Вы находитесь на странице: 1из 705

ndice general

1. Lgica matemtica 9
1.1. Formas proposicionales . . . . . . . . . . . . . . . . . . . . . . . . . . . . . . . . . . 9
1.1.1. Operaciones entre proposiciones lgicas . . . . . . . . . . . . . . . . . . . . 11
1.1.2. Tarea . . . . . . . . . . . . . . . . . . . . . . . . . . . . . . . . . . . . . . . 18
1.2. Construccin de tablas de verdad . . . . . . . . . . . . . . . . . . . . . . . . . . . . 21
1.2.1. Operaciones con frmulas lgicas y sus propiedades . . . . . . . . . . . . . . 22
1.2.2. Tautologas y falacias . . . . . . . . . . . . . . . . . . . . . . . . . . . . . . 28
1.2.3. Tarea . . . . . . . . . . . . . . . . . . . . . . . . . . . . . . . . . . . . . . . 30
1.3. Transformacin de frmulas . . . . . . . . . . . . . . . . . . . . . . . . . . . . . . . 31
1.3.1. Formas normales . . . . . . . . . . . . . . . . . . . . . . . . . . . . . . . . . 33
1.3.2. Consecuencias lgicas . . . . . . . . . . . . . . . . . . . . . . . . . . . . . . 34
1.3.3. Tarea . . . . . . . . . . . . . . . . . . . . . . . . . . . . . . . . . . . . . . . 36
1.4. Expresiones de la lgica de predicados . . . . . . . . . . . . . . . . . . . . . . . . . 38
1.4.1. Leyes de la lgica de predicados . . . . . . . . . . . . . . . . . . . . . . . . . 40
1.4.2. Interpretacin de frmulas en la lgica de predicados . . . . . . . . . . . . . 43
1.4.3. Forma normal prenexa . . . . . . . . . . . . . . . . . . . . . . . . . . . . . . 46
1.4.4. Tarea . . . . . . . . . . . . . . . . . . . . . . . . . . . . . . . . . . . . . . . 49
2. Teora de conjuntos 51
2.1. Conjuntos . . . . . . . . . . . . . . . . . . . . . . . . . . . . . . . . . . . . . . . . . 51
2.1.1. Formas de expresar un conjunto . . . . . . . . . . . . . . . . . . . . . . . . 51
2.2. Conjuntos nitos e innitos . . . . . . . . . . . . . . . . . . . . . . . . . . . . . . . 52
2.2.1. Conjunto nito . . . . . . . . . . . . . . . . . . . . . . . . . . . . . . . . . . 52
2.2.2. Conjunto innito . . . . . . . . . . . . . . . . . . . . . . . . . . . . . . . . . 52
2.2.3. Nocin de pertenencia . . . . . . . . . . . . . . . . . . . . . . . . . . . . . . 52
2.2.4. Igualdad de conjuntos . . . . . . . . . . . . . . . . . . . . . . . . . . . . . . 52
2.2.5. Conjuntos vaco . . . . . . . . . . . . . . . . . . . . . . . . . . . . . . . . . . 53
2.2.6. Conjunto unitario . . . . . . . . . . . . . . . . . . . . . . . . . . . . . . . . 53
2.2.7. Conjunto universal . . . . . . . . . . . . . . . . . . . . . . . . . . . . . . . . 53
2.2.8. Subconjunto . . . . . . . . . . . . . . . . . . . . . . . . . . . . . . . . . . . 53
2.2.9. Conjunto de partes . . . . . . . . . . . . . . . . . . . . . . . . . . . . . . . . 55
2.2.10. Conjunto potencia . . . . . . . . . . . . . . . . . . . . . . . . . . . . . . . . 55
2.3. Operaciones con conjuntos . . . . . . . . . . . . . . . . . . . . . . . . . . . . . . . . 55
2.3.1. Unin de conjuntos . . . . . . . . . . . . . . . . . . . . . . . . . . . . . . . . 55
2.3.2. Propiedades de la unin de conjuntos . . . . . . . . . . . . . . . . . . . . . 56
2.3.3. Interseccin de conjuntos . . . . . . . . . . . . . . . . . . . . . . . . . . . . 56
2.3.4. Propiedades de la interseccin conjuntos . . . . . . . . . . . . . . . . . . . . 57
2.3.5. Diferencia de conjuntos . . . . . . . . . . . . . . . . . . . . . . . . . . . . . 58
1
NDICE GENERAL 2
2.3.6. Complemento de un conjunto . . . . . . . . . . . . . . . . . . . . . . . . . . 59
2.3.7. Propiedades del complemento de un conjunto . . . . . . . . . . . . . . . . . 60
2.3.8. Diferencia simtrica . . . . . . . . . . . . . . . . . . . . . . . . . . . . . . . 61
2.3.9. Cardinalidad . . . . . . . . . . . . . . . . . . . . . . . . . . . . . . . . . . . 62
2.4. Tarea . . . . . . . . . . . . . . . . . . . . . . . . . . . . . . . . . . . . . . . . . . . 64
3. Nmeros reales 75
3.1. Nmeros naturales . . . . . . . . . . . . . . . . . . . . . . . . . . . . . . . . . . . . 75
3.2. Nmeros primos y compuestos . . . . . . . . . . . . . . . . . . . . . . . . . . . . . 77
3.3. Nmeros enteros . . . . . . . . . . . . . . . . . . . . . . . . . . . . . . . . . . . . . 78
3.4. Nmeros racionales . . . . . . . . . . . . . . . . . . . . . . . . . . . . . . . . . . . . 80
3.5. Tarea . . . . . . . . . . . . . . . . . . . . . . . . . . . . . . . . . . . . . . . . . . . 81
3.6. Fracciones . . . . . . . . . . . . . . . . . . . . . . . . . . . . . . . . . . . . . . . . . 82
3.7. Tarea . . . . . . . . . . . . . . . . . . . . . . . . . . . . . . . . . . . . . . . . . . . 86
3.8. Nmeros reales . . . . . . . . . . . . . . . . . . . . . . . . . . . . . . . . . . . . . . 90
3.9. Igualdades y desigualdades numricas . . . . . . . . . . . . . . . . . . . . . . . . . 93
3.10. Tarea . . . . . . . . . . . . . . . . . . . . . . . . . . . . . . . . . . . . . . . . . . . 100
3.11. Smbolo sumatoria . . . . . . . . . . . . . . . . . . . . . . . . . . . . . . . . . . . . 103
3.12. Smbolo producto . . . . . . . . . . . . . . . . . . . . . . . . . . . . . . . . . . . . . 112
3.13. Tarea . . . . . . . . . . . . . . . . . . . . . . . . . . . . . . . . . . . . . . . . . . . 114
3.14. Induccin matemtica . . . . . . . . . . . . . . . . . . . . . . . . . . . . . . . . . . 116
3.15. Tarea . . . . . . . . . . . . . . . . . . . . . . . . . . . . . . . . . . . . . . . . . . . 121
3.16. Factorial y frmula del binomio de Newton . . . . . . . . . . . . . . . . . . . . . . 125
3.17. Tarea . . . . . . . . . . . . . . . . . . . . . . . . . . . . . . . . . . . . . . . . . . . 133
3.18. Progresiones . . . . . . . . . . . . . . . . . . . . . . . . . . . . . . . . . . . . . . . . 137
3.19. Tarea . . . . . . . . . . . . . . . . . . . . . . . . . . . . . . . . . . . . . . . . . . . 149
4. Expresiones algebraicas 155
4.1. Expresin numrica . . . . . . . . . . . . . . . . . . . . . . . . . . . . . . . . . . . . 155
4.2. Tarea . . . . . . . . . . . . . . . . . . . . . . . . . . . . . . . . . . . . . . . . . . . 157
4.3. Potencia con exponente entero . . . . . . . . . . . . . . . . . . . . . . . . . . . . . 160
4.4. Tarea . . . . . . . . . . . . . . . . . . . . . . . . . . . . . . . . . . . . . . . . . . . 169
4.5. Potencia con exponente racional . . . . . . . . . . . . . . . . . . . . . . . . . . . . 170
4.6. Tarea . . . . . . . . . . . . . . . . . . . . . . . . . . . . . . . . . . . . . . . . . . . 181
4.7. Potencia de un nmero positivo . . . . . . . . . . . . . . . . . . . . . . . . . . . . . 186
4.8. Tarea . . . . . . . . . . . . . . . . . . . . . . . . . . . . . . . . . . . . . . . . . . . 190
4.9. Magnitudes directa e inversamente proporcionales . . . . . . . . . . . . . . . . . . . 194
4.10. Razones y proporciones . . . . . . . . . . . . . . . . . . . . . . . . . . . . . . . . . 195
4.10.1. Proporcionalidad directa . . . . . . . . . . . . . . . . . . . . . . . . . . . . . 196
4.10.2. Proporcionalidad inversa . . . . . . . . . . . . . . . . . . . . . . . . . . . . . 196
4.10.3. Proporcin . . . . . . . . . . . . . . . . . . . . . . . . . . . . . . . . . . . . 202
4.11. Tarea . . . . . . . . . . . . . . . . . . . . . . . . . . . . . . . . . . . . . . . . . . . 202
4.12. Regla de tres y porcentajes . . . . . . . . . . . . . . . . . . . . . . . . . . . . . . . 206
4.12.1. Regla de tres simple . . . . . . . . . . . . . . . . . . . . . . . . . . . . . . . 206
4.12.2. Regla de tres compuesta . . . . . . . . . . . . . . . . . . . . . . . . . . . . . 207
4.12.3. Porcentajes . . . . . . . . . . . . . . . . . . . . . . . . . . . . . . . . . . . . 208
4.13. Tarea . . . . . . . . . . . . . . . . . . . . . . . . . . . . . . . . . . . . . . . . . . . 211
NDICE GENERAL 3
5. Polinomios 214
5.1. Deniciones generales . . . . . . . . . . . . . . . . . . . . . . . . . . . . . . . . . . 214
5.2. Suma, resta y producto de polinomios . . . . . . . . . . . . . . . . . . . . . . . . . 216
5.3. Produtos notables . . . . . . . . . . . . . . . . . . . . . . . . . . . . . . . . . . . . 218
5.4. Tarea . . . . . . . . . . . . . . . . . . . . . . . . . . . . . . . . . . . . . . . . . . . 221
5.5. Divisin de polinomios . . . . . . . . . . . . . . . . . . . . . . . . . . . . . . . . . . 223
5.5.1. Mtodo normal . . . . . . . . . . . . . . . . . . . . . . . . . . . . . . . . . . 223
5.5.2. Mtodo de coecientes separados . . . . . . . . . . . . . . . . . . . . . . . . 225
5.5.3. Mtodo de Horner . . . . . . . . . . . . . . . . . . . . . . . . . . . . . . . . 225
5.5.4. Regla de Runi . . . . . . . . . . . . . . . . . . . . . . . . . . . . . . . . . 226
5.5.5. Teorema del resto . . . . . . . . . . . . . . . . . . . . . . . . . . . . . . . . 228
5.6. Tarea . . . . . . . . . . . . . . . . . . . . . . . . . . . . . . . . . . . . . . . . . . . 229
5.7. Mtodos de factorizacin . . . . . . . . . . . . . . . . . . . . . . . . . . . . . . . . . 235
5.7.1. Factor comn . . . . . . . . . . . . . . . . . . . . . . . . . . . . . . . . . . . 235
5.7.2. Mtodo de identidades . . . . . . . . . . . . . . . . . . . . . . . . . . . . . . 236
5.7.3. Mtodo del aspa . . . . . . . . . . . . . . . . . . . . . . . . . . . . . . . . . 236
5.7.4. Mtodo de evaluacin . . . . . . . . . . . . . . . . . . . . . . . . . . . . . . 237
5.7.5. Mtodo de articios de clculo . . . . . . . . . . . . . . . . . . . . . . . . . 238
5.7.6. Factorizacin recproca . . . . . . . . . . . . . . . . . . . . . . . . . . . . . . 239
5.8. Tarea . . . . . . . . . . . . . . . . . . . . . . . . . . . . . . . . . . . . . . . . . . . 239
5.9. Mximo comn divisor y mnimo comn multiplo . . . . . . . . . . . . . . . . . . . 244
5.9.1. Divisiones sucesivas . . . . . . . . . . . . . . . . . . . . . . . . . . . . . . . 245
5.9.2. Por factorizacin . . . . . . . . . . . . . . . . . . . . . . . . . . . . . . . . . 248
5.10. Tarea . . . . . . . . . . . . . . . . . . . . . . . . . . . . . . . . . . . . . . . . . . . 249
5.11. Fracciones algebraicas . . . . . . . . . . . . . . . . . . . . . . . . . . . . . . . . . . 251
5.12. Tarea . . . . . . . . . . . . . . . . . . . . . . . . . . . . . . . . . . . . . . . . . . . 253
6. Ecuaciones y sistemas de ecuaciones 257
6.1. Ecuaciones algebraicas . . . . . . . . . . . . . . . . . . . . . . . . . . . . . . . . . . 257
6.2. Tarea . . . . . . . . . . . . . . . . . . . . . . . . . . . . . . . . . . . . . . . . . . . 264
6.3. Sistemas de ecuaciones lineales de dos ecuaciones en dos incgnitas . . . . . . . . . 268
6.4. Tarea . . . . . . . . . . . . . . . . . . . . . . . . . . . . . . . . . . . . . . . . . . . 272
6.5. Sistemas de ecuaciones lineales de ms de 2 variables . . . . . . . . . . . . . . . . . 277
6.6. Tarea . . . . . . . . . . . . . . . . . . . . . . . . . . . . . . . . . . . . . . . . . . . 280
6.7. Fracciones parciales . . . . . . . . . . . . . . . . . . . . . . . . . . . . . . . . . . . . 284
6.8. Tarea . . . . . . . . . . . . . . . . . . . . . . . . . . . . . . . . . . . . . . . . . . . 289
6.9. Ecuaciones cuadrticas . . . . . . . . . . . . . . . . . . . . . . . . . . . . . . . . . . 290
6.10. Tarea . . . . . . . . . . . . . . . . . . . . . . . . . . . . . . . . . . . . . . . . . . . 297
6.11. Ecuacin simtrica de tercer y cuarto grados . . . . . . . . . . . . . . . . . . . . . 301
6.12. Tarea . . . . . . . . . . . . . . . . . . . . . . . . . . . . . . . . . . . . . . . . . . . 303
6.13. Ecuaciones de orden superior . . . . . . . . . . . . . . . . . . . . . . . . . . . . . . 304
6.14. Tarea . . . . . . . . . . . . . . . . . . . . . . . . . . . . . . . . . . . . . . . . . . . 308
6.15. Sistemas de ecuaciones no lineales . . . . . . . . . . . . . . . . . . . . . . . . . . . 311
6.16. Tarea . . . . . . . . . . . . . . . . . . . . . . . . . . . . . . . . . . . . . . . . . . . 326
7. Desigualdades e inecuaciones 355
7.1. Desigualdades con una incgnita y de primer grado . . . . . . . . . . . . . . . . . . 355
7.1.1. La recta real . . . . . . . . . . . . . . . . . . . . . . . . . . . . . . . . . . . 355
7.1.2. Segmentos, desigualdades e intervalos . . . . . . . . . . . . . . . . . . . . . 355
7.1.3. Operaciones entre desigualdades . . . . . . . . . . . . . . . . . . . . . . . . 358
NDICE GENERAL 4
7.1.4. Valor absoluto . . . . . . . . . . . . . . . . . . . . . . . . . . . . . . . . . . 358
7.2. Desigualdades de primer grado con una incgnita . . . . . . . . . . . . . . . . . . . 361
7.3. Tarea . . . . . . . . . . . . . . . . . . . . . . . . . . . . . . . . . . . . . . . . . . . 366
7.4. Desigualdad de segundo grado . . . . . . . . . . . . . . . . . . . . . . . . . . . . . . 370
7.5. Tarea . . . . . . . . . . . . . . . . . . . . . . . . . . . . . . . . . . . . . . . . . . . 374
7.6. Desigualdades de orden superior . . . . . . . . . . . . . . . . . . . . . . . . . . . . 382
7.7. Tarea . . . . . . . . . . . . . . . . . . . . . . . . . . . . . . . . . . . . . . . . . . . 384
8. Funciones algebraicas 388
8.1. Funciones . . . . . . . . . . . . . . . . . . . . . . . . . . . . . . . . . . . . . . . . . 388
8.2. Tarea . . . . . . . . . . . . . . . . . . . . . . . . . . . . . . . . . . . . . . . . . . . 400
8.3. Funcin inversa . . . . . . . . . . . . . . . . . . . . . . . . . . . . . . . . . . . . . . 408
8.4. Paridad de una funcin . . . . . . . . . . . . . . . . . . . . . . . . . . . . . . . . . 411
8.5. Tarea . . . . . . . . . . . . . . . . . . . . . . . . . . . . . . . . . . . . . . . . . . . 412
8.6. Monotona de una funcin . . . . . . . . . . . . . . . . . . . . . . . . . . . . . . . . 414
8.7. Tarea . . . . . . . . . . . . . . . . . . . . . . . . . . . . . . . . . . . . . . . . . . . 419
8.8. Operaciones con funciones . . . . . . . . . . . . . . . . . . . . . . . . . . . . . . . . 427
8.9. Tarea . . . . . . . . . . . . . . . . . . . . . . . . . . . . . . . . . . . . . . . . . . . 430
8.10. Grca de una funcin . . . . . . . . . . . . . . . . . . . . . . . . . . . . . . . . . . 432
8.11. Tarea . . . . . . . . . . . . . . . . . . . . . . . . . . . . . . . . . . . . . . . . . . . 441
9. Funciones exponenciales y logartmicas 443
9.1. Expresiones exponenciales y logartmicas . . . . . . . . . . . . . . . . . . . . . . . . 443
9.2. Tarea . . . . . . . . . . . . . . . . . . . . . . . . . . . . . . . . . . . . . . . . . . . 449
9.3. Ecuaciones exponenciales y logartmicas . . . . . . . . . . . . . . . . . . . . . . . . 451
9.4. Tarea . . . . . . . . . . . . . . . . . . . . . . . . . . . . . . . . . . . . . . . . . . . 453
9.5. Desigualdades exponenciales y logartmicas . . . . . . . . . . . . . . . . . . . . . . 458
9.6. Tarea . . . . . . . . . . . . . . . . . . . . . . . . . . . . . . . . . . . . . . . . . . . 461
9.7. Funciones exponenciales y logartmicas . . . . . . . . . . . . . . . . . . . . . . . . . 464
9.8. Tarea . . . . . . . . . . . . . . . . . . . . . . . . . . . . . . . . . . . . . . . . . . . 472
10.Funciones hiperblicas 474
10.1. Funciones hiperblicas directas e inversas . . . . . . . . . . . . . . . . . . . . . . . 474
10.1.1. Funcin seno hiperblico . . . . . . . . . . . . . . . . . . . . . . . . . . . . . 477
10.1.2. Funcin coseno hiperblico . . . . . . . . . . . . . . . . . . . . . . . . . . . 478
10.1.3. Funcin tangente hiperblica . . . . . . . . . . . . . . . . . . . . . . . . . . 479
10.1.4. Funcin cotangente hiperblica . . . . . . . . . . . . . . . . . . . . . . . . . 481
10.1.5. Funcin secante hiperblica . . . . . . . . . . . . . . . . . . . . . . . . . . . 482
10.1.6. Funcin cosecante hiperblica . . . . . . . . . . . . . . . . . . . . . . . . . . 482
10.2. Tarea . . . . . . . . . . . . . . . . . . . . . . . . . . . . . . . . . . . . . . . . . . . 483
11.Funciones trigonomtricas 486
11.1. Angulos . . . . . . . . . . . . . . . . . . . . . . . . . . . . . . . . . . . . . . . . . . 486
11.1.1. Medicin del ngulo en grados . . . . . . . . . . . . . . . . . . . . . . . . . 487
11.1.2. Medida radial del ngulo . . . . . . . . . . . . . . . . . . . . . . . . . . . . 487
11.2. Crculo unitario . . . . . . . . . . . . . . . . . . . . . . . . . . . . . . . . . . . . . . 489
11.3. Funciones trigonomtricas de un ngulo . . . . . . . . . . . . . . . . . . . . . . . . 492
11.4. Identidades trigonomtricas . . . . . . . . . . . . . . . . . . . . . . . . . . . . . . . 507
11.5. Frmulas de adicin . . . . . . . . . . . . . . . . . . . . . . . . . . . . . . . . . . . 514
11.6. Frmulas de arcos dobles y mitad . . . . . . . . . . . . . . . . . . . . . . . . . . . . 529
NDICE GENERAL 5
11.7. Tarea . . . . . . . . . . . . . . . . . . . . . . . . . . . . . . . . . . . . . . . . . . . 539
11.8. Ecuaciones trigonomtricas . . . . . . . . . . . . . . . . . . . . . . . . . . . . . . . 548
11.9. Tarea . . . . . . . . . . . . . . . . . . . . . . . . . . . . . . . . . . . . . . . . . . . 554
11.10.Desigualdades trigonomtricas . . . . . . . . . . . . . . . . . . . . . . . . . . . . . . 559
11.11.Tarea . . . . . . . . . . . . . . . . . . . . . . . . . . . . . . . . . . . . . . . . . . . 561
11.12.Funciones trigonomtricas . . . . . . . . . . . . . . . . . . . . . . . . . . . . . . . . 562
11.12.1.Funcin seno . . . . . . . . . . . . . . . . . . . . . . . . . . . . . . . . . . . 563
11.12.2.Funcin Coseno . . . . . . . . . . . . . . . . . . . . . . . . . . . . . . . . . . 570
11.12.3.Funcin Tangente . . . . . . . . . . . . . . . . . . . . . . . . . . . . . . . . 573
11.12.4.Funcin Cotangente . . . . . . . . . . . . . . . . . . . . . . . . . . . . . . . 575
11.13.Tarea . . . . . . . . . . . . . . . . . . . . . . . . . . . . . . . . . . . . . . . . . . . 577
11.14.Expresiones trigonometricas inversas . . . . . . . . . . . . . . . . . . . . . . . . . . 580
11.15.Tarea . . . . . . . . . . . . . . . . . . . . . . . . . . . . . . . . . . . . . . . . . . . 586
11.16.Ecuaciones trigonomtricas inversas . . . . . . . . . . . . . . . . . . . . . . . . . . 588
11.17.Tarea . . . . . . . . . . . . . . . . . . . . . . . . . . . . . . . . . . . . . . . . . . . 589
11.18.Funciones trigonomtricas inversas . . . . . . . . . . . . . . . . . . . . . . . . . . . 591
11.18.1.Funcin arco seno . . . . . . . . . . . . . . . . . . . . . . . . . . . . . . . . 592
11.18.2.Funcin arco coseno . . . . . . . . . . . . . . . . . . . . . . . . . . . . . . . 595
11.18.3.Funcin arco tangente . . . . . . . . . . . . . . . . . . . . . . . . . . . . . . 596
11.18.4.Funcin arco cotangente . . . . . . . . . . . . . . . . . . . . . . . . . . . . . 598
11.19.Tarea . . . . . . . . . . . . . . . . . . . . . . . . . . . . . . . . . . . . . . . . . . . 599
12.Curvas dadas implcitamente 602
12.1. Deniciones generales . . . . . . . . . . . . . . . . . . . . . . . . . . . . . . . . . . 602
12.2. Curvas implcitas . . . . . . . . . . . . . . . . . . . . . . . . . . . . . . . . . . . . . 603
12.3. Tarea . . . . . . . . . . . . . . . . . . . . . . . . . . . . . . . . . . . . . . . . . . . 605
13.Curvas dadas paramtricamente 606
13.1. Ecuaciones parametricas . . . . . . . . . . . . . . . . . . . . . . . . . . . . . . . . . 606
13.2. Tarea . . . . . . . . . . . . . . . . . . . . . . . . . . . . . . . . . . . . . . . . . . . 607
13.3. Curvas paramtricas . . . . . . . . . . . . . . . . . . . . . . . . . . . . . . . . . . . 608
13.4. Tarea . . . . . . . . . . . . . . . . . . . . . . . . . . . . . . . . . . . . . . . . . . . 611
14.Curvas dadas en coordenadas polares 613
14.1. Transformaciones . . . . . . . . . . . . . . . . . . . . . . . . . . . . . . . . . . . . . 613
14.2. Tarea . . . . . . . . . . . . . . . . . . . . . . . . . . . . . . . . . . . . . . . . . . . 615
14.3. Grca en coordenadas polares . . . . . . . . . . . . . . . . . . . . . . . . . . . . . 617
14.4. Tarea . . . . . . . . . . . . . . . . . . . . . . . . . . . . . . . . . . . . . . . . . . . 620
14.5. Interseccin de curvas . . . . . . . . . . . . . . . . . . . . . . . . . . . . . . . . . . 622
14.6. Tarea . . . . . . . . . . . . . . . . . . . . . . . . . . . . . . . . . . . . . . . . . . . 623
15.Nmeros complejos 624
15.1. Deniciones . . . . . . . . . . . . . . . . . . . . . . . . . . . . . . . . . . . . . . . . 624
15.2. Operaciones con los nmeros complejos . . . . . . . . . . . . . . . . . . . . . . . . 637
15.3. Tarea . . . . . . . . . . . . . . . . . . . . . . . . . . . . . . . . . . . . . . . . . . . 646
15.4. Potencia de un nmero complejo . . . . . . . . . . . . . . . . . . . . . . . . . . . . 659
15.4.1. Potencia de la unidad imaginaria . . . . . . . . . . . . . . . . . . . . . . . . 659
15.4.2. Potenciacin de un nmero complejo . . . . . . . . . . . . . . . . . . . . . . 660
15.4.3. Extraccin de la raz cuadrada de un nmero complejo . . . . . . . . . . . . 661
15.5. Tarea . . . . . . . . . . . . . . . . . . . . . . . . . . . . . . . . . . . . . . . . . . . 663
NDICE GENERAL 6
15.6. Forma trigonomtrica . . . . . . . . . . . . . . . . . . . . . . . . . . . . . . . . . . 676
15.6.1. Forma trigonomtrica de un nmero complejo . . . . . . . . . . . . . . . . . 676
15.6.2. Producto de nmeros complejos dados en forma trigonomtrica . . . . . . . 683
15.6.3. Divisin de nmeros complejos dados en forma trigonomtrica . . . . . . . 684
15.6.4. Potenciacin de un nmero complejo dado en forma trigonomtrica . . . . . 685
15.6.5. Radicacin de nmeros complejos dados en forma trigonomtrica . . . . . . 687
15.6.6. Tarea . . . . . . . . . . . . . . . . . . . . . . . . . . . . . . . . . . . . . . . 694
15.7. Forma exponencial de un nmero complejo . . . . . . . . . . . . . . . . . . . . . . 700
15.8. Tarea . . . . . . . . . . . . . . . . . . . . . . . . . . . . . . . . . . . . . . . . . . . 704
Introduccin
He aqu una obra de colaboracin, en que se pretende aunar experiencias diversas de publica-
ciones anteriores y practica docente, mediante una labor de conjunto y critica mutua. Destinado el
libro a servir de base a cursos formativos de iniciacin universitaria en el Departamento de Ciencias
Exactas, la seleccin del material se ha guiado por los planes de estudio de las Universidades y
Escuelas Politcnicas, pero sin sujetarse a ningn programa determinado, antes bien, con afn de
superacin y aliento renovador.
Un curso es una relacin de cuestiones fundamentales, lgicamente encadenadas; es una ex-
cursin exploradora por el campo de una ciencia; es como un plano que sirve de preparacin y
gua para el estudio de los tratados. Un tratado general debe contener, en cambio, una exposicin
sistemtica del organismo de una ciencia; debe ser la cantera de donde puedan extraerse cursos
variados.
Pero un tratado no debe ser una enciclopedia, ni puede ser un libro de historia de la ciencia. No
busque, pues, el lector en estas pginas multitud de cuestiones con las que acaso est encariado,
pero que por su inters muy secundario o exclusivamente histrico no deben gurar en un libro
moderno; pues, adems de distraer la atencin, quitan tiempo y espacio preciosos para poder llegar
en plazo prudencial hasta los problemas actuales de la matemtica superior.
Tambin he puesto especial cuidado en omitir toda clase de detalles superuos o secundarios,
que slo cansancio y desorientacin producen. Detenindose solamente en las estaciones principales,
es posible llegar en poco tiempo bastante lejos; mientras que perderse en una selva de minucias y
casos particulares, que confunden y oscurecen los troncos primarios, es condenarse voluntariamente
a no salir nunca de lo elemental.
No busque tampoco el lector en estas pginas disquisiciones metafsicas sobre los nmeros ir-
racionales, imaginarios, etc. Pasada ya la poca subsiguiente a toda ampliacin del concepto de
nmero; vencida la inevitable resistencia que la inercia opone siempre a todo concepto nuevo, estas
nociones han perdido desde hace casi medio siglo todo su antiguo misterio.
Huyendo de la general tendencia a elevar por abstraccin los asuntos elementales, he prescindi-
do de todo formalismo, esforzndome por el contrario, en elementalizar las cuestiones difciles sin
menoscabo del rigor.
El rigor constituye hoy un mandato imperativo en todo libro de matemtica. Toda demostracin
no rigurosa se considera como un valor nulo.
En un curso de Algebra, el alumno dispone bsicamente de tres recursos: el profesor, el libro y
el tiempo que dedique al trabajo duro. Este ltimo es el ms importante. Es durante ese tiempo,
7
NDICE GENERAL 8
cuando trata de resolver problemas y utiliza el libro para aprender a resolverlos, cuando obtiene
el mayor provecho. Este libro de Algebra Superior, est diseado para utilizarse estudiando los
ejemplos que facilitan el aprendizaje de las tcnicas del lgebra.
El objetivo fundamental de este trabajo, es proporcionar un libro a partir del cual, el estudi-
ante de Algebra adquiera en el curso la mayor destreza y profundidad en la resolucin de problemas.
El estudiante, que por s solo utiliza una coleccin de problemas, ha de ser su propio corrector;
por tanto, debe observar con el mayor cuidado si no ha omitido alguna parte del razonamiento, y
debe ser, adems, muy exigente consigo mismo.
Este trabajo lo dedico a la memoria de mis padres:
Lus Garca y Gladys Arcos.
Para consultas y sugerencias, remitirse a jofregaar@hotmail.com
Joe Garca Arcos
Profesor de Matemticas
Escuela Politcnica del Ejrcito
Sangolqu, Marzo de 2011
Captulo 1
Lgica matemtica
1.1. Formas proposicionales
La lgica matemtica se ocupa del anlisis de las proposiciones y demostraciones del razon-
amiento lgico, proporciona ideas claras y precisas sobre la naturaleza de la conclusin deductiva,
desarrolla el pensamiento funcional y hace una contribucin esencial al desarrollo del pensamiento
cientco y creador. Esto se maniesta, por ejemplo, en la correcta comprensin de las estructuras
lgicas y las tareas formales, en el reconocimiento de las semejanzas de los diferentes fenmenos
lgicos, en la aplicacin de las leyes y reglas lgicas y en la pretensin de claridad, sencillez y
economa en la expresin lingstica.
Una de las propiedades de la forma de expresin matemtica, es la de representar los objetos,
las imgenes mentales, los vnculos y las relaciones mediante smbolos (signos), y combinarlos entre
s.
Denicin 1.1 Constante
Una constante es un signo que tiene una determinada signicacin ja.
Es decir; una constante tiene, en todo el desarrollo de una investigacin o en la solucin de una
tarea, siempre la misma signicacin.
Denicin 1.2 Variable
Una variable es un signo que representa cualquier elemento de un dominio bsico previamente
establecido.
Esto quiere decir que una variable se puede sustituir por el signo de cualquier elemento del
dominio bsico. Entonces se habla de la sustitucin de la variable, o de la interpretacin de la
variable.
Denicin 1.3 Trmino
Por trmino entendemos las constantes, las variables y sus combinaciones mediante los signos de
operacin y los signos tcnicos.
Los trminos son, por tanto, las denominaciones de los objetos matemticos o las combina-
ciones de signos donde se presentan variables, constantes y signos de operaciones, y que mediante
la interpretacin de las variables se omiten en las designaciones de los objetos matemticos. El ob-
jeto matemtico, identicado como un trmino, y en cuya denominacin se omite este calicativo
9
CAPTULO 1. LGICA MATEMTICA 10
despus de la interpretacin de las variables, se conoce como valor del trmino.
Las proposiciones son estructuras lingsticas cuyo valor de verdad es, o verdadero o falso. La
lgica clsica, a travs de sus axiomas y principios, ha hecho algunas consideraciones sobre el con-
tenido de verdad de una proposicin. El principio de la bivalencia expresa: Toda proposicin o es
falsa o es verdadera.
De este principio se pueden deducir dos teoremas.
1. El teorema de la tercera posibilidad excluida, expresa:
Toda proposicin es falsa o verdadera.
2. El teorema de la contradiccin excluida, expresa:
Ninguna proposicin es falsa y verdadera al mismo tiempo.
En las observaciones posteriores veremos que los dos teoremas, considerados en conjunto, ex-
presan exactamente lo mismo que el principio de la bivalencia. Por consiguiente, se puede proceder
a la inversa; es decir deducir el teorema de la bivalencia a partir del principio de la tercera posibil-
idad excluida y del principio de la contradiccin excluida.
A cada proposicin se le hace corresponder un valor de verdad, o falso F o verdadero V. Es por
esta razn que tambin se habla de una lgica bivalente. La asignacin de los valores de verdad F o
V de una proposicin, no es tan sencillo de determinar. Aunque en el principio de la bivalencia se
expresa claramente que una proposicin es falsa o verdadera, no se puede decir inmediatamente si
cada proposicin es falsa o verdadera. En matemticas existen actualmente muchas proposiciones
que hasta el momento no han podido ser demostradas, concebida, la demostracin, como una
aseveracin de la verdad, a continuacin se dan dos ejemplos de este tipo de proposiciones.
Ejemplo 1.1 La proposicin: Todo nmero par que sea mayor que 4, se puede representar
como la suma de dos nmeros primos, excepto el 2, existe desde el ao 1742. Hasta el momento
no se ha podido demostrar si es una proposicin falsa o verdadera. (Suposicin de Goldbach).
Denicin 1.4 Forma proposicional
Una estructura lingstica que contiene por lo menos una variable libre, se convierte en una proposi-
cin, cuando se sustituyen todas las variables por smbolos, que denotan objetos del dominio bsico,
recibe el nombre de forma proposicional.
Ejemplo 1.2 8 + x <12 con x N no representa evidentemente ninguna proposicin. Esta
sucesin de signos no es ni falsa ni verdadera. Mediante las sustituciones de la variable x podemos
formar proposiciones falsas y verdaderas. As, con las sustituciones 0, 1, 2, 3 obtenemos siem-
pre proposiciones verdaderas, y cualquier otra sustitucin dar lugar a proposiciones falsas. En
este caso, encontramos una expresin lingstica especial que no es una proposicin, pero que, sin
embargo, se convierte en una proposicin mediante la sustitucin de la variable.
A las expresiones matemticas de este tipo se las denomina formas proposicionales. Las variables
en tales expresiones se denominan variables libres. Las formas proposicionales surgen cuando entre
los trminos que contienen variables se coloca un determinado signo de relacin. De forma anloga
al convenio establecido para la notacin de trminos, denotamos una forma proposicional con
P(x
1
, x
2
, ..., x
n
). Todos los elementos cuyos smbolos convierten una forma proposicional en una
proposicin, constituyen el conjunto solucin de esta forma proposicional. El conjunto solucin
comprende solamente aquellos elementos del dominio bsico cuyos smbolos convierten una forma
proposicional en una proposicin verdadera. Las formas proposicionales se pueden clasicar en la
forma siguiente: aquellas formas proposicionales que mediante una sustitucin por lo menos, se
CAPTULO 1. LGICA MATEMTICA 11
pueden transformar en una proposicin verdadera, se denominan interpretables. Todas las dems
se denominan no interpretables. Entre las interpretables se destacan las formas proposicionales de
validez general, que son aquellas que al hacer cualquier sustitucin por los elementos del dominio
bsico se convierten en una proposicin verdadera.
Ejemplo 1.3 (x+y)
2
= x
2
+2xy +y
2
x, y R. En toda sustitucin de x e y por elementos
del dominio bsico se obtiene una proposicin verdadera. Este ejemplo es, por tanto, una forma
proposicional de validez general en el conjunto de los nmeros reales. El conjunto solucin es el
conjunto de todos los pares (x, y) donde x e y son elementos de un dominio bsico; luego, en este
caso coincide con el conjunto base de solucin. Este ejemplo trata entonces de una identidad.
Ejemplo 1.4 Sea (x +y)
2
= x
2
+y
2
x, y R.
(x +y)
2
x
2
+y
2
Valor de verdad
0 0 V
9 9 V
36 36 V
9 5 F
361 193 F
La presente tabla muestra que a partir de esta forma proposicional se pueden obtener proposi-
ciones falsas y verdaderas. El conjunto solucin es, un subconjunto propio del conjunto base de
solucin. El conjunto solucin consta, de los pares ordenados de elementos del dominio bsico.
Este ejemplo trata entonces de una neutralidad.
Ejemplo 1.5 x
2
5x + 10 = 0 x R. En el dominio bsico no hay elementos que satisfagan
esta forma proposicional, es decir, toda sustitucin la convierte en una proposicin falsa. Este
ejemplo trata por consiguiente de una contradiccin.
1.1.1. Operaciones entre proposiciones lgicas
En esta seccin trataremos exclusivamente las proposiciones y las formas proposicionales.
Primeramente, introduciremos algunas combinaciones de proposiciones, mediante las cuales a su vez
se obtienen otras proposiciones. Despus obtendremos mediante deniciones las funciones proposi-
cionales y ms tarde las funciones veritativas. En todas las operaciones con proposiciones sealare-
mos el proceso de abstraccin circunstancia - proposiciones - valores de verdad.
Denicin 1.5 Proposicin
Denominaremos proposicin a una frase narrativa que puede calicarse como verdadera o falsa,
pero no ambas al mismo tiempo.
Los valores verdadero y falso mencionados en la denicin se denominan valores de certeza o
valores de verdad. As cuando una proposicin se considere verdadera o falsa diremos que dicha
proposicin tiene valor de certeza verdadero o falso.
Ejemplo 1.6 Las siguientes frases son proposiciones:
- La tierra es plana.
- 547 es un nmero primo.
- Los nmeros irracionales son complejos.
- Los nmeros complejos son un subconjunto de los reales.
- La Escuela Politcnica del Ejercito es un instituto de educacin superior.
- No es verdadero que 3 sea un entero par o 7 un primo.
CAPTULO 1. LGICA MATEMTICA 12
- 2n = n
2
para alguna n N.
- 289301 + 1 es un nmero primo.
- Si un rbol tiene n vrtices, entonces tiene exactamente n - 1 aristas.
- 2n + n es un nmero primo para una innidad de n.
- Todo entero par mayor que 4 es la suma de dos nmeros primos.
- Las matemticas son divertidas.
- Los rboles son ms interesantes que las matrices.
Ejemplo 1.7 Las siguientes frases no son proposiciones:
- Porqu es importante la lgica proposicional?
- 323789 ext 205
- Porqu es importante la induccin?
- x - y = y - x.
Es importante hacer notar que el valor de verdad de una proposicin no es trabajo ni parte
de la lgica aqu tratada, por tal razn dichos valores los supondremos ya asignados. Todas las
proposiciones constituyen una clase que, a su vez, se descompone en dos subclases, en la clase de
las proposiciones verdaderas V y en la clase de las proposiciones falsas F.
La verdad o falsedad de las proposiciones no puede ser demostrada inmediatamente en todos
los casos, pero, para toda proposicin, independientemente de que an no haya sido comprobada ni
refutada, solo cabe una de las dos posibilidades, es verdadera V o es falsa F. El proceso de negacin
lo denominamos operacin lgica de un lugar. Los enlaces de dos proposiciones, como resultado de
los cuales se obtiene una proposicin nica se denominan operaciones lgicas de dos lugares.
Denicin 1.6 Funcin proposicional de n-lugares
Cuando a cada n-uplo de proposiciones se le hace corresponder unvocamente una proposicin, esta
correspondencia se denomina funcin proposicional de n-lugares.
Se entiende por n-uplo, un conjunto de n elementos dependientes del orden, en este caso proposi-
ciones. De todas las funciones proposicionales, las llamadas funciones proposicionales clsicas tienen
una gran importancia por las razones siguientes:
a) porque las restantes funciones se pueden representar en trminos de estas.
b) porque en la lgica formal tradicional se han tratado especialmente las cinco siguientes fun-
ciones:
Nombre Argumento Funciones proposicionales Nmero de lugares
Negacin P No P Uno
Conjuncin P, Q P y Q Dos
Disyuncin P, Q P o Q Dos
Implicacin P, Q Si P, entonces Q Dos
Equivalencia P, Q P exactamente cuando Q Dos
En estas funciones proposicionales el valor de verdad de la proposicin resultante depende so-
lamente de los valores de verdad de los argumentos correspondientes, y no de su contenido, y se
denominan funciones proposicionales extensionales. Adems de las funciones proposicionales clsi-
cas existen otras funciones proposicionales que son extensionales.
En el transcurso de las observaciones hemos hecho abstraccin del contenido concreto de las
proposiciones o de los enlaces de proposiciones y, alcanzado las etapas de abstraccin de las fun-
ciones proposicionales.
CAPTULO 1. LGICA MATEMTICA 13
Continuamos el proceso de abstraccin sobre la base de las armaciones ya hechas. Si tambin
realizamos el paso de transicin de las proposiciones a los valores de verdad, entonces obtenemos
las funciones veritativas correspondientes a las funciones proposicionales.
Denicin 1.7 Funcin veritativa de n-lugares
Cuando a cada n-uplo de valores de verdad se le hace corresponder unvocamente un valor de
verdad, entonces esta correspondencia recibe el nombre de funcin veritativa de n lugares.
Las funciones proposicionales y las funciones veritativas pertenecen a diferentes niveles de ab-
straccin. Por este motivo es conveniente introducir otros smbolos para las funciones veritativas.
Con la negacin de una proposicin queremos expresar la idea de que esto no se reere a la cir-
cunstancia que a ella corresponde. Cuando negamos una proposicin P, entonces obtenemos otra
proposicin P, es decir, la negacin de P. A travs de esta operacin obtenemos una proposicin
cuyo valor de verdad es contrario al valor de verdad de P.
Frecuentemente en el lenguaje comn, una negacin se expresa mediante prejos que indican
negacin o mediante adverbios de negacin. Por ejemplo, se dice en lugar de no regular, irregular,
y en lugar de no un, simplemente ningn, etc. Al formular ciertas negaciones pueden surgir con
mucha facilidad algunas confusiones, cuando simplemente se expresa la negacin mediante antn-
imos o contrarios.
Por ejemplo, negro y blanco, pequeo y grande, positivo y negativo, orden y caos son, en cierto
sentido, contrarios que no pueden ser expresados a travs de una negacin. Aqu deniremos la
negacin como una funcin veritativa, aunque la denominacin de negacin se utiliza tambin para
la funcin proposicional P y para la operacin negacin.
Denicin 1.8 Negacin
Se denomina negacin a la funcin veritativa de un lugar, cuyos valores se jan de la manera
siguiente:
P P
V F
F V
La negacin corresponde a la funcin proposicional de un lugar P. La armacin P es ver-
dadera cuando la proposicin P es falsa, y P es falsa cuando la proposicin P es verdadera.
Mediante la negacin de una proposicin P se obtiene una nueva proposicin P cuyo valor de
verdad es opuesto al valor de verdad de P.
Ejemplo 1.8 Sea P: Los billetes de $ 5000 contienen la egie de Rumiahui.
La negacin de P es la proposicin:
P: Los billetes de $ 5000 no contienen la egie de Rumiahui.
Dadas las proposiciones P, Q consideremos la construccin de proposiciones de la forma (P y
Q).
Ejemplo 1.9 En la bsqueda de un profesor la ESPE publica el siguiente aviso:
Se solicita profesor con especialidad en Pedagoga y lgebra.
Se presentan cuatro candidatos A, B, C y D con las caractersticas siguientes:
A: Tiene ambas especialidades (V, V)
B: Slo tiene la especialidad de Pedagoga (V, F)
C: Slo tiene la especialidad de lgebra (F, V)
CAPTULO 1. LGICA MATEMTICA 14
D: Slo tiene la especialidad de Qumica (F, F)
Como podemos darnos cuenta, puesto que A cumpli con los requisitos, entonces A es el ganador
del empleo.
Denicin 1.9 Conjuncin
Se denomina conjuncin o producto lgico de las proposiciones P y Q, dadas en este orden, a la
funcin veritativa de dos lugares que se obtiene enunciando Q luego de enunciar P, unidas ambas
por la palabra y, cuyos valores de verdad se jan de la siguiente manera:
P Q P Q
V V V
V F F
F V F
F F F
El enlace de proposiciones P Q es verdadera cuando ambas proposiciones P, Q son verdaderas.
Una conjuncin cuyo valor de verdad es V, expresa que las circunstancias que se reejan a travs
de las proposiciones parciales, existen en conjunto. Pero, cuando una conjuncin tiene el valor de
verdad F, esto signica que, por lo menos, una de sus proposiciones parciales no reeja correc-
tamente una circunstancia. En las explicaciones posteriores, consideraremos el enlace de varias
proposiciones como una conjuncin si aparece la expresin y/o sus sinnimos.
Ejemplo 1.10 Sean las proposiciones:
P: 2 es un divisor de 10
Q: 5 es un divisor de 10
La conjuncin de P y Q es la siguiente proposicin:
P Q: 2 es un divisor de 10, pero tambin 5 es un divisor de 10
Por lo tanto la proposicin P Q es verdadera.
Estudiaremos ahora proposiciones de la forma (P o Q) y (o P o Q).
Ejemplo 1.11 Consideremos ahora el siguiente aviso:
Se solicita profesor con especialidad de Pedagoga o lgebra
Se presentan cuatro candidatos A, B, C y D con las caractersticas siguientes:
A: Tiene ambas especialidades (V, V)
B: Slo tiene la especialidad de Pedagoga (V, F)
C: Slo tiene la especialidad de Algebra (F, V)
D: Slo tiene la especialidad de Qumica (F, F)
En este caso slo D no podr ser seleccionado.
El punto central de esta parte lo constituye el uso de la palabra o, la cual puede ser utilizada
en un sentido exclusivista o ... o ... o no exclusivista. Por este motivo procederemos en dos pasos
intermedios.
Denicin 1.10 Disyuncin
Se denomina disyuncin a la funcin veritativa bivalente cuyos valores se jan de la manera sigu-
iente:
P Q P Q
V V V
V F V
F V V
F F F
CAPTULO 1. LGICA MATEMTICA 15
La disyuncin corresponde a la funcin proposicional bivalente P o Q. Segn la denicin
anterior P o Q es verdadera cuando, como mnimo, una de las proposiciones enlazadas es verdadera.
En discusiones posteriores, el enlace de varias proposiciones con o recibe el nombre de disyuncin.
Entonces, un enlace de proposiciones de este tipo representa una proposicin verdadera cuando
todas las proposiciones enlazadas son verdaderas.
Ejemplo 1.12 - 2 3 = 6 3 + 2 = 5
- 75 % de 45 m es 135/4 m, 33,75 m.
A causa de la extensionalidad de las funciones proposicionales las proposiciones enlazadas
pueden o no, tener relaciones de contenido entre s. La abstraccin hecha de las relaciones de
contenido entre las proposiciones enlazadas es necesaria para poder fundamentar la relacin lgica.
Mediante la denicin anterior se ha determinado el sentido de la palabra o. Para nosotros son de
gran inters an los valores de verdad de las proposiciones parciales. En otros enlaces se proceder
de forma similar.
Denicin 1.11 Alternativa
Se denomina alternativa a la funcin veritativa de dos lugares cuyos valores se jan de la manera
siguiente:
P Q P Q
V V V
V F V
F V V
F F F
La alternativa corresponde a la funcin proposicional de dos lugares o P o Q es verdadero
cuando una de las dos proposiciones es verdadera. Es falso cuando ambas proposiciones son ver-
daderas o falsas. La alternativa es igualmente extensional.
En las explicaciones que demos posteriormente, un enlace de ms de dos proposiciones con o
... o recibe el nombre de alternativa. En el uso diario del lenguaje se dice frecuentemente o en
lugar de o ... o, actuando esta palabra, en tales casos, de forma excluyente.
Cuando en el lenguaje familiar corriente se habla de una disyuncin, se hace referente a la
alternativa que hemos denido. Estos hechos hay que tenerlos siempre en cuenta.
Ejemplo 1.13 - La suma de los siete primeros nmeros naturales es o par o impar.
- 1969 es o un nmero primo o divisible por 9.
Un verdadero enlace mediante la alternativa de ambas proposiciones reeja que de dos circun-
stancias posibles existe exactamente una. Para evitar las confusiones se debe utilizar, en tales casos,
siempre o ... o. o puede usarse en el lenguaje comn pero con otro sentido, cuando se quiere
decir que las dos circunstancias enlazadas entre s no pueden existir en conjunto. Como mximo,
esto puede referirse a una de ellas.
El conocimiento de estas distintas interpretaciones de o en el lenguaje comn es muy im-
portante para la conclusin lgica y, adems, nos motiva a velar por la exactitud de nuestras
formulaciones.
A continuacin formularemos, mediante la disyuncin, algunos teoremas importantes de la
lgica de las proposiciones.
CAPTULO 1. LGICA MATEMTICA 16
Teorema 1.1 Toda proposicin es verdadera o falsa.
Este teorema se denomina, teorema del tercero excluido. Esto podemos representarlo mediante
la funcin proposicional P o P. Esta funcin proposicional es una identidad, porque para
cualquier argumento P, siempre obtenemos una proposicin verdadera.
Teorema 1.2 Toda proposicin es o verdadera o falsa.
Este teorema se denomina, principio de la bivalencia. Este principio expresa que entre una
proposicin y su negacin no hay una tercera posibilidad, y que una proposicin no puede ser
simultneamente verdadera y falsa.
Proposiciones como P Q y P Q que resultan de combinar otras proposiciones reciben el
nombre de proposiciones compuestas.
Es posible una proposicin compuesta G

= G(P
1
, P
2
, ..., P
n
) sea verdadera sin importar qu
asignaciones de verdad se hayan hecho a las proposiciones P
1
, P
2
, ..., P
n
.
Ejemplo 1.14 En el Instituto de Ciencias Bsicas existe el siguiente reglamento: Para que
un estudiante pueda tomar materias de avance de primer nivel, tiene que haber aprobado materias
concatenadas de prepolitcnico. En cules de los siguientes casos se viola el reglamento?
A: Toma avances y aprob materias concatenadas (V, V).
B: Toma avances y no aprob materias concatenadas (V, F).
C: No toma avances pero aprob materias concatenadas (F, V).
D: Ni toma avances ni aprob materias concatenadas (F, F).
Un poco de reexin nos conduce a aceptar que se viola el reglamento en el caso B.
Denicin 1.12 Implicacin
Se denomina implicacin o condicional a la funcin veritativa de dos lugares cuyos valores de
verdad se jan de la manera siguiente:
P Q P Q
V V V
V F F
F V V
F F V
La proposicin P se denomina hiptesis o antecedente y la proposicin Q, conclusin o conse-
cuente. Considrese el problema de asignar un valor de verdad a la proposicin implicacin si P,
entonces Q. En efecto, si la hiptesis P es verdadera y la conclusin Q es tambin verdadera (esto
es, la hiptesis y la conclusin son ambas verdaderas), entonces la proposicin condicional si P,
entonces Q debe ser verdadera.
Por otra parte, si la hiptesis P es verdadera y la conclusin Q es falsa, entonces si P, entonces
Q debe ser falsa. (No se debe deducir una conclusin falsa de una hiptesis verdadera). La deni-
cin normal dice que si P, entonces Q es verdadera en caso de que la hiptesis P sea falsa, sin
considerar el valor de verdad de la conclusin Q. En las ulteriores explicaciones, al enlace de varias
proposiciones con si P, entonces Q lo llamaremos implicacin.
En el lenguaje ordinario, la hiptesis y la conclusin en una proposicin implicacin estn
normalmente relacionadas, pero en lgica no se requiere que la hiptesis y la conclusin en una
CAPTULO 1. LGICA MATEMTICA 17
proposicin implicacin se reeran al mismo tema.
Tienen inters particular las proposiciones implicacin verdaderas. Los teoremas de matemti-
cas con frecuencia se expresan como proposiciones implicacin. Una demostracin de un teorema
de esta forma lo constituye la vericacin de que la proposicin implicacin es verdadera.
Sean P

= P(P
1
, P
2
, ..., P
n
) y Q

= Q(P
1
, P
2
, ..., P
n
) proposiciones compuestas y supngase
que P Q es verdadera. Se sabe que si P es falsa, P Q es verdadera, no importando si Q es
verdadera o falsa. Por otra parte, si P es verdadera, Q tambin debe serlo, pues en caso contrario
P Q sera falsa.
Ejemplo 1.15 Sean las siguientes proposiciones:
P : Hoy es 30 de Febrero.
Q : Entre 5 y 15 hay nmeros primos.
P Q : Si hoy es 30 de febrero, entonces entre 5 y 15 hay nmeros primos.
Esta proposicin compuesta es verdadera, ya que podemos decir que toda implicacin, cuyo primer
miembro sea falso tiene el valor de verdad V, sin tener en cuenta si el primero y el segundo
miembros tienen relacin de contenido o no.
Ejemplo 1.16 La proposicin compuesta si entre 5 y 15 hay nmeros primos, entonces entre
13 y 15 hay nmeros primos, es falsa, ya que el primer miembro de esta implicacin es verdadero
y su segundo miembro es falso.
Otra proposicin compuesta de gran utilidad es
P si y slo si Q
Este enunciado se interpreta como:
(Si P, entonces Q) y (si Q, entonces P)
Determnese el valor de verdad de la primera proposicin. Supngase que P y Q son ambas
verdaderas. Entonces las dos proposiciones implicacin de la segunda son verdaderas. Y como la
conectiva y resulta verdadera para ambas verdaderas, se tiene que la segunda tambin lo es.
Dado que la primera se interpreta como la segunda, se considera que la primera es verdadera
cuando ambas P y Q lo son. Si P y Q son falsas, nuevamente las dos proposiciones implicacin
de la segunda son verdaderas. En consecuencia, la segunda es verdadera. Por lo tanto, si ambas
proposiciones P y Q son falsas, se considera que la primera es verdadera.
Si P es falsa y Q es verdadera, entonces la segunda proposicin implicacin en la segunda es
falsa. Ahora bien, cuando en la conectiva y uno de los valores es falso, el resultado es falso. Por
consiguiente, se considera que la primera es falsa si P es falsa y Q es verdadera. Esto motiva la
siguiente denicin.
Denicin 1.13 Equivalencia
Se denomina implicacin o condicional a la funcin veritativa de dos lugares cuyos valores de
verdad se jan de la manera siguiente:
P Q P Q
V V V
V F F
F V F
F F V
CAPTULO 1. LGICA MATEMTICA 18
Otra forma de enunciar P si y slo si Q es P es una condicin necesaria y suciente para
Q.
As mismo, P si y slo si Q en ocasiones se escribe P ssi Q.
La proposicin compuesta (Si P, entonces Q) y (si Q, entonces P), tambin podemos expresarla en
smbologa lgica de la siguiente manera
P Q

= (P Q) (Q P)
Ejemplo 1.17 Sean
P: El nmero 2013 es divisible por 3.
Q: La suma de las cifras bsicas de 2013 es divisible por 3.
P Q: El nmero 2013 es divisible por 3 cuando la sumas de sus cifras bsicas es divisible por 3.
Esta proposicin es verdadera, ya que ambos enlaces son verdaderos.
Ejemplo 1.18 Si P Q es una proposicin implicacin, entonces denominamos:
Q P recproca de P Q.
P Q inversa de P Q.
Q P contrapositiva de P Q.
Ejemplo 1.19 Implicacin: Si 2272 es divisible por 4, entonces 2272 es un nmero par.
Recproca: Si 2272 es un nmero par, entonces 2272 es divisible por 4.
Contrapositiva: Si 2272 no es un nmero par, entonces 2272 no es divisible por 4.
Inversa: Si 2272 no es divisible por 4, entonces 2272 no es un nmero par.
Ejemplo 1.20 Implicacin: Si un tringulo es equiltero, entonces es issceles.
Recproca: Si un tringulo es issceles, entonces es equiltero.
Contrapositiva: Si un tringulo no es issceles, entonces tampoco es equiltero.
Inversa: Si un tringulo no es equiltero, entonces tampoco es issceles.
1.1.2. Tarea
1. Suponga que x, y, z R. Represente en forma simblica los enunciados dados tomando:
P: x < y, Q: y < z, R: x < z
a) (x y e y < z) o x z;
b) No es cierto que (x < y e y < z);
c) x < y o no es verdad que (y < z y x < z);
d) (No es verdad que (x < y y (x < z o y < z))) o (x y y x < z).
2. Sean P, Q, R las proposiciones:
P: Est lloviendo, Q: El Sol est brillando, R: Hay nubes en el cielo.
Traduzca la siguiente notacin lgica, utilizando P, Q, R y conectivos lgicos.
a) Est lloviendo y el Sol est brillando;
b) Si est lloviendo, entonces hay nubes en el cielo;
c) Si no est lloviendo, entonces el Sol no est brillando y hay nubes en el cielo;
d) El Sol est brillando si y slo si no est lloviendo;
e) Si no hay nubes en el cielo, entonces el Sol est brillando.
3. Sean P, Q, R como en el ejercicio anterior. Traduzca lo siguiente a oraciones en espaol:
a) (P Q) R; b) (P R) Q; c) P (Q R); d) (P (Q R));
e) (P Q) R.
CAPTULO 1. LGICA MATEMTICA 19
4. Sean p : tengo un loro y q : tengo un gato, escribir en lenguaje corriente y luego simplicar
(p (q)) (p)
Resp: p (q): tengo un loro y no tengo un gato.
5. A un blanco se han efectuado tres tiros. Sea P
i
la proposicin el blanco ha sido batido
por el i-simo tiro, i = 1, 2, 3. Qu signican las siguientes proposiciones:
a) P
1
P
2
P
3
; b) P
1
P
2
P
3
; c) (P
1
P
2
) P
3
?
Cules de estas tres proposiciones son verdaderas si P
3
es verdadera y P
1
y P
2
, falsas?
6. Cules de las siguientes expresiones son proposiciones? Proporcione los valores de verdad
de las proposiciones:
a) x
2
= x para toda x R; b) x
2
= x para alguna x R; c) x
2
= x;
d) x
2
= x para exactamente una x R; e) xy = xz implica y = z;
f ) xy = xz implica y = z para toda x, y, z R.
7. Considere la frase ambigua x
2
= y
2
implica x = y para todo x, y:
a) Transforme esta frase en una proposicin no ambigua cuyo valor de verdad sea verdadero;
b) Transforme esta frase en una proposicin no ambigua cuyo valor de verdad sea falso.
8. Formule verbalmente las expresiones simblicas contenidas en los siguientes literales, uti-
lizando las proposiciones:
P : Hoy es lunes, Q : Est lloviendo, R : Hace calor.
a) P (Q R); b) (P Q) R; c) (P (Q R)) (R (Q P));
d) (P(P(QR))) (P(RQ)); e) P (QR); f ) (PQ) R;
g) (P (Q R)) (R (Q P)); h) (P (P (Q R))) (P (R Q)).
9. En los siguientes literales, represente cada proposicin en la forma de una proposicin
condicional:
a) Para todo nmero x R, [x[ < 2 siempre que 0 < x < 2;
b) Una condicin suciente para que una funcin f sea integrable es que f sea continua.
10. Enuncie la recproca, la inversa y la contrapositiva de cada uno de los literales del ejercicio
anterior.
11. Proporcione las recprocas, las inversas y las contrapositivas de las siguientes proposiciones:
a) Si soy listo entonces soy rico; b) Si x
2
= x entonces x = 0 o x = 1;
c) Si 2 + 2 = 4 entonces 2 + 4 = 8.
12. a) Muestre que n = 3 es un contraejemplo de la armacin n
3
< 3n para toda n N;
b) Puede encontrar otros contraejemplos?
13. a) Muestre que x = 1 es un contraejemplo de (x + 1)
2
x
2
para toda x R;
b) Encuentre otro contraejemplo;
c) Puede servir de contraejemplo cualquier nmero no negativo? Explique su respuesta.
14. Encuentre contraejemplos de las siguientes armaciones:
a) 2
n
1 es primo para toda n 2; b) 2
n
+ 3
n
es primo para toda n N;
c) 2
n
+n es primo para todo entero impar positivo n.
15. a) Proporcione un contraejemplo para: x > y implica x
2
> y
2
para toda x, y R. Su
respuesta debe ser un par ordenado;
b) Cmo debe restringir x e y para que sea verdadera la proposicin de la parte a)?
CAPTULO 1. LGICA MATEMTICA 20
16. Exprese en forma simblica cada uno de los enunciados, suponiendo que x, y, z R y que
P : x < y, Q : y < z, R : x < z :
a) Si x < y, entonces y z; b) Si (x < y e y < z), entonces x < z;
c) Si (x y e y < z), entonces x z;
d) Si no es verdad que (x < z e y < z), entonces x z;
e) x < y si y slo si (y < z y x < z);
f ) Si es falso que (x < y y (ya sea x < y o y < z)), entonces (x y, entonces x < z).
17. Cules de las proposiciones P, Q, R deben ser verdaderas y cules falsas para que
((P P) Q) R
sea verdadera?
18. Represente simblicamente cada una de las proposiciones condicionales dadas a contin-
uacin. Escriba su recproca, inversa y contrapositiva tanto con smbolos como con palabras.
Determine tambin el valor de verdad para la proposicin condicional, para su recproca,
inversa y para su contrapositiva:
a) Si 4 < 6, entonces 9 > 12; b) Si 4 > 6, entonces 9 > 12;
c) [1[ < 3 si 3 < 1 < 3; d) [4[ < 3 si 3 < 4 < 3.
19. Proporcione la recproca, inversa y contrapositiva de cada una de las siguientes proposi-
ciones:
a) Si x +y = 1 entonces x
2
+y
2
1; b) Si 2 + 2 = 4 entonces 3 + 3 = 8.
20. Considere la proposicin: si x > 0 entonces x
2
> 0 para x N:
a) Proporcione la recproca, inversa y contrapositiva de la proposicin;
b) Cul de las siguientes proposiciones es verdadera: la proposicin original, la recproca,
la inversa o la contrapositiva?
21. Determine los valores de verdad de las siguientes proposiciones compuestas:
a) Si 2 + 2 = 4, entonces 2 + 4 = 8; b) Si 2 + 2 = 5, entonces 2 + 4 = 8;
c) Si 2 + 2 = 4, entonces 2 + 4 = 6; d) Si 2 + 2 = 5, entonces 2 + 4 = 6;
e) Si la tierra es plana, entonces Vicente Rocafuerte fue el primer presidente de Ecuador;
f ) Si la tierra es plana, entonces Sixto Durn-Ballen es presidente de Ecuador en el perido
92 - 96;
g) Si Sixto Durn-Ballen es presidente de Ecuador en el perido 92 - 96, entonces la tierra
es plana;
h) Si Sixto Durn-Ballen es presidente de Ecuador en el perido 92 - 96, entonces 2 + 2 =
4.
22. Supngase que sabemos que P Q es falso. Proporcione los valores de verdad para:
a) P Q; b) P Q; c) Q P; d) P Q; e) P Q;
f ) Q P; g) Q P; h) P Q; i) P Q; j) (P Q).
23. Un lgico le dijo a su hijo Si no terminas tu cena, te irs directo a dormir y no vers
televisin. Termin su cena y fue enviado directamente a la cama. Disctalo.
24. A la pregunta de cul de tres estudiantes estudiaba lgica fue obtenida una respuesta
correcta: si la estudiaba el primero, tambin lo haca el tercero, pero no era cierto que si la
estudiaba el segundo lo haca asmismo el tercero. Quin estudiaba lgica?
CAPTULO 1. LGICA MATEMTICA 21
25. Luis, Carlos, Joe, Fred ocuparon en la olimpiada de matemticas los cuatro primeros
puestos. Cuando les preguntaron acerca de la distribucin de los puestos, dieron las tres
siguientes respuestas:
a) Fred - primero, Carlos - segundo; b) Fred - segundo, Luis - tercero;
c) Joe - segundo, Luis - cuarto.
Cmo se distribuyeron los puestos si en cada una de las respuestas slo una de las arma-
ciones era verdadera?
26. Determine cul de cuatro estudiantes dio el examen si sabemos que:
a) Si lo dio el primero, el segundo tambin;
b) Si lo dio el segundo, el tercero tambin o bien el primero no lo dio;
c) Si no lo dio el cuarto, lo dio el primero, pero el tercero no;
d) Si el cuarto lo dio, el primero tambin.
27. Para una expedicin de ocho pretendientes A, B, C, D, E, F, G, H hay que elegir seis
especialistas: bilogo, hidrlogo, sinptico, radista, mecnico y mdico. Las funciones del
bilogo pueden ser realizadas por E y G, las del hidrlogo, B y F. Las del sinptico, F y G,
las del radista, C y D, las del mecnico, C y H, las del mdico, A y D. Aunque algunos de los
pretendientes tienen dos especialidades, en la expedicin cada uno puede realizar slo una
funcin. Quin y en calidad de qu ha de incluirse en la expedicin si F no puede ir sin B,
D sin H y sin C, C no puede ir simultneamente con G, y A no puede ir junto con B?
1.2. Construccin de tablas de verdad
El enunciado G

= P [(Q R) Q] incluye tres proposiciones: P, Q y R, cada una puede
ser verdadera o falsa de manera independiente. Existen en total 2
3
= 8 combinaciones posibles de
los valores de verdad para P, Q y R y la tabla de verdad para G deber dar el valor de verdad de
G para cada uno de los casos.
Denicin 1.14 Combinaciones
Si una proposicin compuesta G consta de n enunciados, habr 2
n
combinaciones de valores de
verdad, es decir, n las en la tabla de verdad de G.
Una tabla que despliega todos los valores de verdad de una frmula, para todas las posibles
interpretaciones que pueda tener, se denomina tabla de verdad de la frmula. Esta tabla puede
construirse sistemticamente de la siguiente manera:
1. Las primeras n columnas se encabezan con las variables proposicionales; y se construyen ms
columnas para las combinaciones parciales de enunciados y se culmina con el enunciado dado.
2. Bajo cada una de las primeras n columnas, se enlistan las 2
n
n-adas posibles de los valores de
verdad de los componentes del enunciado G. Cada n-tupla se enlista en una la separada.
3. Para cada la se calculan sucesivamente los valores de verdad restantes.
Ejemplo 1.21 Sea G

= (P Q) (P Q), construir la correspondiente tabla de verdad:


P Q P Q P Q G
V V V V V
V F F V F
F V V V V
F F V V F
Ejemplo 1.22 Sea G

= [(P Q) Q] P), construir la correspondiente tabla de verdad:


CAPTULO 1. LGICA MATEMTICA 22
P Q P Q (P Q) Q G
V V V F V
V F F F V
F V V V V
F F F F V
Ejemplo 1.23 Sea G

= [(P Q) P] Q), construir la correspondiente tabla de verdad:


P Q P Q (P Q) P G
V V V F V
V F V F V
F V V V V
F F F F V
Ejemplo 1.24 Sea G

= (P Q) (Q P), construir la correspondiente tabla de verdad:


P Q P Q Q P G
V V V V V
V F F F V
F V V V V
F F V V V
Ejemplo 1.25 Sea G

= (P Q) [(P Q) (Q P)], construir la correspondiente tabla


de verdad:
P Q P Q P Q Q P (P Q) (Q P) G
V V V V V V V
V F F F V F V
F V F V F F V
F F V V V V V
1.2.1. Operaciones con frmulas lgicas y sus propiedades
En el estudio de las funciones proposicionales hemos utilizado las variables P, Q, R, ... para
designar las proposiciones. Estas variables podemos interpretarlas con elementos de un dominio
bsico, es decir, con proposiciones. Su dominio est formado solamente por dos elementos, los
valores de verdad V y F. Las constantes en este caso las constituyen los conectores lgicos. Mediante
el enlace lineal de las variables con valores de verdad P, Q, etc., y conectores, as como mediante
la aplicacin de los signos tcnicos (parntesis), podemos formar series de signos.
Denicin 1.15 Frmula bien formada
Una frmula bien formada, se dene dentro de la lgica proposicional en los siguientes trminos
recursivos:
1) Las variables P, Q, ... son frmulas.
2) a) Si P es una frmula, entonces P tambin es una frmula.
b) Si P y Q son frmulas entonces P Q, P Q, P Q, P Q tambin son frmulas.
3) Una serie de signos P, Q, ... es una frmula solo cuando se trata de los casos 1 y 2.
En la representacin simblica se interpretan los signos , , , , , que reciben el nom-
bre de conectores, como signos de funciones proposicionales y tambin como signos de funciones
veritativas. A los literales tales como P, Q, R,... que son usados para denotar proposiciones se
denominan frmulas atmicas o tomos. No es difcil reconocer que expresiones como P , P no
CAPTULO 1. LGICA MATEMTICA 23
son frmulas. Cuando no exista confusin se suprimen los parntesis asignando rangos decrecientes
a los conectores proposicionales de la siguiente manera; , , , , de manera que al conector
proposicional con mas alto rango se lo evalue al nal.
Ejemplo 1.26 1) P Q R = P (Q R);
2) P Q R S = P Q (R S) = P [Q (R S)].
Ahora vamos a establecer una relacin entre los valores de verdad y las funciones veritativas
por una parte y las expresiones, por otra. Las variables P, Q, ... las utilizamos ahora como variables
del valor de verdad, y de igual forma los conectores proposicionales , , , , como signos de
las funciones veritativas clsicas.
Sobre la base de las armaciones hechas podemos indicar el correspondiente valor de verdad
para cada interpretacin de las variables P, Q, ... con los valores de verdad. En las expresiones
complicadas de la lgica proposicional tambin es posible calcular de esta forma, en nitos pasos,
los valores de verdad, al hacer las diferentes interpretaciones de las variables.
Comparando las tablas de verdad podemos decidir si dos frmulas G y H tienen la misma tabla
de valores de verdad. Con esto tambin podemos mostrar si una frmula formada a partir de G
y H, es una identidad de la lgica proposicional. La igualdad de las tablas de valores de verdad
y la identidad de la lgica proposicional, sin embargo, no son exactamente lo mismo. La igualdad
de la tabla de valores de verdad es una relacin entre dos frmulas; y la propiedad de ser una
identidad es una peculiaridad de una frmula. Cuando nos interesamos por la igualdad de la tabla
de valores de verdad, entonces comparamos los valores de verdad de dos frmulas en todas las
sustituciones posibles. Cuando nos interesamos por la validez general de una frmula, queremos
establecer si esta determinada frmula toma, en cada interpretacin, el valor de verdad V. En este
caso, se determina el valor de verdad de una nueva frmula formada a partir de las frmulas G y
H en todas las sustituciones posibles. De las frmulas con las mismas tablas de verdad, G y H, se
pueden formar siempre identidades de la lgica proposicional, es decir, frmulas de validez general.
Teorema 1.3 Una frmula doblemente negada tiene la misma tabla de valores de verdad que la
correspondiente frmula dada, es decir; P

= P es una identidad de la lgica proposicional.
Demostracin
P P P
V F V
F V F
Teorema 1.4 Para la conjuncin, la disjuncin y la equivalencia se cumplen la ley conmutativa
y la ley asociativa con respecto a la igualdad de las tablas de valores de verdad. Para la implicacin
no se cumple ni la ley asociativa, ni la ley conmutativa.
Demostracin
P Q P Q Q P P Q Q P P Q Q P
V V V V V V V V
V F V V F F F F
F V V V F F F F
F F F F F F V V
Dado que G
1
= (P Q) R y G
2
= P (Q R), entonces
CAPTULO 1. LGICA MATEMTICA 24
P Q R (P Q) R P (Q R) (P Q) R P (Q R) G
1
G
2
V V V V V V V V V
V V F V V F F F F
V F V V V F F F F
V F F V V F F V V
F V V V V F F F F
F V F V V F F V V
F F V V V F F V V
F F F F F F F F F
En lgica las proposiciones idnticamente verdaderas o bien idnticamente falsas desempean
importante papel. Las proposiciones idnticamente verdaderas son siempre verdaderas independi-
ente de si las proposiciones que las forman son verdaderas o falsas.
Teorema 1.5 Para las proposiciones idnticamente verdaderas e idnticamente falsas, con todo
P son ciertas las siguientes frmulas:
P P

= V; P V

= V; P F

= P
P P

= F; P V

= P; P F

= F
Demostracin
P P Q P V P F P P P V P F
V V V V F V F
F V V F F F F
Teorema 1.6 Las equivalencias siguientes
P Q

= Q P; P Q

= Q P
P Q

= Q P; P Q

= Q P
son identidades de la lgica proposicional.
Demostracin
P Q P Q Q P P Q Q P
V V V V V V
V F F F V V
F V V V V V
F F V V F F
P Q Q P P Q Q P
F F V V
V V V V
V V F F
V V V V
Teorema 1.7 Las equivalencias siguientes
(P Q)

= P Q; (P Q)

= P Q
(P Q) P

= P; (P Q) P

= P
(P Q) Q

= Q; (P Q) Q

= Q
P Q

= P Q; P Q

= (P Q) (Q P)
son identidades de la lgica proposicional.
Demostracin
CAPTULO 1. LGICA MATEMTICA 25
P Q (P Q) P Q (P Q) P Q (P Q) P (P Q) P
V V F F F F V V
V F F F V V V V
F V F F V V F F
F F V V V V F F
(P Q) Q (P Q) Q P Q P Q P Q (P Q) (Q P)
V V V V V V
F F F F F F
V V V V F F
F F V V V V
Teorema 1.8 La conjuncin es, con respecto a la disjuncin en ambos lados, distributiva y
viceversa, es decir, que las siguientes frmulas son identidades de la lgica proposicional
P (Q R)

= (P Q) (P R); (Q R) P

= (Q P) (R P)
P (Q R)

= (P Q) (P R); (Q R) P

= (Q P) (R P)
Demostracin
P Q R P (Q R) (P Q) (P R) (Q R) P (Q P) (R P)
V V V V V V V
V V F V V V V
V F V V V V V
V F F F F F F
F V V F F F F
F V F F F F F
F F V F F F F
F F F F F F F
P (Q R) (P Q) (P R) (Q R) P (Q P) (R P)
V V V V
V V V V
V V V V
V V V V
V V V V
F F F F
F F F F
F F F F
Teorema 1.9 Conjuntamente con la distributividad se cumple que la implicacin, con respecto
a las dems funciones veritativas, es distributiva a la derecha, pero no distributiva a la izquierda,
es decir, que las siguientes frmulas son de validez general
P (Q R)

= (P Q) (P R); P (Q R)

= (P Q) (P R)
P (Q R)

= (P Q) (P R); P (Q R)

= (P Q) (P R)
Demostracin
CAPTULO 1. LGICA MATEMTICA 26
P Q R P (Q R) (P Q) (P R) P (Q R) (P Q) (P R)
V V V V V V V
V V F F F V V
V F V F F V V
V F F F F F F
F V V V V V V
F V F V V V V
F F V V V V V
F F F V V V V
P (Q R) (P Q) (P R) P (Q R) (P Q) (P R)
V V V V
F F F F
V V F F
V V V V
V V V V
V V V V
V V V V
V V V V
Teorema 1.10 Si la conclusin, segundo miembro, de una implicacin es igualmente una impli-
cacin, entonces las dos premisas (primeros miembros), se pueden unir formando una sola premisa
P (Q R)

= (P Q) R; (P Q) R

= (P R) (Q R)
Demostracin
P Q R P (Q R) (P Q) R (P Q) R (P R) (Q R)
V V V V V V V
V V F F F F F
V F V V V V V
V F F V V V V
F V V V V V V
F V F V V V V
F F V V V V V
F F F V V V V
Ejemplo 1.27 Utilizando las leyes de la lgica proposicional, demostrar que:
(P Q) (P Q)

= (P Q) (P Q).
Solucin
(P Q) (P Q)

= [(P Q) (P Q)] [(P Q) (P Q)]

= [(P Q) (P Q)] [(P Q)) (P Q)]

= [(P Q) (P Q)] [(P Q) (P Q)]

= (P Q P Q) [(P Q) (P Q)]

= [(P P) (Q Q)] [(P Q) (P Q)]

= (V V) [(P Q) (P Q)]

= V [(P Q) (P Q)]

= (P Q) (P Q).
CAPTULO 1. LGICA MATEMTICA 27
Ejemplo 1.28 Utilizando las leyes de la lgica proposicional, demostrar que:
[(P Q) P] Q

= Q P.
Solucin
[(P Q) P] Q

= [(P Q) P] Q

= [(P Q) P] Q

= (P) Q

= P Q

= Q P.
Ejemplo 1.29 Utilizando las leyes de la lgica proposicional, demostrar que:
[(P Q) (P R)] (Q R)

= Q (P R).
Solucin
[(P Q) (P R)] (Q R)

= [(P Q) (P R)] (Q R)

= (P Q) (P R) (Q R)

= (P Q) (P R) Q R

= Q R (P R)

= Q [(R P) (R R)]

= Q [(R P) V]

= Q (R P)

= Q (P R).
Ejemplo 1.30 Utilizando las leyes de la lgica proposicional, demostrar que:
[(P Q) R] [(Q P) R]

= (P Q) R.
Solucin
[(P Q) R] [(Q P) R]

= [(P Q) R] [(Q P) R]

= [(P Q) R] [(Q P) R]

= [(P Q) R] [(Q P) R]

= [(P Q) R] [(Q P) R]

= [(P Q) (Q P) R] [R (Q P) R]

= [(P Q Q) (P Q P)] R V

= [(P Q) (P Q)] R

= (P Q) R

= (P Q) R

= (P Q) R.
Ejemplo 1.31 Utilizando las leyes de la lgica proposicional, demostrar que:
[(P Q) P] (P Q)

= P Q.
CAPTULO 1. LGICA MATEMTICA 28
Solucin
[(P Q) P] (P Q)

= [(P Q) P] (P Q)

= [(P Q) P] (P Q)

= P P Q

= P Q

= P Q.
1.2.2. Tautologas y falacias
Denicin 1.16 Tautologa
Si una proposicin compuesta es siempre verdadera bajo todas sus interpretaciones, independien-
temente de los valores de vericacin de sus componentes, decimos que la proposicin compuesta
es una tautologa.
Es decir, a un enunciado que es verdadero para todos los valores posibles de sus variables
proposicionales se le denomina tautologa. Cuando se comprueba que una equivalencia es una
tautologa, signica que sus dos partes componentes son siempre o ambas verdaderas o ambas
falsas, para cualesquier valores de las variables proposicionales. Por tanto los dos lados son slo
diferentes maneras de proponer el mismo enunciado y se dice que son logicamente equivalentes.
Denicin 1.17 Falacia
Una frmula G es una falacia, si G es una tautologa.
Ejemplo 1.32 Utilizando una tabla de verdad, determinar si la frmula
G = (P Q) (P Q)
es tautologa.
Solucin
P Q P Q P Q (P Q) ( P Q)
V V V V V
V F F F V
F V V V V
F F V V V
Por lo tanto G si es tautologa.
Ejemplo 1.33 Utilizando una tabla de verdad, determinar si la frmula
G = (Q P) (P Q)
es tautologa.
Solucin
P Q Q P P Q (Q P) (P Q)
V V V V V
V F V F F
F V F V V
F F V V V
Por lo tanto G no es tautologa.
CAPTULO 1. LGICA MATEMTICA 29
Ejemplo 1.34 Utilizando una tabla de verdad, determinar si la frmula
G = (P Q) (Q P)
es tautologa.
Solucin
P Q P Q Q Q (P Q) ( Q P)
V V V V V
V F F F V
F V V V V
F F V V V
Por lo tanto G si es tautologa.
Ejemplo 1.35 Utilizando una tabla de verdad, determinar si la frmula
G = (P Q) [(P Q) (Q P)]
es tautologa.
Solucin
P Q P Q (P Q) (Q P (P Q) [(P Q) (Q P)]
V V V V V
V F F F V
F V F F V
F F V V V
Por lo tanto G si es tautologa.
Ejemplo 1.36 Utilizando una tabla de verdad, determinar si la frmula
G = [(P Q) (Q R)] (P R)
es tautologa.
Solucin
P Q R (P Q) (Q P P Q [(P Q) (Q R)] (P R)
V V V V V V
V V F F F V
V F V F V V
V F F V F V
F V V V V V
F V F F V V
F F V V V V
F F F V V V
Por lo tanto G si es tautologa.
Ejemplo 1.37 Utilizando una tabla de verdad, determinar si la frmula
G = [P (Q R)] [(P Q) R]
es tautologa.
Solucin
CAPTULO 1. LGICA MATEMTICA 30
P Q R (P (Q R) (P Q) R [P (Q R)] [(P Q) R]
V V V V V V
V V F F F V
V F V V V V
V F F V V V
F V V V V V
F V F V F F
F F V V V V
F F F V F F
Por lo tanto G no es tautologa.
1.2.3. Tarea
1. Construya la tabla de verdad para cada una de las siguientes proposiciones:
a) (P Q) [(P Q) (P Q)]; b) [(P Q) R] (P Q);
c) [(P Q) (P R)] (Q P)]; d) P P; e) (P Q) R;
f ) (P P) P; g) P Q; h) (P Q).
2. Utilizando las leyes de la lgica proposicional, demostrar que
P Q

= (P Q) (P Q)
3. Utilizando las leyes de la lgica proposicional, demuestre o refute:
a) P Q

= (P Q) (P Q); b) P (Q R)

= (P Q) (P R);
c) (P Q) R

= P (Q R).
4. Simplique las siguientes frmulas y diga cuales son tautologas y cuales falacias:
a) P (P Q)] (P Q); b) (P Q) [(R P) Q].
5. Simplique las siguientes frmulas y diga cuales son tautologas y cuales falacias:
a) (R Q) (P Q R) (P Q R); b) (P Q) (R Q);
c) (P Q) Q [(R Q) P].
6. Simplique las siguientes frmulas y diga cuales son tautologas y cuales falacias:
a) (P Q) (R S) (P S); b) (P Q) (P R) R;
c) (P Q) (P R) (Q S).
7. Simplique las siguientes frmulas y diga cuales son tautologas y cuales falacias:
a) (P Q) Q (P R); b) (P Q) (P R) (Q R);
c) (P Q) Q (P (R S)].
8. Simplique las siguientes frmulas y diga cuales son tautologas y cuales falacias:
a) (P S) (P Q) [(S R) T] (Q R);
b) P (Q P) [(Q R) S];
c) (P Q) (R Q) (R S) [(S P) T].
9. Simplique las siguientes frmulas y diga cuales son tautologas y cuales falacias:
a) (P Q) [(P Q) (Q P)]; b) [(P Q) (Q R) P] R;
c) [P (Q R)] (Q R) [(S R) P] S.
CAPTULO 1. LGICA MATEMTICA 31
1.3. Transformacin de frmulas
La igualdad de los valores de verdad de dos proposiciones la hemos demostrado hasta ahora
utilizando las tablas completas de valores de verdad. Con su ayuda pudimos decidir si una frmula
dada es o no una identidad de la lgica proposicional.
Por esta va hemos conocido mumerosas frmulas con las tablas de valores. Otras identidades,
es decir; las leyes de la lgica proposicional, las obtenemos a partir de las frmulas dadas y medi-
ante sustituciones o transformaciones en frmulas equivalentes.
En esta seccin veremos cmo obtener equivalencias e implicaciones lgicas sin utilizar tablas
de verdad. Tambin explicaremos el signicado de teorema y de demostracin. Empezaremos con
dos reglas tiles, que sin embargo deben manejarse con cuidado.
Teorema 1.11 Si en una frmula de validez general, es decir, en una identidad de la lgica
proposicional, se sustituye una variable proposicional por una frmula cualquiera en todos los lu-
gares donde se presenta la frmula correspondiente, entonces se obtiene nuevamente una frmula
de validez general.
Teorema 1.12 Cuando en una frmula G se sustituye una cierta subfrmula G
1
por una fr-
mula G
2
, que toma los mismos valores de verdad que G
1
, entonces la frmula obtenida F tiene los
mismos valores de verdad que la frmula G. La frmula G, una vez sustituida G
1
debe sustituirse
por G
2
en todos los lugares donde esta se presenta.
Ejemplo 1.38 Consideremos la proposicin
G

= [P (P Q)] Q
que es una tautologa. Si reemplazamos, cada vez que aparece P, por la proposicin
G
1

= Q R
obtenemos la tautologa
H

= [(Q R) ((Q R) Q)] Q.


Si en cambio reemplazamos Q, cada vez que aparece, por G
1
, obtenemos la tautologa
H

= [P (P (Q R))] (Q R).
Ejemplo 1.39 Consideremos la proposicin
G

= [(P Q) (P R)] [Q (P R)]


que no es una tautologa. Obtenemos proposiciones lgicamente equivalentes si reemplazamos P Q
por su equivalencia lgica P Q o si reemplazamos una o las dos veces que aparece P R por
P R. Podemos tambin reemplazar (P Q) (P R) por P (Q R). De esta manera G
es lgicamente equivalente a las siguientes proposiciones entre otras:
[(P Q) (P R)] [Q (P R)]
[(P Q) (P R)] [Q (P R)]
[(P (Q R)] [Q (P R)].
CAPTULO 1. LGICA MATEMTICA 32
Denicin 1.18 Frmula vlida
Una frmula G es vlida o constituye una tautologa, si y slo si es verdadera bajo todas las
interpretaciones. En caso contrario la frmula G es invlida.
Denicin 1.19 Frmula inconsistente
Una frmula G se denomina inconsistente o insatisfactible, si y slo si es falsa bajo todas las
interpretaciones. En caso contrario la frmula G es consistente o satisfactible.
De las deniciones anteriores, las observaciones siguientes son obvias:
1. Una frmula es vlida, si y slo si su negacin es inconsistente.
2. Una frmula es inconsistente, si y slo si su negacin es vlida.
3. Una frmula es invlida, si y slo si hay por lo menos una interpretacin bajo la cual la
frmula es falsa.
4. Una frmula es inconsistente, si y slo si hay por lo menos una interpretacin bajo la cual
la frmula es verdadera.
5. Si una frmula es vlida, entonces es consistente pero no viceversa.
6. Si una frmula es inconsistente, entonces es invlida pero no viceversa.
Ejemplo 1.40 Vericar la validez o inconsistencia de la frmula:
[(P Q) (Q R)] (P R)
Solucin
P Q R (P Q) (Q R) P R [(P Q) (Q R)] (P R)
V V V V V V
V V F F F V
V F V F V V
V F F V F V
F V V V V V
F V F F V V
F F V V V V
F F F V V V
Por lo tanto G es una frmula vlida.
Ejemplo 1.41 Vericar la validez o inconsistencia de la frmula:
[(P (Q R)] [(P Q) R]
Solucin
P Q R (P (Q R) (P Q) R [(P (Q R)] [(P Q) R]
V V V V V V
V V F F F V
V F V V V V
V F F V V V
F V V V V V
F V F V F F
F F V V V V
F F F V F F
Por lo tanto G no es una frmula vlida.
CAPTULO 1. LGICA MATEMTICA 33
1.3.1. Formas normales
En lgica matemtica es muy importante el poder transformar frmulas de una forma a otra,
especialmente a las denominadas formas normales. Para lograr estas transformaciones de frmulas,
se utiliza el concepto de equivalencias de frmulas.
Denicin 1.20 Frmulas equivalentes
Las frmulas G y H son equivalentes si los valores de verdad de G y H son los mismos bajo todas
las interpretaciones de estas frmulas.
Por supuesto que nuestro inters no se limita a estudiar una simple clasicacin de los enun-
ciados del lenguaje; pero tampoco intentamos internarnos en el fascinante mundo de la deduccin
lgica sin antes estar seguros de conocer y comprender algunos conceptos elementales. Las dos
formas normales que nos interesa obtener y que son utilizadas en prueba mecnica de teoremas,
son la forma normal conjuntiva y la forma normal disjuntiva.
Denicin 1.21 Forma normal conjuntiva
Una frmula G se dice que est en forma normal conjuntiva si y slo si G tiene la forma
G

= G
1
G
2
G
n
n N
donde cada una de las frmulas G
1
, G
2
, ..., G
n
, se expresan como una conjuncin de literales.
Ejemplo 1.42 Expresar la frmula
G

= (Q P) (P Q)
en forma normal conjuntiva.
Solucin
(Q P) (P Q)

= (Q P) (P Q)

= (Q P) (P Q)

= (Q P) (P Q)

= [Q (P Q)] [P (P Q)].
Ejemplo 1.43 Expresar la frmula
G

= (P Q) [(P Q) (Q P)]
en forma normal conjuntiva.
Solucin
(P Q) [(P Q) (Q P)]

= (P Q) (P Q)

= [(P Q) (P Q)] [(P Q) (P Q)].


Denicin 1.22 Forma normal disjuntiva
Una frmula G se dice que est en forma normal disjuntiva si y slo G si tiene la forma
G

= G
1
G
2
G
n
n N
donde cada una de las frmulas G
1
, G
2
, ..., G
n
, se expresan como una disjuncin de literales.
CAPTULO 1. LGICA MATEMTICA 34
Ejemplo 1.44 Expresar la frmula
G

= (Q P) (P Q)
en forma normal disjuntiva.
Solucin
(Q P) (P Q)

= (Q P) (P Q)
Ejemplo 1.45 Expresar la frmula
G

= (P Q) [(P Q) (Q P)]
en forma normal conjuntiva.
Solucin
(P Q) [(P Q) (Q P)]

= (P Q) (P Q)

= [(P Q) (P Q)] [(P Q) (P Q)]

= (P Q) (P Q)

= (P Q) (P Q)
Un hecho que es muy importante anotar, es que cualquier frmula de la lgica proposicional
puede ser transformada a una de las formas normales, utilizando las leyes de la lgica proposicional.
1.3.2. Consecuencias lgicas
Denicin 1.23 Consecuencia lgica
Dadas las frmulas G
1
, G
2
, ..., G
n
y una frmula G, G se denomina consecuencia lgica de G
1
,
G
2
, ..., G
n
si y slo si para cualquier interpretacin en la cual G
1
G
2
G
n
es verdad, G
tambin lo es G
1
, G
2
, ..., G
n
se denominan axiomas de G.
Teorema 1.13 Dadas las frmulas G
1
, G
2
, ..., G
n
y una frmula G, G es una consecuencia
lgica de G
1
, G
2
, ..., G
n
si y slo si la frmula (G
1
G
2
G
n
) G es vlida.
Demostracin
Suponga que G es una consecuencia lgica de G
1
, G
2
, ..., G
n
. Sea I una interpretacin ar-
bitraria. Si G
1
, G
2
, ..., G
n
son verdaderos en I, entonces por denicin de consecuencia lgica
G es verdadero en I. Entonces (G
1
G
2
G
n
) G es verdadero en I. Por otra parte, si
G
1
, G
2
, ..., G
n
son falsos en I, entonces (G
1
G
2
G
n
) G es verdadero en I. As, de-
mostramos que (G
1
G
2
G
n
) G es verdadero bajo cualquier interpretacin. Esto es,
(G
1
G
2
G
n
) G es una frmula vlida.
Supongamos que (G
1
G
2
G
n
) G es una frmula vlida. Para cualquier interpretacin
I, si G
1
G
2
G
n
es verdadero en I, G debe ser verdadero en I. Por consiguiente G es una
consecuencia lgica de G
1
, G
2
, ..., G
n
.
Ejemplo 1.46 Sean
G
1
P (Q R)
G
2
(P S) R
G Q
Pruebe si G es consecuencia lgica de G
1
y G
2
.
Solucin
CAPTULO 1. LGICA MATEMTICA 35
Debemos probar que la frmula [P (Q R)] [(P S) R] Q, es verdadera o falsa,
decir:
[P (Q R)] [(P S) R] Q

= [P (Q R)] [P S R] Q

= [(P Q R) (P S R)] Q

= (P Q R) (P S R) Q

= (P Q R) (P Q R) S

= V.
Lo cual indica que G es consecuencia lgica de G
1
y G
2
.
Ejemplo 1.47 Sean
G
1
P Q)
G
2
Q R
G
3
R
G R
Pruebe si G es consecuencia lgica de G
1
, G
2
y G
3
.
Solucin
Debemos probar que la frmula [(P Q) (Q R) R] R, es verdadera o falsa, decir:
[(P Q) (Q R) R] R

= [(P Q) (Q R) R] R

= (P Q) (Q R) (R R)

= V.
Lo cual indica que G es consecuencia lgica de G
1
, G
2
y G
3
.
Teorema 1.14 Dadas las frmulas G
1
, G
2
, ..., G
n
y una frmula G, G es una consecuencia
lgica de G
1
, G
2
, ..., G
n
si y slo si la frmula G
1
G
2
G
n
G es inconsistente.
Demostracin
Por el teorema anterior, G es una consecuencia lgica de G
1
, G
2
, ..., G
n
si y slo si la frmula
(G
1
G
2
G
n
) G es vlida. As, G es una consecuencia lgica de G
1
, G
2
, ..., G
n
si y slo
si la negacin de (G
1
G
2
G
n
) G es inconsistente
[(G
1
G
2
G
n
) G]

= [(G
1
G
2
G
n
) G]

= (G
1
G
2
G
n
) G

= (G
1
G
2
G
n
) G

= G
1
G
2
G
n
G
Por lo tanto, concluimos que el teorema es verdadero.
Ejemplo 1.48 Sean
G
1
P (Q R)
G
2
(P S) R
G Q
Pruebe si G es consecuencia lgica de G
1
y G
2
.
Solucin
CAPTULO 1. LGICA MATEMTICA 36
Debemos probar que la frmula [P (Q R)] [(P S) R] Q, es verdadera o falsa,
decir:
[P (Q R)] [(P S) R] Q

= [(P Q R) (P S R)] Q

= (P Q R) (P Q R) S

= (P Q R) (P Q R) S

= F.
Lo cual indica que G es consecuencia lgica de G
1
y G
2
.
Ejemplo 1.49 Sean
G
1
P Q)
G
2
Q R
G
2
R
G R
Pruebe si G es consecuencia lgica de G
1
, G
2
y G
3
.
Solucin
Debemos probar que la frmula [P Q) (Q R) R] R, es verdadera o falsa, decir:
[P Q) (Q R) R] R

= (P Q) (Q R) (R R)

= F.
Lo cual indica que G es consecuencia lgica de G
1
, G
2
y G
3
.
1.3.3. Tarea
1. Determine la validez o inconsistencia, luego transforme a una de sus formas normales las
siguientes frmulas::
a) [P (P Q)] (P Q);
b) (P Q) [(R P) Q];
c) (R Q) (P Q R) (P Q R);
d) (P Q) Q [(R Q) P].
2. Determine la validez o inconsistencia, luego transforme a una de sus formas normales las
siguientes frmulas:
a) (P Q) (R S) (P S);
b) (P Q) (P R) R;
c) (P Q) (P R) (Q S);
d) (P Q) Q (P R);
e) (P Q) (P R) (Q R);
f ) [(P Q) (Q R) P] R.
3. Determine la validez o inconsistencia, luego transforme a una de sus formas normales las
siguientes frmulas:
a) (P Q) Q [P (R S)];
b) (P S) (P Q) [(S R) T] (Q R);
c) P (Q P) [(Q R) S];
d) (P Q) (R Q) (R S) [(S P) T];
e) (P Q) [(P Q) (Q P)];
f ) [P (Q R)] (Q R) [(S R) P] S.
CAPTULO 1. LGICA MATEMTICA 37
4. Decir cual de los siguientes enunciados son consecuencia lgica:
a)
G
1
(P Q) R)
G
2
S T
G
3
U L
G
4
P U
G
5
S L
G R
b)
G
1
(P Q) R
G
2
R S
G
3
(P Q)
G
4
(S T) U
G U
c)
G
1
(P Q) (R S)
G
2
(P Q)
G
3
(R S) (T U)
G T U
5. Los alumnos son estudiosos o los estudiosos reprueban. Si los estudiosos reprueban, en-
tonces los inteligentes son felices o los alumnos no son estudiosos. Los alumnos son estudiosos
y los inteligentes no son felices. No es verdad que los inteligentes son felices. Los estudiantes
no reprueban?
6. Juego ftbol o estudio. Si paso el examen no estudio. Sucede que no voy a jugar ftbol.
En consecuencia no pas el examen.
7. La lgica es fcil. Si el lgebra es hermosa, entonces la Lgica no es fcil o la Matemtica
es la reina de las ciencias. El Algebra es hermosa. En consecuencia, la Matemtica es la reina
de las ciencias.
8. Ayer no fue mircoles o maana no es martes. Hoy es jueves y ayer fue mircoles. Hoy es
lunes si y slo si maana es martes. En consecuencia, hoy es lunes.
9. Luis har un viaje a Europa si logra terminar su carrera. Luis termina su carrera, y si
hace un viaje a Europa, entonces no asiste a nuestra reunin anual. En consecuencia, Luis
no asistir a nuestra reunin anual.
10. Si faltan ejercicios o encuentro premisas, entonces acabo la tarea. Si el libro est claro y no
me falta creatividad, entonces encuentro premisas. No acabo la tarea. En consecuencia me
falta creatividad o el libro no est claro.
11. Si ganamos el campeonato, recibimos el premio. Si jugamos y ganamos el campeonato,
recibiremos el premio. Jugaremos y ganaremos el campeonato. En consecuencia, recibiremos
el premio.
CAPTULO 1. LGICA MATEMTICA 38
12. Repruebo el examen o sigo mis estudios. Si repruebo el examen, perder la beca y me ir
de la ciudad. No perder la beca o no me ir de la ciudad. Luego, seguire estudiando.
13. Los aviones son veloces o las diligencias respetan los semforos. Si los hombres vuelan y las
bicicletas no contaminan, entonces no es verdad que las diligencias respetan los semforos.
Los hombres vuelas y las bicicletas no contaminan. En conclusin, los aviones son veloces.
1.4. Expresiones de la lgica de predicados
El clculo proposicional es una teora de la lgica, completa y autnoma, pero totalmente inade-
cuada para la mayor parte de las matemticas. El problema reside en que el clculo proposicional
no permite el uso de un nmero innito de proposiciones. Adems, la notacin es difcil para mane-
jar un gran nmero nito de proposiciones.
Por ejemplo, con frecuencia encontramos una sucesin innita de proposiciones P(x) con ndices
en N. La armacin informal P(x) es verdadera para toda x signica P(0) es verdadera, P(1)
es verdadera, P(2) es verdadera, etc. El nico simbolismo que podramos utilizar, segn el clculo
proposicional sera P(0) P(1) P(2) ..., pero no es aceptable en el clculo proposicional.
En forma similar, la armacin informal P(x) es verdadera para alguna x correspondera al
inaceptado P(0) P(1) P(2) .... Para darle la vuelta a este problema, necesitamos dos smbolos
nuevos: uno que signique para todo y otro que signique para algn.
Entonces necesitamos saber las reglas para utilizar los nuevos smbolos y combinarlos con los
viejos. Este sistema de smbolos y reglas se llama clculo de predicados. Los nuevos smbolos que
introduciremos se llaman cuanticadores.
Supongamos que P(x)/x U es una familia con ndices en un conjunto U que puede se
innito; el conjunto U se llama el dominio de individuos o universo de individuos.
Mediante la introduccin de existe ... es conrmada la existencia de por lo menos un elemento
del conjunto base que satisface la forma proposicional dada. Esta proposicin es una proposicin
existencial. Proposiciones con la formulacin una parte, casi todo, la mayora, algunos, etc., son
tambin proposiciones existenciales. Cuando hablamos de proposiciones existenciales, nos referi-
mos tambin a proposiciones particulares, ya que estas no se reeren a todos los elementos del
conjunto que nos interesa, sino solo a una parte. En este caso denominamos a la cuanticacin,
particularidad.
De forma anloga, se denomina a las proposiciones en que aparece la formulacin para todos,
proposiciones universales o generales, ya que estas se reeren a todos los elementos del conjunto de
variables. Tal cuanticacin se denomina tambin generalizacin. La cuanticacin particularidad
y generalizacin son operaciones de la lgica de predicados.
Partiendo de las formas proposicionales relacionadas previamente con los operadores, tales
como existe ..., para todo ..., no existe ningn ..., hemos obtenido proposiciones falsas o
verdaderas. Para estos operadores denominados tambin cuanticadores, se han introducido en la
lgica matemtica signos especiales.
CAPTULO 1. LGICA MATEMTICA 39
El cuanticador existencial (particularizador) existe (por lo menos) un ... es simbolizado con
?. Si el smbolo ? se encuentra ante una forma proposicional P(x), esto quiere decir que existe
por lo menos un elemento del conjunto fundamental que posee la propiedad reejada en la forma
proposicional P(x). Utilizamos las escrituras x P(x).
La tachadura vertical o la relacin que se establece entre el smbolo y el smbolo ,, debe
expresar que no existe ningn elemento del conjunto fundamental que posea la propiedad indicada
en la forma proposicional P(x).
El cuanticador universal (operador universal, generalizador) para todo ... se representa con
el smbolo . Si el smbolo se encuentra ante una forma proposicional P(x), esto quiere
decir que la propiedad reejada en la forma proposicional P(x) es aplicable para cada elemento
del dominio de individuos. El cuanticador universal forma pareja con una variable, x, signica,
para todo x ....
La tachadura vertical o la relacin que se establece entre el cuanticador universal y el smbolo
, debe expresar que la propiedad reejada en P(x) no es aplicable para todos los elementos del
dominio de individuos.
La lgica de predicados o lgica de primer grado, nos ensea que para la cuanticacin slo
son admisibles las variables de individuos. Las variables de individuos cuanticados dejan de ser
variables libres para convertirse en variables ligadas. Para crear expresiones de la lgica de predi-
cados utilizamos adems de los smbolos para las variables de individuos, constantes de individuos,
variables predicativas, cuanticadores y los conectores de la lgica proposicional.
En la lgica proposicional comprobamos el valor de verdad de una expresin mediante la susti-
tucin de las variables de dicha expresin por sus valores de verdad, teniendo en cuenta las disposi-
ciones correspondientes. El valor de verdad de una expresin de la lgica de predicados depende
no solo del cuanticador sino tambin de las variables de individuos y del conjunto de individuos
tomado como base, as como de la sustitucin o interpretacin de las variables predicativas.
A la proposicin compuesta x P(x) se le asignan valores de verdad de la manera siguiente:
x P(x) es verdadero si P(x) es verdadero para toda x en U; en cualquier otro caso x P(x) es
falsa
La proposicin compuesta x P(x) tiene los siguientes valores de verdad:
x P(x) es verdadero si P(x) es verdadera para al menos una x en U; x P(x) es falso si P(x)
es falsa para toda x en U
Analicemos la proposicin x P(x) de manera ms detallada. La expresin P(x) se llama
predicado. Para formar una oracin hay que tener un sujeto. Por ejemplo, el predicado ... es mas
poblada que Quito se transforma en la oracin Guayaquil es mas poblada que Quito al dar co-
mo sujeto Guayaquil. Si llamamos P al predicado la oracin podra escribirse como P(Guayaquil).
Cada sujeto da una oracin.
En nuestra lgica simblica dar un predicado es establecer una funcin que produce una proposi-
cin siempre que le demos un elemento del dominio de individuos, esto es, una funcin proposicional
- valuada con dominio de individuos U. Seguimos nuestra prctica usual y denotamos tal funcin
por P(x). La variable x en la expresin P(x) se llama variable libre del predicado. En tanto x
vara en U los valores de verdad de P(x) pueden variar. En contraste, la proposicin x P(x)
CAPTULO 1. LGICA MATEMTICA 40
tiene un signicado jo y un valor de verdad que no vara con x. La variable x en x P(x) se
llama variable acotada; est acotada por el cuanticador . Como x P(x) tiene un signicado jo
y un valor de verdad sera intil y poco natural cuanticarla de nuevo. Esto es, sera vano intro-
ducir x[ x P(x)] y x[ x P(x)] ya que sus valores de verdad son los mismos que los de x P(x).
Podemos tambin considerar predicados que son funciones de ms de una variable, posiblemente
de ms de un dominio de individuos, y en tales casos el uso de varios cuanticadores resulta natural.
Ejemplo 1.50 Con estos ejemplos en mente vamos a dar una descripcin ms detallada y
formal. Sean U
1
, U
2
, ..., U
n
conjuntos no vacos. Un predicado de n argumentos sobre U
1
x U
2
x ... x U
n
es una funcin P(x
1
, x
2
, ..., x
n
) con dominio de individuos U
1
x U
2
x ... x U
n
y los
valores de la funcin son proposiciones. Las variables x
1
, x
2
, ..., x
n
para P(x
1
, x
2
, ..., x
n
) son todas
variables libres para el predicado y cada x
j
vara en su correspondiente dominio de individuos U
j
. El
trmino libre es la abreviacin de libre para sustitucin, queriendo decir que la variable x
j
est
disponible en caso de que queramos sustituir un valor particular de U
j
cada vez que aparezca x
j
.
Si sustituimos x
j
por un valor, digamos que por ejemplo sustituimos x
1
por a, en P(x
1
, x
2
, ..., x
n
)
obtenemos el predicado P(a, x
2
, ..., x
n
) que es libre en las restantes n 1 variables x
2
, ..., x
n
pero
ya no lo es en x
1
. Al aplicar un cuanticador x
j
o x
j
a un predicado P(x
1
, x
2
, ..., x
n
) obtenemos
un predicado x
j
P(x
1
, x
2
, ..., x
n
) o x
j
P(x
1
, x
2
, ..., x
n
) cuyos valores dependen nicamente de las
restantes n1 variables. Decimos que el cuanticador liga la variable x
j
, haciendo que x
j
sea una
variable acotada para el predicado. Al aplicar n cuanticadores, uno para cada variable, obtenemos
que todas las variables estn acotadas y obtenemos una proposicin cuyo valor de verdad puede
determinarse aplicando las reglas para x y x, para los dominios de individuos U
1
, U
2
, ..., U
n
.
Ejemplo 1.51 Anteriormente notamos que un predicado de n argumentos se transforma en
un predicado de (n 1) argumentos cuando se liga una de las variables con un cuanticador. Su
valor de verdad depende de los valores de verdad de las restantes (n 1) variables libres y en
particular no depende de qu nombre elijamos para llamar la variable acotada. De esta manera si
P(x) es predicado de un argumento con dominio de individuos U, entonces x P(x), y P(y) y
z P(z) tienen todas el mismo valor de verdad, es decir P(n), es verdadero para toda n en U y
falso en cualquier otro caso. De manera semejante, si Q(x, y) es un predicado de dos argumentos
con dominio de individuos U y V , entonces y Q(x, y), t Q(x, t) y s Q(x, s) describen todas el
mismo predicado de un argumento, a saber, el predicado que es verdadero para una x dada en U
si y slo si Q(x, V ) es verdadero para alguna V en V que es el dominio de la segunda variable. Por
otro lado, el predicado x Q(x, x), no es el mismo que los tres ltimos. La diferencia consiste en
que el cuanticador en este caso liga las dos variables libres.
Otra prctica comn es dar una descripcin del dominio de individuos justo despus de la variable
cuanticada. Por ejemplo, en lugar de sea R el dominio de individuos ... x P(x) podramos
escribi x R P(x). De manera similar, x R n P(x
n
> x) se lee como hay un nmero real
x tal que para toda n en P, x
n
> x o como hay un nmero real x tal que x
n
> x para toda n en
P.
1.4.1. Leyes de la lgica de predicados
Las ideas de demostracin y de teorema que se discuti para el clculo proposicional, pueden
extenderse al mbito del clculo de predicados. No es sorprendente que con ms expresiones posi-
bles tengamos tambin mayores complicaciones. Una relacin moderadamente completa de este
tema puede formar una parte sustancial de otro libro. En esta seccin nos limitaremos a discutir
algunas de las ms bsicas y tiles conexiones entre los cuanticadores y los operadores lgicos.
En el captulo anterior utilizamos la expresin proposicin compuesta de manera informal para
CAPTULO 1. LGICA MATEMTICA 41
describir proposiciones construidas a partir de proposiciones ms simples.
Las leyes de la lgica de predicados que no se pueden obtener por medio de la sustitucin de
las leyes de la lgica proposicional, son por ejemplo:
1. x P(x) P(a)
x P(x) P(a) prueba que, si cada individuo de un conjunto posee una determinada
propiedad P, entonces existe tambin un individuo determinado a que posee esta propiedad.
2. P(a) x P(x)
P(a) x P(x) prueba que, si un individuo determinado de un conjunto de individuos
posee una determinada propiedad P, existe entonces, por lo menos un individuo a con esta
propiedad.
Toda expresin de la lgica proposicional con validez general puede convertirse en una expresin
de la lgica de predicados con validez general, pero el recproco es falso.
Podramos intentar obtener, por medio de la ssustitucin de una expresin de la lgica proposi-
cional satisfactible sin validez general, una expresin de la lgica de predicados igualmente satis-
factible, pero sin validez general. Pongamos por ejemplo en la neutralidad de la lgica proposicional
P Q para la variable proposicional
P

= x[P(x) P(x)]
y para
Q

= x[P(x) P(x)]
de esta forma obtenemos la expresin
x[P(x) P(x)] Q

= x[P(x) P(x)].
Esta expresin es una contradiccin.
Por el contrario resulta que: Toda expresin de la lgica proposicional, no ejecutable, satis-
factible, es tambin una expresin de la lgica de predicados, no ejecutable, satisfactible.
Algunas equivalencias de la lgica de predicados, que expresan la relacin que se establece entre
los cuanticadores y reciben especial atencin. Una equivalencia de la lgica de predicados tiene
tanta validez general como una equivalencia de la lgica proposicional, si coinciden en cada caso
los valores de verdad de ambos trminos en iguales sustituciones de sus variables.
Se obtiene una proposicin verdadera en cada sustitucin de las variables del dominio, a partir
de un conjunto no vaco dado, y en cada sustitucin de las variables del predicado P. Esta expre-
sin es una forma, en la lgica de predicados del conocido teorema del tercer excluido de la lgica
proposicional.
Las identidades de la lgica de predicados (leyes) se pueden obtener de las identidades lgicas
proposicionales si las variables son sustituidas por formas proposicionales de la lgica de predicados
en las expresiones de la lgica proposicional correspondiente.
En muchos casos nos encontramos que estas expresiones tienen que ver con formas proposi-
cionales, que se han obtenido mediante la combinacin de dos o ms proposiciones como dos formas
CAPTULO 1. LGICA MATEMTICA 42
proposicionales. La traduccin de expresiones de la lgica de predicados en el lenguaje comn es
generalmente ms fcil que la traduccin en direccin contraria. Sobre todo existen dicultades
cuando se presentan, por ejemplo, dos o ms operadores.
Teorema 1.15 Las siguientes equivalencias son vlidas:
x y P(x, y)

= y x P(x, y) y x y P(x, y)

= y x P(x, y)
Demostracin
Para demostrar que x y P(x, y)

= y x P(x, y) es una tautologa, debemos revisar que esta
proposicin es verdadera para todos los dominios del discurso posibles. Por la denicin de ,
necesitamos revisar solamente que y x P(x, y) es verdadera para un dominio dado si y slo si
x y P(x, y) es verdadera para ese dominio.
Supongamos que x y P(x, y) tiene valor verdadero. Entonces y P(x
0
, y) es verdadera para
alguna x
0
en el universo, por lo tanto P(x
0
, y
0
) es verdadera para alguna y
0
en el dominio. De ah
que x P(x, y
0
) es verdadera y por lo tanto y x P(x, y) es verdadera. La implicacin en la otra
direccin es similar.
Ms an, las dos proposiciones x y P(x, y) y y x P(x, y) son lgicamente equivalentes a la
proposicin (x, y) P(x, y) donde (x, y) vara sobre D
1
x D
2
, con D
1
y D
2
los dominios del discurso
de las variables x e y respectivamente.
Teorema 1.16 Es vlida la siguiente identidad:
x y P(x, y)

= y x P(x, y)
Demostracin
Para poder demostrar este teorema, asumimos que si la parte izquierda de esta proposicin es
verdadero entonces existe x
0
en el dominio de discurso tal que y P(x
0
, y) es verdadero y as
P(x
0
, y) es verdadero para toda y. Por lo tanto, para cada y, x P(x, y) es verdadero; de hecho
la misma x
0
sirve para cada y. Como x P(x, y) es verdadero para toda y, el lado derecho de la
proposicin tiene valor de verdad verdadero. De esta manera la proposicin es una tautologa.
Por otra parte el recproco de esta proposicin, es decir y x P(x, y)

= x y P(x, y) no es en
general verdadero. Para enfatizar la diferencia, supongamos que x e y varan sobre un dominio D
de tres elementos, digamos D = a, b, c. El predicado de 2 argumentos P(x, y) tiene nueve posibles
valores;
P(a, a); P(a, b); P(a, c); P(b, a); P(b, b); P(b, c); P(c, a); P(c, b); P(c, c).
Entonces x y P(x, y) es verdadero si y P(x
0
, y) es verdadero para alguna x
0
. Como x
0
tiene
que ser igual a a, b o c vemos que x y P(x, y) es verdadero si y slo si todas las proposiciones de
una de las las dadas arriba son verdaderas. En contraste, y x P(x, y) sera verdadera siempre
que al menos una proposicin de cada columna sea verdadera.
Por ejemplo si consideramos un predicado P(x, y) con valores de verdad
P(a, a) P(a, b) P(a, c) P(b, a) P(b, b) P(b, c) P(c, a) P(c, b) P(c, c)
V F F F F V F V V
entonces y x P(x, y) ser verdadera en tanto que x y P(x, y) ser falsa. Para esta eleccin
de predicado P(x, y), x P(x, y) es verdadera para toda y pero la x adecuada depende de la y,
ninguna x nica sirve para toda y.
Teorema 1.17 Las identidades siguientes son vlidas:
x P(x)

= x [P(x)]; x P(x)

= x [P(x)];
x P(x)

= x [P(x)]; x P(x)

= x [P(x)].
CAPTULO 1. LGICA MATEMTICA 43
Ejemplo 1.52 Las leyes de DeMorgan pueden utilizarse repetidamente para negar cualquier
proposicin cuanticada
w x y z P(w, x, y, z)
es sucesivamente lgica equivalente a
w[x y z P(w, x, y, z)]; w x[y z P(w, x, y, z)];
w x y[z P(w, x, y, z)]; w x y z[P(w, x, y, z)];
Esto ilustra la regla general: La negacin de un predicado cuanticado es lgicamente equiva-
lente a la proposicin que se obtiene al sustituir cada por y cada por y reemplazando el
mismo predicado por su negacin.
Ejemplo 1.53 La negacin de
x y z (x < z < y) es x y z [(x < z < y)].
Aplicando las leyes de DeMorgan vemos que la negacin es lgicamente equivalente a
x y z [(z x) (z y)]
Ejemplo 1.54 La negacin de
x y (x < y x
2
< y
2
) es x y [(x < z x
2
< y
2
)].
Aplicando las leyes de DeMorgan vemos que la negacin es lgicamente equivalente a
x y [(x < y) (x
2
y
2
)]
1.4.2. Interpretacin de frmulas en la lgica de predicados
En la lgica proposicional una interpretacin es una asignacin de valores de verdad a tomos.
En la lgica de predicados, puesto que hay variables involucradas, hay que hacer ms que eso. Para
denir una interpretacin para una frmula en la lgica de predicados, tenemos que especicar dos
cosas, el dominio y una asignacin a constantes, smbolos de funcin y smbolos de predicado que
ocurren en la frmula. A continuacin se da la denicin formal de interpretacin de una frmula
en la lgica de predicados.
Denicin 1.24 Interpretacin de una frmula
Una interpretacin de una frmula G en la lgica de predicados, consicte de un dominio D no
vaco, y una asignacin de valores a cada constante, smbolos de funcin, y smbolos de predicado
que ocurre en G de la siguiente manera:
1. A cada constante asignamos un elemento en D;
2. A cada smbolo de funcin asignamos una aplicacin de D
n
a D, D
n
= x
1
, x
2
, ..., x
n
D
3. A cada smbolo de predicado asignamos una aplicacin de D
n
a V, F.
Algunas veces para enfatizar el dominio D, hablaremos de una interpretacin de la frmula sobre
D. Cuando evaluamos el valor de verdad de una frmula en una interpretacin sobre el dominio
D, x ser interpretada como para todos los elementos x en D, y x como hay un elemento en
D. Para cada interpretacin de una frmula sobre un dominio de individuos D, la frmula puede
ser evaluada a V o F de acuerdo a las siguientes reglas:
CAPTULO 1. LGICA MATEMTICA 44
1. Si los valores de verdad de las frmulas H y G son evaluadas, entonces los valores de verdad
de las frmulas H, H G, H G, H G, H G son evaluadas de la siguiente manera:
H G H H G H G H G H G
V V F V V V V
V F F V F F F
F V V V F V F
F F V F F V V
2. x H es evaluada a V si el valor verdadero de H es valuado a V para cada d D, de otra
manera es evaluado a F.
3. x H es evaluado a V si el valor de verdad de H es V para por lo menos un d D, de otra
manera es evaluada a F.
Se puede notar fcilmente que cualquier frmula conteniendo variables libres no puede ser
evaluada. En adelante asumiremos, ya sea que las frmulas no contienen variables libres o que las
variables son tratadas como constantes.
Ejemplo 1.55 Considere la frmula x y P(x, y), D = 1, 2
P(1, 1)

= V ; P(1, 2)

= F; P(2, 1)

= F; P(2, 2)

= V.
Si x = 1, podemos ver que hay un y tal que P(1, y) es verdadero. Si x = 2 hay tambin un y
denominado 2 tal que P(2, y) es verdadero, por consiguiente en las interpretaciones de arriba, para
cada x en D hay un y tal que P(x, y) es verdadero, esto es x y P(x, y) es verdadero en esta
interpretacin.
Ejemplo 1.56 Considere la frmula x [P(x) Q(f(x), k)]. Hay una constante k, un smbolo
de funcin f de un lugar, un smbolo de predicado P de un lugar, y un smbolo de predicado Q de
dos lugares. La siguiente es una interpretacin I. Dominio D = 1, 2.
Asignacin para k: a = 1.
Asignacin para f: f(1) = 2; f(2) = 1.
Asignaciones para P y Q:
P(1) = F; P(2) = V ; Q(1, 1) = V ; Q(1, 2) = V ; Q(2, 1) = F; Q(2, 2) = V.
Si x = 1, entonces
P(x) Q(f(x), k) = P(1) Q(f(1), k) = P(1) Q(2, 1) = F F = V.
Si x = 2, entonces
P(x) Q(f(x), k) = P(2) Q(f(2), k) = P(2) Q(1, 1) = V V = V.
Puesto que P(x) Q(f(x), k) es verdadero para todos los valores de x en D, la frmula
x [P(x) Q(f(x), k)]
es verdadera bajo las interpretaciones I.
Ejemplo 1.57 Evaluar los valores de verdad de las siguientes frmulas bajo las interpreta-
ciones dadas en el ejemplo anterior.
CAPTULO 1. LGICA MATEMTICA 45
1. x [P(f(x)) Q(x, f(k))];
2. x [P(x) Q(x, k)];
3. x y [P(x) Q(x, y)].
Para 1): Si x = 1, entonces
P(f(x)) Q(x, f(k)) = P(f(1)) Q(1, f(1)) = P(2) Q(1, f(1)) = P(2) Q(1, 2) = V V = V.
Si x = 2, entonces
P(f(x)) Q(x, f(k)) = P(f(2)) Q(2, f(1)) = P(1) Q(2, 1) = F F = F.
Puesto que hay un elemento en el dominio D, esto es x = 1 tal que P(f(x))Q(x, f(k)) es verdadero,
el valor de verdad de la frmula x [P(f(x)) Q(x, f(k))] es verdadera bajo la interpretacin I.
Para b): Si x = 1, entonces
P(x) Q(x, k) = P(1) Q(1, 1) = F V = F.
Si x = 2, entonces
P(x) Q(x, k) = P(2) Q(2, 1) = V F = F.
Puesto que no hay elemento en el dominio D tal que P(x) Q(x, k) sea verdadero, la frmula
x [P(x) Q(x, k)]
es evaluada a falsa bajo la interpretacin I.
Para c): Si x = 1, entonces P(x) = P(1) = F.
Por consiguiente P(x) Q(x, y) = F para y = 1 e y = 2. Puesto que existe un x, que es x = 1, la
frmula y [P(x)Q(x, y)] es falsa, la frmula x y [P(x)Q(x, y)] es falsa bajo la interpretacin
I, esto es, la frmula es falsicada por I.
Denicin 1.25 Frmula consistente
Una frmula G es consistente (satisfactible) si y slo si existe una interpretacin I tal que G es
evaluada verdadero en I. Si una frmula G es verdadera en una interpretacin I, decimos que I es
un modelo de G e I satisface a G.
Denicin 1.26 Frmula vlida
Una frmula G es vlida si, y slo si cada interpretacin de G satisface a G.
Denicin 1.27 Frmula inconsistente
Una frmula G es inconsistente (insatisfactible) si y slo si, no existe una interpretacin que
satisface a G.
Las relaciones entre validez (inconsistencia) y consecuencias lgicas, como se indica en la lgica
proposicional, son tambin verdaderas para la lgica de predicados. En efecto, la lgica de predica-
dos puede ser considerada como una extensin de la lgica proposicional. Cuando una frmula en
la lgica de predicados no contiene variables y cuanticadores, puede ser tratada justo como una
frmula en la lgica proposicional.
Ejemplo 1.58
1. x P(x) y P(y) es inconsistente;
CAPTULO 1. LGICA MATEMTICA 46
2. x P(x) y P(y) es vlido;
3. P(k) x P(x) es consistente;
4. x P(x) y P(y) es vlido.
En la lgica de predicados, puesto que hay un nmero innito de elementos en el dominio D, en
general, hay un nmero innito de interpretaciones de una frmula. Por consiguiente al contrario de
la lgica proposicional, no es posible vericar la validez e inconsistencia de una frmula, evaluando
la frmula bajo todas las posibles interpretaciones.
1.4.3. Forma normal prenexa
En la lgica proposicional hemos introducido dos formas normales, la forma normal conjuntiva
y la forma normal disjuntiva. En la lgica de predicados hay una forma normal llamada forma
normal Prenexa. La razn para considerar una forma normal Prenexa de una frmula es simplicar
procedimientos de prueba.
Denicin 1.28 Forma normal prenexa
Una frmula G en la lgica de predicados se dice que es una forma normal Prenexa si y slo si, la
frmula G est en la forma
(Q
1
x
1
)(Q
2
x
2
)...(Q
n
x
n
)(M)
donde cada (Q
i
x
i
), i = 1, 2, ..., n ya sea x
i
o x
i
, y M es una frmula que no contiene cuan-
ticadores, (Q
1
x
1
)(Q
2
x
2
)...(Q
n
x
n
) es llamada el prejo y M es llamada la matriz de la frmula
G.
Dada una frmula G, consideraremos un mtodo de transformarla en una forma normal Prenexa.
Esto se logra primero considerando algunos pasos bsicos de frmulas equivalentes en la lgica de
predicados. Recordemos que dos frmulas G y H son equivalentes si, y slo si los valores de verdad
de G y H son los mismos bajo cada interpretacin.
Los pares bsicos de frmulas equivalentes dadas en la lgica proposicional son todava verdad
para la lgica de predicados, adicionalmente hay otros pares de frmulas equivalentes conteniendo
cuanticadores, y que se estudiaron en secciones anteriores. Consideraremos estos pares adicionales
de frmulas equivalentes.
Sea G una frmula que contiene una variable libre x, para enfatizar que la variable libre est en
G, representamos G por G[x]. Sea H una frmula que no contiene variable x, tenemos los siguientes
pares de frmulas equivalentes, donde Q es ya sea o :
1. (Qx)G[x] H

= (Qx)(G[x] H);
2. (Qx)G[x] H

= (Qx)(G[x] H);
3. (xG[x])

= x(G[x]);
4. (xG[x])

= x(G[x]).
Las leyes 1 y 2 son obviamente verdaderas puesto que H no contiene x, por consiguiente
puede ser introducida en el alcance del cuanticador Q. Las leyes 3 y 4 no son difciles de
probar. Sea I cualquier interpretacin arbitraria sobre el dominio D.
CAPTULO 1. LGICA MATEMTICA 47
Si (x G[x]) es verdadera en I, entonces x G[x] es falsa en I. Esto signica que hay un
elemento d en D tal que G[d] es falso. Esto es G[d] es verdadero en I. Por consiguiente,
x (G[x]) es verdadera en I. Por otra parte si (x G[x]) es falsa en I, entonces x G[x] es
verdadera en I. Esto signica que G[x] es verdadera para cada elemento x en D, esto es G[x]
es falso para cada elemento x en D, por consiguiente, x (G[x]) es falsa en I. Puesto que
(x G[x]) y x (G[x]) siempre asume el mismo valor de verdad para cada interpretacin
arbitraria, por denicin, (x G[x])

= x (G[x]). As la ley 3 es probada e igualmente
podemos probar la ley 4.
Supongamos que F[x] y G[x] son dos frmulas que contienen x,
5. x F[x] x G[x]

= x (F[x] G[x])
6. x F[x] x G[x]

= x (F[x] G[x])
Esto es, el cuanticador universal y el existencial , pueden distribuirse sobre y ,
respectivamente. Sin embargo el cuanticador universal y existencial no pueden distribuirse
sobre y respectivamente. Esto es
x F[x] x G[x] ,= x (F[x] G[x])
x F[x] x G[x] ,= x (F[x] G[x])
Para casos como estos tenemos que hacer algo especial. Puesto que cada variable ligada en
una frmula puede ser considerada como una variable renombrable, cada variable x puede
ser renombrada z, y la frmula x G[x] se transforma en z G[z].
Supongamos que escogemos la variable z que no aparece en F[x]. Entonces
x F[x] x G[x]

= x F[x] z G[z]

= xz (F[x] G[z])
Similarmente, renombrando todas las x que ocurren en x G[x] como z, podemos tener
x F[x] x G[x]

= x F[x] z G[z]

= xz (F[x] G[z])
Por consiguiente, para estos dos casos podemos todava pasar todos los cuanticadores a la
izquierda de la frmula. En general, tenemos
7. (Q
1
x)F[x] (Q
2
x)G[x]

= (Q
1
x)(Q
2
x)(F[x] G[z])
8. (Q
3
x)F[x] (Q
4
x)G[x]

= (Q
3
x)(Q
4
x)(F[x] G[z])
donde Q
1
, Q
2
, Q
3
y Q
4
son ya sea o , y z no aparece en F[x].
Naturalmente si Q
1
= Q
2
= y Q
3
= Q
4
= , entonces no tenemos que renombrar las x en
(Q
2
x)G[x] o (Q
4
x)G[x]. Podemos usar las leyes 5 y 6 directamente. Usando las leyes de la lgica
proposicional y las leyes 1 - 8, podemos siempre transformar una frmula dada en forma normal
Prenexa.
La siguiente es una gua del procedimiento de transformacin:
PASO 1: Use las leyes
1. F G

= (F G) (G F);
CAPTULO 1. LGICA MATEMTICA 48
2. F G

= F G;
Para eliminar las conectividades lgicas y .
PASO 2: Repetidamente use las leyes
3. (F)

= F;
4. (F G)

= F G;
5. (G G)

= F G;
6. (x F[x])

= x (F[x]);
7. (x F[x])

= x (F[x]);
para traer los signos de negacin inmediatamente antes de los tomos.
PASO 3: Renombrar las variables ligadas si es necesario.
PASO 4: Use las leyes
8. (Qx)F[x] G

= (Qx)(F[x] G);
9. (Qx)F[x] G

= (Qx)(F[x] G);
10. x F[x] x G[x]

= x (F[x] G[x]);
11. x F[x] x G[x]

= x (F[x] G[x]);
12. (Q
1
x)F[x] (Q
2
x)G[x]

= (Q
1
x)(Q
2
x)(F[x] G[z]);
13. (Q
3
x)F[x] (Q
4
x)G[x]

= (Q
3
x)(Q
4
x)(F[x] G[z]).
para mover los cuanticadores a la izquierda de la frmula y obtener una forma normal
Prenexa.
Ejemplo 1.59 Transformar la frmula x P(x) x Q(x) en forma normal prenexa.
Solucin
x P(x) x Q(x)

= x P(x) x Q(x)

= xP(x) x Q(x)

= x [P(x) Q(x)].
Ejemplo 1.60 Transformar la frmula x y z [P(x, z) P(y, z)] u Q(x, y, u) en
forma normal Prenexa.
Solucin
x yz [P(x, z) P(y, z)] u Q(x, y, u)

= x yz [P(x, z) P(y, z)] u Q(x, y, u)

= x yz [P(x, z) P(y, z)] u Q(x, y, u)

= x y z u P(x, z) P(y, z) u Q(x, y, u).


CAPTULO 1. LGICA MATEMTICA 49
1.4.4. Tarea
1. Sea A = 1, 2, 3, 4 el conjunto universal. Determine el valor de verdad de cada enunciado:
a) x : x + 3 < 6; b) x : x
2
10 8; c) x : x
2
> 1 x + 2 = 0;
d) x : 2x
2
+x = 15.
Resp: a) Falso; b) Verdadero; c) Verdadero; d) Falso.
2. Determine el valor de verdad de las siguientes proposiciones, siendo N el universo:
a) x y (2y = x); b) y x (2x = y); c) x y (2x = y);
d) y x (2y = x); e) x y [(2y = x)].
3. Determine el valor de verdad de las siguientes proposiciones, siendo R el universo:
a) x y (xy = 1); b) x y [(x + y)
2
= x
2
+ y
2
]; c) x y (x
2
+ y
2
+ 1 = 2xy);
d) x y [(x + 2y = 4) (2x y = 2)].
4. Determine el valor de verdad de las siguientes proposiciones, siendo R el universo:
a) x R x
2
x; b) x R 2x = x; c) x R
2x1
4x2
=
1
2
;
d) x R x
2
+ 2x + 1 0.
5. Negar los siguientes enunciados:
a) y p(y) x(q(x)); b) x(p(x)) x q(x); c) x y (p(x, y) q(x, y)).
Resp: a) y p(y) x q(x); b) x p(x) x(q(x)); c) y(p(x, y) q(x, y)).
6. Negar las siguientes armaciones:
a) x y [(x +y es impar) (x es impar y es impar)];
b) x y (x +y = 5 y = x); c) x y (x < y x
2
y);
d) x y z (x < y x +z = y).
7. Averiguar el valor de verdad siendo U = R:
a) x R (x < 0 x < 3); b) x R (x
2
0 x
4
= x
3
);
c) x R, y R (x
2
+y
2
= 1); d) x R, y R (y < x 2y < 10).
Resp: a) Verdadero; b) Verdadero; c) Falso; d) Falso.
8. Considere el universo U de todos los profesores de ciencias bsicas. Sea P(x) el predicado
a x le gusta la lgica matemtica:
a) Exprese la proposicin no a todos los profesores de ciencias bsicas les gusta la lgica
matemtica, utilizando smbolos de la lgica de predicados;
b) Haga lo mismo para a todos los profesores de ciencias bsicas no les gusta la lgica
matemtica;
c) Escriba el signcado de x P(x)

= x [P(x)] para U y P(x);


d) Haga lo mismo con x P(x)

= x [P(x)].
9. Escriba la negacin de las siguientes frmulas:
a) x P(x, x) [y z P(y, z) x P(x, x)];
b) x y x P(f(x, y), y) [x P(f(x, y), y) z [f(z, x) = y];
c) x [P(x) Q(x)] [x P(x) x Q(x)];
d) x y P(x, y) y P(f(x, y), y);
e) x y P(x, y) y P(y, y);
f ) x [y x P(x, y) Q(x)] [y x P(x, y) Q(f(y, y))].
10. Considere la siguiente interpretacin: D = 1, 2.
Asignaciones a las constantes k y t: k = 1 y t = 2.
Asignaciones para la funcin f: f(1) = 2 y f(2) = 1.
CAPTULO 1. LGICA MATEMTICA 50
Asignaciones para el predicado P: P(1, 1) = V ; P(1, 2) = V ; P(2, 1) = F; P(2, 2) = F.
Evale el valor de verdad de las siguientes frmulas en cada interpretacin:
a) P(k, f(k)) P(t, f(t)); b) x y P(y, x);
c) x y [P(x, y) P(f(x), f(y))].
11. Dadas las siguientes frmulas, hallar la correspondiente forma normal prenexa:
a) x y [P(x, y) P(y, x)]; b) x y [P(x) P(y)] x = y;
c) x y (x = y) [x P(x) x P(x)].
12. Escriba la negacin de las siguientes frmulas:
a) x [P(x) Q(x)] [x P(x) x Q(x)];
b) x y P(x, y) y P(f(x, y), y);
c) x y P(x, y) y P(y, y);
d) x [y x P(x, y) Q(x)] [y x P(x, y) Q(f(y, y))];
e) x [P(x) Q(x)] [P(x) Q(x)];
f ) x y P(f(y, x), x) y P(f(y, f(z, x)), f(z, x));
g) x P(x, x) [y z P(y, z) x P(x, x)];
h) x y x P(f(x, y), y) [x P(f(x, y), y) z [f(z, x) = y].
Captulo 2
Teora de conjuntos
2.1. Conjuntos
Casi todos los objetos matemticos son ante todo conjuntos, independientemente de otra
propiedad adicional que posean. Por consiguiente, la teora de los conjuntos es, en cierto sentido,
la base sobre la cual se construye toda la matemtica. A pesar de esto, la teora de los conjuntos,
se aprende, y se usa fcilmente.
Denicin 2.1 Conjunto
Un conjunto es cualquier coleccin bien denida de objetos llamados elementos o miembros del
conjunto.
Se usan letras maysculas como A, B, C, ..., para indicar conjuntos y letras minsculas como
a, b, c, ..., para indicar miembros o elementos de los conjuntos.
Ejemplo 2.1 Son ejemplos de conjuntos, los siguientes:
a. Las letras de alfabeto.
b. Los nmeros pares.
c. Los miembros de un equipo de ftbol.
A continuacin se enuncian las siguientes condiciones para denir un conjunto:
1. Los elementos que forman el conjunto han de ser entes bien denidos.
2. Para cada uno de estos elementos no hay otra alternativa que la de pertenecer o no al conjunto.
3. Para cada par de elementos a considerar no hay otra alternativa que la de estar formado o no
por elementos distintos.
2.1.1. Formas de expresar un conjunto
Hay dos caminos para denir o determinar un conjunto, mtodos que los lgicos designan por
extensin y por comprensin.
Por extensin
Para expresar que el conjunto S consta de los elementos a, b, c, escribiremos S = a, b, c, con
ello damos la extensin del conjunto S al enunciar cada uno de los elementos que lo componen. Es
decir, se declara individualmente todos los elementos del conjunto.
51
CAPTULO 2. TEORA DE CONJUNTOS 52
Por comprensin
Por otra parte, los conjuntos innitos slo pueden denirse por comprensin, es decir, dando
un criterio que permita reconocer para cada ente arbitrario, si pertenece o no al conjunto. Es decir,
se declara una propiedad que caracteriza a todos los elementos del conjunto.
2.2. Conjuntos nitos e innitos
2.2.1. Conjunto nito
Denicin 2.2 Conjunto nito
Aquel conjunto que consta de cierto nmero de elementos distintos cuyo proceso de conteo tiene
lmite, se denomina conjunto nito.
Ejemplo 2.2 Sea
A = x/x = provincias de Ecuador
Que se lee A es el conjunto de las x, tales que x son las provincias de Ecuador. A es un conjunto
nito porque si es posible contar todas las provincias de Ecuador.
2.2.2. Conjunto innito
Denicin 2.3 Conjunto innito
Aquel conjunto que consta de un nmero indeterminado de elementos distintos, se denomina con-
junto innito.
Ejemplo 2.3 Sea
A = z/z = arena en el mar
Que se lee A es el conjunto de las z, tales que z son los granos de arena en el mar. A es un
conjunto innito porque no se puede contar el nmero de granos de arena, es innito.
2.2.3. Nocin de pertenencia
Se indica el hecho de que x es un elemento del conjunto A escribiendo x A y se indica el
hecho de que x no es un elemento del conjunto A escribiendo x / A.
Ejemplo 2.4 Sea A = 1, 3, 5, 7. Entonces 1 S, 3 S pero 2 / A.
Ejemplo 2.5 Si S = x/x es un nmero natural menor que 4, es el conjunto 1, 2, 3 de-
scrito anteriormente, enlistando sus elementos.
Ejemplo 2.6 Sea S = x/x es un nmero real y x
2
= 1, dado que el cuadrado de un
nmero real x es siempre positivo.
2.2.4. Igualdad de conjuntos
Denicin 2.4 Igualdad de conjuntos
Los conjuntos son totalmente determinados cuando se conocen todos sus miembros. As pues, se
dice que dos conjuntos A y B son iguales si tienen los mismos elementos y se escribe A = B.
Ejemplo 2.7 Si A = 1, 2, 3 y B = x/x es un nmero natural y x
2
< 16, entonces A = B.
CAPTULO 2. TEORA DE CONJUNTOS 53
2.2.5. Conjuntos vaco
Denicin 2.5 Conjunto vaco
Cuando la condicin impuesta es contradictoria, no existe ningn elemento que la cumpla, se dice
que dene un conjunto vaco, que suele simbolizarse por .
Ejemplo 2.8 Son vacos los conjuntos siguientes: tringulos equilteros rectngulos; nmeros
primos pares mayores que 2.
2.2.6. Conjunto unitario
Denicin 2.6 Conjunto unitario
Un conjunto que tiene un nico elemento, se denomina conjunto unitario.
Ejemplo 2.9 Sea
A = Los meses del ao, cuyo nombre empieza con F
2.2.7. Conjunto universal
Denicin 2.7 Conjunto universal
El conjunto que contiene a todos los elementos de otros conjuntos, se denomina conjunto universal
o referencial. Se denota con la letra U.
Ejemplo 2.10 Sea
A = Todos los nmeros
Este es un conjunto universal, porque contiene todos los nmeros de los conjuntos R, N, Z, C, ....
2.2.8. Subconjunto
Denicin 2.8 Subconjunto
Si todos los elementos de A son tambin elementos de B, esto es si cuando x A, entonces x B,
decimos que A es un subconjunto de B o que A est contenido en B y se escribe A B. Si A no
es un subconjunto de B, se escribe A B.
Los conjuntos A y B son iguales si y solamente si B est incluido en A y A est incluido en B.
El conjunto vaco se considera subconjunto de todo conjunto.
Si A no es subconjunto de B, entonces hay por lo menos un elemento de A que no pertenece a
B.
CAPTULO 2. TEORA DE CONJUNTOS 54
Subconjunto propio e impropio
Denicin 2.9 Subconjunto propio e impropio
Si A U y A ,= , A ,= U, el conjunto A se denomina subconjunto propio del conjunto U. Los
subconjuntos y U del conjunto U reciben el nombre de impropios.
Es decir, dado A B, entonces el subconjunto A es subconjunto propio del conjunto B, si por
lo menos un elemento del conjunto B no es elemento del conjunto A. Pero si todos los elementos
de A son iguales a los elementos de B, ya no es un subconjunto, en este caso los conjuntos son
iguales.
Ejemplo 2.11 Se tiene que Z
+
Z. Adems, si Q indica el conjunto de todos los nmeros
racionales, entonces Z Q.
Ejemplo 2.12 Determine si la proposicin 2 A = 2, 2, 5 es verdadera o falsa.
Solucin
Es falsa pues 2 A como elemento, pero no como subconjunto.
Ejemplo 2.13 Sea A = 1, 2, 3, 4, 5, 6, B = 2, 4, 5, C = 1, 2, 3, 4, 5. Entonces B A,
B C, C A. Sin embargo A B, A B, C B.
Denicin 2.10 Subconjunto de s mismo
Si A es cualquier conjunto, entonces A A. Esto es, cualquier conjunto es subconjunto de s
mismo.
Sea A un conjunto y sea S = A, A, por tanto, puesto que A y A son elementos de S,
tenemos que A S y A S. De esto se sigue que A S y A S. Sin embargo, no es
verdad que A S.
Dado que una implicacin es verdadera si la hiptesis es falsa, se sigue que A.
Ejemplo 2.14 Dados los conjuntos A, B, C, demuestre las siguientes expresiones:
1. Si A B, B C, entonces A C;
2. Si A B, B C, entonces A C;
3. Si A B, B C, entonces A C;
4. Si A B, B C, entonces A C.
Solucin
Procederemos a demostrar cada uno de los literales de forma minuciosa:
1. Sea x A. Como A B, x B. Entonces con B C, x C. De ah que x A entonces
x C y A C.
2. Sea x A. A B entonces x B. B C entonces x C. De ah que A C. Como A B,
existe y B, donde y / A. Con B C, y C. En consecuencia, A C e y C, con y / A, de
modo que A C.
3. Si x A, entonces A B entonces x B y B C entonces x C. De ah que A C. Como
B C, existe y C con y / B. Adems, A B e y / B entonces y / A. En consecuencia, A C
e y C con y / A entonces A C.
4. Como A B, resulta que A B. Entonces, el resultado se obtiene de 3).
Ejemplo 2.15 Para cualquier conjunto A, A; A si A ,= .
Solucin
Si el primer resultado no es verdadero, entonces A, de modo que hay un elemento x del
universo con x , pero x / A. Pero x es imposible. Adems, si A ,= , entonces hay un
elemento a A y a / , de modo que A.
CAPTULO 2. TEORA DE CONJUNTOS 55
2.2.9. Conjunto de partes
Denicin 2.11 Conjunto de partes
Todo conjunto integrado por la totalidad de subconjuntos que se puede formar a partir de un con-
junto dado, se denomina conjunto de partes y se denota P(A).
Ejemplo 2.16 Indique todos los subconjuntos del conjunto de tres elementos a, b, c.
Solucin
El conjunto de tres elementos tiene los subconjuntos impropios y a, b, c y los subconjuntos
propios: a, b, c, a, b, a, c, b, c.
2.2.10. Conjunto potencia
Denicin 2.12 Conjunto potencia
Si A es un conjunto, entonces al conjunto de todos los subconjuntos de A se le llama conjunto
potencia de A. Tienen la misma connotacin del connunto de conjuntos.
Es decir, el conjunto potencia es el nmero de subconjuntos que se puede formar con elementos
del conjunto, incluyendo el vaco. Se calcula con P
A
= 2
n
, donde n es el nmero de elementos del
conjunto A o cardinalidad del conjunto A.
Ejemplo 2.17 Indique el nmero de subconjuntos o conjunto potencia del conjunto a, b, c, d.
Solucin
Aqu n = 4, por consiguiente
P
A
= 2
4
= 16.
2.3. Operaciones con conjuntos
2.3.1. Unin de conjuntos
Mientras que en aritmtica se realiza operaciones de suma, resta y multiplicacin, en el ca-
so de conjuntos se realiza operaciones de unin, interseccin y diferencia de conjuntos, con un
comportamiento similar al de la aritmtica.
Denicin 2.13 Unin de conjuntos
Si A y B son conjuntos, se dene su unin como el conjunto que tiene todos los elementos que
pertenecen a A o a B y se indica como
A B = x/x A x B.
Es decir, la unin de dos conjuntos A y B es el conjunto formado por todos los elementos que
pertenecen al conjunto A, al conjunto B o a ambos conjuntos. En la unin de conjuntos no se
repiten los elementos que pertenecen a ambos conjuntos.
Obsrvese que x A B si x A o x B o x pertenece a ambos conjuntos.
Ejemplo 2.18 Sean A = a, b, c, e, f y B = b, d, r, s. Puesto que AB consta de todos los
elementos que pertenecen tanto a A como a B, entonces A B = a, b, c, d, e, f, r, s.
Se puede ilustrar la unin de dos conjuntos con un diagrama de Venn como sigue. Si A y B son
los conjuntos dados en la gura, entonces A B es el conjunto de puntos en la regin sombreada.
CAPTULO 2. TEORA DE CONJUNTOS 56
2.3.2. Propiedades de la unin de conjuntos
Las operaciones con conjuntos que se acaban de denir satisfacen muchas propiedades alge-
braicas; algunas de stas se parecen a las propiedades algebraicas que se satisfacen en el sistema
de los nmeros reales.
A continuacin, damos las propiedades ms importantes sobre las operaciones de conjuntos:
1. Propiedad conmutativa: Es decir, el orden de los conjuntos no altera la unin.
A B = B A.
2. Propiedad asociativa: Si son ms de dos conjuntos los que se unen, pueden asociarse de
manera libre.
A (B C) = (A B) C.
3. Propiedad de idempotencia:
A A = A.
4. Propiedad del conjunto universal:
A U = U.
5. Propiedad del conjunto vaco:
A = A.
2.3.3. Interseccin de conjuntos
Denicin 2.14 Interseccin de conjuntos
Si A y B son conjuntos, su interseccin se dene como el conjunto que contiene a todos los
elementos que pertenecen tanto a A como a B y se indica como
A B = x/x A x B.
Es decir, la interseccin de dos conjuntos A y B es el conjunto de elementos comunes a A y B. Es
posible ilustrar la interseccin de dos conjuntos por el diagrama de Venn como sigue. Si A y B son
los conjuntos dados en la gura, entonces A B es el conjunto de puntos en la regin sombreada.
CAPTULO 2. TEORA DE CONJUNTOS 57
Ejemplo 2.19 Sean A = a, b, c, e, f, B = b, e, f, r, s y C = a, t, u, v. Los elementos b,
e y f, son los nicos que pertenecen a A y B por lo cual A B = b, e, f. De igual manera,
A C = a. No existen elementos que pertenezcan tanto a A como a B, por lo cual B C = .
2.3.4. Propiedades de la interseccin conjuntos
A continuacin, damos las propiedades ms importantes sobre interseccin de conjuntos:
1. Propiedad conmutativa: Es decir, el orden de los conjuntos no altera la interseccin.
A B = B A.
2. Propiedad asociativa: Es posible cambiar el orden de asociacin y no se altera el resultado.
A (B C) = (A B) C.
3. Propiedad distributiva:
A (B C) = (A B) (A C);
A (B C) = (A B) (A C).
4. Propiedad de idempotencia:
A A = A.
5. Propiedad del conjunto universal:
A U = A.
6. Propiedad del conjunto vaco:
A = .
Ejemplo 2.20 Pruebe o refute las siguientes relaciones para los conjuntos A, B U:
a) P(A B) = P(A) P(B); b) P(A B) = P(A) P(B).
Solucin
Sea U = 1, 2, 3, A = 1, B = 2, entonces:
a) 1, 2 P(A B), pero 1, 2 / P(A) P(B).
b) C P(AB) C AB C AC B C P(A)C P(B) C P(A)P(B),
de modo que P(A B) = P(A) P(B).
Ejemplo 2.21 Demuestre que
A (B C) = (A B) (A C).
Solucin
Daremos una demostracin de que los conjuntos son subconjuntos uno del otro. Consideremos
primero x A (B C). Entonces x est en A necesariamente.
Tambin x est en B C. As que, o bien x B, en cuyo caso x A B, o x C, en tal caso
x A C. En cualquiera de los dos casos tenemos que x (A B) (A C). Esto muestra que
A(B C) (AB) (AC). Consideremos ahora y (AB) (AC). Entonces y AB
o y A C y consideramos los dos casos por separado. Si y A B, entonces y A y y B,
luego y B C y por lo tanto y A (B C). Anlogamente si y A C entonces y A y
y C, por lo tanto y B C y por eso y A (B C). As, en ambos casos, y A (B C) y
hemos demostrado que (A B) (A C) A (B C). Acabamos de demostrar la contencin
contraria, por lo que los dos conjuntos son iguales.
CAPTULO 2. TEORA DE CONJUNTOS 58
Ejemplo 2.22 Prubense o reftense las siguientes relaciones:
1. Para conjuntos A, B, C U, A C = B C A = B.
2. Para conjuntos A, B, C U, A C = B C A = B.
3. Para conjuntos A, B, C U, A C = B C, A C = B C A = B.
Solucin
1. Sea U = 1, 2, 3, A = 1, B = 2, C = 3. Entonces AC = BC = , pero A ,= B.
2. Para U = 1, 2, A = 1, B = 2, C = U, A C = B C, pero A ,= B.
3. x A x A C x B C. Si x B, entonces A B. Si x C, entonces
x AC = BC y x B. En ambos casos, A B. As mismo, y B y BC = AC,
de modo que y A o y C. Si y C, entonces y BC = AC. En cualquier caso, y A
y B A. De ah que A = B.
Denicin 2.15 Conjuntos disjuntos
A dos conjuntos que no tienen elementos comunes, se les llama conjuntos disjuntos.
La siguiente gura ilustra un diagrama de Venn con dos conjuntos disjuntos.
Las operaciones unin e interseccin pueden generalizarse para tres o ms conjuntos. As pues,
A B C = x/x A x B x C
A B C = x/x A x B x C
La regin sombreada en la segunda gura es la unin de los conjuntos A, B y C, la regin
sombreada en la tercera gura es la interseccin de los conjuntos A, B y C.
En general, si A
1
, A
2
, ..., A
n
son subconjuntos de U, entonces A
1
A
2
... A
n
se indica como

n
i=1
A
i
, y A
1
A
2
... A
n
se indica

n
i=1
A
i
.
Sea A = 1, 2, 3, 4, 5, 7, B = 1, 3, 8, 9, C = 1, 3, 6, 8. Entonces AB C es el conjunto de
elementos que pertenecen a A, B y C. Por tanto A B C = 1, 3.
2.3.5. Diferencia de conjuntos
Denicin 2.16 Diferencia de conjuntos
Si A y B son conjuntos, se dene la diferencia del conjunto A menos el conjunto B, el conjunto
formado por elementos del conjunto A que no son elementos del conjunto B y se indica
AB = x/x A x / B.
CAPTULO 2. TEORA DE CONJUNTOS 59
La diferencia tambin se denota AB.
Ejemplo 2.23 Sea A = a, b, c y B = b, c, d, e. Entonces AB = a y B A = d, e.
Si A y B son los conjuntos en la gura, entonces A B y B A son los conjuntos de puntos
en las regiones sombreadas.
Ejemplo 2.24 Para conjuntos cualesquiera A, B y C se cumple que
(A B) C = (AC) (B C).
Solucin
Se tiene que demostrar la igualdad de dos conjuntos A = B si y slo si se cumple que A B y
B A.
1) Veamos que (A B) C (AC) (B C) se cumple.
Sea x un elemento cualquiera de (A B) C, es decir x (A B) y x / C (x A x B) y
x / C. Deben analizarse por separados dos casos segn la inferencia a partir de una alternativa.
Caso 1: Tenemos que x A y x / C. Entonces x A C, de lo que resulta a su vez que
x (AC) (B C).
Caso 2: Tenemos que x B y x / C. Entonces x B C, de lo que resulta a su vez que
x (AC) (BC). De x (AB) C x (AC) (BC), resulta, segn la inferencia
de para todo, la tesis 1).
2) Veamos tambin que (AC) (B C) (A B) C se cumple.
Sea x un elemento cualquiera de (A C) (B C), es decir, x (A C) x (B C). Aqu
tambin tienen que analizarse dos casos segn la inferencia a partir de una alternativa.
Caso 1: Tenemos que x (A C). Entonces x A y x / C, de lo cual resulta a su vez que
x (A B) y x C, es decir, x (A B) C.
Caso 2: Tenemos que x (B C). Entonces x B y x / C, de lo cual resulta, de la misma
forma, que x (A B) y x / C, es decir, x (A B) C.
De x (AC) (BC) x (AB) C. Resulta nalmente la tesis 2). De 1) y 2) resulta,
que el la identidad es verdadera.
2.3.6. Complemento de un conjunto
Denicin 2.17 Complemento
Si U es un conjunto universal y contiene a A, entonces a U A se le llama complemento de A y
se indica
A = x U/x / A.
CAPTULO 2. TEORA DE CONJUNTOS 60
Ejemplo 2.25 Sea A = x/x es un nmero entero y x 4. Entonces A = x/x es un nmero entero y x <
4.
Si A es el conjunto en la gura, su complemento es la regin sombreada.
Ejemplo 2.26 Demostrar que (A B) A B.
Solucin
Utilizando las reglas del lgebra de conjuntos, obtenemos lo siguiente:
(A B) A = A (A B) = (A A) (A B)
= (A A) (A B) = (A B)
= (A B) = A B.
Ahora es claro que A B B, ya que x A B implica que x est en B.
2.3.7. Propiedades del complemento de un conjunto
A continuacin, damos las propiedades ms importantes sobre el complemento de un conjunto:
1. (A) = A;
2. A A = U;
3. A A = ;
4. = U;
5. U = ;
6. A B = A B;
7. A B = A B.
Ejemplo 2.27 Las siguientes proposiciones son equivalentes para los conjuntos A, B U:
a) A B; b) A B = B; c) A B = A; d) B A.
Solucin
Para probar este problema, se demuestra que a) b), b) c), c) d) y d) a).
a) ): Si A, B son conjuntos cualesquiera, entonces B A B. Para la inclusin opuesta, si
x A B, entonces x A o x B, pero como A B, en ambos casos se tiene x B; de modo
que A B B y resulta la igualdad.
b) c): Dados los conjuntos A, B, siempre se tiene AB A. Para la inclusin opuesta, sea
y A. Si se tiene A B = B, y A y A B y B y A B, entonces
A A B y se concluye que A = A B.
c) d): z B z / B z / A B, pues A B B. Con A B = A,
z / A B z / A x A, de modo que B A.
d) a): Por ltimo, w A w / A y como B A, w / A w / B. Entonces
w / B w B y A B.
Ejemplo 2.28 Demuestre lo siguiente:
a) (A B) A B; b) A B A B.
Solucin
a) Para demostrar que (A B A B, consideramos un elemento x en (A B. Entonces
x / A B. En particular, x / A, por lo que tenemos que x A. Anlogamente, x / B y por lo
CAPTULO 2. TEORA DE CONJUNTOS 61
tanto x B. De aqu tenemos que x AB. Hemos demostrado que (A B implica que x AB
y por lo tanto (A B A B.
b) Para demostrar la inclusin contraria A B (A B, consideramos x en A B. Entonces
x A y por lo tanto x / A. Tambin x B y por lo tanto x / B. Dado que x / A y x / B,
concluimos que x / A B, es decir, x (A B). En consecuencia A B (A B.
Ejemplo 2.29 Simplique la expresin (A B) C B.
Solucin
(A B) C B = (A B) C B
= ((A B) C) B
= (A B) (C B)
= (A B) (B C)
= [(A B) B] C
= B C.
2.3.8. Diferencia simtrica
Denicin 2.18 Diferencia simtrica
Si A y B son dos conjuntos, se dene su diferencia simtrica como el conjunto de todos los ele-
mentos que pertenezcan a A o B, pero no a ambos y se indica como
A B = x U / (x A x / B) (x B x / A).
Es decir, Para dos conjuntos A y B, la diferencia simtrica es lo que queda de ambos conjuntos
despus de eliminar los elementos de su interseccin.
Ejemplo 2.30 Sea A = a, b, c, d y B = a, c, e, f, g. Entonces A B = b, d, e, f, g.
Si A y B son como se indica en la gura, su diferencia simtrica es la regin sombreada. Es
fcil ver que
A B = (AB) (B A).
Ejemplo 2.31 Si A, B U, si, y slo si, A B = A B, A y B son disjuntos.
Solucin
Se comienza con A, B disjuntos. Si x A B, entonces x A o x B (o ambos). Pero como A
y B son disjuntos, x / A B, de modo que x A B. Por tanto, como x A B implica que
x A B, resulta A B A B.
Para la inclusin opuesta, si y A B, entonces y A o y B. (Aunque y / A B, que no se
utiliza aqu.) De modo que y A B. Por tanto, A B A B y resulta que A B = A B.
A la inversa, si A B = A B y A B = , sea x A B. Entonces, x A y x B, de modo
que x A B. Sin embargo, x / A B, lo cual contradice la igualdad de conjuntos dada. En
consecuencia, A y B son disjuntos.
Ejemplo 2.32 Por la observacin hecha en el ejemplo anterior, resulta
A B = x/x A B, x / A B = (A B) (A B) = (A B) A B
CAPTULO 2. TEORA DE CONJUNTOS 62
de modo que
A B = (A B) (A B)
= (A B) (A B)
= (A B) (A B)
= (A B) (A B)
= (A B) (A B)
=
_
(A B) A

_
(A B) B

=
_
(A A) (B A)

_
(A B) (B B)

=
_
U (B A)

_
(A B) U

= (B A) (A B)
= (A B) (A B)
= (A B) (A B)
= A B
= (A B) (A B)
= (A B) (A B)
= A B.
2.3.9. Cardinalidad
Supngase ahora que A y B son subconjuntos nitos del conjunto universal U. Se usa fre-
cuentemente la frmula [AB[, para la cardinalidad de la unin. Si A y B son disjuntos, esto es, si
AB = , entonces cada elemento de AB aparece en A o en B pero no en ambos; por lo tanto,
[A B[ = [A[ +[B[. Si A y B se sobreponen como lo muestra la gura, entonces A B pertenece
a ambos conjuntos y la suma [A[ + [B[ incluye el nmero de elementos en A B dos veces. Para
corregir esta duplicacin, se restar [A B[. Por consiguiente, se tiene el principio de adicin: Si
A y B son conjuntos nitos, entonces
[A B[ = [A[ +[B[ [A B[.
Esta situacin para tres conjuntos es ms complicada. Se expondr el principio de adicin para
tres conjuntos: Sean A, B y C conjuntos nitos. Entonces
[A B C[ = [A[ +[B[ +[C[ [A B[ [B C[ [A C[ +[A B C[.
Ejemplo 2.33 Una compaa de computacin necesita contratar a 25 programadores para tar-
eas de programacin de sistemas y a 40 para programacin de aplicacin. De estos empleados, se
espera que 10 realicen tareas de dos tipos. Cuntos programadores debern contratar?
Solucin
A es el nmero de programadores de sistema empleados y B el nmero de programadores de apli-
caciones. Tenemos [A[ = 25, [B[ = 40 y [A B[ = 10. As pues, el nmero que debemos emplear
es
[A B[ = [A[ +[B[ [A B[ = 25 + 40 10 = 25.
Ejemplo 2.34 Se hace una encuesta de los medios de transporte urbano ms comunes. A cada
persona se le pregunta si el bus urbano, el trolebs o el automvil es el medio ms usado para ir al
trabajo. Se permite ms de una respuesta. El resultado de la encuesta se da a continuacin:
CAPTULO 2. TEORA DE CONJUNTOS 63
a) 30 personas escogieron el bus urbano;
b) 35 personas escogieron el trolebs;
c) 100 personas escogieron el automvil;
d) 15 personas escogieron el bus urbano y el trolebs;
e) 15 personas escogieron el bus urbano y el automvil;
f ) 20 personas escogieron el bus trolebs y el automvil;
g) 5 personas escogieron los tres medios de transporte.
Cuntas personas respondieron a la encuesta?
Solucin
Sean A, B y C los conjuntos de las personas que escogieron bus urbano, trolebs y automvil
respectivamente. Se sabe que [A[ = 30, [B[ = 35, [C[ = 100, [A B[ = 15, [A C[ = 15,
[B C[ = 20, entonces
[A B C[ = (30 + 35 + 100) (15 + 15 + 20) + 5 = 120.
Ejemplo 2.35 Una compaa de fertilizantes anuncia su fertilizante CC en la revista Gua.
La rma entrevist a 100 compradores de fertilizantes en forma casual. Estas entrevistas revelaron
que 25 personas usan CC, 20 personas leen Gua y 5 personas leen Gua y compran CC:
a) Cuntas personas no leen Gua?
b) Cuntas personas o leen Gua o compran CC?
c) Qu porcentaje de las personas que leen Gua compran CC?
d) Puede la compaa concluir que anunciar en Gua le incrementar las ventas?
Solucin
Antes de comenzar a responder cualquier pregunta dibujemos un diagrama de Venn para organizar
la informacin disponible. Denamos A y B as: A = conjunto de personas que compran CC y B
= conjunto de personas que leen Gua. Si C es un conjunto, entonces n(C) indica el nmero de
elementos que ste contiene. En trminos de esta notacin, tenemos que n(A) = 25, n(B) = 20,
n(A B) = 5, n(U) = 100.
Inicialmente slo tenemos elementos para una regin bsica denominada AB. Colocamos inmedi-
atamente 5 en esta regin. Ahora A es la unin de los dos conjuntos AB y AB. Puesto que el
disco que representa a A tiene 25 elementos y hay 5 elementos en una parte, entonces debe haber
20 elementos en la otra. As n(A B) = 20. En forma similar n(B A) = 20 5 = 15.
Puesto que n(U) = 100 y U es la unin de conjuntos disjuntos A B, A B, B A y A B,
tenemos
100 = n(U) = 20 + 5 + 15 +n(A B).
Podemos concluir que n(A B) = 100 40 = 60.
Ahora regresemos a las preguntas:
a) n(B) = 60 + 20 = 80;
b) n(A B) = 20 + 5 + 15 = 40;
c) Si 5 personas de las 20 que leen Gua tambin compran CC, la fraccin de aquellas es
1
4
;
d) La fraccin de la muestra de 100 compradores de fertilizantes que compran CC es
25
100
=
1
4
.
Los resultados indican que sta fraccin no cambia por anunciar en Gua.
Ejemplo 2.36 En una clase de 50 alumnos de primer nivel de universidad, 30 estudian lge-
bra, 25 analtica y 10 lgebra y analtica. Cuntos alumnos estudian una de las dos materias?
Solucin
Sea U la clase de 50 alumnos, A el subconjunto de los que estudian lgebra y B el de los que
estudian analtica. Para responder a la pregunta, se necesita [A B[. En la gura los nmeros
de las regiones se obtienen de la informacin: [A[ = 30, [B[ = 25, [A B[ = 10. Por tanto,
[AB[ = 45 ,= [A[ +[B[ pues [A[ +[B[ cuenta dos veces a los alumnos de AB. Para evitar esta
CAPTULO 2. TEORA DE CONJUNTOS 64
sobre cuenta se resta [A B[ de [A[ +[B[ y se obtiene la frmula correcta:
[A B[ = [A[ +[B[ [A B[.
2.4. Tarea
1. Sea A = 1, 2, 4, a, b, c. Responda si lo siguiente es verdadero o falso:
a) 2 A; b) 3 A; c) c / A; d) A; e) / A; f ) a A.
2. Sea A = x/x es un nmero real y x < 6. Responda si lo siguiente es verdadero o falso:
a) 3 A; b) 6 A; c) 5 / A; d) 8 / A; e) 8 A; f) 3, 4 / A.
3. Escriba el conjunto en la forma x/P(x), donde P(x) es una propiedad que los elementos
del conjunto tienen en comn:
a) 2, 4, 6, 8, 10; b) a, e, o, i, u; c) 1, 4, 9, 16, 25, 36; d) 2, 1, 0, 1, 2.
4. Dados los conjuntos
U = 2, 1, 0, 1, 2, 3, 4, 5, A = 2, 1, 3, 5, B = 1, 0, 3, 4
determine cada una de las siguientes relaciones:
a) A B; b) A B; c) A B; d) A B; e) A B; f ) A B.
5. Demuestre lo siguiente:
a) A A B; b) A B A; c) AA = ; d) AB = A B;
e) A(AB) B; f ) Si C A y C B, entonces C A B;
g) Si A C y B C, entonces A B C.
6. Sea A = 1, 2, 3, 4, 5. Cul de los siguientes conjuntos es igual a A?
a) 4, 1, 2, 3, 5; b) 2, 3, 4; c) 1, 2, 3, 4, 5, 6;
d) x/x es un entero y x
2
25; e) x/x es un entero positivo y x 5;
f ) x/x es un nmero racional positivo y x 5.
7. Cul de los siguientes conjuntos es vaco?
a) x/x es un nmero real y x
2
1 = 0; b) x/x es un nmero real y x
2
+ 1 = 0;
c) x/x es un nmero real y x
2
= 9; d) x/x es un nmero real y x = 2x + 1;
e) x/x es un nmero real y x = x 1.
8. Sea A = 1, 2, 5, 8, 11. Responda si lo siguiente es verdadero o falso:
a) 5, 1 A; b) 8, 1 A; c) 1, 6 A; d) 1, 8, 2, 11, 5 A;
e) A;
f ) 2 A; g) 3 / A; h) A 11, 2, 5, 1, 8, 4.
CAPTULO 2. TEORA DE CONJUNTOS 65
9. Sea A = x/x es un entero y x
2
< 16. Responda si lo siguiente es verdadero o falso:
a) 0, 1, 2, 3 A; b) A; c) 3, 2, 1 A; d) A 3, 2, 1, 0, 1, 2, 3;
e) x/x es un entero y [x[ < 4 A.
10. Sea A = 1, 2, 3, ..., 15:
a) Cuntos subconjuntos de A contienen todos los enteros impares de A?
b) Cuntos subconjuntos de A contienen exactamente tres enteros impares?
c) Cuntos subconjuntos de A de ocho elementos contienen exactamente tres enteros im-
pares?
11. Si A = 1, 2, encuentre P(A).
12. Demuestre que A = B si y slo si A B y B A.
13. Dados los conjuntos
U = a, b, c, d, e, f, g, h, k, A = a, b, c, g, B = d, e, f, g
C = a, c, f, D = f, h, k.
Obtenga:
a) A B;
b) B C;
c) A C;
d) B D;
e) AB;
f ) A;
g) A B;
h) A C;
i) A D;
j) B D;
k) C D;
l) A (B C);
m) B C;
n) C B;
o) B;
p) C D;
q) A B;
r) A B C;
s) A D;
t) (A B) C;
u) A B C;
v) A B;
w) C D;
x) C D;
y) C D;
z) C .
14. En una encuesta hecha a 120 personas se encontr que a 71 personas les gusta escuchar
msica clsica, a 80 personas les gusta escuchar msica nacional, y que a 42 de ellas les
gustaba ambos tipos de msica:
a) A cuntas personas, de las encuestadas, les gusta la msica clsica, pero no la msica
nacional?
b) A cuntas personas no les gusta ninguna de las dos?
Resp: a) 29 personas; b) 11 personas.
15. En un zoolgico hay 80 animales de 11 meses de nacidos. A tal edad se les ensean dos
aspectos: ambientacin y a cambio de alimentacin. Hay 40 animales ambientndose, 30 cam-
biando su alimentacin y 20 aprendiendo ambas cosas:
a) Cuntos animales se ambientan, pero no cambian su alimentacin?
b) Cuntos cambian su alimentacion, sin cambiar su ambiente?
c) Cuntos animales cambian su alimentacin o su ambiente?
Resp: a) 20 animales se ambientan sin cambiar su alimentacin; b) 10 cambian su
alimentacin sin cambiar su ambientacin; c) 50 animales cambian su alimentaci;on o su
ambiente.
CAPTULO 2. TEORA DE CONJUNTOS 66
16. En un grupo de 90 alimentos, 36 productos contienen azcar, 32 tienen cido ctrico y 32
conservador; 6 productos contienen a la vez, azcar, cido ctrico y conservador; 12 contienen
cido ctrico y azcar, 10 contienen conservador y azcar, y nalmente 8 contienen cido
ctrico y conservador:
a) Cuntos productos contienen exclusivamente cido ctrico?
b) Cuntos slo azcar?
c) Cuntos contienen slo conservador?
d) Cuntos de los productos contienen cido ctrico y conservador, pero no azcar?
e) Cuntos productos no contienen ninguna de las sustancias mencionadas?
Resp: a) 18; b) 20; c) 20; d) 2; e) 14.
17. En un restaurant de 900 comidas servidas durante cierto da laboral se obtuvo la siguiente
informacin:
370 incluyeron lete de pescado;
290 incluyeron carne asada;
214 incluyeron tinga de pollo
30 incluyeron lete y carne asada;
40 incluyeron lete y tinga;
20 incluyeron carne asada y tinga;
20 incluyeron lete, carne asada y tinga.
a) Cuntas comidas llevaron exclusivamente lete?
b) Cuntas comidas llevaron exclusivamente carne asada?
c) Cuntas no llevaron ninguno de los tres?
d) Cuntas llevaron lete o carne asada, pero no tinga?
Resp: a) 320 comidas llevaron slo lete; b) 260 tienen slo carne asada; c) 96
comidas llevaron ninguno de los tres; d) 590 comidas que llevaron lete o carne asada,
pero no tinga.
18. En una encuesta a 40 personas sobre sus deportes olmpicos preferidos, se encontr que
a 20 les gusta la gimnasia, a 20 la natacin y a 12 el ciclismo. A 5 de estas personas les
gustan simultneamente los tres deportes, a 8 la gimnassia y la natacin, a 7 la gimnassia y
el ciclismo, y a 6 la natacin y el ciclismo:
a) A cuntas personas les gusta la natacin y el ciclismo pero, no la gimnassia?
b) A cuntas les gusta la gimnassia o el ciclismo, pero no la natacin?
c) A cuntas les gusta uno o dos de estos deportes, pero no los tres conjuntamente?
Resp: a) 1; b) 16; c) 31.
19. Se interrog a 300 jvenes acerca de la adiccin al tabaco, alcohol, drogas o alguna com-
binacin de stas. Se encontr que 122 lo eran al alcohol, 212 al tabaco y 97 a las drogas, 67
eran adictos tanto al alcohol como al tabaco, 50 al alcohol y a las drogas, 44 al tabaco y a
las drogas, y solamente 7 lo eran a los tres tipos:
a) Cuntos son adictos al alcohol pero no al tabaco?
b) Cuntos son adictos al alcohol y las drogas, pero no al tabaco?
c) Cuntos son adictos al tabaco o a las drogas, pero no al alcohol?
Resp: a) 11 jvenes; b) 24 jvenes; c) 64 jvenes.
20. En una encuesta aplicada a 260 estudiantes de preparatoria se obtuvieron los siguientes
datos:
CAPTULO 2. TEORA DE CONJUNTOS 67
64 toman un curso de matemticas
94 toman un curso de computacin
58 toman un curso de administracin
28 toman cursos de matemticas y administracin
26 toman cursos de matemticas y computacin
22 toman cursos de administracin y computacin
14 toman los tres cursos.
a) Cuntos de los estudiantes de la encuesta no toman ninguno de los tres cursos?
b) Cuntos de los estudiantes de la encuesta toman slo el curso de computacin?
21. Una encuesta aplicada a 500 televidentes produce la siguiente informacin:
285 ven programas cmicos
195 ven programas deportivos
115 ven programas culturales
45 ven programas cmicos
70 ven programas deportivos
50 ven programas culturales
50 no ven ningn programa.
a) Cuntos entrevistados ven los tres tipos de programas?
b) Cuntos entrevistados ven slo uno de los tres?
22. Cundo es AB = B A? Explique.
23. Sean
U = 1, 2, 3, ..., 9, 10, A = 1, 2, 3, 4, 5, B = 1, 2, 4, 8,
C = 1, 2, 3, 5, 7, D = 2, 4, 6, 8.
Determine las relaciones:
a) (A B) C;
b) A (B C);
c) C D;
d) C D;
e) (A B) C;
f ) A (B C);
g) (B C) D;
h) B (C D);
i) (A B) (C D).
24. En cada parte, encuentre el conjunto con el menor nmero de elementos posible, que con-
tenga a los conjuntos dados como subconjuntos:
a) a, b, c, a, d, e, f, b, c, e, g; b) 1, 2, 1, 3, ; c) 1, a, 2, b.
25. Dados los conjuntos
U = 1, 2, 3, 4, 5, 6, 7, 8, 9, A = 1, 2, 4, 6, 8, B = 2, 4, 5, 9
C = x/x es un nmero entero y x
2
16, D = 7, 8.
Obtenga:
CAPTULO 2. TEORA DE CONJUNTOS 68
a) AB;
b) B A;
c) A;
d) A B;
e) A D;
f ) B C;
g) A D;
h) B C;
i) C D;
j) C;
k) C D;
l) B C;
m) A B C;
n) B C D;
o) A B C;
p) A A;
q) A A;
r) A A;
s) A B;
t) A B;
u) A (B C);
v) (A B) D;
w) B C D;
x) A (C D).
26. Dados los conjuntos
U = a, b, c, d, e, f, g, h, A = a, c, f, g, B = a, e, C = b, h.
Obtenga:
a) A;
b) B;
c) A B;
d) A B;
e) C;
f ) AB;
g) A B;
h) B C;
i) A A;
j) C C;
k) A B;
l) B C.
27. Dados los conjuntos U = R, A = x/x es una solucin de x
2
1 = 0, B = 1, 4.
Obtenga:
a) A; b) B; c) A B; d) A B.
28. Suponga que A y B son subconjuntos de un conjunto universal U con n(U) = 100. En-
cuentre el nmero de elementos en cada una de las regiones bsicas del diagrama de Venn si:
a) n(A) = 40, n(B) = 70 y n(A B) = 20;
b) n(A) = 30, n(B) = 60 y n(A B) = 85;
c) n(A) = 35, n(A B) = 5 y n(A B) = 32;
d) n(B A) = 20, n(B) = 30 y n(A B).
29. Una encuesta de 1.000 personas mayores de 40 aos revel que 312 fumaban, 80 tenan
cncer y 660 ni fumaban ni tenan cncer. Dibuje un diagrama de Venn para responder estas
preguntas:
a) Cuntas personas de las encuestadas fumaban y tenan cncer?;
b) Qu porcentaje de fumadores tenan cncer?
c) Puede la encuesta indicar que fumar produce cncer?
30. Considere los siguientes conjuntos:
A = 2, 4, 5, 7, B = x/x Z y x es un cuadrado perfecto
C = x/x Z y x
2
= 4, D = 1, 2, 0.
Cules pares de conjuntos son disjuntos?
31. Se dan los conjuntos A, B y C. Con ayuda de las operaciones de unin e interseccin
escriba un conjunto que conste de los elementos pertenecientes:
a) A los tres conjuntos; b) Por lo menos a dos de dichos conjuntos;
c) Por lo menos a un conjunto.
CAPTULO 2. TEORA DE CONJUNTOS 69
32. Suponga que A y B son subconjuntos de un conjunto universal U con n(U) = 100. En-
cuentre el nmero de elementos en cada una de las regiones bsicas del diagrama de Venn si:
a) n(A) = 40, n(B) = 70 y n(A B) = 20;
b) n(A) = 30, n(B) = 60 y n(A B) = 85;
c) n(A) = 35, n(A B) = 5 y n(A B) = 32;
d) n(B A) = 20, n(B) = 30 y n(A B) = 47.
33. Dados los conjuntos
U = 2, 1, 0, 1, 2, 3, 4, 5, A = 2, 1, 3, 5, B = 1, 0, 3, 4.
Determine cada una de las siguientes relaciones:
a) AB; b) AB; c) AB; d) A B; e) AB; f ) AB.
34. Verique que [A B[ = [A[ +[B[ [A B[ dados los conjuntos:
a) A = 1, 2, 3, 4, B = 2, 3, 5, 6, 8;
b) A = a, b, c, d, e, f, B = a, c, f, g, h, i, r;
c) A = 1, 2, 3, 4, B = 5, 6, 7, 8, 9;
d) A = x/x es un nmero entero positivo < 8;
B = x/x es un nmero entero tal que 2 x 5;
e) A = a, b, c, d, e, B = f, g, r, s, t, u;
f ) A = x/x es un nmero entero positivo y x
2
16,
B = x/x es un nmero entero negativo y x
2
25.
35. Si A y B son conjuntos disjuntos tales que [A B[ = [A[, qu se puede decir de B?
36. Calcule la diferencia simtrica A B si:
a) A = 1, 3, 4, 6, 9 y B = 1, 2, 3, 7; b) A = 1, 3, 4, 6, 9 y B = 2, 5;
c) A = 1, 3, 4, 6, 9 y B = 3, 6, 9.
37. Determine AB:
a) A = 1, 3, 5, 7, B = 5, 9, 11, 13;
b) A = 1, 0, 1, 4, 6, 7, 9, 11, B = 2, 4, 6, 8, 10;
c) A = 1, 2, 3, 4, B = 6, 8, 10, 12;
d) A = 4, 6, 8, B = 2, 4, 6, 8, 10.
38. Una encuesta de 200 obreras mayores de 30 aos revela que 60 tienen grado preuniversi-
tario, 80 ganan ms de $ 4.000 al ao y 30 tienen grado preuniversitario y ganan ms de $
4.000 al ao. Dibuje un diagrama de Venn para responder estas preguntas:
a) Cuntas mujeres ni tienen grado preuniversitario ni ganan ms de $ 4.000 al ao?
b) Qu porcentaje de las mujeres que tienen grado preuniversitario ganan ms de $ 4.000
al ao?
c) Indican los resultados de la encuesta que las mujeres con grado preuniversitario tienen
mayores probabilidades de mejorar sus ingresos?
CAPTULO 2. TEORA DE CONJUNTOS 70
39. Una encuesta de 1.000 personas mayores de 40 aos revel que 312 fumaban, 80 tenan
cncer y 660 ni fumaban ni tenan cncer. Dibuje un diagrama de Venn para responder estas
preguntas:
a) Cuntas personas de las encuestadas fumaban y tenan cncer?
b) Qu porcentaje de fumadores tenan cncer?
c) Puede la encuesta indicar que fumar produce cncer?
40. Considere los conjuntos
A = (x, y) R
2
/2xy = 4; B = (x, y) R
2
/x+3y = 9; C = (x, y) R
2
/y = 2x.
Encuentre lo siguiente:
a) A B; b) A C; c) B C; d) A C.
41. Demuestre las siguientes proposiciones. Supngase un universo U:
a) Si A B, C D, entonces A C B D y A C B D;
b) A B si, y slo si, A B = ;
c) A B si, y slo si, A B = U.
42. Demuestre que la igualdad A(BC) = (AB)C es cierta cuando y slo cuando A C.
43. Demuestre las igualdades:
a) A(AB) = A B;
b) A (B C) (A B) C;
c) (AB) C = A(B C);
d) (AB) C = (A C) (B C);
e) (AB) (B A) = (AB) (AB);
f ) (A C) B (AB) C.
44. Demuestre que la inclusin AB C es cierta cuando y slo cuando A B C.
45. Determine en qu razn (X Y, X Y, X = Y ) se encuentran los conjuntos X e Y si:
a) X = A (B C), Y = (A B) (A C);
b) X = (A B) C, Y = (AC) (B C);
c) X = A(B C), Y = (AB) (AC).
46. Sean
A = x N/2 < x 6, B = x N/1 < x < 4, C = x N/x
2
4 = 0.
De qu elementos constan los conjuntos:
a) B C; b) A B C; c) A B C; d) (A B) (B C).
47. Los conjuntos A y B estn compuestos, respectivamente, de los elementos a = 4n + 2,
b = 3n, n N. Hallar A B.
CAPTULO 2. TEORA DE CONJUNTOS 71
48. Suponga que el conjunto A contiene n elementos, el conjunto B, m elementos y la inter-
seccin A B, k elementos. Hallar el nmero de elementos de A B.
49. Sea que A N y cada elemento de A es un nmero mltiplo bien a 2, o bien a 3, o bien a 5.
Hallar el nmero de elementos del conjunto A si entre ellos tenemos: 70 nmeros mltiplos a
2; 60 nmeros mltiplos a 3; 80 nmeros mltiplos a 5; 32 nmeros mltiplos a 6; 35 nmeros
mltiplos a 10; 38 nmeros mltiplos a 15; 20 nmeros mltiplos a 30.
50. Considere los siguientes conjuntos:
A = 2, 4, 5, 7, B = x/x Z y x es un cuadrado perfecto,
C = x/x Zyx
2
= 4, D = 1, 2, 0.
Cules pares de conjuntos son disjuntos?
51. Se dan los conjuntos A, B y C. Con ayuda de las operaciones de unin e interseccin
escriba un conjunto que conste de los elementos pertenecientes:
a) A los tres conjuntos; b) Por lo menos a dos de dichos conjuntos;
c) Por lo menos a un conjunto.
52. Si A B = A C, deber ser B = C? Explique.
53. Si A B = A C, deber ser B = C? Explique.
54. Demostrar que:
a) (A B) (A B) (A B) (A B) = ;
b) (A B) (A B) (A B) (A B) = U;
c) A B = (A B) (A B) (A B).
55. Demuestre que las igualdades A B = B y A B = A son ciertas cuando y slo cuando
A B.
56. Demuestre que el resultado es siempre cierto o bien d un ejemplo especco para demostrar
que no lo es:
a) Si A X = B X, entonces A = B; b) Si A X = B X, entonces A = B;
c) Si AB = C B, entonces A = C; d) Si AB = AC, entonces A = C;
e) (AB) (B A) = (A B) (A B).
57. Determine si la relacin es o no correcta:
a) (A B) (C D) = [(A C) (B C)] [(A B) D];
b) A (B C) = A (B C);
c) (A B C) D = [(A B) D] [(A B) C];
d) A (B C) = (A B) (A C).
CAPTULO 2. TEORA DE CONJUNTOS 72
58. Para los conjuntos A, B, C U, y mediante diagramas de Venn, analice la veracidad o
falsedad de las siguientes relaciones:
a) A (B C) = (A B) (A C); b) A (B C) = (A B) (A C);
c) A (B C) = (A B) (A C); d) A (B C) = (A B) (A C);
e) A(B C) = (AB) (AC); f ) A(B C) = (AB) (AC);
g) A (B C) = (A B) C.
59. Sean A y B subconjuntos tomados al azar del conjunto U. Demuestre las igualdades:
a) AB = AB; b) (AB)(AB) = AB; c) (AB)(AB) = AB.
60. Sea A U, B U. Hallar el conjunto X U verdaderas y cules falsas. Para las falsas,
proporcione un ejemplo en el que la armacin no se cumpla:
a) A (B C) = (A B) C para todo conjunto A, B, C;
b) A B A B implica que A = B;
c) (A ) B = B para todo conjunto A, B;
d) A ( B) = A siempre que A B;
e) A B = A B para todo conjunto A, B.
61. Hallar los subconjuntos A y B del conjunto U si se sabe que para todo conjunto X U
es cierta la igualdad X A = X B.
62. A B = U equivale a A B y a A B. Igualmente, A B = equivale a A B y
a A B. Demostrar que A B, es equivalente a A B = y a A B = U. Igualmente
A B, equivale a A B = y a A B = U. En particular, resulta B X A si y slo si
(X A) (B X) = o (X A) (B X) = U.
63. Demuestre las siguientes relaciones:
a) Si A = B y B = C, entonces A = C;
b) Si A = B, entonces A X = B X;
c) Si A = B, entonces A X = B X;
d) Si A = B, entonces A = B;
e) Si A B, entonces A B = B;
f ) Si A B, entonces A B = A;
g) Si A B, entonces B A;
h) Si A B, entonces AB = ;
i) Si A B = , entonces B A = B.
64. Demuestre que la igualdad A(BC) = (AB)C es cierta cuando y slo cuando A C.
65. Para cualquier conjunto A, qu es A A?qu es A ?
66. Utilizando las leyes de la teora de conjuntos, simplique las siguientes relaciones:
a) A B (A B C);
b) (A B) (A B C D) (A B);
c) (AB) (A B);
d) A (B A).
67. Demuestre las igualdades:
CAPTULO 2. TEORA DE CONJUNTOS 73
a) A(AB) = A B;
b) (AB) C = A(B C);
c) (AB) (B A) = (AB) (AB);
d) (AB) C = (A C) (B C).
68. Demuestre lo siguiente:
a) A (B C) = (A B) (A C);
b) A B (A C) (B C);
c) (A B C) = A B C;
d) A B A y A A B para todo conjunto A, B;
e) Si A B y A C, entonces A B C;
f ) Si A C y B C, entonces A B C;
g) A B si y slo si B A;
h) A B si y slo si A B = B.
69. Demuestre o refute lo siguiente:
a) A B = A C implica B = C;
b) A B = A C implica B = C;
c) A B = A C y A B = A C implica B = C;
d) A B A B implica A = B;
e) A B = A C implica B = C.
70. Demuestre que el complemento relativo no es conmutativo; es decir, A B = B A no
siempre es verdadero.
71. Demuestre que el complemento relativo no es asociativo; es decir, (AB)C = A(BC)
no siempre es verdadero.
72. Demuestre que (AB) C A(B C) para todo conjunto A, B, C.
73. Demuestre las inclusiones:
a) A (B C) (A B) C; b) (A C) B (AB) C.
74. Sean los conjuntos
A = x N/2 < x 6, B = x N/1 < x < 4, C = x N/x
2
4 = 0
De qu elementos constan los conjuntos:
a) B C; b) A B C; c) A B C; d) (A B) (B C).
75. Suponga que el conjunto A contiene n elementos, el conjunto B, m elementos y la inter-
seccin A B, k elementos. Hallar el nmero de elementos de A B.
76. Sea que A N y cada elemento de A es un nmero mltiplo bien a 2, o bien a 3, o bien a 5.
Hallar el nmero de elementos del conjunto A si entre ellos tenemos: 70 nmeros mltiplos a
2; 60 nmeros mltiplos a 3; 80 nmeros mltiplos a 5; 32 nmeros mltiplos a 6; 35 nmeros
mltiplos a 10; 38 nmeros mltiplos a 15; 20 nmeros mltiplos a 30.
CAPTULO 2. TEORA DE CONJUNTOS 74
77. Sean A y B subconjuntos tomados al azar del conjunto U. Demuestre las igualdades:
a) (AB) (AB) = AB; b) (AB) (AB) = AB; c) AB = AB.
78. Para los conjuntos A, B, C U, y mediante diagramas de Venn, analice la veracidad o
falsedad de las siguientes relaciones:
a) A (B C) = (A B) (A C);
b) A(B C) = (AB) (AC);
c) A (B C) = (A B) (A C);
d) A(B C) = (AB) (AC);
e) A (B C) = (A B) (A C);
f ) A (B C) = (A B) (A C);
g) A (B C) = (A B) C.
Captulo 3
Nmeros reales
3.1. Nmeros naturales
Los nmeros naturales pueden compararse entre s y en este caso est claro cul de los dos
nmeros es mayor. Todos los nmeros naturales dispuestos en el orden de crecimiento forman una
serie de nmeros naturales, es decir: 1, 2, 3, 4, ..., etc. A todo nmero natural le corresponde su
lugar en dicha serie. En lo sucesivo la serie de nmeros naturales se designar con la letra N.
Adicionar dos nmeros naturales m y n signica hallar en la serie de nmeros naturales un
nmero p, p > m. El nmero mencionado p se denomina suma de los nmeros m y n; se denota
con p = m+n, mientras que los nmeros m y n se llaman sumandos.
Para sumar varios nmeros naturales, es necesario adicionar al principio los dos primeros, luego
aadir a la suma obtenida el siguiente nmero natural, etc.
Multiplicar un nmero natural m por otro nmero natural n signica encontrar un nmero
natural q igual a n, si m = 1 y; a la suma de m nmeros, cada uno de los cuales es n, siempre que
m > 1. El citado nmero q se denomina producto de los nmeros m y n; se denota como q = mn,
y los nmeros m y n se denominan factores.
Para multiplicar varios nmeros naturales, se debe multiplicar al principio los dos primeros
nmeros, luego multiplicar el nmero natural obtenido por el siguiente nmero natural, etc.
A continuacin, damos a conocer las leyes principales de adicin y multiplicacin de los nmeros
naturales:
1. m+n = n +m ley conmutativa de la adicin.
2. (r +m) +n = r + (m+n) ley asociativa de la adicin.
3. mn = nm ley conmutativa de la multiplicacin.
4. (rm)n = r(mn) ley asociativa de la multiplicacin.
5. (r +m)n = rn +mn ley distributiva de la adicin respecto a la multiplicacin.
75
CAPTULO 3. NMEROS REALES 76
Si el nmero m gura en calidad de factor k veces, donde k es nmero natural superior a la
unidad, entonces el producto m m ... m
. .
k veces
se denomina k-sima potencia del nmero m y se rep-
resenta por m
k
, es decir, por denicin m
k
= m m ... m
. .
k veces
. Adems, de acuerdo con la denicin,
m
1
= m.
Las propiedades de las potencias, son las siguientes:
1. m
k
m
n
= m
k+n
.
2. (m
k
)
r
= m
kr
.
3. m
k
n
k
= (mn)
k
.
Sustraer de un nmero natural n otro nmero natural m signica encontrar un nmero natural
p tal, que sea p = n m.
No siempre existe tal nmero natural p que se cumpla la igualdad anterior para cualesquiera
nmeros naturales n y m. Si n > m, tal nmero existe y es nico. Este se denomina diferencia o
resto entre los nmeros n y m; el nmero n se llama minuendo, y m, sustraendo.
Dividir un nmero natural n por otro nmero natural m signica hallar un nmero natural q
tal que se verique la igualdad p =
n
m
.
No siempre existe tal nmero natural q que se verique la igualdad para cualesquiera nmeros
naturales n y m. Si dicho nmero existe, entonces m y q se denominan divisores del nmero n.
Apoyndose en las leyes principales de adicin y multiplicacin de los nmeros naturales y en
las deniciones de las operaciones de sustraccin y divisin, se puede armar lo siguiente:
1. Si el nmero m es un divisor de los nmeros p y q, entonces m ser el divisor de la suma
p +q.
2. Si m es un divisor de los nmeros p y q, siendo p > q, entonces el nmero m ser el divisor
de la diferencia p q.
3. Si m = n, entonces m+k = n +k para cualquier nmero natural k.
4. Si m = n, entonces mr = n r para cualquier nmero natural r tal, que sea m > r.
5. Si m = n, entonces mp = np para cualquier nmero natural p.
6. Si m = n, entonces para cualquier nmero natural q que es el divisor del nmero m.
Veamos un nmero nuevo, a saber, el nmero cero. Para designarlo se emplea el smbolo 0. El
cero no es un nmero natural y se considera un nmero precedente a todos los nmeros naturales.
La serie de nmeros naturales junto con el nmero cero lleva el nombre de serie natural ampliada.
En la serie ampliada de nmeros naturales se pueden denir las operaciones de adicin y
multiplicacin; con este objeto es suciente aadir a las deniciones de la adicin y multiplicacin
de los nmeros naturales las deniciones correspondientes de la adicin y multiplicacin, en las
cuales interviene el nmero cero:
CAPTULO 3. NMEROS REALES 77
1. 0 +n = n + 0 = n.
2. 0 + 0 = 0.
3. 0 n = n 0 = 0.
4. 0 0 = 0.
Por denicin, la potencia nula de todo nmero natural m es la unidad, es decir, m
0
= 1. La
divisin por cero y la elevacin del cero a potencia nula son operaciones no determinadas.
3.2. Nmeros primos y compuestos
El conjunto de nmeros naturales se compone de la unidad y de nmeros primos y compuestos.
Un nmero natural superior a la unidad se denomina primo, si es divisible solamente por s
mismo y por la unidad. Un nmero natural superior a la unidad se llama compuesto, si tiene por
lo menos un divisor distinto de la unidad y de s mismo.
Todo nmero compuesto p puede ser escrito en forma de un producto de nmeros primos.
Ejemplo 3.1 El nmero 713 se descompone en 2331. En este caso suele decirse que el nmero
compuesto p est descompuesto en factores primos.
Cuando un nmero se descompone en factores primos, algunos de estos ltimos pueden encon-
trarse en la descomposicin varias veces. Tal factor primo se escribe, elevado a una potencia que
muestra cuntas veces l interviene como factor.
Ejemplo 3.2 El nmero 2701125 se descompone en 3
2
5
3
7
4
.
Cualquier nmero natural puede ser escrito en la forma
p = p
n
1
1
p
n
2
2
... p
n
k
k
(3.1)
donde p
1
, p
2
, ..., p
k
son diferentes divisores primos del nmero p, y n
1
, n
2
, ..., n
k
sealan cuntas
veces dichos divisores se repiten en la descomposicin del nmero p. La descomposicin (1) de
un nmero natural p en factores primos es nica, es decir, no existen otros nmeros primos que
sean divisores del nmero p, y las potencias n
1
, n
2
, ..., n
k
no pueden sustituirse por otras potencias.
Si los nmeros naturales p
1
y p
2
son divisibles por un mismo nmero natural p, este ltimo se
denomina divisor comn de los nmeros p
1
y p
2
. El nmero natural mximo por el que se dividen
p
1
y p
2
lleva el nombre de mximo comn divisor (MCD) de dichos nmeros. Si el MCD de dos
nmeros es igual a la unidad, se llaman recprocamente primos.
Si los nmeros naturales p
1
y p
2
son recprocamente primos y el nmero natural p es divisible
tanto por p
1
como por p
2
, entonces p se divide por el producto p
1
p
2
.
Ejemplo 3.3 Determine el mximo comn divisor de los nmeros 55125 y 16335.
Solucin
Como 55125 = 3
2
5
3
7
2
y 16335 = 3
3
5 11
2
, entonces el MCD = 3
2
5.
Se llama mnimo comn mltiplo (MCM) de dos nmeros naturales p
1
y p
2
un nmero natural
mnimo que es divisible por p
1
y por p
2
.
CAPTULO 3. NMEROS REALES 78
Ejemplo 3.4 Determine el mnimo comn mltiplo de los nmeros 55125 y 16335.
Solucin
Como 55125 =3
2
5
3
7
2
y 16335 =3
3
5 11
2
, entonces
MCM = 3
3
5
3
7
2
11
2
.
Si, como resultado de la divisin de un nmero natural p por otro nmero natural m, se obtiene
el nmero natural q tal que q =
p
m
, se dice que p se divide por m.
Dividir enteramente un nmero natural p por otro nmero natural m signica encontrar dos
nmeros q y r, de la serie natural ampliada, que verique la igualdad p = mq +r, con la particu-
laridad de que r satisfaga la condicin 0 r < m. El nmero q se denomina cociente y el nmero
r, resto de la divisin. Si r = 0, se dice que el nmero natural p se divide por el nmero natural m
exactamente.
Ejemplo 3.5 Determnese el MCD(1144; 360).
Solucin
Como 1144 = 360 3 + 64, entonces MCD(1144; 360) = MCD(360; 64).
Como 360 = 64 5 + 40, entonces MCD(360; 64) = MCD(64; 40).
Como 64 = 40 1 + 24, entonces MCD(64; 40) = MCD(40; 24).
Como 40 = 24 1 + 16, entonces MCD(40; 24) = MCD(24; 16).
Como 24 = 16 1 + 8, entonces MCD(24; 16) = MCD(16; 8).
Como 16 = 8 2, entonces MCD(16; 8) = MCD(8; 0) = 8, es decir,
MCD(1144; 360) = 8.
Una vez determinado el MCD(p; m), resulta posible hallar tambin el mnimo comn mltiplo
de estos nmeros MCM(p; m), de la siguiente manera
MCM(p; m) =
p m
MCD(p; m)
.
Ejemplo 3.6 Determnese el MCM(1144; 360).
Solucin
Como el MCD(1144; 360) = 8, entonces
MCM(p; m) =
1144 360
8
= 51480.
3.3. Nmeros enteros
Anteriormente sealamos que la sustraccin no es siempre realizable dentro del conjunto de
nmeros naturales. Por esta razn tenemos la necesidad de ampliar el conjunto de nmeros natu-
rales.
En este anlisis, introduciremos nuevos nmeros, nmeros naturales de signo menos, es decir,
nmeros del tipo m, donde m es un nmero natural, y los llamaremos nmeros enteros negativos,
notados por Z

.
Diremos que dos nmeros enteros negativos m y n son iguales, si son iguales los nmeros
m y n.
CAPTULO 3. NMEROS REALES 79
Examinemos un conjunto de nmeros, que incluye todos los nmeros naturales, el cero y todos
los nmeros negativos. Convengamos en considerar que dos nmeros de dicho conjunto son iguales,
si ambos son nmeros naturales iguales, si ambos son nmeros enteros negativos iguales, o si cada
uno de ellos es cero.
Denamos ahora las operaciones de adicin y multiplicacin para los nmeros de este conjunto.
Si ambos nmeros que han de ser adicionados o multiplicados pertenecen a una serie natural ampli-
ada, entonces, las operaciones de adicin y multiplicacin para dichos dos nmeros se determinan
de igual forma.
Si uno de los nmeros o ambos nmeros, que deben sumarse o multiplicarse, son enteros nega-
tivos, las operaciones de adicin y multiplicacin para estos dos nmeros se realizan de la siguiente
manera:
1. (m) + (n) = (m+n);
2. (m) + 0 = 0 + (m) = m;
3. (m) +n =
_

_
(mn), siempre que m > n
n m, cuando m < n
0, si m = n
;
4. (m)n = m(n) = (mn);
5. (m)(n) = mn;
6. (m) 0 = 0 (m) = 0.
Un conjunto de nmeros que incluye todos los nmeros naturales, el cero y todos los nmeros
enteros negativos con las deniciones de igualdad y de operaciones de adicin y multiplicacin, se
denomina conjunto de nmeros enteros y se representa por Z, mientras que los nmeros menciona-
dos llevan el nombre de nmeros enteros.
Las leyes principales de adicin y multiplicacin de los nmeros enteros son anlogas a aquellas
que se usan para sumar y multiplicar los nmeros naturales.
Para las operaciones de adicin y multiplicacin de los nmeros enteros se introducen opera-
ciones inversas, las de sustraccin y divisin, excepto de la divisin por cero. La operacin de
sustraccin es en este caso siempre realizable, y la operacin de divisin, no siempre es posible. Sin
embargo, al igual que para los nmeros naturales, los nmeros enteros siempre admiten la divisin
entera.
Dividir un nmero entero a por un nmero natural m con resto, signica hallar dos nmeros
enteros q y r tales, que sea vlida la igualdad a = mq +r, con la particularidad de que r satisface
la condicin 0 r < m.
A continuacin damos algunas propiedades:
1. Sea a un nmero entero cualquiera y sea m, cualquier nmero natural. Entonces, existe el
nico par de nmeros enteros q y r que satisface las condiciones a = mq +r y 0 r < m.
2. Todo nmero par a puede escribirse en la forma a = 2q, donde q es cierto nmero entero.
CAPTULO 3. NMEROS REALES 80
3. Todo nmero impar a puede escribirse en la forma a = 2q + 1, donde q es cierto nmero
entero.
4. Cualquier nmero entero a, que se divide exactamente por cierto nmero natural k, puede
ser escrito en la forma a = kq, donde q es un nmero entero.
5. Cualquier nmero entero a, que no se divide exactamente por cierto nmero natural k,
puede escribirse en la forma a = kq +r, donde r es uno de los nmeros 1, 2, ..., k 1, y q es
un nmero entero.
3.4. Nmeros racionales
De acuerdo con lo expuesto anteriormente, en el conjunto de nmeros naturales no son siempre
realizables las operaciones de sustraccin y divisin. Es por esta razn que surge la necesidad de
introducir nmeros nuevos.
Introduzcamos en este anlisis nmeros nuevos: fracciones con signo menos, es decir, nmeros
del tipo
m
n
, donde m y n son nmeros naturales. La fraccin
m
n
se denomina opuesto de la
fraccin
m
n
.
Un conjunto de nmeros compuesto por todos los nmeros del tipo
p
q
, donde q es un nmero
natural y p, un nmero entero, con las deniciones de igualdad y de operaciones de adicin y
multiplicacin, que acabamos de introducir, recibe el nombre de conjunto de nmeros racionales
y se representa por la letra Q; los propios nmeros se denominan racionales. Si p es un nmero
natural, entonces
p
q
se llama nmero racional positivo o fraccin positiva. Si, en cambio, p es un
nmero negativo, el nmero
p
q
se denomina nmero racional negativo o fraccin negativa.
Est claro que el conjunto de nmeros enteros es una parte del conjunto de nmeros racionales.
Para las operaciones de adicin y multiplicacin de nmeros racionales se introducen opera-
ciones inversas; las de sustraccin y divisin, y en este caso, ambas operaciones, a excepcin de
la divisin ilegtima por cero, son siempre realizables. Las leyes principales de adicin y multipli-
cacin de nmeros racionales son anlogas a las leyes correspondientes de adicin y multiplicacin
de nmeros enteros.
Si un nmero racional r gura como factor k veces, k > 1, entonces el producto r r ... r
. .
k veces
se
llama k-sima potencia del nmero r y se representa por rk. Adems por denicin, r
1
= r.
Al igual que en el caso de los nmeros naturales, son vlidas las siguientes leyes de las potencias
de los nmeros racionales:
1. r
m
r
k
= r
m+k
;
2. r
m
s
m
= (rs)
m
;
3. (r
k
)
m
= r
km
;
4.
_
r
s
_
k
=
r
k
s
k
, si s ,= 0;
CAPTULO 3. NMEROS REALES 81
5.
r
k
r
m
= r
km
, si k > m, r ,= 0.
Por denicin, r
0
= 1 para cualquier nmero racional r, salvo el nmero cero.
3.5. Tarea
1. Simplique las siguientes expresiones:
a)
9
16
+
7
12

5
6
+
1
2

5
6
+
1
4
;
b)
5
11

1
33
+ 1 +
2
3
2;
c) (0, 7 + 1, 75 0, 5)
_
1
0, 625 0, 3
_
;
d)
3
5
+
1
2

7
10
+
1
3

5
6
;
e)
1
5
+
3
4

1
2
+
2
3
;
f )
5
12

7
18
+ 4
1
6
2 +
4
9
;
g)
(1 0, 2)(0, 3 1, 5) +

0, 09
0, 3 (1, 4 5 0, 3)
2
;
h)
2
9

1
3
+
4
5

7
15
;
i)
_
1
2
_
2

144 +
2
3

_
9
4

_
1
36
25
;
j)
(2, 5 + 0, 25 0, 875) 0, 1875
0, 6 3, 5
;
k)
1, 08 1 0, 1
1 0, 8
0, 1;
l)
_
1 + 0, 2 0, 01
1 0, 5
2
1, 6
_
0, 04
_
2
.
Resp: a)
11
48
; b)
1
11
; c)
83
36
; d)
1
10
; e)
67
60
; f ) 6; g)
11
500
; h)
2
9
;
i)
4
3
; j)
100
21
; k)
49
100
; l) 400.
2. Simplique las siguientes operaciones:
a)
1
2
25
0, 75
0, 5
3
5
+ 0, 7
_
_
_
1
2

7
16
3
8

16
5
_
_
_
6
5
b)
0, 8 (1, 2) + 0, 3 3 (7, 08)
_
0, 75 0, 5 + 0, 625
5 (0, 5) 3, 75
_
2
(1 + 0, 5)
2

0, 01
c)
_
_
16
25
4 0, 3
_
1
3
4
+ 2, 25
_
2
d)
_
_
7
36
7 0, 6
_
5 +
4
9
+ 1, 25
_
1
e)
3
7
+
1
3
(2)
32
27

9
4
_

3
14
_
_
2
3
4
_
1
2
3
4
+ 2
3
4
+ 2
f )
9
4
1
7
53

3
7
6 +
12
5

3
5
_
2
5

2
3
+
4
5
1 +
3
8
__
5
1
2
+
37
4
_
Resp: a)
17
16
; b)
1
100
; c)
3
11
; d)
39
70
; e)
4
25
; f ) 60.
3. De dos ciclistas que tienen que hacer el mismo camino, uno ha hecho los
12
13
y el otro los
1
15
. Cul ha recorrido mayor camino?
4. Un automvil recorre 163 km en 3 horas; otro 103 km en 2 horas, y un tercero 275 km en
4 horas. Cul de ellos tiene una velocidad promedio menor?
CAPTULO 3. NMEROS REALES 82
3.6. Fracciones
Anteriormente sealamos que la divisin no es siempre realizable dentro del conjunto de nmeros
naturales. Para que la divisin sea siempre realizable, hay que considerar nmeros nuevos, las frac-
ciones, lo cul denimos a continuacin.
Un nmero igual a la k-sima parte del nmero uno se designa
1
k
, siendo k un nmero natu-
ral mayor que la unidad. Si la parte aducida se toma m veces, entonces la designacin del nuevo
nmero obtenido ser
m
k
, siendo m un nmero natural. Un nmero que se determina segn esta
regla con ayuda de dos nmeros naturales p y q (q > 1) y que se nota en forma
p
q
, se llama fraccin
o cociente de los nmeros naturales p y q, y en este caso p se denomina numerador de esta fraccin
y el nmero q, denominador.
Todo nmero natural puede ser considerado como una fraccin cuyo denominador es la unidad,
es decir, cualquier nmero natural n puede escribirse en forma de una fraccin
n
1
.
Dos fracciones
p
q
y
m
k
se dice son iguales, si el producto del numerador de la primera fraccin
por el denominador de la segunda es igual al producto del numerador de la segunda fraccin por
el denominador de la primera, es decir
p
q
=
m
k
, pk = qm.
Ejemplo 3.7 Los fracciones
3
2
y
12
8
son iguales, porque
12
8
=
3 4
2 4
=
3
2
.
Se denomina suma de dos fracciones una fraccin cuyo numerador es igual a la suma de los
productos del numerador de la primera fraccin por el denominador de la segunda y del numerador
de la segunda fraccin por el denominador de la primera, mientras que el denominador es igual al
producto de los denominadores de dichas fracciones, es decir
p
q
+
m
k
=
pk +qm
qk
.
Ejemplo 3.8 Dadas las fracciones
2
3
y
3
4
, entonces
2
3
+
3
4
=
2 4 + 3 3
12
=
8 + 9
12
=
17
12
.
Se llama producto de dos fracciones, una fraccin cuyo numerador es igual al producto de los
numeradores de estas fracciones y el denominador, al producto de los denominadores, es decir
p
q

m
k
=
pm
qk
Ejemplo 3.9 Dadas las fracciones
5
2
y
3
7
, entonces
5
2

3
7
=
5 3
2 7
=
15
14
.
CAPTULO 3. NMEROS REALES 83
Dividir una fraccin
p
q
por otra fraccin
m
n
signica hallar tal fraccin
r
k
que se verique
p
q

m
n
=
p
q

n
m
=
r
k
.
A diferencia de los nmeros naturales, la divisin para las fracciones es siempre realizable.
Ejemplo 3.10 Dadas las fracciones
7
5
y
4
3
, entonces
7
5

4
3
=
7
5

3
4
=
21
20
.
Sustraer de una fraccin
p
q
otra fraccin
m
n
signica hallar tal fraccin
r
s
que cumpla la igualdad
p
q

m
n
=
pn qm
qn
=
r
s
.
Ejemplo 3.11 Dadas las fracciones
5
2
y
3
7
, entonces
5
2

3
7
=
5 7 3 2
14
=
35 6
14
=
29
14
.
Al igual que en el caso de los nmeros naturales, la sustraccin de las fracciones no es siempre
realizable.
Denicin 3.1 Fraccin irreducible
La fraccin
p
q
se denomina irreducible, si los nmeros p y q son recprocamente primos.
A continuacin damos algunas leyes de las fracciones:
1.
p
q
+
m
k
=
m
k
+
p
q
ley conmutativa de la adicin.
2.
_
p
q
+
m
k
_
+
r
n
=
p
q
+
_
m
k
+
r
n
_
ley asociativa de la adicin.
3.
p
q

m
k
=
m
k

p
q
ley conmutativa de la multiplicacin.
4.
_
p
q

m
k
_

r
n
=
p
q

_
m
k

r
n
_
ley asociativa de la multiplicacin.
5.
_
p
q
+
m
k
_

r
n
=
p
q

r
n
+
m
k

r
n
ley distributiva de la adicin respecto a la multiplicacin.
6. Si el numerador y el denominador de una fraccin dada se multiplican o se dividen por un
mismo nmero natural k, se obtendr una fraccin igual a la dada
p
q
=
kp
kq
.
7. Si
p
q
es una fraccin irreducible, la fraccin
m
n
ser igual a ella cuando, y slo cuando,
m = kp y n = kq, donde k es un nmero natural.
CAPTULO 3. NMEROS REALES 84
Ejemplo 3.12 Simplique la expresin
__
3
1
2
2
1
3
_

6
5

_
5
1
2

_
3
2
1
2
5
_
+
_
1
2
+
1
3
___
.
Solucin
F =
__
7
2

7
3
_

5
6

_
11
2

_
3
2

7
5
_
+
_
1
2
+
1
3
___
=
__
21 14
6
_

5
6

_
11
2

_
15 14
10
_
+
_
3 + 2
6
___
=
_
7
6

5
6

_
11
2

1
10
+
5
6
__
=
_
7
6

5
6

_
11
20
+
5
6
__
=
_
35
36

_
33 + 50
60
__
=
_
35
36

83
60
_
=
15 35 9 83
540
=
525 747
540
=
37
90
.
A continuacin examinamos aquellas fracciones
p
q
, cuyo denominador q = 10
k
, donde k es un
nmero natural.
Para cada fraccin de este tipo se ha adoptado una forma especial de representacin: se escribe
el numerador de la fraccin y, al contar, por el lado derecho, k cifras, se separan stas con una
coma; si en el numerador hay menos cifras que k, por ejemplo, n cifras (n < k), entonces se escribe
el numerador y delante de la primera cifra de ste se ponen k n ceros, luego se pone la coma y
delante de sta se pone un cero ms; en cambio, si en el numerador hay k cifras, entonces se escribe
el numerador, delante de la primera cifra de ste se marca una coma y se pone un cero delante de
la coma.
Ejemplo 3.13 Las cifras
5671
100
,
93
10000
,
543
1000
pueden ser escritas como: 56,71; 0,0093; 0,543.
Una fraccin escrita de la forma a
0
, a
1
a
2
a
3
...a
k
se denomina fraccin decimal nita, donde k es
un nmero natural, a
0
es un nmero perteneciente a la serie natural ampliada, cualquiera de a
1
,
a
2
, a
3
, ..., a
k
, uno de los nmeros 0, 1, 2, 3, 4, 5, 6, 7, 8, 9.
Toda fraccin decimal nito se transforma fcilmente en una ordinaria. Con este n se debe
escribir en el numerador un nmero entero que se obtiene, si se elimina la coma de la fraccin
decimal, y se escribe en el denominador el nmero 10 a una potencia que sea igual a la cantidad
de cifras que se tienen en la fraccin decimal tras la coma, despus de lo cual la fraccin puede ser
simplicada por un factor comn, si existe. Escribir una fraccin ordinaria en forma de fraccin
decimal nita signica hallar una fraccin decimal nita que sea igual a la dada.
CAPTULO 3. NMEROS REALES 85
Ejemplo 3.14 Simplique la expresin
__
0,5
2,5

1
0,25
_

19
3

_
1
0,5

_
0,25
0,75
0,2
_
+
_
0,6 +
1
0,75
___
.
Solucin
F =
__
1
2
5
2

1
1
4
_

3
19

_
1
1
2

_
1
4
3
4

1
5
_
+
_
3
5
+
1
3
4
___
=
__
1
5
4
_

3
19

_
2
_
1
3

1
5
_
+
_
3
5
+
4
3
___
=
__
1 20
5
_

3
19

_
2
_
5 3
15
_
+
_
9 + 20
15
___
=
_

19
5

3
19

_
2
2
15
+
29
15
__
=
_

3
5

_
4
15
+
29
15
__
=
_

3
5

33
15
_
=
9 + 33
15
=
42
15
.
Ser conveniente llamar fraccin decimal peridica innita, aquella en la cul despus de la
coma viene una innidad de cifras, con la particularidad de que una cifra o un conjunto ordenado
de cifras se repiten a partir de cierto lugar tras la coma.
Se denomina fraccin decimal peridica innita, una fraccin que puede ser escrita en la forma
a
0
,a
1
a
2
a
3
...a
k
..., donde a
0
es un nmero perteneciente a la serie natural ampliada, para cualquier
nmero natural m, en el m-simo lugar tras la coma gura uno de los nmeros 0, 1, 2, 3, 4, 5,
6, 7, 8, 9, con la particularidad de que o bien a
0
es distinto de cero, o bien, si a
0
es igual a cero,
existe al menos un nmero natural q tal, que en el q-simo lugar tras la coma gure uno de los
nmeros 0, 1, 2, 3, 4, 5, 6, 7, 8, 9; existen tales nmeros naturales r y p que para cualquier nmero
natural n r es vlida la igualdad a
n+p
= a
n
, y en este caso el conjunto ordenado de cifras se
llama perodo de la fraccin decimal peridica innita.
Al escribir una fraccin decimal peridica innita, los puntos suspensivos se ponen despus del
perodo que se repiti varias veces, es decir, cuando se hace claro cul nmero es el perodo de esta
fraccin.
Cualquier fraccin ordinaria
p
q
puede ser representada de modo nico en forma de una fraccin
decimal peridica innita y, viceversa, toda fraccin decimal peridica innita puede ser represen-
tada de modo nico en forma de una fraccin ordinaria
p
q
.
Ejemplo 3.15 La fraccin 7,45232323... es una fraccin decimal peridica innita de perodo
23.
CAPTULO 3. NMEROS REALES 86
Con el n de convertir una fraccin decimal peridica innita en una fraccin ordinaria, se debe
sustraer del nmero que precede al segundo perodo otro nmero, que precede al primer perodo,
y representar dicha diferencia como el numerador, poniendo en el denominador la cifra 9 tantas
veces, cuantas cifras hay en el perodo, y escribir despus de los nueves tantos ceros, cuantas cifras
se encuentran entre la coma y el primer perodo.
Ejemplo 3.16 Simplique la expresin
__
0, 333...
0, 666...

1, 0333..
0, 0222...
_

75
2
+
_
177
5
_
0, 12525...
0, 71515...
0, 222...
_
+
_
0, 666... +
1
0, 2525...
___
.
Solucin
F =
__
1
3
2
3

1
1
30
1
45
_

2
75
+
_
177
5

_
1251
990
7157
990

2
9
_
+
_
2
3
+
1
25
99
___
=
__
1
2

93
2
_

2
75
+
_
177
5

_
124
708

2
9
_
+
_
2
3
+
1
99
25
___
=
_

92
2

2
75
+
_
177
5

_
31
177

2
9
_
+
_
2
3
+
1
99
25
___
=
_

92
75
+
_
177
5

_
279 354
1593
_
+
_
50 + 297
75
___
=
_

92
75
+
_
177
5

_

75
1593
_
+
347
75
__
=
_

92
75
+
_

177
5

75
1593
+
347
75
__
=
_

92
75
+
_

15
9
+
347
75
__
=
_

92
75
+
_
375 + 1041
225
__
=
_

92
75
+
666
225
_
=
276 + 666
225
=
390
225
=
26
15
.
Para que no haya dos representaciones diferentes de una misma fraccin decimal nita, se con-
viene en no tener en el perodo el nmero 9. Entonces, cada fraccin decimal nita puede ser escrita
de modo nico en forma de una fraccin decimal peridica innita de perodo 0, y, viceversa, cada
fraccin de esta ndole es una fraccin decimal nita.
De este modo, se puede constatar que cualquier fraccin decimal peridica innita es otra forma
de representacin de cierta fraccin ordinaria bien determinada.
3.7. Tarea
1. Determine el MCD y mcm de los nmeros siguientes:
CAPTULO 3. NMEROS REALES 87
a) 1320, 900, 1260;
b) 17600, 2640, 5390, 1386;
c) 4536, 684, 5184, 1728;
d) 5430, 3560, 2670;
e) 1704, 12780, 2130, 994;
f ) 220320, 7344, 6120;
g) 4530, 3630, 1260;
h) 1010, 6060, 56560, 11110;
i) 2530, 20240, 50600;
j) 48, 280, 720;
k) 90, 315, 225, 405, 450;
l) 9240, 64680, 27720, 46200;
m) 81, 540, 162, 243;
n) 84, 189, 210, 105.
Resp: a) MCD = 2
2
5 3, mcm = 2
3
5
2
3
2
7 11;
b) MCD = 2 11, mcm = 2
6
5
2
11 3
2
7
2
;
c) MCD = 2
2
3
2
, mcm = 2
6
3
4
7 19;
d) MCD = 2 5, mcm = 2
3
3 5 181 89;
e) MCD = 2 71, mcm = 2
3
3
2
5 7 71;
f ) MCD = 2
3
3
2
17, mcm = 2
5
3
4
5 17;
g) MCD = 2 3 5, mcm = 2
2
3
2
5 151 11
2
7;
h) MCD = 2 5 101, mcm = 2
4
3 5 7 11 101;
i) MCD = 2 5 11 23, mcm = 2
4
5
2
11 23;
j) MCD = 2
3
, mcm = 2
4
3
2
5 7;
k) MCD = 3
2
5, mcm = 2 3
4
5
2
7;
l) MCD = 2
3
3 5 7 11, mcm = 2
3
3
2
5
2
7
2
11;
m) MCD = 3
3
, mcm = 2
2
3
5
5;
n) MCD = 3 7, mcm = 2
2
3
3
5 7.
2. Simplique las expresiones:
a)
1
2
__
3
2
1
3
2
_
+ 15
_
1
5

1
3
__
3
1
2
+ 1
__
;
b) 2
1
4
_
1
2

1
4
_
+ 3
1
2
_
3
2

5
4
__
3
4

1
2
_

3
2
;
c)
1
3
_
2
3

1
2
_
+
4
3
_
1
2

1
3
__
2
1
3
+ 1
1
2
_
;
d)
3
4
_
2
3
_
1
2

3
4
_

8
3
_
1
3
+
1
2
__
1
4

1
3
__
2
1
3
;
e)
3
2
_
1
4

1
3
_
+
3
2
_
1
3
_
2
3

1
2
_
+
3
2
_
;
f )
_
2
3

1
2
_
+
1
2
_
2
3
_
4
3

1
2
_

1
2
_
1
2

1
3
__
;
g)
2
3
_
1
5

1
3
_
+
1
14
_
1
3

1
5
__
1
3
+
3
5
_
;
h)
1
4
_
2
3
_
4
3

5
2
_

4
3
_
1
2
+
2
5
__
+
_
1
3

1
2
_
;
i)
_
1
2
+
5
3
_

_
3
2

5
4
__
1
4

3
2
_
1
3
2
;
j)
4
3
_
3
2
_
4
3

3
2
_
+
5
2
_
1
3

1
4
__
+
_
2
1
3

1
2
_
;
k)
1
4
+
_
1
2
_
1
3

1
2
_
+
1
3
_
1
2
_
4
3

3
4
___
;
l)
8
3
_
3
7
_
1
4
+
1
3
__
1
3

1
4
_

4
3
_
1
2

5
3
__
+ 3
1
2
;
CAPTULO 3. NMEROS REALES 88
m)
_
1
4

1
2
__
1
2
+
1
3
_

_
2
3

1
4
__
4
3
+
3
2
_
;
n)
3
2
_
1
2
_
4
3
_
1
3

1
2
_

1
5
_
2
3

1
4
__
4
3

1
2
__
+
3
2
_
;
o)
_
3
2

1
4
__
1
2
_
3
4
+
3
2
_
2
1
2
_
+
_
4
3

1
2
_
;
p)
2
3
__
5
2
+ 1
_

3
2
_
2
3
_
1
2
+
1
3
_
+
4
3
_
2
3

5
2
___
;
q)
5
4
_
1
4

3
8
__
8
3
+ 1
_

5
3
_
5
7
_
2
3

1
5
_

1
3
_
;
r)
6
5
_
5
2
_
1
3

5
2
_

5
3
_
3
2
_
5
3

9
2
_
+
5
2
_
1
5

2
3
___
;
s)
15
4
_
4
15
_
3
16

1
4
_

2
5
_
5
3
_
4
3

7
2
4
1
2
___
;
t)
_
1
4
_
1
2

_
3
2

5
4
__
+
3
2
_
5
2

7
4
__

_
9
2
+
1
4
_
;
u)
2
5
_
5
4
_
3
8

3
4
_
+
3
4
_
5
8
+
7
4
__

15
2
_
7
2

5
4
_
;
v)
5
2
_
14
5
_
1
3

1
4
_

2
5
_
1
3
+
1
2
__

4
13
_
4
5
+
1
2
_
;
w)
15
8
__
1
5
+
1
3
_

16
5
_
2
5

2
3
__
+
_
1
15
+ 1
_
;
x)
3
2

15
2
_
4
5

2
3
__
1
3

1
5
_

1
2
_
4
3
_
1
2
+
3
4
_

4
3
_
;
y)
8
3
_
5
2

5
3
__
1
2

1
3
_

3
2
_
4
3
1
__
1
12
+
1
3
_
;
z)
3
8
_
8
5
_
5
2

4
3
_
+ 5
1
2
_

1
4
_
4
_
1
5

1
3
_
+ 1
1
3
_
.
Resp: a)
4
5
; b)
23
32
; c)
49
54
; d)
167
72
; e)
53
24
; f )
29
72
; g)
2
25
;
h)
119
180
; i)
1
48
; j)
16
9
; k)
19
72
; l)
208
27
; m)
25
18
; n)
65
32
; o)
85
96
;
p)
38
9
; q)
55
96
; r)
13
3
; s)
581
24
; t)
57
16
; u)
327
20
; v)
13
20
; w)
11
3
;
x)
6
5
; y)
35
216
; z)
41
16
.
3. Simplique la siguiente expresin:
a)
0, 777...
1 0, 111...

2 + 0, 333...
0, 0777...
+
_
0, 111...
6, 25
;
b)
1,333 0,066... 0,303030...
0,333...
+
2
5

4
11
;
c)
_
3
5
0, 05454...
_
1
0, 4 0, 1 5 +
2
9
;
d)
5
6
(0, 5 1)
2

1
100
(0, 1)
3
+ 0, 666...
3
_

0, 25
2
.
4. Tres personas desean repartir 180 libros, 240 juguetes y 360 chocolatines, respectivamente,
entre un cierto nmero de nios, de al modo que cada uno reciba un nmero exacto de libros,
CAPTULO 3. NMEROS REALES 89
de juguetes y de chocolatines. Cul es el mayor nmero de nios que puede beneciarse en
esa forma?
5. Se desean acondicionar 1830 latas de aceite y 1170 latas de hierba en un cierto nmero de
cajones que contengan el mismo nmero de latas, sin que sobre ninguna y sin mezclar las
latas. Cul ser el mayor nmero posible de latas que puedan ponerse en cada cajn?
6. Un jardinero desea colocar 720 plantas de violetas, 240 de pensamientos, 360 de jacintos
y 480 de miositis en el menor nmero posible de canteros que contengan el mismo nmero
de plantas, sin mezclar las mismas. Qu cantidad de plantas debe contener cada cantero?
Cuntos canteros hay?
7. Cul es el menor nmero posible que dividido por 132, 450 y 342 da en cada caso un resto
de 5?
8. Se tienen 160 cl y 168 cl de extractos distintos. Se quieren envasar en el menor nmero
posible de frascos iguales sin mezclar los extractos. Cul es el nmero de frascos de cada
clase?
9. Se tienen tres cubos de 84 cm
3
, 270 cm
3
y 330 cm
3
. Cul es el mayor volumen en cm
3
que cabe un nmero exacto de veces en cada uno de ellos?
10. Cuatro buques parten para el mismo destino: el primero, cada 10 das; el segundo, cada
8; el tercero, cada 9, y el cuarto, cada 15. Cuntos das transcurren entre dos salidas si-
multneas consecutivas?
11. Dos ruedas dentadas se engranan una sobre la otra; la primera tiene 48 dientes y tarda
4 segundos en cada vuelta; la segunda tiene 104 dientes. Se las pone en movimiento y se
pregunta al cabo de cunto tiempo se encontrarn en la misma posicin que al comenzar.
12. Dos letreros luminosos se encienden con intermitencias de 42 segundos y 54 segundos, re-
spectivamente. A las 20 horas 15 minutos se encienden simultneamente. A qu hora vuelven
a encenderse juntos?
13. Se quiere alambrar un terreno de forma trapezoidal tal que sus lados son, respectivamente,
de: 320 m, 104 m, 396 m y 84 m, deseando que los postes resulten equidistantes y que en
cada esquina haya un poste. Cul es la mxima distancia a que pueden colocarse y, en tal
caso, cuntos postes se necesitan?
14. Simplique las expresiones:
a) 4
2
5
3
2
5
+
_
1
4
2
1
4
+
_
2
3
1
3
2

5
2
_

1
2
_
;
CAPTULO 3. NMEROS REALES 90
b)
3
2

_
2
3

_
3
1
3
+ 1
2
3
__

_
25
5
3
+
_
2
1
2
_
2
_
;
c) 1
2
3

__
1
3
+ 1
3
2
3
1
3
_

_
1
6

2
3
+ 1
_
4
1
2
_
;
d)
__
4
5

2
5
_

1
3
+
5
6
3
1
2

5
2
__

3
2
5
2
3
5
5
2
5
_

5
3
2
1
2
3
;
e)
14
3
+ 2
_
1
3
+ 5
13
3
_

_
7
2
4
1
2
_
3
2
5
2
1
5
__
+ 5
3
2
;
f )
25
6
_
3
2

_
3
2
3

5
2
_
6
5
_
+ 4
1
2
3
3
2
_
1
2
+
2
3
_
;
g) 4
4
5

12
5
_
2
1
2
1
5
2
_

_
1
3
_
3
2
+ 3
3
2
_
5
1
3

2
3
_
;
h)
_
349
63
_
50
9

35
4

_
8
11

147
16

7
5
__
+
97
19
_

63
5
+
5
4
;
i)
41
4

_

_
_
3
10

3
20
_

2
3
_
47
25
+
53
25
_
80

3
2

3
100
_
61
20

53
20
_
4 +
2
5
_

_
;
j)
33
25

_

_
117
100

13
10
+
42
5

6
7

_

_
6
_
23
10
+ 5
25
4
_
7
8
1
80
+
69
10
_

_
_

_
;
k)
1
10
_
4207
30

1661
12
_

109
6

1
5
_
1499
18

2557
30
_

8
3
.
Resp: a)
157
102
; b)
1123
52
; c)
43
6
; d)
44
5
; e)
162
5
; f )
157
36
; g) 4;
h)
373
28
; i)
1
2
; j)
2
33
; k)
14
11
.
3.8. Nmeros reales
El conjunto de todas las fracciones decimales innitas, se denomina conjunto de nmeros reales
y se nota con la letra R, mientras que toda fraccin decimal innita lleva el nombre de nmero
real. La fraccin decimal innita positiva se llamar nmero real positivo y la fraccin decimal
innita negativa, nmero real negativo; la fraccin decimal peridica innita nula, de perodo cero,
nmero cero.
Por cuanto las fracciones decimales innitas pueden ser tanto peridicas como aperidicas, todo
nmero real es o bien racional o bien irracional.
Denicin 3.2 Nmeros reales iguales
Dos nmeros reales positivos son iguales, si b
k
= a
k
para todos los nmeros k pertenecientes a una
serie natural ampliada.
De dos nmeros reales positivos, el primero es mayor que el segundo, siempre que o bien a
k
= b
k
,
pero a
k+1
> b
k+1
.
CAPTULO 3. NMEROS REALES 91
Denicin 3.3 Nmeros reales opuestos
Dos nmeros reales se llaman opuestos, si b
k
= a
k
para todos los nmeros k pertenecientes a
una serie natural ampliada. Dos nmeros reales negativos son iguales, si son iguales los nmeros
opuestos de ellos.
De dos nmeros negativos el mayor es aquel cuyo nmero opuesto es menor. Un nmero posi-
tivo es mayor que el cero y que cualquier nmero negativo. El nmero cero es mayor que cualquier
nmero negativo.
La suma de un nmero real con el nmero opuesto, es el nmero cero.
El producto de dos nmeros reales negativos es igual al producto de los nmeros positivos, op-
uestos de los primeros. El producto de dos nmeros reales de signos diferentes, es igual al nmero
negativo opuesto del producto de los nmeros. El producto de dos nmeros, uno de los cuales es
cero, es igual a cero.
A continuacin se dan las leyes principales de adicin y multiplicacin de los nmeros reales:
1. a +b = b +a ley conmutativa de la adicin.
2. (a +b) +c = a + (b +c) ley asociativa de la adicin.
3. ab = ba ley conmutativa de la multiplicacin.
4. (ab)c = a(bc) ley asociativa de la multiplicacin.
5. (a +b)c = ac +bc ley distributiva de la adicin respecto a la multiplicacin.
Para las operaciones de adicin y multiplicacin de los nmeros reales se introducen operaciones
inversas; las de sustraccin y divisin, las cuales denimos a continuacin.
Denicin 3.4 Sustraccin de nmeros reales
Sustraer de un nmero real a otro nmero real b, signica hallar un nmero real c tal que c = ab.
Denicin 3.5 Divisin de nmeros reales
Dividir un nmero real a por otro nmero real b, distinto de cero, signica hallar un nmero real
c tal que c =
a
b
.
En el conjunto de nmeros reales las operaciones de sustraccin y divisin son siempre realiz-
ables, a excepcin de la divisin por cero.
Denicin 3.6 Potencia n-sima de nmeros reales
Si un nmero real a gura en calidad de factor n veces, n es un nmero natural, n > 1, entonces
el producto a a ... a
. .
n veces
recibe el nombre de n-sima potencia del nmero a y se nota a
n
. Adems,
por denicin a
1
= a.
Las propiedades de la potencia de los nmeros reales son anlogas a las de la potencia de los
nmeros racionales.
En relacin con el concepto de potencia de los nmeros reales surge con frecuencia un problema
en el que se pide hallar, para el nmero natural dado n y para el nmero real dado no negativo a,
otro nmero real no negativo b tal, que tenga lugar la igualdad b
n
= a.
CAPTULO 3. NMEROS REALES 92
Denicin 3.7 Raz aritmtica de n-simo grado
Un nmero no negativo b tal, que la n-sima potencia suya es el nmero dado a, es decir, b
n
= a,
se denomina raz aritmtica de n-simo grado del nmero no negativo a y se representa por b =
n

a.
Teorema 3.1 Para cualquier nmero natural n y para todo nmero no negativo a existe una
raz aritmtica de n-simo grado del nmero a y esta raz es la nica en el conjunto de nmeros
no negativos.
Las races aritmticas pueden ser nmeros racionales o irracionales. Sealemos que, por deni-
cin, la raz aritmtica del nmero 0 es cero.
Denicin 3.8 Magnitud absoluta
Se denomina magnitud absoluta de un nmero natural a; este mismo nmero, si a es positivo; cero,
si a es cero; el nmero opuesto de a, si a es negativo. La magnitud absoluta de un nmero real a
se designa [a[, notada
[a[ =
_

_
a, si a > 0
0, si a = 0
a, si a < 0
La magnitud absoluta de un nmero tiene las siguientes propiedades:
1) La magnitud absoluta de una suma algebraica es menor o igual a la suma de las magnitudes
absolutas de los sumandos, es decir
[a +b[ [a[ +[b[
2) La magnitud absoluta de la diferencia de dos nmeros es mayor o igual a la diferencia de las
magnitudes absolutas del minuendo y el sustraendo, es decir
[a b[ [a[ [b[
3) La magnitud absoluta del producto de un nmero nito de factores es igual al producto de
sus magnitudes absolutas, es decir
[a
1
a
2
...a
n
[ = [a
1
[[a
2
[...[a
n
[
4) La magnitud absoluta del cociente es igual al cociente de las magnitudes absolutas del divi-
dendo y el divisor, es decir

a
b

=
[a[
[b[
Observemos que en virtud de la denicin de raz aritmtica de un nmero no negativo es
vlida, para cualquier nmero real, la igualdad

a
2
= [a[.
Examinemos la cuestin referente a la existencia de la raz algebraica de un nmero real.
Sea a = 0, entonces para todo nmero natural n existe una, y slo una raz algebraica de
n-simo grado, que es el nmero b, igual a cero. Suponga que a es un nmero positivo y n, un
nmero natural impar, n = 2k+1. En este caso existe una, y slo una, raz aritmtica, b
1
=
2k+1

a,
de este nmero y no hay otras races algebraicas reales de l. De este modo, existe una sola raz
CAPTULO 3. NMEROS REALES 93
algebraica de grado impar de un nmero positivo, la raz aritmtica.
Suponga que a es un nmero positivo y n, un nmero natural par, n = 2k. En este caso existe
y, adems, una sola raz aritmtica, b
1
=
2k

a, y una raz algebraica real, b


2
=
2k

a, del nmero
citado. De este modo, existen dos races algebraicas reales de grado par del nmero positivo a:
b
1
=
2k

a y b
2
=
2k

a.
Sean ahora, a un nmero negativo y n, un nmero natural par, n = 2k. Por cuanto cualquier
nmero real distinto de cero a la potencia par es un nmero positivo, y el nmero 0 a toda potencia
natural es cero, no existe ningn nmero real b tal, que b
2k
sea un nmero negativo. Esto quiere
decir que no existe raz algebraica real de grado par de un nmero negativo.
Suponga ahora que a es un nmero negativo y n, un nmero natural impar, n = 2k + 1. En-
tonces existe un nmero real negativo b tal, que b
n
= a. Es decir,
2k+1
_
[a[ es la raz algebraica
real del nmero negativo a.
Pasemos ahora a la interpretacin geomtrica de los nmeros reales. Sea dada una recta hor-
izontal. Esta tiene dos direcciones mutuamente opuestas. Llamaremos positiva la direccin a la
derecha y negativa, la direccin a la izquierda. Fijemos en la recta cierto punto 0 y llammoslo
punto de referencia. El punto 0 divide la recta en dos partes denominadas rayos. El rayo dirigido
a la derecha se llamar positivo y el rayo dirigido a la izquierda, negativo. Sea dado un segmento,
tomado por unidad de longitud; en estos casos se dice que se ha introducido una escala.
Denicin 3.9 Recta numrica
Se denomina recta numrica aquella en la que se han elegido el punto de referencia, la direccin
positiva y se ha introducido la escala.
A todo punto de la recta numrica se le puede poner en correspondencia un nmero real,
rigindose por la siguiente regla:
1. Al punto elegido 0 le pondremos en correspondencia el nmero cero.
2. A todo punto N en el rayo positivo le pondremos en correspondencia el nmero positivo
a, donde a es la longitud del segmento 0N.
3. A todo punto M en el rayo negativo le pondremos en correspondencia el nmero negativo
b, donde [b[ es la longitud del segmento 0M.
De este modo, a cualquier punto de la recta numrica, con la escala elegida, se le ha puesto en
correspondencia un nico nmero real.
3.9. Igualdades y desigualdades numricas
Todas las deniciones mencionadas anteriormente se pueden escribir de otra forma, haciendo
uso de la comparacin de los nmeros reales con el nmero cero: dos nmeros reales, a y b, son
iguales, si, y slo si, la diferencia entre ellos es nula, es decir,
a = b a b = 0
el nmero a es mayor que el nmero b, si, y slo si, la diferencia a b es positiva, es decir,
a > b a b > 0
CAPTULO 3. NMEROS REALES 94
el nmero a es inferior al nmero b, si, y slo si, la diferencia b a es positiva, o si la diferencia
a b es negativa, es decir,
a < b a b < 0 b a > 0.
Si dos nmeros estn unidos entre s mediante el signo de igualdad se dice que se ha dado una
igualdad numrica. Sin embargo, dicha igualdad puede ser cierta y puede ser incierta. Anloga-
mente, si dos nmeros estn unidos mediante cualquier signo de desigualdad, suele decirse que
viene dada una desigualdad numrica, la cual puede ser cierta o incierta.
A continuacin se dan las propiedades principales de las igualdades y desigualdades:
Teorema 3.2 Si los nmeros a, b y c son tales, que a = b y b = c, entonces a = c.
Teorema 3.3 Si los nmeros a, b, c y d son tales, que a = b y c = d, entonces a +c = b +d.
Teorema 3.4 Si los nmeros a, b, c y d son tales, que a = b, c = d, entonces ac = bd.
Teorema 3.5 Para cualesquiera nmeros reales a, b y c las igualdades a = b y a + c = b + c
son equivalentes, es decir, la validez de la igualdad a = b predetermina la validez de la igualdad
a +c = b +c, y, viceversa, de la validez de la igualdad a +c = b +c sigue la validez de la igualdad
a = b.
Teorema 3.6 Para cualesquiera nmeros reales a y b para todo nmero real c, distinto de cero,
las igualdades a = b y ac = bc son equivalentes.
Para las desigualdades numricas, tenemos las siguientes propiedades:
Teorema 3.7 Si los nmeros a, b y c son tales, que a > b y b > c, entonces a > c.
Demostracin
Como a c = (a c) +(b d) = (a b) +(b c). Por cuanto a > b, tenemos a b > 0; por cuanto
b > c, tenemos b c > 0, pero la suma de dos nmeros positivos es positiva, por lo cual a c > 0,
es decir a > c.
Teorema 3.8 Si los nmeros a, b, c y d son tales, que a > b, c > d, entonces a +c > b +d.
Demostracin
Como (a + c) (b + d) = (a b) + (c d). Por cuanto a > b, el nmero a b es positivo; por
cuanto c > d, entonces c d es tambin un nmero positivo, la suma de dos nmeros positivos es
positiva, por lo cual (a +c) (b +d) > 0, es decir, a +c > b +d.
Este teorema, asegura que si dos o mas desigualdades del mismo sentido se pueden sumar
miembro a miembro; como resultado se obtendr una desigualdad del mismo sentido.
Teorema 3.9 Si los nmeros a, b, c y d son tales, que a > b y c < d, entonces a c > b d.
Demostracin
Como (a c) (b d) = (a b) + (d c). Por cuanto a > b, el nmero a b es positivo; por
cuanto c < d, entonces d c es tambin un nmero positivo; la suma de dos nmeros positivos es
positiva, por lo cual (a c) (b d) > 0, es decir, a c > b d.
Este teorema, asegura que las desigualdades de sentido contrario se pueden restar miembro
a miembro; como resultado obtendremos una desigualdad del mismo sentido que la desigualdad
minuendo.
CAPTULO 3. NMEROS REALES 95
Teorema 3.10 Si a, b, c y d son nmeros positivos y, adems, a > b y c > d, entonces ac > bd.
Demostracin
Como ac bd = (ac bd) + (bc bc) = (ac bc) + (bc bd) = c(a b) + b(c d). Puesto
que a > b, entonces a b es un nmero positivo; por cuanto c es un nmero positivo y como
el producto de nmeros positivos es positivo, entonces c(a b) es un nmero positivo; de modo
anlogo se demuestra que b(cd) es tambin un nmero positivo; la suma de dos nmeros positivos
es positiva, por lo cual ac bd > 0, es decir, ac > bd.
Este teorema, asegura que si dos o mas desigualdades del mismo sentido se pueden multiplicar
entre si miembro a miembro si todos sus miembros son positivos; como resultado se obtiene una
desigualdad del mismo sentido.
Teorema 3.11 Para cualesquiera nmeros reales a, b y c, las desigualdades a > b y a + c >
b + c son equivalentes, es decir, la validez de la desigualdad a > b predetermina que es vlida la
desigualdad a + c > b + c, y, viceversa, de la validez de la desigualdad a + c > b + c se desprende
la validez de la desigualdad a > b.
Demostracin
Sea a > b. Entonces (a +c) (b +c) = (a b) + (c c) = (a b) > 0, es decir, a +c > b +c. Sea
(a +c) > (b +c). Entonces a b = (a b) + (c c) = (a +c) (b +c) > 0, es decir, a > b.
Teorema 3.12 Para cualesquiera nmeros reales a y b y para todo nmero positivo c, las de-
sigualdades a > b y ac > bc son equivalentes, es decir, si c > 0, entonces a > b si ac > bc.
Demostracin
Sea a > b, entonces ac bc = (a b)c. Por cuanto c y (a b) son nmeros positivos, el produc-
to de ellos es un nmero positivo, es decir, ac bc > 0, o bien ac > bc. Sea ac > bc, entonces
(ab)c = acbc > 0. Si el producto de dos nmeros es positivo y uno de ellos es tambin positivo,
entonces ser positivo el otro nmero, es decir, por cuanto c > 0, tenemos ab > 0, es decir, a > b.
Teorema 3.13 Para cualesquiera nmeros reales a y b y para todo nmero negativo c, las de-
sigualdades a > c y ac > bc son equivalentes, es decir, si c < 0, entonces a > b si ac < bc.
Hasta ahora se han usado los signos de igualdad (=) y de la desigualdad rigurosa (< o bien
>). A veces estos signos son insucientes. Hay problemas, donde se necesitan desigualdades no
rigurosas.
Denicin 3.10 Desigualdad cierta e incierta
Una desigualdad numrica a b se considera cierta para a < b y para a = b, y es incierta slo en
el caso en que a > b.
Una desigualdad numrica a b se considera cierta tanto para a > b como para a = b; se considera
incierta slo en el caso cuando a < b.
Para las desigualdades no rigurosas son vlidas las propiedades anteriores, si sustituimos en
ellos el signo de desigualdad rigurosa por el signo de desigualdad no rigurosa.
Denicin 3.11 Desigualdad doble
Diremos que se verica la desigualdad doble a < b < c, siempre que sean vlidas a la vez dos
desigualdades a < b y b < c; se verica la desigualdad doble a b < c, si son vlidas a la vez dos
desigualdades; a b y b < c; se verica la desigualdad doble a < b c, cuando son vlidas a la
vez dos desigualdades: a < b y b c; se verica la desigualdad a b c, siempre que sean vlidas
a la vez dos desigualdades: a b y b c.
CAPTULO 3. NMEROS REALES 96
Ejemplo 3.17 Demostrar que si a > 0, b > 0, c > 0, entonces
bc
a
+
ac
b
+
ab
c
a +b +c.
Solucin
Utilicemos las siguientes desigualdades
1
2
_
bc
a
+
ac
b
_
c,
1
2
_
ac
b
+
ab
c
_
a,
1
2
_
bc
a
+
ab
c
_
b
(estas desigualdades son vlidas, porque a la izquierda de cada una de stas se encuentran las
medias aritmticas y a la derecha, las medias geomtricas de los nmeros positivos). Sumando
estas desigualdades, trmino a trmino, obtenemos la desigualdad que fue necesario demostrar.
Ejemplo 3.18 Demostrar la desigualdad
a
3
+b
3
2

_
a +b
2
_
3
donde a > 0, b > 0.
Solucin
Sustituyamos esta desigualdad por una equivalente:
a
3
+b
3
2

_
a +b
2
_
3
0.
Sacndola del parntesis y reagrupndola puede escribirse en una forma equivalente:
3
8
(a +b)(a b)
2
0.
Ya que a > 0 y b > 0, esta desigualdad es evidente con lo que queda demostrada la validez de la
desigualdad inicial equivalente a la primera.
Ejemplo 3.19 Demuestre la desigualdad
(a
m
+b
m
)
1
m
(a
n
+b
n
)
1
n
para a 0, b 0, m > n > 0.
Solucin
Si a = 0 b = 0, entonces la armacin a demostrar es evidente. Ahora bien, sean a > 0 y b > 0.
est claro que uno de estos nmeros no supera al otro. Por ejemplo, sea 0 < a b. En este caso
0 <
a
b
1, y ya que m > n, entonces
0 <
_
a
b
_
m

_
a
b
_
n
y 1 +
_
a
b
_
m
1 +
_
a
b
_
n
.
De la ltima desigualdad se deduce que
_
1 +
_
a
b
_
m
_
1
n

_
1 +
_
a
b
_
n
_
1
n
.
Luego, ya que
1 +
_
a
b
_
m
1 y 0 <
1
m
<
1
n
CAPTULO 3. NMEROS REALES 97
obtenemos que
_
1 +
_
a
b
_
m
_
1
m

_
1 +
_
a
b
_
n
_
1
n
.
Ahora se puede escribir que
_
1 +
_
a
b
_
m
_
1
m

_
1 +
_
a
b
_
m
_
1
n

_
1 +
_
a
b
_
n
_
1
n
de donde se deduce que
_
a
m
+b
m
b
m
_ 1
m

_
a
n
+b
n
b
n
_1
n
.
Ya que b > 0, de la ltima desigualdad se deriva la desigualdad que fue propuesta para la de-
mostracin.
Ejemplo 3.20 Demuestre que para cualquier nmero entero positivo n es vlida la desigualdad
1
9
+
1
25
+... +
1
(2n + 1)
2
<
1
4
.
Solucin
Al deducir que
2
(2k + 1)
2
<
1
2k

1
2k + 2
sustituimos la suma del primer miembro de la desigualdad a demostrar por una expresin mayor
1
3
2
+
1
5
2
+... +
1
(2n + 1)
2
<
1
2
__
1
2

1
4
_
+
_
1
4

1
6
_
+... +
_
1
2n

1
2n + 2
__
.
No obstante, esta ltima expresin es igual a
1
2
_
1
2

1
2n + 2
_
=
1
4

1
4n + 4
y es, evidentemente, menor que
1
4
. Por consiguiente, la suma
1
9
+
1
25
+... +
1
(2n + 1)
2
ser mucho menos de
1
4
.
Ejemplo 3.21 Demuestre que para cualquier nmero natural n > 1 es vlida la desigualdad
1 +
1

2
+
1

3
+... +
1

n
> 2
_
n + 1 1
_
.
Solucin
Para la demostracin vamos a reducir cada sumando de la suma del primer miembro:
1

k
>
2

k +

k + 1
= 2
_
n + 1 1
_
.
Por eso, el primer miembro de la desigualdad a demostrar puede ser reducido:
1 +
1

2
+
1

3
+... +
1

n
> 2(

1) + 2(

2)+
... + 2
_
n

n 1
_
+ 2
_
n + 1

n
_
.
Ya que el segundo miembro de la ltima desigualdad es precisamente igual a 2

n + 12, entonces
la desigualdad a demostrar es justa.
CAPTULO 3. NMEROS REALES 98
Ejemplo 3.22 Demuestre que si n es un nmero entero mayor que 1, entonces
n! <
_
n + 1
2
_
n
.
Solucin
Lo justo de esta desigualdad va a deducirse de la validez de la desigualdad siguiente, que es igual
a la primera:
(n!)
2
<
_
n + 1
2
_
2n
.
Multipliquemos el nmero n! = 1 2...k...(n1) n por el nmero n! = n (n1)...(nk +1)..,2 1
disponindolos uno debajo del otro:
1 2...k...(n 1) n
n (n 1)...(n k + 1)..,2 1
Al multiplicar los nmeros de cada columna, obtendremos que
(1 n) [2(n 1)]...[k(n k + 1)]...[(n 1) 2] (n 1).
Para obtener (n!)
2
es necesario multiplicar los miembros de este rengln. Aplicando a cada miembro
de este rengln una desigualdad anteriormente estudiada, obtenemos
_
k(n k + 1)
k +n k + 1
2
=
n + 1
2
, k = 1, 2, ..., n
Con la cual el signo de desigualdad se logra aqu slo cuando k = n = k+1, es decir, para k =
n+1
2
.
En otras palabras, solamente para los n impares, y slo para un miembro de nuestro rengln de
esta desigualdad es posible el signo de igualdad. Por esto, para todos los parntesis y corchetes,
con excepcin de uno, son vlidas las desigualdades
[k(n k + 1)] <
_
n + 1
2
_
2
.
Puesto que en el rengln hay n miembros, obtenemos que
(n!)
2
<
__
n + 1
2
__
n
.
Ejemplo 3.23 Demuestre que para cualquier nmero real 1 y cualquier nmero entero
positivo n es vlida la desigualdad
(1 +)
n
1 +n.
Solucin
La desigualdad para n = 1 es, evidentemente, vlida. Supongamos que es vlida la desigualdad
(1 + )
k
1 + k y demostremos que lo es tambin la desigualdad (1 + )
k+1
1 + (k + 1).
Efectivamente
(1 +)
k+1
= (1 +)
k
(1 +) (1 +k)(1 +) = 1 + (k + 1) +k
2
1 + (k + 1).
De tal modo, la desigualdad inicial es justa.
CAPTULO 3. NMEROS REALES 99
Ejemplo 3.24 Demuestre que si el producto n 2 de los nmeros positivos es igual a 1, su
suma es mayor o igual a n, es decir, si x
1
x
2
...x
n
= 1, x
1
> 0, x
2
> 0, ..., x
n
> 0, entonces
x
1
+x
2
+... +x
n
n.
Solucin
Tomemos la hiptesis de la induccin y cualesquier nmeros positivos x
1
, ..., x
k
, x
k+1
, que satisfacen
la condicin x
1
...x
k1
x
k
x
k+1
= 1. Si cada uno de estos nmeros es igual a 1, entonces la suma
x
1
+... +x
k
+x
k+1
= k + 1, por razn de que la desigualdad demostrada es vlida.
Si resulta que esto no es as, entonces entre ellos se encontrar un nmero mayor que 1 y un nmero
manor que 1. Supongamos que x
k
> 1 y x
k+1
< 1. Entonces tenemos la igualdad
x
1
...x
k1
(x
k
x
k+1
) 1.
esto es el producto de k nmeros, a causa de que es aplicable la hiptesis de la induccin y, podemos
armar que
x
1
+... +x
k1
+x
k
x
k+1
k.
Pues, entonces
x
1
+... +x
k1
+x
k
+x
k+1
k x
k
x
k+1
+x
k
+x
k+1
= k + 1 + (x
k
1)(1 x
k+1
)
> k + 1
porque x
k
1 > 0 y 1 x
k+1
> 0, lo que era necesario demostrar.
Ejemplo 3.25 Demuestre que si x
2
+y
2
= 1, entonces

2 x +y

2.
Solucin
Resolucin algebraica. Anotemos una desigualdad evidente: (x y)
2
0, o bien, x
2
+ y
2
2xy.
De aqu se deduce:
2(x
2
+y
2
) x
2
+ 2xy +y
2
.
Por cuanto x
2
+ y
2
= 1, entonces de la ltima desigualdad tenemos (x + y)2 2, de donde
[x +y[

2, es decir,

2 x +y

2.
Ejemplo 3.26 Sea a +b = 2, donde a y b son los nmeros reales. Demuestre que a
4
+b
4
2.
Solucin
Notemos que si uno de los nmeros a y b es negativo entonces la desigualdad es casi evidente. Por
ejemplo, sea b < 0. En este caso a > 2 y la desigualdad a
4
+ b
4
2 es correcta, porque b
4
> 0 y
a
4
> 16. Por lo tanto, consideremos en lo posterior que a 0 y b 0.
Primera resolucin: Ya que a + b = 2, entonces (a + b)
2
= 4. Valindose de la desigualdad entre
la media aritmtica y la media geomtrica ab
a
2
+b
2
2
, tenemos 4 = (a + b)
2
= a
2
+ b
2
+ 2ab
2(a
2
+b
2
), es decir, 2 a
2
+b
2
. Al elevar al cuadrado esta desigualdad (cosa que es justa porque
en ambos miembros se hallan nmeros positivos) obtendremos 4 (a
2
+b
2
)
2
.
Sobre la base de la desigualdad entre las medias aritmticas y geomtrica a
2
b
2

a
4
+b
4
2
. Por eso
tenemos
4 (a
2
+b
2
)
2
= a
4
+b
4
+ 2a
2
b
2
2(a
4
+b
4
)
de donde 2 a
4
+b
4
, lo que era necesario demostrar.
Segunda resolucin: Consideremos de nuevo que a 0 y b 0. Ya que a + b = 2, entonces
(a +b)
4
= 16, o bien
(a +b)
4
= (a
2
+ 2ab +b
2
)(a
2
+ 2ab +b
2
) = a
4
+b
4
+ 4ab(a
2
+b
2
) + 6a
2
b
2
= 16.
CAPTULO 3. NMEROS REALES 100
Puesto que a
2
+b
2
= 4 2ab, puede expresarse la ltima desigualdad as:
a
4
+b
4
= 16 16ab + 2a
2
b
2
.
Si podemos demostrar que 1616ab+2a
2
b
2
2, entonces nuestra desigualdad quedar demostrada.
En las condiciones del problema tenemos ab 1. Efectivamente,

ab
a +b
2
. Ya que a + b = 2,
entonces

ab 1, de donde ab 1.
De esa manera, nos hace falta demostrar la desigualdad 16 16ab + 2a
2
b
2
2 a condicin de que
ab 1.
Designemos x = ab. En este caso es necesario demostrar la desigualdad x
2
8x = 7 0 a condicin
de que x 1. Las races del trinomio de segundo grado x
2
8x + 7 son: x
1
= 1, x
2
= 7. Por lo
tanto, la ltima desigualdad puede escribirse como: (x 1)(x 7) 0.
Pero, para x 1 esta desigualdad es evidente. Por esto hemos obtenido 16 16ab + 2a
2
b
2
2, lo
que era necesario demostrar.
Tercera resolucin: Sean a = 1+c, b = 1c. Ya que hemos supuesto ms arriba que a 0 y b 0,
se deduce que 1 c 1. Por esto podemos deducir que
(1 +c)
4
1 + 4c, (1 c)
4
1 4c.
De tal modo
a
4
+b
4
= (1 +c)
4
+ (1 c)
4
(1 + 4c) + (1 4c) = 2.
En conclusin, sealemos que tiene lugar una armacin ms general: si a + b = 2, entonces
a
n
+b
n
2 para cualquier nmero entero positivo n.
3.10. Tarea
1. Demuestre las desigualdades:
a) (a
2
+b
2
)(a
4
+b
4
) (a
3
+b
3
)
3
;
b) (a +b +c)
2
3(a +b +c)abc;
c) (a +b +c)(ab +bc +ac) 9abc;
d) (ab +bc +ac)
2
3(a +b +c)abc;
e) (a +b +c)
2
3(a
2
+b
2
+c
2
);
f )
bc
a
+
ac
b
+
ab
c
a +b +c;
g)
a
3
+b
3
+c
3
a
2
+b
2
+c
2

a +b +c
3
;
h)
1
a
+
1
b
+
1
c

1

bc
+
1

ca
+
1

ab
;
i) a
4
+b
4
a
3
b +ab
3
;
j) a
2
+b
2
2[ab[.
2. Demuestre que para cualesquiera nmeros reales a, b, c, d es vlida la desigualdad:
a) a
4
+b
4
+c
4
+d
4
4abcd; b) a
2
+b
2
+c
2
ab +ac +bc.
3. Demuestre que la media aritmtica de dos nmeros positivos no es menor que su media
geomtrica, es decir
a +b
2

ab.
4. Demostrar que si a +b +c = 1, donde a > 0, b > 0, c > 0, entonces
(1 a)(1 b)(1 c) 8abc.
CAPTULO 3. NMEROS REALES 101
5. Demostrar que para cualesquiera nmeros reales a, b y c
a
2
+b
2
+c
2
ab +bc +ac.
6. Demuestre que si a, b, c son nmeros positivos y no iguales entre s, entonces
a) (a +b +c)(a
1
+b
1
+c
1
) > 9; b) (a +b +c)(a
2
+b
2
+c
2
) > 9abc.
7. Demostrar que
a
2
+b
2
+c
2
+ 3 2(a +b +c).
8. Demuestre que para cualquier x e y reales se satisface la desigualdad
x
2
+ 2xy + 3y
2
+ 2x + 6y + 3 0.
9. Demuestre que para cualquier x e y reales se satisface la desigualdad
x
2
+ 2xy + 3y
2
+ 2x + 6y + 3 0.
10. Demuestre que el polinomio x
8
x
5
+x
2
x+1 es positivo para todos los valores reales de x.
11. Demuestre que si a +b = c, a > 0, b > 0, entonces
a
2
3
+b
2
3
> c
2
3
.
12. Sea n un nmero positivo. Demuestre la desigualdad
_
1 +
1
n
_
n
<
_
1 +
1
2n
_
2n
.
13. Demuestre que para cualquier nmero entero positivo n es vlida la desigualdad
1
n + 1
+
1
n + 2
+... +
1
3n + 2
> 1.
14. Demuestre que si n > 2 es un nmero entero positivo, entonces
a) (n!)
2
> n
n
; b) n! > 2
n1
.
15. Demuestre que es vlida la desigualdad
(a +b)
n
< 2
n
(a
n
+b
n
)
Para cualesquier a y b positivas y cualquier nmero entero positivo n.
16. Demuestre las desigualdades:
a)
_
a 1 +
1
b
__
b 1 +
1
c
__
c 1 +
1
a
_
1 abc = 1, a, b, c > 0;
b)
x
3
(1 +y)(1 +z)
+
y
3
(1 +z)(1 +x)
+
z
3
(1 +x)(1 +y)

3
4
, xyz = 1, x, y, z > 0.;
CAPTULO 3. NMEROS REALES 102
c)
ab
a
5
+b
5
+ab
+
bc
b
5
+c
5
+bc
+
ac
a
5
+c
5
+ac
1, abc = 1, a, b, c > 0;
d)
1
a
3
(b +c)
+
1
b
3
(a +c)
+
1
c
3
(a +b)

3
2
, abc = 1, a, b, c > 0;
e) abc +bcd +acd +abd
1
27
+
176
27
abcd, a +b +c +d = 1, a, b, c, d > 0;
f )
a
3
b + 2c + 3d
+
b
3
a +c +d
+
c
3
a +b +d
+
d
3
a +b +c

1
3
, ab +bc +cd+ad = 1, a, b, c, d > 0;
g) 0 xy +yz +xz 2xyz
7
27
, x +y +z = 1, x, y, z 0;
h)
1
x
1
y
1
z
2
1
+
1
x
2
y
2
z
2
2

8
(x
1
+x
2
)(y
1
+y
2
) (z
1
+z
2
)
2
, x
1
x
2
> 0, y
1
, y
2
, z
1
, z
2

R, x
1
y
1
> z
2
1
, x
2
y
2
> z
2
2
;
i) (a +b c)
2
(b +c a)
2
(c +a b)
2
(a
2
+b
2
c
2
)(b
2
+c
2
a
2
)(a
2
+c
2
b
2
), a, b, c R;
j)
_
(a
2
b +b
2
c +c
2
a)(ab
2
+bc
2
+ca
2
) abc +
3
_
(a
3
+abc)(b
3
+abc)(c
3
+abc), a, b, c >
0;
k)
_
a
4
+b
4
+c
4
+
_
a
2
b
2
+b
2
c
2
+c
2
a
2

_
a
3
b +b
3
c +c
3
a +
_
ab
3
+bc
3
+ca
3
; a, b, c
>0.
l) 7(pq +qr +rp) 2 + 9pqr, p +q +r = 1, p, q, r > 0;
m)
a
2
a
2
+ 2bc
+
b
2
b
2
+ 2ac
+
c
2
c
2
+ 2ab
1
bc
a
2
+ 2bc
+
ac
b
2
+ 2ac
+
ab
c
2
+ 2ab
, a, b, c > 0;
n)
(b +c a)
2
(b +c)
2
+a
2
+
(a +c b)
2
(a +c)
2
+b
2
+
(a +b c)
2
(a +b)
2
+c
2

3
5
, a, b, b > 0;
o)
1
1 +ab
+
1
1 +bc
+
1
1 +ac

3
2
, a
2
+b
2
+c
2
= 3, a, b, c > 0;
p) (ab +bc +ac)
_
1
(a +b)
2
+
1
(b +c)
2
+
1
(a +c)
2
_

9
4
, a, b, c > 0;
q) a(1b
2
)(1c
2
) +b(1c
2
)(1a
2
) +c(1a
2
)(1b
2
)
4

3
9
, ab+bc+ac = 1, a, b, b > 0;
r)
1
2
(a +b)
2
+
1
4
(a +b) a

b +b

a, a, b > 0;
s)
a
b
+
b
c
+
c
a

a +b
a +c
+
b +c
a +b
+
a +c
b +c
, a, b, c > 0;
t)
a
a
2
+ 1
+
b
b
2
+ 1
+
c
c
2
+ 1

9
10
, a +b +c = 1, a, c, c
3
4
;
u)
1
1 +a +b
+
1
1 +b +c
+
1
1 +a +c

1
a + 2
+
1
b + 2
+
1
c + 2
, abc = 1, a, b, c > 0;
v)
_
a
2
+ (1 b)
2
+
_
b
2
+ (1 c)
2
+
_
c
2
+ (1 a)
2

2
2
, a, b, c R;
w)
_
a
2
ab +b
2
+
_
b
2
bc +c
2

_
a
2
+ac +c
2
, a, b, c > 0;
x)
3

abc +
[a b[ +[b c[ +[c a[
3

a +b +c
3
, a, b, c > 0;
y)
3
_
(a +x)(b +y)(c +z)
3

abc +
3

xyz, a, b, c, x, y, z > 0;
z)
a
a +
_
(a +b)(a +c)
+
b
b +
_
(b +c)(a +b
+
c
c +
_
(a +c)(b +c)
1, x, y, z > 0.
17. Demuestre las desigualdades:
a)
a

1 a
+
b

1 b
+
c

1 c

_
3
2
, a +b +c = 1, a, b, c > 0;
b)
1
2

(a +b)(1 ab)
(1 +a
2
)(1 +b
2
)

1
2
, a, b R;
CAPTULO 3. NMEROS REALES 103
c)
1
1
1+a
+
1
1+b
+
1
1+c

1
1
a
+
1
b
+
1
c

1
3
, a, b, c > 0;
d)
_
1
a
1
__
1
b
1
__
1
c
1
_

_
3
a +b +c
1
_
3
, 0 < a, b, c <
1
2
;
e)
9
4(a +b +c)

a
(a +b)(a +c)
+
b
(b +c)(a +b)
+
c
(a +c)(b +c)
, a, b, c > 0.
18. Demuestre que la suma de los catetos de un tringulo rectngulo no es mayor que la diag-
onal de un cuadrado construido sobre la base de la hipotenusa.
19. Demuestre que la suma de cubos de los catetos de un tringulo rectngulo es menor que
el cubo de la hipotenusa.
20. Demuestre que el cuadrado tiene mayor rea que cualquier rectngulo del mismo permetro.
21. Demuestre que el rea de un tringulo arbitrario no supera un cuarto del cuadrado de su
medio permetro.
3.11. Smbolo sumatoria
La aritmtica universal y la Teora de nmeros prescinden en cada conjunto de la naturaleza
y el orden de sus elementos, considerando todos los objetos como equivalentes. El Anlisis com-
binatorio, prescinde tambin de la naturaleza de los objetos, pero no del orden y, por tanto, no
considera todos los objetos como equivalentes, sino que necesita distinguir entre s los elementos
de cada conjunto, designndolos por letras distintas, o con otra notacin que impida confundir
uno con otro. Los problemas de la combinatoria son innitos; solamente nos ocuparemos de los
tres fundamentales, estudiando los diversos modos de ordenar los elementos de un conjunto, o de
agrupar stos, o de coordinar dos conjuntos.
Durante mucho tiempo se ha considerado la Combinatoria completamente desligada de la ar-
itmtica; pero el moderno mtodo de establecer el concepto de nmero, es una de tantas pruebas
del papel preponderante que la nocin de orden adquiere en las ms variadas teoras matemticas,
de modo cada vez ms acentuado.
Denicin 3.12 Sucesin
Una sucesin es un conjunto ordenado de nmeros formados de acuerdo con una ley dada y se
representa por a
i
, donde i varia de k hasta p.
Una sucesin es nita si contiene un nmero limitado de trminos, es decir, si la sucesin tiene
un ltimo trmino. Sea dado un conjunto nito de elementos, no necesariamente distintos. Con-
vengamos en considerar que todos los elementos estn enumerados de cierta forma y se les han
atribuido unos nmeros que varan seguidamente a partir del nmero k hasta algn p. Los elemen-
tos se designarn mediante una letra, indicando el nmero. El propio nmero, el cual se llamar
en adelante ndice, puede ocupar en la designacin un lugar arbitrario. El ndice puede disponerse
al lado de la letra encerrado entre parntesis, abajo cerca de la letra, arriba cerca de la letra, etc.
CAPTULO 3. NMEROS REALES 104
Esto no tiene ninguna importancia. Con mayor frecuencia lo escribiremos al lado de la letra, abajo,
a la derecha.
Una suma de elementos a
k
, a
k+1
, a
k+2
, ..., a
p2
, a
p1
, a
p
se designar mediante el smbolo

de la forma siguiente:
a
k
+a
k+1
+a
k+2
+... +a
p2
+a
p1
+a
p
=
p

i=1
a
i
El ndice i en esta frmula se denomina ndice de adicin. Nada variar si lo designamos con
cualquier otra letra. A veces, bajo el signo de la suma se indicar de manera explcita aquella
totalidad de los ndices segn los cuales se realiza la adicin. Por ejemplo, la suma en consideracin
podra ser escrita de la forma siguiente:
a
k
+a
k+1
+a
k+2
+... +a
p2
+a
p1
+a
p
=
p

i=1
a
i
=

1ip
a
i
Teorema 3.14 Para todo par de elementos a
i
, b
i
, la suma de estos elementos es igual a c
i
,
donde a
i
y b
i
dependen del ndice de adicin i, entonces:
p

i=k
(a
i
b
i
) =
p

i=k
a
i

i=k
b
i
.
Demostracin
Para demostrar la propiedad, desarrollamos el miembro izquierdo de la identidad:
p

i=k
(a
i
b
i
) = (a
k
b
k
) + (a
k+1
b
k+1
) +... + (a
p
b
p
)
= (a
k
+a
k+1
+... +a
p
) (b
k
+b
k+1
+... +b
p
)
=
p

i=k
a
i

i=k
b
i
.
Teorema 3.15 Si todo elemento a
i
es igual al producto del elemento b
i
y elemento , donde
no depende del ndice de adicin i, entonces:
p

i=k
a
i
=
p

i=k
b
i
=
p

i=k
b
i
.
Demostracin
Desarrollando el miembro izquierdo, obtenemos:
p

i=k
b
i
= b
k
+b
k+1
+... +b
p
= (b
k
+b
k+1
+... +b
p
)
=
p

i=k
b
i
.
CAPTULO 3. NMEROS REALES 105
Teorema 3.16 Para todo nmero real, entonces:
p

i=k
= (p k + 1).
Demostracin
Por el teorema anterior, haciendo b
i
= 1 y variando i de 0 hasta p, tenemos lo siguiente:
p

i=0
b
i
= b
0
+b
1
+... +b
p
= + + +... +
. .
p trminos
= +p
= (p + 1).
Ahora bien, si hacemos que i vare desde 1 hasta p, obtenemos la siguiente identidad:
p

i=1
b
i
= b
1
+b
1
+... +b
p
= + +... +
. .
p trminos
= p.
Como
p

i=0
= (p + 1) = (p 0 + 1) y
p

i=1
= p = (p 1 + 1)
podemos concluir que
p

i=k
= (p k + 1).
Teorema 3.17 Para todo p k, entonces:
p

i=k
(a
i
a
i1
) = a
p
a
k1
.
Demostracin
Para demostrar esta propiedad, desarrollamos el miembro izquierdo de la identidad:
p

i=k
(a
i
a
i1
) = (a
k
a
k1
) + (a
k+1
a
k
) +... + (a
p1
a
p2
) + (a
p
a
p1
)
= a
p
a
k1
.
Ejemplo 3.27 Determinar si cada una de las igualdades son verdaderas o falsas:
a)
100

i=0
i
4
=
100

i=1
i
4
; b)
100

i=0
2 = 200; c)
100

i=0
(2 +i) = 2 +
100

i=0
i; d)
100

i=1
(i + 1)
2
=
99

i=0
i
2
.
Solucin
CAPTULO 3. NMEROS REALES 106
a)
100

i=0
i
4
= 0
4
+
100

i=1
i
4
=
100

i=1
i
4
, Verdadera;
b)
100

i=0
2 = (100 0 + 1) 2 = 101 2 = 202 ,= 200, Falsa;
c)
100

i=0
(2 +i) =
100

i=0
2 +
100

i=0
i = (100 0 + 1) 2 +
100

i=0
i = 202 +
100

i=0
i ,= 2 +
100

i=0
i, Falsa;
d)
100

i=1
(i + 1)
2
=
100

i=1
(i
2
+ 2i + 1) =
100

i=1
i
2
+ 2
100

i=1
i +
100

i=1
1 = 15150 +
99

i=1
i
2
,=
99

i=1
i
2
, Falsa.
Ejemplo 3.28 Evaluar la siguiente suma:
S
n
=
n

i=1
i
Solucin
Hgase i
2
(i 1)
2
= 2i 1:
i = 1 : 1
2
0
2
= 2 1 1
i = 2 : 2
2
1
2
= 2 2 1
i = 3 : 3
2
2
2
= 2 3 1
.
.
.
.
.
.
i = n : n
2
(n 1)
2
= 2 n 1
Sumando ambos miembros, obtenemos n
2
= 2
n

i=1
i n, de donde
n

i=1
i =
n
2
+n
2
. Por lo tanto
n

i=1
i =
n(n + 1)
2
Este ejercicio es interesante, ya que el profesor Butter de Braunschweig lo dio a resolver a sus
alumnos, entre los que estaba Carl Friedrich Gauss (1777 - 1855), que en aquel momento con nueve
aos de edad, hizo un ejercicio especial para mantenerlos tranquilos (como el crey). Butter puso
la tarea de sumar los nmeros del 1 hasta el 40. Esto es un caso especial del problema de sumar
los nmeros del 1 hasta n. La mayora de los alumnos calcularon: 1 +2 = 3; 3 +3 = 6; 6 +4 = 10;
... etc. Pero no as Gauss. El orden los nmeros por pares: 1 +40; 2 +39; ...; 20 +21 y obtuvo 20
pares, cuya suma en cada caso dio 41. Slo necesitaba multiplicarlos: 20 x 41 = 820.
Podemos suponer que la suma de los nmeros naturales del 1 hasta n es siempre
S
n
=
n
2
(n + 1)
Ejemplo 3.29 Evaluar la siguiente suma:
S
n
=
n

i=1
i
2
Solucin
CAPTULO 3. NMEROS REALES 107
Hgase i
3
(i 1)
3
= 3i
2
3i + 1:
i = 1 : 1
3
0
3
= 3 1
2
3 1 + 1
i = 2 : 2
3
1
3
= 3 2
2
3 2 + 1
i = 3 : 3
3
2
3
= 3 3
2
3 3 + 1
.
.
.
.
.
.
i = n : n
3
(n 1)
3
= 3 n
2
3 n + 1
Sumando ambos miembros, obtenemos n
3
= 3
n

i=1
i
2
3
n

i=1
i +n. Por el ejemplo anterior, tenemos
que n
3
= 3
n

i=1
i
2
3
n(n + 1)
2
+ n. Realizando operaciones adecuadas, obtenemos el resultado
buscado
n

i=1
i
2
=
n(n + 1)(2n + 1)
6
Ejemplo 3.30 Evaluar la siguiente suma:
S
n
=
n

i=1
i
3
Solucin
Hgase i
4
(i 1)
4
= 4i
3
6i
2
+ 4i 1:
i = 1 : 1
4
0
4
= 4 1
3
6 1
2
+ 4 1 1
i = 2 : 2
4
1
4
= 4 2
3
6 2
2
+ 4 2 1
i = 3 : 3
4
2
4
= 4 3
3
6 3
2
+ 4 3 1
.
.
.
.
.
.
i = n : n
4
(n 1)
4
= 4n
3
6n
2
+ 4n 1
Sumando ambos miembros, obtenemos n
4
= 4
n

i=1
i
3
6
n

i=1
i
2
+ 4
n

i=1
i n. Por los ejemplos
anteriores, tenemos que n
4
= 4
n

i=1
i
3
n(n + 1)(2n + 1) + 2n(n + 1) n. Realizando operaciones
adecuadas, obtenemos el resultado buscado
n

i=1
i
3
=
n
2
(n + 1)
2
4
Ejemplo 3.31 Hallar las siguientes sumas:
a)
n

i=1
8(i 1); b)
n

i=1
(ai +b)
2
; c)
n

i=1
i
2
n
3
; d)
n

i=1
_
a +
(b a)i
n
__
b a
n
_
;
e)
n

i=1
(i 1)
3
_
b a
n
_
; f )
n

i=1
(2i + 1)(i + 6).
Solucin
CAPTULO 3. NMEROS REALES 108
a) Desarrollamos la sumatoria:
n

i=1
8(i 1) = 8
n

i=1
i
n

i=1
8
= 8
_
n
2
+n
2
_
(n 1 + 1) 8
= 4n(n 1).
b) Desarrollamos la sumatoria:
n

i=1
(ai +b)
2
=
n

i=1
(a
2
i
2
+ 2abi +b
2
)
= a
2
n

i=1
i
2
+ 2ab
n

i=1
i +
n

i=1
b
2
)
= a
2
_
n(n + 1)(2n + 1)
6
_
+ 2ab
_
n
2
+n
2
_
+ (n 1 + 1)b
2
=
a
2
n
3
3
+
a(a + 2b)n
2
2
+
(a
2
+ 6ab + 6b
2
)n
6
.
c) Desarrollamos la sumatoria:
n

i=1
i
2
n
3
=
1
n
3
n

i=1
i
2
=
1
n
3

n(n + 1)(2n + 1)
6
=
(n + 1)(2n + 1)
6n
2
.
d) Desarrollamos la sumatoria:
n

i=1
_
a +
(b a)i
n
__
b a
n
_
=
b a
n
n

i=1
a +
b a
n
n

i=1
(b a)i
n
=
b a
n
(n 1 + 1)a +
b a
n

b a
n

n
2
+n
2
=
(b a)[(a +b)n a +b]
2n
.
e) Desarrollamos la sumatoria:
n

i=1
(i 1)
3
_
b a
n
_
=
b a
n
n

i=1
(i
3
3i
2
+ 3i 1)
=
b a
n
_
n

i=1
i
3
3
n

i=1
i
2
+ 3
n

i=1
i
n

i=1
1
_
=
b a
n
_
n
2
(n + 1)
2
4

3n(n + 1)(2n + 1)
6
+
3n(n + 1)
2
n
_
=
b a
n
_
n
2
(n + 1)
2
4

3n(n + 1)(2n + 1)
6
+
3n(n + 1)
2
n
_
=
(b a)n(n 1)
2
4
CAPTULO 3. NMEROS REALES 109
f ) Desarrollamos la sumatoria:
n

i=1
(2i + 1)(i + 6) =
n

i=1
(2i
2
+ 7i + 6)
= 2
n

i=1
i
2
+ 7
n

i=1
i +
n

i=1
6
=
2n(n + 1)(2n + 1)
6
+
7(n
2
+n)
2
+ 6n
=
4n
3
+ 27n
2
+ 59n
6
.
Ejemplo 3.32 Hallar la suma:
S
n
=
n

k=1
4
(k + 1)(k + 2)
Solucin
Aplicando fracciones parciales, descomponemos la siguiente expresin:
4
(k + 1)(k + 2)
=
A
k + 1
+
B
k + 2
eliminando denominadores y resolviendo luego el sistema de ecuaciones, obtenemos que A = 4 y
B = 4. Reemplazamos en la ecuacin inicial y desarrollando, obtenemos:
n

k=1
4
(k + 1)(k + 2)
= 4
n

k=1
_
1
k + 1

1
k + 2
_
= 4
_
1
2

1
3
+
1
3

1
4
+... +
1
n

1
n + 1
+
1
n + 1

1
n + 2
_
= 4
_
1
2

1
n + 2
_
=
2n
n + 2
.
Ejemplo 3.33 Hallar la suma:
S
n
=
n

k=1
1
(4k 3)(4k + 1)
Solucin
Aplicando fracciones parciales, descomponemos la siguiente expresin:
1
(4k 3)(4k + 1)
=
A
4k 3
+
B
4k + 1
eliminando denominadores y resolviendo luego el sistema de ecuaciones, obtenemos que A =
1
4
y
CAPTULO 3. NMEROS REALES 110
B =
1
4
. Reemplazamos en la ecuacin inicial y desarrollando, obtenemos:
n

k=1
1
(4k 3)(4k + 1)
=
1
4
n

k=1
_
1
4k 3

1
4k + 1
_
=
1
4
_
1
1
5
+
1
5

1
9
+
1
9
+... +
1
4n 7

1
4n 3
+
1
4n 3

1
4n + 1
_
=
1
4
_
1
1
4n + 1
_
=
n
4n + 1
.
Ejemplo 3.34 Hallar la suma:
S
n
=
n

k=1
k
2
(2k 1)(2k + 1)
Solucin
Dividimos la fraccin y obtenemos la expresin:
S
n
=
1
4
_
n

k=1
1 +
n

k=1
1
(2k 1)(2k + 1)
_
Hacemos la descomposicin en fracciones parciales
1
(2k 1)(2k + 1)
=
A
2k 1
+
B
2k + 1
de donde obtenemos A =
1
2
y B =
1
2
. Reemplazamos estos valores en la ecuacin inicial, obte-
niendo
S
n
=
1
4
_
n +
1
2
n

k=1
_
1
2k 1

1
2k + 1
_
_
=
1
4
_
n +
1
2
_
1
1
3
+
1
3

1
5
+... +
1
2n 3

1
2n 1
+
1
2n 1

1
2n + 1
__
=
1
4
_
n +
1
2
_
1
1
2n + 1
__
=
n
2
_
n + 1
2n + 1
_
.
Supongamos ahora que los elementos estn marcados con dos ndices, cada uno de los cuales
vara independientemente. Aceptemos para estos elementos una designacin general a
ij
y sea, por
ejemplo, k i p; m j n.
Teorema 3.18 Para todo p k y n m, entonces:
p

i=k
n

j=m
a
ij
=
n

j=m
p

i=k
a
ij
, i, j N.
CAPTULO 3. NMEROS REALES 111
Demostracin
Dispongamos los elementos en forma de una tabla rectangular
a
km
a
k(m+1)
. . . a
kn
a
(k+1)m
a
(k+1)(m+1)
. . . a
(k+1)n
.
.
.
.
.
.
.
.
.
.
.
.
a
pm
a
p(m+1)
. . . a
pm
Est claro que independientemente del orden en que se realiza la sumacin, el resultado ser el
mismo. Por ello, al tomar en consideracin la designacin introducida para la suma, tenemos:
(a
km
+a
k(m+1)
+. . . +a
kn
) + (a
(k+1)m
+a
(k+1)(m+1)
+. . . +a
(k+1)n
) +. . .
+(a
pm
+a
p(m+1)
+. . . +a
pn
) =
n

j=m
a
kj
+
n

j=m
a
(k+1)j
+. . . +
n

j=m
a
pj
=
p

j=k
_
_
n

j=m
a
ij
_
_
.
Por otro lado, esta misma suma es igual a la siguiente identidad:
(a
km
+a
(k+1)m
+. . . +a
pm
) + (a
k(m+1)
+a
(k+1)(m+1)
+. . . +a
p(m+1)
) +. . .
+(a
kn
+a
(k+1)n
+. . . +a
pn
) =
p

i=k
a
im
+
p

i=k
a
i(m+1)
+. . . +
p

i=k
a
in
=
n

j=m
_
p

i=k
a
ij
_
.
Por consiguiente, podemos concluir que
p

i=k
_
_
n

j=m
a
ij
_
_
=
n

j=m
_
p

i=k
a
ij
_
Si convenimos en realizar la adicin siempre de manera sucesiva segn los ndices de las sumas
dispuestas de derecha a la izquierda, los parntesis pueden ser omitidos y, en denitiva, obtenemos:
p

i=k
n

j=m
a
ij
=
n

j=m
p

i=k
a
ij
Este teorema nos indica que al sumar segn dos ndices se puede cambiar el orden de la adicin.
Teorema 3.19 Si a
ij
=
i
b
ij
, donde
i
no depende del ndice j, entonces:
p

i=k
n

j=m

i
b
ij
=
p

i=k

i
n

j=m
b
ij
, i, j N.
La demostracin de este teorema se la puede realizar de forma anloga a la del anterior.
CAPTULO 3. NMEROS REALES 112
Ejemplo 3.35 Hallar la siguiente suma:
n

i=1
m

j=1
(i + 5j)
Solucin
n

i=1
m

j=1
(i + 5j) =
n

i=1
_
_
m

j=1
i + 5
m

j=1
j
_
_
=
n

i=1
_
mi +
5(m
2
+m)
2
_
= m
n

i=1
i +
5(m
2
+m)
2
n

i=1
1
= m
n
2
+n
2
+
5(m
2
+m)
2
n
=
mn(5m+n + 6)
2
.
3.12. Smbolo producto
El producto de los elementos a
k
, a
k+1
, a
k+2
, ..., a
p2
, a
p1
, a
p
se designar mediante el smbolo
de tal forma que
a
k
a
k+1
a
k+2
... a
p2
a
p1
a
p
=
p

i=k
a
i
.
Teorema 3.20 Para todo p k se cumple qua
p

i=k
a
i
b
i
=
_
p

i=k
a
i
__
p

i=k
b
i
_
Demostracin
Desarrollando el miembro izquierdo, obtenemos:
p

i=k
a
i
b
i
= (a
k
)(b
k
)(a
k+1
)(b
k+1
)(a
k+2
)(b
k+2
)...(a
p
)(b
p
)
= (a
k
a
k+1
a
k+2
...a
p
)(b
k
b
k+1
b
k+2
...b
p
)
=
_
p

i=k
a
i
__
p

i=k
b
i
_
Teorema 3.21 Si a
i
= b
i
, entonces
p

i=k
b
i
=
pk+1
p

i=k
b
i
, p k
CAPTULO 3. NMEROS REALES 113
Demostracin
Por el teorema anterior, haciendo a
i
= 1; tenemos
p

i=0
a
i
b
i
= (a
0
b
0
)(a
1
b
1
)(a
2
b
2
)...(a
p
b
p
)
= (b
0
)(b
1
)(b
2
)...(b
p
)
= (...)
. .
p factores
(b
0
b
1
b
2
...b
p
)
=
p
p

i=k
b
i
=
p+1
p

i=k
b
i
.
p

i=1
a
i
b
i
= (a
1
b
1
)(a
2
b
2
)(a
3
b
3
)...(a
p
b
p
)
= (b
1
)(b
2
)(b
3
)...(b
p
)
= (...)
. .
p factores
(b
1
b
2
b
3
...b
p
)
=
p
p

i=1
b
i
.
Como
p

i=0
(a
i
b
i
) =
p+1
p

i=0
b
i
=
p0+1
p

i=0
b
i
y
p

i=1
(a
i
b
i
) =
p
p

i=1
b
i
=
p1+1
p

i=1
b
i
podemos concluir que
p

i=k
b
i
=
pk+1
p

i=k
b
i
Teorema 3.22 Para todo p k, entonces:
p

i=k
a
i
a
i1
=
a
p
a
k1
, si cadaa
i1
,= 0.
Demostracin
desarrollamos el miembro izquierdo de la igualdad:
p

i=k
a
i
a
i1
=
a
k
a
k1

a
k+1
a
k

a
k+2
a
k+1
...
a
p1
a
p2

a
p
a
p1
=
a
p
a
k1
.
CAPTULO 3. NMEROS REALES 114
Ejemplo 3.36 Hallar los siguientes productos:
a)
n

i=1
10
i
; b)
n

i=1
n

j=1
2
ij
.
Solucin
a) Desarrollando el producto, tenemos
n

i=1
10
i
= 10 10
2
10
3
... 10
n
= 10
1+2+3+...+n
= 10
1
2
n(n+1)
.
b) Desarrollando el producto, tenemos
n

i=1
n

j=1
2
ij
=
n

i=1
_
2
i1
2
i2
2
i3
... 2
in
_
=
n

i=1
_
2
1
2
2
2
3
... 2
n
_
2
ni
=
n

i=1
2
ni
2

1
2
n(n+1)
=
_
n

i=1
2

1
2
n(n+1)
__
n

i=1
2
ni
_
=
_
2

1
2
n(n+1)
_
n
_
2
n
2
2n
2
3n
... 2
n
2
_
= 2

1
2
n
2
(n+1)
_
2
n+2n+3n+...+n
2
_
= 2

1
2
n
2
(n+1)
2
1
2
n
2
(n+1)
= 1.
3.13. Tarea
1. Desarrolle y calcule
a)
3

k=1
2

j=1
_
1
j
+k
_
; b)
2

i=1
i

j=1
2ji.
Resp: a)
33
2
; b) 14.
2. Si a
k
=
1
3
k(k + 1)(k + 2) demuestre que a
k
a
k1
= k(k + 1) y de aqu calcule el valor de
n

i=1
i(i + 1).
Resp:
1
3
n(n + 1)(n + 2).
CAPTULO 3. NMEROS REALES 115
3. Calcular las sumatorias:
a)
n

j=1
(j + 1)
3
; b)
n

k=1
(n k + 1); c)
n+1

k=1
(1 + 2
k1
).
Resp: a)
1
4
(n + 1)
2
(n + 2)
2
1; b)
1
6
n(n + 1)(n + 2); c) n + 2
n+1
.
4. Demuestre que:
p

i=0
(1)
i1
4(i + 1)
(2i + 1)(2i + 3)
=
1
3
+
(1)
n1
2n + 3
5. Demuestre y calcule:
2n

k=1
(1)
k
k
2
=
n

k=1
(4k 1)
Resp: 2n
2
+n.
6. Calcular las sumatorias:
a)
n

k=1
k
4
+k
2
+ 1
k
4
+k
; b)
n

k=1
2k + 1
k
2
(k + 1)
2
; c)
n

k=1
log
_
k + 1
k
_
k
;
d)
n

k=1
log
_
1 +
1
k
2
+ 2k
_
.
Resp: a)
n(n + 2)
n + 1
; b) 1
1
(n + 1)
2
; c) log
(n + 1)
n
n!
; d) log
2(n + 1)
n + 2
.
7. Calcular
n

k=2
_
5k + 3

5k 2

25k
2
+ 5k 6

3
k4
6
k
4
2
k+2
_
Resp:
1

5n + 3

1
32
(3
2n2
1) +
1
2

_
1
2
_
n
.
8. Se dene (n + 1)! = n!(n + 1). Por tanto, (n + 1)! = 1 2 3 ... n (n + 1). Calcular:
a)
n

k=1
kk!; b)
n

k=1
(k
2
+ 1)k!; c)
2n1

k=n+1
k
(k + 1)!
; d)
n1

k=0
k + 1 k
2
(k + 1)!
.
Resp: a) (n + 1)! 1!; b) (n + 1)!n; c)
1
(n + 1)!

1
(2n)!
; d) 1
1
n!
+
1
(n 1)!
.
9. Hallar las sumas:
a)
n

k=1
1
(3k 2)(3k + 1)
; b)
n

k=1
1
k(k + 1)(k + 2)(k + 3)
.
10. Hallar las sumas:
a)
n

k=20
8(k 1); b)
n

k=20
(ak +b)
2
; c)
n

k=20
k
2
n
3
; d)
124

k=1
(2k 3);
CAPTULO 3. NMEROS REALES 116
e)
50

k=0
_
k
k + 1

1
k + 2
_
.
11. Hallar las sumas:
a)
n

i=1
n

j=1
ij; b)
n

i=1
n

j=1
n

k=1
(2j i)
2
k.
12. Dados x
1
= 1, x
2
= 2, x
3
= 3, x
4
= 7 y x
5
= 4, evale
5

i=1
(2x
i
3) y
5

i=1
(x
i
+ 2)
2
.
13. Sea x =
1
n
n

i=1
x
i
. Demustre que
n

i=1
(x
i
x)
2
=
n

i=1
x
2
i

1
n
_
n

i=1
x
i
_
2
.
14. Hallar el nmero de esferas en un apilamiento sobre una base rectangular cuyos lados con-
tienen 15 y 20 esferas, si el tope es una lnea.
Resp: 1840.
15. Demuestre que la suma de todos los nmeros naturales impares que son menores que 6n y
que no son mltiplos de 3, es 6n
2
.
16. Demuestre que la suma de los productos en parejas (distintas) de los n primeros nmeros
naturales impares es
1
6
n(n 1)(3n
2
n 1).
17. Demuestre que la suma de los productos de todas las parejas de nmeros distintos que se
pueden sumar con los n primeros nmeros naturales es
1
24
n(n
2
1)(3n + 2).
18. Esferas iguales son apiladas en forma de una pirmide de base cuadrada. Hallar el nmero
de esferas en una pirmide incompleta que tiene n capas si cada lado de la base contiene 2n
esferas.
Resp:
1
6
n(2n + 1)(7n + 1)..
19. Calcular los siguientes productos:
a)
n

i=10
2; b)
n

i=21
10
i
; c)
n

i=12
n

j=12
2
ij
; d)
n

i=1
m

j=1
p

k=1
2
i+j+k
.
3.14. Induccin matemtica
Existe una inmensidad de armaciones que dependen de un nmero natural n. Cmo se deben
entender tales armaciones? Por cuanto hay una innidad de nmeros naturales, cada armacin
contiene, de hecho, un nmero innito de armaciones.
CAPTULO 3. NMEROS REALES 117
Surge la pregunta: cmo podemos convencernos de la validez de una armacin dependiente
de un nmero natural? Con el n de demostrar las armaciones dependientes de un nmero natural
n se emplea, a menudo, el mtodo general de demostracin, el principio de induccin matemtica
completa. Este mtodo est basado en los axiomas de los nmeros naturales.
Es un hecho intuitivamente claro que si S es un conjunto no vaco que slo contenga enteros
no negativos, entonces S contendr un entero mnimo. Esto es, hay un entero m en S tal que
m s para todo entero s de S. Este hecho intuitivamente evidente se llama principio de buena or-
denacin. Utilizamos el principio de buena ordenacin para explicar la demostracin por induccin.
Sea P(n) una expresin que depende de un entero n no negativo. Deseamos poder demostrar
que la expresin P(n) se verica para toda n 0. No obstante, slo se dan los dos siguientes
hechos:
i) La expresin P(0) se verica.
ii) Siempre que k sea un entero positivo tal que P(k 1) se verique, entonces tambin P(k) se
vericar.
El mtodo de demostracin por induccin es un principio general que arma que si se satis-
facen tanto i) como ii), entonces P(n) se vericara para todo n Z no negativo. Para ver porqu
esta conclusin absoluta deber deducirse de i) y ii), supngase que hay algunos enteros n para
los cuales P(n) es falsa. Sea S el conjunto de todos los enteros n para los cuales P(n) es falsa.
Este S no es el conjunto vaco. Por el principio de buena ordenacin, hay un entero mnimo en S.
Sea k el entero mnimo de S. Entonces P(k) es falsa. Como P(0) se verica, debemos tener k > 0.
Por tanto, k1 es un entero no negativo y no puede estar en S. Por consiguiente, P(k1) se verica.
Pero ahora i) nos indica que P(k) se verica. Hemos producido la contradiccin de que P(k) es
tanto verdadera como falsa. El nico escape de esta contradiccin es que S debe ser el conjunto
vaco. Por consiguiente P(n) se verica para toda n. Muchas veces deseamos sustituir i) y ii) por:
i) Hay un entero t tal que P(t) se verique;
ii) Siempre que se veriquen k 1 t y P(k 1), entonces se vericar P(k).
Se deduce entonces, con un razonamiento anlogo al presentado antes, que P(n) se verica para
todos los enteros n t.
Otro tipo de demostracin por induccin (llamado segundo principio de induccin) es el sigu-
iente. Supngase que:
i) La expresin P(0) es verdadera;
ii) Siempre que k sea un entero positivo tal que P(0), P(1), P(2),..., P(k 1) sean todas ver-
daderas, entonces P(k) tambin es verdadera.
El segundo principio de induccin arma ahora que si ambos, i) y ii) se satisfacen, entonces
P(n) se verica para todo entero n no negativo. Para ver esto, se recurre nuevamente al principio
de buena ordenacin. Si P(n) no se verica siempre, hay algunos valores de n para los cuales P(n)
es falsa. Sea k el mnimo entero posible tal que para P(k) sea falsa. Como P(0) se verica, debe
vericarse que k > 0. Entonces P(0), P(1), ..., P(k 1) son verdaderas (porque k es mnimo) y, en
consecuencia por ii), P(k) tambin se verica.
Ahora tenemos la contradiccin de que P(k) es verdadera y falsa. Esta contradiccin indica
que no puede haber valores de n para los cuales P(n) sea falsa. Por tanto, P(n) se verica para
toda n 0. Por supuesto que se puede modicar el segundo principio de induccin en la forma
siguiente. Supngase que:
CAPTULO 3. NMEROS REALES 118
i ) La expresin P(t) se verica;
ii ) Siempre que k sea un entero tal que k > t y P(t), P(t +1), P(t +2), ..., P(k 1) se veriquen
todas, entonces tambin P(k) se vericar.
Se desprende entonces que P(n) se verica para todos los enteros n t. A continuacin, justicamos
lo antes mencionado con el siguiente teorema.
Teorema 3.23 Sea P(n) una proposicin que contiene a la variable n, cuyo universo es el
conjunto N de enteros positivos y que tiene la propiedad de que P(n) es verdadera o falsa, pero no
ambas al mismo tiempo, para cada n N. Si P(n) satisface las dos condiciones:
i) P(1) es verdadera,
ii) P(k 1) P(k) k N, entonces, P(n) es verdadera n N.
Demostracin
Sea Z
+
el conjunto de todos los enteros positivos, para el cual P(n) es verdadera; es decir:
Z
+
= X/X N y P(n) es verdadera.
De i) observamos que 1 Z
+
y, de ii) observamos que (k 1) Z
+
k Z
+
. Por lo tanto Z
+
es
un conjunto inductivo y, por la denicin de Z
+
sabemos que Z
+
N.
Por otro lado, por la denicin del conjunto N de enteros positivos sabemos que N Z
+
. Por
consiguiente, Z
+
= N; es decir, el conjunto de todos los enteros positivos para el cual se cumple
P(n), es el conjunto N.
Para concluir, podemos decir que el principio de induccin matemtica completa se emplea
frecuentemente en la demostracin de las armaciones que son vlidas no para todos los nmeros
naturales n, sino slo para n superiores o iguales a cierto nmero natural p.
Ejemplo 3.37 Haciendo uso del mtodo de induccin matemtica demuestre que
1
2
+ 2
2
+ 3
2
+... +n
2
=
n(n + 1)(2n + 1)
6
Solucin
1) Comprobamos la validez de la frmula dada para n = 1. el primer miembro es igual a la
unidad. El segundo miembro
1(1 + 1)(2 1 + 1)
6
= 1. Por lo tanto, la frmula es justa para n = 1.
2) Suponiendo que la frmula dada tambin es justa para cierto nmero n (n > 1), demostremos
que para n + 1 tiene lugar la misma frmula:
1
2
+ 2
2
+ 3
2
+... +n
2
+ (n + 1)
2
=
(n + 1)[(n + 1) + 1][2(n + 1) + 1]
6
Efectivamente
1
2
+ 2
2
+ 3
2
+... +n
2
+ (n + 1)
2
=
n(n + 1)(2n + 1)
6
+ (n + 1)
2
=
(n + 1)[n(2n + 1) + 6(n + 1)]
6
=
(n + 1)(2n
2
+ 7n + 6)
6
=
(n + 1)(n + 2)(2n + 3)
6
=
(n + 1)[(n + 1) + 1][2(n + 1) + 1]
6
CAPTULO 3. NMEROS REALES 119
que es lo que se quera demostrar. Por consiguiente, basndonos en el mtodo de induccin
matemtica, sacamos la conclusin de que la frmula dada es cierta para todo nmero natural
n.
Ejemplo 3.38 Demostrar que si n es un nmero entero positivo, 4
n
+15n1 se divide por 9.
Solucin
Si n = 1, el nmero 4
n
+15n1 es igual a 18, es decir, se divide por 9. Supongamos que 4
k
+15k1
se divide por 9 y sea n = k + 1. Entonces
4
k+1
+ 15(k + 1) 1 = 4(4
k
+ 15k 1) 45k + 18
= 4(4
k
+ 15k 1) 9(5k 2).
Pero, segn la hiptesis de induccin, 4k + 15k 1 se divide por 9, a causa de que el segundo
miembro, y junto con ste el primer miembro de la igualdad se dividen por 9.
Ejemplo 3.39 Demostrar que, si a y b son nmeros positivos y a < b, para cualquier n natural
es vlida la desigualdad a
n
< b
n
.
Solucin
Pata n = 1, la armacin es evidente. Supongamos que a
k
< b
k
; al multiplicar esta desigualdad por
el nmero positivo a, obtenemos a
k+1
< ab
k
. pero b es un nmero positivo debido a que b
k
a < b
k
b,
es decir,
a
k+1
< b
k+1
.
Ejemplo 3.40 Valindonos del proceso de induccin, demuestre la desigualdad:
1
9
+
1
25
+... +
1
(2n + 1)
2
<
1
4
.
Solucin
Para n = 1 esta desigualdad tiene la forma
1
9
<
1
4
, es decir, es vlida. supongamos que la desigualdad
a demostrar es vlida para n = k:
1
9
+
1
25
+... +
1
(2k + 1)
2
<
1
4
.
Para n = k + 1 el primer miembro es igual a
1
9
+
1
25
+... +
1
(2k + 3)
2
=
_
1
9
+
1
25
+... +
1
(2k + 1)
2
_
+
1
(2k + 3)
2
.
Segn la hiptesis de induccin, la suma entre los corchetes es menor que
1
4
, por eso
1
9
+
1
25
+... +
1
(2k + 3)
2
<
1
4
+
1
(2k + 3)
2
.
Es claro, que de la desigualdad obtenida de ninguna manera se puede deducir que su primer
miembro es menor que
1
4
. de tal modo, segn el mtodo de induccin, la demostracin queda sin
solucin. Sin embargo, esta desigualdad se demuestra fcilmente por otro mtodo.
El mtodo de induccin matemtica es cmodo para determinar las sumas de un nmero nito
de sumandos.
CAPTULO 3. NMEROS REALES 120
Ejemplo 3.41 Hallar la suma
1 + 3 + 5 +... + (2n 1)
Solucin
Designemos esta suma con S
n
, es decir
S
n
= 1 + 3 + 5 +... + (2n 1)
Para obtener una expresin para S
n
que no necesite la adicin algebraica de n sumandos, calculemos
algunos primeros valores de esta suma:
S
1
= 1, S
2
= 1 + 3 = 4, S
3
= 1 + 3 + 5 = 9, S
4
= 1 + 3 + 5 + 7 = 16.
Vemos que estos valores son los cuadrados secesivos de nmeros naturales. es natural suponer que
S
n
= n
2
. Para demostrar la validez de esta igualdad utilicemos el mtodo de induccin matemtica.
tenemos:
1) S
1
= 1 + 1
2
. Por lo tanto, la frmula es justa para n = 1;
2) suponiendo que ella es justa para cierto n, demostremos que para n + 1 tiene lugar la frmula
S
n+1
= (n + 1)
2
. En efecto
S
n+1
= S
n
+ [2(n + 1) 1] = n
2
+ (2n + 1) = (n + 1)
2
que es lo que se necesitaba demostrar. por consiguiente, basndonos en el mtodo de induccin
matemtica sacamos la conclusin de que la frmula S
n
= n
2
es justa para todo nmero natural n
y
1 + 3 + 5 +... + (2n 1) = n
2
.
Ejemplo 3.42 En un plano hay trazadas n rectas de las cuales dos no son paralelas y tres
no pasan por un mismo punto. determinar entre cuntas partes queda dividido el plano con estas
rectas.
Solucin
Al trazar los dibujos requeridos, podemos anotar la siguiente correlacin entre el nmero n de
rectas que renen las condiciones del problema y el nmero a
n
de partes en las que estas rectas
dividen el plano:
n 1 2 3 4 5 ...
a
n
2 4 7 11 16 ...
Es fcil ver que en calidad del trmino general de la sucesin a
n
conviene emplear la expresin
a
n
= 1 +
n(n + 1)
2
Esta frmula se comprueba fcilmente pata los primeros valores de n, sin embargo, de ah no
se deduce que da respuesta al problema planteado. Esta armacin requiere una demostracin
complementaria aplicando el mtodo de induccin matemtica.
Al prescindir de la seleccin recin efectuada, demostremos que n rectas (de las cuales dos no
son paralelas y tres no pasan por un mismo punto) dividen el plano en a
n
partes, donde a
n
se
calcula segn la frmula dada.
CAPTULO 3. NMEROS REALES 121
Es evidente que cuando n = 1, la frmula general es vlida. aplicando la hiptesis de induccin,
examinemos (k +1)-sima recta que satisface la condicin del problema. Eliminando de stas arbi-
trariamente k rectas, podemos decir que dividen el plano en 1+
k(k + 1)
2
partes. ahora adicionamos
(k + 1)-sima recta. dado que no es paralela a ninguna de las rectas precedentes, intersecar, por
consiguiente, todas las k rectas. Ya que no pasar por ninguno de los puntos de interseccin de las
rectas precedentes, pasar entonces por un fracmento k + 1, en los que ya fue dividido el plano, y
dividir en dos partes cada uno de estos fragmentos, es decir, resultarn aadidos k+1 fragmentos.
Por consiguiente, el nmero total de fragmentos en los cuales se divide el plano k + 1 rectas es:
1 +
k(k + 1)
2
+k + 1 = 1 +
(k + 1)[(k + 1) + 1]
2
= a
k+1
.
3.15. Tarea
1. Demuestre que si x N y f(x) = x
4
+ 6x
3
+ 11x
2
+ 6x, f(x) es divisible para 24.
2. Demuestre que si f(x) = x
2
(x
2
+ 14) + 49, donde x es un nmero impar, entonces f(x) es
divisible para 64.
3. Demuestre que 1
2
+ 2
2
+ 3
2
+... +n
2
=
n(n + 1)(2n + 1)
6
.
4. Demuestre que 1
3
+ 2
3
+ 3
3
+... +n
3
=
_
n(n + 1)
2
_
2
.
5. Demuestre que la suma de los cubos de tres nmeros reales sucesivos se divide por 9.
6. Demuestre que f(n) = 3
2n+1
+ 40n 67 es divisible para 64.
7. Demuestre que f(n) = n
4
+ 6n
3
+ 11n
2
+ 6n es divisible para 24, con cualquier n N.
8. Demuestre que si x N, entonces f(x) = x
5
5x
3
+ 4x es divisible para 120.
9. Demuestre que si x es un nmero mutuamente primo con 6, entonces f(x) = x
3
1 es
divisible para 24.
10. Demuestre que si x N, entonces f(x) = 2x
3
+ 3x
2
+x es divisible para 6.
11. Con qu valores de x N la expresin f(x) = x
4
+ 4 es un nmero primo?
12. Demuestre que si x es un nmero par, entonces f(x) =
x
12
+
x
2
8
+
x
3
24
es un nmero entero.
CAPTULO 3. NMEROS REALES 122
13. Demuestre que si n es cualquier entero positivo, entonces
1
3
(n
3
+ 2n) es un entero.
14. Si u
i+1
= 2u
i
+ 1, i N. Demostrar que u
n
+ 1 = 2
n1
(u
1
+ 1).
15. Sabiendo que
4 =
3
u
1
= u
1
+
3
u
2
= u
2
+
3
u
3
= ... = u
n
+
3
u
n+1
Demuestre que u
n
=
3
n+1
3
3
n+1
1
.
16. Si u
1
= 0 y u
n+1
= (1 +x)u
n
nx, n N, demuestre que
u
n
=
1
x
[1 +nx (1 +x)
n
], x ,= 0.
17. Demuestre que n N: 2
n+4
> (n + 4)
2
.
18. Demuestre que n Z, n 1, h 1: (1 +h)
n
1 +nh.
19. Demuestre que los nmeros de la forma u
n
= 2
2n+1
9n
2
+ 3n 2 son divisibles por 54.
20. Demostrar que u
n
= 3
4n+2
+ 5
2n+1
es mltiplo de 14.
21. Se denen los nmeros a
1
, a
2
, a
3
, ... mediante a
1
=

2 y a
n+1
=

2a
n
. demuestre que
a
n
< 2 para todo n.
22. Demuestre que si u
0
= 2, u
1
= 3, ..., u
k+1
= 3u
k
2u
k1
, entonces n 0, u
n
= 2
n
+ 1.
23. Demuestre que n(n + 1)(n + 2)...(n +p 1) es divisible por p.
24. Demuestre que 1
3
+ 3
3
+... + (2n + 1)
3
= (n + 1)
2
(2n
2
+ 4n + 1), n 0.
25. Demuestre que el producto (2n + 1) nmeros reales negativos es un nmero negativo.
26. Demuestre que para n > 2, la suma de los ngulos interiores de un polgono regular de n
lados es (n 2).
27. Sean u
1
= 10, u
2
= 47, ..., u
n
= 23u
n1
60u
n2
, n 3. Demuestre que u
n
= 20
n1
+3
n+1
.
28. Dado que a
0
= 12, a
1
= 11,..., a
n+2
= a
n+1
+ 6a
n
, n 0. Demuestre que a
n
=
7 3
n
+ 5(2)
n
.
CAPTULO 3. NMEROS REALES 123
29. Sean a
1
= 0, a
2
= 1, ..., a
n+1
= n(a
n
+a
n1
), n 2. Demostrar que
a
n
= n!
_
1
2!

1
3!
+... +
(1)
n
n!
_
30. Sean a
1
= a, a
2
= b, a
3
=
1
3
(a
1
+ 2a
2
), a
4
=
1
3
(a
2
+ 2a
3
), ... a, b R, a ,= b. Demuestre
que
a
n
= a
3c
c + 3
_
_

c
3
_
n1
1
_
, c = b a.
31. Demuestre que si x N, entonces f(x) =
x
5
120
+
x
4
12
+
7x
3
24
+
5x
2
12
+
x
5
es un nmero entero.
32. Demuestre que para todo n N:
a) 1 + 2 + 3 +... +n =
n(n + 1)
2
;
b) 1 2 + 2 3 +... +n(n + 1) =
n(n + 1)(n + 2)
3
;
c)
_
1
1
2
__
1
1
3
_
...
_
1
1
n + 1
_
=
1
n + 1
;
d) 1 4 + 2 7 + 3 10 +... +n(3n + 1) = n(n + 1)
2
;
e)
_
1
1
4
__
1
1
9
__
1
1
16
_
...
_
1
1
(n + 1)
2
_
=
n + 2
2n + 2
;
f ) 1 1! + 2 2! +... +n n! = (n + 1)! 1;
g)
0
1!
+
1
2!
+
2
3!
+... +
n 1
n!
= 1
1
n!
;
h)
1
2
1 3
+
2
2
3 5
+... +
n
2
(2n 1)(2n + 1)
=
n(n + 1)
2(2n + 1)
;
i)
1
1 3 5
+
2
3 5 7
+... +
n
(2n 1)(2n + 1)(2n + 3)
=
n(n + 1)
2(2n + 1)(2n + 3)
;
j)
1
1 2 3
+
1
2 3 4
+... +
1
n(n + 1)(n + 2)
=
1
2
_
1
2

1
(n + 1)(n + 2)
_
;
k) 1 2 3 + 2 3 4 +... +n(n + 1)(n + 2) =
n(n + 1)(n + 2)(n + 3)
4
;
l) 2 1
2
+ 3 2
2
+... + (n + 1)n
2
=
n(n + 1)(n + 2)(3n + 1)
12
;
m)
1
1 2 3 4
+... +
1
n(n + 1)(n + 2)(n + 3)
=
1
3
_
1
6

1
(n + 1)(n + 2)(n + 3)
_
;
n) 1 +x +x
2
+... +x
n
=
x
n+1
1
x 1
, donde x ,= 1;
o) 7 + 77 + 777 +... + 777..,7 =
7(10
n+1
9n 10)
81
;
p) 1
1
2
+
1
3

1
4
+... +
1
2n 1

1
2n
=
1
n + 1
+... +
1
2n
;
q)
1
2
2
1
+
1
3
2
1
+... +
1
(n + 1)
2
1
=
3
4

1
2(n + 1)

1
2(n + 2)
;
r)
1
1 4
+
1
4 7
+
1
7 10
+... +
1
(3n 2)(3n + 1)
=
n
3n + 1
;
s)
1
1 5
+
1
5 9
+
1
9 13
+... +
1
(4n 3)(4n + 1)
=
1
4n + 1
;
CAPTULO 3. NMEROS REALES 124
t)
1
a(a + 1)
+
1
(a + 1)(a + 2)
+... +
1
(a +n 1)(a +n)
=
n
a(a +n)
;
u) 1
2
+ 3
2
+... + (2n 1)
2
+
n(4n
2
1)
3
;
v) 1 2 + 2 3 + 3 4 +... +n(n 1) =
n(n 1)(n + 1)
3
;
w) 1 2
2
+ 2 3
2
+ 3 4 +... + (n 1)n
2
=
n(n
2
1)(3n + 2)
12
.
33. Demuestre las identidades:
a) x + 2x
2
+ 3x
3
+... +nx
n
=
x (n + 1)x
n+1
+nx
n+2
(1 x)
2
, donde x ,= 1;
b)
x + 1
2
+
x + 3
4
+
x + 7
8
+... +
x + 2
n
1
2
n
=
(x 1)(2
n
1)
2
n
+n;
c)
1
1 +x
+
2
1 +x
2
+
4
1 +x
4
+... +
2
n
1 +x
2
n
=
1
x 1
+
2
n+1
1 x
2
n+1
, donde [x[ , = 1;
d)
x
1 x
2
+
x
2
1 x
4
+
x
4
1 x
8
+... +
x
2
n1
1 x
2
n
=
1
1 x

x x
2
n
1 x
2
n
, donde [x[ , = 1;
e)
_
x
1
x
_
2
+... +
_
x
n

1
x
n
_
2
=
1
x
2
1
_
x
2n+2

1
x
2
n
_
2n 1.
34. Demuestre la validez de las armaciones:
a) f(n) = 6
2n
1 es divisible para 35;
b) f(n) = 4
n
+ 15n 1 es divisible para 9;
c) f(n) = 2
5n+3
+ 5
n
3
n+2
es divisible para 17;
d) f(n) = 6
2n
+ 3
n+2
+ 3
n
es divisible para 11;
e) f(n) = 3
2n+2
8n 9 es divisible para 64;
f ) f(n) = 3
3n+2
+ 5 2
3n+1
es divisible para 19;
g) f(n) = 2
n+5
3
4n
+ 5
3n+1
es divisible para 37;
h) f(n) = 7
n+2
+ 8
2n+1
es divisible para 57;
i) f(n) = 11
n+2
+ 12
2n+1
es divisible para 133;
j) f(n) = 2
n+2
3
n
+ 5n 4 es divisible para 25;
k) f(n) = 5
2n+1
+ 2
n+4
+ 2
n+1
es divisible para 23;
l) f(n) = 3
2n+2
5
2n
3
3n+2
2
2n
es divisible para 1053;
m) f(n) = n
6
+ 3n
5
+ 6n
4
7n
3
2n es divisible para 24;
n) f(n) = n
7
n es divisible para 7;
o) f(n) = 11
n+2
+ 12
2n+1
es divisible para 133;
p) f(n) = n
3
+ (n + 1)
3
+ (n + 2)
3
es divisible para 9.
35. Demustrese que
n

i=1
(1 +x
2
i1
) =
1 x
2
n
1 x
, x ,= 1.
Cul es el valor del producto cuando x = 1?
36. Compruebe que si x
1
, x
2
, ..., x
n
y x son conjuntos, entonces:
a) x (x
1
x
2
... x
n
) = (x x
1
) (x x
2
) ... (x x
n
);
b) (x
1
x
2
... x
n
) = x
1
x
2
... x
n
.
CAPTULO 3. NMEROS REALES 125
37. Demuestre las desigualdades:
a) n! > 2
n1
, si n > 2;
b) 2
n
n! < n
n
, si n > 2;
c) (n!)
2
<
_
(n + 1)(2n + 1)
6
_
n
;
d) (2n!) <
_
2n + 1
2
_
2n
;
e)
a
1
a
2
+
a
2
a
3
+... +
a
n1
a
n
+
a
n
a
1
n, si a
1
> 0, ..., a
n
> 0;
f )
_
a
2
+b
2
+c
2
a +b +c
_
a+b+c
> a
a
b
b
c
c
, si a, b, c son los nmeros enteros positivos distintos;
g)
1
n + 1
+
1
n + 2
+... +
1
3n + 1
> 1;
h)
x
1
+x
2
+... +x
n
n

n

x
1
x
2
...x
n
, donde x
1
, x
2
, ..., x
n
son nmeros positivos arbitrarios;
i) 2
n1
(a
n
+b
n
) > (a +b)
n
, a +b > 0, a ,= b, n > 1;
j)
(2n)!
(n!)
2
>
4
n
n + 1
, n > 1;
k) (2n)! < 2
2
n
(n!)
2
;
l)

n
n

n! <
n + 1
2
;
m)
1
2

3
4

5
6
...
2n 1
2n
<
1

3n + 1
;
n)
1
2n

1 3 5 ... (2n 1)
2 4 6 ... 2n
;
o)
n
2
< 1 +
1
2
+
1
3
+... +
1
2
n
1
< n;
p)
_
1 +
1
n
_
n
<
_
1 +
1
2n
_
2n
;
q) 1
2
+ 2
2
+ 3
2
+... + (n 1)
2
<
n
3
3
< 1
2
+ 2
2
+ 3
2
+... +n
2
.
38. Sean x
1
, x
2
, ..., x
n
nmeros positivos tomados al azar, con la particularidad de que
x
1
x
2
...x
n
= 1. Probar que x
1
+x
2
+... +x
n
1.
39. Demostrar la siguiente desigualdad:
_
n

k=1
(a
k
+b
k
)
2
_1
2

_
n

k=1
a
2
k
_1
2
_
n

k=1
b
2
k
_1
2
, a
k
, b
k
R.
3.16. Factorial y frmula del binomio de Newton
Denicin 3.13 Factorial
Sea n N. Se dene n! (factorial de n) de la siguiente manera:
n! = 1 2 3 ... n
Es decir, si n es un nmero natural, entonces n! es el producto de los n primeros nmeros
naturales. Adems, es fcil ver que n! = n(n 1)!, para cualesquiera n N. Los factoriales au-
mentan considerablemente a medida que se incrementa el nmero. Por fortuna, no es generalmente
CAPTULO 3. NMEROS REALES 126
necesario desarrollar un factorial por completo, ya que cuando se utilizan suelen presentarse en
grupos, y es posible eliminarlos. Los factoriales algunas veces comprenden operaciones de adicin y
sustraccin. Cuando los nmeros estn dentro del parntesis y el signo factorial fuera, es necesario
completar la adicin o sustraccin antes de determinar el factorial.
Para calcular la suma de los primeros n nmeros naturales hay una frmula cmoda
1 + 2 + 3 +... +n =
n(n + 1)
2
.
Para el producto de los primeros n nmeros naturales tal frmula no existe, pero esta magnitud,
que se encuentra con frecuencia en el anlisis combinatorio y otras partes de la matemtica, tiene
una designacin especial: n! (factorial de n). as pues, por denicin
1 2 3 ... n = n!
El signo de admiracin est elegido, quizs, para la designacin debido al hecho de que incluso para
valores comparativamente pequeos de n, el nmero n! es muy grande; para mostrar lo rpido que
crece n! con el aumento de n escribamos estos nmeros para n de 1 a 10:
1! = 1, 2! = 1 2 = 2, 3! = 1 2 3 = 6 4! = 3! 4 = 24, ; 5! = 4! 5 = 120,
6! = 5! 6 = 720, 7! = 6! 7 = 5040, 8! = 7! 8 = 40320, 9! = 8! 9 = 362880,
10! = 9! 10 = 3628800.
De la denicin de n! se deduce que las factoriales de dos nmeros naturales vecinos n y n + 1
estn relacionadas por la frmula
(n + 1)! = (n + 1) n!
Notemos que si en esta igualdad se sustituye n = 0, obtenemos 1! = 1 0!, por eso se supone
0! = 1; este acuerdo resulta frecuentemente cmodo en distintas frmulas generales.
Ejemplo 3.43 Demustrese la frmula (n + 1)! n! = n n!.
Solucin
Hagamos uso del mtodo de induccin matemtica. Tenemos:
1. para n = 1(1 + 1)! 1! = 1 1!, de donde 1 = 1, por lo tanto, la frmula es justa;
2. suponiendo su validez para cierto n demostremos que para n + 1 tiene lugar la frmula
(n + 2)! (n + 1)! = (n + 1)(n + 1)!. Efectivamente
(n + 2)! (n + 1)! = (n + 1)(n + 2)n! (n + 1)n!
= (n + 1)[(n + 2) 1]n!
= (n + 1)(n + 1)n!
= (n + 1)(n + 1)!
que es lo que se quera demostrar. Por consiguiente, sacamos la conclusin de que la frmula es
justa para todo n natural.
CAPTULO 3. NMEROS REALES 127
Ejemplo 3.44 Hallar la suma
1 1! + 2 2! + 3 3! +... +n n!
Solucin
Reemplacemos cada sumando por la diferencia segn la frmula
(n + 1)! n! = n n!
obtenemos
(1 + 1)! 1! + (2 1)! 2! + (3 + 1)! 3! +... + (n + 1)! n!
= 2! 1! + 3! 2! + 4! 3! +... + (n + 1)! n! = (n + 1)! 1
ya que todos los sumandos en el primer miembro de la igualdad, a excepcin del segundo y el
penltimo, se suprimen recprocamente. Por consiguiente
1 1! + 2 2! + 3 3! +... +n n! = (n + 1)! 1.
En las matemticas se utilizan ampliamente los magncos nmeros llamados coecientes bino-
miales. Estos tienen la designacin especial
_
n
k
_
y se hallan por la frmula
_
n
k
_
=
n!
k!(n k)!
donde n son nmeros enteros no negativos y k, nmeros enteros no negativos que satisfacen la
condicin 0 k n. Si el numerador y denominador de la fraccin se reducen eliminando (nk)!,
obtenemos la frmula
_
n
k
_
=
n(n 1)...(n k + 1)
k!
que es cmoda de guardar en la memoria y con cuya ayuda es ms fcil realizar los clculos. El
denominador de esta fraccin est formado por el producto de todos los primeros k nmeros natu-
rales y el numerador, por el producto de k nmeros naturales escritos en el orden de decrecimiento,
comenzando con el nmero n. En el anlisis combinatorio esta frmula dene el coeciente binomial
_
n
k
_
como nmero de combinaciones de n elementos tomados k a k.
Denicin 3.14 Coeciente binomial
Sean n , m N. Se dene el coeciente binomial de la siguiente manera
_
n
k
_
=
_
0, si 0 n k
n!
k!(nk)!
, si n k 0
Ejemplo 3.45 Calcular
_
20
6
_
.
Solucin
Tenemos
_
20
6
_
=
20!
6! 14!
=
20 19 18 17 16 15
1 2 3 4 5 6
= 38760.
CAPTULO 3. NMEROS REALES 128
Con ayuda de los coecientes binomiales se demuestran muchas armaciones matemticas y,
en particular, una frmula muy importante del binomio de Newton
(a +b)
n
=
_
n
0
_
a
n
+
_
n
1
_
a
n1
b +... +
_
n
k
_
a
nk
b
k
+... +
_
n
n
_
b
n
a cuyo nombre se debe tambin la denominacin de los coecientes
_
n
k
_
. Esta frmula suele
escribirse brevemente como:
(a +b)
n
=
n

k=0
_
n
k
_
a
nk
b
k
.
De esta frmula, en particular, para n = 2 y n = 3 obtenemos las frmulas conocidas:
(a +b)
2
=
_
2
0
_
a
2
+
_
2
1
_
ab +
_
2
2
_
b
2
= a
2
+ 2ab +b
2
(a +b)
3
=
_
3
0
_
a
3
+
_
3
1
_
a
2
b +
_
3
2
_
ab
2
+
_
3
3
_
b
3
= a
3
+ 3a
2
b + 3ab
2
+b
3
Teorema 3.24 Para todo a, b R y n N, entonces
(a +b)
n
=
n

k=0
_
n
k
_
a
nk
b
k
Demostracin
Para la demostracin de este teorema utilizaremos el principio de induccin matemtica
i) Probamos cuando n = 1:
(a +b)
1
=
1

k=0
_
1
k
_
a
1k
b
k
=
_
1
0
_
a
1
b
0
+
_
1
1
_
a
0
b
1
= a +b.
ii) Cuando n = r:
(a +b)
r
=
r

k=0
_
r
k
_
a
rk
b
k
Cuando n = r + 1:
(a +b)
r+1
= (a +b)(a +b)
r
= (a +b)
r

k=0
_
r
k
_
a
rk
b
k
=
r

k=0
_
r
k
_
a
rk+1
b
k
. .
(1)
+
r

k=0
_
r
k
_
a
rk
b
k+1
. .
(2)
CAPTULO 3. NMEROS REALES 129
Es necesario poder expresar estas dos sumatorias como una sola, para eso se requiere que ambas
sumatorias tengan el mismo lmite. En (1), hacemos que k = j y en (2), hacemos que k = j 1;
entonces:
(a +b)
r+1
=
r

j=0
_
r
j
_
a
rj+1
b
j
. .
(1)
+
r+1

j=1
_
r
j 1
_
a
rj+1
b
j
. .
(2)
= a
r+1
b
0
+
r

j=1
_
r
j
_
a
rj+1
b
j
+
r

j=1
_
r
j 1
_
a
rj+1
b
j
+a
0
b
r+1
= a
r+1
b
0
+
r

j=1
__
r
j
_
+
_
r
j 1
__
a
rj+1
b
j
+a
0
b
r+1
como
_
r
j
_
+
_
r
j 1
_
=
_
r + 1
j
_
y haciendo que
a
r+1
b
0
=
_
r + 1
0
_
a
r+1
b
0
y a
0
b
r+1
=
_
r + 1
r + 1
_
a
0
b
r+1
entonces
(a +b)
r+1
= a
r+1
b
0
+
r

j=1
_
r + 1
j
_
a
rj+1
b
j
+a
0
b
r+1
=
_
r + 1
0
_
a
r+1
b
0
+
r

j=1
_
r + 1
j
_
a
rj+1
b
j
+
_
r + 1
r + 1
_
a
0
b
r+1
=
r+1

j=0
_
r + 1
j
_
a
rj+1
b
j
=
r+1

j=0
_
r + 1
j
_
a
(r+1)j
b
j
haciendo j = k, obtenemos
r+1

k=0
_
r + 1
k
_
a
(r+1)k
b
k
.
Ejemplo 3.46 Emplee el teorema del binomio para demostrar que se cumple
3
n
=
n

i=0
2
i
_
n
i
_
Solucin
CAPTULO 3. NMEROS REALES 130
Haciendo 3
n
= (1 + 2)
n
y desarrollando este binomio, resulta
(1 + 2)
n
=
_
n
0
_
1
n0
2
0
+
_
n
1
_
1
n1
2
1
+
_
n
2
_
1
n2
2
2
+... +
_
n
n
_
1
nn
2
n
=
_
n
0
_
2
0
+
_
n
1
_
2
1
+
_
n
2
_
2
2
+... +
_
n
n
_
2
n
=
n

i=0
2
i
_
n
i
_
Ejemplo 3.47 De cuntas maneras se puede seleccionar de un grupo de 25 personas, un
comit formado por 4 personas?
Solucin
En virtud de que un comit es un grupo no ordenado de personas, la respuesta es
_
25
4
_
=
25!
4!(25 4)!
= 12650.
Ejemplo 3.48 De cuntas maneras se puede seleccionar de un grupo de 9 hombres y 8 mu-
jeres, un comit de 3 hombres y 2 mujeres?
Solucin
Al igual que en el ejemplo anterior, se puede ver que es posible seleccionar los 3 hombres de
_
9
3
_
= 84 maneras y las 2 mujeres de
_
8
2
_
= 28 maneras. El comit puede formarse en dos pasos
sucesivos: elegir a los hombres y despus a las mujeres. Por el principio de multiplicacin, el nmero
total de comits es (84)(28) = 2352.
A continuacin demostraremos algunas propiedades algebraicas de los nmeros combinatorios
Teorema 3.25 Para todo m, n N, donde m n, entonces:
_
m
n
_
=
_
m
mn
_
Demostracin
Partiendo del miembro derecho, tenemos
_
m
mn
_
=
m!
(mn)![m(mn)]!
=
m!
(mn)!(mm+n)!
=
m!
n!(mn)!
=
_
m
n
_
.
Teorema 3.26 Para todo m, n N, donde m n, entonces:
_
m
n
_
=
_
m1
n 1
_
+
_
m1
n
_
CAPTULO 3. NMEROS REALES 131
Demostracin
Partiendo del miembro derecho, tenemos
_
m1
n 1
_
+
_
m1
n
_
=
(m1)!
(n 1)!(mn)!
+
(m1)!
n!(mn 1)!
=
n(m1)!
n(n 1)!(mn)!
+
(mn)(m1)!
n!(mn)!
=
m(m1)!
n!(mn)!
=
m!
n!(mn)!
=
_
m
n
_
.
Teorema 3.27 Para todo m, n N, entonces:
_
m
0
_
+
_
m+ 1
1
_
+... +
_
m+n
n
_
=
_
m+n + 1
n
_
Demostracin
Partiendo del miembro derecho, tenemos
_
m+n + 1
n
_
=
_
m+n
n
_
+
_
m+n
n 1
_
_
m+n
n 1
_
=
_
m+n 1
n 1
_
+
_
m+n 1
n 2
_
_
m+n 1
n 2
_
=
_
m+n 2
n 2
_
+
_
m+n 2
n 3
_
_
m+n + 1
n
_
=
_
m+n
n
_
+
_
m+n 1
n 1
_
+
_
m+n 2
n 2
_
+
_
m+n 2
n 3
_
como
_
m+ 2
1
_
=
_
m+ 1
1
_
+
_
m+ 1
0
_
entonces
_
m+n + 1
n
_
=
_
m+n
n
_
+
_
m+n 1
n 1
_
+... +
_
m+ 2
1
_
=
_
m+n
n
_
+
_
m+n 1
n 1
_
+... +
_
m+ 1
1
_
+
_
m+ 1
0
_
=
_
m+n
n
_
+
_
m+n 1
n 1
_
+... +
_
m+ 1
1
_
+
_
m
0
_
Teorema 3.28 Para todo m, n, r N, entonces:
_
m
n
__
n
r
_
=
_
m
r
__
mr
n r
_
CAPTULO 3. NMEROS REALES 132
Demostracin
Partiendo del miembro derecho, tenemos
_
m
r
__
mr
n r
_
=
m!
r!(mr)!

(mr)!
(n r)!(mr n +r)!
=
m!
r!(mn)!(n r)!

n!
n!
=
m!
n!(mn)!

n!
r!(n r)!
=
_
m
n
__
n
r
_
.
Teorema 3.29 Para todo n N, entonces:
_
n
0
_
+
_
n
1
_
+... +
_
n
n
_
= 2
n
Demostracin
Sea 2
n
= (1 + 1)
n
, entonces
(1 + 1)
n
=
n

k=0
_
n
k
_
1
nk
1
k
=
_
n
0
_
1
n0
1
0
+
_
n
1
_
1
n1
1
1
+
_
n
2
_
1
n2
1
2
+... +
_
n
n
_
1
nn
1
n
=
_
n
0
_
+
_
n
1
_
+... +
_
n
n
_
Teorema 3.30 Para todo m, n, r N, entonces:
_
n +m
r
_
=
_
n
0
__
m
r
_
+
_
n
1
__
m
r 1
_
+... +
_
n
r
__
m
0
_
Demostracin
Como (1 + x)
n+m
= (1 + x)
n
(1 + x)
m
. Aplicando el teorema del binomio a cada miembro de la
identidad, resulta:
(1 +x)
n+m
=
_
n +m
0
_
+
_
n +m
1
_
x +... +
_
n +m
r
_
x
r
+... +
_
n +m
n +m
_
x
n+m
(1 +x)
n
(1 +x)
m
=
__
n
0
_
+
_
n
1
_
x +... +
_
n
n
_
x
r
_ __
m
0
_
+
_
m
1
_
x +... +
_
m
m
_
x
r
_
Para probar la proposicin, basta observar que
_
n +m
r
_
es el coeciente de x
r
en el polinomio
del primer miembro y que
_
n
0
__
m
r
_
+
_
n
1
__
m
r 1
_
+... +
_
n
r
__
m
0
_
es el coeciente x
r
en el polinomio que se obtiene al efectuar el producto de los dos polinomios del
segundo miembro
__
n
0
_
+
_
n
1
_
x +... +
_
n
n
_
x
r
_ __
m
0
_
+
_
m
1
_
x +... +
_
m
m
_
x
r
_
CAPTULO 3. NMEROS REALES 133
_
n
0
__
m
r
_
x
r
+
_
n
1
_
x
_
m
r 1
_
x
r1
+... +
_
n
r
_
x
r
_
m
0
_
__
n
0
__
m
r
_
+
_
n
1
__
m
r 1
_
+... +
_
n
r
__
m
0
__
x
r
=
_
n +m
r
_
x
r
_
n
0
__
m
r
_
+
_
n
1
__
m
r 1
_
+... +
_
n
r
__
m
0
_
=
_
n +m
r
_
.
Teorema 3.31 Para todo m, n, r N, entonces:
_
n +m
m
_
=
_
m
0
__
n
0
_
+
_
m
1
__
n
1
_
+... +
_
m
n
__
n
n
_
Demostracin
Como (1 +x
1
)
m
= x
m
(1 +x)
m
, multiplicando ambos miembros por (1 +x)
n
se obtiene
(1 +x)
n
(1 +x
1
)
m
= (1 +x)
n
x
m
(1 +x)
m
= x
m
(1 +x)
n+m
Aplicando el teorema del binomio a cada uno de los tres factores
(1 +x)
n+m
, (1 +x)
n
y (1 +x
1
)
m
resulta
x
m
(1 +x)
n+m
= x
m
__
m+n
0
_
+
_
m+n
1
_
x +... +
_
m+n
m
_
x
m
+... +
_
m+n
m+n
_
x
m+n
_
= (1 +x)
n
(1 +x
1
)
m
=
__
n
0
_
+
_
n
1
_
x +... +
_
n
n
_
x
n
_ __
m
0
_
+
_
m
1
_
x +... +
_
m
n
_
x
n
+... +
_
m
m
_
x
m
_
Igualando los trminos independientes, es decir, los trminos en el que el exponente de la x es cero,
se tiene que
_
m+n
m
_
=
_
n
0
__
m
0
_
+
_
n
1
__
m
1
_
xx
1
+... +
_
n
n
__
m
n
_
x
n
x
n
=
_
n
0
__
m
0
_
+
_
n
1
__
m
1
_
+... +
_
n
n
__
m
n
_
.
3.17. Tarea
1. Hllese la suma
0!
1!
+
1
2!
+
2
3!
+... +
n 1
n!
Resp: 1
1
n!
.
2. Deduzca las frmulas para las sumas:
a) S
n
=
1
1 3
+
1
3 5
+... +
1
(2n 1)(2n + 1)
;
b) S
n
=
1
1 4
+
1
4 7
+... +
1
(3n 2)(3n + 1)
;
CAPTULO 3. NMEROS REALES 134
c) S
n
=
1
1 5
+
1
5 9
+... +
1
(4n 3)(4n + 1)
;
d) S
n
=
1
1 6
+
1
6 11
+... +
1
(5n 4)(5n + 1)
;
e) S
n
= 1
2
2
2
+ 3
2
4
2
+... + (1)
n1
n
2
;
f ) S
n
=
1
1 2
+
1
2 3
+... +
1
n (n + 1)
3. Para todo n N, entonces:
_
n
0
_

_
n
1
_
+
_
n
2
_
... + (1)
n
_
n
n
_
= 0
4. Determine n, si
a)
_
n + 1
n + 1
_

_
n + 3
n
_
= 15(n + 2); b)
1

n
4

=
1

n
5

+
1

n
6

.
5. Demostrar las siguientes identidades:
a)
_
n
r
_
=
n
r
_
n 1
r 1
_
;
b)
_
n +m
m
_
=
m

k=0
_
n +k 1
k
_
, m, n > 0;
c)
_
n
k
_
=
_
n 1
k
_
+
_
n 2
k 1
_
+... +
_
n k 1
0
_
, n > k;
d)
_
n
r
_
=
n
nr
_
n 1
r
_
;
e)

n + 1
r + 1

n 1
r 1

n
r

n 1
r 1

n
r

n + 1
r + 1

n 1
r 1

= r, n > 1, 0 < r < n;


f )
1
n+1
_
2n
n
_
=
_
2n 1
n 1
_

_
2n 1
n + 1
_
;
g)
_
2n
n
_
= (n + 1)
_
2n
n
_
(n + 1)
_
2n
n 1
_
;
h)
_
m+n
2
_

_
m
2
_

_
n
2
_
= mn;
i)
_
r
r
_
+
_
r + 1
r
_
+
_
r + 2
r
_
+... +
_
n
r
_
=
_
n + 1
r + 1
_
;
j)
_
n
k 1
_
+ 2
_
n
k
_
+
_
n
k + 1
_
=
_
n + 2
k + 1
_
.
6. Realice una eleccin apropiada para a y b en el teorema del binomio y derive para obtener
la identidad siguiente:
n(1 +x)
n1
=
n

k=1
_
n
k
_
kx
k1
CAPTULO 3. NMEROS REALES 135
7. Aplique el resultado del problema anterior para demostrar que:
2
n1
n =
n

k=1
_
n
k
_
8. Demostre la identidad:
n

k=0
_
n +k
n
_
=
_
2n + 1
n + 1
_
9. En el desarrollo
_
3
2
x
2

1
3x
_
9
. Hallar:
a) El quinto trmino; b) El trmino que contiene a x
5
;
c) El trmino independiente de x.
Resp: a)
_
9
4
_
_
3
2
_
5
_

1
3
_
4
x
6
; b) No existe tal trmino; c)
7
18
.
10. Encontrar el coeciente de x
n
, en (1 x +x
2
)(1 +x)
2n+1
.
Resp:
_
2n + 1
n
_

_
2n + 1
n 1
_
+
_
2n + 1
n 2
_
.
11. Si x
r
se encuentra en el desarrollo de
_
x +
1
x
_
n
, hallar su coeciente.
Resp: El coeciente es
_
n
nr
2
_
, solo hay solucin si n r es par o cero.
12. Demuestre que los coecientes de x
2
y x
3
en el desarrollo de (x
2
+ 2x + 2)
n
son 2
n1
n
2
y
1
3
n(n
2
1)2
n1
.
13. Encuentre el valor de n, si
_
n
n 2
_
= 10.
Resp: n = 5.
14. Encuentre el trmino central de
_
x +
1
x
_
12
.
Resp:
_
12
6
_
.
15. Hllese la relacin que debe existir entre r y n, para que los coecientes de los trminos
de lugares 3r y r + 2 en el desarrollo de (1 +x)
2n
, sean iguales.
Resp: n = 2r.
16. Demostrar
_

n
k=0
_
n
k
__
2
=

2n
k=0
_
2n
k
_
.
17. Demuestre que
n1

i=0
n

j=1
_
n 1
i
__
n
j
_
= 2
n1
(2
n
1).
CAPTULO 3. NMEROS REALES 136
18. Demuestre que
_
n
0
_
2
+ 2
_
n
1
_
2
+ 3
_
n
2
_
2
+... + (n + 1)
_
n
n
_
2
=
(n + 2)(2n 1)!
n!(n 1)!
.
19. Demuestre que el coeciente del trmino central de (1 + x)
2n
, es igual a la suma de los
coecientes de los dos trminos centrales de (1 +x)
2n1
.
20. Demuestre que
a)

n
k=0
(k + 1)
_
n
k
_
x
k
= (1 +x)
n1
[1 + (n + 1)x];
b)

n
k=0
(2k + 1)
_
n
k
_
5
k
= 6
n1
(10n + 6).
21. Demuestre que
n

k=0
1
k + 2
_
n
k
_
=
1 +n2
n+1
(n + 1)(n + 2)
.
22. En el desarrollo
_
x
3

3
2

x
_
9
. Determine:
a) El sptimo trmino; b) El trmino que contiene a x
7
; c) La suma de los coe-
cientes de los dos trminos centrales.
Resp: a)
567
16
; b) No existe; c)
147
16
.
23. Determine el coeciente de x
15
en el desarrollo
_
3x
x
3
6
_
9
.
Resp: 283,5.
24. Encuentre el trmino independiente de x en los desarrollos:
a)
_
x
1
x
2
_
3n
; b)
_
x +
1
x
_
3
_
x
1
x
_
5
c) (2x + 1)
_
1 +
2
x
_
700
.
Resp: a)
(1)
n
(3n)!
n!(2n)!
; b) 0; c) 2801.
25. Encuentre el coeciente de
1
x
en el desarrollo de
(1 +x)
_
1 +
1
x
_
n
Resp:
(2n)!
(n1)!(n+1)!
.
26. Determine el valor de k si los coecientes de x
k
y de x
k+1
en el desarrollo (3x + 2)
19
son
iguales.
Resp: k = 11.
27. Encuentre el coeciente de x
4
en:
a) (1 x)(1 +x)
5
; b) (1 +x)(1 x)
n
.
Resp: a) 5; b)
1
24
n(n 1)(n 2)(n 7).
CAPTULO 3. NMEROS REALES 137
3.18. Progresiones
Denicin 3.15 Progresin aritmtica
Una sucesin se dice que es una progresin aritmtica si la diferencia entre cualquier trmino y el
anterior es la misma a lo largo de toda la sucesin. La diferencia algebraica entre cada trmino y
el anterior se denomina diferencia comn, y se denota por d.
Si a es el primer trmino y d es la diferencia comn de una progresin aritmtica, los trminos
sucesivos de la progresin aritmtica son a, a +d, a + 2d, a + 3d, ...
Teorema 3.32 La suma de n trminos de una progresin aritmtica con primer trmino a y
diferencia comn d est dado por
S
n
=
n
2
[2a + (n 1)d].
Demostracin
Si a es el primer trmino y d es la diferencia comn de una progresin aritmtica, la sucesin es
a, a +d, a + 2d, a + 3d, ....
Si la sucesin consta de n trminos y si k denota el ltimo trmino, k = a+(n1)d. El penltimo
trmino ser k d, el antepenltimo trmino ser k 2d, etc. Si S
n
representa la suma de estos n
trminos, entonces
S
n
= a + (a +d) + (a + 2d) +... + (k 2d) + (k d) +k
Si escribimos esta progresin en orden inverso, la suma es la misma, de modo que
S
n
= k + (k d) + (k 2d) +... + (a + 2d) + (a +d) +a
Sumando los dos valores de S
n
, obtenemos
2S
n
= (a +k) + (a +d +k d) + (a + 2d +k 2d) +... + (k d +a +d) + (k +a)
= (a +k) + (a +k) + (a +k) +... + (a +k) + (a +k) + (a +k)
Podemos observar que hay n trminos en el lado derecho y cada uno es igual a (a + k). En
consecuencia
2S
n
= n(a +k) S
n
=
n
2
(a +k)
Sustituyendo el valor de k de la ecuacin k = a + (n 1)d en la ecuacin anterior, obtenemos
S
n
=
n
2
[a +a + (n 1)d]
=
n
2
[2a + (n 1)d].
Ejemplo 3.49 Dada la sucesin 2, 9, 16, 23, 30, ..., calcular:
a) El vigsimo tercer trmino; b) El n-simo trmino.
Solucin
La sucesin dada es una progresin aritmtica, porque
d = 9 2 = 16 9 = 23 16 = 30 23 = 7
CAPTULO 3. NMEROS REALES 138
En consecuencia, la diferencia comn es d = 7. Tambin a = 2.
a) Cuando n = 23, obtenemos
k = 2 + (23 1)7 = 156.
b) Como k = a + (n 1)d, entonces el n-simo trmino es
k = 2 + (n 1)7 = 7n 5.
Ejemplo 3.50 Qu trmino de la sucesin 5, 14, 23, 32, ..., es 239?
Solucin
Como la sucesin es una progresin aritmtica, obtenemos que d = 9, entonces de
k = a + (n 1)d 239 = 5 + (n 1)9 n = 27
Por lo tanto 239 corresponde al trmino 27.
Ejemplo 3.51 La suma de tres nmeros en progresin aritmtica es 12 y su producto es 48.
Determine tales nmeros.
Solucin
Conviene tomar a d, a, a + d como los tres nmeros en progresin aritmtica, pues de su suma
igual a 12 se obtiene de inmediato que a = 4 y por tanto de (4d)4(4+d) = 48, se obtiene d = 2,
as los nmeros son 2, 4, 6 y 6, 4, 2.
Ejemplo 3.52 El ltimo trmino de la sucesin 20, 18, 16, ..., es - 4. Calcule el nmero de
trminos de esta sucesin.
Solucin
Como esta sucesin es una progresin aritmtica, d = 2 y a = 20, por lo tanto
4 = 20 + (n 1)(2) n = 13.
De esta manera podemos decir que la sucesin tiene 13 trminos.
Ejemplo 3.53 Si los trminos cuarto y noveno de una progresin aritmtica son 9 y 27 re-
spectivamente, encuentre el vigsimo octavo trmino.
Solucin
Como estos trminos pertenecen a una progresin aritmtica, entonces el n-simo trmino esta
dado por k = a + (n 1)d, lo cual indica que el cuarto trmino est dado por a + 3d = 9 y el
noveno trmino por a + 8d = 27. Resolviendo este sistema, obtenemos que d =
18
5
y a =
9
5
. De
esta manera podemos calcular el vigsimo octavo trmino que est dado por
k =
9
5
+ (28 1)
18
5
=
477
5
.
Ejemplo 3.54 El tercer trmino de una progresin aritmtica es a y el trmino de lugar 21
es a + 36b, con a y b reales dados, no nulos a la vez. determine la progresin aritmtica.
Solucin
Por hiptesis a
3
= a
1
+ 2d = a y a
2
= a
1
+ 20d = a + 36b de donde resolviendo el sistema para
a
1
y d se obtiene a
1
= a 4b y d = 2b por tanto resulta a
n
= 2bn + a 6b que es la progresin
aritmtica pedida.
Ejemplo 3.55 Determine la suma de los 100 primeros trminos de una progresin aritmtica,
cuyo tercer trmino es 4 veces el primero y su sexto trmino es 17.
CAPTULO 3. NMEROS REALES 139
Solucin
a
3
= 4a
1
y a
6
= 17 conducen a resolver el sistema
_
a
1
+ 2d = 4a
1
a
1
+ 5d = 17
de donde a
1
= 2 y d = 3, por tanto
S
100
= 50[4 + 99 3] = 15050.
Ejemplo 3.56 Dos cuerpos que se encuentran a la distancia de 153 metros uno del otro, se
mueven al encuentro mutuo. El primero recorre 10 metros por segundo, y el segundo recorrio 3
metros en el primer segundo; en cada segundo siguiente recorre 5 metros mas que en el anterior.
Despus de cuntos segundos los cuerpo se encuentran?
Solucin
Supongamos que el encuentro se produce despus de x segundos, en tal caso el primer cuerpo
recorrio un camino igual a 10x, el segundo cuerpo recorrio un camino igual a la suma de los
terminos de la progresin aritmtica:
S = 3 + (3 + 5) + (3 + 5 2) +... + [3 + 5(x 1)].
Por los datos del problema
10x +S = 153 10x +
5x + 1
2
x = 153
Resolviendo esta ecuacin cuadrtica, hallamos que x = 6.
Ejemplo 3.57 Pueden los nmeros que expresan las longitudes de los lados de un triangulo y
su permetro, formar una progresin aritmtica?
Solucin
Supongamos que las longitudes de los lados forman una progresin aritmtica, en este caso se los
puede designar por a, a + d, a + 2d, siendo su permetro igual a 3a + 3d. La diferencia entre el
permetro y el lado mayor es
(3a + 3d) (a + 2d) = 2a +d
y, puesto que 2a +d > d, el permetro no es el cuarto trmino de la progresin aritmtica.
Ejemplo 3.58 En una progresin aritmtica si los trminos de lugares p, q y r son respecti-
vamente, a, b y c. Demuestre que
(q r)a + (r p)b + (p q)c = 0
Solucin
Por hiptesis se tienen
_

_
a
1
+ (p 1)d = a
a
1
+ (q 1)d = b
a
1
+ (r 1)d = c
de este sistema de ecuaciones, se obtiene:
a
1
d = a pd = b qd = c rd
CAPTULO 3. NMEROS REALES 140
y de aqu
_

_
p q =
1
d
(a b)
q r =
1
d
(b c)
r p =
1
d
(c a)
Multiplicando la primera ecuacin por c, la segunda por a y la tercera por b, se tiene
(q r)a + (r p)b + (p q)c = 0
Ejemplo 3.59 Encuentre la suma de todos los nmeros entre 100 y 1000, que sean divisibles
por 14. Solucin
El primer nmero despus del 100, divisible por 14 es 112, luego a
1
= 112 y d = 14, entonces
a
n
= 112 + (n 1)14 < 1000 n < 64, 43
luego n = 64 con lo que
S
64
= 32[2 112 + 63 14] = 35392.
Ejemplo 3.60 Si la suma de m trminos de una orogresin aritmtica es a la suma de n
trminos, como m
2
es a n
2
. Demuestre que
a
m
a
n
=
2m1
2n 1
Solucin
Como
S
m
S
n
=
m
2
n
2
entonces
m[2a
1
+ (m1)d]
n[2a
1
+ (n 1)d]
=
m
2
n
2
d = 2a
1
por lo tanto
a
m
a
n
=
a
1
+ (m1)d
a
1
+ (n 1)d
=
a
1
+ (m1)2a
1
a
1
+ (n 1)2a
1
=
2m1
2n 1
.
Ejemplo 3.61 En una progresin aritmtica cuyo primer trmino es a, si la suma de los p
primeros trminos es cero, demuestre que la suma de los siguientes q trminos es
a(p +q)q
1 p
Solucin
Por hiptesis tenemos
S
p
=
p
2
[2a + (p 1)d] = 0, p ,= 0 2a + (p 1)d = 0
CAPTULO 3. NMEROS REALES 141
de donde d =
2a
1p
, p ,= 1; por otra parte S = S
p+q
S
p
, S es la suma de los q siguientes trminos,
ahora como S
p
= 0, entonces
S = S
p+q
=
p +q
2
_
2a + (p +q 1)
2a
1 p
_
=
a(p +q)q
1 p
.
Ejemplo 3.62 Si la suma de los primeros p trminos de una progresin aritmtica es q y la
suma de los q primeros trminos es p. Demuestre que la suma de los primeros p + q trminos es
(p +q).
Solucin
Nos dicen que
_
S
p
=
p
2
[2a
1
+ (p 1)d] = q
S
q
=
q
2
[2a
1
+ (q 1)d] = p
resolviendo ste sistema de ecuaciones, obtenemos
_
d =
2(p+q)
pq
a
1
=
q
2
+(p1)(p+q)
pq
por tanto
S
p+q
=
p +q
2
[2a
1
+ (p +q 1)d]
y reemplazando los valores de a
1
y d, obtenemos luego de simplicar, que
S
p+q
= (p +q).
Ejemplo 3.63 En una progresin aritmtica se conoce la suma S
m
de los m primeros trminos
y la suma S
n
de los n primeros trminos. Calcular la diferencia de la progresin aritmtica.
Solucin
De inmediato
_
S
m
=
m
2
[2a
1
+ (m1)d]
S
n
=
n
2
[2a
1
+ (n 1)d]
de donde
_
2nS
m
= 2nma
1
+n(m1)d
2mS
n
= 2nma
1
+m(n 1)d
sumando miembro a miembro resulta
2(mS
n
nS
m
) = dmn(mn) d =
2(mS
n
nS
m
)
mn(mn)
, m ,= n.
Ejemplo 3.64 Si log
k
x, log
m
x, log
n
x estn en progresin aritmtica, demuestre que
n
2
= (kn)
log
k
m
Solucin
Como log
k
x, log
m
x, log
n
x estn en progresin aritmtica, entonces
log
m
x log
k
x = log
n
x log
m
x
CAPTULO 3. NMEROS REALES 142
llevando a base 10 se tiene
2 log x
log m
=
log x
log k
+
log x
log n
2 log k log n = log mlog n + log mlog k
log n
2
= log
k
m(log n + log k) n
2
= (kn)
log
k
m
Ejemplo 3.65 Una persona debe pagar una deuda de $ 360000 en 40 cuotas que forman una
progresin aritmtica cuando 30 de los pagos estn cubiertos la persona fallece, dejando la tercera
parte de la deuda sin pagar. Calcule el valor del primer pago.
Solucin
Sean a
1
y d el primer trmino y la diferencia de la progresin aritmtica en cuestin, entonces
_
S
40
= 20[2a
1
+ 39d] = 360000
S
30
= 15[2a
1
+ 29d] =
2
3
360000
de donde resolviendo el sistema de ecuaciones, se obtiene d = 200 y a
1
= 5100.
Supongamos que a
k1
, a
k
, a
k+1
son tres trminos sucesivos de una progresin aritmtica. En
tal caso, por propiedad de la progresin tendremos:
a
k
a
k1
= a
k+1
a
k
2a
k
= a
k1
+a
k+1
a
k
=
a
k1
+a
k+1
2
.
Denicin 3.16 Media aritmtica
Se llama media aritmtica la semisuma de dos nmeros; por lo tanto, cualquier trmino de una
progresin aritmtica (excepto el primero) es la media aritmtica de dos de sus trminos contiguos.
Ejemplo 3.66 Intercalar 7 medias aritmticas entre los numeros 8 y 20.
Solucin
Esto signica que se deben hallar 7 nmeros tales que junto con los nmeros dados 8 y 20 formen
una progresin aritmtica; el primer trmino de esta progresin es el 8, el noveno, el nmero 20.
Tendremos que
a
9
= a
1
+ 8d 20 = 8 + 8d d = 1,5.
La progresin buscada ser:
8; 9,5; 11; 12,5; 14; 15,5; 17; 18,5; 20.
Ejemplo 3.67 Dada la progresin aritmtica 35x, ..., 3x; x R, x ,= 0. Calcular a
n
sabiendo
que existen 17 trminos entre los extremos.
Solucin
De inmediato a
1
= 35x y a
19
= 3x, entonces
35x + 18d = 3x d =
19
9
x
por tanto
a
n
= 35x + (n 1)
19
9
x.
Ejemplo 3.68 Hallar la relacin entre x e y, de manera que el medio aritmtico de lugar r,
entre x y 2y, sea el mismo que el medio aritmtico de lugar r entre 2x e y. Habiendo n medios
aritmticos interpolados en cada caso.
CAPTULO 3. NMEROS REALES 143
Solucin
Para el primer caso:
2y = x + (n + 1)d
1
d
1
=
2y x
n + 1
a
r
= x +rd
1
para el segundo caso
y = 2x + (n + 1)d
2
d
2
=
y 2x
n + 1
b
r
= 2x +rd
2
Ahora por hiptesis a
r
= b
r
de donde
x +r
2y x
n + 1
= 2x +r
y 2x
n + 1
x(n r + 1) = yr
Denicin 3.17 Progresin geomtrica
Una sucesin de trminos es una progresin geomtrica si la razn de cada trmino anterior es
siempre la misma. Esta razn constante se denomina razn comn de la progresin geomtrica.
Cada trmino de una progresin geomtrica se obtiene multiplicando al anterior por la razn
comn. Si b es el primer trmino y r es la razn comn, los trminos sucesivos de la progresin
geomtrica son
b, br, br
2
, br
3
, ...
En esta progresin geomtrica, observamos que la potencia de r en cualquier trmino es menor en
uno a la anterior. As que, el n-simo trmino est dado por t = br
n1
Teorema 3.33 Si b es el primer trmino y r la razn comn de una progresin geomtrica,
entonces la suma S
n
de n-trminos de la progresin geomtrica est dada por
S
n
=
b(1 r
n
)
1 r
, r ,= 1.
Demostracin
Los n-trminos de la progresin geomtrica dada son
b, br, br
2
, br
3
, ..., br
n2
, br
n1
.
Por tanto, la suma de estos trminos es
S
n
= b +br +br
2
+br
3
+... +br
n2
+br
n1
Multiplicamos ambos lados por r, y obtenemos
rS
n
= br br
2
br
3
br
4
... br
n1
br
n
Sumando estas dos ecuaciones, advertimos que todos los trminos se cancelan excepto el primer
trmino de la primera ecuacin y el ltimo trmino de la segunda ecuacin, lo que resulta
S
n
rS
n
= b br
n
(1 r)S
n
= b(1 r
n
) S
n
=
b(1 r
n
)
1 r
.
Multiplicando el numerador y el denominador de la ecuacin por -1, obtenemos la frmula alter-
nativa S
n
=
b(r
n
1)
r1
. Esta frmula por lo general se usa cuando r > 1, mientras que la ecuacin
S
n
=
b(1r
n
)
1r
es ms til cuando r < 1. La frmula S
n
=
b(r
n
1)
r1
es vlida slo cuando r ,= 1. Si
n = 1, la progresin geomtrica se transforma en b +b +b +... +b
. .
n trminos
cuya suma es igual a nb.
CAPTULO 3. NMEROS REALES 144
Ejemplo 3.69 Encuentre el dcimo tercer trmino de la sucesin 3, 6, 12, 24, ...
Solucin
Como esta sucesin es una progresin geomtrica, entonces r = 2. Por lo tanto, el dcimo tercer
trmino ser
t = (3)(2)
131
= 12288.
Ejemplo 3.70 Encuentre el n-simo trmino de la sucesin
2
9
,
1
3
,
1
2
, ...
Solucin
Como r =
3
2
, por lo tanto se trata de una progresin geomtrica y el n-simo trmino estar dado
por
t =
_
2
9
__

3
2
_
n1
=
(1)
n1
3
_
3
2
_
n2
.
Ejemplo 3.71 El segundo y quinto trmino de una progresin geomtrica son 24 y 81, respec-
tivamente. Determine la sucesin y el dcimo trmino.
Solucin
El segundo y quinto trminos quedan determinados por ar = 24 y ar
4
= 81 respectivamente.
Igualando estas dos ecuaciones, obtenemos que r =
3
2
y a = 16, por lo tanto el trmino genrico es
l = (16) (3/2)
n1
=
_
32
3
_ _
3
2
_
n
y el dcimo trmino es
19683
32
.
Ejemplo 3.72 Determine la suma de:
a) S
n
=
n

i=1
1
(5

13
)
i
; b) S
n
=
n

i=2
(1)
i
_
_
3
5
_
i
.
Solucin
a) Desarrollando el smbolo de sumatoria, obtenemos
S
n
=
1
(5

13
) +
1
(5

13
)
2
+
1
(5

13
)
3
+... +
1
(5

13
)
n
de donde podemos calcular r =
1
5

13
, lo cual indica que se trata de una progresin geomtrica y,
de esta manera podemos encontrar la suma pedida
S
n
=
(5

13)
n
1
(4

13)(5

13)
n
.
b) Como
S
n
=
n

i=1
(1)
i
_
_
3
5
_
i

i=1
(1)
i
_
_
3
5
_
i
=
_
3
5
+
_
_
3
5
_
2

_
_
3
5
_
3
+... + (1)
n
_
_
3
5
_
n

_
3
5
_
podemos encontrar que r =
_
3
5
lo cual nos indica que se trata de una progresin geomtrica y
de esta manera encontramos el valor de la identidad pedida:
S
n
=

_
3
5
_
1
_

_
3
5
_
n
_
1 +
_
3
5
+
_
3
5
=
(1)
n
(

3)
n+1
+ 3(

5)
n1
(

5)
n
(

5 +

3)
.
CAPTULO 3. NMEROS REALES 145
Ejemplo 3.73 La suma de los 6 primeros trminos de una progresin geomtrica es 9 veces
la suma de los tres primeros trminos, determine su razn. (a
1
,= 0, r ,= 1)
Solucin
Como
S
6
= 9S
3
a
1
r
6
1
r 1
= 9a
1
r
3
1
r 1
(r
3
1)(r
3
+ 1) = 9(r
3
1)
como r ,= 1, entonces
r
3
+ 1 = 9 r = 2
Ejemplo 3.74 El producto de tres nmeros en prpgresin geomtrica es 27 y la suma de sus
recprocos es 3. Encuentre tales nmeros.
Solucin
En este caso conviene tomar
a
r
, a, ar como los tres nmeros en progresin geomtrica, por tanto
a
r
a ar = 27 a = 3
luego
r
3
+
1
3
+
1
3r
= 3 r
2
8r + 1 = 0 r = 4

15
y los nmeros son
1
4

15
, 3, 3
_
4

15
_
Ejemplo 3.75 En una progresin geomtrica si los trminos de lugares p, q y r son respecti-
vamente: a, b y c. Demuestre que
a
qr
b
rp
c
pq
= 1
Solucin
Sea x el primer trmino e y la razn de la progresin geomtrica, luego
xy
p1
= a, xy
q1
= b, xy
r1
= c
de donde obtenemos
_

_
a
qr
= x
qr
y
(p1)(qr)
b
rp
= x
rp
y
(q1)(rp)
c
pq
= x
pq
y
(r1)(pq)
multiplicando miembro a miembro, nalmente obtenemos
a
qr
b
rp
c
pq
= 1
Ejemplo 3.76 Calcular la suma
2 +
a +b
ab
+
a
2
+b
2
a
2
b
2
+... +
a
n
+b
n
a
n
b
n
Solucin
Reordenando la suma, obtenemos
S = 1 +
1
a
+
1
a
2
+... +
1
a
n
+ 1 +
1
b
+
1
b
2
+... +
1
b
n
=
_
1
a
_
n+1
1
1
a
1
+
_
1
b
_
n+1
1
1
b
1
=
a
n+1
1
a
n
(a 1)
+
b
n+1
1
b
n
(b 1)
.
CAPTULO 3. NMEROS REALES 146
Ejemplo 3.77 Si a, b, c, d estn en progresin geomtrica, demuestre que
(b c)
2
+ (c a)
2
+ (d b)
2
= (a d)
2
Solucin
Como a, b, c, d estn en progresin geomtrica, entonces
b
a
=
c
b
=
d
c

_

_
b
2
= ac
c
2
= bd
bc = ad
Ahora
(b c)
2
+ (c a)
2
+ (d b)
2
= 2b
2
+ 2c
2
+a
2
+d
2
2ac 2bc 2bd
= 2ac + 2bd +a
2
+d
2
2ac 2ad 2bd
= a
2
+d
2
2ad
= (a d)
2
Ejemplo 3.78 Encuentre la suma de n trminos de la sucesin cuyo k-simo trmino es
a
k
= (2k + 1)2
k
Solucin
Como
S
n
=
n

k=1
(2k + 1)2
k
2S
n
=
n

k=1
(2k + 1)2
k+1
de donde restando miembro a miembro estas sumas, se tiene
2S
n
S
n
=
n

k=1
(2k + 1)2
k+1

k=1
(2k + 1)2
k
entonces
S
n
= (2n + 1)2
n+1
+
n1

k=1
(2k + 1)2
k+1

k=2
(2k + 1)2
k
3 2
= (2n + 1)2
n+1
+
n1

k=1
(2k + 1)2
k+1

n1

k=1
(2k + 3)2
k+1
3 2
= (2n + 1)2
n+1
+
n1

k=1
(2)2
k+1
6
= (2n + 1)2
n+1

n1

k=1
2
k+2
6
= (2n + 1)2
n+1
8
2
n1
1
2 1
6
= n 2
n+2
2
n+1
+ 2.
CAPTULO 3. NMEROS REALES 147
Ejemplo 3.79 Un ventilador gira a 1200 revoluciones por minuto (rpm). Despus de apagar el
motor del ventilador, ste disminuye gradualmente su velocidad de manera que cada segundo efecta
slo 90 % de las revoluciones del segundo anterior. Cuntas revoluciones efectuar el ventilador
durante el primer minuto, despus de apagarlo?
Solucin
Cuando gira a 1200 rpm, el ventilador girar
1200
60
o 20 revoluciones por segundo. El nmero
de revoluciones por segundo para los segundos posteriores a apagar el ventilador, formarn una
progresin geomtrica donde b
1
= 18 y r = 0,9; entonces,
18, 18(0,9), 18(0,9)
2
, ..., 18(0,9)
n1
Como un minuto tiene 60 segundos, el problema se reduce a encontrar S
60
, lo que se puede lograr
por aplicacin de la frmula para la obtencin se S
n
es una progresin geomtrica:
S
60
=
18(1 0,9
60
)
1 0,9

18
0,1
= 180 revoluciones.
Ejemplo 3.80 Cuatro nmeros forman una progresin geomtrica decreciente. Sabiendo que
la suma de los trminos extremos es igual a 27, y la suma de los trminos medios, igual a 18,
hallar su progresin.
Solucin
Tenemos el sistema
_
a
1
+a
1
q
3
= 27
a
1
q +a
1
q
2
= 9
Dividamos la primera ecuacin por la segunda:
1 q +q
2
q
= 3
Resolviendo esta ecuacin cuadrtica, obtendremos q = 2

3. Solamente q = 2

3 satisface
las condiciones del problema dado, puesto que la progresin debe ser decreciente y, por eso [q[ < 1.
El primer trmino de la progresin lo hallamos de la correlacion
a
1
(q +q
23
) = 9 a
1
=
3
2
(9 + 5

3)
Ejemplo 3.81 La suma de tres numeros positivos, que forman una progresin aritmtica, es
igual a 21. Si a estos nmeros les sumamos respectivamente 2, 3 y 9, los nuevos nmeros forman
una progresin geomtrica. Hallar esos nmeros.
Solucin
Supongamos que x, y y z son los nmeros buscados. En tal caso x +y +z = 21, y, puesto que los
nmeros x, y, z forman una progresin aritmtica, tendremos que 2y = x+z. Por las condiciones del
problema x+2, y+3, z +9 componen una progresin geomtrica, es decir, (y+3)2 = (x+2)(z +9).
Se obtuvo el sistema de ecuaciones
_

_
x +y +z = 21
2y = x +z
(y + 3)
2
= (x + 2)(z + 9)
Resolviendo este sistema de ecuaciones, obtenemos que los nmeros buscados son 3, 7 y 11.
CAPTULO 3. NMEROS REALES 148
Ejemplo 3.82 Calcular los ngulos de un cuadriltero sabiendo que estos ngulos estn en
progresin geomtrica y que el ngulo mayor es 9 veces el segundo.
Solucin
Supongamos r > 1, entonces los ngulos son a, ar, ar
2
y ar
3
tales que a < ar < ar
2
< ar
3
y
9ar = ar
3
, de donde r = 3. Por otra parte de la geometra elemental sabemos que
a +ar +ar
2
+ar
3
= 360

a + 3a + 9a + 27a = 360

a = 9

luego los ngulos resultan ser: 9

, 27

, 81

y 243

. Si se supone r < 1, r =
1
3
y a = 243

y resultan
los mismos ngulos.
Ejemplo 3.83 En un cuadrado de lado a se inscribe otro cuadrado cuyos vrtices dividen los
lados del primer cuadrado en la razn 1 : 1. En el segundo cuadrado se inscribe un tercer cuadrado
que divide a los lados del anterior en la misma razn y as sucesivamente. Encontrar la suma de
los permetros y reas de n de estos cuadrados, cules son estas sumas si n .
Solucin
Permetro:
P
1
= 4a, P
2
= 4

2
2
a, P
3
= 4
_

2
2
_
2
a, ..., P
n
= 4
_

2
2
_
n1
a
S
P
n
= 4a
_
_
1 +

2
2
+
_

2
2
_
2
+... +
_

2
2
_
n1
_
_
= 4a
1
_

2
2
_
n1
1

2
2
Si n , entonces
S
P
=
8
2

2
a.
Area:
A
1
= a
2
, A
2
=
_

2
2
_
2
a
2
, A
3
=
_

2
2
_
4
a
2
, ..., A
n
=
_

2
2
_
2(n1)
a
2
S
A
n
= a
2
_
1
_
1
2
_
n
1
1
2
_
si n , entonces S
A
= 2a
2
.
Ejemplo 3.84 Se deja caer una pelota de goma desde una altura h, en el primer rebote la
pelota sube hasta el tercio de la altura h, en el segundo rebote sube hasta el tercio de la nueva
altura y as sucesivamente. Calcule la distancia que recorre la pelota antes de detenerse.
Solucin
Se debe tener que
H = h +
1
3
h +
1
3
_
1
3
h
_
+
1
3
_
1
3
_
1
3
h
__
+...
= h
_
1 +
1
3
+
_
1
3
_
2
+
_
1
3
_
3
+...
_
Se trata de una serie geomtrica de razn r =
1
3
< 1, por tanto la suma de innitos trminos ser
H = h
1
1
1
3
=
3
2
h.
CAPTULO 3. NMEROS REALES 149
Supongamos que a, a, a son tres terminos consecutivos de una progresin geomtrica, donde el
subndice k es un numero natural cualquiera mayor que 1. En tal caso tendremos:
a
k
a
k1
=
a
k+1
a
k
cada una de estas relaciones es igual a la razon de la progresin q. Por una propiedad de la
progresin tendremos:
a
2
k
= a
k1
a
k+1
.
Denicin 3.18 Media geomtrica
El nmero cuyo cuadrado es igual al producto de dos nmeros dados, se llama su media geomtri-
ca. Es decir, todo trmino de una progresin geomtrica es la media geomtrica de dos trminos
equidistantes a l.
Ejemplo 3.85 Intercalar entre los nmeros 2 y 1458 cinco medias geomtricas.
Solucin
La condicion del problema es: hallar cinco nmeros tales que junto con los nmeros dados 2 y
1458 formen una progresin geomtrica cuyo primer trmino sea a
1
= 2 y el sptimo trmino sea
a
7
= 1458. Tendremos que
a
7
= a
1
q
6
1458 = 2q
6
729 = q
6
q =
6

729 = 3
Son posibles dos progresiones:
2, 6, 18, 54, 162, 486, 1458 o 2, 6, 18, 54, 162, 486, 1458.
3.19. Tarea
1. Sea a
n
una progresin aritmtica en la que todos sus trminos y su diferencia d son
distintos de cero. Demustrese que son vlidas la igualdades siguientes:
a) a
n
=
a
n1
+a
n+1
2
, n 2 N;
b) a
k
+a
nk+1
= a
1
+a
n
, k = 1, 2, ..., n;
c)
n

k=1
1
a
k
a
k+1
=
1
d
_
1
a
1

1
a
n+1
_
;
d)
n

k=1
1
a
k
a
k+1
a
k+2
=
1
2d
_
1
a
1
a
2

1
a
n+1
a
n+2
_
;
e)
n

k=1
1
a
k
a
k+1
a
k+2
a
k+3
=
1
3d
_
1
a
1
a
2
a
3

1
a
n+1
a
n+2
a
n+3
_
.
2. En una progresin aritmtica cuyo primer trmino es 4 y el orden n, 34. Si la suma de los
n primeros trminos es 247, determine n y la diferencia d.
Resp: 13 y
5
2
.
3. Sumar 19 trminos de la sucesin
3
4
,
2
3
,
7
12
,...
Resp: 0.
CAPTULO 3. NMEROS REALES 150
4. Interpolar 9 medios aritmticos entre
1
4
y
39
4
.
Resp:
3
4
,
7
4
,
11
4
,
15
4
,
19
4
,
23
4
,
27
4
,
31
4
,
35
4
.
5. Sumar 25 trminos de la sucesin
3

5
,
4

5
,

5,...
Resp: 15

5.
6. La suma de 4 nmeros enteros de una progresin aritmtica es 24 y su producto es 945.
Hallar los nmeros.
Resp: 3, 5, 7 y 9.
7. Encontrar la suma de todos los nmeros entre 14 y 84 inclusive extrayendo los mltiplos
de 3.
Resp: 1152.
8. Dados tres nmeros en progresin aritmtica con diferencia d, d N; se sabe que uno de
ellos es mltiplo de d. Demostrar que el producto de ellos es divisible por 6d
3
.
9. Si a, b, c estn en progresin aritmtica y f(x) = px +q en que f : R R es una funcin
con p ,= 0. Demuestre que f(a), f(b), f(c) tambin estn en progresin aritmtica.
10. En la ecuacin x
4
(3m + 4)x
3
+ (m + 1)
2
= 0 determine m tal que sus races estn en
progresin aritmtica.
Resp: m = 2.
11. Si la suma de m trminos de una progresin aritmtica es igual a la suma de los siguientes
n trminos y tambin a la suma de los siguientes p trminos, entonces demuestre que
(m+n)
_
1
m

1
p
_
= (m+p)
_
1
m

1
n
_
12. La suma de cinco trminos en una progresin aritmtica es 20 y el producto entre el mayor
y el menor es -20. Cules son los trminos?.
Resp: -2, 1, 4, 7, 10 o bien 10, 7, 4, 1, -2.
13. Demuestre que la suma de un nmero impar de trminos consecutivos de una progresin
aritmtica es igual al trmino central multiplicado por el nmero de trminos.
14. Una sucesin a
1
, a
2
, ..., a
n
satisface la igualdad

n
k=1
a
k
= 3n
2
+ 2n. Demuestre que la
sucesin es una progresin aritmtica y encuentre una expresin para a
n
en trminos de n
nicamente.
Resp: a
n
= 6n 1.
CAPTULO 3. NMEROS REALES 151
15. Si en una progresin aritmtica la suma de los m primeros trminos es igual a la suma de
los n primeros trminos, demostrar que la suma de los m+n trminos es nula.
16. En un tringulo rectngulo los lados estn en progresin aritmtica. Demostrar que la
diferencia de la progresin es igual al radio de la circunferencia inscrita al tringulo.
17. La suma de tres nmeros en progresin aritmtica es 9 y la suma de sus recprocos es nula.
Determine la suma de los 20 primeros trminos de esta progresin aritmtica.
Resp: 30
_
2 17

3
_
.
18. Una persona contrae una deuda que debe pagar en tres aos en cuotas mensuales que se
incrementan cada mes en una cantidad ja. Si al trmino de los dos primeros aos la persona
ha pagado la mitad de la deuda y la primera cuota del tercer ao es de $ 122000. Determine
el total que la persona paga al nal de los tres aos.
Resp: $ 3456000.
19. Demustrese que si los nmeros positivos a, b, c son trminos consecutivos de una progresin
aritmtica, los nmeros
1

b +

c
,
1

c +

a
,
1

a +

b
tambin son trminos consecutivos de la progresin aritmtica.
20. Demustrese que si los nmeros positivos a
1
, a
2
, ..., a
n
son los trminos consecutivos de
una progresin aritmtica, entonces:
1

a
1
+

a
2
+
1

a
2
+

a
3
+... +
1

a
n1
+

a
n
=
n 1

a
1
+

a
n
21. Sea S
n
la suma de los primeros n trminos de una progresin aritmtica. Demustrese que:
a) S
n+3
= 3S
n+2
3S
n+1
+S
n
; b) S
3n
= 3(S
2n
S
n
).
22. Comprubese o reftese la siguiente aseveracin: si t
1
, t
2
, t
3
, t
4
, es una progresin arit-
mtica nita, entonces Cost
1
, Cost
2
, Cost
3
, Cost
4
, tambin es una progresin aritmtica.
23. Si el costo de un automvil sufre una depreciacin anual de 12 %, cul ser su valor de-
spus de 5 aos, si el precio original del mismo era de 8600 dlares? (Sugerencia: El valor al
trmino de cada ao es 88 % del valor al trmino anterior.)
24. El movimiento de una clase especca de hormigas depende de la temperatura. Aparente-
mente, las hormigas duplican su velocidad de desplazamiento por cada 10
o
C de aumento en
la temperatura. Si una hormiga se desplaza a una velocidad de 60 cm/min cuando la tem-
peratura ambiente es 10
o
C, cul ser la velocidad de desplazamiento a 40
o
C?, a 50
o
C?
CAPTULO 3. NMEROS REALES 152
25. Se considera que una estrella de magnitud 6 emite una unidad de luz, mientras que una
estrella de magnitud 5 emitir 2.5 veces la luz de una estrella de magnitud 6, y una estrella
de magnitud 4 emitir 2.5 veces la luz de una estrella de magnitud 5, y as sucesivamente.
Cuntas unidades de luz emitir una estrella de magnitud 2? Y una estrella de magnitud 3?
26. La poblacin mundial en 1970 se estim en 3.7 x 109 habitantes. La tasa de crecimien-
to anual aproximada es 2 %. Suponiendo que la tasa se mantendr constante, estmese la
poblacin mundial en los aos 1990 y 2000.
27. Si cada coneja da a luz tres conejitos, cuntos conejos de la octava generacin sern de-
scendientes de una coneja en la primera generacin?
28. Una pelota de hule que cae desde una altura de 3 metros siempre rebotar un tercio de la
distancia de la cual cay previa al rebote. determnese la altura alcanzada en el quinto rebote.
29. El peldao inferior de una escalera de 18 peldaos mide 45 cm. La longitud de cada pel-
dao es 1.6 cm. ms corto que el anterior, en el sentido ascendente. Con ayuda de las teclas
de sumando constante o constante automtica de una calculadora electrnica manual, desar-
rllese una tabla que muestre la longitud de todos los peldaos.
30. Un medio de cultivo se siembra con n bacterias. Si el nmero de bacterias se duplica cada
2 horas, obtngase el nmero de bacterias existentes en el cultivo despus de 24 horas.
31. En 1791, Benjamin Franklin don 4000 dlares para ser empleados en prstamos a artesanos
casados que necesitaban ayuda econmica. Durante 100 aos, este dinero estuvo sometido a
un inters compuesto de 5,6 % anual. Calclese el valor aproximado del fondo en 1891.
32. Sea S
n
la suma de los primeros n miembros de una progresin geomtrica. Demustrese
que
S
n
(S
3n
S
2n
) = (S
2n
S
n
)
2
.
33. Demustrese que la sucesin b
n
de nmeros diferentes de cero es una progresin ge-
omtrica si y slo si con cada n 3 se verica la igualdad
(b
2
1
+b
2
2
+... +b
2
n1
)(b
2
2
+b
2
3
+... +b
2
n
) = (b
1
b
2
+b
2
b
3
+... +b
n1
b
n
)
2
.
34. La suma de tres nmeros en progresin geomtrico es 70, si los extremos son amplica-
ciones por 4 y el del medio por 5, la serie est en progresin aritmtica. Hallar los nmeros.
35. Hallar una progresin aritmtica cuyo primer trmino es 1, y tal que los trminos de lugares
2, 10 y 34 se encuentran en progresin geomtrica.
36. Si
1
ba
,
1
2b
,
1
bc
estn en progresin aritmtica, demostrar que a, b, c estn en progresin
geomtrica.
CAPTULO 3. NMEROS REALES 153
37. Si m es el producto de n nmeros en progresin geomtrica, p su suma y q la suma de sus
recprocos, demuestre que
m
2
=
_
p
q
_
n
38. Sea k, (k ,= 0), un nmero dado. Encontrar los nmeros a, b, c sabiendo que a, b, c estn
en progresin geomtrica; a, b +k, c estn en progresin aritmtica y a +k, b +k, c estn en
progresin geomtrica.
39. Si el medio aritmtico entre a y b es el doble que el medio geomtrico entre a y b, demuestre
que
a
b
=
2 +

3
2

a
b
=
2

3
2 +

3
40. En una progresin geomtrica de 5 trminos, la razn es la cuarta parte del primer trmino
y la suma de los dos primeros trminos es 24. Hallar tales trminos.
Resp: 8, 16, 32, 64, 128 o bien -12, 36, -108, 324, -972.
41. La suma de los primeros cinco trminos de una progresin geomtrica es 422, y la suma de
los trminos segundo al sexto es 633. Determine la progresin geomtrica.
Resp: 32, 48, 72, 108 162.
42. Dividir el nmero 221 en tres partes que formen una progresin geomtrica de modo que
el tercer nmero sobrepase al primero 136.
Resp: 17, 51, 153.
43. Si a, b, c estn en progresin geomtrica y f(x) = e
x
en que f : R R es una funcin.
Demuestre que f(a), f(b), f(c) tambin estn en progresin geomtrica.
44. La suma de k nmeros de una progresin geomtrica de razn 2 es 1533 y el ltimo tr-
mino es 768. Determine los k nmeros t luego calcule la suma de 10 primeros trminos de la
progresin geomtrica.
Resp: k = 9, a
1
= 3, S
10
= 3069.
45. Si cada trmino de una progresin geomtrica se resta del trmino siguiente, demuestre
que las diferencias sucesivas forman otra progresin geomtrica con la misma razn que la
primera progresin geomtrica.
46. Si a
1
= 0, a
2
= 1, ..., a
n
=
1
2
(a
n1
a
n2
); demuestre que
a
n
=
2
3
_
1
_

1
2
_
n1
_
47. Demuestre que, si 2u
1
= a +b, 2u
2
= b +u
1
, 2u
3
= u
1
+u
2
, ... entonces
3u
n
= a
_
1
_

1
2
_
n
_
+b
_
2 +
_

1
2
_
n
_
CAPTULO 3. NMEROS REALES 154
48. Si S es la suma de n nmeros en progresin geomtrica y S es la suma de los recprocos
de dichos nmeros, entonces S : S es el producto del primer nmero por el ltimo.
49. Si S
1
, S
2
, ..., S
p
son las sumas de las series geomtricas de primeros trminos 1, 2, ..., p
respectivamente y de razones
1
2
,
1
3
, ...,
1
p+1
respectivamente. Demuestre que
S
1
+S
2
+... +S
p
=
1
2
p(p + 1)
Captulo 4
Expresiones algebraicas
4.1. Expresin numrica
Con ayuda de los nmeros, los signos de operaciones y del parntesis se componen diferentes
expresiones numricas.
Denicin 4.1 Valor numrico
Si en una expresin numrica se pueden realizar todas las operaciones indicadas en ella, el nmero
real, obtenido como resultado de las operaciones cumplidas, se denomina valor numrico de la
expresin numrica dada.
En lugar de las expresiones numricas resulta a menudo, ms cmodo analizar las expresiones,
en las cuales en algunos lugares guran letras en vez de nmeros. Toda expresin de esta ndole se
denomina expresin matemtica.
Denicin 4.2 Expresin algebraica
La expresin matemtica en la cual con los nmeros y las letras se realizan operaciones de adi-
cin, sustraccin, multiplicacin, divisin, elevacin a potencia natural y extraccin de una raz
aritmtica, recibe el nombre de expresin algebraica.
Denicin 4.3 Expresin algebraica racional
Una expresin algebraica se llama racional, si participan en ella slo las operaciones de adicin,
multiplicacin, sustraccin, divisin y elevacin a potencia natural. Una expresin racional se llama
entera respecto de la letra dada, si no contiene la operacin de divisin por la letra dada o por una
expresin en que gura esta letra.
La expresin racional fraccionaria respecto de una letra dada es una expresin racional que
contiene la operacin de divisin por cierta expresin en la que gura esta letra.
Denicin 4.4 Expresin algebraica irracional
Una expresin algebraica se denomina irracional, si en ella se prev la operacin de extraccin de
una raz aritmtica respecto de las letras que la integran.
Sean dadas dos expresiones algebraicas que se denotan con las letras A y B. Denamos para
ellas las operaciones aritmticas.
Denicin 4.5 Suma de expresiones algebraicas
Adicionar dos expresiones algebraicas A y B signica escribir formalmente la expresin algebraica
A+B, denominada suma de las expresiones A y B.
155
CAPTULO 4. EXPRESIONES ALGEBRAICAS 156
Ejemplo 4.1 Sean A = 2a b y B = a 3b +c, entonces
A+B = (2a b) + (a 3b)
= 3a 4b +c.
Denicin 4.6 Producto de expresiones algebraicas
Multiplicar dos expresiones algebraicas A y B signica escribir formalmente la expresin algebraica
AB denominada producto de las expresiones A y B.
Ejemplo 4.2 Dadas A = 5a 3b y B = 3a + 2b 5c, entonces
AB = (5a 3b)(3a + 2b 5c)
= 15a
2
+ 10ab 25ac + 9ab 6b
2
+ 15bc
= 15a
2
+ 19ab 25ac + 15bc 6b
2
.
Si hay necesidad de adicionar varias expresiones algebraicas, se suman primeramente las dos
primeras expresiones y luego a la suma obtenida se le adiciona la tercera expresin, etc. De modo
anlogo se dene tambin el producto de varias expresiones algebraicas. Si en un producto una
misma expresin algebraica A interviene como factor n veces (n > 1, n N), se escribe A
n
en
lugar del producto A A ... A
. .
n veces
.
Denicin 4.7 Diferencia de expresiones algebraicas
Sustraer de una expresin algebraica A otra expresin algebraica B signica escribir formalmente
la expresin algebraica AB, llamada diferencia de las expresiones A y B.
Ejemplo 4.3 Sean A = 9a + 4b +c y B = 5a + 3b c +d, entonces
AB = (9a + 4b +c) (5a + 3b c +d)
= 4a +b + 2c d.
Denicin 4.8 Divisin de expresiones algebraicas
Dividir una expresin algebraica A por otra expresin algebraica B signica escribir formalmente la
expresin algebraica AB, denominada cociente de la divisin de la expresin A por la expresin
B.
Ejemplo 4.4 Sean A = a 2b +c y B = 2a b 2c d, entonces
A
B
=
a 2b +c
2a b 2c d
.
Ejemplo 4.5 Simplique la expresin:
3
_
a

1
2
b
2

_
a

5
2
b
4
_

1
3

_
a
2
b

1
3
.
Solucin
A =
3
_
1
a
1
2

1
b
2

_
1
a
5
2

1
b
4
_

1
3

_
1
a
2

1
b
1
3
=
1
a
1
6

1
b
2
3

_
a
5
2
b
4
_1
3

1
a

1
b
1
6
=
1
a
7
6

1
b
5
6
a
5
6
b
4
3
=

b
3

a
.
CAPTULO 4. EXPRESIONES ALGEBRAICAS 157
Ejemplo 4.6 Simplique la expresin:

3a

7
2
b
8
a
2
3
b

1
2

4a
10
b
6

1
a

3
2
b
3
.
Solucin
A =

_3b
8
b
1
2
a
7
2
a
2
3

4b
6
a
10

a
3
2
a
3
=

_
3b
17
2
a
25
6

4b
3
a
17
2
=
_
3b
17
2
a
25
6

_
4b
3
a
17
2
_
1
4
_
1
2
=
3
1
2
b
17
4
a
25
6

_
4b
3
a
17
2
_
1
8
=
3
1
2
b
17
4
a
25
6

4
1
8
b
3
8
a
17
16
=
3
1
2
2
1
4
b
37
8
a
151
48
=
4

18b
37
48

a
151
.
La sustitucin de una expresin analtica por otra idnticamente igual a ella en cierto conjunto,
lleva el nombre de transformacin idntica en este conjunto de la expresin dada.
Al realizar transformaciones idnticas de una expresin es posible la variacin de su dominio.
La variacin del dominio de la expresin es tambin posible como resultado de ciertas otras
transformaciones, por lo que, despus de efectuar la transformacin de la expresin dada, siempre
hay que saber responder a la pregunta en qu conjunto ella es idntica a la obtenida.
Una expresin algebraica lleva el nombre de racional si ella slo contiene operaciones de sumar,
multiplicar, restar, dividir y elevacin a una potencia entera.
4.2. Tarea
1. Simplique la expresin:
a)
_
1
x
2
+ 3x + 2
+
2x
x
2
+ 4x + 3
+
1
x
2
+ 5x + 6
_
2

(x 3)
2
+ 12x
2
;
b)
x
2
x y)(x z)
+
y
2
(y z)(y x)
+
z
2
(z x)(z y)
.
2. Demuestre que si x +y +z = 0, entonces x
3
+y
3
+z
3
= 3xyz.
CAPTULO 4. EXPRESIONES ALGEBRAICAS 158
3. Demuestre que si x +y +z = 0, donde x ,= 0, y ,= 0, z ,= 0, entonces
_
x y
z
+
y z
x
+
z x
y
__
z
x y
+
x
y z
+
y
z x
_
= 9.
4. Simplique las expresiones racionales:
a)
5x
2
x 4
x
3
1
;
b)
x
6
+x
4
+x
2
+ 1
x
3
+x
2
+x + 1
;
c)
x
4
+x
2
2
x
6
+ 8
;
d)
x
4
x
2
12
x
4
+ 8x
2
+ 15
;
e)
2x
4
+ 7x
2
+ 6
3x
4
+ 3x
2
6
;
f )
5x
4
+ 5x
2
3x
2
y 3y
x
4
+ 3x
2
+ 2
;
g)
x
4
+x
2
y
2
+y
4
x
6
y
6
;
h)
2x
2
+xy y
2
x +y
;
i)
x
4
10x
2
+ 169
x
2
+ 6x + 13
.
Resp: a)
5x + 4
x
2
+x + 1
; b)
x
4
+ 1
x + 1
; c)
x
2
1
x
2
2x
3
+ 4
; d)
x
2
4
x
2
+ 5
;
e)
2x
2
+ 3
3x
2
3
; f )
5x
2
3y
x
2
+ 2
; g)
1
x
2
y
2
.
5. Simplique las expresiones racionales:
a)
1
1 x

1
1 +x

2x
1 +x
2

4x
3
1 +x
4

8x
7
1 +x
8
;
b)
1
1 x
+
1
1 +x
+
2
1 +x
2
+
4
1 +x
4
+
8
1 +x
8
+
16
1 +x
1
6
;
c)
1
x(x + 1)
+
1
(x + 1)(x + 2)
+
1
(x + 2)(x + 3)
+
1
(x + 3)(x + 4)
+
1
(x + 4)(x + 5)
;
d)
x
x
2
1
+
x
2
+x 1
x
3
x
2
+x 1
+
x
2
x 1
x
3
+x
2
+x + 1

2x
3
x
4
1
;
e)
_
y
x +y
+x
__
x
x y
y
_

_
x
x +y
+y
__
y
x y
x
_
;
f )
1
x
+
1
y +z
1
x

1
y +z

_
1 +
y
2
+z
2
x
2
2yz
_
;
g)
1
(x y)(x z)
+
1
(y z)(y x)
+
1
(z x)(z y)
;
h)
x +y
(y z)(z x)
+
y +z
(z x)(x y)
+
z +x
(x y)(y z)
;
i)
x z
x
2
+xz +z
2

x
3
z
3
x
2
y yz
2

_
1 +
z
x z

1 +z
z
_

z(1 +z) x
yz
;
j)
x
8y
3
+
1
4y
2
x
2
+ 2xy + 2y
2

x
8y
3

1
4y
2
x
2
2xy + 2y
2

1
4y
2
(x
2
+ 2y
2
)
+
1
4y
2
(x
2
2y
2
)
;
k)
x y
x +y
+
y z
y +z
+
z x
z +x
+
(x y)(y z)(z x)
(x +y)(y +z)(z +x)
;
l)
x
3
y xy
3
+y
3
z yz
3
+z
3
x zx
3
x
2
y xy
2
+y
2
z yz
2
+z
2
x zx
2
;
m)
(x
2
y
2
)
3
+ (y
2
z
2
)
3
+ (z
2
x
2
)
3
(x y)
3
+ (y z)
3
+ (z x)
3
;
n)
y z
(x y)(x z)
+
z x
(y z)(y x)
+
x y
(z x)(z y)
;
CAPTULO 4. EXPRESIONES ALGEBRAICAS 159
o)
x
2
(u y)(u z)
(x y)(x z)
+
y
2
(u z)(u x)
(y z)(y x)
+
z
2
(u x)(u y)
(z x)(z y)
.
Resp: a)
16x
15
1 x
16
; b)
32
1 x
32
; c)
5
x(x + 5)
; d)
x
x
2
1
; e) 2x;
f )
(x +y +z)
2
2yz
; g) 0; h) 0; i)
1
x +z
; j)
2x
4
x
8
16y
8
; k) 0; l) x +y +z;
m) (x +y)(y +z)(z +x); n)
2
x y
+
2
y z
+
2
z x
; o) u
2
.
6. Demuestre que si x, y, z R, de la igualdad
(x y)
2
+ (y z)
2
+ (z x)
2
= (x +y 2z)
2
+ (y +z 2x)
2
+ (z +x 2y)
2
se deduce que a = b = c.
7. Demuestre que con x R, (x 1)(x 3)(x 4)(x 6) + 10 es un nmero positivo.
8. Encuentre el menor valor de la expresin (x 1)(x 3)(x 4)(x 6) + 10.
9. Demuestre que si x +y +z = 0,
x
5
+y
5
+z
5
5
=
x
3
+y
3
+z
3
3

x
2
+y
2
+z
2
2
.
10. Demuestre que si x +y +z = 0,
x
7
+y
7
+z
7
7
=
x
5
+y
5
+z
5
5

x
2
+y
2
+z
2
2
.
11. Demuestre que si
a
x
+
b
y
+
c
z
= 1 y
x
a
+
y
b
+
z
c
= 0, entonces
a
2
x
2
+
b
2
y
2
+
c
2
z
2
= 1.
12. Demuestre que si
x
y z
+
y
z x
+
z
x y
= 0, donde x ,= y, x ,= z, y ,= z, entonces
x
(y z)
2
+
y
(z x)
2
+
z
(x y)
2
= 0.
13. Demuestre que si x +y +z = 0, entonces
x
5
(y
2
+z
2
) +y
5
(x
2
+z
2
) +z
5
(y
2
+x
2
) =
(x
3
+y
3
+z
3
)(x
4
+y
4
+z
4
)
2
.
14. Simplique la expresin:
a)
_

ab +b

a +

b
_

_

a

a +

b
+

b
+
2

ab
a b
_
;
CAPTULO 4. EXPRESIONES ALGEBRAICAS 160
b)
a + 2 +

a
2
4
a + 2

a
2
4
+
a + 2

a
2
4
a + 2 +

a
2
4
;
c)
a
1
+b
1
+ 2(

a +

b)
1
_
1

a
+
1

b
_
_
ab a

ab
a +

ab
_
1
;
d)
a
_

a +

b
2b

a
_
1
+b
_

a +

b
2a

b
_
1
_
a +

ab
2ab
_
1
+
_
b +

ab
2ab
_
1
;
e)
1
2
ab

8a
3
b +
1
3
ab

18ab
3
a
2
_
2b
a
b
2
_
2a
b
;
f )
_

a +
ab
2
+c

ab
2
+c
_

_
b

a +b
_
ab
2
+c
_
;
g)
a
1
2
b
1
2
c
1
6

_
c

1
2
a

1
3
b

1
3

5
6
c

2
3
b
5
6
_
;
h)
_
_
a
2
3
b
1
_
2

_
a
2
b
1
_1
2

_
b
2
3
_

3
2
_
2
;
i) ab
3
_
b
a
2
ab
3
_
a
b
2
+
a
b
3

ab
4

b
a
3

a
4
b;
j)
_
3

1 +a
+

1 a
_

_
3

1 a
2
+ 1
_
;
k)
_
a 4b
a +

ab 6b

a 9b
a + 6

ab + 9b
_

1
2
a
1
2
3b
1
2
;
l)
_
a +

a
2
b
2
a

a
2
b
2

a
2
b
2
a +

a
2
b
2
_

4a

a
2
b
2
b
2
;
m)
6

a
_
3(a b)
a
2
3
+a
1
6
b
1
2

a
5
6
a

1
6
b
a
1
2
+b
1
2
_
;
n)
_
a

1
2
b

1
3
a

3
4
b

5
6

a
3
b
5
_2
7
;
o)
a
1
+b
1
a
2
+b
2
c
2
a
2
b
2
+ 2a
1
b
1
_
1
1
a+b+c
_
.
4.3. Potencia con exponente entero
Anteriormente se deni la operacin de elevacin a potencia con exponente natural de cualquier
nmero real. En esta seccin se dan las deniciones de elevacin de un nmero a potencia nula y
a potencia con exponente negativo.
Denicin 4.9 Potencia con exponente natural
Sean a un nmero real cualquiera y n, cualquier nmero natural. Entonces, se denomina potencia
del nmero a con exponente natural n, un nmero que se escribe en la forma a
n
y que se determina
CAPTULO 4. EXPRESIONES ALGEBRAICAS 161
como a
n
= a a ... a
. .
n veces
, si n 2 y a
n
= a, si n = 1. Si a es un nmero cualquiera real distinto de
cero. Se denomina potencia nula de este nmero la unidad, es decir a
0
= 1 para cualquier nmero
real a distinto de cero.
La potencia nula del nmero cero no est denida y el smbolo 0
0
se considera sin sentido.
Denicin 4.10 Potencia con exponente negativo
Sea a un nmero real cualquiera distinto de cero y n, cualquier nmero natural. Se llama potencia
del nmero a con exponente negativo, el nmero a
n
=
1
a
n
para cualquier nmero real a, distinto
de cero, y para todo nmero entero negativo.
La potencia entera negativa del nmero cero no est denida y el smbolo 0
n
se considera sin
sentido.
As pues, la potencia natural se determina para cualquier nmero real, mientras que la potencia
nula y entera negativa se denen slo para cualquier nmero real, distinto de cero.
Si a es un nmero real cualquiera distinto de cero, entonces se puede enunciar la denicin de
potencia con exponente entero, la cual representa la reunin de las deniciones anteriores.
Denicin 4.11 Potencia con exponente entero
Sea a un nmero real cualquiera distinto de cero y k, cualquier nmero entero; entonces, por
nmero a
k
se entiende aquel nmero que se determina como a
k
= a, si k = 1; a
k
= a a ... a
. .
k veces
, si
k es un nmero natural 2; a
k
= 1, si k = 0; a
k
=
1
a
r
, si k es un nmero entero negativo. En
este caso el nmero a
k
se denomina potencia con exponente entero, el nmero a es la base de la
potencia, el nmero k, el exponente de la potencia.
La potencia par de un nmero positivo o negativo es un nmero positivo; la potencia impar de
un nmero positivo es un nmero positivo, la potencia impar de un nmero negativo es un nmero
negativo.
Sean a y b cualesquiera nmeros reales distintos de cero, y sean n y m cualesquiera nmeros
enteros, entonces se cumplen las siguientes propiedades:
Teorema 4.1 Sean a y b cualesquiera nmeros reales distintos de cero, y sea k cualquier nmero
entero, entonces:
(ab)
k
= a
k
b
k
.
Demostracin
La validez de esta propiedad para k natural (k = n, n N) se deduce de las leyes principales de
adicin y multiplicacin de nmeros reales:
(ab)
k
= (ab)
n
= (ab) (ab) ... (ab)
. .
n veces
= a a ... a
. .
n veces
b b ... b
. .
n veces
= a
n
b
n
= a
k
b
k
.
CAPTULO 4. EXPRESIONES ALGEBRAICAS 162
Sea k = 0,
(ab)
k
= (ab)
0
= 1
= 1 1
= a
0
b
0
= a
k
b
k
.
es decir,
(ab)
k
= a
k
b
k
.
Supongamos que k = m, y m es un nmero natural. Por denicin de potencia con exponente
negativo
(ab)
k
= (ab)
m
=
1
(ab)
m
=
1
a
m
b
m
=
1
a
m

1
a
m
= a
m
b
m
= a
k
b
k
.
Teorema 4.2 Sean a y b cualesquiera nmeros reales distintos de cero, y sea k cualquier nmero
entero, entonces:
_
a
b
_
k
=
a
k
b
k
.
Teorema 4.3 Sean a y b cualesquiera nmeros reales distintos de cero, y sean k y r cualesquiera
nmeros enteros, entonces:
a
k
a
r
= a
k+r
.
Demostracin
Con el n de demostrar esta propiedad, examinemos cada uno de los seis casos posibles:
Caso 1. k = n, r = m: cuando k = n, r = m, la validez de esta propiedad, se desprende de las leyes
principales de adicin y multiplicacin de los nmeros reales:
a
k
a
r
= a
n
a
m
= (a a ... a)
. .
n veces
(a a ... a)
. .
m veces
= a a ... a
. .
n+m veces
= a
n+m
= a
k+r
.
Caso 2. k = n, r = m: sea k = n, r = m, donde n y m son nmeros naturales; entonces, por
denicin de potencia con exponente entero negativo, tenemos
a
k
a
r
= a
n

1
a
m
=
a
n
a
m
.
CAPTULO 4. EXPRESIONES ALGEBRAICAS 163
Supongamos que n > m, entonces
a
n
a
m
= a
n
a
m
= a
nm
= a
n+(m)
= a
k+r
.
Sea n = m, entonces, por denicin de potencia con exponente nulo, obtenemos
a
n
a
m
= 1
= a
0
= a
n+(m)
= a
k+r
.
Sea n < m, entonces,
a
n
a
m
=
1
a
m
1
a
n
=
1
a
m
a
n
=
1
a
mn
= a
(mn)
= a
m+n
= a
n+(m)
= a
k+r
.
Caso 3. k = n, r = m: supongamos que k = n, r = m, donde n y m son nmeros naturales.
Este caso es anlogo al caso en que k = n, r = m.
Caso 4. k = n, r = m, n, m N: sea k = n, r = m, donde n y m son nmeros naturales,
entonces,
a
k
a
r
= a
n
a
m
=
1
a
n

1
a
m
=
1
a
m+n
= a
(n+m)
= a
nm
= a
(n)+(m)
= a
k+r
.
Caso 5. k Z, r = 0: sea k un nmero entero cualquiera y sea r = 0, entonces,
a
k
a
r
= a
k
1
= a
k
= a
k+0
= a
k+r
.
CAPTULO 4. EXPRESIONES ALGEBRAICAS 164
Caso 6. k = 0, r Z: supongamos que k = 0 y r es un nmero entero cualquiera, entonces,
a
k
a
r
= 1 a
r
= a
r
= a
0+r
= a
k+r
.
Teorema 4.4 Sean a y b cualesquiera nmeros reales distintos de cero, y sean k y r cualesquiera
nmeros enteros, entonces:
a
k
a
r
= a
kr
.
Demostracin
Para demostrar esta propiedad con k y r naturales (k = n, r = m, n N, m N), examinemos
tres casos:
Caso 1. Si n > m, entonces n = m+s, donde s N:
a
k
: a
r
= a
n
: a
m
=
a
n
a
m
=
(a a a ... a)
. .
m veces
(a a a ... a)
. .
s veces
(a a a ... a)
. .
m veces
= a
s
= a
nm
= a
kr
.
Caso 2. Si n = m, entonces
a
k
: a
r
= a
n
: a
m
=
a
n
a
m
=
(a a a ... a)
. .
m veces
(a a a ... a)
. .
m veces
= 1.
Por denicin, a
0
= 1. Por lo tanto,
a
k
: a
r
= a
n
: a
m
= a
0
= a
nm
= a
kr
.
CAPTULO 4. EXPRESIONES ALGEBRAICAS 165
Caso 3. Si m > n, entonces m = n +t, donde t N:
a
k
: a
r
= a
n
: a
m
=
a
n
a
m
=
(a a a ... a)
. .
n veces
(a a a ... a)
. .
n veces
(a a a ... a)
. .
t veces
=
1
(a a a ... a)
. .
t veces
= a
t
= a
(mn)
= a
nm
= a
kr
.
Cabe sealar que en este caso n m no es un nmero natural.
Teorema 4.5 Sea a cualquier nmero real distinto de cero, y sean k y r cualesquiera nmeros
enteros, entonces:
(a
k
)
r
= a
kr
.
Demostracin
Con el objeto de demostrar esta propiedad, examinemos los seis casos posibles: Caso 1. Supong-
amos que k = n, r = m, donde n y m son nmeros naturales:
(a
k
)
r
= (a
n
)
m
= (a
n
) (a
n
) (a
n
) ... (a
n
)
. .
m veces
= (a a a ... a)
. .
n veces
... (a a a ... a)
. .
n veces
. .
m veces
= a a a ... a
. .
nm veces
= a
nm
= a
kr
.
Caso 2. Supongamos que k = n, r = m, donde n y m son nmeros naturales. Entonces:
(a
k
)
r
= (a
n
)
m
=
1
(a
n
)
m
=
1
a
nm
= a
nm
= a
n(m)
= a
kr
.
CAPTULO 4. EXPRESIONES ALGEBRAICAS 166
Caso 3. Supongamos que k = n, r = m, donde n y m son nmeros naturales. La validez de esta
propiedad se demuestra igual que en el caso de k = n, r = m.
Caso 4. Supongamos que k = n, r = m, donde n y m son nmeros naturales. Entonces:
(a
k
)
r
= (a
n
)
m
=
1
(a
n
)
m
=
1
1
a
nm
= 1
1
a
nm
= a
nm
= a
(n)(m)
= a
kr
.
Caso 5. Supongamos que k es un nmero entero cualquiera y r = 0, entonces:
(a
k
)
r
= (a
k
)
0
= 1
= a
0
= a
k0
= a
kr
.
Caso 6. Supongamos que k = 0 y r es un nmero entero cualquiera, entonces:
(a
k
)
r
= (a
0
)
r
= 1
r
= 1
= a
0
= a
0r
= a
kr
.
Por consiguiente, la propiedad queda demostrada.
Ejemplo 4.7 Simplique la expresin:
_
27
10
5
_
1
9
_
8
3
12
+
_
1
2
_
2
_
1
3
2
_
8
3
8
_

_
41
_
3
2
_
12
_
.
CAPTULO 4. EXPRESIONES ALGEBRAICAS 167
Solucin
A =
_
27
10
5 9
8
3
12
+ 2
2
3
16
3
8
_
:
_
41 3
24
_
=
_
3
30
5 3
28
+ 2
2
3
24
_

_
41 3
24
_
=
3
30
5 3
28
+ 2
2
3
24
41 3
24
=
3
24
_
3
6
5 3
4
+ 2
2
_
41 3
24
=
3
6
5 3
4
+ 2
2
41
=
729 405 + 4
41
=
328
41
= 8.
Ejemplo 4.8 Simplique la expresin:
(2) (3)
17
(3)
16
9
7
15
.
Solucin
A =
2 3
17
3
16
9
7
15
=
(2 3 1)3
16
(3
2
)
7
3 5
=
(6 1)3
16
3
14
3 5
=
3
16
5
3
15
5
= 3.
Ejemplo 4.9 Simplique la expresin:
8(4
2
)
4
3
3
27
2
+ 90 6
3
4
7
(3
2
)
2
24(6
2
)
4
(2
4
)
2
+ 144(2
3
)
4
(9
2
)
2
4
2
.
Solucin
CAPTULO 4. EXPRESIONES ALGEBRAICAS 168
A =
2
3
(2
4
)
4
3
3
(3
3
)
2
+ 3
2
5 2 3
3
2
3
(2
2
)
7
3
4
3 2
3
(3
2
2
2
)
4
2
8
+ 3
2
2
4
2
12
(3
4
)
2
2
4
=
2
3
2
16
3
3
3
6
+ 3
2
5 2 3
3
2
3
2
14
3
4
3 2
3
3
8
2
8
2
8
+ 3
2
2
4
2
12
3
8
2
4
=
2
19
3
9
+ 3
9
5 2
18
2
19
3
9
+ 3
10
2
20
=
2
18
3
9
(2 + 5)
2
19
3
9
(1 + 3 2)
=
7
14
=
1
2
..
Ejemplo 4.10 Calcule el volumen V de un cubo de arista
3
4
metros.
Solucin
El volumen V de un cubo de arista a es V = a
3
. Tenemos que a =
3
4
m, por lo tanto, el volumen
del cubo es
V = a
3
=
_
3
4
m
_
3
=
27
64
m
3
.
Ejemplo 4.11 Escriba con una ecuacin La tercera ley de Kepler que enuncia: El cuadrado
del perido de revolucin de un planeta alrededor del Sol es proporcional al cubo del semieje mayor
de la rbita del planeta.
Solucin
Si T es el periodo y a el semieje mayor, entonces
T
2
= ka
3
donde k es una constante de proporcionalidad.
Ejemplo 4.12 La velocidad de luz es v = 2, 99 10
10 cm
seg
. Calcule la distancia recorrida por la
luz en un da y exprsela en notacin cientca.
Solucin
En un da hay 24 horas, en una hora 60 minutos y en un minuto 60 segundos. Por lo tanto, en un
da hay t = (24)(60)(60) = 86400 segundos, es decir, t = 8, 6400 10
4
segundos.
La distancia d se calcula con la frmula d = vt. En este ejercicio,
d =
_
2, 99 10
10
cm
seg
_
(8, 64 10
4
seg)
con lo que,
d = 25, 8336 10
14
cm
= 2, 58336 10
15
cm.
CAPTULO 4. EXPRESIONES ALGEBRAICAS 169
Ejemplo 4.13 El nmero de Avogadro 6, 022 10
23
, es el nmero de molculas contenidas en
un mol. Si un mol de H
2
O tiene 18 gr, calcule la masa de una molcula de agua.
Solucin
La masa de una molcula de agua es
m =
18
6, 022 10
23
gr
=
18
6, 022
10
23
gr
= 2, 989 10
23
gr.
4.4. Tarea
1. Simplique las expresiones:
a)
1
4
1
1
8

1
2
_
1

5
2
2
_

1
3
_
2

3
4

1
2

1
8
5
2
_

1
25
_ ;
b)
0, 4
_
1
2
1
_
+ (1, 2)
2
0, 5
+ 2(0, 1)
2

_
3
4
1
_
(2)
1
1
2
;
c)
_
3
5

2
3
_
2

_
1
2
_
2

5
3
_
1
3 7
7
2
(3)
1
;
d)
_
_
3
2
3
_
6
4
1
12
+
1
3

6
5
+
_

3
4
_
2

2
3
;
e)
_
4
0, 8 1, 2
_
1

_
2(0, 1) + (1 0, 5)
2

0, 1;
f )
_

2
_
4
_
_
2
1
2
_
3

_
2
3
_
2
_

_
(0, 1)
2
_
5
2
_
3
_
;
g)
8
_
4
2
_
4
3
3
27
2
+ 90 6
3
4
7

_
3
2
_
2
24 (6
2
)
4
(2
4
)
2
+ 144 (2
3
)
4
(9
2
)
2
4
2
.
2. Simplique las expresiones:
a)
_
_
10
6
_
2
+ 5
3
25
4
2
3

_
2
3
_
2
5
13

_
4
2
_
2
_

_
_
5
5
_
1
_
1
2
_
2
10
5
_
;
b)
_
9
_
1
2
_
2
18
2

_
2
2
_
2
_
1
3
2
_
5

_
3
3
_
2
_
1
6
_
6
_
__
1
2
_
4

1
_
3
2
_
4
;
c)
_
2
3
+
5
6

2
9
_

2
9
_

9
4
_

2
9
_
+
_

2
3
_
(3)
3

_
5
2
3
_

1
3
_
;
d)
_
3
_
3
4
+ 1
_
6
+
_
_
3
_
8
5
2
_
3
_
1
(2)
2
+
3
4
_

1
2
_
2
(2)
3
(3)
2
+ (2)
4
3
(1)
2

_
1 +
1
2
_
1
;
e) (3)
1
(1)
3

_
1
1
2
_
2
+
_
5
5
6
_
1

_
1
1
5
_
1
.
3. Simplique las expresiones:
a)
0, 2
2
1, 5 + 0, 1 0, 4 0, 2
(1 0, 6) 0, 02
;
b)
1 0, 5
2
0, 125
2
(0, 6 0, 5)
2
;
CAPTULO 4. EXPRESIONES ALGEBRAICAS 170
c)
_
0, 8
0, 8 + 1

_
0, 5
0, 7
2
__
2
(0, 3)
2
;
d)
(1 0, 5)
2
0, 25 + 0, 75
0, 5 (1 0, 25)
0, 3;
e)
(0, 5 0, 6) 0, 1
(0, 5 1)
2
;
f )
2
_
0, 9 1
3
_
2
+
(0, 5 1)
2
0, 5
+ 5 (0, 1);
g)
(0, 5 0, 1 1, 55)
2
0, 75 1
.
4.5. Potencia con exponente racional
Denicin 4.12 Raz aritmtica de n-simo grado
Sea n un nmero natural y a, un nmero positivo. Entonces el nmero positivo b tal, que b
n
= a
lleva el nombre de raz aritmtica de n-simo grado del nmero a y se designa b =
n

a.
De esta denicin resulta vlida la siguiente armacin:
n

a
_

_
a es un nmero positivo
n es un nmero natural
n

a es un nmero positivo
(
n

a)
n
= a.
Para todo nmero positivo a existe una, y slo una, raz aritmtica de n-simo grado.
A continuacin damos a conocer la denicin de elevacin de un nmero entero a una potencia
con exponente racional aprovechando con este n la denicin de elevacin a potencia entera y la
denicin de raz aritmtica de un nmero positivo.
Denicin 4.13 r-sima potencia
Sea a un nmero positivo y r =
p
q
, un nmero racional, con la particularidad de que q es un nmero
natural mayor que cero. El nmero positivo b tal, que b =
q

a
q
lleva el nombre de r-sima potencia
del nmero a y se denota b = a
r
, es decir a
p
q
=
q

a
p
.
Supongamos que a y b son cualesquiera nmeros positivos y k, t, cualesquiera nmeros racionales.
Resultan vlidas las siguientes propiedades, llamadas propiedades de las potencias con exponentes
racionales.
Teorema 4.6 Al elevar a potencia un producto, se puede elevar a esa potencia cada uno de los
factores y multiplicar los resultados obtenidos:
(ab)
k
= a
k
b
k
.
Demostracin
Sea k =
p
q
donde q es un nmero natural, entonces:
_
(ab)
p
q
_
q
=
_
q
_
(ab)
p
_
q
= (ab)
p
= a
p
b
p
=
_
q

a
p
_
q
_
q

b
p
_
q
=
_
a
p
q
_
q
_
b
p
q
_
q
=
_
a
p
q
b
p
q
_
q
CAPTULO 4. EXPRESIONES ALGEBRAICAS 171
As pues, ((ab)
k
)
q
= (a
k
b
k
)
q
, esta igualdad es equivalente a la igualdad (ab)
k
= a
k
b
k
.
Teorema 4.7 Si se eleva a potencia una fraccin, se pueden elevar a esta potencia el numerador
y el denominador de la fraccin por separado y dividir el primer resultado por el segundo:
_
a
b
_
k
=
a
k
b
k
Teorema 4.8 Al multiplicar potencias de bases iguales se suman los exponentes:
a
k
a
t
= a
k+t
.
Demostracin
Supongamos que k =
p
q
, t =
m
n
. Entonces a
k
a
t
= a
p
q
a
m
n
. Por tanto
_
a
p
q
a
m
n
_
qn
=
_
a
p
q
_
qn _
a
m
n
_
qn
=
__
a
p
q
_
q
_
n
_
_
a
m
n
_
n
_
q
=
__
q

a
p
_
q
_
n
__
n

a
m
_
n
_
q
= (a
p
)
n
(a
m
)
q
= a
pn
a
mq
= a
pn+mq
=
_
nq

a
pn+mq
_
nq
=
_
a
pn+mq
nq
_
nq
As pues, tomando en consideracin que
pn +mq
nq
= k +t
tenemos (a
k
a
t
)
qn
= (a
k+t
)
qn
, esta igualdad es equivalente a la igualdad a
k
a
t
= a
k+t
.
Teorema 4.9 Al dividir potencias de bases iguales se restan los exponentes:
a
k
a
t
= a
kt
.
Teorema 4.10 Si se eleva a potencia una potencia, los exponentes se multiplican:
(a
k
)
t
= a
kt
.
Demostracin
CAPTULO 4. EXPRESIONES ALGEBRAICAS 172
Supongamos que k =
p
q
, t =
m
n
, entonces (a
k
)
t
=
_
a
p
q
_m
n
. Por tanto
_
_
a
p
q
_m
n
_
nq
=
__
_
a
p
q
_m
n
_
n
_
q
=
__
n
_
_
a
p
q
_
m
_
n
_
q
=
__
a
p
q
_
m
_
q
=
_
a
p
q
_
mq
=
__
a
p
q
_
q
_
m
=
__
q

a
p
_
q
_
m
= (a
p
)
m
= a
pm
=
_
pn

a
pm
_
qn
=
_
a
pm
qn
_
qn
As pues, ((a
k
)
t
)
nq
= (a
kt
)
nq
, la validez de esta igualdad predetermina la validez del teorema.
Teorema 4.11 Supongamos que a es un nmero positivo, k =
p
q
un nmero racional, mientras
que q y n son nmeros naturales. En este caso
a
p
q
= a
pn
qn
.
Demostracin
Por tanto
_
a
p
q
_
qn
=
__
a
p
q
_
q
_
n
=
_
q
_
(a
p
)
q
_
n
= (a
p
)
n
= a
pn
=
_
qn

a
pn
_
qn
=
_
a
pn
qn
_
qn
As pues,
_
a
p
q
_
qn
=
_
a
pn
qn
_
qn
, de donde precisamente proviene la validez de esta propiedad.
Para las races aritmticas, las propiedades demostradas anteriormente se expresan de la sigu-
iente manera:
1) Al extraer la raz de un producto se puede extraer la raz de igual exponente de cada factor y
multiplicar los resultados obtenidos, es decir
n

ab =
n

a
n

b
CAPTULO 4. EXPRESIONES ALGEBRAICAS 173
2) Para extraer la raz de una fraccin, se puede extraer la raz, de igual exponente, del numerador
y denominador por separado y dividir el primer resultado para el segundo, es decir
n
_
a
b
=
n

a
n

b
3)
n

a
m

a =
nm

a
n+m
;
4)
n

a
m

a
=
nm

a
mn
;
5) Al elevar a potencia una raz se puede elevar a esa potencia el nmero subradical sin variar
el ndice de la raz. Adems al extraer la raz de una potencia se puede dividir el exponente del
radicando por el ndice de la raz, si esa divisin se cumple enteramente, es decir
_
n

a
_
m
=
n

a
m
= a
m
n
6) Al extraer la raz de una raz se puede extraer la raz de grado igual al producto de los ndices
de las dos races, permaneciendo el resultado sin variacin, es decir
m
_
n

a =
nm

a
7) El ndice de la raz y el exponente del radicando se pueden dividir por su factor comn, es
decir
nm

a
m
=
n

a
Se denomina radical doble, a una expresin de la forma
_
A

B
Todo radical doble se puede descomponer en la suma o diferencia de dos radicales simples. En
general:
_
A

B =

y
De donde se deduce que
__
A+

B =

x +

y (I)
_
A

B =

y (II)
Para calcular x y y, procedemos de la siguiente manera:
1) Sumando (I) + (II):
2

x =
_
A+

B +
_
A

B
Elevando al cuadrado:
4x = A+

B + 2
_
A+

B
_
A

B +A

B
x =
2A+ 2

A
2
B
4
=
A+

A
2
B
2
haciendo C =

A
2
B, entonces
x =
A+C
2
CAPTULO 4. EXPRESIONES ALGEBRAICAS 174
2) Restando (I) - (II):
2

y =
_
A+

B
_
A

B
Elevando al cuadrado:
4y = A+

B 2
_
A+

B
_
A

B +A

B
y =
2A2

A
2
B
4
=
A

A
2
B
2
haciendo C =

A
2
B, entonces
x =
AC
2
Sustituyendo los valores de x y y en (I) y (II):
_
_
_
_
A+

B =
_
A+C
2
+
_
AC
2
_
A+

B =
_
A+C
2
+
_
AC
2
Es decir que, para transformar races dobles, en races simples, A
2
B debe ser un nmero
cuadrado perfecto.
Un radical de la forma
_
A+

B +

C +

D
se puede descomponer en radicales simples de la siguiente manera:
Sea
_
A+

B +

C +

D =

x +

y +

z
el objetivo es calcular x, y y z en funcion de los valores conocidos A, B, C y D. Se procede
elevando al cuadrado la expresin
__
A+

B +

C +

D
_
2
=
_
x +

y +

z
_
2
A+

B +

C +

D = x +y +z + 2

xy + 2

xz + 2

yz
identicando los trminos racionales e irracionales, tenemos:
_

_
x +y +z = A (1)
2

xy =

B (2)
2

xz =

C (3)
2

yz =

D (4)
que es un sistema de cuatro ecuaciones con tres incgnitas. Resolviendo en el sistema conformado
por las ecuaciones (2), (3) y (4) se obtiene x, y y z. La ecuacin (1) es la ecuacin de comprobacin
de los valores obtenidos.
Un radical de la forma
_
A+

D
CAPTULO 4. EXPRESIONES ALGEBRAICAS 175
en este caso, los radicales simples deben llevar algn signo negativo:
Sea
_
A+

D =

x +

z
Elevamos al cuadrado la expresin
__
A+

D
_
2
=
_
x +

z
_
2
A+

D = x +y +z + 2

xy 2

xz 2

yz
identicando los trminos racionales e irracionales, tenemos:
_

_
x +y +z = A (1)
2

xy =

B (2)
2

xz =

C (3)
2

yz =

D (4)
que es un sistema de cuatro ecuaciones con tres incgnitas. Resolviendo en el sistema conformado
por las ecuaciones (2), (3) y (4) se obtiene x, y y z. La ecuacin (1) es la ecuacin de comprobacin
de los valores obtenidos.
El radical de la forma
3
_
A

B
se puede descomponer en radicales simples, de la siguiente manera:
1)
3
_
A+

B:
Haciendo
3
_
A+

B = x +

y
elevando al cubo
_
3
_
A+

B
_
3
= (x +

y)
3
A+

B = x
3
+ 3x
2

y + 3x(

y)
2
+ (

y)
3
= x
3
+ 3xy + 3x
2

y +y

y
igualando las partes racionales e irracionales
_
A = x
3
+ 3xy (1)

B = 3x
2

y +y

y (2)
Restando (1) - (2) y ordenando
A

B = x
3
+ 3x
2

y + 3x(

y)
2
(

y)
3
= (x

y)
3
extrayendo la raz cbica queda demostrado que
3
_
A+

B = x +

y
CAPTULO 4. EXPRESIONES ALGEBRAICAS 176
donde, conocidos los valores de A y B se debe calcular x e y en funcin de los anteriores.
2)
3
_
A

B:
Como
_
3
_
A+

B = x +

y (1)
3
_
A

B = x

y (2)
Multiplicando (1) y (2)
3
_
(A+

B)(A

B) = (x +

y)(x

y)
3
_
A
2
B = x
2
y
haciendo C =
3

A
2
B se tendr
C = x
2
y y = x
2
C (3)
De (1) se sabe que
A = x
3
+ 3xy
sustituyendo el valor de y
A = x
3
+ 3x(x
2
C) = 4x
3
3xC (4)
de donde por tanteos, se encuentra el valor de x que sustituyendo en (3) da el valor de y.
Ejemplo 4.14 Simplique la expresin:
3
_
2
3
2
_
3
2
+

6 +

150.
Solucin
Simplicamos la expresin:
A =
_
3
2

2
3

_
2
2

3
2
+

6 +

25 6
=

6 +

6 + 5

6
= 6

6.
Ejemplo 4.15 Simplique la expresin:
2
_
5

48 + 3
_
40

12 2
_
15

27.
Solucin
Simplicamos la expresin:
A = 2
_
20

3 + 6
_
10

12 2
_
45

3
= 4
_
5

3 + 6
_
20

3 6
_
5

3
= 4
_
5

3 + 12
_
5

3 6
_
5

3
= 10
_
5

3.
CAPTULO 4. EXPRESIONES ALGEBRAICAS 177
Ejemplo 4.16 Simplique la expresin:
15
_
1,04
3
5
_
5
5
9
+ 6
_
1
18
5
_
0,02 +

300.
Solucin
Simplicamos la expresin:
A = 15
_
26
25

3
5
_
50
9
+ 6
_
1
18
5
_
1
50
+

300
= 15
1
5

26
3
5

5
3

2 + 6
1
3
_
1
2
5
1
5
_
1
2
+ 10

3
= 3

26

2 + 2
_
1
2

_
1
2
+ 10

3
= 3

26

2 +

2
1
2

2 + 10

3
= 3

26
1
2

2 + 10

3.
Ejemplo 4.17 Simplique la expresin:
30
3
_
1
12
+ 3
1
2
3
_
2
3
+ 5
3

144.
Solucin
Simplicamos la expresin:
A = 30
3
_
1
12
+
7
2
3
_
2
3
+ 5
3

2
3
18
= 30
3
_
1
12
+
7
2
3
_
2
3
+ 10
3

18
= 30
1
3
_
1
12
+
7
2
3

2
3

3
+ 10
3

18
=
60 + 7
3

8 + 20
3

216
2
3

12
=
60 + 7
3

2
3
+ 20
3

6
3
2
3

12
=
60 + 14 + 120
2
3

12
=
97
3

12
.
Ejemplo 4.18 Simplique la expresin:
__
2
1
2
3
1
2
_
: 4
1
6
_
:
__
4

1
2
:
_
2

1
3
3

1
3
__ __
2

5
6
4

2
3
_
: 3
5
6
__
.
Solucin
CAPTULO 4. EXPRESIONES ALGEBRAICAS 178
Simplicamos la expresin:
A =
_
2
1
2
3
1
2
4
1
6
_
:
__
1
2
:
1
2
1
3
3
1
3
_ _
1
2
5
6
4
2
3
: 3
5
6
__
=
_
2
1
2
3
1
2
2
1
3
_
:
__
1
2
:
1
2
1
3
3
1
3
_ _
1
2
5
6
2
4
3
: 3
5
6
__
= 2
1
6
3
1
2
:
_
2
1
3
3
1
3
2

1
2
13
6
3
5
6
_
= 2
1
6
3
1
2
:
_
1
2
17
6
3
1
2
_
= 2
1
6
3
1
2
2
17
6
3
1
2
= 2
3
3
= 24.
Ejemplo 4.19 Simplique la expresin:
_
__
3

1
2
2

1
3
_
:
_
3

3
4
2

5
6
__
:
_
1
864
_1
4
_
2
7
.
Solucin
Simplicamos la expresin:
A =
_
_
1
3
1
2
2
1
3
:
1
3
3
4
2
5
6
_
:
_
1
864
_1
4
_
2
7
=
_
3
3
4
2
5
6
3
1
2
2
1
3
:
_
1
27 32
_1
4
_
2
7
=
_
3
3
4
2
5
6
3
1
2
2
1
3
_2
7
:
_
1
3
3
4
2
5
4
_2
7
=
_
3
1
4
2
1
2
_2
7
3
3
14
2
5
14
= 3
1
14
2
1
7
3
3
14
2
5
14
= 3
2
7
2
1
2
.
Ejemplo 4.20 Simplique la expresin:
_
3
5
2
5
4
3
2
5
4
_
16 :
_
27
1
5

5
3
__ _
25 3
1
2
: 2
1
4
__1
5
Solucin
CAPTULO 4. EXPRESIONES ALGEBRAICAS 179
Simplicamos la expresin:
A =
_
3
5
2
5
4
3
2
5
4
_
16 :
1
27 5
5
3
_
_
25 3
1
2
2
1
4
__1
5
=
_
3
5
2
5
4
3
2
5
4
_
16 27 5
5
3
_
_
5
2
3
1
2
2
1
4
__1
5
=
_
3
5
2
5
4
3
2
5
4
16 27 5
5
3
5
2
3
1
2
2
1
4
_1
5
=
3
1
2
5
4
15
2
1
4
2
4
5
3
3
5
5
1
3
5
2
5
3
1
10
2
1
20
= 3
6
5
5 2
= 3
1
5
30.
A continuacin estudiaremos las propiedades principales del tipo de desigualdades para la po-
tencia con exponente racional:
Teorema 4.12 Supongamos que a > 1 y r =
p
q
es nmero racional positivo p > 0, q > 0.
Entonces, a
r
> 1.
Demostracin
_
a
p
q
_
q
=
_
q

a
p
_
q
= a
p
Las condiciones a > 1 y a
p
> 1
p
son equivalentes, quiere decir, de la condicin a > 1 se desprende
que a
p
> 1, mas, en este caso,
_
a
p
q
_
q
> 1
q
, es decir, (a
r
)
q
> 1
q
, de lo cual, segn la misma
propiedad, resulta que a
r
> 1.
Teorema 4.13 Sea 0 < a < k, y r =
p
q
un nmero racional positivo p > 0, q > 0. Entonces,
a
r
< 1.
Teorema 4.14 Supongamos que a > 1 y k, t son nmeros racionales tales que k > t. Entonces,
a
k
> a
t
.
Demostracin
Por cuanto k t es un nmero racional positivo, entonces, conforma a la propiedad 1, a
kt
> 1.
Al multiplicar esta desigualdad por el nmero positivo a
t
, obtenemos a
t
(a
kt
) > a
t
. De aqu que
a
k
> a
t
, es decir esta propiedad queda demostrada.
Teorema 4.15 Supongamos que 0 < a < 1, y sean k y t nmeros racionales tales, que k > t.
Entonces, a
k
< a
t
.
La operacin inversa a la potenciacin se denomina radicacin; mediante esta operacin, si
estn dados la potencia y su exponente, se busca la base de la potencia. La operacin de radicacin
o extraccin de raz, se ja con el signo

; adems, sobre este signo se escribe el ndice de la raz
y slo en el caso de la raz cuadrada el ndice de raz.
Extraer la raz n-sima del nmero a signica hallar un nmero x tal, que despus de elevar a
la potencia n obtenemos el mismo nmero a, es decir
n

a = x, si x
n
= a.
CAPTULO 4. EXPRESIONES ALGEBRAICAS 180
Ejemplo 4.21 Demuestre que

2 es irracional.
Solucin
Si x R y x
2
= 2, entonces x no es racional. La propiedad de ser irracional es un tipo de propiedad
negativa y no es fcil de vericar directamente. Sin embargo, podemos demostrar que x racional
junto con x
2
= 2 conduce a una contradiccin. Demostremos que

2 es irracional por contradiccin.


Supongamos que x R, x
2
= 2 y x es racional. Por denicin de nmero racional tenemos que
x =
p
q
donde p, q Z y q ,= 0. Reduciendo la fraccin cuanto sea necesario podemos suponer que p
y q no tienen factores comunes. En particular p y q no pueden ser ambos pares. Como 2 = x
2
=
p
2
q
2
tenemos p
2
= 2q
2
y por lo tanto p
2
es par. Esto implica que p es par. Entonces p = 2k para algn
k Z. Luego (2k)
2
= 2q
2
y por lo tanto q
2
= 2k
2
. As q
2
y q son tambin pares. Pero entonces p y
q son ambos pares contradiciendo lo que inicialmente se estableci. Por lo tanto

2 es irracional.
Ejemplo 4.22 Demuestre que

3 +

2 es un nmero irracional.
Solucin
Suponga que

3 +

2 Q, entonces

3 +

2 =
1

2
es el cociente de dos nmeros racionales,
de donde el nmero

2 =
(

3+

2)(

2)
2
Q, lo que contradice la naturaleza irracional de

2.
Por lo tanto, la suposicin es falsa y el nmero

3 +

2 es irracional.
Ejemplo 4.23 Demustrese que para cualesquiera nmeros positivos a y b se verica la de-
sigualdad
a
2
3
+b
2
3
> (a +b)
2
3
.
Solucin
Denotemos a+b con c y examinemos las fracciones
a
c
y
b
c
. Por cuanto
a
c
+
b
c
= 1, entonces 0 <
a
c
< 1,
0 <
b
c
< 1. De aqu que
_
a
c
_1
3
< 1,
_
b
c
_1
3
< 1,
es decir
_
a
c
_
1
2
3
< 1,
_
b
c
_
1
2
3
< 1.
Por consiguiente
a
c
<
_
a
c
_2
3
y
b
c
<
_
b
c
_2
3
.
De acuerdo con la propiedad de las desigualdades numricas se verica tambin la desigualdad
_
a
c
_2
3
+
_
b
c
_2
3
>
a
c
+
b
c
de donde, teniendo en cuenta que
a
c
+
b
c
= 1, llegamos a que se verica la desigualdad
_
a
c
_2
3
+
_
b
c
_2
3
> 1.
Teniendo en cuenta que c es un nmero positivo y multiplicando esta desigualdad por c2/3, con-
cluimos que se verica la desigualdad
a
2
3
+b
2
3
> c
2
3
.
CAPTULO 4. EXPRESIONES ALGEBRAICAS 181
La raz de ndice par de un nmero positivo tiene dos valores reales inversas. La raz de ndice
impar tiene el mismo signo que el nmero subradical. La raz de ndice par de un nmero negativo
no es un nmero real. Tales races se denominan nmeros imaginarios.
Denicin 4.14 Valor aritmtico de la raz
El valor no negativo de la raz de ndice par de un nmero no negativo se denomina valor aritmtico
de la raz.
Cuando hay que calcular aproximadamente la magnitud numrica de una fraccin, que contiene
en el denominador un radical, frecuentemente se hace necesario dividir por un nmero de muchas
cifras, lo que es incmodo. Sin embargo, la fraccin dada se puede transformar de manera que el
denominador se convierta en un nmero racional. Esta transformacin se denomina, racionalizacin
de denominadores.
Denicin 4.15 Radicales semejantes
Dos o varios radicales se denominan semejantes, si se diferencian slo por los coecientes, pero
tienen idnticas expresiones subradicales e iguales ndices del radical o no dieren en nada.
Frecuentemente los radicales aparentan ser no semejantes; sin embargo, despus de reducirlos a
la forma elemental se puede descubrir su semejanza. Los radicales semejantes se reducen del mismo
modo que los monomios racionales semejantes.
Al sumar o restar radicales se relacionan entre s con el signo ms o menos y se reducen a
radicales semejantes, si stos existen.
Al multiplicar y dividir polinomios irracionales se utilizan las mismas reglas que al multiplicar
y dividir polinomios racionales.
Ejemplo 4.24 Simplique la expresin:
9
5

7
+
22
7 +

7 +

5
Solucin
A =
9
_
5 +

7
_
_
5

7
_ _
5 +

7
_ +
22
_
7

5
_
_
7 +

5
_ _
7

5
_

5
_
7 +

5
_ _
7

5
_
=
9
_
5 +

7
_
25 7
+
22
_
7

5
_
49 5

5
7 5
=
9
_
5 +

7
_
18
+
22
_
7

5
_
44

5
2
=
5 +

7
2
+
7

5
2

5
2
=
5 +

7 + 7

7 +

5
2
= 6.
4.6. Tarea
1. Simplique la expresin:
a)
_
18
_
4

17
_
2
; b)
_
54
_
2

3
_
2
; c)
4
_
48
_
2

7
_
4
; d)
4
_
2
_
11 3
_
4
.
CAPTULO 4. EXPRESIONES ALGEBRAICAS 182
2. Simplique la expresin:
a) 2
_
5

48+3
_
40

122
_
15

27; b) 2
3

0, 125+
4

0, 0016; c)
4

0, 0001
5

0, 00032.
3. Simplique la expresin:
a) 3
_
1
27

5
6

27 0, 1

75 + 2
_
1
3
; b) 30
3
_
1
12
+ 3
1
2
3
_
2
3
+ 5
3

144;
c)
_
6 2

15
_

3
3
+

20.
4. Simplique la expresin:
a) 2
_
252

175
_

_
112

63

28
_
;
b)

12 2

27 3

48 + 2

75 + 3

108;
c)

176 2

275 +

1584

891;
d) 15

1, 04
3
5
_
5
5
9
+ 6
_
1
18

_
5

0, 02

300
_
.
5. Expresar cada uno de los cocientes, en la forma a +b

c:
a)
1 +

2
3 2

2
;
b)
2 3

6
4 +

6
;
c)
_
6 3
_ _
2 +

3
_
_
8 2
_ _
2 + 1
_ ;
d)
2

6 8 + 3

2
_
3

2
_
2 +

6 +

3
_
2

3
_ ;
e)
3 + 4

3
2 +

3
;
f )
1

6
_
1 +

6
_
2
;
g)
_
8 1
_
3
_
2

6 +

3
_
4
9
_
8 + 1
_
5
;
h)
_
2 +

5 +

3
_ _
2

3
_
_
3 +

5
_
2
.
6. Simplique la expresin:
a)
_
12 +

140
_
8 +

28 +
_
11 2

30
_
7 2

6;
b)

_
2

3 +

9 + 5

3
_
3(

3 + 2) +
_
4 + 2

3;
c)

_
1 + 2

_
1 + 2

_
1 + 2

1 +... + 2
_
1 + 2
_
3 + 2

2;
d)
_

2 1
__
56 + 40

2
_
34 + 26

2 +
_
23 + 37

2
_
;
e)

a +b +

a b
_
a +

a
2
b
2

_
c

c
2
d
2

c +d

c d
;
f )
_
2x + 2

x
2
1
_
2 + 2
_
2x
2
+ 2x + 2

x
4
+ 2x
3
2x 1
;
g)
_
9 4

2 + 2
_
3 + 2

2 +
_
12 + 8

2
_
15 10

2 +
_
13 + 4

10
_
11 2

10
;
CAPTULO 4. EXPRESIONES ALGEBRAICAS 183
h)
_
10 + 2

6 + 2

10 + 2

15;
i)
_
24 + 4

15 + 4

21 + 2

35;
j)
_
a + 3b + 4 + 4

a + 4

3b + 2

3ab;
k)
_
14 + 2

10 2

14 2

35;
l)
3
_
7 + 5

2;
m)
3
_
54 30

3;
n)
3
_
14

5 + 18

3;
o)
42x
2
9x
3
10

42x
2
9x
3
24

42x
2
9x
3
24 6
;
p)
1
3
_
7 + 5

2
+
1
3
_
26 + 15

3
+
1
3
_
9

3 + 11

2
;
q)
4
_
3 +

7
_
_
13

7
_
5

7;
r)
_
a + 5b + 3
_
2ab +b
2

_
a +
_
2ab +b
2
+b;
s)

_
6 +

_
6 +

6 +... +
_
_
6 + 4

2 + 7

2;
t)
1

2 +

5
+
1

3 +

2
+
1

5 +

3
+
3

5 +

3
2

6
.
Resp: a) 0; b) 2; c) 1 +

2; d) 7; e)

2
2
; f ) 1; g) 3;
h)

2 +

3 +

5; i) 2

3 +

5 +

7; j) 2 +

a +

3b; k)

2 +

7;
l) 1 +

2; m) 3

3; n)

3 +

5; o)

42x
2
9x
3
24 4; p) 1; q) 2;
r)

2b; s) 3; t)

2
2
.
7. Transformar a radicales simples la expresin:
a)
_
5x 2 + 2
_
6x
2
7x 3;
b)
_
7x + 16y + 4 + 2
_
21xy + 39y
2
+ 56x + 92y 32;
c)
_
5x 2 +
_
24x
2
14x 5;
d)

x +
1
2
_
2x
1
4
;
e)
_
a +b +c +
_
c(2a + 2b +c)
_
a +b +c
_
c(2a + 2b +c).
Resp: a)

3x + 1 +

2x 3; b)

7x + 13y 4 +

3y + 8; c)
_
6x5
2
+
_
4x+1
2
;
d)
_
1
8
+
_
x
1
8
; e)

2c.
8. Simplique la expresin:
a)
m
_

2 1
2m
_

2 + 1
4m
_

2 + 1
8m
_
3 +

8;
CAPTULO 4. EXPRESIONES ALGEBRAICAS 184
b)
3
_

2 1
8
_
3 + 2

2
6
_

2 + 1
12
_
5

2 7
;
c)
_
x
3
3x + (x
2
1)

x
2
4
_
x
3
3x (x
2
1)

x
2
4
_
x +

x
2
4
_
x

x
2
4
;
d)
3
_
3
4

a
2
+
_
9

a
4
1 +
3
_
3
4

a
2

_
9

a
4
1
3

a +
3
_
3
4

a
2
+
_
9

a
4
1 +
_
3
4

a
2

_
9

a
4
1
;
e)

6ny
_
(

an +

ny)(

ax +

ax ny)
_
(

an

ny)(

ax +

ax ny)
;
f )

a b +

b c +

c a
_
_
(a b)(b c) +
_
(a b)(c a) +
_
(b c)(c a)
;
g)
_
4 +

15
_
3
2
+
_
4

15
_
3
2
_
6 +

35
_
3
2

_
6

35
_
3
2
;
h)
_
26 +

675
_
26

675
3
_
26 +

675 +
3
_
26

675
;
i)
_
1 +x

1 +x

1 x
+
1 x

1 +x
2
+x 1
__
_
x
2
1
1
x
_
;
j)
x
3
3x 2 + (x
2
1)

x
2
4
x
3
3x + 2 + (x
2
1)

x
2
4

_
x + 2
x 2
;
k)
_
5
_
5 +

5
_
5

2
_
5 +

2
+

2
_
5

2
_
2
+ 2

23;
l)

_
10

4
_
6 +
_
6 +

6 +...;
m)
3

2
_
9 + 2

18

3
_
8 + 2

12
+

6
_
5 + 2

6
;
n)
10

18
_
3 +

10 +

18

8
_
3 +

5;
o)
48
3

21
3

3 +
3

35
3

5
;
p)
3
3

2
3

4
3

2 2
;
q)
1
3

4+
3

2+1
+
3
3

4
3

2+1
1
3

4+
3

2+1

3
3

4
3

2+1
+
3

2;
r)
6
2 +

2
4

2
;
s)
(x 1)
_
1 +x
3

x
2
_
1 +
3

x +x
3

x
2
;
t)
3

3 +
6

9
.
CAPTULO 4. EXPRESIONES ALGEBRAICAS 185
Resp: a) 1; b) 1; c) x + 1; d)
3

3; e)

3; f )

2; g)
7
13
; h)
5

2
4
;
i) -1; j)
x+1
x1
; k) 10; l) 3; m) 0; n) 3;
o)
_
3

25
3

15 +
3

9
_ _
3

49
3

7 + 1
_
; p) 1
3

2; q) 0; r) 2 + 2
4

2
4

8;
s)
3

x 1; t)
1
2
(
6

3 1)(
3

9 +
3

3 + 1).
9. Simplique la expresin:
a)
20
7 +

6 +

14 +

21
; b)
2 + 2

2
1 +

2 +

3 +

6
; c)
3
_
20

2 + 12

6+
3
_
20

2 12

6.
Resp: a) 7 +

21

14; b)

3 1; c) 2

2.
10. Simplique la expresin:
a) 2

12 4

27 + 4

24;
b) 6
3

54 + 10
3

2 +
3

150;
c)
_
3

3 +
3

2
_ _
3

9
3

4
_
+
3

12
3

18;
d)
_
3

10
_ _
3 +

10
_
;
e)
_
3

5
3

2
_
3

3 +
3

6;
f )
_

5
_
3
_

5
_
4

5
_
2
.
11. Simplique la expresin:
a)
3

2
+
4

6 +

5
;
b)
1

2
+
1

2 1
+
1

2 + 1
;
c)
2

5 +

3 +

2
;
d)
12

15

6 +

35

14
;
e)
4
3

5 1
+
7
3

5 +
3

2
;
f )
3
_
54 + 30

3 +
3
_
54 30

3;
g)
3
_

5 + 2
3
_

5 2;
h)
3
_
72 + 32

5
3
_
72 32

5.
12. Simplique la expresin:
a)
_
2
3

2
_
3
_
2

2
4
_
2
3

2;
b)
3
3

25 +
3

10 +
3

4
;
c)
1
3

3
3

2
;
d)
4

8
4

2
+
4

6
4

160
4

5
.
13. Expresar cada uno de los cocientes, en la forma a +b
3

2 +c
3

4:
a)
1 + 3
3

2
3

4
3
3

4
;
b)
1
1 +
3

2
3

4
;
c)
5 4
3

2 2
3

4
1 + 2
3

2 3
3

4
;
d)
1 +
3

2
1
3

2
;
e)
1 +
3

2 + 2
3

4
3

2
;
f )
1
3

2 2
3

4
1 2
3

2
;
g)
3 + 2
3

2 +
3

4
2
3

2 +
3

4 1
;
h)
3

2
1 +
3

2
.
CAPTULO 4. EXPRESIONES ALGEBRAICAS 186
4.7. Potencia de un nmero positivo
Todo lo analizado anteriormente, permite dar la denicin de potencia real de un nmero
positivo. Obsrvese que el nmero a
k
existe y, adems, es nico para cualquier nmero real k.
Denicin 4.16 Potencia de un nmero positivo
Sean dados un nmero positivo a y un nmero real k. Por nmero a se entiende un nmero positivo
que se determina segn la siguiente regla:
1.- Si k > 0:
a) k = m, m es un nmero natural, entonces
a
k
=
_
_
_
a, para m = 1
a a ... a
. .
n veces
, para m 2
b) k =
1
q
, donde q es un nmero natural, entonces a
k
=
q

a;
c) k =
p
q
, donde p, q son nmeros naturales, entonces a
k
=
q

a
p
;
d) k es un nmero irracional, entonces:
i) Si a > 1, el nmero p
k
ser mayor que a
r
1
y menor que a
s
t
, donde r
i
es cualquier aproximacin
racional del nmero k por defecto y s
t
, cualquier aproximacin racional del nmero k por exceso;
ii) Si 0 < a < 1, entonces a
k
es un nmero menor que a
r
i
y mayor que a
s
t
;
iii) Si a = 1, entonces a
k
= 1.
2.- Si k < 0, entonces a
k
=
1
a
|k|
.
El nmero a
k
recibe el nombre de potencia, el nmero a es la base de la potencia y k, el exponente
de la potencia.
La potencia de un nmero positivo posee las siguientes propiedades principales: si a y b son
nmeros positivos, y k y r, cualesquiera nmeros reales, entonces:
1.- (ab)
k
= a
k
b
k
;
2.-
_
a
b
_
k
=
a
k
b
k
;
3.- a
k
a
r
= a
k+r
;
4.-
a
k
a
r
= a
kr
;
5.- (a
k
)
r
= a
kr
.
Ejemplo 4.25 Simplique la expresin:
_
2
3

2
__
3
3
_
3
4

3
4
_
_
3
3

3
_
3
2
Solucin
A =
_
2
3

2
_1
2

_
_
3
3
_
3
4

3
_1
1

__
3
3

3
_1
4
_
3
2
= 2
1
2
2
1
6

_
3
1
2

_
3
4

3
_1
6

_
3
3

3
_1
8
_
3
2
= 2
2
3

_
3
1
2
3
1
6
3
1
24
3
1
8
3
1
24
_3
2
= 2
2
3

_
3
17
24
3
1
6
_3
2
= 2
2
3

_
3
13
24
_3
2
= 2
2
3
3
13
16
= 2
2
3
3

13
16
.
CAPTULO 4. EXPRESIONES ALGEBRAICAS 187
Ejemplo 4.26 Simplique la expresin:
_
3
_
16
4
_
8

2
_2

3
_
32
4

2
_
2
4
_
4
3

4
Solucin
A =
_
(16)
1
3

_
8

2
_ 1
12
_
(32)
1
3
2
1
12
2
1
2

_
4
3

4
_1
8
= (16)
2
3

_
8

2
_1
6
(32)
1
3
2
7
12
4
1
8
4
1
24
= 2
8
3
8
1
6
2
1
12
2
5
3
2
7
12
2
1
4
2
1
12
= 2
8
3
2
1
2
2
1
12
2
5
3
2
7
12
2
1
4
2
1
12
= 2
35
6
.
Ejemplo 4.27 Simplique la expresin:
1
3

3
3

2
Solucin
Para simplicar esta expresin, multiplicamos y dividimos para el factor racionalizante, es decir:
A =
_
3

3
_
2
+
3

3
3

2 +
_
3

2
_
2
_
3

3
3

2
_
_
_
3

3
_
2
+
3

3
3

2 +
_
3

2
_
2
_
=
3

9 +
3

6 +
3

4
_
3

3
3

2
_ _
3

9 +
3

6 +
3

4
_
=
3

9 +
3

6 +
3

4
_
3

3
_
3

_
3

2
_
3
=
3

9 +
3

6 +
3

4.
Ejemplo 4.28 Simplique la expresin:
_
6
_
9 + 4

5 +
3
_
2 +

5
_

3
_

5 2
Solucin
Para simplicar esta expresin, multiplicamos y dividimos para el factor racionalizante, es decir:
A =
6
_
(9 + 4

5)(

5 2)
2
+
3
_
(

5 + 2)(

5 2)
=
6
_
(9 + 4

5)(9 4

5) +
3

5 4
=
6

81 80 + 1
= 2.
Ejemplo 4.29 Simplique la expresin:
_

ab +b

a +

b
__

a

a +

b
+

b
+
2

ab
a b
_
CAPTULO 4. EXPRESIONES ALGEBRAICAS 188
Solucin
Para simplicar esta expresin, hacemos la siguiente transformacin:
A =
_

ab +b

a +

b
__

a

a +

b
+

b
+
2

ab
a b
_
=
_

b
_
_
_

a
2

ab +

ab +

b
2
+ 2

ab
_

a +

b
__

b
_
_
_
=
_

b
_

a +

b
_
2
_

a +

b
__

b
_
=
_

b
_

a +

b
=

a +

b.
Si a 0, b 0, a ,= 0.
A continuacin estudiamos las principales propiedades de la potencia de un nmero positivo
del tipo de desigualdad.
Teorema 4.16 Si a > 1 y k > 0, entonces a
k
> 1.
Demostracin
Si k =
p
q
es un nmero racional (p y q son nmeros naturales), entonces la propiedad de a
k
> 1
ya se demostr anteriormente. Si k es un nmero irracional, elegimos cualquier nmero racional
positivo r que aproxima k por defecto, en este caso a
k
> a
r
. Al mismo tiempo a
r
> 1. Conforme a
la propiedad de transitividad de las desigualdades, la validez de dos igualdades a
k
> a
r
y a
r
> 1
predetermina la validez de la desigualdad a
k
> 1.
Teorema 4.17 Si a > 1 y k < 0, entonces a
k
< 1.
Demostracin
El nmero r = k es positivo, por lo cual, al aplicar el teorema anterior, tenemos a
r
> 1. Multi-
plicando ambos miembros de esta igualdad por el nmero positivo a
k
, segn la propiedad de las
desigualdades tenemos a
r
a
k
> a
k
; segn la denicin de potencias concluimos que a
r
a
k
= a
k+r
=
a
0
= 1, por consiguiente a
k
< 1.
Teorema 4.18 Si a > 1 y a
k
> 1, entonces k > 0.
Demostracin
Supongamos que a
k
> 1 y a > 1, pero k 0, es decir, o bien k = 0 o bien k < 0. Si k = 0, entonces
a
k
= 1 por denicin. Si k < 0 y a > 1, entonces, aplicando el teorema anterior, tenemos a
k
< 1.
As pues, si k 0, entonces a
k
1, lo que contradice la suposicin de que a
k
> 1.
Teorema 4.19 Si a > 1 y a
k
< 1, entonces k < 0.
Si a > 1, entonces las condiciones a > 1 y k > 0 son equivalentes; adems, son equivalentes las
condiciones a < 1 y k < 0, es decir, si a > 1, entonces:
a
k
> 1 k > 0; a
k
< 1 k < 0.
CAPTULO 4. EXPRESIONES ALGEBRAICAS 189
Teorema 4.20 Si 0 < a < 1 y k > 0, entonces a
k
< 1.
Demostracin
Examinemos el nmero b >
1
a
. Por cuanto b > 1, entonces, aplicando el teorema 1, tendremos
b
k
= 1. Multipliquemos ambos miembros de esta desigualdad por el nmero positivo a
k
. Segn la
propiedad de las desigualdades tenemos: b
k
a
k
> a
k
. Segn la propiedad de las potencias tenemos
b
k
a
k
= (ab)
k
= 1
0
= 1, por lo cual a
k
< 1.
Teorema 4.21 Si 0 < a < 1 y k < 0, entonces a
k
> 1.
Teorema 4.22 Si 0 < a < 1 y a
k
> 1, entonces k < 0.
Teorema 4.23 Si 0 < a < 1 y a
k
< 1, entonces k > 0.
Si 0 < a < 1, entonces las condiciones a
k
> 1 y k < 0 son equivalentes, adems, son equivalentes
las condiciones a
k
< 1 y k > 0, es decir, si 0 < a < 1, entonces:
a
k
> 1 k < 0; a
k
< 1 k > 0.
Si a > 0 y a ,= 1, entonces las condiciones a
k
= 1 y k = 0 son equivalentes, es decir, si a > 0 y
a ,= 1, se tiene:
a
k
= 1 a = 0.
Teorema 4.24 Si a > 1 y k
1
> k
2
, entonces a
k
1
> a
k
2
.
Teorema 4.25 Si a > 1 y k
1
< k
2
, entonces a
k
1
< a
k
2
.
Teorema 4.26 Si a > 1 y a
k
1
> a
k
2
, entonces k
1
> k
2
.
Teorema 4.27 Si a > 1 y a
k
1
< a
k
2
, entonces k
1
< k
2
.
Si a > 1, entonces las condiciones a
k
1
> a
k
2
y k
1
> k
2
son equivalentes; adems, son equivalentes
las condiciones a
k
1
< a
k
2
y k
1
< k
2
, es decir, si a > 1, entonces:
a
k
1
> a
k
2
k
1
> k
2
; a
k
1
< a
k
2
k
1
< k
2
.
Teorema 4.28 Si 0 < a < 1 y k
1
> k
2
, entonces a
k
1
< a
k
2
.
Teorema 4.29 Si 0 < a < 1 y k
1
< k
2
, entonces a
k
1
> a
k
2
.
Teorema 4.30 Si 0 < a < 1 y a
k
1
> a
k
2
, entonces k
1
< k
2
.
Teorema 4.31 Si 0 < a < 1 y a
k
1
< a
k
2
, entonces k
1
> k
2
.
Si 0 < a < 1, entonces las condiciones a
k
1
> a
k
2
y k
1
< k
2
son equivalentes; adems, son
tambin equivalentes las condiciones a
k
1
< a
k
2
y k
1
> k
2
, es decir, si 0 < a < 1, se tiene:
a
k
1
> a
k
2
k
1
< k
2
; a
k
1
< a
k
2
k
1
> k
2
.
Si a > 0 y a ,= 1, entonces las condiciones a
k
1
= a
k
2
y k
1
= k
2
son equivalentes, es decir, si a > 0
y a ,= 1, entonces:
a
k
1
= a
k
2
k
1
= k
2
.
Si k > 0, el concepto de operacin de elevacin a una potencia puede extenderse al conjunto de
todos los nmeros no negativos, puesto que, por denicin 0
k
= 0, si k > 0.
CAPTULO 4. EXPRESIONES ALGEBRAICAS 190
4.8. Tarea
1. Simplique la siguiente expresin:
a)
_
(1 + 4 5 + 2)
2

3
+
5
_
9
2
(3)
5
_
(8)(2)(4 + 6)
_
3
_
(35 7)(10 + 7 + 2)
_
6
+
_
_
(15)(3)(3 + 5)
_
2
b)
3

375
3

3 +
5

192
5

32 + [(2)(3)]
2
[(2)(6)
3
(11)]
0

3
_
(8)(30 + 3)
3
;
c) [2(3) + 8]
5
+
_
(200) (10 + 2) +
3
_
1372 (5 1) (1 + 2 4)
2
+
4
_
(512 4) (1 + 9);
d)
_
5
2
_
1

4
5
_
2
+1
1

1
6
_
1

_
_
1
3
4
_

25
16
+(2)
3
+(1)
2

2
3

_
1
4

3
8
_
1
;
e) [(6 2 + 5) (9 6)]
5

_
(2)(1)

4 6 +
_
3

27

16 (3)
+
3
_
5

169;
f )
_
1
_

1
2
_
2
10
_

2
3
_
2
2
_
(288)(
1
2
)
25

_
1
3
4
_
3
;
g)
(
1
6

5
6
+3)(
4
3
2)
2+
25
12

1
2

_
_

1
5
_

2+
1
2

1
2

_
_

5
3
_

3
5
_
_
1
.
2. Simplique la siguiente expresin:
a)
_
1
3
_
2
_

3
5
_
(12)
3
2

_

2
3
_
1
+
_
5
_

1
64
_
1
;
b)
3

8 (8)
3
+ (3)(2)
3

_
5 +

16 + (2)
2

64;
c)
_

2
3
_
3
1
3
_

15
4
_
+ 2
_

5
3
_
+
3
2
(3);
d)
5
_
(1215) 5
_
(49)(16)
3
_
216 (125) (1 + 28);
e) 1
1+2
2
7
+
2
9

3
14
_

1
2
_

_
2
3

1
24

3
_
3
3
8
;
f )
_

7
3
_
(14) + 2
_
3
2
9

5
_

32
243

_

3
2
_
;
g)

2
3

15
4

2
25
5 + 1
4
_

1
9
_
1

_
1
1
3
_
2
+
_
_

3
28
_
(21)
;
h)
3

_
_
2
3
+
_
3
2
_
2
_
3
_
1 +
7
8
_
1
10
_
1
_
_
3
5
_
1
+
_
1 +
5
6
_
_

_
1
5

_

1
100
_
_

1
2
_
2
1
_
1
;
i)
6
2
5
6
2
3

5
3

4
15
+
1
2

_
2
3

7
5
+ 1
2
3
+ 1
;
j)
_
6
_
5

5
_
2

_
6
_
25
3

25
_
4
2
3

_
3
_
5
_
5

5
_
1
3
;
k)
_
_
_
3

2
3
_
4
4
_
8
3

4
9
_
4
3
_
2
3

4
_
_
_
1
2

3
_
3
9

9;
CAPTULO 4. EXPRESIONES ALGEBRAICAS 191
l)
_

16 +

144
3
_
3
3
_
4

4 +
5
_
3

64
_

256

4.
3. Simplique la expresin:
a) (

32 +

45

98)(

72

500

8);
b)
3
_
20 +

392 +
3
_
20

392;
c)
1
3

2 1
;
d)
3
_
9

3 11

2;
e)
_
27 10

2;
f )
3
1 +

3
.
4. Simplique la expresin:
_
2 +

3
_
2 +
_
2 +

2 +
_
2 +
_
2 +

2
_
2 +
_
2 +

3
5. Transforme la expresin:
_
x
2
4x + 4 +
_
x
2
+ 6x + 9.
6. Simplique la expresin:
a)

x +y
2
y
+ 2

x +y
2
y
2

x;
b)
_
x
3

x 2x
3

y +
3
_
x
2
y
2
3

x
2

xy
+
3
_
x
2
y
3
_
xy
2
3

x
3

y
_

x
2
;
c)
_
1

x +

x + 1
+
1

x 1
_

_
1 +
_
x + 1
x 1
_
.
7. Simplique las expresiones:
a) 2x
2
5xy + 2y
2
con x =

6 +

5 y y =

5;
b) 3x
2
+ 4xy 3y
2
con x =

5 +

2
y y =

5 +

2
;
c) 4x
3
+ 2x
2
8x + 7 con x =
1
2
(

3 + 1);
d)
x +y 1
x y + 1
con x =

x + 1

xy + 1
y y =

xy +

xy 1
;
e)

a +x +

a x

a +x

a x
con x =
2ab
1 +b
2
;
f ) 2a
_
1 +x
2

_
x +
_
1 +x
2
_
1
con x =
1
2
_

ab
1

ba
1
_
.
8. Simplique las expresiones:
CAPTULO 4. EXPRESIONES ALGEBRAICAS 192
a)
_
7 + 4

3;
b)
_
3 2

2);
c)
__
5 + 2

6 +
_
5 2

6
_

3
2
;
d)
2 +

2 +
_
2 +

3
+
2

2
_
2

3
;
e)
_
4

2 + 2

6;
f )
4
_
17 +

288;
g)
4
_
28 16

3;
h)
_
17 4
_
9 + 4

5;
i)

3 +
_
5
_
13 +

48.
9. Simplique las expresiones:
a)
1
4

5
4

2
;
b)
1
3

15
3

7
;
c)
_

5 +

3
_

3
;
d)
1
1 +

2 +

3
;
e)
1
3

4 +
3

6 +
3

9
;
f )
1
_

2 +
3

3
;
g)
1
4

2 +
4

4 +
4

8 + 2
;
h)
1

14 +

21 +

15 +

10
;
i)
2 +

6
2

2 + 2

6 2
.
10. Simplique las expresiones:
a)
_
4

8
_

2 + 1
_
4

8 +
_

2 1
_
_
4

8
_

2 1
;
b) (2

3)
3
_
26 + 15

3;
c)
2
3

2
1 +

3
;
d)
_
5 2

6 (5 + 2

6)(49 20

6)

27 3

18 + 3

12

8
;
e)
_
6 + 4

2 +
_
6 + 4

2
+
6 4

2
_
6 4

2
_
2
;
f )
_
3
[
3
]64
3

25
+
3

40
3

8 +
3

10

25
_
1
(13 4
3

5 2
3

25) +
3

25;
g)
3

6 +
_
847
27
+
3

6
_
847
27
;
h)
_
5

2 + 7
3
_
5

2 7.
11. Simplique la expresin:
a)
_

ab +b

a +

b
_

_

a

a +

b
+

b
+
2

ab
a b
_
;
b)
a + 2 +

a
2
4
a + 2

a
2
4
+
a + 2

a
2
4
a + 2 +

a
2
4
;
CAPTULO 4. EXPRESIONES ALGEBRAICAS 193
c)
a
1
+b
1
+ 2
_

a +

b
_
1

_
1

a
+
1

b
_
_
ab a

ab
a +

ab
_
1
;
d)
a
_

a +

b
2b

a
_
1
+b
_

a +

b
2a

b
_
1
_
a +

ab
2ab
_
1
+
_
b +

ab
2ab
_
1
;
e)
1
2
ab

8a
3
b +
1
3
ab

18ab
3
a
2

_
2b
a
b
2

_
2a
b
;
f )
_

a +
ab
2
+c

ab
2
+c
_

_
b

a +b
_
ab
2
+c
_
;
g)
2a
1/3
a
2/3
3a
1/3

a
2/3
a
5/3
a
2/3

a + 1
a
2
4a + 3
;
h)
4
_
6a(5 + 2

6)
_
3

2a 2

3a;
i)
3
_

5
6
_
8 + 2

15 +

a
3
_

24 +

12
6
_
8 2

15 2
3

2a +
3

a
2
;
j)
[(
4

a +
4

b)
2
(
4

a +
4

b)
2
]
2
(16a + 4b)
4a b
+
10

a 3

b
2

a +

b
;
k)

_
a
2
+
4
a
2
_
2
8
_
a +
2
a
_
2
+ 48;
l)
a
2
+ 2a 3 + (a + 1)

a
2
9
a
2
2a 3 + (a 1)

a
2
9
;
m)

a +
_
a
2
4
a
+

a
_
a
2
4
a
;
n)
_
_
3

(x
2
+ 1)
_
1 +
1
x
2
+
3

(x
2
1)
_
1
1
x
2
_
_
2
;
o)
_
4

ab

ab
1

ab
+
1
4

ab
4

ab
_
:
4

ab
1
4

a
3
b
3

1
4

ab

ab

ab
;
p)
m+n

m+

n
:
_
m+n

mn
+
n
m

mn

m
n +

mn
_
;
q)
_
1 +a

1 +a

1 a
+
1 a

1 a
2
1 +a
_
_
_
1
a
2
1
1
a
_
;
r) 2a

_
1 +
1
4
_
_
a
b

_
b
a
_
2
:
_

_
1
2
_
_
a
b

_
b
a
_
+

_
1 +
1
4
_
_
a
b

_
b
a
_
2
_

_;
s)
_
1

a 4

a
1

2
3

a
3

a
4

64a
_
2

_
a
2
+ 8a + 16;
CAPTULO 4. EXPRESIONES ALGEBRAICAS 194
t)
_
_

_
a +b
2

ab
1
_
1
+

_
a +b
2

ab
+ 1
_
1
_
_
:
_
_

_
a +b
2

ab
1
_
1

_
a +b
2

ab
+ 1
_
1
_
_
;
u)
_
_

(1 a)
3

1 +a
a

3
_
3a
2
4 8a + 4a
2
_
_
1

_
3a

a
2

1 a
2
_
1
;
v)
_
(1 a)
1/4
2(1 +a)
3/4
+
(1 +a)
1/4
(1 a)
3/4
2
_
(1 a)
1/2
_
1 +a
1 a
_
1/4
;
w)
_
a +

a
2
1
a

a
2
1
+
1
a

a
2
+1
1 +
a

a
2
1
_
:
_
a
1
a
_
1
a
;
x) b
__
a
4

a +
4

a
2
b
3
4

a
3
+
4

a
2
b

ab
_
:
_
4

a
4

b
_

a
_
1
;
y)
_
3

a
2
b
3

ab
2
3

a
2
2
3

ab +
3

b
2

a +b
3

a
2

b
2
_
_
6

a
6

b
_
1
+
6

a;
z)
_
a 2b
3

a
2

4b
2
+
3

2a
2
b +
3

4ab
2
3

a
2
+
3

4b
2
+
3

16ab
_
:
a
3

a +b
3

2b +b
3

a +a
3

2b
a +b
.
12. Simplique la expresin:
a)
(a b)
3
(

a +

b)
3
+ 2a

a +b

b
a

a +b

b
+
3(

ab a)
a b
;
b)
_
1
a
1/3
a
1/6
+ 1
+
1
a
1/3
+a
1/6
+ 1

2a
1/3
2
a
2/3
a
1/3
+ 1
_1

1
4
a
4/3
;
c)
_
_
4

b(
4

a
4

b) + 2
4

ab
(
4

a
4

b)
2

_
_
b
a
+ 1
_
1
+ 1
_
_
1/2

ab;
d)
_
(a +b)(
3

a
2

b
2
)
1
(
3

a
2
b
3

ab
2
)(
3

b
3

a)
2
(
6

a +
6

b)(
3

b +
6

ab 2
3

a)
_
1
+ 2
6

a.
4.9. Magnitudes directa e inversamente proporcionales
Dos magnitudes son directamente proporcionales, cuando al multiplicar o dividir el valor de
una de ellas por un nmero, el valor correspondiente de la otra queda multiplicado o dividido por
el mismo nmero.
Dos magnitudes son inversamente proporcionales cuando el producto entre dos cantidades cor-
respondientes es constante. A esta constante se le denomina constante de proporcionalidad.
Para realizar el reparto de una cantidad de forma inversamente proporcional a unas cantidades,
es equivalente a repartirla de forma directamente proporcional a los inversos de las cantidades.
Haremos lo siguiente:
1. Se suman las cantidades inversas a repartir.
CAPTULO 4. EXPRESIONES ALGEBRAICAS 195
2. Se divide la cantidad por esta suma. El cociente nos dar la constante de proporcionalidad.
3. Para calcular cada parte basta con multiplicar cada cantidad por esa constante.
Ejemplo 4.30 Cules de los siguientes pares de magnitudes son directamente proporcionales?
1. La velocidad de un automvil y el tiempo que tarda en realizar un mismo recorrido.
(No) son directamente proporcionales. Si la velocidad se hace doble, triple, ..., el tiempo
necesario para hacer el mismo recorrido no es doble, triple, ...
2. La distancia recorrida por un automvil y el tiempo empleado, manteniendo la misma
velocidad.
(S) son directamente proporcionales. Si la distancia se hace doble, triple, ..., el tiempo deber
ser doble, triple, ...
3. La longitud del lado de un cuadrado y la supercie del mismo.
(No) son directamente proporcionales. Si la longitud se hace doble, triple, ..., la supercie no
es doble, triple, ...
4. La edad de una persona y su estatura.
(No) son directamente proporcionales. Si la edad se hace doble, triple, ..., la estatura no es
doble, triple, ...
Ejemplo 4.31 Si por un auto se paga $ 8000, por 2 se paga $ 16000, por 3 se paga $ 24000,
entonces tenemos que
_

_
1 8000
2 16000
3 24000
La razn entre cada medida de la magnitud precio y el nmero de autos que le corresponden, es la
misma. Es decir
8000
1
=
16000
2
=
24000
3
= 8000
Si designamos por x el nmero de autos y por y el precio correspondiente, se tiene
y
x
= 6 y = 6x.
4.10. Razones y proporciones
Las razones y proporciones tienen una gran aplicacin en diversas reas; por ejemplo, en inge-
niera se emplean las escalas para realizar maquetas, en el rea contable, para realizar movimientos
nancieros, etc.
Una razn es la comparacin por cociente de dos nmeros. Este cociente se interpreta como el
nmero de veces que uno de ellos es mayor que el otro, esto se expresa como
A
B
=
C
D
, B ,= 0 y D ,= 0.
En una razn, al trmino A se le llama antecedente y al trmino B, consecuente.
CAPTULO 4. EXPRESIONES ALGEBRAICAS 196
4.10.1. Proporcionalidad directa
La proporcionalidad directa entre las cantidades x y y est dada por una expresin de la forma
y = x
Esto signica que la variable y tiene una variacin proporcional a la variable x: Cuanto aumenta
x, con el mismo tanto , aumenta y.
La proporcionalidad directa aparece comnmente en las relaciones entre las variables principales
de fenmenos o procesos naturales. Para ejemplo, cuando se dice: Durante una reaccin de primer
orden, la cantidad de un reactivo que permanece por unidad de tiempo es proporcional a la cantidad
que reacciona, si Q
t
es la cantidad de reactivo al tiempo t y Q+t + 1 es la cantidad por reaccionar
una unidad de tiempo despus, se habla de una relacin de la forma
Q
t+1
= Q
t
donde es la constante de proporcionalidad.
Se observa que si la variable x es directamente proporcional a la variable y, entonces de las
parejas relacionadas (x
1
, y
1
), (x
2
, y
2
) mediante las igualdades y
1
= x
1
, y
2
= x
2
se obtiene, al
dividirlas miembro a miembro
y
1
y
2
=
x
1
x
2
=
x
1
x
2
la cual se conoce como la regla de tres. Esto nos permite resolver problemas sin tener que calcular
la constante de proporcionalidad .
4.10.2. Proporcionalidad inversa
Existe otro tipo de proporcionalidad entre las cantidades x y y, que tiene la forma
y =

x
Esta es llamada proporcionalidad inversa con constante . Tal tipo de proporcionalidad aparece
tambin en los procesos y fenmenos de la naturaleza. Por ejemplo, cuando se dice: En un gas ideal
a temperatura constante, la presin que ejerce el gas es inversamente proporcional al volumen que
ocupa, esto puede escribirse como
P =

V
donde P es la variable presin, V es el volumen del gas y es la constante de proporcionalidad.
Algunos de los principios ms conocidos de la ciencia pueden expresarse como variaciones. A
continuacin se mencionan algunas:
Las reas de las guras semejantes son directamente proporcionales a los cuadrados de las lneas
correspondientes.
Los volmenes de los slidos semejantes son directamente proporcionales a los cubos de las lneas
correspondientes.
Los volmenes de los gases son inversamente proporcionales a la presin absoluta y directamente
proporcionales a la temperatura absoluta.
En cualquier reaccin qumica entre sustancias A y B, la cantidad de la sustancia A que interviene
en la reaccin es directamente proporcional a la cantidad de la sustancia B que tambin interviene.
CAPTULO 4. EXPRESIONES ALGEBRAICAS 197
Ejemplo 4.32 Escriba, mediante una frmula, las siguientes proposiciones:
a) w vara directamente como x e y.
b) w es directamente proporcional a x e inversamente proporcional a y.
c) w es directamente proporcional al cubo de x e inversamente al cuadrado de z.
d) w es directamente proporcional a la raz cbica de b e inversamente a la raz cuadrada de c.
e) R es directamente proporcional a w y a la raz cuadrada de x e inversamente proporcional al
cubo de h.
Solucin
a) Si es la constante proporcionalidad entre las variables dadas, la relacin se escribe
w = xy.
b) Utilizamos una sola constante para escribir la relacin entre las variables, que quedan
w =
x
y
.
c) Si se utiliza a como constante de proporcionalidad, lo anterior se escribe
V =
x
3
z
2
.
d) Se tiene en este caso, la relacin
w =

3

c
donde es una constante de proporcionalidad.
e) Para este caso, se utiliza igualmente una sola constante de proporcionalidad para todas las
variables, obtenindose la relacin
R =

xw
h
3
.
Ejemplo 4.33 La variable N es inversamente proporcional a y. Adems se sabe que N = 20
cuando y = 0, 35. Calcular la relacin entre las variables dadas.
Solucin
Ya que para alguna se tiene que N =

y
, de las condiciones N = 20, y y = 0, 35 se tiene que
20 =

0,35
. Esto implica que = 20 0, 35 = 7. De esta forma la relacin entre N y y es N =
7
y
.
Ejemplo 4.34 P es inversamente proporcional a V . Si V = 30 litros cuando P = 2 atms-
feras, hallar V cuando P = 25 atmsferas.
Solucin
Ya que P =

V
y 2atmsferas =

30 litros
, entonces = 60 atm x lt, lo que implica que
P =
60
V
.
De esta manera, si P = 25 atm, entonces 25 =
60
V
, lo cual implica que
V =
60 atm x lt
25 atm
= 2, 4 lt.
Ejemplo 4.35 La variable C es directamente proporcional a d
2
. Si C = 80 cuando d = 12,
hallar C cuando d = 15.
CAPTULO 4. EXPRESIONES ALGEBRAICAS 198
Solucin
Si c = d
2
entonces de la pareja de igualdades 80 = 12
2
y C = 15
2
se tiene que, al dividir
miembro por miembro la segunda igualdad y la primera
C
80
=
15
2
12
2
=
15
2
12
2
obtenemos el valor de la constante
C =
_
15
12
_
2
80 = 1, 5625.
Ejemplo 4.36 La variable v es directamente proporcional a

h. Si v = 28 cuando h = 3,
hallar v cuando h = 12.
Solucin
Ya que v es directamente proporcional a

h entonces v =

h, lo que nos lleva a las igualdades


28 =

3 y v =

12. Al dividir miembro a miembro la segunda igualdad entre la primera se


tiene que
v
28
=

12

3
=

12

3
=
_
12
3
= 2.
De esta forma v = 2 28 = 56.
Ejemplo 4.37 La variable R es directamente proporcional a l e inversamente proporcional a
d
2
. Si R = 35 cuando l = 110 y d = 0, 006. Hallar R cuando l = 75 y d = 0, 004.
Solucin
De la relacin R =
l
d
2
y de las condiciones dadas se tienen las igualdades 35 =
110
0, 006
2
y R =
75
0, 004
2
nuevamente, al dividir la segunda ecuacin miembro a miembro con la primera se obtiene
R
35
=
75
0,004
2
110
0,006
2
=
75 0, 006
2
110 0, 004
2
=
75
110

_
0, 006
0, 004
_
2
= 1, 534.
lo cual implica que R = 1, 534 35 = 53, 69.
Ejemplo 4.38 El hidrgeno usado para inar globos se obtiene haciendo pasar vapor de agua
sobre una malla de hierro al rojo vivo. Si con 390 gr de hierro se obtienen 2,2 m
3
de hidrgeno,
cunto hierro se necesitar para obtener 33 m
3
?
Solucin
CAPTULO 4. EXPRESIONES ALGEBRAICAS 199
Denotemos por h la cantidad de hierro necesario en g para obtener H m
3
de hidrgeno. Entonces
es claro que la relacin entre una pareja (h
1
, H
1
) y (h
2
, H
2
) viene dada por la regla de tres
H
1
h
1
=
H
2
h
2
.
De esta manera, para las condiciones dadas se tiene la relacin
2, 2
390
=
33
h
lo cual nos dice que el hierro necesario para obtener 33 m
3
de hidrgeno es
h =
33
2, 2
390 = 5850 gr.
Ejemplo 4.39 La distancia area entre los puertos A y B es de 325 km. Los puertos distan
18 cm en un mapa. Cul es la distancia area entre los puertos C y D que distan 23 cm en el
mismo mapa?.
Solucin
Sea D la distancia area entre A y B y d su correspondiente distancia sobre el mapa. De la
relacin de proporcionalidad directa se tiene que en correspondientes (d
1
, D
1
), (d
2
, D
2
) se cumple
la igualdad
D
1
d
1
=
D
2
d
2
.
De esta forma, para las condiciones se tiene la ecuacin
325
18
=
D
23
que equivale a
D =
325
18
23 = 415, 27 km.
Ejemplo 4.40 Un disco de 40,6 cm de dimetro pesa 2,570 gr. Cul ser el dimetro de un
disco del mismo espesor que pesa 945 gr?
Solucin
Ya que ambos discos tienen el mismo espesor, apenas varan sus reas segn el cuadrado de sus
dimetros. Como el material es el mismo, se tiene que la densidad es igual y entonces el peso del
disco vara segn vare el rea y, por lo tanto, depende de cmo vara el diametro.
Por otro lado, las reas de guras semejantes son directamente proporcionales a los cuadrados de
sus lneas correspondientes. De esta forma, se guarda una relacin de proporcionalidad para las
parejas (d
2
1
, P
1
) y (d
2
2
, P
2
) de la forma
P
1
d
2
1
=
P
2
d
2
2
donde P es el peso del disco y d es su dimetro. Por lo tanto, para P
1
= 2, 570, d
1
= 40, 6 y
P
2
= 945 se cumple una igualdad
2, 570
40, 6
2
=
945
d
2
2
lo que implica que
d
2
=
_
945
2, 570
40, 6
2
=
_
945
2, 570
40, 6.
CAPTULO 4. EXPRESIONES ALGEBRAICAS 200
es el dimetro del disco mencionado.
Ejemplo 4.41 Una esfera de hierro de 6,3 cm de dimetro pesa 850 gr. Cunto pesar otra
esfera de hierro de 9,2 cm de dimetro?
Solucin
Como las esferas son semejantes, entonces sus volmenes son proporcionales a los cubos de sus
radios. Por lo tanto, sus pesos correspondientes P
1
, P
2
guardan la relacin de proporcionalidad con
los cubos de los dimetros
P
1
d
3
1
=
P
2
d
3
2
donde d
1
es el dimetro de la esfera de peso P
1
y d
2
es el de la esfera de peso P
2
. Para los datos
dados P
1
= 850, d
1
= 6, 3, d
2
= 9, 2, P
2
=? se cumple la relacin
850
6, 3
3
=
P
2
9, 2
3
lo cual implica que el peso P
2
buscado es
P
2
=
850
6, 3
3
9, 2
3
= 850
_
9, 2
6, 3
_
3
= 2647, 04 gr.
Ejemplo 4.42 La frmula D =
PL
3
th
3
, da la deexin de una viga, de longitud L entre los
puntos de apoyo, con una carga P en el centro, una anchura t y un grosor h. Si D es 4 cuando
P = 250, L = 12, h = 3 y t = 2, 5, hallar D cuando P es 400, L es 10, h es 4 y t es 2.
Solucin
De la relacin de proporcionalidad
D =
PL
3
th
3
sujeta a los argumentos dados, se tiene la igualdad
4 =
250 12
3
2, 5 3
3
lo cual implica que la constante de proporcionalidad es
=
4 2, 5 3
3
250 12
3
=
270
898560
= 0, 0003.
De esta manera, la relacin obtenida es
D =
0, 0003PL
3
th
3
Esto implica que para los argumentos P = 400; L = 10, h = 4, t = 2 se tiene una deexin
D =
0, 0003 400 10
3
2 4
3
= 0, 937.
CAPTULO 4. EXPRESIONES ALGEBRAICAS 201
Ejemplo 4.43 La cantidad C del agua que sale por un oricio en el fondo de un depsito es
directamente proporcional a la raz cuadrada de la altura h de la supercie libre del lquido. El
caudal es de 85 litros/minuto cuando la altura es de 2,56 m:
a) Encuentre una frmula de C dependiendo de h.
b) Calcule C cuando h = 4, 62 mt.
c) Encuentre h cuando C = 62 litros/minuto.
Solucin
La relacin entre las variables C y h tiene la forma C =

h para alguna constante real .


Si para h = 2, 56 m se tiene que C = 85 lt/min, entonces se tiene la igualdad 85 =

2, 56 lo que
nos da constante de proporcionalidad =
85

2, 56
= 53, 125. De esta forma C = 53, 125

h lo que
responde al inciso a).
Utilizando esta relacin, se tiene que si h = 4, 62 mt, entonces el valor asociado a C es C =
53, 125

4, 62 = 114, 18 lt/min, lo cual responde la pregunta b).


Finalmente, si C = 62 lt/min entonces, de la relacin 462 = 53, 125

h se obtiene la igualdad
h =
_
62
53, 125
_
2
= 1, 36 mt.
Ejemplo 4.44 Un hombre de 1,70 mt de estatura pesa 75 kg. Otro hombre, de constitucin
parecida, mide 1,80 mt. Cul ser el peso del segundo?
Solucin
Ya que ambos hombres tienen una constitucin parecida, podemos suponer que tienen en su forma
voluminosa una semejanza, y que por tanto, sus longitudes correspondientes (tallas) son propor-
cionales a sus pesos. Esto es, si l
1
es la talla asociada al peso P
1
del primer hombre y l
2
es la talla
asociada al peso P
2
del otro hombre, entonces es justa una relacin
P
1
l
3
1
=
P
2
l
3
2
Ya que para nuestro caso l
1
= 1, 7 mt y P
1
= 75 kg, entonces para el segundo hombre se cumple
75
1, 7
3
=
P
2
1, 8
3
es decir, el peso P
2
del hombre es
P
2
=
75 1, 8
3
1, 7
3
= 75
_
1, 8
1, 7
_
3
= 89, 02 kg.
Ejemplo 4.45 La distancia del horizonte, en el mar, es directamente proporcional a la raz
cuadrada de la altura del observador sobre el nivel del mar. Si el horizonte est a 7,2 km para una
altura de 4,1 mt, hallar la distancia correspondiente a una altura de 110 mt.
Solucin
Entindase por d la distancia del horizonte en el mar y por h a la altura de un observador sobre el
nivel del mar. Entonces, es justa una relacin entre tales variables
d =

h
CAPTULO 4. EXPRESIONES ALGEBRAICAS 202
Dadas las condiciones d = 7, 2 km y h = 4, 1 m se tiene que
7, 2 =
_
4, 1 = 2, 02
lo que implica que
=
7, 2
2, 02
= 3, 56
es la constante de proporcionalidad buscada.
As, la relacin entre las variables es d = 3, 56

h. De esta manera, para h = 110 mt se tiene una


distancia al horizonte
d = 3, 56

110 = 37, 33 km.


Ejemplo 4.46 Una persona, al comprar una torre de 100 cds, verica que 4 estn defectu-
osos, encuentre la razn.
Solucin
La razn que se obtiene es
4 cds defectuosos
100 torre de cds
Simplicando esta razn, se tiene
4
100
=
1
25
Lo cual se interpreta como: de cada 25 cds, 1 est defectuoso.
4.10.3. Proporcin
Denicin 4.17 Proporcin
Se llama proporcin a la equivalencia entre dos razones y, se representa por
A
B
=
C
D
, B ,= 0 y D ,= 0.
En una proporcin, a los trminos A y D se les denomina extremos y a B y C, medios.
Ejemplo 4.47 Una persona, compr una torre de 100 cds y pag por ella $ 23. Si necesita
600 cds, cunto deber pagar?
Solucin
En este caso, tenemos
100 cds
$23
=
600 cds
x
100x = 23 600
x =
23 600
100
x = $138.
Es decir, las 6 torres de 100 cds cuestan 138 dlares.
4.11. Tarea
1. Determine el extremo desconocido en las siguientes proporciones:
CAPTULO 4. EXPRESIONES ALGEBRAICAS 203
a)
3
2
x
1, 333... 1
=
x
(9 0, 666...)
1
;
b)
x
2
5
0, 1
=
1
2
(3 0, 6)
(0, 1)
2
1
;
c)
x

5
4
=
2
3
1
12
;
d)
x
1
2

_
4
9
=
1
2

_
1
16
0, 099...
;
e)
2 +
1
3
_
1
4
=
1
3

1
5
x
;
f )
x
3
=
8
9, 6
;
g)
1
4
1
0, 5
_
1
16
=
0, 5
_
1
16
x
;
h)
x
1
5
=
5
3
4
9
;
i)
(3 + 0, 5)
_
1
49
x
=
x
1
1
2
;
j)
(0, 1)
2
_
1
5
9
_
1
3
5
_
2
=
(0, 5 1)
_
1
2
3
_
x
;
k)
x
3
4
0, 05
=
3
4
0, 05
1, 8 0, 4
;
l)
(0, 2)
2
+ 1, 46
x
=
(1, 1)
2
+ 0, 29
0, 5
;
m)
2
5
1
6
=
2
5
x
;
n)
x
1 + 0, 2
=
1
(0, 1 + 0, 3)
2
;
o)
(1 0, 2)
2
0, 4
=
0, 4
x
;
p)
_
1
5
9
x
=
x
_
3
_
1
19
27
_
2

8
3
;
q)
_
3
1
2
_
2
x
=
x
2
3
_
1
7
8
;
r)
x
0, 3
1
2
=
1
6
0, 5
_
1
2
3
_
2
;
s)
0, 1(1 0, 1)
0, 1 1 0, 4
=
1
4
1
x
;
t)
(1, 2 0, 3)
2
(0, 1)
3
=
x
0, 7 1, 3
;
u)
5

8
=
10

2
x
;
v)
0, 5
1
2
=
2
x
;
w)

5
8
1
2

3
=
1
2

3
x
;
x)
x
3

2
=

2
0, 03
.
2. Determine el extremo desconocido en las siguientes proporciones:
a)
0, 2 4, 333...
_
1
81
=
x
5
34
0, 444...
;
b)
5, 1515...
_
1 +
5
4
x
=
(1 1, 4)
2
1, 2 +
1
2
0, 4
;
c)
_
3
1
2
_
3
x
=
_
1
9
25
(0, 1)
3
_
1
3
4
;
d)
0, 055... 2
1
x
=
0, 888...
3
_
7
8
1
;
e)
1, 5 1
0, 333... 2
=
_
1
16
25
x
;
f )
0, 666... 1
1, 111...
=
x
1
2
3 2x
;
g)
x
1, 222... 2
=
1, 222... 2
_
1
2
3
_
2
;
h)
(0, 1)
2
_
1
5
_
2
3
_
8
25
5
=
_
1
5
_
2
3
_
8
25
5
x
;
i)
0, 5(0, 1 1)
x
=
x
(0, 222... + 2)
1
;
j)
x
3, 033...
=
0, 888... 1
1
0,555...
+ 0, 222...
;
k)
(1, 1)
2
0, 1 0, 2
2, 5 8 0, 1
=
x
1 0, 1
3

0, 343
.
3. Determine el extremo desconocido en las siguientes proporciones:
CAPTULO 4. EXPRESIONES ALGEBRAICAS 204
a)
_
1
_
1
2
1
_
2
_
21
x
=
x
0, 111...
_
2 +
1
4
;
b)
(1, 222... 0, 333...)
1
1
_
2
3
1
2+
1
3
_
1
=
(1, 5 1, 999...)
3
x
;
c)
_
3
4
2
_
(0, 1515...)
1
x
=
x
(1, 3636...) 5
;
d)
(0, 2)
1
+ (0, 1 0, 01) (0, 3)
2

1 0, 6
=
_
(1 0, 7)
2
0, 1
x
;
e)
x
3, 333...
=
3
_
1
37
64
(0, 5 1)
2

1
92
1, 022... (2, 5)
2
;
f )
18
_
4
3

1
2
_
_
1, 111...
_
1
3
_
2
_
2
x
=
(0, 666...)
2
(1, 222...)
1
(1 0, 5)
2
.
4. Dos nmeros, cuya suma es 28, guardan entre s la relacin
3
4
. Cules son esos nmeros?
5. Descomponer el nmero
35
6
en dos partes tales que cuya razn sea
3
2
.
6. La suma de los cuadrados de dos nmeros positivos es 25. Si la razn entre ellos es
2
1, 5
.
Cules son los nmeros?
7. Calcular los dos nmeros naturales tales que su diferencia es 9 y su razn es
11
8
.
8. Calcular dos nmeros naturales de 2 cifras cada uno cuya razn es
7
2
y tales que tienen
iguales la cifra de las unidades y la de las decenas dieren en 3.
9. La diferencia de los cuadrados de dos nmeros es 5 y el cociente de los nmeros es 1,5.
Calcular esos nmeros.
10. Podemos considerar que una gota de agua tiene forma cbica cuya arista mide aproxi-
madamente 1 mm = 1 x 10
3
mt:
a) Calcular el volumen de una gota.
b) Calcular el nmero de gotas que caben en un tinaco de 1000 litros.
Resp: a) 1 x 10
9
mt
3
; b) 1 x 10
9
gotas.
11. Suponiendo que un protn tenga forma cbica, cuya arista sea de 10
13
cm, calcule su
volumen.
Resp: 10
39
.
CAPTULO 4. EXPRESIONES ALGEBRAICAS 205
12. Considerando que la masa de un protn es de 10
24
gramos, determine su densidad (la
densidad de un cuerpo se obtiene al dividir su masa entre su volumen).
Resp: 10
15
cm.
13. Al colocar con mucho cuidado sobre una supercie libre de un recipiente con agua, una
gota de aceite cuyo volumen es V = 6x10
2
cm
3
, la misma se dispersa y forma una capa
muy na cuya rea es A = 2x10
4
cm
2
. Calcule el espesor de esta lmina de aceite.
Resp: 3 x 10
6
cm.
14. Si V es directamente proporcional a m e inversamente al cuadrado de t, calcular V = 2
cuando m = 15 y t = 6.
15. v es directamente proporcional a d
2
. Si C = 80 cuando d = 12, Hallar C cuando d = 15.
16. R es directamente proporcional a la cuarta potencia de T e inversamente a la raz cuadrada
de N. Calcular si R =
1
3
cuando T = 2 y N = 36.
17. La variable M es directamente proporcional a d
2
. Si M = 12 gr cuando d = 8 cm, calcular
M cuando d = 12 cm.
18. La variable N es inversamente proporcional a d
2
. Si N = 10,890 plantas por hectrea
cuando las plantas distan d = 2 mt, hallar N cuando d = 5, 5 mt.
19. Si la variable v vara conjuntamente como la raz cuadrada de g y la raz cuadrada de h.
Si v = 14 mt/seg cuando g = 9, 8 mt/seg
2
y h = 10 mt, hallar v cuando g = 9, 81 mt/seg
2
y
h = 2 mt.
20. La variable V es directamente proporcional a r
4
y p e inversamente proporcional a l. Si
V = 120 cuando r = 0, 012, p = 20 y l = 30, calcular V cuando r = 0, 015, p = 36 y l = 25.
21. La variable a es directamente proporcional a v
2
e inversamente proporcional a r. Si a = 540
cuando v = 84 y r = 5, hallar a cuando v = 119 y r = 4.
22. Un matraz Erlenmeyer de 250 ml tiene una altura de 12,6 cm
2
. Qu altura debera tener
otro matraz de la misma forma para que su capacidad sea 500 ml?
23. El anlisis de una pintura muestra un 54 % de pigmento y un 46 % de aglomerante. El
pigmento est compuesto de 15 % de la sustancia A, 60 % de la sustancia B, y 25 % de la
sustancia C. Cul es el porcentaje de cada sustancia en la pintura?
CAPTULO 4. EXPRESIONES ALGEBRAICAS 206
24. Se recorta de un mapa el perl de una nca y se encuentra que pesa 42,78 gr. Una seccin
rectangular de 12,2 por 20,2 cm, del mismo mapa, pesa 5,31 gr. Si la escala del mapa es de
2,5 cm por 50 mt, hallar el rea de la nca en metros cuadrados.
25. Para abastecer de agua a una ciudad de 50.000 habitantes se usa un tubo de 62 cm de
dimetro. Si se espera alcanzar una poblacin de 120.000 individuos en un tiempo de 30 aos,
qu dimetro debe tener la nueva tubera?
26. La potencia necesaria para impulsar una lancha es proporcional al cubo de su velocidad. Si
un motor de 5 HP permite alcanzar una velocidad de 16 km/h, qu potencia se necesitara
para conseguir una velocidad de 22 km/h?
27. Se compra un lote de sosa, que contiene 52 % en peso de agua de cristalizacin, a 17,5
centavos por libra. Cuando se vende al por menor, se encuentra que el contenido en agua a
descendido a 37 %. Cul debe ser el precio de venta para obtener una ganancia de un 40 %?
4.12. Regla de tres y porcentajes
La regla de tres es una operacin que tiene por objeto hallar el cuarto trmino de una pro-
porcin, cuando se conocen tres. En una regla de tres, siempre debe existir un supuesto y pregunta.
En una regla de tres el supuesto est constituido por los datos de la parte del problema que ya
se conoce y la pregunta por los datos de la parte del problema que contiene la incgnita.
De acuerdo a la relacin con la incgnita, puede ser directa cuando los aumentos en una variable
provocan aumento en la otra variable o inversa cuando los aumentos en una variable provocan
disminucin en la otra variable.
4.12.1. Regla de tres simple
Los problemas en los que los elementos mantienen una relacin proporcional directa o inversa,
se resuelven mediante la regla de tres simple. Es simple cuando solamente intervienen en ella dos
magnitudes, esta a su vez puede ser:
1. Regla de tres simple directa: La regla de tres simple directa es una relacin que
se establece entre tres o ms valores conocidos y una incgnita. Normalmente se usa cuando se
puede establecer una relacin de linealidad entre todos los valores involucrados. Normalmente se
representa de la siguiente manera:
_
Supuesto : A B
Pregunta : C x
Ax = BC x =
BC
A
.
Siendo A, B y C valores conocidos y x la incgnita cuyo valor queremos averiguar. Esto se lee de
la siguiente forma: x es a C como A es a B.
CAPTULO 4. EXPRESIONES ALGEBRAICAS 207
Ejemplo 4.48 Si 5 telfonos cuestan 150 dlares, cunto costarn 25 telfonos?
Solucin
Estas cantidades son directamente proporcionales y sabemos que la proporcin se forma igualando
las razones directas:
_
Supuesto : 5 150
Pregunta : 25 x
x =
25 150
5
= 750 dlares.
2. Regla de tres simple inversa: Cuando la cantidad aumenta y la otra disminuye pro-
porcionalmente se dice que existe una relacin inversa. Esta es una regla de tres simple inversa.
En las reglas de tres inversas las relaciones se establecen entre pares de cantidades que van de ms
a menos o de menos a ms:
_
Supuesto : A B
Pregunta : C x
Cx = AB x =
AB
C
.
Ejemplo 4.49 Si 5 personas realizan una labor en 8 das, en cuntos das podran hacer la
misma tarea 12 personas?
Solucin
A ms personas, menos das. Estas cantidades son inversamente proporcionales y sabemos que la
proporcin se forma igualando la razn directa de las dos primeras con la razn inversa de las dos
ltimas o viceversa:
_
Supuesto : 5 8
Pregunta : 12 x
x =
5 8
12
= 3
1
3
das.
4.12.2. Regla de tres compuesta
Cuando la cantidad de magnitudes que aparece en un problema es mayor que dos, nos enfrenta-
mos a un problema que se puede resolver mediante una regla de tres compuesta. Estos problemas
son equivalentes a varios problemas de regla de tres simple encadenados. De acuerdo a si las mag-
nitudes de cada uno de ellos son directa o inversamente proporcionales, encontraremos tres casos:
1. Regla de tres compuesta directa: Si cada una de las magnitudes que aparecen es
directamente proporcional a la magnitud de la cantidad que se quiere calcular, el problema se
llama regla de tres compuesta directa.
Ejemplo 4.50 Un paseo de n de ao para 30 personas por 15 das cuesta 65700 dlares.
Cunto costar en iguales condiciones, el paseo a 25 personas, durante 8 das?
Solucin
Para resolver una regla de tres compuesta se consideran, consecutivamente, dos reglas de tres sim-
ples. Procedemos de la siguiente manera:
_
Supuesto : 30 67500
Pregunta : 25 x
CAPTULO 4. EXPRESIONES ALGEBRAICAS 208
x =
25 67500
30
= 56250 dlares.
Al plantear la segunda regla de tres simple aparece como dato x = 56250 dlares hallados en la
primera regla de tres:
_
Supuesto : 15 56250
Pregunta : 8 x
x =
8 56250
15
= 30000 dlares.
Este problema tambin se puede resolver de la siguiente manera:
_
Supto : 30 pers 15 das $ 67500
Pregta : 25 pers 8 das $ x
x =
25 8 67500
30 15
= 30000 dlares.
Es decir, el paseo les costar a las 25 personas, durante 8 das, 30000 dlares.
2. Regla de tres compuesta inversa: La regla de tres simple inversa es un mtodo para
hallar una cantidad que forma proporcin con otras cantidades conocidas de dos o ms magnitudes
inversamente proporcionales. Dos magnitudes son inversamente proporcionales cuando al aumentar
una cantidad, disminuye la otra.
Ejemplo 4.51 5 obreros trabajando 8 horas diarias han realizado 150 metros de una obra en
5 das. Cuntos das necesitarn 8 obreros, trabajando 8 horas diarias, para hacer 120 metros de
la misma obra?
Solucin
Para resolver este problema, procedemos de la siguiente manera:
_
Supuesto : (+)5 obr. (+)8 h.d. ()150 mt. (+)5 das
Pregunta : ()8 obr. ()8 h.d. (+)120 mt. x das
De esta manera obtenemos:
x =
5 8 5 120
8 8 150
= 2
1
2
das.
3. Regla de tres compuesta mixta: Si hay algunas directas y otras inversamente pro-
porcionales a la de la incgnita, se llama regla de tres compuesta mixta.
La regla de tres compuesta, tambin se puede solucionar por el mtodo de las proporciones que
consiste en descomponer la regla de tres compuesta en reglas de tres simples y luego multiplicar
ordenadamente las proporciones formadas. Al formar cada regla de tres simple, se considera que
las dems magnitudes no varan.
4.12.3. Porcentajes
Los problemas del tanto por ciento, se resuelven ya sea aplicando regla de tres o por medio de
fracciones. Un porcentaje es una forma de expresar una proporcin o fraccin como una fraccin
CAPTULO 4. EXPRESIONES ALGEBRAICAS 209
de denominador 100.
La expresin p por ciento de a signica p centsimos de a, es decir
p
100
x a = p x
a
100
El p por ciento de a se denota tambin por el signo p % de a. El porcentaje aparece en la vida
diaria, en el comercio, en las ciencias naturales, etc., y su smbolo es %.
Ejemplo 4.52 Determine el 65 % de 32000.
Solucin
Procedemos de la siguiente manera:
_
100 32000
65 x
x =
65 32000
100
= 20800.
Es decir, el 65 % de 32000 es 20800.
Ejemplo 4.53 Tenemos una receta para hacer pastel de 1 kg. pero queremos hacer uno de 1,5
kg. Si la receta original dice que debemos usar
2
3
de tazas de azcar. Cul ser la cantidad de
azcar que debemos usar ahora?
Solucin
Si a un kilogramo de pastel le asociamos el 100 %, entonces medio kilogramo corresponde al 50 %,
lo que indica que la cantidad de azcar usada sera
2
3
100 % +
2
3
50 % =
2
3

50
100
+
2
3

50
100
=
2
3
+
2
3

1
2
=
2
3
_
1 +
1
2
_
=
2
3

3
2
= 1.
Es decir, debemos usar 1 taza de azcar.
Ejemplo 4.54 Una barra de metal de 5 kg. tiene 2 kg. de bronce y 3 kg. de aluminio:
a) Determine la cantidad de cobre y estao en la barra si se sabe que el bronce es una aleacin
con 70 % de cobre y 30 % de estao.
b) Qu porcentaje de cobre tiene la barra de metal?
Solucin
a) En virtud de que la barra tiene 2 kg. de bronce, entonces, en la barra hay, (2 kg.)(0,7) = 1,4
kg. de cobre y (2 kg.)(0,3) = 0,6 kg. de estao.
b) En la barra de 5 kg. hay 1,4 kg. de cobre. Por tanto, en la barra hay
1,4
5
100 = 0, 28100 = 28 %
de cobre.
Ejemplo 4.55 La supercie de nuestro planeta consta de 70 % de agua y 30 % de tierra. De
este ltimo 30 %,
2
5
partes es cultivable. Qu porcentaje de la supercie total del planeta es cul-
tivable?
CAPTULO 4. EXPRESIONES ALGEBRAICAS 210
Solucin
Sea T la supercie total del planeta. Entonces, 0, 3T es tierra, de la cual
2
5
0, 3T = 0, 12T es
cultivable. Por lo tanto, el porcentaje del planeta cultivable es
0, 12T
T
100 = 0, 12 100 = 12 %.
Ejemplo 4.56 Cuando una persona pide dinero prestado debe pagar un inters durante el
tiempo que dura el prstamo, denotmoslo por i. El capital es la cantidad que se presta denotado
por c. La tasa o rdito, es el tanto por ciento que se paga en un tiempo determinado, r. El tiempo
que dura el prstamo lo denotaremos por t. Se tiene la relacin i = crt:
a) Cul es el inters que se debe pagar por un prstamo de $400 durante 5 meses si el rdito
es 2 % mensual?
b) Cul es el inters que se debe pagar por un prstamo de $400 durante 3 meses, si la tasa es
de 24 % anual?
c) Nos prestan $500 con inters mensual del 2 %. Cunto pagaremos a n de mes para liquidar
completamente la deuda?
Solucin
a) i = crt = 400 0, 02 5 = 40 dlares.
b) i = crt = 400 0, 24
3
12
= 24 dlares.
c) El inters a pagar es i = crt = 500 0, 02 1 = 10 dlares. Por lo tanto, para liquidar la deuda
debemos pagar 500 + 10 = 510 dlares.
Ejemplo 4.57 Si el radio del cilindro disminuye en un 10 % mientras que su altura aumenta
en un 12 % en qu tanto porciento varan:
a) El volumen del cilindro.
b) El rea lateral del cilindro.
Solucin
Sea r: radio inicial del cilindro, h: altura inicial del cilindro
a) Si el radio disminuye en un 10 % entonces el nuevo radio R ser
R = r 0, 1r = 0, 9r
Si la altura aumenta en un 12 % entonces la nueva altura H ser
H = h + 0, 12h = 1, 12h
luego el nuevo volumen ser
V = R
2
H
= (0, 9r)
2
(1, 12h)
2
= 0, 9072r
2
h
= r
2
h 0, 0928r
2
h
esto es, el volumen disminuye en 0, 0928r
2
h unidades; es decir, disminuye en 9, 28 %.
b) El rea del cilindro ser
A = 2RH
= 2(0, 9r)(1, 12h)
= 1, 008(2rh)
= 2rh + 0, 008(2rh)
luego el rea lateral aumenta en 0, 8 %.
CAPTULO 4. EXPRESIONES ALGEBRAICAS 211
4.13. Tarea
Regla de tres simple, directa e inversa
1. En 50 litros de agua de mar hay 1300 gramos de sal. Cuntos litros de agua de mar
contendrn 1000 gramos de sal?
2. Un auto consume 2 galones de gasolina cada 100 kilmetros. Si quedan en el depsito 2
litros de gasolina, cuntos kilmetros recorrer el auto?
3. Un ganadero tiene comida suciente para alimentar 150 vacas durante 15 das. Cuntos
das podr alimentar con la misma cantidad de comida a 200 vacas?
4. Para envasar cierta cantidad de ron se necesitan 10 toneles de 150 litros de capacidad cada
uno. Se desea envasar la misma cantidad de ron empleando 12 toneles. Cul deber ser la
capacidad de esos toneles?
5. Un avin tarda 2 minutos para recorrer 4,5 kilmetros. Cunto tarda en recorrer con la
misma velocidad 150 kilmetros?
6. Un obrero gana 25 dlares por 8 horas de trabajo. Cunto tiempo ha trabajado para
ganar 110 dlares?
7. Se compra 15 metros de cinta a 0,18 dlares el metro. Cuntos metros de otra cinta de
0,12 dlares el metro se puede comprar con el mismo dinero?
8. En un da de trabajo de 8 horas, un obrero ha hecho 10 cajas. Cuntas horas tardar en
hacer 25 de esas mismas cajas?
9. El jugo de naranja de una cierta marca viene en latas de 220 cm
3
y cuesta 0,33 dlares, el
de otra marca viene en latas de 250 cm
3
y cuesta 0,40 dlares. Cul resulta ms barato?
10. Ocho obreros han tardado 24 horas para realizar cierto trabajo. Cunto tiempo hubiesen
empleado para hacer el mismo trabajo 4 obreros?
11. Cul ser la altura de una columna que produce una sombra de 4,5 metros, sabiendo que
a la misma hora una varilla vertical de 0,49 metros arroja una sombra de 0,63 metros?
12. Un comerciante compr 33 kg de arroz a razn de 0,90 dlares el kg. Cuntos kg de arroz
de 1,10 dlares podra haber comprado con esa misma suma de dinero?
13. Un alimento para perros se venda en paquetes de 800 gramos a 48 dlares, y ahora se
vende en paquetes de 2 kilogramos a 1,12 dlares. Aument o rebajo el precio del kilogramo?
Cunto fue el aumento o la disminucin?
14. Se lma un partido de ftbol de modo tal que la cmara capte 48 imgenes en 3 segundos; la
otra cmara capta 450 imgenes en 0.5 minuto. Cul lmacin resulta ms lenta? Cuntas
imgenes por segundo lma la segunda cmara?
15. Si para pintar 180 m
2
se necesitan 24 kg de pintura, cuntos kg se necesitarn para pintar
una supercie rectangular de 12 m de largo por 10 m de ancho?
16. Para hacer 96 m
2
de un cierto gnero se necesitan 30 kg de lana; cuntos kg se necesitarn
para tejer una pieza de 0,90 m de ancho por 45 m de largo?
17. La longitud de los
4
5
de un camino es de 550,20 m. Cul es la longitud del camino?
CAPTULO 4. EXPRESIONES ALGEBRAICAS 212
18. Un trabajo puede ser realizado por 80 obreros en 42 das. Si el plazo para terminarlo es de
30 das, cuntos obreros debern aumentarse?
19. A razn de 70 km por hora un automovilista emplea 2 horas 30 minutos para recorrer
cierta distancia. Qu tiempo emplear para recorrer la misma distancia a razn de 45 km
por hora?
20. Cinco motores consumen 7200 kg de combustible en 42 horas de funcionamiento; para
cuntas horas alcanzar esa misma cantidad de combustible, si funcionan slo 3 de esos
motores?
21. Con 15 kg de algodn se teje una tela de 120 m de largo y 95 cm de ancho; qu largo
tendr una tela de igual calidad que la anterior de 90 cm de ancho tejida con la misma
cantidad de algodn?
22. Un automvil recorre 100 km en 1 h 32 m. En qu tiempo recorrer 60 km?
23. Doce obreros han hecho la mitad de un trabajo en 18 horas. A esa altura de la obra 4
obreros abandonan el trabajo. Cuntas horas tardan en terminarlo los obreros que quedan?
24. Para empapelar una habitacin se necesitan 15 rollos de papel de 0,45 m de ancho. Cuntos
rollos se necesitarn, si el ancho fuera de 0,75 m?
25. Si 65 hectreas producen 2920 kg de trigo. Cuntos kg producirn 2340 hectreas de la
misma calidad de tierra?
26. Con 15 kg de hierro se han hecho 420 tuercas de 4 pulgadas. Cuntas tuercas semejantes
a las anteriores, pero de 3 pulgadas, se pueden hacer con la misma cantidad de hierro?
27. Un ganadero tiene 36 ovejas y alimento para ellas por el trmino de 28 das. Con 20
ovejas ms, sin disminuir la racin diaria y sin agregar forraje, durante cuntos das podr
alimentarlas?
28. Si los
3
5
de un campo tienen una supercie de 25,20 hectreas. Cul es la supercie del
campo expresada en m
2
.
Regla de tres compuesta, directa, inversa y mixta
1. Para construir una autista de 8 kilmetros, se emplearon 40 operarios por 60 das traba-
jando 8 horas diarias. Cuntos das tardarn 35 operarios trabajando 8 horas diarias en
construir 5 kilmetros?
2. Una familia compuesta de 6 personas consume en 2 das 3 kg de pan, cuntos kg de pan
sern consumidos en 5 das, estando dos personas ausentes?
3. Con 9 arados de disco se rotulan 36.9 hectreas en 48 horas, cuntas hectreas se rotularn
con 15 arados en 120 horas?
4. Para cavar una zanja de 78 m de largo, 9
o
cm de ancho y 75 cm de profundidad, se necesitan
39 obreros, cuntos obreros habr que disminuir para hacer en el mismo tiempo una zanja
de 60 m de largo, 0,50 m de ancho y 45 cm de profundidad?
5. En un colegio con 120 alumnos pupilos se han gastado en manutencin 120 dlares durante
6 das. Habiendo disminuido el nmero de alumnos en
1
3
, cunto se gastar durante un mes
de 30 das?
CAPTULO 4. EXPRESIONES ALGEBRAICAS 213
Porcentajes
1. En cierto pas, la poblacin masculina representa el 48 % y una de cada 7 mujeres tiene el
hbito de fumar. Supongamos que la poblacin es de 100.000.000 de habitantes:
a) Cuntas mujeres fumadoras hay?
b) Qu porcentaje de la poblacin representan las mujeres fumadoras?
Resp: a) 7600.000; b) 7,6 %.
2. La poblacin de un cultivo de bacterias aumenta 10 % en la primera hora y disminuye el
mismo porcentaje en la segunda hora. Si la poblacin original era de 5500:
a) Calcule el nmero de bacterias despus de dos horas.
b) Qu porcentaje representa de la poblacin original?
Resp: a) 5445; b) 99 %.
3. Al analizar una pintura se encontr con un 55 % de colorante y con un 45 % de aglomerante.
El colorante est compuesto de 20 % del material A, 55 % del material B y 25 % del material
C. Cul es el porcentaje de cada material en la pintura analizada?
Resp: Hay 11 % de material A, 30 % del material B y 13 % del material C.
4. Un material se desintegra de tal forma que cada 100 aos se consume el 0, 8 % de la cantidad
que queda por desintegrarse. Si en el ao 2000 se tienen 600 kg. de tal material:
a) Qu cantidad se tendra en 2101?
b) Qu cantidad se tendra en 2201?
c) Qu cantidad se tendra en 2501?
d) Qu cantidad se tendra en 2901?
Resp: a) 595,2 kg; b) 590,4 kg; c) 576,3 kg; d) 558,1 kg.
Captulo 5
Polinomios
5.1. Deniciones generales
Denicin 5.1 Polinomio
Una expresin racional en la que se prevn solamente dos operaciones respecto de las letras que
lo integran, a saber, multiplicacin y elevacin a potencia natural, se denomina monomio. Una
expresin racional se denominar polinomio, si es entera respecto de toda letra que gura en dicha
expresin. En particular, una expresin racional que contiene una sola letra y que es entera respecto
de esta letra, se denomina polinomio entero respecto de una letra. El grado, es la mnima expresin
algebraica formada por un solo trmino algebraico.
De la denicin de polinomio y de las reglas que rigen las operaciones sobre expresiones alge-
braicas se desprende que la suma, la diferencia y el producto de dos polinomios sern polinomios.
Por regla general, los monomios se transforman idnticamente conforme a determinadas leyes de
operaciones, reuniendo juntos todos los nmeros que integran el monomio y escribindolos ante
las letras del monomio, y tambin, reuniendo juntas las letras iguales que integran el monomio y
escribindolas en forma de una potencia natural de dicha letra. Realizada tal transformacin, un
monomio se considera escrito en la forma estndar, mientras que el factor numrico que precede a
las letras del monomio se denomina coeciente del monomio dado.
Cuando varios nmeros estn sometidos a operaciones enteras: suma, resta y multiplicacin, la
expresin se llama entera. Toda expresin entera puede reducirse a un monomio o a una suma de
monomios, que llamaremos polinomio.
Segn las reglas de operaciones sobre las expresiones algebraicas, todo polinomio siempre puede
transformarse en una forma en la que el polinomio se componga de varios monomios escritos en la
forma estndar y unidos entre s mediante los signos de adicin y de sustraccin; por esta razn se
dice que un polinomio es la suma algebraica de monomios.
Los trminos semejantes de un polinomio son sus monomios escritos en forma estndar y que
se diferencian en nada ms que los coecientes. Reducir los trminos semejantes de un polinomio
signica sustituir la suma algebraica de los trminos semejantes por un solo trmino idnticamente
igual a dicha suma.
Un polinomio en x es una expresin algebraica de la forma
p(x) = a
0
+a
1
x +a
2
x
2
+... +a
n1
x
n1
+a
n
x
n
214
CAPTULO 5. POLINOMIOS 215
Es una suma formal de n+1 trminos, siendo el primero a
0
. El smbolo n representa un entero que
es positivo o cero. El trmino a
i
x
i
se denomina trmino general del polinomio y es el trmino i. El
primer trmino a
0
se llama trmino constante de p(x) y ajusta la frmula para el trmino general
si se acuerda identicar x
0
con 1, a
n
con a
n
x
n
. Los smbolos a
0
, a
1
, ..., a
n
se llaman coecientes
de p(x).
Los polinomios se clasican en ordenados y desordenados, completos e incompletos, homogneos
y heterogneos. Es decir:
1. Polinomio ordenado: Un polinomio ordenado respecto a una de sus letras, es aquel en
que el exponente de la letra llamada ordenatriz es constantemente mayor o menor en cada
termino que en el que le precede, si no lo tiene igual.
2. Polinomio desordenado: Un polinomio desordenado es aquel que no presenta este req-
uisito. La ordenacin de un polinomio puede ser ascendente o descendente, segn que los
exponentes de la letra ordenatriz vayan aumentando o disminuyendo, desde el primero al
ltimo.
3. Polinomio completo: Un polinomio completo respecto a una de sus letras, es aquel
en el que existen todos los sucesivos exponentes de ella, desde el mayor hasta el cero, que
corresponde al trmino independiente de la letra. Polinomio incompleto es aquel en el que
falta alguno o algunos de los trminos. Las propiedades de un polinomio completo son las
siguientes:
a) Si el polinomio es de grado n, entonces el nmero de trminos es igual a n + 1.
b) El grado del polinomio completo es igual al nmero de trminos menos 1.
c) La diferencia de grados relativos de dos trminos consecutivos es igual a la unidad.
d) El trmino independiente contiene a la variable con exponente cero.
4. Polinomio homogneo: Un polinomio homogneo es el que tiene todos sus trminos del
mismo grado, llamado grado de homogeneidad.
Denicin 5.2 Polinomio nulo
Cuando los coecientes de un polinomio son todos cero, se le llama polinomio nulo. Se conviene
en que todos estos polinomios son iguales y se identican con el nmero cero.
Denicin 5.3 Polinomios iguales
Si p(x) y q(x) son dos polinomios arbitrarios, se pueden agregar, de ser necesario, trminos con
coecientes cero a uno de ellos y escribir entonces ambos polinomios como en la frmula general
y con la misma n. Entonces se conviene en que, por denicin, dos polinomios p(x) y q(x) son
iguales si los coecientes correspondientes son nmeros iguales.
Si p(x) no es el polinomio cero, tiene por lo menos un coeciente distinto de cero. Se pueden
entonces borrar sucesivamente aquellos trminos primeros que tengan coeciente cero hasta llegar
a un nuevo trmino primero con coecientes distintos de cero. Esto permite escribir cualquier poli-
nomio como en la frmula general con a
n
,= 0. Cuando es as, se dice que a
n
es el coeciente nal
de p(x) y n es el grado de p(x).
Un polinomio como el de la frmula general con n = 0 se reduce a su primero (y ltimo) trmino
a
0
x
0
= a
0
= a
n
. Se le llama un polinomio constante. Se le da el grado cero aun en el caso que
sea el polinomio cero. Se har referencia algunas veces a los polinomios de grado n > 0 como a
CAPTULO 5. POLINOMIOS 216
polinomios no constantes.
Un polinomio de grado n = 1, 2, 3, 4, se puede tambin llamar polinomio lineal, cuadrtico,
cbico y curtico respectivamente.
5.2. Suma, resta y producto de polinomios
En virtud de las reglas de operaciones con las expresiones algebraicas podemos concretar las
leyes que rigen las operaciones sobre los polinomios de la siguiente manera:
Denicin 5.4 Suma de polinomios
Para adicionar dos polinomios, se deben escribir todos los trminos seguidos del primer polinomio
y, luego, todos los trminos del segundo polinomio, conservando para cada monomio el signo que
est delante de su coeciente despus de lo cual es necesario reducir los trminos semejantes.
Denicin 5.5 Resta de polinomios
Para sustraer de un polinomio otro polinomio, se deben escribir todos los trminos seguidos del
primer polinomio, conservando inalterable el signo de cada monomio que est delante de su coe-
ciente, a continuacin se escriben todos los trminos del segundo polinomio, cambiando por op-
uestos todos los signos que estn delante de los coecientes de los monomios del segundo polinomio,
despus de lo cual es necesario reducir los trminos semejantes.
Los polinomios se suman sumando los coecientes correspondientes y se restan restando los
coecientes correspondientes.
Si n es el mayor de los grados de dos polinomios p(x) y q(x), se puede escribir
p(x) = a
0
+a
1
x +a
2
x
2
+... +a
n1
x
n1
+a
n
x
n
y
q(x) = b
0
+b
1
x +b
2
x
2
+... +b
n1
x
n1
+b
n
x
n
Si tienen grado distinto, uno de los coecientes a
n
y b
n
es cero. Cuando se han escrito as, se tiene
p(x) +q(x) = (a
0
+b
0
) + (a
1
+b
1
)x + (a
2
+b
2
)x
2
+... + (a
n1
+b
n1
)x
n1
+ (a
n
+b
n
)x
n
Estos son polinomios cuyo grado no es mayor que n.
El resultado obtenido puede extenderse a sumas de varios polinomios y enunciarse de la sigu-
iente manera: El grado de una suma de polinomios no es mayor que el mayor de los grados de
todos los polinomios que se suman.
Esto no solamente es vlido para sumas sino tambin para diferencias. En realidad vale para
sumas de mltiples constantes de polinomios.
p(x) q(x) = (a
0
b
0
) + (a
1
b
1
)x + (a
2
b
2
)x
2
+... + (a
n1
b
n1
)x
n1
+ (a
n
b
n
)x
n
Supresin de signos:
1.- Cuando el signo est precedido del signo +, se elimina este signo sin producir ningn cambio.
2.- Cuando est precedido del signo -, se elimina el signo cambiando todos los signos de suma o
resta que se encuentra dentro de l.
3.- Cuando tiene que ir precedido del signo +, se escribe el signo sin realizar ningn cambio.
CAPTULO 5. POLINOMIOS 217
4.- Cuando tiene que ir precedido del signo -, se escribe el signo, cambiando los signos de suma
y de resta de todos los trminos que se introduce.
Ejemplo 5.1 Smense y rstense los polinomios
p(x) = 3x
5
4x
2
+ 2x + 1 y q(x) = 2x
4
+ 5x
3
+ 3x 2
Solucin
Para sumar los polinomios, hacemos
3 +0 +0 4 +2 +1
0 2 +5 +0 +3 2
3 2 +5 4 +5 1
El polinomio resultante es p(x) +q(x) = 3x
5
2x
4
+ 5x
3
4x
2
+ 5x 1.
Para restar los polinomios, hacemos
3 +0 +0 4 +2 +1
0 +2 5 +0 3 +2
3 +2 5 4 5 +3
El polinomio resultante es p(x) q(x) = 3x
5
+ 2x
4
5x
3
4x
2
x + 3.
El producto de
p(x) = a
0
+a
1
x +a
2
x
2
+... +a
n1
x
n1
+a
n
x
n
de grado n y
q(x) = b
0
+b
1
x +b
2
x
2
+... +b
m1
x
m1
+b
m
x
m
de grado m es la suma de todos los productos (a
i
x
i
)(b
j
x
j
) = a
i
b
j
x
i+j
. Todos los trminos con el
mismo exponente para x se combinan sumando sus coecientes. Si p(x) o q(x) es el polinomio cero,
su producto es cero. En caso contrario a
n
,= 0, b
m
,= 0 y p(x)q(x) tiene precisamente un trmino
de grado n + m, a saber, a
n
b
m
x
n+m
. Este es el trmino de mximo grado y resulta que el grado
de p(x)q(x) es n +m.
Denicin 5.6 Producto de polinomios
Para multiplicar un monomio por un polinomio, se debe multiplicar dicho monomio por cada ter-
mino del polinomio, escribir los trminos segundos del producto con aquellos signos que tenan los
trminos del polinomio, si delante del coeciente del monomio esta el signo mas, y con los signos
opuestos, si el coeciente del monomio tiene el signo menos, a continuacin se debe escribir en la
forma estndar cada monomio del producto y reducir los trminos semejantes. Si el polinomio no
admite reduccin, tampoco habr trminos semejantes en el producto, que ser otro polinomio de
igual nmero de trminos.
Este resultado es vlido para los productos de varios polinomios. Los polinomios que se multi-
plican factores y se enuncia el resultado de la siguiente manera.
Teorema 5.1 El grado de un producto de polinomios distintos de cero es igual a la suma de
los grados de sus factores. El coeciente inicial de cualquier producto es igual al producto de los
coecientes iniciales de los productos, y el trmino constante de un producto es igual al producto
de los trminos constantes de sus factores. El producto de polinomios distintos de cero es distinto
de cero y es una constante si y slo si todos sus factores son constantes.
CAPTULO 5. POLINOMIOS 218
El grado de un producto es la suma de los grados. Estos grados son todos nmeros naturales y
su suma puede ser cero nicamente cuando los grados que se sumen sean cero todos, es decir, los
factores son constantes.
El producto de un polinomio p(x) = a
0
+a
1
x+a
2
x
2
+... +a
n1
x
n1
+a
n
x
n
por un polinomio
constante k es
kp(x) = ka
0
+ka
1
x +ka
2
x
2
+... +ka
n1
x
n1
+ka
n
x
n
En particular
(1)p(x) = (a
0
) + (a
1
x) + (a
2
x
2
) +... + (a
n1
x
n1
) + (a
n
x
n
)
y cualquier diferencia
p(x) q(x) = p(x) + (1)q(x)
Se puede ahora establecer que cualquier expresin algebraica obtenida mediante la aplicacin
a x y a las constantes, de un nmero nito de operaciones enteras de adicin, sustraccin y mul-
tiplicacin es un polinomio en x. Tambin se dan las deniciones de suma y multiplicacin de
polinomios de manera que se satisfacen las leyes para las operaciones enteras y para los exponentes
enteros no negativos. Entonces resultar cierto que si se sustituye x por cualquier nmero en dos
expresiones formalmente distintas del mismo polinomio, los nmeros resultantes sern un mismo
nmero.
Ejemplo 5.2 Multiplicar los polinomios
p(x) = 2x
3
3x
2
+ 2x 1 y q(x) = x
4
+ 3x
3
2x
2
+ 4
solucin
Ubicamos los coecientes de los polinomios de la siguiente manera, y luego pro-cedemos a multi-
plicar ordenadamente:
0 +2 3 +2 1
1 +3 2 +0 +4
0 +2 3 +2 1
0 +6 9 +6 3
0 4 +6 4 +2
0 0 0 0 0
0 +8 12 +8 4
0 +2 +3 11 +11 +1 10 +8 4
Por tanto, el polinomio resultante tiene la forma
p(x)q(x) = 2x
7
+ 3x
6
11x
5
+ 11x
4
+x
3
10x
2
+ 8x 4.
5.3. Produtos notables
Haciendo uso de las reglas de adicin y multiplicacin de polinomios y de las propiedades que
poseen las igualdades de expresiones algebraicas, obtendremos igualdades idnticas, las cuales se
denominan formulas de multiplicacin reducida.
En muchos problemas aparecen una y otra vez para ser multiplicados, algunos factores que
son expresiones algebraicas de un cierto tipo. En consecuencia, vale la pena aprender a escribir
rpidamente los productos. Cuando es necesario calcular varios polinomios, resulta que ciertos
tipos de expre-siones guran con tanta frecuencia que se justica el desarrollo de frmulas para el
clculo.
CAPTULO 5. POLINOMIOS 219
Denicin 5.7 Cuadrado de la suma de dos nmeros
El cuadrado de la suma de dos nmeros es igual al cuadrado del primer nmero, ms el producto
duplicado del primer nmero por el segundo, mas el cuadrado del segundo numero:
(x +y)
2
= x
2
+ 2xy +y
2
Denicin 5.8 Cuadrado de la diferencia de dos nmeros
El cuadrado de la diferencia de dos nmeros es igual al cuadrado del primer nmero, menos el
producto duplicado del primer nmero por el segundo, mas el cuadrado del segundo numero:
(x y)
2
= x
2
2xy +y
2
Denicin 5.9 Diferencia entre los cuadrados de dos nmeros
La diferencia entre los cuadrados de dos nmeros es igual al producto de la diferencia de estos
nmeros por la suma de los mismos:
x
2
y
2
= (x y)(x +y)
Denicin 5.10 Cubo de la suma de dos nmeros
El cubo de la suma de dos nmeros es igual al cubo del primer nmero, mas el producto triplicado
del cuadrado del primer numero por el segundo, mas el pro-ducto triplicado del primer numero por
el cuadrado del segundo y mas el cubo del segundo numero:
(x +y)
3
= x
3
+ 3x
2
y + 3xy
2
+y
3
De la misma forma, podemos denir las restantes:
(x y)
3
= x
3
3x
2
y + 3xy
2
y
3
x
3
+y
3
= (x +y)(x
2
xy +y
2
)
x
3
y
3
= (x y)(y
2
+xy +y
2
)
...
Las frmulas aducidas permiten observar cierta regularidad con ayuda de la cual podemos es-
cribir la frmula para (a b)
n
, donde n es un nmero natural cualquiera. Es fcil ver que habr
en total n + 1 trminos: El primer trmino es el primer nmero a la potencia n; en cada trmi-
no subsiguiente la potencia del primer nmero ser en una unidad menor que su potencia en el
trmino antecedente, y en el ltimo trmino la potencia del primer nmero es nula; el segundo
nmero tiene en el primer trmino la potencia nula, en el segundo trmino, la primera potencia,
en cada trmino subsiguiente la potencia del segundo nmero ser en una unidad mayor que su
potencia en el trmino antecedente, y en el ltimo trmino el segundo nmero gura a la potencia n.
El coeciente de cada termino puede hallarse con ayuda del triangulo de Pascal:
1
1 1
1 2 1
1 3 3 1
1 4 6 4 1
1 5 10 10 5 1
1 6 15 20 15 6 1
1 7 21 35 35 21 7 1
. . .
CAPTULO 5. POLINOMIOS 220
Es simple la regla conforme a la cual se forman las lneas del triangulo de Pascal. Cada lnea
puede obtenerse de la lnea superior anterior del modo siguiente: En el intervalo entre cualesquiera
nmeros vecinos de la lnea superior, pero ms abajo, se escribe la suma de estos, y en los extremos
se ponen las unidades. El numero de la lnea ensea a que potencia se eleva el binomio a+b, mien-
tras que los nmeros de dicha lnea son los coecientes de los trminos correspondientes escritos
en el orden estudiado mas arriba.
Si se necesita escribir la formula para (a +b)
n
, donde n es un numero grande, esta claro que el
calculo de los coecientes del segundo miembro con ayuda del triangulo de Pascal ser engorroso,
razn por la cual resulta deseable conocer otra formula para calcular (a + b)
n
. Tal formula existe
y lleva el nombre de binomio de Newton, teniendo por expresin
(a +b)
n
=
_
n
0
_
a
n
+
_
n
1
_
a
n1
b +... +
_
n
k
_
a
nk
b
k
+... +
_
n
n
_
b
n
donde
_
n
k
_
=
n!
k!(n k)!
, 0! = 1, k! = 1 2 3 ... k para cualquier k natural. De la formula para
el binomio de Newton se deduce fcilmente la formula (a b)
n
. Denotemos d = b y apliquemos
la formula del binomio de Newton:
(a b)
n
= (a +d)
n
=
_
n
0
_
a
n
+
_
n
1
_
a
n1
d +... +
_
n
k
_
a
nk
d
k
+... +
_
n
n
_
d
n
Sustituyendo b por d, obtenemos
(a b)
n
=
_
n
0
_
a
n

_
n
1
_
a
n1
b +... + (1)
k
_
n
k
_
a
nk
b
k
+... + (1)
n
_
n
n
_
b
n
En una serie de problemas, al operar con polinomios, resulta ms fcil examinarlo no en la forma
estndar, sino en forma de un producto. La transformacin idntica de un polinomio en la forma
de un producto de polinomios se llama descomposicin del polinomio en factores. En forma general
todas las formulas de multiplicacin reducida son precisamente formulas que rigen la descomposi-
cin del polinomio en factores. Adems de la aplicacin de las formulas de multiplicacin reducida
existen tambin otros procedimientos para descomponer los polinomios en factores, por ejemplo, la
agrupacin o procedimiento consistente en sacar el factor comn del parntesis. Para descomponer
un polinomio en factores son tiles todos los procedimientos.
Hay un procedimiento ligeramente distinto del que se acaba de usar para calcular polinomios.
Diere principalmente en la forma de seleccionar y ordenar los trminos. Es un mtodo valioso
para ahorrar lugar y tiempo. A este procedimiento se le puede llamar procedimiento del exponente
jo. Se selecciona un exponente jo i y se calculan todos los coecientes de la potencia x
i
que hay
en los trminos cuya suma es p(x). Entonces la suma de estos coecientes es el coeciente de x
i
en p(x). El procedimiento empieza con la determinacin del coeciente de la potencia mxima de
x que aparece en todos los trminos de p(x). Este coeciente ser el coeciente inicial de p(x) a
menos que sea cero. El procesamiento termina con el clculo de a
0
= p(0).
Ejemplo 5.3 Calcular
p(x) = (3x 2)
2
(x + 1) x(2x + 1)(2x 1) 5(x + 1)x
2
.
Solucin
Se escribe
p(x) = (9x
2
12x + 4)(x + 1) x(4x
2
1) 5x
3
5x
2
CAPTULO 5. POLINOMIOS 221
El coeciente de x
3
es 9 - 4 - 5 = 0, el de x
2
es 9 - 12 - 5 = -8 y el de x es -12 + 4 + 1 = -7.
Tambin p(0) = 4. Por consiguiente
p(x) = 8x
2
7x + 4.
Encontremos en seguida una regla para escribir con facilidad el cuadrado de un polinomio de
tres o ms trminos.
Denicin 5.11 Cuadrado de un polinomio
El cuadrado de un polinomio es igual a la suma de los cuadrados de los trminos por separado ms
el doble de la suma algebraica de los productos obtenidos al multiplicar a cada trmino por la suma
de los trminos que lo siguen.
Ejemplo 5.4 Desarrolle el cuadrado del polinomio
(2x 3y +z 2u).
Solucin
Desarrollamos directamente, aplicando la regla
(2x 3y +z 2u)
2
= 4x
2
+ 9y
2
+z
2
+ 4u
2
12xy + 4xz 8xu 6yz + 12yu 4zu.
5.4. Tarea
1. Calcule a, b y c para que se verique la igualdad:
(x
3
2x +a)(bx +c) = 3x
4
+ 2x
3
6x
2
x + 2
2. Determine la suma y diferencia:
a)
_
p(x) = 3x
4
2x
3
+ 4x
2
1
q(x) = 5x
3
+ 3x
2
4x + 3
b)
_
p(x) = 2x
4
+ 4x
3
4x
2
+ 2
q(x) = 3x
4
2x
2
+ 3x 1
c)
_
p(x) = x
4
5x
3
+ 2x
2
+ 3
q(x) = 2x
4
+ 5x
2
2x + 3
d)
_
p(x) = x
4
3x
2
+ 5x 3
q(x) = 4x
3
+ 3x
2
5x + 3
e)
_
p(x) = x
3
5x
2
+ 8x 3
q(x) = 2x
4
+ 3x
2
5x + 2
f )
_
p(x) = 6x
4
2x
3
+x
2
3
q(x) = x
4
12x
2
5x + 4
Resp: a) 3x
4
+ 3x
3
+ 7x
2
, 3x
4
7x
3
+ x
2
+ 4x 4; b) 5x
4
+ 4x
3
6x
2
+ 3x + 1,
x
4
+4x
3
2x
2
3x+3; c) 3x
4
5x
3
+7x
2
2x+6, x
4
5x
3
3x
2
+2x; d) 4x
3
x
4
,
x
4
4x
3
6x
2
+ 100x 6; e) 2x
4
+x
3
2x
2
+ 3x 1, 2x
4
+x
3
8x
2
+ 13x 5;
f ) 7x
4
2x
3
11x
2
5x + 1, 5x
4
2x
3
+ 13x
2
+ 5x 7.
3. Dados los polinomios
_

_
p(a, b) = 3a
2
b 5ab
2
2a
2
q(a, b) = 4a2b ab
2
b
2
r(a, b) = 8a
2
b + 5a
2
7b
2
h(a, b) = 5ab
2
10a
2
b + 4a
2
CAPTULO 5. POLINOMIOS 222
Calcular las siguientes operaciones: a) p +q r h; b) p q [p + (q +h) r].
Resp: a) 9a
2
b 11a
2
11ab
2
+ 6b
2
; b) 12a
2
b 5a
2
5ab
2
+ 7b
2
.
4. Determine los siguientes polinomios:
a) x(x
2
+ 1) 3x(x + 3) + 2(x
2
x)
2
;
b) 2(x
2
+ 3) 2x(x 3) + 6(x
2
x 1);
c) (2x
2
+x 1)(x 3) (2x 1)(2x + 1);
d) (3x 1)(3x + 1) (4x 3)
2
2(2x
2
+ 16x 16).
5. Determine los siguientes polinomios:
a) p(x) = (3x
2
4x)(x + 1) (2x + 1)(x 2);
b) p(x) = (2x + 3)(2x 3)x (2x
2
+ 2x + 1)(x 1);
c) p(x) = 2x(x
2
+ 2x 1) (x
2
+ 3x + 2)(x 1);
d) p(x) = (2x
2
+ 3x 1)(x
2
2x 2) x
3
(2x 1);
e) p(x) = (x
2
x 1)(x
3
+ 2x + 1) + (x + 1)(x
3
+ 2);
f ) p(x) = (x + 1)(x
2
+ 1) 2(2x
2
1)(x 1) + 3x
2
(x 2);
g) p(x) = 3(x ky)
2
+ 2(x ky)(kx +y) 4(kx +y)
2
;
h) p(x) = (x ky)
2
+ 6(x ky)(kx +y) 9(kx +y)
2
;
i) p(x) = 2(x ky)
2
+ (x ky)(kx +y) 3(kx +y)
2
.
Resp: a) p(x) = 3x
3
+x
2
x2; b) p(x) = 2x
3
8x+1; c) p(x) = x
3
+2x
2
x+2;
d) p(x) = 11x
2
4x+2; e) p(x) = x
5
+2x
3
x
2
x+1; f ) p(x) = x
2
+3x1.
6. Determine el producto de los polinomios:
a)
_
p(x) = 3x
4
2x
3
+ 4x
2
1
q(x) = 5x
3
+ 3x
2
4x + 3
b)
_
p(x) = 2x
4
+ 4x
3
4x
2
+ 2
q(x) = 3x
4
2x
2
+ 3x 1
c)
_
p(x) = x
4
5x
3
+ 2x
2
+ 3
q(x) = 2x
4
+ 5x
2
2x + 3
d)
_
p(x) = x
4
3x
2
+ 5x 3
q(x) = 4x
3
+ 3x
2
5x + 3
e)
_
p(x) = x
3
5x
2
+ 8x 3
q(x) = 2x
4
+ 3x
2
5x + 2
f )
_
p(x) = 6x
4
2x
3
+x
2
3
q(x) = x
4
12x
2
5x + 4
Resp: a) 15x
7
x
6
+ 2x
5
+ 29x
4
27x
3
+ 9x
2
+ 4x 3; b) 8x
8
+ 12x
7
16x
6

2x
4
+ 24x
4
16x
3
+ 6x 2; c) 2x
8
10x
7
+ 9x6 27x
5
+ 29x
4
19x
3
+ 21x
2
6x + 9;
d) 4x
7
3x
6
7x
5
+ 8x
4
+ 18x
3
43x
2
+ 30x 9; e) 2x
7
10x
6
+ 19x
5
26x
4
+
51x
3
59x
2
+ 31x 6; f ) 6x
8
2x
7
71x
6
6x
5
+ 19x
4
13x
3
+ 40x
2
+ 15x 12.
7. Efectuar los siguientes productos notables:
a) (2x
2
+ 3y)(2x
2
3y);
b) (2x + 3y)
2
;
c) (2x + 1)(3x + 4)(2x + 1)(3x + 4);
d) (x +y)(x
2
xy +y
2
);
e) (x + 1)(x
5
x
4
+x
3
x
2
+x 1);
f ) (x
2
1)(x
4
+x
2
+ 1);
g) (x + 2)(x
2
2x + 4)(x 2)(x
2
+ 2x + 4);
h) (x a)
2
+ ((x b)
2
+ (x c)
2
+x
2
;
CAPTULO 5. POLINOMIOS 223
i) 3(x 2y)
2
+ 2(x 2y)(x + 2y) + (3y x)(3y +x) (2x 3y)
2
;
j) (x y)(x +y)(x
2
+y
2
)(x
4
+y
4
);
k) (2x +y)(4x
2
2xy +y
2
)(8x
3
y
3
).
Resp: a) 4x
4
9y
2
; b) 4x
2
+12xy+9y
2
; c) 36x
4
+132x
3
+169x
2
+88x+16; d)
x
3
+y
3
; e) x
6
1; f ) x
6
1; g) x
6
64; h) 4x
2
2(a +b +c)x+a
2
+b
2
+c
2
;
i) 4y
2
; j) x
8
y
8
; k) 64x
6
y
6
.
8. Desarrollar los siguientes binomios:
a) (x
2
+y
3
)
6
; b)
_
1
2
x
2
+y
3
_
5
; c)
_
1
x

1
y
_
7
; d) (0, 3x
3
y
2
ab)
3
.
Resp: a) x
12
+ 6x
10
y
3
+ 15x
8
y
6
+ 20x
6
y
9
+ 15x
4
y
12
+ 6x
2
y
15
+y
18
;
b)
1
32
(x
10
+ 10x
8
y
3
+ 40x6y
9
+ 80x
2
y
12
+ 32y
15
);
c)
1
x
7
y
7
(x
7
7xy
6
+ 21x
2
y
5
35x
3
y
4
+ 35x
4
y
3
21x
5
y
2
+ 7x
6
y x
7
);
d)
1
1000
(27x
9
y6 270abx
6
y
4
+ 900a
2
b
2
x
3
y
2
1000a
3
b
3
).
9. Hallar:
a) El tercer trmino de (x y)
5
;
b) El quinto trmino de (x
2
2y)
9
;
c) El penltimo trmino de (2x y
2
)
6
;
d) El trmino central de (3x
2
y
2
)
8
;
e) El coeciente de x
21
en (2x
4
x)
9
;
f ) El trmino central de
_
2
3
x
3
2x
_
6
.
Resp: a) 10x
3
y
2
; b) 2016x
10
y
9
; c) 12xy
10
; d) 5670x
8
y
8
; e) 2016;
f ) 20.
5.5. Divisin de polinomios
5.5.1. Mtodo normal
El algoritmo de la divisin para los nmeros enteros tiene un anlogo para polinomios que se
enuncia de la siguiente manera:
Sean p(x) y q(x) dos polinomios tales que q(x) no es una constante. Entonces existen polinomios
nicos c(x) y r(x) tales que el grado de r(x) es menor que el de q(x) y p(x) = c(x)q(x) + r(x).
Cuando el grado de p(x) es menor que el de q(x), el polinomio c(x) = 0 y p(x) = r(x). Por otra
parte, el grado de c(x) es igual al de p(x) menos el de q(x).
Las siguientes son sus propiedades ms importantes:
1.- En toda divisin, el grado del cociente es igual al grado del dividendo menos el grado del
divisor.
2.- En toda divisin el grado del dividendo es mayor o igual que el grado del divisor.
3.- En toda divisin el grado del divisor es mayor que el grado del resto (excepto polinomios
homogneos).
4.- En toda divisin el grado mximo del resto es igual al grado del divisor menos uno (en el caso
de divisin de polinomios homogneos, no se cumple esta propiedad).
5.- En el caso de polinomios homogneos, el grado del resto es mayor que el grado del divisor.
CAPTULO 5. POLINOMIOS 224
Para dividir dos polinomios se procede en el siguiente orden:
i) Se divide los signos mediante la regla de signos.
ii) Se divide los coecientes.
iii) Se divide los literales aplicando la teora de exponentes.
Ejemplo 5.5 Dividir los monomios:
16x
4
y
8
z
5
y 4x
2
y
5
z
4
Solucin
16x
4
y
8
z
5
4x
2
y
5
z
4
=
16
4
x
42
y
85
z
54
= 4x
2
y
3
z.
Para dividir polinomios por el mtodo normal, se procede de la siguiente manera:
a) Se ordenan los polinomios, generalmente en forma descendente.
b) Se escribe stos en lnea horizontal, uno a continuacin del otro y utilizando el signo de la
divisin aritmtica.
c) Se divide el primer trmino del dividendo, entre el primer trmino del divisor, lo cual da el
primer trmino del cociente.
d) Este primer trmino se multiplica por cada uno de los trminos del divisor y se resta de los
correspondientes trminos del dividendo (se cambian de signo los productos).
e) Se incorpora al residuo, el siguiente trmino del divisor. Se divide el primer trmino del resto
obtenido, entre el primer trmino del divisor y se obtiene el segundo trmino del cociente.
f ) Se procede como en el paso 4, y as sucesivamente, hasta terminar la divisin.
Ejemplo 5.6 Dividir el polinomio
p(x) = 2x
5
4x
4
+ 3x
3
2x2 + 3x 1
entre
q(x) = x
2
3x + 4
Solucin
Aplicando los pasos enunciados anteriormente, obtenemos:
2x
5
4x
4
+ 3x
3
2x
2
+ 3x 1 x
2
3x + 4
2x
5
+ 6x
4
8x
3
2x
3
+ 2x
2
+x 7
2x
4
5x
3
2x
2
+ 3x 1
2x
4
+ 6x
3
8x
2
x
3
10x
2
+ 3x 1
x
3
+ 3x
2
4x
7x
2
x 1
7x
2
21x + 28
22x + 27
El cociente es: c(x) = 2x
3
+ 2x
2
+x 7
El resto es: r(x) = 22x + 27.
CAPTULO 5. POLINOMIOS 225
5.5.2. Mtodo de coecientes separados
Adems de las consideraciones del mtodo normal, debe tenerse en cuenta lo siguiente:
a) Se trabaja solamente con los coecientes y sus signos.
b) En caso de faltar un trmino, se coloca en su lugar cero, tanto en el dividendo como en el
divisor.
c) Se procede a dividir estos coecientes siguiendo los pasos del mtodo normal, de esta manera
se obtiene los coecientes del cociente con sus signos.
d) Para determinar el grado del cociente y el resto se aplica las siguientes propiedades:
i) El grado del cociente es igual al grado del dividendo menos el grado del divisor.
ii) El grado del resto es igual al grado del divisor menos uno.
e) Este mtodo es recomendable para polinomios de una sola variable.
Ejemplo 5.7 Dividir el polinomio
p(x) = 2x
5
4x
4
+ 3x
3
2x
2
+ 3x 1
entre
q(x) = x
2
3x + 4
Solucin
Ubicamos los coecientes de los polinomios de forma ordenada, de la siguiente manera:
2 4 3 2 3 1 1 3 4
2 6 8 2 2 2 7
2 5 2 3 1
2 6 8
1 10 3 1
1 3 4
7 1 1
7 21 28
22 27
Por tanto
p(x) = (2x
3
+ 2x
2
+x 7)(x
2
3x + 4) + (22x + 27)
El cociente es: c(x) = 2x
3
+ 2x
2
+x 7
El resto es: r(x) = 22x + 27.
5.5.3. Mtodo de Horner
Es un caso particular del mtodo de coecientes separados y se emplea para la divisin de
polinomios de cualquier grado. Se procede de la siguiente forma:
a) Se escribe los coecientes del dividendo en una la de izquierda a derecha con su propio signo.
b) Se escribe los coecientes del divisor en una columna de arriba hacia abajo, a la izquierda del
primer trmino del dividendo; el primero de ellos con su propio signo y los restantes con signos
cambiados.
c) El primer trmino del dividendo se divide entre el primer trmino del divisor, obtenindose el
primer trmino del cociente, el cual se anota en la ltima la del cuadro.
d) Se multiplica este trmino del cociente solamente por los trminos del divisor a los cuales se
les cambio su signo, colocndose los resultados a partir de la segunda columna a la derecha.
e) Se reduce la siguiente columna (efectuando la operacin indicada) y se coloca este resultado en
la parte superior para dividirlo entre el primer coeciente del divisor y obtener el segundo trmino
CAPTULO 5. POLINOMIOS 226
del cociente.
f ) Se multiplica este cociente por los trminos del divisor a los cuales se cambio de signo, colocn-
dose los resultados a partir de la tercera columna a la derecha.
g) Se contina este procedimiento hasta obtener un trmino debajo del ltimo trmino del div-
idendo, separando inmediatamente los trminos del cociente y resto. El nmero de trminos del
resto est dado por el nmero de trminos que tiene el ltimo paso.
h) Se suma verticalmente obtenindose los coecientes del residuo. El grado del cociente y del
resto se obtiene tal como se indic en el mtodo de coecientes separados.
Ejemplo 5.8 Dividir el polinomio
p(x) = 8x
5
+ 14x
4
+ 5x
3
+ 16x
2
+ 3x + 2
entre
q(x) = 4x
2
+x + 3
Solucin
Grado del cociente = Grado del dividendo - Grado del divisor = 5 - 2 = 3.
Grado del residuo = Grado del divisor - 1 = 2 - 1 = 1
4 8 14 5 16 3 2
1 2 6
3 9
1 3
2 6
2 3 1 2 4 4
Cociente: c(x) = 2x
3
+ 3x
2
x + 2
Resto: r(x) = 4x 4
5.5.4. Regla de Runi
Este mtodo se utiliza para dividir polinomios cuando el divisor es un binomio de primer grado.
Se presenta tres casos:
1.- Cuando el divisor es de la forma x b.
2.- Cuando el divisor es de la forma ax b.
3.- Cuando el divisor es de la forma ax
n
b.
Primer caso: Forma del divisor x b.
a) Se escribe los coecientes del dividendo en lnea horizontal. Completando previamente, si fuese
necesario.
b) Se escribe el trmino independiente del divisor, con signo cambiado, un lugar a la izquierda y
un lugar abajo del primer coeciente del dividendo.
c) Se divide como en el caso de Horner, teniendo presente que el primer coeciente del cociente,
es igual al primer coeciente del dividendo.
d) Para obtener los coecientes del cociente, se separa la ltima columna, la cual constituye el
resto.
Ejemplo 5.9 Dividir el polinomio
p(x) = 4x
4
5x
3
+ 6x2 + 7x + 8 entre q(x) = x + 1
Solucin
Ubicamos los coecientes de los polinomios de forma ordenada, de la siguiente manera:
CAPTULO 5. POLINOMIOS 227
4 5 6 7 8
1 4 9 15 8
4 9 15 8 16
Grado del cociente = Grado del dividendo - Grado del divisor = 4 - 1 = 3
Cociente: c(x) = 4x
3
9x
2
+ 15x 8
Resto: 16.
Segundo caso: Forma del divisor ax b.
a) Se transforma el divisor a la primera forma, sacando en factor comn el primer coeciente del
divisor:
ax b = a
_
x
b
a
_
b) Se divide entre x
b
a
operando como el primer caso.
c) Los coecientes del cociente obtenido son divididos entre el coeciente de x del divisor.
d) El resto obtenido no se altera.
Ejemplo 5.10 Dividir el polinomio
p(x) = 18x
5
29x
3
5x
2
12x 16 entre q(x) = 3x + 2
Solucin
Factorizamos el denominador
18 0 29 5 12 16

2
3
12 8 14 6 12
18 12 21 9 18 4
Grado del cociente = Grado del dividendo - Grado del divisor = 5 - 1 = 4
Verdaderos coecientes del cociente:
18 12 21 + 9 18
3
= 6 4 7 + 3 6
El cociente es: c(x) = 6x
4
4x
3
7x
2
+ 3x 6
El resto es: r(x) = 4
Tercer caso: Forma del divisor ax
n
b.
La resolucin slo es posible por el mtodo de Runi cuando los exponentes de la variable del
dividendo son mltiplos enteros de la variable del divisor. El procedimiento se explica a travs del
siguiente ejemplo.
Ejemplo 5.11 Dividir el polinomio
p(x) = 6x
36
+ 17x
27
16x
18
+ 17x
9
+ 12 entre q(x) = 3x
9
+ 1
Solucin
1) Se observa que los coecientes de la variable del dividendo sean mltiplos del exponente de la
variable del divisor.
2) Se factoriza el divisor
3
_
x
9
+
1
3
_
3) Se divide como en el primer caso.
4) Cada uno de los coecientes del cociente obtenido, se divide entre coeciente de x del divisor.
CAPTULO 5. POLINOMIOS 228
6 +17 16 +17 +12

1
3
2 5 +7 8
6 +15 21 +24 4
Grado del cociente = Grado del dividendo - Grado del divisor = 36 - 9 = 27.
Verdaderos coecientes del cociente:
6 + 15 21 + 24
3
= 2 + 5 7 + 8
El cociente es: c(x) = 2x
27
+ 5x
18
7x
9
+ 8
El resto es: r(x) = 4.
5.5.5. Teorema del resto
Consiste en hallar el resto de una divisin sin realizar la divisin. El resto de dividir un poli-
nomio en x, racional y entero, entre un binomio de la forma ax b, es igual al valor numrico que
adquiere dicho polinomio cuando se reemplaza en l x por
b
a
.
Para hallar el resto se procede de la siguiente manera:
a) Se iguala el divisor a cero:
ax b = 0.
b) Se despeja x
x =
b
a
c) Se reemplaza en el polinomio dividendo la variable x por:

b
a
d) Se efecta operaciones, el resultado es el valor del resto
r = p
_

b
a
_
Ejemplo 5.12 Dividir el polinomio
p(x) = 6x
4
+ 3x
3
19x2 + 14x 15 entre q(x) = 2x 3
Solucin
Hacemos q(x) = 0:
2x 3 = 0 x =
3
2
Encontramos el resto
r = 6
_
3
2
_
4
+ 3
_
3
2
_
3
19
_
3
2
_
2
+ 14
_
3
2
_
15 = 3
CAPTULO 5. POLINOMIOS 229
5.6. Tarea
1. Usar la divisin normal para calcular el cociente y el residuo de las divisiones siguientes:
a) x
3
3x
2
+ 2x 1 entre x 2;
b) x
4
14x
3
+ 2x
2
+ 49x 36 entre x + 2;
c) x
4
+x
2
+x 2 entre x + 3;
d) x
5
x
3
32x entre x 3;
e) 3x
3
+ 2x
2
+ 5x + 10 entre x + 2;
f ) x
7
6x
5
+ 8x
3
3x
2
7x + 1 entre x + 1;
g) x
4
+ 10x
3
+ 22x
2
7x + 5 entre x + 4;
h) x
4
+x
3
22x
2
+ 15x 32 entre x 4.
Resp: a) c(x) = x
2
x, r(x) = 1; b) c(x) = x
3
16x
2
+ 34x 19, r(x) = 2;
c) c(x) = x
3
3x
2
+10x29, r(x) = 85; d) c(x) = x
4
+3x
3
+8x
2
+24x+40, r(x) = 120;
e) c(x) = 3x
2
4x+13, r(x) = 16; f ) c(x) = x
6
x
5
5x
4
+5x
3
+3x
2
6x1, r(x) = 2;
g) c(x) = x
3
+ 6x
2
2x + 1, r(x) = 1; h) c(x) = x
3
+ 5x
2
2x + 7, r(x) = 4.
2. Demostrar por medio de la divisin normal que:
a) (x 2)
2
es un factor de x
4
4x
3
+ 5x
2
4x + 4;
b) (x + 3)
2
es un factor de x
4
17x
2
+ 6x + 90;
c) (x + 1)
2
es un factor de 2x
5
+ 6x
4
+ 5x
3
x
2
3x 1;
d) (x 5)
2
es un factor de x
3
75x + 250;
e) (x + 4)
2
es un factor de x
4
+ 8x
3
128x 256.
3. Encuentre un polinomio de segundo grado sabiendo que es divisible por x + 2 y por x 4
y que el coeciente del trmino de mayor grado es 1.
4. Encuentre un polinomio de segundo grado sabiendo que sus dos races son 1 y -3, y que el
trmino independiente es 6.
5. Encuentre un polinomio de segundo grado p(x), sabiendo que p(4) = 22 y que una de sus
races es 2.
6. Hallar m y n para que el polinomio x
5
+mx
3
+n sea divisible por x + 1 y por x 1.
7. Hallar el valor de m para que el polinomio 2x
3
+mx 3 sea divisible por x 2.
8. Dado el polinomio p(x) = x
3
+ 3x
2
+ 6x + a, calcule el valor de a para que p(x) sea
divisible por x 1. El resto de ls divisin de p(x) entre x 1 sea igual a 15.
9. Hallar el valor de a para que el polinomio 2x
3
+ax
2
5x + 4 sea divisible por x + 1.
10. Hallar a, b y c sabiendo que en la divisin 4x
2
8x + 3 entre 2x + 1 se obtiene ax +b de
cociente y c de resto.
CAPTULO 5. POLINOMIOS 230
11. Calcule a y b para que el polinomio p(x) = x
3
+ ax
2
+ bx + b sea divisible por x 2 y
adems se cumpla p(1) = 10.
12. Hallar el valor de a para que x + 5 sea factor del polinomio x
3
4x 12a.
13. Hallar a y b para que el polinomio x
3
+ax
2
+bx + 5 sea divisible por x
2
+x + 1.
14. Calcule el valor de a para que el polinomio 3x
2
5x + a verique que sea divisible por
x 2. El resto de la divisin entre x 2 sea 8.
15. Hallar el valor de a para que el polinomio x
3
ax
2
ax + 1 sea divisible por x 1.
16. Hallar el valor de a para que al dividir el polinomio x
3
3x
2
ax+12 por x3 se obtenga
9 de resto.
17. Calcule a y b para que el polinomio x
3
ax
2
+7x +b sea divisible por x 5 y de un resto
de 9 al dividir por x 2.
18. Hallar el valor de a de forma que al dividir el trinomio 3x
2
+ax + 9 por x + 2, se obtenga
el mismo resto que al dividir 2x + 3x
3
+ 3 por x + 2.
19. El polinomio x
2
+bx +c es divisible entre x + 1. Sabiendo que si lo dividimos entre x 1
y x 3 se obtiene el mismo resto, hallar los valores de b y c.
20. Efecte la divisin entre los polinomios 3x
4
8x
3
+ 9x
2
2x 7 y x
2
x 1.
21. Efecte la divisin entre los polinomios 4x
5
2x
3
+ 3x y x
2
x + 2.
22. Hallar los valores a y b para que el polinomio p(x) = x
4
+ 2x
2
+ax +b se pueda expresar
en la forma p(x) = (x + 1)(x 2)(x
2
+x + 1).
23. Hallar los valores de a y b para que el polinomio p(x) = x
3
+ax +b tenga como raz doble
x = 1.
24. Hallar b, c y d para que el polinomio p(x) = x
3
+bx
2
+cx+d sea divisible por x+1, x2
y de resto 4 al dividirlo por x.
25. Encuentre un polinomio p(x) y un nmero real k que veriquen (x + 1)p(x) + k =
3x
5
x
2
+ 6x 12.
CAPTULO 5. POLINOMIOS 231
26. Hallar a y b para que el polinomio x
3
2x
2
+ (a 1)x + b sea divisible por x 1 y al
dividirlo por x de 2 de resto.
27. Hallar un polinomio de segundo grado, cuyo coeciente principal sea 2, que se anule para
x = 2 y que su valor numrico para x = 4 sea 5.
28. Hallar a y b para que el polinomio x
2
+ ax + b sea divisible por x 1 y adems verique
que al dividir por x + 1 se obtenga el mismo resto que al dividir por x + 3.
29. Hallar a y b para que el polinomio x
3
+ 6x
2
+ax +b sea divisible por x
2
4.
30. Hallar a y b para que el polinomio x
3
+ax
2
+bx + 5 sea divisible por x
2
+x + 1.
31. Hallar el valor de k para que el polinomio x
k
1 sea divisible por x + 1.
32. Sabiendo que x
2
+ 2 es un factor del polinomio p(x) = x
6
7x
2
6, expresar p(x) como
un producto de factores cuadrticos.
33. Dados los polinomios p(x) = x
5
+ ax
4
+ x
3
+ 2x
2
+ bx 1, q(x) = x
3
2x
2
+ 1 y
r(x) = x
2
3x 1. Hallar a y b en R tales que r(x) sea el resto de dividir p(x) por q(x).
34. Determinar los valores de a y b en R de modo que el polinomio p(x) = 2x
5
9x
4
+14x
3

(2a + 1)x
2
+ 2ax +b sea divisible por (x 2)
2
.
35. Demuestre que x+c es un factor del polinomio p(x) = x
n
+c
n
para todo n entero positivo
impar.
36. El polinomio p(x) = x
5
x
4

31
4
x
3
+3x
2
+
57
4
x admite como raz a
1
2

5. Encontrar las
restantes races de p(x).
37. El polinomio p(x) = x
3
+bx
2
+cx +d admite la raz r. Demuestre que las otras dos races
de p(x) son las races del polinomio q(x) = x
2
+ (b +r)x
d
r
.
38. En los problemas siguientes, obtenga el cociente y el resto de la divisin:
a) 6x
3
7x
2
+ 14x 8 dividido por 3x 2;
b) 25x
2
+ 6x
4
13x
3
12x
5
+ 7 + 3x dividido por 2 + 3x
2
;
c) x
6
1 dividido por x
3
1;
d) (2 i)x
3
+ (6 4i)x
2
+ (4 i)x (3 + 9i) dividido por x + (3 i).
CAPTULO 5. POLINOMIOS 232
39. a) Para qu valores de A y B se verica que A(2x 3) +B(x 2) = x?
b) Para qu valores de A, B y C se verica que A(x1)(x2) +B(x+2)(x2) +C(x+
2)(x 1) = x
2
5x 2?
40. Encuentre, sin efectuar la diviosin, el resto en las siguientes divisiones:
a) 4x
3
5x
2
1 dividido por x +
3
2
;
b) x
8
x
5
x
3
+ 1 dividido por x +i;
c) 2ix
5
x
4
(1 +i)x
3
8i dividido por
1
2
x 2i.
41. a) Sea p(x) = x
5
32. El cociente de dividir p(x) por d(x) es el polinomio q(x) =
x
4
2x
3
+ 4x
2
8x + 16 y el resto es -64. Hallar el polinomio d(x).
b) Sea p(x) =
1
2
x
5

9
2
x
4

9
2
x
3
+
21
4
x
2

3
4
x +
11
4
. El cociente de dividir p(x) por d(x) es
el polinomio q(x) =
1
2
x
3
5x
2
+x
3
4
y el resto es r(x) = x + 2. Hallar d(x).
42. a) El resto de dividir 2x
3
+ 2x
2
+ 6x + 14 por x
2
+k es 8. Hallar k.
b) Si x
3
+x
2
+ 3 se divide por x k el resto es 1. Hallar k.
43. Determine el polinomio p(x) de menor grado con coecientes reales que tenga races:
a) -1, -3, 4 y tal que p(2) = 5; b) -3 y -2 raz doble y p(1) = 4.
44. Resuelva en C las ecuaciones:
a)
2
3
x
2
(5 2x)(x
2
+x + 1) = 0; b)
_
x
1
2
_
(x
2
+ 9)
2
(2x
3
1) = 0.
45. Para cada uno de los siguientes polinomios se conoce una raz. determine las races restantes:
a) p(x) = x
3
x
2
4x 6, raz 3;
b) p(x) = x
4
+ 8x
3
+ 26x
2
+ 72x + 153, raz -3i;
c) p(x) = x
3
(1 i)x
2
(6 +i)x 6i, raz 3.
46. Sea p(x) = x
5
+Ax
3
70x
2
+Bx30. Sabiendo que -1 es raz de multiplicidad dos, obtenga
A y B.
47. a) El polinomio p(x) = x
4
+
3
2
x
3
+
3
2
x
2
+
3
2
x r, admite las races racionales r y 2r.
Halle todas las races de p(x).
b) El polinomio p(x) = 9x
3
+ 54x
2
x + k, admite como races r y r. Calcule las races
de p(x) y el valor de k.
48. La profundidad x a la cual ota una esfera slida de radio r y densidad d que se sumerge
en el agua es una raz positiva de la ecuacin x
3
r
2
dx
2
14d = 0. Halle la profundidad,
con aproximacin de dos cifras decimales, a la cual ota una esfera de 4 cm. de dimetro si
est hecha de un material cuya densidad es 0,5.
CAPTULO 5. POLINOMIOS 233
49. Para qu valores de k sern nmeros reales los ceros de p(x) = x
2
2kx k 1.
50. Sea p(x) un polinomio de grado n con coecientes reales. Muestre que ninguna recta par-
alela al eje X puede intersectar a la grca de p(x) ms de n veces.
51. Utilizando el mtodo de coecientes separados, dividir los siguientes polinomios:
a) 3x
4
2x
3
+x
2
3x + 1 entre x + 2;
b) 2x
3
+ 6x
2
x + 1 entre x + 2;
c) x
4
+ 2x
2
+ 1 entre x 1;
d) x
4
3x
2
+ 2x entre 2x 1;
e) x5 2x4 3x + 1 entre x
2
1;
f ) 3x
3
+ 6x
2
2x + 1 entre 2x + 3.
Resp: a) c(x) = 3x
3
8x
2
+17x 37, r(x) = 75; b) c(x) = 2x
2
+2x 5, r(x) = 11;
c) c(x) = x
3
+x
2
+ 3x + 3, r(x) = 4; d) c(x) =
1
2
x
3
+
1
4
x
2

11
8
x +
5
16
, r(x) =
5
16
;
e) c(x) = x
3
2x
2
+x 2, r(x) = 2x; f ) c(x) =
3
2
x
2

3
4
x
17
8
, r(x) =
59
16
.
52. Utilizando el mtodo de Horner, dividir los siguientes polinomios:
a) 3x
4
2x
3
+x
2
3x + 1 entre x + 2;
b) 2x
3
+ 6x
2
x + 1 entre x + 2;
c) x
4
+ 2x
2
+ 1 entre x 1;
d) x
4
3x
2
+ 2x entre 2x 1;
e) x
5
2x
4
3x + 2 entre x
2
1;
f ) 6x
4
+ 3x
3
2x + 1 entre 2x + 3.
Resp: a) c(x) = 3x
3
8x
2
+17x 37, r(x) = 75; b) c(x) = 2x
2
+2x 5, r(x) = 11;
c) c(x) = x
3
+x
2
+ 3x + 3, r(x) = 4; d) c(x) =
1
2
x
3
+
1
4
x
2

11
8
x +
5
16
, r(x) =
5
16
;
e) c(x) = x
3
2x
2
+x 2, r(x) = 2x; f ) c(x) =
3
2
x
2

3
4
x
17
8
, r(x) =
59
16
.
53. Efectuar las siguientes divisiones por el mtodo de Horner:
a) x
4
+x
3
+ 7x
2
6x + 8 entre x
2
+ 2x + 8;
b) x
5
5x
4
+ 9x
3
6x
2
x + 2 entre x
2
3x + 2;
c) x
8
y
8
entre x
3
+x
2
y +xy
2
+y
3
.
Resp: a) c(x) = x
2
x + 1, r(x) = 0; b) c(x) = x
3
2x
2
+x + 1, r(x) = 0;
c) c(x, y) = x
5
x
4
y +xy
4
y
5
, r(x, y) = 0.
54. Usar la regla de Runi para efectuar las divisiones:
a) x
3
3x
2
+ 2x 1 entre x 2;
b) x
4
14x
3
+ 2x
2
+ 49x 36 entre x + 2;
c) x
4
+x
2
+x 2 entre x + 3;
d) x
5
x
3
32x entre x 3;
e) 3x
3
+ 2x
2
+ 5x + 10 entre x + 2;
f ) x
7
6x
5
+ 8x
3
3x
2
7x + 1 entre x + 1;
g) x
4
+ 10x
3
+ 22x
2
7x + 5 entre x + 4;
h) x
4
+x
3
22x
2
+ 15x 32 entre x 4;
i) x
3
3x
2
+ 16x 44 entre x
1
10
;
j) 2x
4
+ 9x
3
+ 14x + 8 entre x +
1
2
;
k) 6x
4
+ 5x
3
+ 10x 4 entre x
1
2
;
l) 3x
4
+ 5x
3
+x
2
+ 17x 6 entre x
1
3
;
CAPTULO 5. POLINOMIOS 234
m) 3x
4
14x
3
57x
2
+ 65x 56 entre x 7;
n) 3x
4
+ 40x
3
+ 85x
2
+ 97x + 99 entre x + 11;
o) x
8
81x
6
2x
3
+ 18x2 + 8x 72 entre x 9;
p) 18x
5
+ 20x
4
+ 29x
3
+ 57x
2
+ 6x entre x +
1
9
;
q) x
4
+ 2x
3
13x
2
+ 13x 21 entre x + 3;
r) x
3
+ 0, 4x
2
0, 18x + 0, 33 entre x 0, 2.
55. Calcule el cociente y el resto de la divisin:
a) 3x
2
7x + 5 entre x
2
x + 1; b) x
3
x entre x
2
1;
c) x
3
8x
2
9x + 7 entre x 3.
56. Determine los valores de k, para que el polinomio q(x) sea divisible exactamente para p(x):
a) p(x) = kx
4
+ (k
2
2)x
3
k(k
2
+ 2)x
2
+k
2
(2 k
2
)x + 2k
3
, q(x) = x 1;
b) p(x) = 4kx
4
+(2k
2
+24k2)x
3
+(12k
2
+35k12)x
2
+6(3k
2
k3)x9k, q(x) = 2x+1;
c) p(x) = k
2
x
4
+ 2k
3
x
3
+ (k
4
1)x
2
2kx k
2
, q(x) = x + 2;
d) p(x) = 2x
4
+ (2k + 3)x
3
+k(3 10k)x
2
+ 3k
2
(2k 5)x + 9k
3
, q(x) = x 3;
e) p(x) = k
3
x
4
+ (k
4
+k
2
)x
3
+ (k
3
8k)x
2
4(2k
2
+ 3)x 12k, q(x) = x + 3;
f ) p(x) = x
5
5kx
4
+ 7k
2
x
3
+k
3
x
2
8k
4
x + 4k
4
, q(x) = x
2
1;
g) p(x) = x
5
+kx
4
6k
2
x
3
14k
3
x
2
11k
4
x 3k
5
, q(x) = x
2
2x 3.
57. Si el polinomio p(x) = (m+2n3)x
4
+(mn+5)x es idnticamente nulo, encuentre los
valores de m y n.
Resp: m =
7
3
, n =
8
3
58. Hallar m, n y p en la siguiente identidad:
7x
2
6x + 1 = m(x 1)(x 2) +n(x 2)(x 3) +p(x 3)(x 1).
Resp: m = 23, p = 17 n = 1
59. Hallar el valor de m para que la divisin sea exacta:
x
4
ma
2
x
2
+a
4
entre x
2
ax +a
2
Resp: m = 1
60. Cul es el valor de m si el polinomio p(x) = x
3
+m(a 1)x
2
+a
2
(mx+a 1) es divisible
entre x a + 1?
Resp: m = 1
61. Determine el valor de m si el polinomio x
3
+ 3x
2
5x +m es divisible entre x + 2.
Resp: m = 14
62. Dado el polinomio x
3
+ 2x
2
a + m, determinar el valor de m para que, al dividirlo por
x +
1
2
se obtenga de resto 1.
Resp: m = a
3
8
CAPTULO 5. POLINOMIOS 235
63. Determinar el valor numrico de m, del trinomio 3x
2
+mx+9, con la condicin de que, al
dividir ste para x +2, d el mismo resto que la divisin de 2x
3
+3x +3 por dicho binomio.
Resp: m = 20
64. Determine m y n si la divisin de x
4
3ax
3
+a
2
x
2
+ma
3
x +na
4
entre x
2
ax +a
2
deja
como resto 7a
3
x + 3a
4
.
Resp: m = 7, n = 1
65. El primer coeciente de un polinomio de segundo grado es 2; al dividirlo por x + 2, el
residuo es 0; al dividirlo por x + 3, el residuo es 9. Encuentre el polinomio.
Resp: 2x
2
+x 6
5.7. Mtodos de factorizacin
Factorizacin es la operacin que tiene por objeto transformar una expresin algebraica racional
y entera en otra equivalente que sea igual al producto de sus factores primos racionales y enteros.
5.7.1. Factor comn
Los principales mtodos para factorizar son los siguientes:
El factor comn de dos o ms expresiones algebraicas es la parte numrica y/o literal que est
repetida en cada una de dichas expresiones. El factor comn puede ser de tres tipos:
A) Factor comn monomio.
B) Factor comn polinomio.
C) Factor comn por agrupacin.
A) Factor comn monomio: Cuando el factor comn en todos los trminos es un
monomio.
Ejemplo 5.13 Factorizar la expresin
72x
2a
y
b
+ 48x
a+1
y
b+1
+ 24xy
2b
.
Solucin
El factor comn es 24x
a
y
b
, de este modo
24x
a
y
b
(3x
a
+ 2xy +y
b
)
B) Factor comn polinomio: Cuando el factor comn que aparece es un polinomio.
Ejemplo 5.14 Factorizar la expresin
(x + 1)
7
(x
2
+ 1)
10
(x + 1)
5
(x
2
+ 1)
11
Solucin
El factor comn es (x + 1)
5
(x
2
+ 1)
10
, luego
(x + 1)
5
(x
2
+ 1)
10
[(x + 1)
2
(x
2
+ 1)] (x + 1)
5
(x
2
+ 1)
10
(x2 + 2x + 1 x
2
1)
CAPTULO 5. POLINOMIOS 236
(x + 1)
5
(x
2
+ 1)
10
(2x) 2x(x + 1)
5
(x
2
+ 1)
10
.
C) Factor comn por agrupacin: Sea
x
m+n
+y
m+n
+ (xy)
m
+ (xy)
n
efectuando operaciones
x
m
x
n
+y
m
y
n
+x
m
y
m
+x
n
y
n
agrupando
(x
m
x
n
+x
m
y
m
) + (y
m
y
n
+x
n
y
n
)
factoricemos cada parntesis
x
m
(x
n
+y
m
) +y
n
(y
m
+x
n
)
el factor comn es el parntesis, as
(x
n
+y
m
)(x
m
+y
n
).
5.7.2. Mtodo de identidades
A) Diferencia de cuadrados
x
2m
y
2n
= (x
m
)
2
(y
n
)
2
= (x
m
y
n
)(x
m
+y
n
)
B) Suma o diferencia de cubos
x
3m
y
3n
= (x
m
)
3
(y
n
)
3
= (x
m
y
n
)(x
2m
x
m
y
n
+y
2n
)
C) Trinomio cuadrado perfecto
x
2m
2x
m
y
n
+y
2n
= (x
m
y
n
)
2
5.7.3. Mtodo del aspa
A) Aspa simple: Se usa para factorizar trinomios de la forma
ax
2n
bx
n
c
o, de la forma
x
2n
bx
n
c
Se descompone en dos factores al primer trmino, ax
2n
o x
2n
, segn sea el caso. Se coloca estos
factores en las puntas de la izquierda del aspa. El trmino in dependiente, incluyendo el signo,
tambin se descompone en dos factores, los que se coloca en las puntas de la derecha del aspa. Los
factores de la expresin dada son la suma horizontal de arriba y la suma horizontal de abajo. El
trmino central debe ser igual a la suma de los productos en aspa.
Ejemplo 5.15 Factorizar la siguiente expresin
x
4n
+ 7x
2n
+ 12
Solucin
El trmino x
4n
lo descomponemos en dos factores: x
2n
y x
2n
. El trmino independiente lo de-
scomponemos en dos factores: 4 y 3. Se coloca los factores en la punta izquierda y derecha del
aspa:
CAPTULO 5. POLINOMIOS 237
El trmino central es la suma de los productos en aspa
3x
2n
+ 4x
2n
= 7x
2n
Los factores son las sumas horizontales de arriba y abajo
x
4n
+ 7x
2n
+ 12 = (x
2n
+ 4)(x
2n
+ 3)
B) Aspa doble: Se usa para factorar polinomios de la forma
ax
2n
bx
n
y
n
cy
2n
dx
n
ey
n
f
y tambin para algunos polinomios de cuarto grado. Se ordena en forma decreciente para una de
las variables; luego, se traza y se ejecuta un aspa simple para los tres primeros trminos con trazo
continuo. A continuacin y, pegada al primer aspa, se traza otro, de tal modo que el producto de
los elementos del extremo derecho de este aspa multiplicados verticalmente sea el trmino inde-
pendiente.
Primer factor: suma de los elementos tomados horizontales de la parte superior.
Segundo factor: suma de los elementos tomados horizontalmente de la parte inferior.
Ejemplo 5.16 Factorizar la siguiente expresin
12x
2
7xy 10y
2
+ 59y 15x 63
Solucin
Tomando los tres primeros trminos:
12x
2
en dos factores: 4x y 3x
10y
2
en dos factores: 5y e 2y
63 en dos factores: 9 y 7
Vericamos los trminos:
8xy 15xy = 7xy segundo trmino
45y + 14y = 59y cuarto trmino
36x + 21x = 15x quinto trmino
Luego, la expresin factorizada es
(4x 5y + 7)(3x + 2y 9).
5.7.4. Mtodo de evaluacin
Este mtodo se aplica a polinomios de una sola variable que se caracteriza por anularse para
algunos de los divisores de su trmino independiente afectado de doble signo, o alguna combinacin.
Ejemplo 5.17 Factorizar la siguiente expresin
2x
4
+x
3
9x
2
4x + 4
Solucin
Los nmeros de prueba son 1, 2, 4,
1
2
. Dividiendo p(x) sucesivamente entre los factores
obtenidos por el mtodo de Runi:
CAPTULO 5. POLINOMIOS 238
2 1 9 4 4
1 2 1 8 4
2 1 8 4 0
2 4 6 4
2 3 2 0
2 4 2
2 1 0
Despus de la divisin se obtiene:
p(x) = (x + 1)(x 2)(x + 2)(2x 1)
5.7.5. Mtodo de articios de clculo
A) Reduccin a diferencia de cuadrados: Consiste en sumar y restar una misma cantidad
a la expresin dada para transformarla en una diferencia de cuadrados.
Ejemplo 5.18 Factorizar la siguiente expresin:
x
4
+ 2x
2
y
2
+ 9y
4
Solucin
Sumamos y restamos 4x
2
y
2
:
x
4
+ 6x
2
y
2
+ 9y
4
4x2y
2
(x
2
+ 3y
2
)
2
4x
2
y
2
(x
2
+ 3y
2
)
2
(2xy)
2
Realizamos la diferencia de cuadrados:
(x
2
+ 3y
2
2xy)(x
2
+ 3y
2
+ 2xy).
B) Sumas y restas: Consiste en sumar y restar una misma cantidad de tal manera que se
forme una suma o una diferencia de cubos.
Ejemplo 5.19 Factorizar la expresin:
x
5
+x
4
+ 1
Solucin
Sumamos y restamos x
3
+x
2
+x:
x
5
+x
4
+ (x
3
+x
2
+x) + 1 (x
3
+x
2
+x)
x
5
+x
4
+x
3
+x
2
+x + 1 x
3
x
2
x
Agrupamos
(x
5
+x
4
+x
3
) + (x
2
+x + 1) (x
3
+x
2
+x)
Sacamos factor comn
x
3
(x
2
+x + 1) + (x
2
+x + 1) x(x
2
+x + 1)
(x
3
x + 1)(x
2
+x + 1)
C) Cambio de variable: Consiste en cambiar una variable por otra, de manera que se
obtenga una forma de factorizacin conocida, o que tenga una forma ms simple.
CAPTULO 5. POLINOMIOS 239
Ejemplo 5.20 Factorizar la expresin:
1 +x(x + 1)(x + 2)(x + 3)
Solucin
Agrupamos:
1 + [x(x + 3)][(x + 1)(x + 2)] 1 + (x
2
+ 3x)(x
2
+ 3x + 2)
Haciendo x
2
+ 3x = y, obtenemos
1 +y(y + 2) y
2
+ 2y + 1 (y + 1)
2
Sustituimos la variable
(x
2
+ 3x + 1)
2
.
5.7.6. Factorizacin recproca
Polinomio recproco: Es aquel cuyos coecientes equidistantes de los extremos son iguales:
Ax
4
+Bx
3
+Cx
2
+Bx +A.
Ejemplo 5.21 Factorizar la siguiente expresin:
6x
4
+ 5x
3
+ 6x
2
+ 5x + 6
Solucin
Extraemos x
2
como factor comn:
x
2
_
6x
2
+ 5x + 6 +
5
x
+
6
x
2
_
Ordenando
x
2
_
6
_
x
2
+
1
x
2
_
+ 5
_
x +
1
x
_
+ 6
_
Haciendo x +
1
x
= y, entonces x
2
+
1
x
2
= y
2
2. Sustituyendo
x
2
[6(y
2
2) + 5y + 6]
Efectuamos la multiplicacin dentro de los corchetes
x
2
(6y
2
+ 5y 6)
Factorizamos el parntesis por el aspa simple, entonces
x
2
(3y 2)(2y + 3)
Reponiendo x
x
2
_
3
_
x +
1
x
_
2
_ _
2
_
x +
1
x
_
+ 3
_
x
2
_
3x
2
+ 3 2x
x
__
2x
2
+ 2 + 3x
x
_
(3x
2
2x + 3)(2x
2
+ 3x + 2).
5.8. Tarea
1. Factorar las expresiones:
CAPTULO 5. POLINOMIOS 240
a) 2ab 2bc ad +cd + 2b
2
bd;
b) x
5m
9x
3m
y
4n
;
c) a
2
(x y) +b
2
(y x) c
2
(x y);
d) 2a
3
x
3
x
6
a
6
+ 2b
3
y
3
+b
6
+y
6
;
e) 1 +xy +a(x +y) a(xy + 1) x y;
f )
1
5
a
7
b
4
x +
9
10
a
5
b
3
x
2
+
27
20
a
3
b
2
x
3
+
27
40
abx
4
;
g) 2a
2
y
5
+aby
5
aby
3
2a
2
y
3
;
h) 5a +ab + 5b +b
2
;
i) 5ax + 3by 5ay 3bx;
j) 3a
5
6a
4
b + 3a
3
b
2
;
k) 5a
5
b
3
+ 5a
2
b
9
;
l) ax bx +by +cy cx ay;
m) 4a
2
c
4
2ac c
3
;
n) 2a
5
c
5
+abc
5
abc
3
2a
2
c
3
.
Resp:
a) (a +b c)(2b d);
b) x
3m
(x
m
+ 3y
2n
)(x
m
3y
2n
);
c) (a
2
b
2
c
2
)(x y);
d) (x
3
+y
3
a
3
+b
3
)(x
3
y
3
a
3
b
3
);
e) (1 a)(x 1)(y 1);
f )
1
40
abx(3x + 2a
2
b)
3
;
g) ay
3
(y + 1)(y 1)(2a +b);
h) (a +b)(b + 5);
i) (x y)(5a 3b);
j) 3a
3
(a b)
2
;
k) 5a
2
b
3
(a +b
2
)(a
2
ab
2
+b
4
);
l) (a b c)(x y);
m) (2a +c
2
)(2a c c
2
);
n) ac
3
(2a
4
c
2
2a +bc
2
b).
2. Factorar las expresiones:
a) 36x
2
84xy + 49y
2
;
b) (x 3y)
2
16z
2
;
c) (x + 2y)
2
4(3x y)
2
;
d) x
8
256;
e) x
5m
9x
3m
y
4n
;
f ) x
6
+ 14x
3
+ 49;
g) (x +y)
3
+ (x y)
3
;
h) (x +y)
2
+ 6a(x +y) + 9a
2
;
i) x
6
64y
6
;
j) 32x
10
+y
25
;
k)
1
5
a
7
b
4
x +
9
10
a
5
b
3
x
2
+
27
40
abx
4
;
l) 125a
3
343b
3
;
m) (2a 3b)
2
(3a 2b)
2
;
n) (a b)
3
8b
3
;
o) 36a
2
b
2
100;
p) (a + 2b)
2
4(3a b)
2
;
q) 8a
3
+ (b 2a)
3
;
r) 9(a 3b)
2
16(b 2a)
2
;
s) 25 49a
2
b
6
c
4
;
t) (2a b)
3
(3b a)
3
;
u) 1 + 1000b
6
;
v) 8(a b)
3
+ 27(b 2a)
3
;
w) a
3
b
6
c
9
8d
6
;
x) 5a
5
b
3
+ 5a
2
b
9
;
y) a
3
b
3
27c
3
;
z) 4a
2
b
4
81c
2
.
Resp:
a) (6x 7y)
2
;
b) (x 3y + 4z)(x 3y 4z);
c) 7x(4y 5x);
d) (x + 2)(x 2)(x
2
+ 4)(x
4
+ 16);
e) x
3m
(x
m
+ 3y
2n
)(x
m
3y
2n
);
f ) (x
3
+ 7)
2
;
g) 2x(x
2
+ 3y
2
);
h) (x +y + 3a)
2
;
i) (x + 2y)(x 2y)(x
2
+ 2xy + 4y
2
)(x
2

2xy + 4y
2
);
j) (2x
2
+y
5
)(16x
8
8x
6
y
5
+4x
4
y
10
2x
2
y
15
+
y
20
);
k)
1
40
abx(3x + 2a
2
b)
3
;
l) (5a 7b)(25a
2
+ 35ab + 49b
2
);
m) 5(a +b)(b a);
n) (a 3b)(a
2
+ 3b
2
);
o) 4(3ab + 5)(3ab 5);
p) 7a(4b 5a);
q) b(12a
2
6ab +b
2
);
r) 5(a +b)(13b 11a);
s) (7ab
3
c
2
+ 5)(5 7ab
3
c
2
);
t) (3a 4b)(3a
2
3ab + 7b
2
);
u) (10b
2
+1)(10b
2
+

30b+1)(10b
2

30b+
1);
CAPTULO 5. POLINOMIOS 241
v) (b 4a)(52a
2
62ab + 19b
2
);
w) (ab
2
c
3
2d
2
)(a
2
b
4
c
6
+ 2ab
2
c
3
d
2
+d
4
);
x) 5a
2
b
3
(a +b
2
)(a
2
ab
2
+b
4
);
y) (ab 3c)(a
2
b
2
+ 3abc + 9c
2
);
z) (2ab
2
+ 9c)(2ab
2
9c).
3. Factorar las siguientes expresiones:
a) 6x
2
+ 7x 3;
b) 6x
2
+ 19x + 10;
c) 2x
2
7x 15;
d) 6x
2
+ 5x + 1;
e) 2x
2
+ 9x + 10;
f ) 3x
2
+ 16x + 16;
g) 3x
2
7x + 4;
h) x
2
+ 4x 5;
i) x
2
+x 6;
j) 2x
2
7x + 3;
k) 2mx
2
+ (mn 2n)x n
2
;
l) 4m
2
x
2
+ 2mx 2;
m) m
2
x
2
+ 4mx + 3;
n) mx
2
+ (m+n)x +n;
o) 3mx
2
+ (mn + 3)x +n;
p) 2x
2
+ 3nx +n
2
;
q) mx
2
(mn n)x n
2
;
r) m
2
x
2
mx 6;
s) mx
2
+ (2mn 1)x 2n;
t) 2nx
2
+(2n
2
1)xn. u) 64(a b)
3
+1.
Resp:
a) (3x 1)(2x + 3);
b) (2x + 5)(3x + 2);
c) (x 5)(2x + 3);
d) (3x + 1)(2x + 1);
e) (x + 2)(2x + 5);
f ) (3x + 4)(x + 4);
g) (3x 4)(x 1);
h) (x 1)(x + 5);
i) (x + 3)(x 2);
j) (x 3)(2x 1);
k) (mx n)(2x +n);
l) (2mx 1)(2mx + 2);
m) (mx + 1)(mx + 3);
n) (mx +n)(x + 1);
o) (3x +n)(mx + 1);
p) (2x +m)(x +n);
q) (x n)(mx +n);
r) (mx + 2)(mx 3);
s) (x + 2n)(mx 1);
t) (x +n)(2mx 1);
u) (4a4b+1)(16a
2
32ab4a+16b
2
+4b+1).
4. Factorar los siguientes polinomios:
a) 2x
2
+xy x y
2
+ 2y 1;
b) 2x
2
+ 3x 2y
2
y + 1;
c) 2x
2
+ 3xy 6x +y
2
4y + 4;
d) 2x
2
6xy + 9x + 4y
2
12y + 9;
e) x
2
xy 3x 6y
2
y + 2;
f ) 2x
2
2xy 3x 4y
2
+ 9y 2;
g) x
2
+xy +x 12y
2
+ 11y 2;
h) 2x
2
+ 5xy + 3x 3y
2
+ 2y + 1;
i) x
2
+ 5x y
2
+y + 6;
j) 3x
2
7xy + 5x 6y
2
+ 7y 2.
Resp:
a) (2x y + 1)(x +y 1);
b) (x +y + 1)(2x 2y + 1);
c) (2x +y 2)(x +y 2);
d) (2x 2y + 3)(x 2y + 3);
e) (x + 2y 1)(x 3y 2);
f ) (2x 4y + 1)(x +y 2);
g) (x 3y + 2)(x + 4y 1);
h) (x + 3y + 1)(2x y + 1);
i) (x +y + 2)(x y + 3);
j) (3x + 2y 1)(x 3y + 2).
5. Factorar los siguientes polinomios:
CAPTULO 5. POLINOMIOS 242
a) x
4
10x
3
+ 36x
2
54x + 27;
b) x
4
+ 9x
3
+ 11x
2
81x 180;
c) x
4
32x
2
+ 256;
d) 36x
4
36x
3
x
2
+ 9x 2;
e) x
4
4x
3
52x
2
+ 112x + 384;
f ) 4x
4
25x
2
+ 36;
g) x
4
20x
3
+ 140x
2
400x + 384;
h) x
4
+ 4x
3
34x
2
76x + 105;
i) x
4
+ 16x
3
+ 86x
2
+ 176x + 105.
Resp:
a) (x 1)(x 3)
3
;
b) (x + 3)(x 3)(x + 4)(x + 5);
c) (x + 4)
2
(x 4)
2
;
d) (2x + 1)(2x 1)(3x 1)(3x 2);
e) (x + 2)(x 4)(x + 6)(x 8);
f ) (x + 2)(x 2)(2x + 3)(2x 3);
g) (x 2)(x 4)(x 6)(x 8);
h) (x 1)(x + 3)(x 5)(x + 7);
i) (x + 1)(x + 3)(x + 5)(x + 7).
6. Factorar los siguientes polinomios:
a) 2x
5
+ 9x
4
26x
3
+ 6x + 9;
b) x
5
4x
4
+x
3
+ 10x
2
4x 8;
c) x
5
+ 7x
4
3x
3
79x
2
46x + 120;
d) 4x
5
7x
3
+x
2
+ 3x 1;
e) x
5
3x
4
8x
3
+ 24x
2
+ 16x 48;
f ) x
5
3x
4
5x
3
+ 15x
2
+ 4x 12;
g) x
5
x
4
2x
3
+ 2x
2
+x 1;
h) x
5
x
4
8x
3
+ 8x
2
+ 16x 16;
i) 2x
5
x
4
36x
3
+ 18x
2
+ 162x 81.
Resp:
a) (x 1)
2
(x + 3)
2
(2x + 1);
b) (x + 1)
2
(x 2)
3
;
c) (x 1)(x + 2)(x 3)(x + 4)(x + 5);
d) (x 1)(x + 2)
2
(2x 1)
2
;
e) (x 3)(x + 2)
2
(x 2)
3
;
f ) (x + 1)(x 1)(x + 2)(x 2)(x 3);
g) (x + 1)
2
(x 1)
3
;
h) (x 1)(x + 2)
2
(x 2)
2
;
i) (x + 3)2(x 3)
2
(2x 1).
7. Factorar los siguientes polinomios:
a) x
6
x
5
11x
4
+ 17x
3
+ 22x
2
52x + 24;
b) 16x
6
96x
5
+ 136x
4
+ 48x
3
71x
2
6x + 9;
c) x
6
+ 2x
5
31x
4
64x
3
+ 224x
2
+ 512x + 256;
d) 2x
6
15x
5
+ 27x
4
+ 26x
3
72x
2
27x + 27;
e) 4x
6
+ 32x
5
15x
4
115x
3
+ 55x
2
+ 111x 72;
f ) 36x
6
+ 60x
5
83x
4
120x
3
+ 83x
2
+ 60x 36;
g) x
6
+ 18x
5
+ 127x
4
+ 444x
3
+ 799x
2
+ 690x + 225;
h) x
6
7x
5
3x
4
+ 151x
3
514x
2
+ 708x 360;
i) 32x
6
+ 112x
5
+ 32x
4
56x
3
22x
2
+ 7x + 3;
j) 81x
6
+ 324x
5
+ 306x
4
72x
3
71x
2
+ 4x + 4.
Resp:
a) (x + 2)(x + 3)(x 1)
2
(x 2)
2
;
b) (x 3)
2
(2x + 1)
2
(2x 1)
2
;
c) (x + 1)
2
(x + 4)
2
(x 4)
2
;
d) (x + 1)
2
(x 3)
3
(2x 1);
e) (x + 8)(x 1)
3
(2x + 3)
2
;
f ) (x + 1)(x 1)(2x + 3)
2
(3x 2)
2
;
g) (x + 1)
2
(x + 3)
2
(x + 5)
2
;
h) (x + 5)(x 3)
2
(x 2)
3
;
i) (x + 3)(2x 1)
2
(2x + 1)
3
;
j) (x + 2)
2
(3x + 1)
2
(3x 1)
2
.
CAPTULO 5. POLINOMIOS 243
8. Factorar los siguientes polinomios:
a) 2x
3
x
2
y 2xy
2
+y
3
;
b) x
4
+ 3x
3
y 7x
2
y
2
27xy
3
18y
4
;
c) 6x
3
+ 11x
2
y xy
2
6y
3
;
d) 4x
4
17x
2
y
2
+ 4y
4
;
e) 6x
4
5x
3
y 20x
2
y
2
+ 25xy
3
6y
4
;
f ) 2x
4
5x
3
y + 5xy
3
2y
4
;
g) x
4
13x
2
y
2
+ 36y
4
;
h) 4x
4
+ 4x
3
y 39x
2
y
2
36xy
3
+ 27y
4
;
i) x
3
2x
2
y xy
2
+ 2y
3
.
Resp:
a) (x +y)(x y)(2x y);
b) (x +y)(x + 2y)(x + 3y)(x 3y);
c) (x +y)(2x + 3)(3x 2y);
d) (x + 2y)(x 2y)(2x +y)(2x y);
e) (x + 2y)(x 2y)(x + 3y)(x 3y);
f ) (x +y)(x y)(2x y)(x 2y);
g) (x + 2y)(x 2y)(x + 3y)(x 3y);
h) (x + 3y)(2x y)(x 3y)(2x + 3y);
i) (x +y)(x y)(x 2y).
9. Descomponer en factores:
a) x
4
1;
b) x
6
1;
c) x
6
+ 1;
d) x
4
18x
2
+ 81;
e) x
12
2x
6
+ 1;
f ) x
5
+x
3
x
2
1;
g) x
4
+ 2x
3
2x 1;
h) 4y
2
z
2
(y
2
+z
2
x
2
)
2
;
i) x
4
x
2
y
2
+y
4
;
j) x
4
+ 4x
2
5;
k) 4x
4
+ 5x
2
+ 1;
l) z
4
(1 +xy)z
2
+xy;
m) x
4
+ 324;
n) x
4
+x
2
+ 1;
o) x
8
+x
4
+ 1;
p) 2x
4
+x
3
+ 4x
2
+x + 2;
q) x
4
+ 3x
3
+ 4x
2
6x 12;
r) (x
2
+x + 3)(x
2
+x + 4) 12;
s) x
5
+x
3
x
2
1;
t) 2x
2
y+4xy
2
x
2
z+az
2
4y
2
z+2yz
2
4xyz;
u) (xy +xz +yz)(x +y +z) xyz;
v) x(y2z)
2
+y(x2z)
2
2z(x+y)
2
+8xyz;
w) x
3
(x
2
7)
2
36x;
x) (x +y)
5
(x
5
+y
5
);
y) x
2
y
2
(y x) +y
2
z
2
(z y) +x
2
z
2
(x z);
z) 8x
3
(y +z) y
3
(2x +z) z
3
(2x y).
Resp:
a) (x 1)(x + 1)(x
2
+ 1);
b) (x 1)(x + 1)(x
2
+x + 1)(x
2
x + 1);
c) (x
2
+ 1)(x
2
+

3x + 1)(x
2

3x + 1);
d) (x 3)
2
(x + 3)
2
;
e) (x 1)
2
(x + 1)
2
(x
2
+x + 1)
2
(x
2
x + 1)
2
;
f ) (x 1)(x
2
+ 1)(x
2
+x + 1);
g) (x 1)(x + 1)
3
;
h) (x y +z)(x +y z)(x +y +z)(x +y +z);
i) (x
2
+xy +y
2
)(x
2
xy +y
2
);
j) (x + 1)(x 1)(x
2
+ 5);
k) (x
2
+ 1)(4x
2
+ 1);
l) (z 1)(z + 1)(z
2
xy);
m) (x
2
+ 6x + 18)(x
2
6x + 18);
n) (x
2
+x + 1)(x
2
x + 1);
o) (x
2
+x + 1)(x
2
x + 1)(x
2
+

3x + 1)(x
2

3x + 1);
p) (x
2
+ 1)(2x
2
+x + 2);
CAPTULO 5. POLINOMIOS 244
q) (x +

2)(x

2)(x
2
+ 3x + 6);
r) x(x + 1)(x
2
+x + 7);
s) (x 1)(x
2
+ 1)(x
2
+x + 1);
t) (x + 2y)(2y z)(x z);
u) (x +y)(y +z)(z +x);
v) (x 2z)(y 2z)(x +y);
w) x(x 1)(x + 1)(x 2)(x + 2)(x 3)(x + 3);
x) 5xy(x +y)(x
2
+xy +y
2
);
y) (x y)(y z)(z x)(xy +yz +zx);
z) (y +z)(2x y)(2x +z)(2x +y z).
10. Descomponer en factores:
a) (x +y +z)
3
(x
3
+y
3
+z
3
);
b) x
4
+ 9;
c) x
4
+y
4
;
d) x
3
+ 5x
2
+ 3x 9;
e) x(x + 1)(x + 2)(x + 3) + 1;
f ) (x + 1)(x + 3)(x + 5)(x + 7) + 15;
g) 2(x
2
+2x1)
2
+5(x
2
+2x1)(x
2
+1) +
2(x
2
+ 1)
2
;
h) (x y)z
3
(x z)y
3
+ (y z)x
3
;
i) (x y)
3
+ (y z)
3
(x z)
3
;
j) (x
2
+y
2
)
3
(y
2
+z
2
)
3
(x
2
z
2
)
3
;
k) x
4
+ 2x
3
y 3x
2
y
2
4xy
3
y
4
;
l) x
2
y
2
+xy
2
+x
2
z +y
2
z +yz
2
+ 3xyz;
m) x
4
+y
4
+z
4
2x
2
y
2
2x
2
z
2
2y
2
z
2
;
n) x
5
+x
4
+x
3
+x
2
+x + 1;
o) x
4
+ 2x
3
+ 3x
2
+ 2x + 1;
p) x
4
2x
3
y 8x
2
y
2
6xy
3
y
4
;
q) x
4
+x
2
+

2x + 2;
r) x
10
+x
5
+ 1.
Resp:
a) 3(x +y)(y +z)(z +x);
b) (x
2
+

6x + 3)(x
2

6x + 3);
c) (x
2
+

2xy +y
2
)(x
2

2xy +y
2
);
d) (x 1)(x + 3)
2
;
e) (x
2
+ 3x + 1)
2
;
f ) (x + 2)(x + 6)(x
2
+ 8x + 10);
g) (3x
2
+ 4x 1)(3x
2
+ 2x + 1);
h) (x y)(y z)(x z)(x +y +z);
i) 3(x y)(y z)(z x);
j) 3(x +z)(x z)(x
2
+y
2
)(y
2
+z
2
);
k) (x
2
xy y
2
)(x
2
+ 3xy +y
2
);
l) (x +y +z)(xy +yz +xz);
m) (x+y+z)(x+yz)(xy+z)(xyz);
n) (x + 1)(x
2
+x + 1)(x
2
x + 1);
o) (x
2
+x + 1)
2
;
p) (x +y)
2
(x
2
4xy y
2
);
q) (x
2
+

2x + 1)(x
2

2x + 2);
r) (x
2
+x+1)(x
8
x
7
+x
5
x
4
+x
3
x+1).
5.9. Mximo comn divisor y mnimo comn multiplo
Si los nmeros naturales p
1
y p
2
son divisibles por un mismo nmero natural p, este ltimo se
denomina divisor comn de los nmeros p
1
y p
2
. El nmero natural mximo por el que se dividen
p
1
y p
2
lleva el nombre de mximo comn divisor (MCD) de dichos nmeros. Si el MCD de dos
nmeros es igual a la unidad, se llaman recprocamente primos.
Si los nmeros naturales p
1
y p
2
son recprocamente primos y el nmero natural p es divisible
tanto por p
1
como por p
2
, entonces p se divide por el producto p
1
p
2
.
CAPTULO 5. POLINOMIOS 245
Se llama mnimo comn mltiplo (mcm) de dos nmeros naturales p
1
y p
2
un nmero natural
mnimo que es divisible por p
1
y por p
2
.
Es decir, el mcm de dos nmeros se obtiene multiplicando cualquiera de ellos por el cociente de
dividir el otro por el MCD de ambos. Si stos son primos entre s, el mcm es su producto.
Para determinar el MCD (mcm) de varios enteros, calclese el MCD (mcm) de dos de ellos;
despus el MCD (mcm) de ste con un tercer entero, y se sigue as, sucesivamente, hasta haber
utilizado todos los enteros dados. El ltimo MCD (mcm) calculado es el MCD (mcm) buscado.
5.9.1. Divisiones sucesivas
Denicin 5.12 Mximo comn divisor
El mximo comn divisor (MCD) de dos o ms polinomios, es el mayor divisor posible de todos
ellos.
Dos polinomios pueden ser divisibles por un mismo polinomio, que se llama entonces comn
divisor.
De todos los divisores comunes de dos polinomios, se asigna especial inters al divisor comn
de grado mximo. Esta expresin se denomina mximo comn divisor. Veremos a continuacin
que el mximo comn divisor es esencialmente nico, y que puede encontrarse por una serie de
operaciones regulares.
Sean dos polinomios dados p(x) y q(x). Dividiendo p(x) por q(x), sea c(x) el cociente y r(x) el
resto tal que
p(x) = c(x)q(x) +r(x)
Si r(x) no es un polinomio idnticamente nulo, podremos continuar dividiendo q(x) por r(x),
obteniendo un cociente c
1
(x) y el resto r
1
(x) tal que
q(x) = c
1
(x)r(x) +r
1
(x)
Nuevamente, si r
1
(x) no es idnticamente nulo, la divisin de r(x) por r
1
(x) lleva a otra iden-
tidad
r(x) = c
1
(x)r
1
(x) +r
2
(x); ...etc.
Desde que el grado de los polinomios q(x), r(x), r
1
(x), ... disminuye y las operaciones pueden
continuarse mientras el ltimo resto obtenido no sea un polinomio idnticamente nulo, debemos
llegar a un resto r
n
(x) que divida exactamente al resto precedente, de manera que tendremos n
identidades:
p(x) = c(x)q(x) +r(x)
q(x) = c
1
(x)r(x) +r
1
(x)
r(x) = c
1
(x)r
1
(x) +r
2
(x)
...
r
n2
(x) = c
n1
(x)r
n1
(x) +r
n
(x)
r
n1
(x) = c
n
(x)r
n
(x).
De estas identidades puede inferirse lo siguiente:
a) Que r
n
(x) es un divisor comn de p(x) y q(x).
CAPTULO 5. POLINOMIOS 246
b) Que cualquier divisor de estos polinomios divide a r
n
(x).
Se cumplen en las expresiones literales las siguientes propiedades:
1.- El MCD de dos o ms expresiones enteras es el producto de sus factores primos comunes,
numricos y literales, elevados a las potencias de menor exponente que de ellos aparezcan en la
descomposicin de dichas expresiones.
2.- Toda cantidad entera que divida a dos o ms expresiones enteras, divide tambin al MCD de
estas.
3.- Si dos o ms cantidades enteras se multiplican o dividen por una misma cantidad, el MCD
quedara multiplicado o dividido por la misma.
4.- Si dos o ms expresiones enteras se dividen por su MCD, los coecientes sern primos entre
s.
5.- El MCD de dos o ms expresiones enteras no vara aunque se multiplique o divida una de ellas
por otra cantidad entera prima con las restantes, ya que entonces los factores primos comunes de
dichas expresiones sern los mismos despus de la multiplicacin o divisin por la cantidad prima.
6.- El MCD de tres o ms expresiones enteras es el mismo que el de todas ellas, excepto dos, y
el MCD de estas dos.
Para hallar el MCD de polinomios deben tenerse en cuenta los siguientes principios:
i) El MCD de dos polinomios, cuando la divisin es exacta, es el divisor, lo cual es evidente.
ii) El MCD de dos polinomios, cuando la divisin es inexacta, es el mismo que el del divisor y el
resto, siempre que el cociente y el resto obtenidos sean enteros.
Ejemplo 5.22 Encuentre el mcd de los siguientes polinomios:
_
p(x) = x
6
+ 2x
5
+x
3
+ 3x
2
+ 3x + 2
q(x) = x
4
+ 4x
3
+ 4x
2
x 2
Solucin
El primer paso para el desarrollo del algoritmo de Euclides es dividir p(x) por q(x). La divisin se
realiza con los coecientes separados como sigue:
1 2 0 1 3 3 2 1 4 4 1 2
1 4 4 1 2 1 2 4
2 4 2 5
2 8 8 2 4
4 10 3 1 2
4 16 16 4 8
6 13 3 10
El primer resto es
r(x) = 6x
3
13x
2
+ 3x + 10
Ahora tenemos que dividir p(x) por r(x). Esta divisin introducir coecientes fraccionarios y, para
evitar este inconveniente, podemos multiplicar q(x) por 6; de esta manera s(x) estar multiplicado
por una constante, la cual no tiene importancia para nuestro propsito. La siguiente operacin es,
por lo tanto:
6 24 24 6 12 6 13 3 10
6 13 3 10 1 11
11 27 4 12
CAPTULO 5. POLINOMIOS 247
Nuevamente, para evitar fracciones, multiplicamos los nmeros de la ltima la por 6; esto cambia
el resto nal de manera que en lugar del s(x) que obtendramos por el procedimiento normal,
tendremos s(x) multiplicado por una constante. La operacin continua as:
66 162 24 72
66 142 33 110
19 57 38
Todos los coecientes tienen aqu el factor 19; simplicando podemos tomar como s(x), al siguiente
polinomio
s(x) = x
2
+ 3x + 2
Tngase en cuenta que en la la en la que se escriben los coecientes del cociente, los nmeros
ya no representan dichos coecientes. Pero esto no tiene importancia, ya que no nos interesan los
coecientes, sino solamente los restos y de estos no nos preocupan los factores constantes. Ahora
tenemos que dividir r(x) por s(x). Esta divisin se realiza de la siguiente manera:
6 13 3 10 1 3 2
6 18 12 6 5
5 15 10
5 15 10
0 0 0
Este procedimiento naliza con resto nulo. Por lo tanto las operaciones terminan aqu y el MCD
pedido es:
x
2
+ 3x + 2
Denicin 5.13 Mnimo comn mltiplo
El mnimo comn mltiplo (mcm) de dos o ms polinomios, es el menor polinomio distinto de cero
que es mltiplo de todos ellos.
El mnimo comn mltiplo (mcm) de dos o ms polinomios, es el menor polinomio distinto de
cero que es mltiplo de todos ellos.
Si el producto de dos polinomios lo dividimos por su MCD, el cociente es el mnimo comn
mltiplo.
El clculo del mnimo comn mltiplo de ms de dos polinomios, consiste en hallar el de dos
de ellos y despus hallar el mcm del siguiente polinomio con el mcm de los dos primeros y as
sucesivamente hasta el ltimo polinomio.
Ejemplo 5.23 Encuentre el mcm de los siguientes polinomios:
_
p(x) = x
6
+ 2x
5
+x
3
+ 3x
2
+ 3x + 2
q(x) = x
4
+ 4x
3
+ 4x
2
x 2
Solucin
El mcm lo calculamos de la siguiente manera: multiplicamos los dos polinomios
p(x)q(x) = x
10
+ 6x
9
+ 12x
8
+ 8x
7
+ 3x
6
+ 15x
5
+ 25x
4
+ 15x
3
x
2
8x 4
Dividimos este polinomio para el MCD =x
2
+ 3x + 2 y obtenemos
mcm = x
8
+ 3x
7
+x
6
x
5
+ 4x
4
+ 5x
3
+ 2x
2
x 2
CAPTULO 5. POLINOMIOS 248
5.9.2. Por factorizacin
En el caso menos frecuente en que se dan polinomios en forma factorizada, su MCD se calcula
por el procedimiento ordinario. En efecto, ste es el producto de todos los polinomios primos que
se encuentran en las diversas factorizaciones, pero con exponentes iguales al menor de los que
aparecen en cada una de las descomposiciones en irreducibles. Este procedimiento es relativamente
limitado.
Ejemplo 5.24 Encuentre el MCD de los siguientes polinomios:
_
p(x) = x
8
2x
6
+x
5
+ 2x
2
x 1
q(x) = x
8
+x
5
+x
4
x 2
Solucin
Factorando estos polinomios dentro de los reales, obtenemos:
p(x) = (x + 1)(x 1)(x
2
+ 1)(x
4
2x
2
+x + 1)
q(x) = (x + 1)(x 1)(x
2
+ 1)(x
4
+x + 2)
Por lo tanto:
MCD = (x + 1)(x 1)(x
2
+ 1).
El mcm de dos polinomios distintos de cero p(x) y q(x) es el polinomio h(x) con coeciente
inicial uno que es divisible por p(x) y q(x) y cuyo grado es el menor de todos los grados de aquellos
polinomios no nulos divisibles por p(x) y q(x). Puede verse que si a
0
, b
0
son los coecientes iniciales
respectivos de p(x) y q(x) respectivamente y si d(x) es su MCD, entonces
p(x)q(x)
a
0
b
0
d(x)
es su mcm h(x). El mcm de un conjunto de polinomios p
1
(x), p
2
(x), ..., p
k
(x) puede calcularse
usando la propiedad de que si h
i
(x) es el mcm de p
1
(x), p
2
(x), ..., p
i
(x) entonces el mcm de p
1
(x),
p
2
(x), ..., p
i
(x)p
i+1
(x) es igual al mcm de h
i
(x) y p
i+1
(x).
Ejemplo 5.25 Encuentre el mcm de los siguientes polinomios:
_
p(x) = x
8
2x
6
+x
5
+ 2x
2
x 1
q(x) = x
8
+x
5
+x
4
x 2
Solucin
Factorando estos polinomios dentro de los reales, obtenemos:
p(x) = (x + 1)(x 1)(x
2
+ 1)(x
4
2x
2
+x + 1)
q(x) = (x + 1)(x 1)(x
2
+ 1)(x
4
+x + 2)
Por lo tanto:
mcm = (x + 1)(x 1)(x
2
+ 1)(x
4
2x
2
+x + 1)(x
4
+x + 2).
CAPTULO 5. POLINOMIOS 249
5.10. Tarea
1. Determine el MCD y mcm entre los polinomios:
a)
_
p(x) = x
5
x
4
2x
3
+ 2x
2
+x 1
q(x) = 5x
4
4x
3
6x
2
+ 4x + 1
;
b)
_
p(x) = 2x
4
+ 2x
3
3x
2
2x + 1
q(x) = x
3
+ 2x
2
+ 2x + 1
;
c)
_
p(x) = x
4
6x
2
8x 3
q(x) = x
3
3x 2
;
d)
_
p(x) = 2x
5
+ 4x
4
+x
3
x
2
+x + 1
q(x) = 6x
5
2x
4
+x
3
+ 2x
2
x + 1
;
e)
_
p(x) = 2x
6
+ 3x
5
+x
4
+ 7x
3
+ 4x
2
+ 4x + 5
q(x) = x
4
x
3
x 1
;
f )
_
p(x) = 10x
6
9x
5
12x
4
+ 2x
2
x 1
q(x) = 4x
5
+x
4
7x
3
8x
2
x + 1
;
g)
_
p(x) = x
3
3x
2
+ 4
q(x) = x
3
2x
2
x + 2
;
h)
_
p(x) = x
4
x
3
7x
2
+x + 6
q(x) = x
3
+x
2
4x 4
;
i)
_
p(x) = x
3
2x + 4
q(x) = x
6
2x
4
+ 4x
3
+x
2
2x + 2
;
j)
_
p(x) = x
3
+ 4x
2
7x 10
q(x) = x
3
7x + 6
;
k)
_
p(x) = x
3
5x
2
4x + 20
q(x) = x
3
2x
2
+ 3x 6
;
l)
_
p(x) = x
4
+x
3
11x
2
+x 12
q(x) = x
4
3x
3
+ 2x
2
3x + 1
.
2. Determine los valores de k, para que xk sea el MCD de los polinomios p(x) y q(x). Cul
es el mcm?:
a)
_
p(x) = x
4
6x
3
+ 9x
2
+ 4x 12
q(x) = x
4
+ 6x
3
+ 5x
2
24x 36
;
b)
_
p(x) = x
4
+ 4x
3
+ 2x
2
4x 3
q(x) = x
4
+ 8x
3
+ 2x
2
80x 75
.
3. Determine los valores de k, para que x
2
+kx+k sea el MCD de los polinomios p(x) y q(x).
Cul es el mcm?:
a)
_
p(x) = x
5
2x
4
11x
3
+ 15x
2
+ 18x + 27
q(x) = x
5
+ 2x
4
+x
3
x
2
2x 1
;
b)
_
p(x) = x
5
+x
4
5x
3
15x
2
16x 6
q(x) = x
5
+ 3x
4
x
3
5x
2
4x + 6
.
CAPTULO 5. POLINOMIOS 250
4. Determine los valores de k, para que el polinomio h(x) sea el MCD entre los polinomios
p(x) y q(x). Cul es el mcm?:
a)
_

_
p(x) = kx
4
(10k 3)x
3
+ (25k 29)x
2
+ 65x + 25
q(x) = kx
4
+ (10k + 3)x
3
+ (25k + 31)x
2
+ 85x + 25
h(x) = 2x
2
+ 3x + 1
;
b)
_

_
p(x) = x
4
+ (k 1)x
3
(k 4)x
2
+ (3k 1)x + 3
q(x) = x
4
+ (k 2)x
3
2(k + 7)x
2
(15k + 2)x 15
h(x) = x
2
3x + 1
.
5. Hallar el MCD de tres polinomios P
1
, P
2
, P
3
, si se conoce que el MCD de P
1
y P
2
es
x
2
x 2 y el MCD de P
2
y P
3
es x
4
+ 5x
2
+ 8x + 2.
Resp: x + 1.
6. Dada la fraccin
x
3
+ 2ax
2
+ (2a + 1)x + 6
x
3
+ 2bx
2
+ (2b + 1)x + 10
Hallar a y b para que sea simplicable. Determinar el MCD del numerador y denominador
si se sabe que es de la forma x
2
+px +q.
Resp: MCD = x
2
+x + 2, a = 2 y b = 1.
7. Hallar a y b y el MCD para que la fraccin simplicada sea
x
3
+ (a + 2)x
2
+ (2a 15)x 15a
x
3
+ (b + 2)x
2
+ (2b 15)x 15b
=
x + 2
x + 1
Resp: MCD = x
2
+ 2x 15, a = 2 y b = 1.
8. El MCD entre P
1
y P
2
es (x +2)(x 1), y su mcm es (x 1)
2
(x
2
4)(x 3). Determinar
P
2
si se conoce que P
1
es x
3
3x + 2.
Resp: x
4
4x
3
x
2
+ 16x 12.
9. El MCD y el mcm de dos polinomios P
1
y P
2
de igual grado son respectivamente x + 1
y x
3
+ 2x
2
x 2. Si se conoce que el trmino independiente del polinomio P
2
es positivo,
determinar P = 2P
1
3P
2
.
Resp: x
2
9x 8.
10. Hallar dos polinomios de cuarto grado si:
_
MCD = 2x
3
2x
2
+ 2x 2
mcm = 2x
5
4x
4
2x + 4
Resp: p
1
(x) = 2x
4
+ 2, p
2
(x) = 2x
4
6x
3
+ 6x
2
6x + 4.
CAPTULO 5. POLINOMIOS 251
11. Tres polinomios de igual grado tienen como MCD a 2x
2
+x 6 y como mcm 2x
5
3x
4

10x
3
+ 15x
2
+ 8x 12. Determine dichos polinomios.
Resp: p
1
(x) = 2x
3
+3x
2
5x 6, p
2
(x) = 2x
3
x
2
7x +6, p
3
(x) = 2x
3
3x
2
8x +12.
12. El MCD de P(x) y R(x) es x
2
+x + 1 y el mcm de los dos polinomios es x
4
+x
3
x 1.
Hallar 2P(x) 3R(x), si se conoce que los polinomios P(x) y R(x) son de igual grado.
Resp: x
3
6x
2
6x 5.
13. Determine P
2
(x) si se conoce que:
_

_
MCD = (2x + 3)(x 5)
p
1
(x) = (3x + 3)(x
2
12x + 35)
mcm = 2(x
2
12x + 35)(2x
2
7x 15)
Resp: p
2
(x) = (2x + 3)(x 5)(2x 10).
5.11. Fracciones algebraicas
Si no se indica explcitamente la regin en la que se estudia cierta igualdad, entonces sta se
examina en el dominio de dos expresiones que guran en los miembros primero y segundo de la
igualdad. Por ello, en adelante no se indicar explcitamente la regin en la cual se vericar una
igualdad, tomando en consideracin que sta es valida en el dominio de dos expresiones que guran
en los miembros primero y segundo de la igualdad.
En una serie de casos se necesita representar una fraccin en forma de una suma de fracciones
con denominadores ms simples. Esto puede realizarse slo en el caso cuando el polinomio en el
denominador de la fraccin se descompone en un producto de polinomios de grado menor.
Denicin 5.14 Fraccin algebraica
Se denomina fraccin algebraica una expresin racional fraccional que es el cociente de la divisin
de un polinomio por otro.
La fraccin algebraica que representa el cociente de la divisin del polinomio p(x) por otro
polinomio q(x) se escribe frecuentemente como
p(x)
q(x)
, con la particularidad de que el polinomio
p(x) se denomina numerador de la fraccin algebraica, y el polinomio q(x), denominador de la
fraccin.
A continuacin se dan algunas de las propiedades ms importantes sobre la igualdad de las
fracciones algebraicas:
1. Si se designa la fraccin algebraica
p(x)
q(x)
con el polinomio h(x), donde q(x) ,= 0, son equivalentes
las igualdades
h(x) =
p(x)
q(x)
y p(x) = h(x)q(x).
2. Dos fracciones
p(x)
q(x)
y
r(x)
h(x)
, donde q(x) ,= 0 y h(x) ,= 0, son iguales si y slo si, se verica la
igualdad
p(x)h(x) = q(x)r(x).
CAPTULO 5. POLINOMIOS 252
3. En la fraccin algebraica
p(x)
q(x)
, donde q(x) ,= 0, se verica la igualdad
p(x)
q(x)
=
[p(x)]
[q(x)]
=
p(x)
[q(x)]
=
[p(x)]
q(x)
.
4. Para cualquier polinomio h(x) ,= 0, en la fraccin algebraica
p(x)
q(x)
, se verica la igualdad
p(x)
q(x)
=
p(x)h(x)
q(x)h(x)
.
5. En la fraccin algebraica
p(x)
q(x)
, donde q(x) ,= 0, se verica la igualdad
p(x)
q(x)
= p(x)
1
q(x)
.
6. En la fraccin algebraica
1
p(x)q(x)
, donde p(x) ,= 0 y q(x) ,= 0, se verica la igualdad
1
p(x)q(x)
=
1
p(x)

1
q(x)
.
7. En las dos fracciones algebraicas
p(x)
q(x)
y
q(x)
p(x)
, donde p(x) ,= 0 y q(x) ,= 0, se verica la igualdad
p(x)
q(x)
=
1
p(x)
q(x)
.
Haciendo uso de las propiedades de adicin y multiplicacin de las expresiones algebraicas y de
las propiedades de fracciones algebraicas, resulta que se cumplen las siguientes igualdades:
p(x)
q(x)
+
r(x)
h(x)
=
p(x)h(x) +q(x)r(x)
q(x)h(x)
y
p(x)
q(x)

r(x)
h(x)
=
p(x)r(x)
q(x)h(x)
.
A menudo se requiere reducir a un denominador comn las fracciones algebraicas, es decir,
escribirlas de un modo tal, que todas estas fracciones tengan un mismo denominador. Para esto
existe el procedimiento siguiente: es menester descomponer cada denominador en factores y, a
continuacin, multiplicar el numerador y el denominador de cada fraccin por el producto de
aquellos factores de los denominadores de las fracciones restantes que no guran en el denominador
dado, lo que no los har variar, segn las propiedades de las fracciones.
Ejemplo 5.26 Simplique las expresiones:
a)
x
4
16
x
4
4x
3
+ 8x
2
16x + 16
; b)
3x
x
3
8

x
2
4
4(x
2
+ 2x + 4)
;
c)
x
2
1
(x 1)x + 1
+
12x
(1 x)x 1
.
Solucin
a) Factoramos numerador y denominador:
(x + 2)(x 2)(x
2
+ 4)
(x 2)
2
(x
2
+ 4)
=
x + 2
x 2
;
CAPTULO 5. POLINOMIOS 253
b) Factoramos numerador y denominador:
3x
x
3
8

4(x
2
+ 2x + 4)
x
2
4
=
3x
(x 2)(x
2
+ 2x + 4)

4(x
2
+ 2x + 4)
(x 2)(x + 2)
=
3x
x 2

4
(x 2)(x + 2)
=
12x
(x 2)
2
(x + 2)
.
c) Factoramos numerador y denominador:
x
2
1
(x 1)x + 1

12x
(x 1)x + 1
=
(x
2
1) 12x
(x 1)x + 1
=
x
2
12x 1
x
2
x + 1
.
Ejemplo 5.27 Simplique las expresiones:
a)
x
4
+ 2x
2
y
2
+ 9y
4
x
2
2xy + 3y
2
; b)
x
x + 2
+
x
2
+ 3x
4 x
2

x + 1
3x 6
;
c)
_
8x
2
y
12y
2
z

xyz
x
3
y
2
z
_

15x
2
y
2
z
3xyz
.
Solucin
a) Factoramos numerador y denominador:
(x
2
+ 2xy + 3y
2
)(x
2
2xy + 3y
2
)
x
2
2xy + 3y
2
= x
2
+ 2xy + 3y
2
;
b) Factoramos numerador y denominador:
x
x + 2
+
x(x + 3)
(2 x)(2 +x)

x + 1
3(x 2)
=
3x(x 2) 3x(x + 3) (x + 1)(x + 2)
3(x + 2)(x 2)
=
x
2
+ 18x + 2
3(x + 2)(2 x)
;
c) Factoramos numerador y denominador:
8x
2
y
12y
2
z

x
3
y
2
z
xyz

3xyz
15x
2
y
2
z
=
2x
2
3yz
x
2
y
1
5xy
=
2x
3
15yz
.
5.12. Tarea
1. Simplique las expresiones:
a)
x
x + 1

x 2
x +x
2
+
1
x
;
b)
1 x
x + 1
+
x + 1
1 x

x
2
+ 1
x
2
1
;
c)
x 2
x
2
+ 6x + 9

x + 3
x
2
41
;
d)
5x
2x 6
+
x 3
x + 3

x
2
6
x
2
9
;
CAPTULO 5. POLINOMIOS 254
e)
x
2
4x + 4
3x
:
x 2
x
2
+ 6x
;
f )
3x
x
2
4

2
x 2

x 3
x
2
+ 4x + 4
;
g)
x
2
+x + 1
x
3
x
2
+x 1
:
x + 1
x 1
;
h)
3xy
x y

x
2
y
2
6x
2
y

x
x +y
;
i)
3x
2
5x + 2
x
3
+ 1

x
2
+ 5x + 4
2x
2
3x + 1
;
j)
x 2y
y
+
y + 3x
x
3;
k)
2x
x 3y

3y
x + 3y

x
2
+ 3xy + 18y
2
x
2
9y
2
;
l)
bx b
x + 1
+
3bx
x 1
+
3bx
2
+bx + 2b
1 x
2
;
m)
_
x +y
x y

x y
x +y
_

x
2
y
2
2xy
;
n)
_
1
x y
x +y
_
:
_
x y
x +y

x +y
x y
_
.
2. Simplique las expresiones:
a)
[3 x[
[2x + 1[ +[x 2[ 6
, si 0 < x < 2;
b)
[

2 + 1[ +[1

2[
[x
2
x + 1[ +x x
2
+[

3 1[
;
c)
[[3x 10[ [2x 7[ 3[
3x 18
, si 4 < x < 5.
Resp: a) -1; b) 2
_
2
3
; c)
1
3
.
3. Simplique las expresiones:
a)
_
2
2x y
+
6y
y
2
4x
2

4
2x +y
_

x
2
4x
2
y
2
;
b)
_
x 2
x
3
+ 1
+
1
x
3
x
2
+x
_

x
3
x
x
2
+ 1
+
2
x
3
+x
2
+x + 1
;
c)
_
_
x +y
x y
_
2
+ 4
_

_
_
x y
x +y
_
2
+ 4
_

x
3
+y
3
x
3
y
3
;
d)
y
2
100
x
2
y
2

y + 10
x y
+
x y
x
2
+ 2xy +y
2

y
2
+xy
y
2
xy
;
e)
_
x
2
5x + 6
x
2
+ 5x + 4

x
2
4x + 3
2x
2
+ 3x + 1
_

x
2
+ 3x 4
2x
2
3x 2
;
f )
_
x
2
4x 45
x
2
14x 15

x
2
12x 45
x
2
6x 27
_

_
y
2
4
y
2
121

y + 11
y + 2
_
;
g)
_
2
2x 1
+
6
1 4x
2

4
2x + 1
_

_
1
4x
2
+ 1
4x
2
1
_
;
h)
_
x
2
2x + 4
4x
2
1

2x
2
x
x
3
+ 8

x + 2
2x
2
+x
_

4
x
2
+ 2x

x + 4
3 6x
;
i)
_
z
3
8
z +x

_
z 2
4z

8z
3
z
2
+xz
__

z
2
+ 2z + 4
2z(x z)
;
j)
_
1
x +y

x +y
x
2
xy +y
2
_

x
6
y
6
x
2
y
2
+
x
3
y
3
xy
;
k)
x
y
_
y
x

x
y
_
y
2
x
2

x
y
_
y
3
x
3

y
4
x
4
___
;
l)
6xy 14y
x 2

x
2
4
4x
2
+x 14

4x 7
4x
2

3x
2
x 14
2x
2
+ 4x
;
m)
_
x +y
x y

x y
x +y
_

_
x +y
x y
+
x y
x +y
_
;
CAPTULO 5. POLINOMIOS 255
n)
(x z)(x u)
(y z)(y u)
+
(x u)(x y)
(z u)(z y)
+
(x y)(x z)
(u y)(u z)
;
o)
x
6
y
6
x
2
y
2

x
6
+y
6
x
2
+y
2

x
4
y
4
(x
2
+y
2
)(x y)
;
p)
x
2
(x y)(x z)
+
y
2
(y z)(y x)
+
z
2
(z x)(z y)
;
q)
x
2
2x + 1
x 3

_
(x + 2)
2
x
2
4x
2
4

3
x
2
x
_
;
r)
_
x
2
+y
2
+xy
x
3
y
3

a
3
b 2a
2
b + 4ab
a
3
+ 8
_

x
2
2x + 1
3x
3
+ 7x 10
;
s)
1
x
2
+ 3x + 2

3
(x + 1)(x
2
+ 5x + 6)
;
t)
1
x
2
+x
+
1
x
2
+ 3x + 2
+
1
x2 + 5x + 6
+
1
x
2
+ 7x + 12
;
u)
6x
3
+ 48x
2
x
3
+ 64
+
x
2
4
x + 4

3x
2
x
2
4x + 16
;
v)
1
x
2
+ 4x + 4

4
x
4
+ 4x
3
+ 4x
2
+
4
x
3
+ 2x
2
;
w)
x
2
2x + 1
x 3

_
(x + 2)
2
x
2
x
2
1

x
x
2
x
_
;
x)
_
1
x
+
1
y
_
_
(x y)
2
+xy

+
_
1
x

1
y
_
_
(x +y)
2
xy

;
y)
x
2
+x 2
x
n+1
3x
n

_
(x + 2)
2
x
2
4x
2
4

3
x
2
x
_
;
z)
(c b)(x c)
(a b)(a c)
+
(x c)(x a)
(b c)(b a)
+
(x a)(x b)
(c a)(c b)
.
4. Simplique las expresiones:
a)
2x 1
2x + 1

2x + 1
2x 1
2x
4x
2
1

1
2x 1
;
b)
2
x 2

x 2
2
1
x + 2
+
4
x
2
4
;
c)
1
x + 1
+ 2
1
x
2
1
4
;
d)
(x y)
2
(x +y)
2
1
x y
x +y
1
;
e)
x +a
x a
+
x a
x +a
x
2
+a
2
x
2
a
2
+
x
2
a
2
x2 +a
2
;
f )
1
x 1

1
x + 1
x +
1
x + 1
x
2
1

1
x
2
+ 1
;
g)
x
x +y

xy 1
y
2
x
2
x
2
1
(y x)
2

_
y 1
x 1
1
_;
h)
x
x
2
4y
2
+
x 2y
x
x
2
4y
2
x
+
x
x 2y
;
i)
x
2
1
1
x 1
1
x
2
1
(x 1)
;
CAPTULO 5. POLINOMIOS 256
j)
x
2
y
4

y
2
x
4
(x
2
+y
2
)
2
x
2
y
2
x
2
y
2
;
k)
1
x
2
2x 2

1
x
2
+ 2x + 2
_
1
x
2

x
2
+ 4
x
4
+ 4
_

1
x
4
4(x + 1)
2
;
l)
6x(x + 1) 3x
2
(x + 1)
2
2x(x + 1) (x + 1)
2
x
2
.
5. Simplique las expresiones:
a)
x
2
x 20
x
2
+ 5x + 4

x
2
x
x
2
+x 2

_
x
2
x
2
+ 3x + 2

x
2
2x 15
x + 3
_
;
b)
y
2
4
x
2
1

x
3
x
y
3
+ 2y
2
+
3yz
2
5z
3
u

15z
2
u
2
9y
2
z

3z
(z + 1)
2
+
2
z + 1
;
c)
_
x
x 2
+
x
2
x
3
+ 8

x
2
2x + 4
2 x
_

8
x
2
4x + 4

x
2
+x + 6
4x + 8
;
d)
__
_
x 1
x + 1
_
2

_
x + 1
x 1
_
2
_

8x
3
+ 8x
x
3
+x
2
x 1
+
1
x + 1
_
(1 x
2
);
e)
1
2(x y)

1
2(x +y)
+
4
y
2
x
2
+
_
x
2
y
2
z
2

u
3
x
yz
3
_

y
4
xz

u
3
y
3
;
f )
_
12x
3
+ 24x
2
14x
2
7x

x
2
+ 2x
2x 1
_

_
16x
2
49
4x
2
+x 14

2x
2
x 1
2x
2
+ 5x + 2
_
;
g)
_
x
(x y)(x z)
+
y
(y z)(y x)
+
z
(z x)(z y)
_

(u + 1)
3
(u 1)
3
3u
3
+u
;
h)
_
x
2
+xy +y
2
x
2
4xy 21y
2

x
2
+ 2xy 3y
2
x
3
y
3
_

1
x 7y
+
x
2
u
2
5x
3
z
3

_
x +u
10x
4

2x 2u
xz
4
_
;
i)
y z
(x y)(x z)
+
z x
(y z)(y x)
+
x y
(z x)(z y)

2
x y

2
y z

2
z x
.
Captulo 6
Ecuaciones y sistemas de ecuaciones
6.1. Ecuaciones algebraicas
Denicin 6.1 Ecuacin
Se denomina ecuacin la igualdad que contiene una o varias letras, bajo las cuales se sobreentien-
den los nmeros incgnitos. Los valores de las incgnitas que satisfacen a la ecuacin dada, se
denominan sus soluciones.
Generalmente las incgnitas se designan con las ltimas letras del abecedario latino x, y, z, u,
v, ... Si la ecuacin contiene slo una incgnita, generalmente su solucin se denomina raz de la
ecuacin.
Resolver una ecuacin o un sistema de ecuaciones signica hallar las soluciones, es decir, todos
los valores de las incgnitas que satisfacen a la ecuacin o al sistema dado.
Denicin 6.2 Ecuacin algebraica
La ecuacin con una incgnita se denomina algebraica si ella se puede reducir de manera que su
primer miembro es un polinomio con respecto a la incgnita, y el segundo miembro sea igual a cero.
Tal tipo de ecuacin se denomina normal. El mayor exponente de la incgnita del primer miembro
de la ecuacin normal se denomina grado de la ecuacin algebraica.
Se denominan coecientes de una ecuacin los factores numricos o literales de las incgnitas,
as como el trmino independiente, es decir, el trmino que no contiene incgnitas.
Denicin 6.3 Ecuaciones equivalentes
Dos ecuaciones con iguales incgnitas se denominan equivalentes si todas las soluciones de la
primera ecuacin son tambin soluciones de la segunda e, inversamente, todas las soluciones de
la segunda ecuacin sirven tambin de soluciones de la primera o si ambas ecuaciones no tienen
solucin.
Las ecuaciones equivalentes tienen las siguientes propiedades:
1. Las ecuaciones p(x) = q(x) y p(x) q(x) = 0 son equivalentes.
2. Las ecuaciones p(x) = q(x) y p(x) +k = q(x) +k son equivalentes para cualquier nmero
real k. Es decir, si a ambos miembros de una ecuacin agregamos un mismo nmero o un
mismo polinomio con respecto a la incgnita la nueva ecuacin es equivalente a la inicial.
257
CAPTULO 6. ECUACIONES Y SISTEMAS DE ECUACIONES 258
3. Las ecuaciones p(x) = q(x) y kp(x) = kq(x) son equivalentes para todo nmero real k
distinto de cero. Es decir, Si ambos miembros de una ecuacin se multiplica o se divide por
un mismo nmero, distinto de cero, la nueva ecuacin es equivalente a la inicial.
4. Supongamos que se sabe que para cualquier nmero real k se verica la igualdad p(x) =
h(x), entonces sern equivalentes las ecuaciones p(x) = q(x) y h(x) = q(x).
Toda ecuacin de primer grado con una incgnita puede reducirse a la forma Ax + B = 0.
El primer miembro de esta ecuacin es un polinomio de primer grado respecto a x, y el segundo
miembro es igual a cero.
Analizaremos el caso en que p(x) es un polinomio de primer grado, es decir, examinemos la
ecuacin
Ax +B = 0 Ax = B x =
B
A
, A ,= 0.
Esta ecuacin elemental x =
B
A
tiene una raz nica que es el nmero
B
A
. Como la ecuacin
Ax+B = 0 equivalente a la ecuacin elemental x =
B
A
, entonces la ecuacin Ax+B = 0 tambin
tiene una sola raz, que es el nmero
B
A
. De este modo, la ecuacin de primer grado con una sola
incgnita tiene una sola raz x =
B
A
.
Si A = 0 y B ,= 0, la ecuacin no tiene races.
Si A = B = 0, la solucin de la ecuacin es un nmero cualquiera; en este caso la ecuacin se
denomina indeterminada.
Ejemplo 6.1 Resolver la ecuacin siguiente:
x 1
2
+
2x + 3
3

3x 1
6
=
1
6
.
Solucin
Realizamos la suma algebraica, igualamos a cero y resolvemos la ecuacin resultante
3(x 1) + 2(2x + 3) (3x 1)
6
=
1
6
4x + 4 = 1
4x + 3 = 0 x =
3
4
.
Ejemplo 6.2 Resolver la ecuacin siguiente:
2x 5
11

x 2
7
= 5x 17
1
2
Solucin
Realizamos la suma algebraica de la expresin
2x 5
11

x 2
7
= 5x
35
2

7(2x 5) 11(x 2)
77
=
10x 35
2
2(3x 13) = 77(10x 35)
Igualamos a cero y resolvemos la ecuacin
6x 26 = 770x 2695 764x 2669 = 0 x =
2669
764
.
CAPTULO 6. ECUACIONES Y SISTEMAS DE ECUACIONES 259
Ejemplo 6.3 Si dos poleas se unen por una correa sus velocidades angulares (revoluciones por
minuto) son inversamente proporcionales a sus dimetros, esto es, w
1
: w
2
= d
2
: d
1
. Hallar la
velocidad de una polea de 15 centmetros de dimetro unida a otra de 12 centmetros de dimetro,
que gira a 100 revoluciones por minuto.
Solucin
Sea w
1
la velocidad conocida, d
1
= 15, w
2
= 100 y d
2
= 12. La frmula dada nos da
w
1
100
=
12
15
w
1
=
12
15
100 w
1
= 80 rpm.
Ejemplo 6.4 La presin de un gas en un recipiente a temperatura constante es inversamente
proporcional al volumen. Si p = 30 cuando v = 45, hallar p cuando v = 25.
Solucin
Aqu
p =
c
v
30 =
c
45
c = 1350
As, pues p =
1350
v
. Si v = 25, entonces p =
1350
25
= 54.
Este problema tambin puede resolver de la siguiente manera: Como
p
1
=
c
v
1
y p
2
=
c
v
2

p
1
p
2
=
v
2
v
1
Tomando v
1
= 25, p
2
= 30, v
2
= 45, entonces
p
1
30
=
45
25
p = 54.
Ejemplo 6.5 Se obtienen polvos para blanquear (hipoclorito de calcio) por reaccin del cloro
y la cal apagada; 74,10 kilogramos de cal y 70,91 kilogramos de cloro producen 127 kilogramos de
polvos y 18,01 kilogramos de agua. Cuntos kilogramos de cal se necesitan para producir 1000
kilogramos de polvos de blanquear?
Solucin
Sea x el nmero de kilogramos de cal que se necesitan. Entonces
x
1000
=
74, 10
127
127x = 74100 x = 583, 46 kilogramos.
Ejemplo 6.6 La carga de seguridad de una viga horizontal apoyada en ambos extremos es
proporcional a su ancho y al cuadrado de su altura e inversamente proporcional a su longitud. Si
una viga de 5 x 10 centmetros de 2,40 metros de longitud soporta 250 kilogramos con seguridad,
Cul ser la carga limite segura para una viga de 4 x 8 centmetros y de 6 metros de larga del
mismo material?
Solucin
Como
S
1
= c
b
1
d
2
1
l
1
y S
2
= c
b
2
d
2
2
l
2
.
Entonces
S
1
S
2
=
b
1
d
2
1
l
2
b
2
d
2
2
l
1

S
1
500
=
4 8
2
8
2 4
2
20
S
1
= 1600 kilogramos.
Ejemplo 6.7 El permetro de un parque circular excede en 10 km a su dimetro. Cunto
mide el radio del parque? Cunto mide el rea del parque?
Solucin
CAPTULO 6. ECUACIONES Y SISTEMAS DE ECUACIONES 260
Denotemos por P al permetro del parque y por d su dimetro, dados en kms. Entonces, si r es el
radio del parque, se cumplen las ecuaciones
P = 2r y d = 2r
Ya que P excede a d en 10 kms, entonces P = d + 10, o equivalentemente, 2r = 2r + 10 que es
una ecuacin lineal en la variable r.
Resolvemos tal ecuacin despejando a r
2r = 2r + 10 2r 2r = 10 r(2 2) = 0 r =
5
1
De esta manera el radio del parque es r =
5
1
km, y por lo tanto, el rea A del parque ser
A = r
2
= r
_
5
1
_
2
=
25
( 1)
2
km
2
.
Ejemplo 6.8 Un sistema de refrigeracin de 20 litros se llena con un anticongelante al 25 %.
Cuntos litros deben ser extrados y reemplazados por anticongelante puro, para elevar la concen-
tracin a un 45 %?
Solucin
Denotemos por l el nmero de litros que hay que reemplazar, extrayndolos con concentracin al
25 %, y agregndolos con una concentracin al 100 %, para obtener nuevamente 20 litros al 45 %.
El proceso se describe por la igualdad
20(al 25 %) l(al 25 %) +l(al 100 %) = 20(al 45 %)
es decir
20
_
25
100
_
l
_
25
100
_
+l
_
100
100
_
= 20
_
25
100
_
20 25
100

25l
100
+
100l
100
=
20 45
100

500 25l + 100l
100
=
900
100
75l = 400 l =
400
75
= 5, 33
De esta manera, se deberan reemplazar 5,33 litros al 25 % por 5,33 litros al 100 % de anticongelante,
para tener 20 litros al 45 %.
Ejemplo 6.9 El tanque de un laboratorio de acuacultura se puede llenar con dos llaves en 50
minutos. Si una de ellas, sola, puede llenarla en una hora y cuarto, cunto tardar en llenar la
otra?
Solucin
Se entiende por T a la capacidad total del tanque, por v
1
a la velocidad de llenado de la primera
llave y por v
2
a la velocidad de llenado de la otra llave. Entonces se tiene que
v
1
=
capacidad del tanque
tiempo de llenado de la primera llave
=
T
75
v
2
=
capacidad del tanque
tiempo de llenado de la segunda llave
=
T
t
donde, el tiempo para la primer llave es de 75 minutos (hora y cuarto) y el tiempo de llenado para
la otra llave es la incgnita t.
CAPTULO 6. ECUACIONES Y SISTEMAS DE ECUACIONES 261
Por otro lado, si v es la velocidad de llenado con las dos llaves, entonces v es la suma de v
1
y v
2
,
es decir, v
1
+v
2
= v. De la denicin se tiene adems que
v =
T
50

T
50
=
T
75
+
T
t
t = 150
Esto es, la segunda llave tendra un tiempo de llenado de t = 150 minutos, es decir, de dos horas
y media.
Ejemplo 6.10 Una leona sale a cazar desde su madriguera a una velocidad promedio de 3
km/h y regresa con partes de sus presas a una velocidad promedio de 2 km /h. Si la cacera no
puede durar ms de 6 horas debido a que tiene que cuidar a sus cachorros, cunto puede alejarse
de su madriguera?
Solucin
Sea D la distancia que puede recorrer a lo ms durante su cacera. Ya que tiene que recorrer la
misma distancia a otra velocidad, se cumple que
Tiempo de ida + Tiempo de vuelta = 6horas
Pero de la denicin de velocidad promedio
Tiempo de ida =
D
Velocidad de ida
=
D
3
horas
Tiempo de vuelta =
D
Velocidad de vuelta
=
D
2
horas
lo cual implica que
D
3
+
D
2
= 6 D =
36
5
kilmetros.
Ejemplo 6.11 Se tienen dos soluciones cidas una A al 20 % de cido y la otra B al 60 % de
cido. Cunto se debera poner de cada solucin para obtener 100 ml de una solucin al 40 % de
cido?
Solucin
Sea s la cantidad de solucin A necesaria para obtener la cantidad requerida al 40 %. Entonces la
cantidad B necesaria es de 100 s. Esto es, la ecuacin que describe el problema es, en mililitros
cantidad deA(al 20 %) + cantidad deB(al 60 %) = 100(al 40 %)
es decir
s
_
20
100
_
+ (100 s)
_
60
100
_
= 100
_
40
100
_
s = 50.
De esta manera, se debern poner 50 ml de la sustancia A y 100 - 50 = 50 ml de la sustancia B.
Ejemplo 6.12 Si la ecuacin C =
5
9
(F32) representa la relacin entre las lecturas expresadas
en grados centgrados y Farhrenheit, de una temperatura, hallar a qu temperatura las dos lecturas
sern iguales.
Solucin
De la ecuacin C =
5
9
(F 32) se obtiene la relacin para F, F =
9
5
C + 32. Denamos la variable
de independencia por T en cada caso, esto es
C =
5
9
(T 32) F =
9
5
T + 32
CAPTULO 6. ECUACIONES Y SISTEMAS DE ECUACIONES 262
Entonces las temperaturas F y C coinciden, si y slo si
F = C
5
9
(T 32) =
9
5
T + 32
5
9
T
32 5
9
=
9
5
T + 32
5
9
T
9
5
T = 32
_
1 +
5
9
_

56
45
T = 32
14
9
T = 40
De esta manera, las lecturas coinciden en C = 40 = F.
Ejemplo 6.13 En un laboratorio de quesos se cuenta con 100 litros de leche con 5 % de grasa.
El nivel permitido de grasa en la leche para ser consumida en la ciudad A es de 3, 5 %. Cuntos
litros de crema pueden separarse para hacer queso con 30 % de grasa, de tal manera que la leche
mantenga el nivel de grasa permitido?
Solucin
Sea C la cantidad de crema separada para hacer queso con 30 % de grasa. Entonces se tiene que
la ecuacin siguiente dene el problema:
(100 litros al 5 %) = C(litros al 30 %) + (100 C)(litros al 3, 5 %)
Esto es, la ecuacin que resuelve al problema se plantea
100
5
100
= C
30
100
+ (100 C)
3, 5
100

500
100
=
30C
100
+
(100 C) 3, 5
100
500 = 30C + 350 3, 5C C =
150
26, 5
= 5, 660 litros
De esta forma, se han de separar 5,66 litros de crema al 30 % para que la leche sobrante, 1005, 66 =
94, 34 litros tengan 3, 5 % de grasa.
Ejemplo 6.14 Una bomba que trabaja sola, llena un estanque en 7 horas, otra lo hara en 8
horas. En cunto tiempo se llena el estanque si trabajan ambas bombas a la vez?
Solucin
Sea x el nmero de horas que tarda el estanque en llenarse (si trabajan ambas bombas a la vez)
y sea V el volumen total del estanque. Si una bomba llena el estanque en 7 horas (entonces en
una hora llenar
1
7
del estanque), luego en x horas llenar
x
7
del estanque, es decir, llenar
xV
7
(unidades de volumen). La otra bomba llena el estanque en 8 horas, entonces en x horas llenar
x
8
del estanque, es decir,
xV
8
(unidades de volumen). Como en x horas, las dos bombas juntas llenan
al estanque entonces se tiene
xV
7
+
xV
8
= V
x
7
+
x
8
= 1
de donde 15x = 56. Luego, trabajando las dos bombas a la vez, el estanque se llena en
56
15
horas.
Ejemplo 6.15 Cuatro estudiantes deciden vivir solos en un departamento y repartir en partes
iguales el arriendo mensual. Sin embargo, encuentran que si aumenta en dos el nmero de estudi-
antes, su cuota mensual se reduce en 20 dlares. Cul es el costo del arriendo?
Solucin
Sea x el costo del arriendo. Si hay cuatro estudiantes entonces la cuota mensual de cada uno es de
x
4
. Si aumentan en dos, entonces la cuota mensual ser de
x
6
. Dadas las hiptesis del problema se
tiene que
x
4
20 =
x
6
6x 480 = 4x x = 240
luego el costo del arriendo es de 240 dlares.
CAPTULO 6. ECUACIONES Y SISTEMAS DE ECUACIONES 263
Ejemplo 6.16 En una prueba de matemticas, el 12 % de los estudiantes de una clase no re-
solvi un problema, el 32 % lo resolvi con algunos errores y los 14 estudiantes restantes obtuvieron
la solucin correcta. Cuntos estudiantes haba en la clase?
Solucin
Sea x el nmero de estudiantes, entonces
12x
100
no resolvieron el problema,
32x
100
resolvieron con al-
gunos errores el problema, 14 resolvieron el problema correctamente. Luego tenemos la siguiente
ecuacin
12x
100
+
32x
100
+ 14 = x 56x = 1400 x = 25
luego haba 25 estudiantes en la clase.
Ejemplo 6.17 Una tienda de antiguedades compr dos muebles en 345 dlares y despus los
vendi y obtuvo un benecio del 40 %. Cunto pag por cada mueble si el primero dej un benecio
de 25 % y el segundo un benecio del 50 %?
Solucin
Sea x dlares lo que le ha costado el primer mueble, entonces el segundo lo compr en (345 x)
dlares. El primero dio un benecio del 25 %, luego se vendi en (x+0, 25x) dlares, es decir 1, 25x
dlares.
El segundo di un benecio del 50 %, luego se vendi en 1, 50(345 x) dlares. Por hiptesis, la
tienda compr los dos muebles en 345 dlares y obtuvo un benecio total del 40 %, luego los dos
muebles se vendieron en 1, 40 345 = 483 dlares.
As, obtenemos la ecuacin
1, 25x + 1, 50(345 x) = 483
x
4
=
69
2
x = 138
El primer mueble se vendi en 138 dlares y el segundo en 207 dlares.
Ejemplo 6.18 Los dimetros de dos cilindros son entre s como 3 : 4 y sus alturas como 5 :
6.Qu tanto por ciento del volumen del mayor de ellos es el volumen del menor?
Solucin
Sean d
1
, d
2
los dimetros de los cilindros y h
1
, h
2
sus respectivas alturas. Por hiptesis del problema
tenemos las siguientes relaciones
d
1
d
2
=
3
4
,
h
1
h
2
=
5
6
adems la frmula del volumen de un cilindro es V = r
2
h, donde r es el radio del cilindro y h la
altura; adems d = 2r donde d es el dimetro del cilindro.
Con todo lo anterior tenemos que
V =

4
d
2
h
y para cada cilindro
V
1
=

4
d
2
1
h
1
, V
2
=

4
d
2
2
h
2
pero
d
1
=
3
4
d
2
, h
1
=
5
6
h
2
luego
V
1
=

4
_
3
4
d
2
_
2
_
5
6
h
2
_
=
45
96


4
d
2
2
h
2
=
45
96
V
2
as
V
1
=
45
96
V
2
Luego V
1
es el 46, 875 % de V
2
.
CAPTULO 6. ECUACIONES Y SISTEMAS DE ECUACIONES 264
6.2. Tarea
1. Resolver las siguientes ecuaciones:
a) 2
2x + 3
3x 1
= 0;
b)
4x 1
2

3x + 1
3
=
5x
2
;
c) 3
5x
3
+
x + 3
2
= 0;
d)
3x 1
2

2x + 1
2
=
1
5
;
e)
5
2
+
x 1
x + 1
= 1;
f )
4x 5
3
+
x 6
2
= 1;
g) 1
1
4

x 6
x + 7
= 2;
h)
(x 1) (5x + 3)
3
= 2
3
4
;
i) 3
1
2
=
x
2

2x 3
3
;
j) 3x
2x + 1
2
= 1;
k)
2x
3
+
3x 4
2
= 3;
l) 1
2
5
x =
3x + 2
3
.
2. Resolver las siguientes ecuaciones:
a)
1
2
+
2x
3

2x + 1
2
+
2x + 2
5
= 0;
b)
(3x + 5) (4x + 1)
3
= x 1;
c)
(x 6) + (8x 1)
2
= 3x 1;
d)
3x 2
4

2x 1
2
= x
x + 1
3
;
e)
5x + 2
3
+
4
5
=
5x + 3
2
1;
f )
1 6x
2

3 2x
3
=
5x
4
+ 3x;
g)
[x 2[
2
+
x + 1
3
= x;
h)

2x 3
2

2x 3
2
= x + 3;
i) [3x 1[ +
3x + 1
2
= x + 2;
j) [x 2[ +[x 1[ = x 3;
k) (x 1)([x[ 1) = 0, 5;
l)
7x + 4
5
x =
[3x 5[
2
.
3. Despeje la variable indicada en cada uno de los siguientes ejercicios:
a) De la frmula para la reactancia de un condensador
X =
1
2fC
despejar la variable C.
b) De la relacin de la velocidad media de un cuerpo
V =
V
t
+V
0
2
despejar la velocidad inicial V
0
.
c) De la frmula
E
e
=
R +r
2
de la cada de tensin, despejar a la variable r.
d) Despejar a de la frmula
C =
Kab
b a
CAPTULO 6. ECUACIONES Y SISTEMAS DE ECUACIONES 265
e) Considere la relacin de la distancia recorrida de un cuerpo en cada libre
d =
1
2
at
2

1
2
a(t 1)
2
despeje la variable t.
f ) De la relacin
1
x
+
1
nx
=
1
f
despeje a la variable x.
g) La ecuacin para una polea diferencial viene dada por
W =
2PR
R r
despeje la variable R.
h) Despeje la variable n de la ecuacin
I =
E
r +
R
n
que se reere a la corriente suministrada por generadores en paralelo.
i) Despeje a la variable w de la ecuacin
wf =
_
w
k
1
_
1
k
j) De la ecuacin de dilatacin de gases
V
1
= V
0
(1 + 0, 00365t)
despeje la variable t.
4. Cunta soldadura, con 50 % de estao, y cunto metal de imprenta con 15 % de estao, es
necesario alear para obtener 80 kg. de soldadura con un 40 % de estao?
5. Un tendero calcul que su reserva de azcar durara 30 das. Como vendi 20 kilos diarios
ms de lo que esperaba, su reserva le dur solamente 24 das. De cuntos kilos dispona?
6. Un granjero compr 100 km
2
de tierra por $ 150.100. Parte de ellos le costaron a $ 500
por km
2
, y el resto a $ 1800. Hallar el nmero de km
2
comprados a cada precio.
7. El rea de un paseo de 4 mt de anchura que rodea un estanque circular es de 1.496 m
2
.
Tomando =
22
7
, hallar el dimetro del estanque.
8. Cunto acero, con un 18 % de tungsteno, debe alearse con otro acero, conteniendo un
12 % de tungsteno, para obtener 3.000 kg de acero al 14, 6 %? Hallar tambin la cantidad de
acero que debe usarse al 12 %.
CAPTULO 6. ECUACIONES Y SISTEMAS DE ECUACIONES 266
9. Cul sera la temperatura nal cuando se mezclan 20 kg de agua a 60

C con 30 kg de agua
a 10

C? En los problemas de intercambio calorco que no impliquen un cambio de estado


se verica: masa x calor especco x disminucin de temperatura en un cuerpo caliente y x
calor especco x aumento de la temperatura en un cuerpo fro.
10. Un reloj mal compensado adelanta 11 seg en 9 horas cuando se lleva verticalmente en el
bolsillo, y atrasa 28 seg en 13 horas cuando se deja en posicin horizontal. Durante cuntas
horas hay que mantenerlo en cada posicin para que no gane ni pierda durante un total de
24 horas de funcionamiento?
11. Cuntos litros de solucin anticongelante al 35 % deben aadirse a tres litros de solucin
al 80 %, para reducir su concentracin al 60 %?
12. Un trozo de alambre de 11
2
3
centmetros de largo ha de dividirse en dos partes tales que
la una sea
2
3
de la otra. Hallar la longitud de la ms corta.
13. Un tren sale de la estacin a 40 kilmetros por hora. Dos horas ms tarde parte un segundo
tren a 60 kilmetros por hora. Dnde alcanzar al primero?
14. Un tanque se vaca por dos tubos, uno de los cuales lo puede vaciar en 30 minutos y el
otro en 25 minutos. Si el tanque est lleno en sus
5
6
y ambos tubos estn abiertos, en cunto
tiempo quedar vaco?
15. A puede hacer un trabajo en 10 das. Despus de llevar 2 das trabajando, B viene a ayu-
darle y juntos acaban en 3 das. En cuntos das podra hacer el trabajo B solo?
16. La resistencia resultante de dos resistencias R
1
y R
2
conectadas en paralelo es tal que
1
R
=
1
R
1
+
1
R
2
. Hallar R si R
1
= 80 y R
2
= 240.
17. A que horas despus del medioda se vuelven a juntar por primera vez las manecillas del
reloj?
18. La reaccin de 65,4 gramos de cinc y 72,9 gramos de cido clorhdrico da 136,3 gramos
de cloruro de cinc y 2 gramos de hidrgeno. Cul es el peso de cido clorhdrico necesario
para una reaccin completa con 300 gramos de cinc y cul es el peso del hidrgeno producido?
19. Una mquina de cambiar monedas cambia billetes de un dlar en monedas de 25 y de 5
centavos. Si recibe 12 monedas, despus de introducir un billete de 1 dlar, cuntas monedas
de cada tipo recibe?
20. Un barco usa receptores de sonido encima y bajo la supercie del agua, para registrar una
explosin, que llega 5 segundos antes al receptor sumergido. El sonido viaja en el aire a unos
1050 pies/seg y, en el agua del mar, a unos 4500 pies/seg.
CAPTULO 6. ECUACIONES Y SISTEMAS DE ECUACIONES 267
a) Cunto tiempo tard cada onda sonora en llegar al barco?
b) A qu distancia del barco se produjo la explosin?
21. Dos empresas le han ofrecido empleo de vendedor. Ambos empleos son esencialmente
iguales, pero una empresa le paga solamente una comisin del 8 %, en tanto que la otra
le ofrece 51 dlares a la semana, ms una comisin del 5 por ciento. Los mejores represen-
tantes de ventas, en cualquiera de las dos empresas, rara vez tienen ventas superiores a 4000
dlares por semana. Antes de aceptar una de estas ofertas, necesita usted saber en qu punto
pagan lo mismo ambas empresas y cul de las dos paga mas a ambos lados de dicho punto.
22. La potencia de una mquina de vapor es proporcional a la presin media en el cilindro y a
la velocidad de rotacin. Si la presin media es 320 newtons por centmetros cuadrados y el
volante da 750 revoluciones por minuto la potencia es 100. Cul es la potencia si la presin
media baja a 240 newtons por centmetros cuadrados y las revoluciones a 600 revoluciones
por minuto?
23. Cunta agua hay que usar para preparar una solucin a 1:5000 de bicloruro de mercurio
con una tableta de 0,5 gramos?
24. El volumen de un cono es proporcional a la altura y al cuadrado del radio. Si el radio es
4 y la altura 6, el volumen es 32. Cul debe ser la altura si el volumen es 12 cuando el
radio es 2?
25. La ley de Newton de la gravitacin dice que la fuerza de atraccin entre dos cuerpos vara
en proporcin directa de sus masas m
1
y m
2
e inversa del cuadrado de la distancia entre ellos.
Dos cuerpos cuyos centros estn a una distancia de 5000 kilmetros se atraen con una fuerza
de 76 newtons. Cul sera la fuerza de atraccin si se triplicaran las masas y la distancia
entre los centros se doblar?
26. Si un cuerpo pesa 100 Newton sobre la supercie terrestre, cul es su peso a 3000 kilmet-
ros de la supercie? Supngase el radio de la tierra 6000 kilmetros aproximadamente.
27. Un taller de imprenta de un peridico cuente con dos maquinas dobladoras para el aco-
modamiento nal del diario vespertino, cuya circulacin es de 35000 ejemplares. La maquina
mas lenta puede doblar los peridicos a una velocidad de 6000 por hora, en tanto que la
otra los dobla a razn de 9000 por hora. Si el uso de la maquina mas lenta se retrasa media
hora, por una leve avera, Cul ser el tiempo total necesario para doblar todo el peridico?
Cunto tiempo empleara cada maquina en esta tarea?
28. Se tiene dos barras de aleacin de oro: una es de 12 quilates y la otra de 18 (el oro de
24 quilates es oro puro; el de 12 quilates corresponde a 12/24 de pureza; el de 18, a 18/24
de pureza y as sucesivamente). Cuntos gramos de cada aleacin se deben mezclar para
obtener 10 gramos de oro de 14 quilates?
CAPTULO 6. ECUACIONES Y SISTEMAS DE ECUACIONES 268
29. Un terremoto emite una onda primaria y otra secundaria. Cerca de la supercie de la tier-
ra, la onda primaria viaja aproximadamente a 5 millas/seg, en tanto que la secundaria viaja
a 3 millas/seg. Por el retraso entre ambas ondas, al llegar a una estacin dada, es posible
calcular la distancia a la que ocurri el temblor. (El epicentro se puede localizar cuando se
obtienen las medidas de dicha distancia, en tres o mas estaciones.) Supongamos que una
estacin midi una diferencia de 16 seg entre la llegada de ambas ondas. Cunto tiempo
viajo cada onda? Y a que distancia de la estacin tuvo lugar el terremoto?
30. Si la suma de los dos ngulos agudos de un triangulo rectngulo corresponde a 90

y su
diferencia es 14

, encuentre ambos ngulos.


31. Encuentre las dimensiones de un rectngulo con 124 centmetros de permetro, si su longi-
tud es 25 % ms grande que su anchura.
32. Un qumico tiene dos concentraciones de acido clorhdrico: una en solucin al 50 % y la
otra al 80 %. Qu cantidad de cada una deber mezclar para obtener 100 ml de una solucin
al 68 %.
6.3. Sistemas de ecuaciones lineales de dos ecuaciones en dos
incgnitas
Denicin 6.4 Ecuacin lineal
Se denomina ecuacin lineal con dos incgnitas, la ecuacin del tipo
Ax +By = C.
Se aprecia fcilmente que esta ecuacin tiene innitas soluciones, puesto que a una de las in-
cgnitas, se le puede dar valores arbitrarios, y el valor de la incgnita correspondiente a ste se
halla de la ecuacin. Las coordenadas de cualquier punto de una recta son las soluciones de la
ecuacin; pero, teniendo en cuenta que los puntos de una recta son innitos, tambin las soluciones
son innitas.
El conjunto de dos ecuaciones
_
A
1
x +B
1
y = C
1
A
2
x +B
2
y = C
2
forma un sistema de ecuaciones lineales con dos incgnitas. El par de nmeros x
0
, y
0
, que satisfacen
a cada ecuacin del sistema se denomina su solucin.
Supongamos dado un sistema lineal con coecientes literales
_
A
1
x +B
1
y = C
1
A
2
x +B
2
y = C
2
donde B
1
,= 0, se requiere hallar su solucin. De la primera ecuacin expresamos y por x:
y =
C
1
A
1
x
B
1
CAPTULO 6. ECUACIONES Y SISTEMAS DE ECUACIONES 269
Este valor de y lo sustituimos en la segunda ecuacin:
A
2
x +B
2
C
1
A
1
x
B
1
= C
2
Obtenemos una ecuacin con una incgnita, la que se reduce a la forma
A
2
B
1
x A
1
B
2
x = B
1
C
2
B
2
C
1
(A
2
B
1
A
1
B
2
)x = B
1
C
2
B
2
C
1
Si el coeciente de x, es decir, la expresin A
2
B
1
A
1
B
2
, es distinto de cero, ambos miembros de
la igualdad anterior se pueden dividir por l; as, obtenemos:
x =
B
2
C
1
B
1
C
2
A
1
B
2
A
2
B
1
Despus de sustituir este valor de x en la primera igualdad, hallamos:
y =
A
1
C
2
A
2
C
1
A
1
B
2
A
2
B
1
El sistema dado tiene una sola solucin si A
1
B
2
A
2
B
1
,= 0, adems, los valores de las incgnitas
se calculan por las frmulas
_

_
x =
B
2
C
1
B
1
C
2
A
1
B
2
A
2
B
1
y =
A
1
C
2
A
2
C
1
A
1
B
2
A
2
B
1
1. Si A
1
B
2
A
2
B
1
= 0, de donde los coecientes de las incgnitas son proporcionales.
2. Si B
2
C
1
B
1
C
2
= 0, el sistema es indeterminado, es decir, tiene innitas soluciones.
3. Si A
1
B
2
A
2
B
1
= 0 y B
2
C
1
B
1
C
2
,= 0, el sistema es incompatible y no tiene soluciones.
A cada uno de los tres casos examinados se le puede dar una interpretacin geomtrica,
partiendo de que en el sistema de coordenadas rectangulares a cada ecuacin lineal, con dos
incgnitas corresponde una recta.
a) Si A
1
B
2
A
2
B
1
,= 0, dos rectas, representadas por las ecuaciones del sistema, se
cortan en un punto; las coordenadas del punto de interseccin representan precisamente
la solucin del sistema.
b) Si A
1
B
2
A
2
B
1
= 0 y B
2
C
1
B
1
C
2
=, las dos rectas correspondientes a las ecuaciones
se confunden en una recta comn; dado que stas tienen innitos puntos comunes, en
consecuencia, el sistema tambin tiene innitas soluciones.
c) Si A
1
B
2
A
2
B
1
= 0 y B
2
C
1
B
1
C
2
,= 0, las rectas correspondientes a las ecuaciones
del sistema, son paralelas, es decir, no tienen ningn punto comn, por lo cual el sistema
no tiene soluciones.
Ejemplo 6.19 Resuelva el sistema de ecuaciones:
_
9x + 3y 2 = 0
10x + 6y 4 = 0
CAPTULO 6. ECUACIONES Y SISTEMAS DE ECUACIONES 270
Solucin
Aplicando las frmulas deducidas anteriormente, obtenemos
x =

2 3
4 6

9 3
10 6

=
12 12
54 30
=
0
24
= 0, y =

9 2
10 4

9 3
10 6

=
36 20
54 30
=
16
24
=
2
3
.
Ejemplo 6.20 Un hombre puede remar aguas abajo 6 kilmetros en 1 hora y regresar en 2
horas. Hallar su velocidad en agua tranquila y la velocidad de la corriente.
Solucin
Sean
x : velocidad en agua tranquila en kilmetros por hora.
y : velocidad del ro en kilmetros por hora.
x +y : velocidad con la corriente.
x y : velocidad en contra.
De esta manera obtenemos
_
x +y = 6
x y = 3
Resolviendo este sistema, obtenemos:
x =

6 1
3 1

1 1
1 1

=
9
2
=
9
2
, y =

1 6
1 3

1 1
1 1

=
3
2
=
3
2
La velocidad en agua tranquila es
9
2
kilmetros por hora y la velocidad de la corriente es
3
2
kilmetros
por hora.
Ejemplo 6.21 Un cierto nmero de estudiantes deben acomodarse en una residencia. Si se
ubicaran dos estudiantes por habitacin entonces quedaran 2 estudiantes sin pieza. Si se ubicaran
3 estudiantes por habitacin entonces sobraran 2 piezas. Cuntas habitaciones disponibles hay
en la residencia y cuntos estudiantes deben acomodarse en ella?
Solucin
Sean
x : nmero de estudiantes.
y : nmero de habitaciones.
Entonces podemos representar el problema mediante las ecuaciones
_
x 2y = 2
x 3(y 2) = 0
Resolviendo este sistema, obtenemos:
x =

2 6
2 3

1 1
1 3

=
18
1
= 18, y =

1 1
2 6

1 1
2 3

=
8
1
= 8
Por lo tanto hay 8 habitaciones y 18 estudiantes.
CAPTULO 6. ECUACIONES Y SISTEMAS DE ECUACIONES 271
Ejemplo 6.22 Resuelva el sistema de ecuaciones:
_
5x 3 = 2y
10x + 4y = 6
Solucin
De igual forma que en los literales anteriores, obtenemos
x =

3 2
6 4

5 2
10 4

=
12 12
20 20
=
0
0
, y =

5 3
10 6

5 2
10 4

30 30
20 20
=
0
0
como los valores de x y y indican una indeterminacin
0
0
, entonces el sistema tiene ms de una
solucin. Para encontrar la solucin general a este tipo de sistemas, hacemos y = t, entonces por
la primera ecuacin obtenemos x =
32t
5
, para cualquier valor de t.
Ejemplo 6.23 Resuelva el sistema de ecuaciones:
_
7x = 8 7y
16y + 16x 8 = 0
Solucin
Haciendo uso de las frmulas deducidas anteriormente, obtenemos
x =

8 7
8 16

7 7
16 16

=
128 56
112 112
=
72
0
.
como tenemos la divisin para cero, entonces el sistema de ecuaciones no tiene solucin.
Ejemplo 6.24 Determine de ser posible los valores de a y b, para que el sistema de ecuaciones
lineales tenga solucin nica, tenga ms de una solucin, no tenga solucin y encuentre la solucin
general del sistema en trminos de a y b:
a)
_

_
1
3
x by = a
ax +
1
2
y = 2
; b)
_

_
1
a
x y =
1
2
x +
1
b
y =
a
b
; c)
_

_
2
3
x +by =
1
2
1
2
ax +y =
1
3
.
Solucin
a) Encontramos la solucin general en trminos de a y b:
x =

a b
2
1
2

1
3
b
a
1
2

=
1
2
a + 2b
1
6
+ab
=
3(a + 4b)
2(1 + 6ab)
, y =

1
3
a
a 2

1
3
b
a
1
2

2
3
a
2
1
6
+ab
=
2(2 3a
2
)
1 + 6ab
Para que el sistema de ecuaciones tenga solucin nica, hacemos
1 + 6ab ,= 0 ab ,=
1
6
CAPTULO 6. ECUACIONES Y SISTEMAS DE ECUACIONES 272
Para que el sistema de ecuaciones tenga ms de una solucin, hacemos
_

_
a + 4b = 0
2 3a
2
= 0
1 + 6ab = 0

_
_
_
a =
_
2
3
b =
1
2

6
Para que el sistema de ecuaciones no tenga solucin, hacemos
1 + 6ab = 0 ab =
1
6
.
b) Encontramos la solucin general en trminos de a y b:
x =

1
2
1
a
b
1
b

1
a
1
1
1
b

=
1
2b
+
a
b
1
ab
+ 1
=
a(1 + 2a)
2(1 +ab)
, y =

1
a
1
2
1
a
b

1
a
1
1
1
b

=
a
ab

1
2
1
ab
+ 1
=
a(2 b)
2(1 +ab)
Para que el sistema de ecuaciones tenga solucin nica, hacemos
1 +ab ,= 0 ab ,= 1.
Para que el sistema de ecuaciones tenga ms de una solucin, hacemos
_

_
a(1 + 2a) = 0
a(2 b) = 0
1 +ab = 0

_
a =
1
2
b = 2
Para que el sistema de ecuaciones no tenga solucin, hacemos
1 +ab = 0 ab = 1.
c) Encontramos la solucin general en trminos de a y b:
x =

1
2
b
1
3
1

2
3
b
a
2
1

=
1
2

b
3
2
3

ab
2
=
3 2b
4 3ab
, y =

2
3
1
2
a
2
1
3

2
3
b
a
2
1

=
2
9

a
4
2
3

ab
2
=
8 9a
6(4 3ab)
Para que el sistema de ecuaciones tenga solucin nica, hacemos
4 3ab ,= 0 ab ,=
4
3
.
Para que el sistema de ecuaciones tenga ms de una solucin, hacemos
_

_
3 2b = 0
8 9a = 0
4 3ab = 0

_
a =
8
9
b =
3
2
.
Para que el sistema de ecuaciones no tenga solucin, hacemos
4 3ab = 0 ab =
4
3
.
6.4. Tarea
1. Resolver los siguientes sistemas de ecuaciones lineales:
CAPTULO 6. ECUACIONES Y SISTEMAS DE ECUACIONES 273
a)
_
5x 2y = 1
3x +y = 2
b)
_
4x + 5y = 3
5x + 2y = 1
c)
_
6x + 2y = 1
8x + 3y = 5
d)
_
2x + 5y = 9
3x 8y = 1
e)
_
x + 5y = 4
3x + 8y = 1
f )
_
7x + 5y = 5
3x + 4y = 9
g)
_
6x + 7y = 9
5x + 4y = 8
h)
_
3x + 7y = 3
5x 6y = 1
i)
_
0, 33x +
1
3
y = 1
1
2
x + 0, 5y = 2
j)
_
1
1
2
x +
1
3
y =
1
2
2
1
3
x +
1
2
y =
1
3
k)
_
3
2
x +
3
4
y =
3
2
1
3
x +
2
3
y =
1
3
l)
_
2x 0, 75y = 1
1
2
0, 52x + 3y =
1
3
m)
_
1
4
x + 0, 31y =
1
5
1
5
x +
1
2
y = 0, 3
n)
_
0, 55x +
1
5
y = 0, 1
1
4
x +
1
3
y = 0, 22
o)
_
5
3
x
2
3
y =
1
3
2
3
x +
4
3
y = 1
p)
_
5
2
x +
3
2
y =
1
2
1
3
x + 3y = 5
2. Resolver los siguientes sistemas de ecuaciones lineales:
a)
_
1, 25x + 1, 35y = 1, 25
0, 25x 0, 35y = 0, 25
b)
_
0, 21x + 0, 31y = 0, 41
0, 52x 0, 62y = 0, 72
c)
_
0, 1x + 0, 2y = 0, 3
x 0, 3y = 0, 4
d)
_
0, 11x + 0, 12y = 0, 13
0, 12x 0, 13y = 0, 14
e)
_
1, 45x + 1, 25y = 0, 5
2, 25x 0, 25y = 1, 5
f )
_
2, 03x + 2, 04y = 2
2, 04x 2y = 1
g)
_
3, 12x + 3, 14y = 3
2, 11x + 2, 13y = 1
h)
_
2, 15x + 2, 25y = 2, 5
3, 15x + 3, 25y = 3, 5
i)
_
0, 7x + 0, 6y = 0, 5
0, 4x + 0, 5y = 0, 3
j)
_
1, 75x + 1, 25y = 0, 5
1, 4x + 1, 5y = 1, 3
3. Resolver los siguientes sistemas de ecuaciones lineales:
a)
_

_
x 2y 1
3
+
0, 5x y + 0, 25
2
=
1
3
x +y
2

1 x +y
3
=
1
2
b)
_

_
3x +y
2
+
x 3y
3
= 2
x + 3y
3
+
3x y
2
= 1
c)
_

_
x +y 1
2

2x y + 3
3
= 2
x + 3y 2
3
+
x +y 2
2
= 3
4. Resolver los siguientes sistemas de ecuaciones lineales:
a)
_
4x
1
3
y = 0
2x +
1
2
y = 1, 3
b)
_
1
1
5
x + 1
1
3
y = 1
1
1
2
x + 1
2
3
y = 2
c)
_
x+y
2

xy
2
= 1
xy
3
+
x+y
3
= 2
d)
_
2
1
2
x 1
1
2
y = 2
1
1
3
x + 2
1
3
y = 2
e)
_
x 3y =
1
3
2x + 4y =
1
2
f )
_
0, 25x 0, 35y = 1
0, 35x + 0, 45y = 1
g)
_
31x + 21y = 20
21x 31y = 30
h)
_
15x 12y = 10
16x + 13y = 1
CAPTULO 6. ECUACIONES Y SISTEMAS DE ECUACIONES 274
5. Determine de ser posible los valores de a y b, para que el sistema de ecuaciones lineales
tenga solucin nica, tenga ms de una solucin, no tenga solucin y encuentre la solucin
general del sistema en trminos de a y b:
a)
_
ax +y = 1
x (a + 1)y = 4
b)
_
x + 3ay = 2
ax 3ay = 1
c)
_
(2a 1)x +ay = 2
ax y = a
d)
_
ax + (3a 2)y = 1
(a + 1)x y = 3
e)
_
ax y = 3a + 1
(3a + 2)x y = a
f )
_
ax + 3ay = 3
2x + 6ay = 2a
g)
_
5x + (7a 2)y = 3
5ax y = 2a
h)
_
(a 1)x + 2ay = 4
3ax (a + 1)y = 2a
i)
_
x (a 2)y = 3
ax +y = 2
j)
_
x +ay = 3
2x by = 2
k)
_
(a +b)x y = 1
ax +by = 2
l)
_
(a 1)x +y = 3
x + (a + 1)y = 1
m)
_
ax +y = a
x +ay = 1
n)
_
x ay = 1
ax y = 2
o)
_
(2a + 1)x y = a
x + (a + 1)y = 0
p)
_
(a b)x +y = 1
ax y = 2
q)
_
ax +by = 3
bx + (a + 1)y = 2
r)
_
x +ay = b
ax y = 1
s)
_
bx + (a 1)y = 2
ax +y = b
t)
_
ax +y = 1
bx +y = 2
u)
_
ax by = 2
x (b + 1)y = 1
v)
_
2ax 3y = 3
3x + 2ay = 1
w)
_
(a + 1)x +by = 2
x +ay = b
x)
_
0, 3x +by = 2, 5
ax 3y = b
6. A y B estn a 30 kilmetros uno del otro. Si parten al mismo tiempo y caminan en la
misma direccin A alcanza a B en 60 horas. Si marcha uno hacia el otro se encuentran a las
5 horas. Cules son las velocidades?
7. Una aleacin contiene tres veces ms cobre que plata y otra contiene cinco veces ms plata
que cobre. Qu cantidad de cada aleacin se ha de utilizar para hacer 14 kilogramos con el
doble de cobre que de plata?
8. A y B trabajando juntos pueden realizar una tarea en 4 das y 4/5 de da; B y C jun-
tos la haran en 4 das y A y C juntos en 3 das y 3/7. Cuntos das gastaran los tres juntos?
9. Si un lote se agranda hacindolo 10 metros ms largo y 5 metros ms ancho, su rea au-
menta en 1050 metros cuadrados. Si su longitud se rebaja en 5 metros y su anchura en 10
metros, el rea disminuye en 1050 metros cuadrados. Hallar las dimensiones del lote.
10. Dos trenes de 400 metros de largo cada uno corren sobre vas paralelas. Si van en la misma
direccin el uno pasa al otro en 20 segundos; pero si van en direcciones contrarias se pasan
en 5 segundos. Hallar la velocidad de cada tren.
11. Un cierto nmero de personas tiene que pagar a partes iguales un total de 72000 dlares.
Si hubiera tres personas menos entonces cada una debera contribuir con 4000 dlares ms.
CAPTULO 6. ECUACIONES Y SISTEMAS DE ECUACIONES 275
Cuntas personas son?
12. Sesenta ejemplares del primer volumen de un libro y 75 ejemplares del segundo volumen
cuestan un total de 405000 dlares. Sin embargo, un descuento del 15 % en el primer volumen
y de un 10 % en el segundo volumen reducir el precio total a 355500 dlares. Determine el
precio de cada volumen.
13. La hipotenusa de un tringulo rectngulo mide 3

5 metros. Determine los catetos sabien-


do que cuando se aumenta uno en un
400
3
% y el otro en un
50
3
% la suma de sus longitudes
vale 14 metros.
14. Dos sacos contienen 140 kilogramos de harina. Si sacamos el 12, 5 % de la harina del primer
saco y la hechamos en el segundo, ambos tendrn la misma cantidad. Cuntos kilos de ha-
rina tiene cada saco?
15. Dos fbricas A y B se comprometen a servir un pedido en 12 das. Despus de dos das la
fbrica A cierra para hacer unas reparaciones, mientras que la fbrica B sigue funcionandop
normalmente. Sabiendo que B tiene un rendimiento del
200
3
% del rendimiento de A, deter-
mine en cuntos das se servir el pedido.
16. Un lingote de aleacin cobre-zinc que pesa 24 kilogramos se sumerge en agua y pierde
26
9
kilogramos en peso. Determine la cantidad de cobre y de zinc en la aleacin, sabiendo que en
el agua el cobre pierde
100
9
% y el zinc
100
7
% de su peso.
17. Encuentre un nmero de dos cifras sabiendo que el cociente que se obtiene al dividirle por
el producto de sus dgitos es igual a
8
3
y, adems, que la diferencia que se obtiene entre el
nmero buscado y el nmero que se obtiene al invertir el orden de los dos dgitos que lo
forman es 18.
18. Determine un nmero de dos cifras sabiendo que el nmero de unidades excede en dos al
nmero de decenas y que el producto del nmero deseado por la suma de sus dgitos es 144.
19. Dos trenes salen al mismo tiempo de las estaciones A y B separadas 600 km y viajan uno
al encuentro del otro. El primer tren llega a B tres horas antes de que el segundo llegue a
A. El primer tren recorre 250 km en el mismo tiempo en que el segundo recorre 200 km.
Encuentre la velocidad de cada tren.
20. Un viajero que va a tomar su tren ha cubierto 3,5 km en una hora y se da cuenta que a
esa velocidad llegar una hora tarde. Entonces recorre el resto de la distancia a la velocidad
de 5 km/hr y llega 30 minutos antes de que salga el tren. Determine la distancia que tena
que recorrer.
CAPTULO 6. ECUACIONES Y SISTEMAS DE ECUACIONES 276
21. La distancia entre A y B por autopista es 10 km. Un ciclista sale de A en direccin a B a
una velocidad constante. Un coche sale de A 15 minutos ms tarde en la misma direccin. Al
cabo de 10 minutos alcanza al ciclista y continua hasta B, donde da la vuelta y al cabo de 50
minutos despus de haber abandonado A encuentra por segunda vez al ciclista. Determine
las velocidades del ciclista y del coche.
22. Un tren correo sale de la estacin A a las 5 de la madrugada en direccin de la estacin B
a 1080 km de distancia. A las 8 de la maana sale de B un tren en direccin de A y viaja
15 km/hr ms aprisa que el tren correo. Cundo se encontrarn sabiendo que el punto de
encuentro es el punto medio entre A y B?
23. A dista 78 km de B. Un ciclista sale de A en direccin de B. Una hora despus, otro ciclista
sale de B en direccin a A y va 4 km/hr ms rpido que el primero. Se encuentran a 36 km
de B. Cunto hace que ha salido cada uno y cules son sus velocidades?
24. Una bandeja rectangular de 20 cm x 90 cm x 25 cm (paraleleppedo rectangular) se usar
para hacer negativos fotogrcos. El agua llega a travs de un tubo de goma y sale a travs de
otro para mantener al agua en agitacin. Se necesitan 5 minutos menos para vaciar la bandeja
a travs del segundo tubo que en llenarla mediante el primero, con el segundo cerrado. Si se
abren ambos tubos, la bandeja completa se vaciar en una hora. Encuentre la cantidad de
agua que deja pasar cada tubo en un minuto.
25. Dos trabajadores, uno de los cuales empieza a trabajar uno y medio das despus que el
otro, pueden completar un trabajo en 7 das. Si cada uno de ellos hiciera el trabajo indi-
vidualmente, el primero habra necesitado 3 das ms que el segundo que empez despus.
Cuntos das tardar cada obrero en realizar el trabajo individualmente?
26. Dos mquinas perforadoras de tneles trabajando en los dos extremos de un tnel tienen
que completar la perforacin en 60 das. Si la primera mquina hace el 30 % del trabajo
asignado, y la segunda el
80
3
%, entonces ambas perforarn 60 metros de tnel. Si la primera
mquina ha realizado
2
3
del trabajo asignado a la segunda, y la segunda 0,3 del trabajo
asignado a la primera, entonces la primera mquina necesitara 6 das ms que la segunda.
Determine cuntos metros de tnel perfora cada mquina por da.
27. Dos cuadrillas de ferroviarios trabajando conjuntamente terminan una reparacin de la sec-
cin de una va en 6 das. Para hacert el 40 % del trabajo, la primera cuadrilla sola necesitara
dos das ms de lo que la segunda cuadrilla sola necesitar, para realizar
40
3
% del trabajo
completo. Determine cuntos das tardara cada cuadrilla en reparar la seccin completa por
separado.
28. Dos obreros juntos completan una cierta tarea en 8 horas. Trabajando individualmente, el
primer obrero podra hacer el trabajo 12 horas ms aprisa que lo podra hacer el segundo.
Cuntas horas tardara cada obrero en hacer individualmente el trabajo?
CAPTULO 6. ECUACIONES Y SISTEMAS DE ECUACIONES 277
29. Dos tuberas tardan 6 horas en llenar una piscina. Una sola la llenara 5 horas ms deprisa
que la otra. Cunto tardar cada tubera sola en llenar la piscina?
30. Dos ruedas estn girando accionadas por una correa sin n; la ms pequea da 400 rev-
oluciones por minuto ms que la grande. Esta da 5 revoluciones en un lapso de tiempo que
es un segundo mayor que el necesario para que la ms pequea de 5 revoluciones. Cuntas
revoluciones por minuto da cada una?
31. Dos motores de combustin interna se sometieron a un ensayo de rendimiento y se encontr
que uno de ellos haba consumido 600 gramos de combustible mientras que el otro, que haba
funcionado 2 horas menos, consumi 384 gramos. Si el primer motor consumiera la misma
cantidad de combustible por hora que el segundo y el segundo lo mismo que el primero,
entonces ambos motores consumiran la misma cantidad de combustible durante el mismo
perodo de funcionamiento que antes. Cunto combustible consume por hora cada motor?
32. Cuatro gras de puerto idnticas se usan para cargar un barco. Cuando llevan 2 horas
trabajando, se ponen a trabajar con ellas 2 gras ms de menor capacidad, con lo que se
completa la carga en 3 horas. Si hubieran empezado a trabajar todas juntas, se habra com-
pletado la carga en 4,5 horas. Determine el tiempo (en horas) necesario para que realice el
trabajo completo una gra sola de las ms potentes y una gra sola de las de menor potencia.
33. Un estanque con cierta cantidad de agua tiene forma de cono invertido. Al agregarle 10
litros, el nivel de agua sube en un 20 %. Si la base del cono fuese reducida en 40 %, mante-
niendo la misma altura resultara un cono que estara lleno con la cantidad de agua inicial.
Sabiendo que la altura del estanque es 50 cm. Calcular el radio inicial y el volumen de agua
contenido en un comienzo.
6.5. Sistemas de ecuaciones lineales de ms de 2 variables
Denicin 6.5 Ecuacin lineal
Una ecuacin lineal sobre R en n variables es una expresin de la forma a
1
x
1
+a
2
x
2
+...+a
n
x
n
= b
donde los a
i
, b son nmeros conocidos y los x
i
son variables. Los a
i
se denominan coecientes de
los x
i
respectivos, y b es el trmino independiente de la ecuacin.
Una solucin de la ecuacin lineal a
1
x
1
+ a
2
x
2
+ ... + a
n
x
n
= b es un conjunto ordenado de n
valores k
1
, k
2
, ..., k
n
tales que a
1
k
1
+a
2
k
2
+... +a
n
k
n
= b.
Denicin 6.6 Sistema de ecuaciones lineales
Un sistema de m ecuaciones lineales en n variables, es una expresin de la forma
_

_
a
11
k
1
+a
12
k
2
+... +a
1n
k
n
= b
1
a
21
k
1
+a
22
k
2
+... +a
2n
k
n
= b
2
...
a
m1
k
1
+a
m2
k
2
+... +a
mn
k
n
= b
m
donde los a
ij
y los b
i
pertenecen a los nmeros reales. El primer subndice en los coecientes indica
el nmero de la ecuacin, y el segundo, el nmero de la variable. Para un sistema de m ecuaciones
CAPTULO 6. ECUACIONES Y SISTEMAS DE ECUACIONES 278
lineales en n variables x
i
, i = 1, 2, ..., n, el conjunto solucin S es el subconjunto de R
n
denido
por S = S
1
S
2
... S
m
donde S
i
es el conjunto solucin de la i-sima ecuacin, i = 1, 2, ..., m.
Una solucin del sistema de ecuaciones lineales
_

_
a
11
x
1
+a
12
x
2
+... +a
1n
x
n
= b
1
a
21
x
1
+a
22
x
2
+... +a
2n
x
n
= b
2
...
a
m1
x
1
+a
m2
x
2
+... +a
mn
x
n
= b
m
es un conjunto ordenado de n valores k
1
, k
2
, ..., k
n
tales que
_

_
a
11
k
1
+a
12
k
2
+... +a
1n
k
n
= b
1
a
21
k
1
+a
22
k
2
+... +a
2n
k
n
= b
2
...
a
m1
k
1
+a
m2
k
2
+... +a
mn
k
n
= b
m
Para cualesquiera sistemas de ecuaciones lineales, se presentan tres tipos de conjunto solucin:
1. Un conjunto solucin que contiene solamente un elemento. Se dice que el sistema tiene
solucin nica y se denomina sistema compatible determinado;
2. Un conjunto solucin que contiene ms de un elemento. En este caso se dice que el sistema
tiene ms de una solucin y se denomina sistema compatible indeterminado;
3. Un conjunto solucin vaco. Se dice que el sistema no tiene solucin y se denomina sistema
incompatible.
Denicin 6.7 Sistema de ecuaciones lineales homogneas
Se llama sistema de m ecuaciones homogneas y n incgnitas, al sistema
_

_
a
11
x
1
+a
12
x
2
+... +a
1n
x
n
= 0
a
21
x
1
+a
22
x
2
+... +a
2n
x
n
= 0
...
a
m1
x
1
+a
m2
x
2
+... +a
mn
x
n
= 0
es decir, cuando todos los trminos independientes son nulos. Se llama sistema de m ecuaciones
no homogneas y n incgnitas, al sistema
_

_
a
11
x
1
+a
12
x
2
+... +a
1n
x
n
= b
1
a
21
x
1
+a
22
x
2
+... +a
2n
x
n
= b
2
...
a
m1
x
1
+a
m2
x
2
+... +a
mn
x
n
= b
m
siempre que al menos un trmino independiente sea diferente de cero.
Se dice que un sistema de ecuaciones lineales es sobredeterminado si hay ms ecuaciones que
incgnitas. Se dice que un sistema de ecuaciones lineales est escasamente determinado si hay
menos ecuaciones que incgnitas.
CAPTULO 6. ECUACIONES Y SISTEMAS DE ECUACIONES 279
Se dice que un sistema de ecuaciones lineales es no susceptible, si errores pequeos en los
coecientes o en el proceso de resolucin slo tienen un efecto pequeo sobre la solucin. Y es
susceptible, si errores pequeos en los coecientes o en el proceso de resolucin tienen un efecto
grande sobre la solucin.
Se dice que un sistema de ecuaciones lineales es equivalente a un segundo sistema de ecuaciones
lineales, si tienen los mismos conjuntos de soluciones.
Dos sistemas de ecuaciones lineales se dice son equivalentes, si uno se obtiene del otro aplicando
una sucesin nita de operaciones elementales.
Ejemplo 6.25 Utilizando el mtodo de operaciones elementales, solucionar el sistema de ecua-
ciones lineales:
_

_
x +y z = 3
2x y + 4z = 3
3x + 2y z = 8
Solucin
Multiplicamos la ecuacin 1 por 2 y luego le restamos la la 2, multiplicamos la la 1 por 3 y luego
restamos la la 3:
_

_
x +y z = 3
y 2z = 1
y 2z = 1
restamos la la dos a la la tres y a la la uno le restamos la segunda la:
_

_
x +z = 2
y 2z = 1
0 = 0
podemos observar que 0 = 0, lo cual indica que el sistema es indeterminado, es decir tiene un
nmero innito de soluciones:
_

_
x = 2 t
y = 1 + 2t
z = t
.
Ejemplo 6.26 Utilizando el mtodo de operaciones elementales, solucionar el sistema de ecua-
ciones lineales:
_

_
3x 4y + 6z = 7
5x + 2y 4z = 5
x + 3y 5z = 3
Solucin
Se multiplica la ecuacin 1 por 5 y luego le restamos 3 veces la la 2, y 3 veces la la 3:
_

_
3x 4y + 6z = 7
13y + 21z = 10
13y + 21z = 2
CAPTULO 6. ECUACIONES Y SISTEMAS DE ECUACIONES 280
restamos la la dos a la la tres:
_

_
3x 4y + 6z = 7
13y + 21z = 10
0 = 12
podemos observar que 0 = -12, lo cual indica que el sistema es inconsistente.
Ejemplo 6.27 Utilizando el mtodo de operaciones elementales, solucionar el sistema de ecua-
ciones lineales:
_

_
x +y +z +u = 0
x + 2y + 3z + 4u = 0
x + 3y + 6z + 10u = 0
x + 4y + 10z + 20u = 0
Solucin
A la segunda la le resto la primera la, a la tercera la le resto la primera la, a la cuarta la le
resto la primera la:
_

_
x +y +z +u = 0
y + 2z + 3u = 0
2y + 5z + 9u = 0
3y + 9z + 19u = 0
A la tercera la le resto 2 veces la segunda la, a la cuarta la le resto 3 veces la segunda la:
_

_
x +y +z +u = 0
y + 2z + 3u = 0
z + 3u = 0
3z + 10u = 0
A la cuarta la le resto 3 veces la tercera la:
_

_
x +y +z +u = 0
y + 2z + 3u = 0
z + 3u = 0
u = 0
Como el sistema se redujo a la forma triangular, entonces el sistema tiene solucin nica. Es decir
x = y = z = u = 0.
6.6. Tarea
1. Resuelva los sistemas de ecuaciones lineales:
a)
_

_
x + 3y z = 1
2x 2y +z = 0
5x + 6y + 3z = 2
b)
_

_
2x +y +z = 8
5x 3y + 2z = 3
7x +y + 3z = 20
c)
_

_
4x 5y + 6z = 3
8x 7y 3z = 9
7x 8y + 9z = 6
d)
_

_
2x y 6z = 3
x 3y + 2z = 5
x +y 4z = 1
CAPTULO 6. ECUACIONES Y SISTEMAS DE ECUACIONES 281
e)
_

_
5x 3z = 4(1 +y)
2(z + 2x) = 8 +y
2y + 3x = 14 z
f )
_

_
x 5y + 2z = 3
2x + 8y z = 1
3x + 3y 5z = 5
g)
_

_
2x y + 3z = 1
x + 5y 2z = 4
3x + 5y 7z = 2
h)
_

_
7x + 4y 3z = 3
4x 3y +z = 1
6x 7y + 3z = 6
i)
_

_
x y + 3z = 0
x + 4y z = 1
x 7y + 3z = 2
j)
_

_
2x +y 5z = 2
x 8y + 6z = 8
x + 3y z = 5
k)
_

_
x + 9y 6z = 8
x + 5y 3z = 1
5x +y z = 7
l)
_

_
7x + 9y z = 0
4x + 6y 3z = 1
x + 7y 5z = 2
m)
_

_
5x 5y + 9z = 5
7x + 6y z = 3
2x + 7y 5z = 0
n)
_

_
7x 9y + 9z = 0
x 12y + 5z = 5
3x 7y + 6z = 6
o)
_

_
x + 5y 9z = 9
5x + 3y + 3z = 0
x y + 3z = 9
p)
_

_
x + 9y 7z = 0
x + 8y + 2z = 1
7x + 7y 4z = 2
q)
_

_
5x + 10y z = 5
x + 9y + 3z = 1
9x + 5y z = 5
r)
_

_
7x + 9y +z = 0
x + 9y 7z = 1
2x 5y + 7z = 2
s)
_

_
x + 9y 7z = 8
x 7y + 9z = 1
x + 8y 2z = 7
t)
_

_
2x 3y + 9z = 9
7x 7y + 4z = 8
5x + 8y 3z = 7
u)
_

_
5x + 5y 7z = 7
2x 9y + 2z = 2
9x + 5y 9z = 9
v)
_

_
x + 9y 7z = 7
3x +y 9z = 3
5x 7y + 9z = 5
w)
_

_
6x 9y + 3z = 3
2x + 4y 8z = 2
7x + 9y 5z = 3
x)
_

_
x 9y + 7z = 5
x + 7y 5x = 4
x 9y 5z = 5
2. Determine de ser posible los valores de a, para que el sistema de ecuaciones lineales tenga
solucin nica, tenga ms de una solucin, no tenga solucin y encuentre la solucin general
del sistema en trminos de a:
a)
_

_
2x 3y +az = 1
ax y +z = 1
x +y +az = 1
b)
_

_
ax + 2ay +z = 2
x +y az = a
2x y +az = 1
c)
_

_
x +y + (a + 1)z = 1
x + (a + 1)y +z = 1
(a + 1)x +y +z = 1
d)
_

_
(a + 1)x +y +z = 2a + 3
(a 1)x y z = 1
2x 4y az = 2
e)
_

_
x ay +az = 1
ax +ay +z = 1
ax y +az = 1
f )
_

_
x +ay +z = 1
ax y +z = a
x +y az = 1
g)
_

_
x (a + 1)y z = 1
ax +y (a + 1)z = 1
x +y z = a
h)
_

_
(a + 1)x y +az = 1
x +y +z = a
3x + 2y az = 1
i)
_

_
x + 3ay z = 0
x 3y +z = a
x +y z = 1
j)
_

_
4x 3y +az = a
2x 3y + 3z = 1
x +y az = a
k)
_

_
x (2a + 1)y z = 1
(2a + 1)x y z = 2
x y (2a + 1)z = 3
l)
_

_
(2a 1)x +y +z = 1
x + (2a 2)y +z = 1
x +y + (2a 3)z = 1
m)
_

_
x +ay + 3z = 2
x y +az = 3
2x +y z = a
n)
_

_
ax +y z = 1
x +a
2
y +z = 0
x +y +az = 1
o)
_

_
(a + 1)x y +z = 1
(a 2)x +y +z = 1
(a 3)x y +z = 1
CAPTULO 6. ECUACIONES Y SISTEMAS DE ECUACIONES 282
p)
_

_
ax + (a 2)y +z = 1
2x (a + 1)y z = 1
3x + (a 1)y +z = 1
q)
_

_
x +y +z = 1
2x +y +z = 1
ax +y z = a
r)
_

_
3ax 2y + 3z = 1
x + 2ay 3z = 1
x +y + 3az = 1
s)
_

_
(a + 2)x +y +z = 2
x + (a + 3)y +z + 3
x +y + (a + 4)z = 4
t)
_

_
0, 2ax 0, 1y +z = 0, 2
0, 1x + 0, 3y +z = 0, 1
0, 3x 0, 4y z = 0, 3
u)
_

_
x + 3ay z = 2
3ax y +z = 1
x +y 3az = 3
v)
_

_
2ax + 2y 3z = 1
x + 3ay +z = 1
x +y + 4z = 1
w)
_

_
x +y (a 3)z = 1
x (a 3)y +z = 1
(a 3)x +y z = 1
x)
_

_
x +y az = a
3
a
2
x y +z = a
2
a
x +y +az = a
3. Determine de ser posible los valores de a y b, para que el sistema de ecuaciones lineales
tenga solucin nica, tenga ms de una solucin, no tenga solucin y encuentre la solucin
general del sistema en trminos de a y b:
a)
_

_
ax +y +z = 4
x +by +z = 3
x + 2by +z = 4
b)
_

_
ax +by +z = 1
x +aby +z = b
x +by +az = 1
c)
_

_
bx (a 1)y z = 1
ax +y (a + 1)z = 1
x +y z = a
d)
_

_
(2b 1)x +y +z = 1
x + (2b 1)y +z = b
x +y + (2b 1)z = 1
e)
_

_
ax +y +z = b
x +ay +z = c
x +y +az = d
f )
_

_
x ay z = b
bx y +z = a
x y +z = 1
g)
_

_
x +y + (b + 1)z = 1
x + (a + 1)y +z = b
(b + 1)x +y +z = 1
h)
_

_
bx 3y +az = 1
ax by +z = 1
bx +y +az = b
i)
_

_
x by +az = 1
bx +ay +z = 1
ax y +az = b
j)
_

_
x + 3y bz = 0
x by +z = a
x +y z = b
k)
_

_
2bx + 2y 3z = 1
x + 3ay +z = b
x +by + 4z = 1
l)
_

_
ax + (b 2)y +z = 1
bx (a + 1)y z = 1
3x + (b 1)y +z = 1
m)
_

_
x +by +az = 2
bx +y +z = 3
2x +y +z = 1
n)
_

_
bx 3y +bz = b
2x by + 3z = 1
x +y az = a
o)
_

_
(b + 1)x y +z = 1
(a + 2)x +y z = b
(b + 3)x y +z = 1
p)
_

_
ax +y +z = b
x +by +z = 3
x +y +az = b
q)
_

_
bx +y z = 1
x +a
2
y +z = b
x +y +bz = 1
r)
_

_
3ax 2y + 3z = b
bx + 2ay 3z = 1
x +y + 3bz = 1
s)
_

_
x (2a + 1)y z = 1
(2a + 1)x y z = b
x y (2b + 1)z = 3
t)
_

_
x +y (a 3)z = b
x (b 3)y +z = 1
(a 3)x +y z = 1
u)
_

_
x +y +z = b
bx +y +z = 1
ax +by z = a
v)
_

_
x + 3ay az = 2
3bx y +z = b
x +y 3z = 3
w)
_

_
0, 2bx 0, 1y +z = 0, 2
0, 1x + 0, 3by +z = 0, 1
0, 3x 0, 4y az = 0, 3
x)
_

_
x +y z = b
3
b
2
x y +z = a
2
a
x +y +z = b
4. Resuelva los sistemas de ecuaciones lineales:
CAPTULO 6. ECUACIONES Y SISTEMAS DE ECUACIONES 283
a)
_

_
x +y +z u = 2
x +y z + 2u = 5
3x y +z + 2u = 3
b)
_

_
2x + 3y z + 2u = 1
3x y + 2z u = 2
x 3y +z 4u = 3
c)
_

_
5x + 4y 3z u = 8
3x + 2y 5z + 2u = 1
x + 5y +z + 3u = 2
d)
_

_
2x 3y + 2z + 3u = 1
x 5y + 2z 3u = 2
3x 4y +z u = 5
e)
_

_
3x + 4y z + 2u = 1
x + 5y 2z + 3u = 2
x +y 4z + 5u = 1
f )
_

_
x + 5y + 3z +u = 5
4x y + 2z u = 1
x + 3y 5z + 2u = 2
g)
_

_
4x +y 3z + 2u = 4
x + 5y + 4z u = 5
2x 3y +z 3u = 3
h)
_

_
5x + 4y z + 3u = 1
x 5y + 4z 3u = 1
5x + 7y z + 5u = 1
i)
_

_
x + 10y 4z u = 2
5x +y 11z +u = 3
x + 11y + 12z 3u = 1
j)
_

_
x +y z 4u = 1
2x + 4y z + 4u = 3
x + 3y 5z + 2u = 2
k)
_

_
2x y +z u = 0
x + 2y 2z u = 2
x +y 3z + 2u = 1
l)
_

_
x + 5y 5z +u = 3
x 3y + 3z u = 0
x +y +z u = 3
5. Resuelva los sistemas de ecuaciones lineales:
a)
_

_
x +y z +u = 2
x y + 3z u = 1
x +y + 2z + 2u = 1
3x y +z +u = 1
b)
_

_
x +y +z +u = 1
x y +z u = 2
2x +y 2z +u = 1
x + 2y z + 2u = 2
c)
_

_
2x y + 2z u = 0
x +y 3z + 3u = 1
3x +y + 3z u = 0
x +y z + 3u = 1
d)
_

_
2x + 3y z +u = 0
3x + 2y 5z +u = 1
x 3y + 2z u = 0
x +y 2z + 3u = 1
e)
_

_
x + 2y z + 2u = 0
x + 3y +z u = 1
3x +y z + 2u = 0
x y + 2z u = 1
f )
_

_
x +y z + 3u = 3
x y + 3z u = 2
x + 3y z +u = 1
3x y +z u = 1
g)
_

_
3x y + 2z +u = 1
x + 3y z +u = 2
x y + 3z u = 1
x +y z + 3u = 1
h)
_

_
5x +y 3z +u = 2
x 3y + 2z u = 2
3x + 2y z + 2u = 1
x +y 2z u = 1
i)
_

_
2x 2y + 3z + 3u = 1
2x + 2y z 2u = 1
x 3y + 2z 3u = 1
x + 3y z + 2u = 1
j)
_

_
x + 2y + 3z u = 3
2x 3y + 2z +u = 1
x 2y 3z +u = 2
2x + 3y z + 2u = 1
k)
_

_
4x 3y + 2z u = 1
5x + 4y 3z +u = 2
x + 4y 5z + 2u = 1
2x y + 2z 3u = 2
l)
_

_
3x y + 3z u = 3
2x + 3y 2z +u = 1
3x + 2y z 3u = 2
4x 3y +z + 3u = 4
m)
_

_
x + 2y z + 3u = 5
x 2y + 3z + 2u = 3
3x +y 2z +u = 1
x y + 3z 2u = 2
n)
_

_
x + 3y +z + 2u = 1
x 2y 2z 2u = 1
x + 2y + 3z +u = 2
x 3y z u = 3
o)
_

_
5x 3y +z + 2u = 1
4x + 3y 2z 2u = 2
3x 2y + 3z 3u = 3
2x y z + 4u = 4
p)
_

_
3x 5y + 7z u = 4
x + 7y 5z 3u = 3
2x 7y + 4z u = 2
x 5y + 4z 3u = 1
q)
_

_
x + 7y 5z + 3u = 5
4x y + 3z 2u = 2
5x + 3y z + 4u = 3
x 6y + 4z u = 2
r)
_

_
7x 12y + 4z 3u = 5
3x + 7y 5z + 4u = 2
2x + 5y 12z u = 3
x + 12y 15z + 2u = 4
CAPTULO 6. ECUACIONES Y SISTEMAS DE ECUACIONES 284
s)
_

_
2x 4y 5z + 6u = 1
x + 3y 3x + 4u = 1
3x y + 2z 3u = 3
x 4y + 2z 5u = 2
t)
_

_
4x 3y z + 5u = 5
x 10y +z 3u = 2
4x + 2y 5z +u = 3
x + 5y 3z + 2u = 1
u)
_

_
3x 4y + 5z u = 4
x 5y + 6z + 2u = 1
2x 4y + 4z u = 3
x + 5y 5z + 3u = 2
6. Resuelva los sistemas de ecuaciones lineales:
a)
_

_
2x + 3y + 7z 5u +v = 5
x + 2y + 4z + 7u 2v = 3
3x + 2y + 4z + 7u v = 7
9x + 6y +z u + 3v = 2
b)
_

_
6x + 3y + 2z + 3u + 4v = 5
4x + 2y +z + 2u + 3v = 4
4x + 2y + 3z + 2u +v = 0
2x +y + 7z + 3u + 2v = 1
c)
_

_
7x + 9y + 4z + 2u 3v = 2
2x 2y +z +u + 4v = 6
5x + 6y + 3z + 2u v = 3
2x + 3y +z +u +v = 0
d)
_

_
x + 2y + 3z 2u + 4v = 4
3x 6y + 5z + 2u = 5
x + 2y + 7z 3u v = 3
2x + 4y + 2z 5u + 2v = 4
e)
_

_
3x 3y + 2z 2u + 4v = 5
2x + 2y + 2z + 3u 3v = 1
4x + 2y 3z + 2u 3v = 2
2x + 5y 5z + 3u 3v = 5
f )
_

_
2x y 6z + 3u + 5v = 1
7x 4y + 2z 15u 3v = 2
x 2y 4z + 9u 6v = 5
x y + 2z 6u + 2v = 7
6.7. Fracciones parciales
Denicin 6.8 Fraccin racional
La expresin
p(x)
q(x)
, donde p(x) y q(x) son polinomios, con la particularidad de que q(x) es un
polinomio no nulo, lleva el nombre de fraccin racional. El polinomio p(x) se denomina numerador
y q(x) denominador de la fraccin racional.
Por lo visto, cada polinomio t(x) es una fraccin racional, en tal caso, p(x) = t(x), q(x) = 1.
Denicin 6.9 Fracciones racionales iguales
Las fracciones racionales
p(x)
q(x)
y
r(x)
s(x)
se consideran iguales si p(x)s(x) = r(x)q(x). De aqu sigue
que dos fracciones racionales con iguales denominadores son iguales si y slo si son iguales sus
denominadores.
La suma
p(x)
q(x)
+
r(x)
s(x)
de las fracciones racionales
p(x)
q(x)
y
r(x)
s(x)
se denomina fraccin racional
p(x)s(x) +r(x)q(x)
q(x)s(x)
y su producto
p(x)
q(x)

r(x)
s(x)
, la fraccin racional
p(x)r(x)
q(x)s(x)
.
La diferencia y el cociente de dos fracciones racionales se determinan como el resultado de
las operaciones inversas a la adicin y multiplicacin. Las operaciones de adicin y multiplicacin
CAPTULO 6. ECUACIONES Y SISTEMAS DE ECUACIONES 285
de fracciones racionales son conmutativas y asociativas; estn ligadas entre s mediante la ley
distributiva.
Denicin 6.10 fraccin racional propia e impropia
La fraccin racional
p(x)
q(x)
lleva el nombre de propia si el grado del polinomio p(x) es menor que el
de q(x). Si el grado de p(x) es mayor o igual que el de q(x), la fraccin racional se llama impropia.
Toda fraccin racional impropia
p(x)
q(x)
puede representarse de un modo nico, en forma de una
suma de un polinomio y cierta fraccin racional propia.
La representacin de una fraccin impropia en esta forma, recibe el nombre de formacin de la
parte entera de una fraccin racional impropia. Toda fraccin racional propia puede ser descom-
puesta, en forma nica, como la suma de un nmero nito de fracciones simples.
Todo polinomio q(x) de coecientes reales q(x) = a
n
x
n
+a
n1
x
n1
+... +a
0
se descompone de
modo nico en forma de un producto de su coeciente principal a
n
, de un nmero nito de poli-
nomios de la forma x a, correspondientes a sus races reales y de un nmero nito de polinomios
de la forma x
2
+px +q, correspondientes a sus races no reales.
Sea
p(x)
q(x)
una fraccin racional propia. Supongamos que los coecientes de los polinomios que
la integran son nmeros reales y la fraccin dada es irreducible, esto signica que el numerador y
el denominador no tienen races comunes. Entonces tenemos los siguientes casos:
1. Sea
p(x)
q(x)
la fraccin racional propia cuyo denominador tiene la forma
q(x) = (x a
1
)(x a
2
)...(x a
n
)
Entonces, para esta fraccin es vlida la siguiente descomposicin
p(x)
q(x)
=
A
1
x a
1
+
A
2
x a
2
+... +
A
n
x a
n
En esta expresin A
1
, A
2
, ..., A
n
, son ciertos nmeros constantes, algunos de los cuales
pueden ser iguales a cero o iguales entre s.
2. Sea
p(x)
q(x)
la fraccin racional propia cuyo denominador tiene la forma
q(x) = (x a
1
)
n
(x a
2
)
m
...(x a
r
)
k
Entonces, para esta fraccin es vlida la siguiente descomposicin
p(x)
q(x)
=
A
1
x a
1
+
A
2
(x a
1
)
2
+... +
A
n
(x a
1
)
n
+
B
1
x a
2
+
B
2
(x a
2
)
2
+...+
+
B
n
(x a
2
)
m
+
C
1
x a
r
+
C
2
(x a
r
)
2
+... +
C
k
(x a
r
)
k
En esta expresin A
1
, A
2
, ..., A
n
, B
1
, B
2
, ..., B
m
, ..., C
1
, C
2
, ..., C
k
son ciertos nmeros
constantes, algunos de los cuales pueden ser iguales a cero o iguales entre s.
CAPTULO 6. ECUACIONES Y SISTEMAS DE ECUACIONES 286
3. Sea
p(x)
q(x)
la fraccin racional propia cuyo denominador tiene la forma
q(x) = (x
2
+a
1
x +b
1
)(x
2
+a
2
x +b
2
)...(x
2
+a
n
x +b
n
)
Entonces, para esta fraccin es vlida la siguiente descomposicin
p(x)
q(x)
=
A
1
x +B
1
x
2
+a
1
x +b
1
+
A
2
x +B
2
x
2
+a
2
x +b
2
+... +
A
n
x +B
n
x
2
+a
n
x +b
n
En esta expresin A
1
, A
2
, ..., A
n
, B
1
, B
2
, ..., B
n
son ciertos nmeros constantes, algunos de
los cuales pueden ser iguales a cero o iguales entre s.
4. Sea
p(x)
q(x)
la fraccin racional propia cuyo denominador tiene la forma
q(x) = (x
2
+a
1
x +b
1
)
m
(x
2
+a
2
x +b
2
)
r
...(x
2
+a
n
x +b
n
)
k
Entonces, para esta fraccin es vlida la siguiente descomposicin
p(x)
q(x)
=
A
1
x +B
1
x
2
+a
1
x +b
1
+
A
2
x +B
2
(x
2
+a
1
x +b
1
)
2
+... +
A
m
x +B
m
(x
2
+a
1
x +b
1
)
m
+
+
C
1
x +D
1
x
2
+a
2
x +b
2
+
C
2
x +D
2
(x
2
+a
2
x +b
2
)
2
+... +
C
r
x +D
r
(x
2
+a
2
x +b
2
)
r
+
E
1
x +F
1
x
2
+a
n
x +b
n
+
E
2
x +F
2
(x
2
+a
n
x +b
n
)
2
+... +
E
k
x +F
k
(x
2
+a
n
x +b
n
)
k
En esta expresin A
1
, A
2
, ..., A
m
, B
1
, B
2
, ..., B
m
, C
1
, C
2
, ..., C
r
, D
1
, D
2
, ..., D
r
, ..., E
1
,
E
2
, ..., E
k
, F
1
, F
2
, ..., F
k
son ciertos nmeros constantes, algunos de los cuales pueden ser
iguales a cero o iguales entre s.
Ejemplo 6.28 Descomponga la fraccin racional en fracciones elementales:
x
6
x
2
+ 1
(x 1)
3
.
Solucin
Dividiendo esta expresin, obtenemos
x
3
+ 3x
2
+ 6x + 10 +
14x
2
24x + 11
(x 3)
3
Analizaremos la fraccin
14x
2
24x + 11
(x 3)
3
=
A
x 1
+
B
(x 1)
2
+
C
(x 1)
3
Eliminamos denominadores y establecemos un sistema de ecuaciones:
14x
2
24x + 11 = A(x 1)
2
+B(x 1) +C
_

_
x
2
: A = 14
x
1
: 2A+B = 24
x
0
: AB +C = 11
CAPTULO 6. ECUACIONES Y SISTEMAS DE ECUACIONES 287
Resolviendo este sistema, obtenemos A = 14, B = 4, C = 1. De donde
14x
2
24x + 11
(x 3)
3
=
14
x 1
+
4
(x 1)
2
+
1
(x 1)
3
Por lo tanto la fraccin racional queda descompuesta por:
x
3
+ 3x
2
+ 6x + 10 +
14x
2
24x + 11
(x 3)
3
=
14
x 1
+
4
(x 1)
2
+
1
(x 1)
3
.
Ejemplo 6.29 Descomponga la fraccin racional en fracciones elementales:
x
6
7x
4
+ 8x
3
8x + 8
x
3
+ 1
Solucin
Dividiendo esta expresin, obtenemos
x
3
7x + 7
x 1
x
3
+ 1
Analizaremos la fraccin
x 1
x
3
+ 1
=
x 1
(x + 1)(x
2
x + 1)
=
A
x + 1
+
Bx +C
x
2
x + 1
Eliminamos denominadores y establecemos un sistema de ecuaciones:
x 1 = A(x
2
x + 1) + (Bx +C)(x + 1)
_

_
x
2
: A+B = 0
x
1
: A+B +C = 1
x
0
: A+C = 1
Resolviendo este sistema, obtenemos A =
2
3
, B =
2
3
, C =
1
3
. De donde

2
3
x + 1
+
2
3
x
1
3
x
2
x + 1
=
1
3
_
2
x + 1
+
1 2x
x
2
x + 1
_
Por lo tanto la fraccin racional queda descompuesta por:
x
3
7x + 7
1
3
_
2
x + 1
+
1 2x
x
2
x + 1
_
.
Ejemplo 6.30 Descomponga la fraccin racional en fracciones elementales:
x + 1
(x + 1)
4
16
Solucin
Analizaremos la fraccin
x + 1
(x 1)(x + 3)(x
2
+ 2x + 5)
=
A
x 1
+
B
x + 3
+
Cx +D
x
2
+ 2x + 5
Eliminamos denominadores y establecemos un sistema de ecuaciones:
x + 1 = A(x + 3)(x
2
+ 2x + 5) +B(x 1)(x
2
+ 2x + 5) + (Cx +D)(x 1)(x + 3)
CAPTULO 6. ECUACIONES Y SISTEMAS DE ECUACIONES 288
_

_
x
3
: A+B +C = 0
x
2
: 5A+B + 2C +D = 0
x
1
: 11A+ 3B 3C + 2D = 1
x
0
: 15A5B 3C = 1
Resolviendo este sistema, obtenemos A =
1
16
, B =
1
16
, C =
1
8
, D =
1
8
. De donde
1
16
x 1
+
1
16
x + 3
+

1
8
x
1
8
x
2
+ 2x + 5
=
1
16
_
1
x 1
+
1
x + 3

2x + 2
x
2
+ 2x + 5
_
.
Ejemplo 6.31 Descomponga la fraccin racional en fracciones elementales:
1
x
4
+ 1
Solucin
Analizaremos la fraccin
1
(x
2
+

2 x + 1)(x
2

2 x + 1)
=
Ax +B
x
2
+

2 x + 1
+
Cx +D
x
2

2 x + 1
Eliminamos denominadores y establecemos un sistema de ecuaciones:
1 = (Ax +B)(x
2

2 x + 1) + (Cx +D)(x
2
+

2 x + 1)
_

_
x
3
: A+C = 0
x
2
:

2 A+B +

2 C +D = 0
x
1
: A

2 B +C +

2 D = 0
x
0
: B +D = 1
Resolviendo este sistema, obtenemos A =

2
4
, B =
1
2
, C =

2
4
, D =
1
2
. De donde

2
4
x +
1
2
x
2
+

2 x + 1
+

2
4
x +
1
2
x
2

2 x + 1
=
1
4
_

2 x + 2
x
2
+

2 x + 1

2 x 2
x
2

2 x + 1
_
.
Ejemplo 6.32 Descomponga la fraccin racional en fracciones elementales:
8x
2
x
4
1
Solucin
Analizaremos la fraccin
8x
2
(x + 1)(x 1)(x
2
+ 1)
=
A
x + 1
+
B
x 1
+
Cx +D
x
2
+ 1
Eliminamos denominadores y establecemos un sistema de ecuaciones:
8x
2
= A(x 1)(x
2
+ 1) +B(x + 1)(x
2
+ 1) + (Cx +D)(x
2
1)
_

_
x
3
: A+B +C = 0
x
2
: A+B +D = 8
x
1
: A+B C = 0
x
0
: A+B D = 0
CAPTULO 6. ECUACIONES Y SISTEMAS DE ECUACIONES 289
Resolviendo este sistema, obtenemos A = 2, B = 2, C = 0, D = 4. De donde
8x
2
x
4
1
=
2
x + 1
+
2
x 1
+
4
x
2
+ 1
.
Ejemplo 6.33 Descomponga la fraccin racional en fracciones elementales:
x
3
x
4
+ 18x
2
+ 81
Solucin
Analizaremos la fraccin
x
3
(x
2
+ 9)
2
=
Ax +B
x
2
+ 9
+
Cx +D
(x
2
+ 9)
2
Eliminamos denominadores y establecemos un sistema de ecuaciones:
x
3
= (Ax +B)(x
2
+ 9) + (Cx +D)
_

_
x
3
: A = 1
x
2
: B = 0
x
1
: 9A+C = 0
x
0
: 9B +D = 0
Resolviendo este sistema, obtenemos A = 1, B = 0, C = 9, D = 0. De donde
x
3
x
4
+ 18x
2
+ 81
=
x
x
2
+ 9

9x
(x
2
+ 9)
2
.
Ejemplo 6.34 Descomponga la fraccin racional en fracciones elementales:
1
x
6
+ 2x
3
+ 1
Solucin
Analizaremos la fraccin
1
(x + 1)
2
(x
2
x + 1)
2
=
A
x + 1
+
B
(x + 1)
2
+
Cx +D
x
2
x + 1
+
Ex +F
(x
2
x + 1)
2
Eliminamos denominadores y establecemos un sistema de ecuaciones:
1 = A(x + 1)(x
2
x + 1)
2
+B(x
2
x + 1)
2
+ (Cx +D)(x + 1)
2
(x
2
x + 1) + (Ex +F)(x + 1)
2
_

_
x
5
: A+C = 0
x
4
: A+B +C +D = 0
x
3
: A2B +D +E = 0
x
2
: A+ 3B +C + 2E +F = 0
x
1
: A2B +C +D +E + 2F = 0
x
0
: A+B +D +F = 1
Resolviendo este sistema, obtenemos A =
2
9
, B =
1
9
, C =
2
9
, D =
1
3
, E =
1
3
, F =
1
3
. De donde
1
x
6
+ 2x
3
+ 1
=
1
9
_
2
x + 1
+
1
(x + 1)
2

2x 3
x
2
x + 1

3x 3
(x
2
x + 1)
2
_
6.8. Tarea
1. Descomponga la fraccin racional en fracciones elementales:
CAPTULO 6. ECUACIONES Y SISTEMAS DE ECUACIONES 290
a)
2x
2
+ 3x 1
x
3
+ 2x
2
x 2
;
b)
3x
2
+ 3x + 1
2x
3
+ 11x
2
+ 4x 5
;
c)
x
2
+ 3x + 6
4x
3
+ 8x
2
9x 18
;
d)
2x
3
+ 3x + 3
3x
3
+ 20x
2
+ 29x 12
;
e)
5x
3
4x
2
+x 3
24x
3
34x
2
5x + 3
;
f )
x
3
4
50x
3
+ 175x
2
2x 7
;
g)
x
4
5x
2
+ 2x 5
12x
3
+ 41x
2
+ 13x 6
;
h)
2x
4
2x
2
3x 1
6x
3
+ 25x
2
+ 31x + 10
;
i)
3x
5
5x
2
+ 2x + 5
x
3
+ 2x
2
33x 90
;
j)
3x
5
5x
2
+ 2x + 5
x
3
+ 2x
2
33x 90
;
k)
x
5
5x
4
+x3 2
6x
3
+ 5x
2
7x 4
;
l)
x
2
+ 2x + 6
(x 1)(x 2)(x 4)
;
m)
1
(x + 1)(x + 2)
2
(x + 3)
3
;
n)
2x
3
+x
2
+x + 2
(x 1)
2
(x
2
+x + 1)
;
o)
x
4
2x
3
+ 3x
2
x + 3
x
3
2x
2
+ 3x
;
p)
x
3
+x
2
+x + 2
x
4
+ 3x
2
+ 2
;
q)
x
3
+x
2
5x + 15
(x
2
+ 5)(x
2
+ 2x + 3)
;
r)
x 1
x
2
(x 2)(x + 1)
2
;
s)
x
4
+ 8x
3
x
2
+ 2x + 1
(x
2
+x)(x
3
+ 1)
;
t)
3x + 5
x
3
x
2
x + 1
;
u)
3x
2
+ 2x 1
(x 1)
2
(x + 2)
;
v)
1 2x
x(x + 1)
2
(x
2
+x + 1)
2
;
w)
x + 4
x
3
+ 6x
2
+ 11x + 6
;
x)
x
5
+ 3x
2
1
(x
2
+ 1)(x
5
1)
;
y)
5x
2
+ 6x 23
(x 2)(x + 1)
2
(x 1)
3
.
2. Descomponga la fraccin racional en fracciones elementales:
a)
6
x
3
1
;
b)
x
5
x
x
8
+ 1
;
c)
x
4
+ 1
x
4
1
;
d)
x
2
x
(x + 1)
4
;
e)
x
2
+x
x
6
+ 1
;
f )
3
x
2
x
5
;
g)
x
4
+ 1
x
6
+ 1
;
h)
2x
2
+ 3
(x
2
+ 1)
2
;
i)
x
5
3x
x
2
(x
5
1)
;
j)
1
x
4
+x
2
+ 1
.
3. Descomponga la fraccin racional en fracciones elementales:
a)
x
4
x
3
x 1
x
3
x
2
;
b)
x
2
2x + 2
x
3
+ 2x
2
8x
;
c)
5x 13
(x
2
5x + 6)
2
;
d)
x
2
+ 5x + 4
x
4
+ 5x
2
+ 4
;
e)
5x
2
+ 3x 1
(x
2
+ 5)(x
4
+ 1)
;
f )
x
4
+ 3x
3
+ 3x
2
5
x
3
+ 3x
2
+ 3x + 1
;
g)
x
6
+ 5x
5
+ 3x
4
+ 3x 1
x(2x 1)
2
(2x
2
x + 3)
2
;
h)
x
5
x
4
+ 4x
2
+ 8x
(x
2
+ 2)
3
;
i)
2
x
4
+ 2x
3
+ 2x
2
+ 2x + 1
;
j)
1
(x
2
4x + 4)(x
2
4x + 5)
;
k)
x
2
+ 5x + 1
6x
3
+ 19x
2
+ 2x 3
;
l)
2x
4
3x
2
+ 2x + 5
10x
3
+ 19x
2
9
;
m)
x
6
+x
5
3x
4
+ 2x
3
1
(x 3)
2
(x
2
+ 2x + 5)
2
;
n)
5x
3
+ 2x
2
x + 1
6x
3
+ 17x
2
59x + 30
.
6.9. Ecuaciones cuadrticas
La ecuacin cuyo primer miembro es un polinomio de segundo grado, con respecto a la in-
cgnita x, y el segundo miembro es igual a cero, se denomina cuadrtica. La forma general de la
ecuacin cuadrtica o ecuacin de segundo grado es Ax
2
+ Bx + C = 0. Los nmeros A y B son
los coecientes del trmino principal y de la incgnita de primer grado, respectivamente y C, el
trmino independiente.
El nmero x, que hace igual a cero el trinomio cuadrado Ax
2
+ Bx + C, se denomina raz de
la ecuacin cuadrtica Ax
2
+Bx +C = 0.
CAPTULO 6. ECUACIONES Y SISTEMAS DE ECUACIONES 291
Si en la ecuacin cuadrtica de la forma general Ax
2
+Bx +C = 0 uno de los dos coecientes,
B o C, es igual a cero, o ambos a la vez son iguales a cero, la ecuacin cuadrtica se denomina
incompleta. Son posibles tres formas de ecuaciones cuadrticas incompletas:
1) Ax
2
+Bx = 0 (C = 0, A ,= 0, B ,= 0): Esta ecuacin se resuelve descomponiendo el primer
miembro en factores, x(Ax + B) = 0. El producto se anula cuando uno de los factores es igual a
cero; por eso, o bien x = 0, o bien Ax
2
+ Bx = 0, de donde x =
B
A
. De este modo, la ecuacin
Ax
2
+Bx = 0 tiene dos races; x
1
= 0, x
2
=
B
A
.
Ejemplo 6.35 Determine las races de la siguiente ecuacin cuadrtica
3x
2
+ 4x = 0.
Solucin
Esta ecuacin la resolvemos de la siguiente manera:
3x
2
+ 4x = 0 x(3x + 4) = 0
_
x = 0
3x + 4 = 0

_
x = 0
x =
4
3
2) Ax
2
+ C = 0 (B = 0, A ,= 0, C ,= 0): La ecuacin Ax
2
+ C = 0, despus de dividir los
trminos por A y pasar el trmino independiente al segundo miembro, la reducimos a la forma
x
2
=
C
A
, de donde x =
_

C
A
. Si los coecientes A y C tienen signos contrarios, tendremos que
c
A
< 0, y por eso la incgnita x tiene dos valores reales de signos contrarios
_
_
_
x
1
=
_

C
A
x
2
=
_

C
A
Ejemplo 6.36 Determine las races de la siguiente ecuacin cuadrtica
5x
2
+ 2 = 0.
Solucin
Esta ecuacin la resolvemos de la siguiente manera:
5x
2
2 = 0 x
2
=
2
5
x =
_
2
5
3) Ax
2
= 0 (A ,= 0, B = C = 0): La ecuacin Ax
2
= 0, puesto que A ,= 0, x
2
= 0, x = 0. Se
dice que el nmero 0 es raz doble de la ecuacin Ax
2
= 0, es decir, x
1
= x
2
= 0.
Ejemplo 6.37 Determine las races de la siguiente ecuacin cuadrtica
2x
2
= 0.
Solucin
Esta ecuacin la resolvemos de la siguiente manera:
2x
2
= 0 x
2
= 0 x = 0.
CAPTULO 6. ECUACIONES Y SISTEMAS DE ECUACIONES 292
La ecuacin cuadrtica, cuyo primer coeciente es igual a 1, es decir, la ecuacin de la forma
x
2
+Px +Q = 0, se llama reducida.
Transformando el primer miembro de la ecuacin cuadrtica reducida
x
2
+ 2
P
2
x +
_
P
2
_
2

_
P
2
_
2
+Q = 0.
En el primer miembro de esta ecuacin se introdujeron como sumandos dos nmeros contrarios
_
P
2
_
2
y
_
P
2
_
2
, lo que, desde luego, no vara la magnitud del primer miembro. Despus de pasar
los ltimos dos sumandos al segundo miembro, tendremos
x
2
+ 2
P
2
x +
_
P
2
_
2
=
_
P
2
_
2
Q
_
x +
P
2
_
2
=
P
2
4
Q.
Extraemos la raz cuadrada de ambos miembros, considerando que
_
P
2
_
2
Q 0 en tal caso
x +
P
2
=

_
P
2
_
2
Q
_
_
_
x
1
=
P
2
+
_
_
P
2
_
2
Q
x
2
=
P
2

_
_
P
2
_
2
Q
.
Esta es precisamente la frmula con la cual se calculan las races de la ecuacin cuadrtica reducida,
lo cual se puede expresar como: Las races de la ecuacin cuadrtica reducida son iguales a la mitad
del segundo coeciente, con signo contrario, ms - menos la raz cuadrada del cuadrado de esta
mitad menos el trmino independiente.
Ejemplo 6.38 Determine las races de la siguiente ecuacin cuadrtica
x
2
+ 4x 5 = 0.
Solucin
Esta ecuacin la resolvemos de la siguiente manera:
x
2
+ 4x 5 = 0
_
_
_
x
1
=
4
2
+
_
_
4
2
_
2
(5)
x
2
=
4
2

_
_
4
2
_
2
(5)

_
x
1
= 1
x
2
= 5
.
Si se necesita hallar las races de la ecuacin cuadrtica de la forma general Ax
2
+Bx+C = 0,
despus de dividir todos los trminos por A (A ,= 0) ella se convierte en reducida
x
2
+
B
A
x +
C
A
= 0.
En tal caso
_

_
x
1
=
B +

B
2
4AC
2A
x
2
=
B

B
2
4AC
2A
Esto se puede expresar como: Las races de la ecuacin cuadrtica de la forma general son iguales a
una fraccin cuyo denominador es el doble del primer coeciente y el numerador es igual al segundo
coeciente, con signo contrario, ms - menos la raz cuadrada del cuadrado de este coeciente menos
el cudruplo del producto del primer coeciente por el trmino independiente.
CAPTULO 6. ECUACIONES Y SISTEMAS DE ECUACIONES 293
Ejemplo 6.39 Determine las races de la siguiente ecuacin cuadrtica
2x
2
+ 3x 1 = 0.
Solucin
Esta ecuacin la resolvemos de la siguiente manera:
2x
2
+ 3x 1 = 0
_
_
_
x
1
=
3+

3
2
42(1)
22
x
2
=
3

3
2
42(1)
22

_
x
1
=
3+

17
4
x
2
=
3

17
4
.
Entre las races de la ecuacin cuadrtica y sus coecientes existe una dependen-cia expresada
en las siguientes propiedades:
1. La suma de las races de la ecuacin cuadrtica reducida es igual al segundo coeciente
con signo contrario, y el producto de las races es igual al trmino independiente.
2. Si la suma de dos nmeros desconocidos es igual a P y su producto es igual a Q, los
nmeros buscados son las races de la ecuacin cuadrtica x
2
Px +Q = 0.
3. Para la ecuacin cuadrtica de la forma general Ax
2
+ Bx + C = 0, despus de reducirla
a la forma x
2
+
B
A
x +
C
A
= 0, tendremos x
1
+x
2
=
B
A
y x
1
x
2
=
C
A
.
Utilizando las propiedades de las races de la ecuacin cuadrtica, todo trinomio de races reales
se puede descomponer en factores:
Ax
2
+Bx +C = A
_
x
2
+
B
A
x +
C
A
_
= A[x
2
(x
1
+x
2
)x +x
1
x
2
]
= (x
2
x
1
x) (x
2
x x
1
x
2
)]
= x(x x
1
) x
2
(x x
1
)]
= A(x x 1)(x x
2
).
Al resolver una ecuacin cuadrtica de coecientes numricos en ciertos casos se obtienen dos
races reales, diferentes entre s; en otros casos, dos races reales iguales, y en los dems, dos races
imaginarias.
En este anlisis tiene especial importancia la expresin = B
2
4AC, llamado discriminante
de la ecuacin se segundo grado. Son posibles tres casos:
CASO 1. A > 0, > 0:
Si el discriminante es un nmero positivo, la ecuacin cuadrtica tiene dos races reales y distintas,
puesto que la expresin

B
2
4AC representa en s dos nmeros contrarios, ms an, ninguno
de ellos es igual a cero; por lo tanto, las fracciones
B

2A
y
B +

2A
tienen diferentes numeradores para denominadores iguales. Con respecto a los signos de los coe-
cientes B y C se pueden efectuar las cuatro suposiciones siguientes:
1. B < 0, C > 0: Si el trmino independiente es positivo, ambas races son de signo igual,
puesto que x
1
x
2
=
C
A
> 0. La suma de las races x
1
+x
2
=
B
A
> 0, y, por eso, ambas races
son positivas.
CAPTULO 6. ECUACIONES Y SISTEMAS DE ECUACIONES 294
2. B > 0, C > 0: Ambas races son negativas y de igual signo, puesto que el signo de la suma
de las races es contrario al signo del coeciente
B
A
> 0.
3. B < 0, C < 0: Las races son de signo contrario, dado que el producto es negativo,
x
1
x
2
=
C
A
< 0. La raz mayor en valor absoluto es positiva, ya que x
1
+x
2
=
B
A
> 0.
4. B > 0, C < 0: Las races son de signo contrario. La raz mayor en valor absoluto es
negativa.
Ejemplo 6.40 Determine las races de las siguientes ecuaciones cuadrticas:
a) 2x
2
5x +1 = 0; b) 3x
2
+2x +4 = 0; c) 4x
2
3x 2 = 0; d) 5x
2
+2x 3 = 0.
Solucin
Estas ecuaciones las resolvemos de la siguiente manera:
a) 2x
2
5x + 1 = 0
_

_
x
1
=
(5) +
_
(5)
2
4 2 1
2 2
x
2
=
(5)
_
(5)
2
4 2 1
2 2

_
x
1
=
5 +

17
4
x
2
=
5

17
4
b) 3x
2
+ 2x + 4 = 0
_

_
x
1
=
2 +

2
2
4 3 4
2 3
x
2
=
2

2
2
4 3 4
2 3

_
x
1
=
1 +

11
3
x
2
=
1

11
3
c) 4x
2
3x 2 = 0
_

_
x
1
=
(3) +
_
(3)
2
4 4 (2)
2 4
x
2
=
(3)
_
(3)
2
4 4 (2)
2 4

_
x
1
=
3 +

41
8
x
2
=
3

41
8
d) 5x
2
+ 2x 3 = 0
_

_
x
1
=
2 +
_
5
2
4 5 (3)
2 5
x
2
=
2
_
2
2
4 5 (3)
2 5

_
_
_
x
1
=
3
5
x
2
= 1
.
CASO 2. A > 0, = 0:
Ambas races son reales e iguales, x
1
= x
2
=
B
2A
, como se desprende de la frmula de las races
de la ecuacin cuadrtica. Si B > 0, ambas races son negativas; para B < 0, ambas races son
positivas.
CASO 3. A > 0, < 0:
La ecuacin cuadrtica no tiene races reales, puesto que la raz cuadrada del nmero negativo

es un nmero imaginario.
Los resultados del anlisis estn expuestos geomtricamente en las grcas del trinomio cuadra-
do. En el caso 1 la parbola corta el eje de abscisas en dos puntos x
1
y x
2
(x
1
y x
2
son races del
trinomio y al mismo tiempo races de la ecuacin cuadrtica).
En el caso 2, la parbola es tangente al eje de abscisas (las dos races se confunden en una).
En el caso 3, la parbola no corta al eje 0X (las races imaginarias)
Ejemplo 6.41 Determine para qu valores reales del parmetro k las races de la ecuacin
(k 3)x
2
2kx + 6k = 0
son reales. Bajo qu condiciones las races sern positivas?
Solucin
CAPTULO 6. ECUACIONES Y SISTEMAS DE ECUACIONES 295
1) Si k ,= 3, la ecuacin es de segundo grado. Las races sern reales si 0, es decir
(2k)
2
4(k 3)6k 0 4k(18 5k) 0 0 k
18
5
luego k
_
0;
18
5

3.
2) Supongamos que k = 3, entonces tenemos la ecuacin lineal
6x + 18 = 0
de donde obtenemos x = 3 R.
As, hemos determinado que las races de la ecuacin sern reales si k
_
0;
18
5

.
Si x
1
y x
2
son las races reales de la ecuacin, se tiene que
x
1
+x
2
=
B
A
x
1
x
2
=
C
A
Luego las races sern positivas si
x
1
+x
2
> 0 x
1
x
2
> 0
Si k = 3 entonces x = 3 > 0
Si k ,= 3, entonces las races sern positivas si
2k
k 3
> 0
6k
k 3
> 0
con k
_
0;
18
5

3. Resolviendo las inecuaciones, obtenemos k


_
3;
18
5

. En consecuencia las races


sern reales y positivas si k
_
3;
18
5

.
Ejemplo 6.42 Una parcela de tierra de 520 m
2
tiene forma rectangular, uno de sus lados
constituye el 65 % del otro. Hallar estos lados.
Solucin
Sabemos que el rea del rectngulo est dado por A = xy. Como y = 0, 65x, entonces 520 = 0, 65x
2
despejando x, obtenemos
x =
_
520
0, 65
x = 20

2
Remplazamos x para obtener y
y = 0, 65
_
20

2
_
= 13

2.
Ejemplo 6.43 Una caja sin tapa de 24 centmetros cbicos de capacidad, se hace de una pieza
cuadrada de cartn cortando cuadrados de 2 centmetros de lado en cada esquina y doblando los
lados. Hallar las dimensiones de la pieza de cartn que se necesita.
Solucin
Sea x la dimensin pedida. La caja tendr por dimensiones (x 4) por (x 4) por 2 y su volumen
ser 2(x 4)(x 4). As, pues
2(x 4)
2
= 24 x 4 = 2

3
_
x
1
= 4 + 2

3
x
2
= 4 2

3
.
Entonces, el cuadrado de cartn necesario tiene 4 + 2

3 centmetros de lado.
CAPTULO 6. ECUACIONES Y SISTEMAS DE ECUACIONES 296
Ejemplo 6.44 Por dos tubos juntos se puede llenar un depsito en 6 horas 40 minutos. Hallar
el tiempo que cada uno solo empleara para llenar el depsito si uno de los tubos puede llenarlo en
3 horas menos que el otro.
Solucin
Sea x, el tiempo en horas que necesita el tubo menor, x3 es el tiempo que necesita el mas grande.
Entonces
1
x
, es la parte que llena en una hora el tubo menor,
1
x3
, parte que llena en una hora el
tubo mayor. Como los dos tubos juntos llenan el depsito en
3
20
del deposito en una hora.
1
x
+
1
x 3
=
3
20
20(x 3) + 20x = 3x(x 3)
3x
2
49x + 60 = (3x 4)(x 15) = 0
_
x
1
=
4
3
x
2
= 15
.
El tubo menor llenara el depsito en 15 horas y el mayor en 12 horas.
Ejemplo 6.45 Si se lanza hacia arriba un objeto con velocidad inicial v metros por segundo,
su altura s metros sobre el suelo despus de t segundos viene dada por s = vt
1
2
gt
2
. Con g = 9, 80
metros por segundos y velocidad inicial 120 metros por segundo, hallar:
a) Cundo est el objeto a 60 metros sobre el suelo.
b) En qu momento alcanza su mayor altura y cul es sta.
Solucin
La ecuacin del movimiento es s = 120t 4, 9t
2
.
a) Si s = 60, entonces
60 = 120t 4, 9t
2
4, 9t
2
120t + 60 = 0 t =
120 115
9, 8

_
t
1
= 24
t
2
=
1
2
Al cabo de t = 0, 5 segundos, el objeto est a 60 metros sobre el suelo y sigue subiendo. Despus
de t = 24 segundos el objeto est a 60 metros sobre el suelo y va cayendo.
b) El objeto est a su mxima altura cuando
t =
b
2a
=
(120)
2(4, 9)
= 12, 24 segundos
Su altura est dada por
120t 4, 9t
2
= 120(12, 24) 4, 9(12, 24)
2
= 734, 9metros.
Ejemplo 6.46 Dos personas salen simultneamente de dos ciudades A y B y van una en
direccin de la otra. La primera persona camina 2 km/hr ms de prisa que la segunda y llega a
B una hora antes que la segunda llegue a A. Si A y B distan 24 km. Cuntos kilmetros recorre
cada una de las personas en una hora?
Solucin
Sea v km/hr la velocidad de la primera persona P
A
, entonces la velocidad de la segunda persona
P
B
ser de (v 2) km/hr. La persona P
A
tardar t
1
=
24
v
horas y P
B
tardar t
2
=
24
v2
horas.
Como P
B
llega una hora ms tarde que P
A
, entonces
t
2
= t
1
+ 1
24
v 2

24
v
= 1
v
2
2v 48
v(v 2)
= 0 v = 8 v = 6
Pero v > 0, as, v = 8 km/hr. Luego la distancia que recorre P
A
en una hora es de d
1
= v 1 = 8
km y P
B
recorre d
2
= (v 2) 1 = 6 km.
CAPTULO 6. ECUACIONES Y SISTEMAS DE ECUACIONES 297
Ejemplo 6.47 Un tren rpido fue obligado a detenerse 16 minutos en un disco rojo. Para
recuperar este tiempo, viaj en un tramo de 80 kilmetros, 10 km/hr ms rpido que lo normal.
Cul es la velocidad normal del tren?
Solucin
Sea v km/hr la velocidad prevista por el tren (donde v > 0). La velocidad real fue de (v + 10)
km/hr.
El tiempo previsto era de t
1
=
80
v
hr, pero realmente t
2
=
80
v+10
hr. Por hiptesis tenemos que
t
1
t
2
=
16
60
es decir
80
v

80
v + 10
=
16
60

v
2
+ 10v 3000
v(v + 10)
= 0 v = 50 v = 60
pero v > 0 luego v = 50 y la velocidad normal del tren fue de 50 km/hr.
6.10. Tarea
1. Formar la ecuacin cuadrtica si sus races son:
a) m+n, mn;
b) m+

n, m

n;
c) 1 +

5, 1

5;
d) 2 +

5, 2

5;
e) 2 3

2, 2 + 3

2;
f ) 1 +

3, 1

3;
g) 5 2

5, 5 + 2

5;
h) 2m

3, 2m+

3;
i) 1 3m

2, 1 + 3m

2;
j)
1
m

2,
1
m
+

2;
k) 3

3
m
, 3 +

3
m
;
l)
2
n
+

2
n
,
2
n
+

2
n
;
m)
n
m
+
n

2
m
,
n
m

n

2
m
;
n)
n
3

3,
n
3
+

3;
o)
5n
2m
+
n

5
m
,
5n
2m

n

5
m
.
2. Para qu valores del coeciente M cada una de las ecuaciones tiene dos races iguales:
a) 4x
2
+Mx +9 = 0; b) x
2
2(1 +3M)x +7(3 +2M) = 0; c) Mx
2
+4x +1 = 0.
3. Qu valor tiene M si la ecuacin
a) 4x
2
+Mx + 9 = 0 tiene una raz igual a AB;
b) x
2
+Mx 18 = 0 tiene una raz igual a 3;
c) Mx
2
15x 7 = 0 tiene una raz igual a 7;
d) x
2
+Mx +A
2
+ 5A+ 6 = 0 tiene una raz igual aA+ 3?.
4. Si las races de la ecuacin Mx
2
+3xM +1 = 0 las designamos por x
1
y x
2
, qu valores
hay que dar al parmetro M para que:
a) x
1
2x
2
=
1
2
; b) 2x
1
+ 3x
2
= 2; c) x
2
1
x
2
2
= 3.
5. Si las races de la ecuacin 3x
2
+ (M 3)x + 2M = 0 las designamos por x
1
y x
2
, qu
valores hay que dar al parmetro M para que:
a) 2x
1
2x
2
= 5; b) x
1
+ 5x
2
= 3; c) x
2
2
+x
2
2
= 2.
CAPTULO 6. ECUACIONES Y SISTEMAS DE ECUACIONES 298
6. Si las races de la ecuacin (M
2
2M + 2)x
2
2x + 3 = 0 las designamos por x
1
y x
2
,
qu valores hay que dar al parmetro M para que:
a) x
2
1
4x
2
2
= 1; b) 9x
2
1
+x
2
2
= 3; c) x
3
1
x
3
2
= 1.
7. Si las races de la ecuacin (M + 2)x
2
Mx + 4 = 0 las designamos por x
1
y x
2
, qu
valores hay que dar al parmetro M para que:
a) x
3
1
+ 8x
3
2
= 1; b) x
2
1
4x
2
2
= 4; c) x
3
1
x
3
2
= 8.
8. Si las races de la ecuacin (M 1)x
2
+ 3x +M 1 = 0 las designamos por x
1
y x
2
, qu
valores hay que dar al parmetro M para que:
a) 8x
3
1
x
3
2
= 1; b) x
4
1
x
4
2
= 1; c) x
2
2
x
2
2
= 4.
9. Si las races de la ecuacin x
2
+ 3x +M = 0 las designamos por x
1
y x
2
, qu valores hay
que dar al parmetro M para que:
a) x
1
x
2
= 6; b) 3x
1
x
2
= 4; c)
x
1
x
2
=
2
5
; d) x
2
1
+x
2
2
= 34.
10. Para qu valores del trmino independiente las races de la ecuacin 3x
2
+2xA = 0 son
entre s como 2 : 3?
11. Formar la ecuacin cuadrtica, cuyas races son iguales a (x
1
+ x
2
)
2
y (x
1
x
2
)
2
, donde
x
1
y x
2
son races de la ecuacin Ax
2
+Bx +C = 0.
12. Conocida la ecuacin Ax
2
+ Bx + C = 0, formar una nueva ecuacin de segundo grado
cuyas races sean inversas a las races de dicha ecuacin.
13. Dada la ecuacin 4x
2
+ Mx + 50 = 0. Para qu valores de M la relacin de las races es
x
1
x
2
=
5
2
?
14. Encuentre las races reales de la ecuacin:
a) 2x(x + 6) = x
2
3x;
b) x + 2x(x 1) = 5;
c) x
2
7x + 6 = 0;
d) 3x
2
7x 1 = 0;
e) (x + 2)
2
= 2(x + 2) + 3 = 0;
f ) x(x 3) 2x(

2 x 3) = 0;
g) x
2
+ 4x 8

8 x + 20 = 0;
h) (x + 1)(x 3) 2(x +

7) = 0;
i) (x 1)(x 5) 3(x + 2) = 0;
j) 7(x
2
+ 5x + 8) = 3(x + 1)(x 2) = 0;
k)
x 5
2
+
2x 1
2 + 3x
=
5x 1
10
1
2
5
;
l)
6x 5
4x 3
=
3x + 3
2x + 5
;
m)
x
x + 2

2x
x + 1
= 1;
n)
x
x + 1

x
x 1
=
1
x
;
o)
x 1
x + 1
+
x 3
x + 3
= 1;
p)
1
x
2
25

1
x + 5
=
1
5
;
q) 1
2
2x
2
x
=
3
(2x
2
x)
2
;
r)

5x 3 +

4x + 3 = 2;
s)

x 8 +

3x 2 = 2;
t) 3x
2
5x +

3x
2
5x + 4 = 16.
CAPTULO 6. ECUACIONES Y SISTEMAS DE ECUACIONES 299
15. Encuentre las races reales de la ecuacin:
a)
_
x + 5 4

x + 1 +
_
x + 2 2

x + 1 = 1;
b)
_
x 2

x 1 +
_
x + 3 4

x 1 = 1;
c)
_
x + 2 + 2

x + 1 +
_
x + 2 2

x + 1 = 2;
d)

1 4x + 4x
2
+

1 + 4x + 4x
2
= 2;
e)
_
[[x[ 4[ +
1
4
+
_
[4 [x[[ +
17
4
= 4.
16. Encuentre las races reales de la ecuacin:
a) [x
2
3x + 3[ = 2;
b) [2x x
2
+ 3[ = 2x;
c) [x
2
+x 1[ = 2x 1;
d) [x
2
x 3[ = x 1;
e) 2[x
2
+ 2x 5[ = x 1;
f ) x
2
+ 3[x[ + 2 = 0;
g) [x
2
1[ +x + 1 = 0;
h) [x
2
9[ +[x 2[ = 5;
i) (x+1)
2
2[x+1[ +1 = 0;
j) [x
2
4[ [9 x
2
[ = 5;
k) [x
2
9[ +[x
2
4[ = 5.
17. Dos turistas se dirigen simultneamente a una ciudad que se encuentra a la distancia de 50
km de ellos. El primero de ellos hace por hora 1.5 km ms, debido a lo cual llega a la ciudad
una hora antes. Cuntos kilmetros por hora hace cada turista?
18. La distancia entre dos ciudades por ro es de 100 km. Un barco pasa esta distancia dos
veces (hacia arriba y hacia abajo) en 9 horas 30 min. Determinar la velocidad del barco en
agua muerta o estancada, si la velocidad de la corriente es de 4 km/hora.
19. Se va a bordear un cuadro de ores rectangular de un jardn que tiene 16 x 24 metros, con
una faja de anchura uniforme que doble su rea. Hallar la anchura x de la faja.
20. Dos obreros trabajando juntos pueden cumplir una tarea dada en 20 horas. El primero de
ellos por separado puede realizar el mismo trabajo 10 horas ms rpidamente que el segundo.
En cuntas horas cada obrero por separado puede realizar la tarea?
21. Si la longitud y anchura de un rectngulo, de 2 por 4 centmetros, aumenta en la misma
cantidad, el rea del nuevo rectngulo medir el doble de la original. Cules son las dimen-
siones del nuevo rectngulo, expresadas hasta centsimos?
22. Una piscina se llena por intermedio de dos tubos en 1,5 horas; el primer tubo por separado
puede llenar la piscina dos horas antes que el segundo tubo solo. En cuntas horas cada uno
de los tubos por separado puede llenar la piscina?
23. Un agricultor estableci que con la existencia de una reserva de semillas de 22,5 toneladas
se puede plantar toda la parcela destinada a la papa. Durante la plantacin se supo que
las semillas eran selectas y por eso se puede disminuir la norma de plantacin propuesta,
aproximadamente, 200 kg por hectrea. Esto condujo al aumento de la supercie de siembra
en 1 hectrea. Cul ha sido la norma de siembra de papa proyectada por hectrea y cul es
la supercie de la parcela inicial?
CAPTULO 6. ECUACIONES Y SISTEMAS DE ECUACIONES 300
24. La distancia entre dos estaciones ferroviarias es de 100 km. El tren rpido recorre este
camino 45 minutos ms rpidamente que el tren de pasajeros ordinario. Hallar la velocidad
de cada tren, si se sabe que la diferencia entre sus velocidades es de 20 km/hora.
25. El interior de una caja cbica se tapiza de material aislante de
1
2
centmetro de espesor.
Hallar la primitiva dimensin interna sabiendo que el volumen ha bajado en 271 centmetros
cbicos.
26. Un turista sali de A a B y hace un promedio de 8 km/hora. Cuando ste recorri 27
km, desde B a su encuentro sali otro turista, quien recorra en una hora la vigsima parte
de todo el camino de B a A y se encontr con el primero despus de tantas horas, como
kilmetros por hora el mismo hace. Determine la distancia de A a B.
27. La anchura de un rectngulo mide 8 pulgadas menos que su longitud. Si su rea es de 33
pulgadas cuadradas, Cules son sus dimensiones?
28. Un rectngulo tiene su longitud 7 centmetros mayor que su ancho; siendo su rea 228
centmetros cuadrados, cules son las dimensiones?
29. Dos mangueras pueden llenar un depsito en 4 horas, cuando se usan ambas al mismo
tiempo. Cuntas horas se necesitaran para que cada manguera por si sola llene el depsito,
si la de menor dimetro tarda 3 horas mas que la de mayor dimetro?
30. La presin p, en libras por pie cuadrado, del viento que sopla a v millas/hora se determina
por medio de la frmula p = 0, 003v
2
. Si el medidor de dicha presin, en un puente, registra
una presin del viento de 14,7 libras/pie2, Cul es la velocidad del viento?
31. Una prensa de imprenta nueva puede hacer un trabajo en 1 hora menos que otra, ms an-
tigua. Juntas, pueden realizar el mismo trabajo en 1,2 horas. Cunto tiempo tardara cada
una sola en efectuar dicho trabajo?
32. Una lancha rpida tarda 1 hora mas en viajar 24 km contra la corriente de un ri que en
el viaje de regreso. Si la lancha viaja a 10 km/h en agua tranquila, Cul es la velocidad de
la corriente en su viaje de 24 km?
33. Cules son las dimensiones del mayor campo rectangular que se puede cercar con 1200
metros de valla?
34. Aproximadamente a que distancia estar el horizonte de un avin que vuela a 2 millas de
altura? Suponemos que el radio de la tierra mide 4000 millas.
CAPTULO 6. ECUACIONES Y SISTEMAS DE ECUACIONES 301
35. Dos embarcaciones se separan perpendicularmente una de la otra, al partir al mismo tiem-
po del mismo muelle; 1 hora despus, estn a 13 km de distancia. Si una de ellas viaja 7
km/h mas aprisa que la otra, Cul es la velocidad de cada una?
36. Una bandera tiene una cruz blanca, de anchura uniforme, sobre fondo rojo. Encuentre la
anchura de dicha cruz, que abarque exactamente la mitad del rea total, dado que la bandera
mide 4 por 3 pies.
37. A 20 millas/hora, un automvil choca con un objeto estacionario, con la misma fuerza que
tendra si hubiera cado 13,5 pies; es decir, como si lo hubieran arrojado de la azotea de una
casa ordinaria de una sola planta. En general, un auto que se mueve a r millas/hora golpea
a un objeto estacionario con una fuerza de impacto equivalente a la que ejercera, al caer
de cierta altura a, dada por la formula a = 0, 0336r
2
. A que velocidad, aproximadamente,
deber desplazarse un auto si se estrellara con tanta fuerza como si hubiera sido arrojado de
un edicio de 12 pisos; es decir, desde 121 pies de altura?
38. En una ciudad, en un da determinado, la ecuacin de la demanda de gasolina es d =
900
p
, y
la ecuacin de la oferta es s = p80, donde d y s denotan el nmero de galones demandados
y suministrados, respectivamente (en millares), al precio de p centavos de dlar por galn.
Encuentre el precio al que la oferta resulta igual a la demanda.
39. Dos turistas A y B salieron simultneamente de distintos lugares al encuentro mutuo. Al
encontrarse result que A recorri 210 km ms que B. Si cada uno de ellos contina su camino
a la velocidad anterior, A llegar al lugar de salida de B despus de 4 das, y B llegar al
lugar de salida de A despus de 9 das. Cuntos kilmetros recorri cada uno de ellos hasta
el encuentro?
40. Si se arroja una echa verticalmente en el aire (desde el suelo), con una velocidad inicial
de 176 pie/seg, su altura y respecto del suelo, t segundos despus de haberla arrojado (sin
tomar en cuenta la resistencia del aire), esta dada por y = 176t 16t2:
a) Encuentre el tiempo en que y = 0, e interprete fsicamente este resultado.
b) Encuentre los tiempos en que y = 1 pies de altura.
41. El mnimo nmero de pies, d, necesarios para obtener, en las mejores condiciones posibles,
a un auto que viaje a una velocidad de v millas/hora, ya incluido el tiempo de reaccin, esta
dado por la frmula d = 0, 044v2 + 1, 1v. Calcule la velocidad de un auto que necesita 165
pies para detenerse, despus de haberse advertido el peligro.
6.11. Ecuacin simtrica de tercer y cuarto grados
Una ecuacin algebraica de tercer grado, se denomina simtrica, si tiene por expresin
Ax
3
+Bx
2
+Bx +A = 0 (A ,= 0)
CAPTULO 6. ECUACIONES Y SISTEMAS DE ECUACIONES 302
Transformemos el polinomio Ax
3
+ Bx
2
+ Bx + A, empleando con este n el mtodo de descom-
posicin de un polinomio en factores. Es evidente que se verican las siguientes igualdades:
Ax
3
+Bx
2
+Bx +A = A(x
3
+ 1) +Bx(x + 1)
= A(x + 1)(x
2
x + 1) +Bx(x + 1)
= (x + 1)[A(x
2
x + 1) +Bx]
= (x + 1)[Ax
2
+ (B A)x +A]
por lo cual la ecuacin Ax
3
+Bx
2
+Bx +A = 0 es equivalente a la ecuacin
(x + 1)[Ax
2
+ (B A)x +A] = 0 (A ,= 0)
Esta ecuacin es, a su vez, equivalente al sistema de ecuaciones:
_
x + 1 = 0
Ax
2
+ (B A)x +A = 0, A ,= 0
Por consiguiente, la ecuacin Ax
3
+Bx
2
+Bx +A = 0 es tambin equivalente a este sistema. La
solucin de este sistema se halla con facilidad, puesto que sta contiene solamente ecuaciones de
primer y segundo grados.
Ejemplo 6.48 Resuelva las siguientes ecuaciones:
a) x
3
2(x + 1) = x; b) 3x
3
3x(x 1) = 7x
2
; c) x
3
x
2
+x 1 = 0.
Solucin
a) x
3
2(x + 1) = x x
3
x 2(x + 1) = 0 x(x
2
1) 2(x + 1) = 0
x(x 1)(x + 1) 2(x + 1) = 0 (x + 1)(x
2
x 2) = 0
(x + 1)(x + 1)(x 2) = 0
Por lo tanto encontramos que x
1
= 1 y x
2
= 2.
b) 3x
3
3x(x 1) = 7x
2
(3x
3
7x
2
) 3x(x 1) = 0
x(3x
2
7x) 3x(x 1) = 0 x(3x
2
10x + 3) = 0 x(3x 1)(x 3) = 0
De esta ecuacin, encontramos x
1
= 0, x
2
=
1
3
y x
3
= 3.
c) (x
3
x
2
) + (x 1) = 0 x
2
(x 1) + (x 1) = 0 (x
2
+ 1)(x 1) = 0
De donde se concluye que la solucin de la ecuacin est dada por x = 1, ya que la ecuacin
x
2
+ 1 = 0 no tiene races reales.
Una ecuacin algebraica de cuarto grado se denomina simtrica, si tiene por expresin
Ax
4
+Bx
3
+Cx
2
+Bx +A = 0, (A ,= 0)
Teniendo en cuenta que A ,= 0, escribamos esta ecuacin en la forma equivalente:
(x
4
+ 1) +
B
A
x(x
2
+ 1) +
C
A
x
2
= 0, (A ,= 0).
Es evidente la validez de la siguiente igualdad:
(x
4
+ 1) +
B
A
x(x
2
+ 1) +
C
A
x
2
= (x
4
+ 2x
2
+ 1) +
B
A
x(x
2
+ 1) +
_
C
A
2
_
x
2
CAPTULO 6. ECUACIONES Y SISTEMAS DE ECUACIONES 303
= (x
2
+ 1)
2
+ 2(x
2
+ 1)
Bx
2A
+
_
Bx
2A
_
2
+x
2
_
C
A
2
B
2
4A
2
_
=
_
x
2
+
B
2A
x + 1
_
2

B
2
4A(C 2A)
4A
2
x
2
.
La validez de esta igualdad predetermina que la ecuacin simtrica es equivalente a la ecuacin
_
x
2
+
B
2A
x + 1
_
2

B
2
4A(C 2A)
4A
2
x
2
= 0, A ,= 0.
Segn sea el nmero B
2
4A(C 2A), son posibles tres casos:
CASO 1. B
2
4A(C 2A) < 0:
La ltima ecuacin y, por lo tanto, la ecuacin equivalente a ella, no tienen races reales.
CASO 2. B
2
4A(C 2A) = 0:
La ltima ecuacin adquiere en este caso la forma
_
x
2
+
B
2A
x + 1
_
2
= 0
Es evidente que esta ecuacin es equivalente a la ecuacin
x
2
+
B
2A
x + 1 = 0
Por consiguiente, el conjunto de races de la ecuacin simtrica de cuarto grado coincide en este
caso con el conjunto de races de la ecuacin cuadrtica
x
2
+
B
2A
x + 1 = 0, A ,= 0.
CASO 3. B
2
4A(C 2A) > 0:
La ltima ecuacin y, por lo tanto, la ecuacin equivalente a ella, son equivalentes al sistema de
ecuaciones cuadrticas
_

_
x
2
+
B +
_
B
2
4A(C 2A)
2A
x + 1 = 0, A ,= 0
x
2
+
B
_
B
2
4A(C 2A)
2A
x + 1 = 0, A ,= 0
cada una de las cuales se resuelve con facilidad.
Ejemplo 6.49 Resuelva las siguientes ecuaciones:
a) x
4
x(x
2
x + 1) = 0; b) (x
2
5x + 7) 2(x 2)(x 3) = 1.
Solucin
a) x
4
x
3
+x
2
x = 0 x
3
(x 1) +x(x 1) = 0
(x
3
+x)(x 1) = 0 x(x
2
+ 1)(x 1) = 0.
Como x
2
+ 1 = 0 no tiene races reales, entonces x
1
= 0 y x
2
= 1 son las races de la ecuacin.
b) x
4
10x
3
+ 37x
2
60x + 36 = 0 (x 2)
2
(x 3)
2
= 0
De aqu se obtiene x
1
= 2 y x
2
= 3.
6.12. Tarea
1. Encuentre las races reales de la ecuacin:
CAPTULO 6. ECUACIONES Y SISTEMAS DE ECUACIONES 304
a)
x
2
1
2x
+
1
x
=
2(x
2
1)
x
;
b)
x
2
4
x
2

x
2
= 1
x
2
4
x
;
c)
x
2
7x + 10
x
2
7x + 12
=
x
2
+ 3x 10
x
2
+ 3x 8
;
d)
x
2
2
+
2
x
2
=
5(2 x
2
)
2x
;
e)
3 5x
x + 2
= 2 +
x 11
x + 4
;
f ) x
2
+ 4x
7
x
2
+ 4x + 5
= 1;
g)
x + 1
x + 3
+
4
x + 7
= 1;
h)
1
x 8
+
1
x 6
+
1
x + 6
+
1
x + 8
= 0;
i)
3
x + 1
+
7
x + 2
=
6
x 1
;
j)
x 1
x
2
+
x + 1
2
=
1
x
2
;
k)
2x + 1
x + 3
+
x
x + 1
=
1
x + 1
;
l)
2
x 14

5
x 13
=
2
x 9

5
x 11
;
m)
x
2
x 1
x
2
+x + 1
+ 1 =
2x + 1
x + 3
;
n)
x
x 1
+
1
x + 1
+
1
x
=
x 1
x
.
2. Encuentre las races reales de la ecuacin:
a)
1
x + 2

x + 1
x
2
x 1
= 1
x + 1
x 1
;
b)
1
2x 1

1
2x + 1
+
1
3x 1

1
3x + 1
= 0;
c)
1
x 1

4
x 2
+
4
x 3

1
x 4
=
1
30
;
d)
x
x 2
+
1
x + 1

x + 1
x 1

x 1
x
= 0;
e) (x + 9)(x 1)(2x
2
+ 16x 20) = 12;
f ) (x
2
+ 5x 7)(2x
2
+ 10x 11) + 1 = 0;
g) (1 x)(x + 2)(x + 3) = 9x
2
x
3
+ 4(1 + 7x).
6.13. Ecuaciones de orden superior
Una ecuacin algebraica se llama binomia, si tiene por expresin
x
n
A = 0
Primeramente examinemos la ecuacin binomia en el caso particular cuando A = 1:
x
n
1 = 0
Para n = 1 esta ecuacin es un caso particular de la ecuacin de primer grado y por ello tiene
la nica raz x
1
= 1. Cuando n = 2, esta ecuacin representa un caso particular de la ecuacin
cuadrtica con discriminante positivo, por lo cual tiene solamente dos races: x
1
= 1 y x
2
= 1.
Mostremos ahora que para n 3, para cualquier n impar, esta ecuacin tiene una sola raz real
x
1
= 1, y para todo n par esta ecuacin tiene solamente dos races reales x
1
= 1 y x
2
= 1.
Sea n un nmero natural impar jo, n 3, es decir, sea n = 2k + 1, donde k es un nmero
natural jo.
Aprovechando la frmula de multiplicacin reducida, obtenemos la validez de la igualdad:
x
2k+1
1 = (x 1)(x
2k
+x
2k1
+... +x
2
+x + 1).
CAPTULO 6. ECUACIONES Y SISTEMAS DE ECUACIONES 305
De la validez de esta igualdad se desprende que, para n = 2k + 1, la ecuacin binomia es
equivalente al sistema de ecuaciones
_
x 1 = 0
x
2k
+x
2k1
+... +x
2
+x + 1 = 0
La primera ecuacin de este sistema tiene la nica raz x = 1, la segunda ecuacin del sistema
no tiene races reales. Con el n de demostrarlo, mostremos que para cualquier x real se verica la
desigualdad
x
2k
+x
2k1
+... +x
2
+x + 1 > 0
En efecto, para cualquier x [1; 0), al escribir el primer miembro de la desigualdad en la
forma
x
2k
+x
2k2
(x + 1) +... +x
2
(x + 1) + (x + 1)
nos convencemos de que el primer sumando de esta suma es positivo y los dems, no negativos.
Quiere decir, para cualquier x [1; 0) la desigualdad es vlida.
Escribiendo el primer miembro de la desigualdad en la forma
x
2k1
(x + 1) +x
2k3
(x + 1) +... +x(x + 1) + 1
nos convencemos de que para cualquier x (; 1) todos los sumandos de esta suma son posi-
tivos. Quiere decir, para todo x (; 1) la desigualdad es vlida.
As pues, se ha demostrado la validez de la desigualdad para cualquier x real y esto signica
que la ecuacin
x
2k
+x
2k1
+... +x
2
+x + 1 = 0
no tiene races reales. Por tanto, la ecuacin tiene, para n = 2k + 1, una sola raz real x
1
= 1.
Sea ahora n = 2k, k es un nmero natural jo y k 2. Aprovechando la frmula de multipli-
cacin reducida, llegamos a que se verica la igualdad idntica
x
2k
1 = (x
2
1)[x
2
(k 1) +x
2
(k 2) +... +x
4
+x
2
+ 1].
Por cuanto esta igualdad idntica es valida, resulta que la ecuacin binomia es equivalente, para
n = 2k (k 2), al sistema de ecuaciones
_
x
2
1 = 0
x
2(k1)
+x
2(k2)
+... +x
4
+x
2
+ 1 = 0
La primera ecuacin de este sistema tiene dos races, x
1
= 1 y x
2
= 1, mientras que la
segunda ecuacin no tiene races reales, puesto que para cualquier x real se verica, evidentemente,
la desigualdad
x
2(k1)
+x
2(k2)
+... +x
4
+x
2
+ 1 > 0.
Por consiguiente, para n = 2k, la ecuacin binomia tiene dos races reales: x
1
= 1 y x
2
= 1.
As pues, cualquiera que sea n impar, la ecuacin binomia tiene una sola raz real x
1
= 1, y para
cualquier n par, solamente dos races reales: x
1
= 1 y x
2
= 1.
Razonando anlogamente, podemos mostrar que:
CAPTULO 6. ECUACIONES Y SISTEMAS DE ECUACIONES 306
1. Para cualquier a positivo, la ecuacin binomia tiene:
a) Una sola raz real x
1
=
n

A, para cualquier n impar;


b) Solamente dos races reales, x
1
=
n

A y x
2
=
n

A, para cualquier n par.


2. Cuando A = 0, la ecuacin binomia tiene una sola raz x
1
= 0.
3. Para cualquier A negativo se puede mostrar que la ecuacin binomia tiene:
a) Una sola raz real, x
1
=
n

A, para cualquier n impar;


b) No tiene races reales, cualquiera que sea n par.
La ecuacin algebraica de la forma
Ax
2n
+Bx
n
+C = 0
se denomina trinomia a condicin de que n 2, A ,= 0, B ,= 0, C ,= 0. Cuando n = 2, la ecuacin
trinomia se llama, adems ecuacin bicuadrada.
Al resolver la ecuacin bicuadrada
Ax
4
+Bx
2
+C = 0, A ,= 0
su primer miembro se transforma por el mtodo de formacin de cuadrado perfecto:
Ax
4
+Bx
2
+C = A
__
x
4
+ 2x
2

B
2A
+
_
B
2A
_
2
+
C
A

B
2
4A
2
__
= A
_
_
x
2
+
B
2A
_
2

B
2
4AC
4A
2
_
En virtud de esta igualdad la ecuacin (17) es equivalente a la siguiente
A
_
_
x
2
+
B
2A
_
2

B
2
4AC
4A
2
_
, A ,= 0
Es evidente que si B
2
4AC < 0, la ecuacin (18) y, por lo tanto, la ecuacin (17), equivalente
a la (18), no tienen races.
Cuando B
2
4AC = 0, la ecuacin (18) adquiere la forma
_
x
2
+
B
2A
_
2
= 0, A ,= 0
La ecuacin (19) es, obviamente, equivalente a la ecuacin
x
2
+
B
2A
= 0, A ,= 0
De este modo, cuando B
2
4AC = 0, la ecuacin bicuadrada equivalente a la ecuacin cuadrtica,
es decir, para
B
2A
< 0 tiene tan slo dos races reales, x
1
=
_

B
2A
y x
2
=
_

B
2A
; para
B
2A
= 0,
la nica raz x
1
= 0; para
B
2A
> 0, no tiene races.
CAPTULO 6. ECUACIONES Y SISTEMAS DE ECUACIONES 307
En cambio, si B
2
4AC > 0, la ecuacin (18) y, por consiguiente, la ecuacin (17), que es
equivalente a (18), son equivalentes al sistema de ecuaciones
_
x
2
+
B
2A

B
2
4AC
2A
= 0, A ,= 0
x
2
+
B
2A
+

B
2
4AC
2A
= 0, A ,= 0
Escribamos este sistema en la forma equivalente
_
x
2
=
B+

B
2
4AC
2A
, A ,= 0
x
2
=
B

B
2
4AC
2A
, A ,= 0
(6.1)
Por cuanto los nmeros que guran en los segundos miembros de las ecuaciones del sistema (21)
son races de la ecuacin cuadrtica
At
2
+Bt +C = 0, A ,= 0 (6.2)
que tiene discriminante positivo = B
2
4AC, entonces el sistema de ecuaciones (21) puede ser
escrito en la forma:
_
x
2
= t
1
, A ,= 0
x
2
= t
2
, A ,= 0
(6.3)
donde t
1
y t
2
son races de la ecuacin (22).
Cuando n > 2, para resolver la ecuacin trinomia
Ax
2n
+Bx
n
+C = 0, A ,= 0
el primer miembro de sta tambin se transforma por el mtodo de formacin de cuadrado perfecto
Ax
2n
+Bx
n
+C = A
_
_
x
n
+
B
2A
_
2

B
2
4AC
4A
2
_
(6.4)
En virtud de esta igualdad, la ecuacin (21) es equivalente a la ecuacin
_
x
n
+
B
2A
_
2
=
B
2
4AC
4A
2
, A ,= 0 (6.5)
Es evidente que si B
2
4AC < 0, la ecuacin (25), y, por tanto, la ecuacin (21) no tiene races.
Si B
2
4AC = 0, la ecuacin (25) es equivalente a la ecuacin binomia
x
n
+
B
2A
= 0, A ,= 0 (6.6)
Por consiguiente, cuando B
2
4AC = 0, la ecuacin trinomia (16) es equivalente a la ecuacin
binomia (26), cuya resolucin fue examinada en el punto anterior.
En cambio, si B
2
4AC > 0, la ecuacin (25) es equivalente al sistema de ecuaciones binomias
_
x
n
+
B
2A

B
2
4AC
2A
= 0, A ,= 0
x
n
+
B
2A
+

B
2
4AC
2A
= 0, A ,= 0
(6.7)
cuya ecuacin, como se mostr ms arriba, puede ser determinada.
CAPTULO 6. ECUACIONES Y SISTEMAS DE ECUACIONES 308
Ejemplo 6.50 Resuelva las siguientes ecuaciones:
a) x
4
+ 2x
2
8 = 0; b) x
5
+x
3
= x
4
; c) x
6
3x
3
+ 2 = 0.
Solucin
a) Si x
2
= t, la ecuacin se transforma en la siguiente
t
2
+ 2t 8 = 0 (t + 4)(t 2) = 0
_
t = 4
t = 2
Reemplazando el cambio original, obtenemos:
_
x
2
= 4
x
2
= 2

_
x =

4, x =

4
x =

2, x =

2
La ecuacin original tiene dos races complejas y dos races reales.
b) x
5
x
4
+x
3
= 0 x
3
(x
2
x + 1) = 0
_
x = 0
x
2
x + 1 = 0
x = 0.
c) Haciendo x
3
= t, obtenemos
t
2
3t + 2 = 0 (t 1)(t 2) = 0
_
t = 1
t = 2
Volviendo a la variable original, obtenemos
_
x
3
1 = 0
x
3
2 = 0

_
(x 1)(x
2
+x + 1) = 0
(x

2)(x
2
+

2 x + 2) = 0

_
x = 1
x =

2
Ejemplo 6.51 Resuelva las siguientes ecuaciones:
a) x
9
2x
5
+x = 0; b) (x
4
+x
2
+ 1)(x
4
+x
2
+ 2) = 12.
Solucin
a) x(x
8
2x
4
+ 1) = 0
_
x = 0
x
8
2x
4
+ 1 = 0

_
x = 0
(x
4
1)
2
= 0
_
x = 0
(x
2
1)(x
2
+ 1) = 0

_
x = 0
(x 1)(x + 1) = 0

_
x = 0
x = 1
x = 1
b) (x
4
+x
2
+ 1)(x
4
+x
2
+ 2) 12 = 0 x
8
+ 2x
6
+ 4x
4
+ 3x
2
10 = 0
(x + 1)(x 1)(x
2
+ 2)(x
4
+x
2
+ 5) = 0
_
x + 1 = 0
x 1 = 0

_
x = 1
x = 1
6.14. Tarea
1. Encuentre las races reales de la ecuacin:
CAPTULO 6. ECUACIONES Y SISTEMAS DE ECUACIONES 309
a) x
3
+ 2x
2
+ 3x + 6 = 0;
b) x
4
+x
3
+ 3x
2
+ 2x + 2 = 0;
c) x
3
+ 4x
2
24 = 0;
d) x
6
9x
3
+ 8 = 0;
e)
x
2
x
x
2
x + 1

x
2
x + 2
x
2
x 2
= 1;
f ) 21x
3
+x
2
5x 1 = 0;
g) 4x
3
+ 10x
2
14x 5 = 0;
h) 3x
4
2x
3
+ 4x
2
4x + 12 = 0;
i) x
2
+
9x
2
(x + 3)
2
= 27;
j) x
3
+ 9x
2
+ 23x + 15 = 0;
k) (x 1)
3
+ (2x + 3)
3
= 27x
3
+ 8;
l) 2x
4
21x
3
+ 74x
2
105x + 50 = 0;
m) x
4
+ 5x
3
+ 4x
2
24x 24 = 0;
n) x
5
4x
4
+ 4x
3
x
2
+ 4x 4 = 0;
o) x
3
x
2

8
x
3
x
2
= 2;
p) (x + 1)(x
2
+ 2) + (x + 2)(x
2
+ 1) = 2;
q) 3
_
x +
1
x
2
_
7
_
1 +
1
x
_
= 0;
r)
(3 +x)(2 +x)(1 +x)
(3 x)(2 x)(1 x)
= 35;
s)
x 2
x 1
+
x + 2
x + 1
=
x 4
x 3
+
x + 4
x + 3

28
15
;
t) 2x
4
x
3
+ 5x
2
x + 3 = 0;
u) 2x
4
4x
3
+ 13x
2
6x + 15 = 0;
v) x
8
15x
4
16 = 0;
w) (x
2
5x + 7)
2
(x 2)(x 3) = 1;
x) x(x 1)(x 2)(x 3) = 15;
y)
x
2
+ 1
x
+
x
x
2
+ 1
= 2, 9;
z)
3
1 +x +x
2
= 3 x x
2
;
2. Encuentre las races reales de la ecuacin:
a) (2x
2
+ 6x 20)(3x
2
14x 5) = 0;
b) (x
3
2x
2
3x + 4)(x
3
7x 6) = 0;
c) (x
6
1)(x
6
9x
4
x
2
+ 9) = 0;
d) (x
2
1)(x
3
+ 5x
2
3x 15) = 0;
e) (x 1)x(x + 1)(x + 2) = 24;
f ) (x + 1)(x + 2)(x + 3)(x + 4) = 3;
g) (8x + 7)
2
(4x + 3)(x + 1) = 4, 5;
h) (x 4, 5)
4
+ (x 5, 5)
4
= 1;
i) (x + 3)
4
+ (x + 5)
4
= 16;
j) 10x
3
3x
2
2x + 1 = 0;
k) 4x
3
3x 1 = 0;
l) 38x
3
+ 7x
2
8x 1 = 0;
m) 4x
3
+ 6x
2
+ 4x + 1 = 0;
n)
x
2
3
+
48
x
2
= 5
_
x
3
+
4
x
_
;
o) 2
_
x
2
+
1
x
2
_
7
_
x +
1
x
_
+ 9 = 0;
p) 4x
2
+ 12x +
12
x
+
4
x
2
= 47;
q) x
2
+x +x
1
+x
2
= 4;
r) 16x
3
28x
2
+ 4x + 3 = 0;
s)
x
2
6x 9
x
=
x
2
4x 9
x
2
6x 9
;
t) (x
4
+ 2x
2
+ 1)(x
4
2x
2
+ 1) = 0;
u) x
5
+ 4x
4
6x
3
24x
2
27x 108 = 0;
v) (x
2
2x 5)
2
2(x
2
2x 3) 4 = 0;
w) (x
3
1)(x
4
+x
3
7x
2
x + 6) = 0;
x) (x
4
1)(x
6
+ 4x
4
x
2
4) = 0;
y) (4x
2
8x + 3)(3x
3
2x
2
7x 2) = 0.
3. Encuentre las races reales de la ecuacin:
a) (x
2
+x + 4)
2
+ 8x(x
2
+x + 4) + 15x
2
= 0;
b)
1
x
2
3x + 3
+
2
x
2
3x + 4
=
6
x
2
3x + 5
;
c) (3x
3
+ 7x 10)(8x3 14x
2
+ 19x 4) = 0;
d) 4x
3
+ 3x
2
5(4x + 3) = 2x
3
5x(2 + 5x x
3
);
e) (x
2
+ 4x)(x
2
+x 6) = (x
3
9x)(x
2
+ 2x 8);
f ) (3x
2
7x + 2)(x
2
9) = (2x
2
5x 3)(9x
2
6x + 1);
g) (x
3
+ 2x
2
5x 6)(x
4
+ 4x
3
x
2
16x 12) = 0;
h) (x
3
+x
2
5x 5)(125x
3
50x
2
25x + 6) = 0;
i) (15x
2
+ 8x + 1)(8x
3
52x
2
+ 94x 35) = 0;
CAPTULO 6. ECUACIONES Y SISTEMAS DE ECUACIONES 310
j) (x
3
x
2
x + 1)(x
5
+x
4
13x
3
13x
2
+ 36x + 36) = 0;
k) (x
4
1)(x
5
x
4
5x
3
+ 5x
2
+ 4x 4) = 0;
l) (x
3
x
2
+x 1)(x
5
+ 2x
4
17x
3
34x
2
+ 16x + 32) = 0;
m) (x
3
x
2
+ 2x 2)(x
4
5x
2
+ 4) = 0;
n) (x
3
6x
2
+ 5x + 12)(x
5
9x
3
12x
2
52x 48) = 0.
4. Encuentre las races reales de la ecuacin:
a)

x 1 +

2x + 6 = 6;
b)
_
x
2
+x 5 +
_
x
2
+ 8x 4 = 5;
c)

x + 1 +
3

2x 6 = 2;
d)
3

x +
3

2x 3 =
3
_
12(x 1);
e)

2x 1 +
3

x 1 = 1;
f ) x
2
+ 3
_
2x
2
3x + 2 = 1, 5(x + 4);
g)
_
2x +
_
6x
2
+ 1 = x + 1;
h)
4

1 x +
4

15 +x = 2;
i) (1 +x
2
)
_
1 +x
2
= x
2
1;
j)
_
1
_
x
4
x
2
= x 1;
k)
_
2x
2
+ 3x + 5 +
_
2x
2
3x + 5 = 3x;
l)
4

x 1 + 2
3

3x + 2 = 4 +

3 x;
m)

x 1 +

2 x = 3;
n)

x + 1

9 x =

2x 12;
o)

2x + 5 +

5x + 6 =

12x + 25;
p)

x

x + 1 +

x + 9

x + 4 = 0;
q) 2x5+2
_
x
2
5x+2

x 5+2

x = 48;
r)
3
_
2x(4x
2
+ 3) 1 12x
3
+x = x
2
11;
s)
_
x
2
+ 1 +
_
x
2
8 = 3;
t) 1 x =
_
1
_
4x
2
7x
4
;
u)
5
_
(x 2)(x 32)
4
_
(x 1)(x 33) =
1;
v)
_
7 +
3
_
x
2
+ 7 = 3;
w)
_
5 +
3

x +
_
5
3

x =
3

x;
x)
_
3x
2
2x + 15 +
_
3x
2
2x + 8 = 7;
y)
_
3x
2
+ 5x + 8
_
3x
2
+ 5x + 1 = 1;
z)
_
x
2
3x + 3 +
_
x
2
3x + 6 = 3.
5. Encuentre las races reales de la ecuacin:
a) x
2
+
_
x
2
+ 20 = 22;
b)
7
_
5 x
x + 3
+
7
_
x + 3
5 x
= 2;
c)
4
_
2 x
3 +x
+
4
_
3 +x
2 x
= 2;
d) x
_
x
2
+ 15

x
4
_
x
2
+ 15 = 2;
e) x
2
4x 6 =
_
2x
2
8x + 12;
f ) 4x 3
3

x 1 = 0;
g)
_
x
2
3x + 5 +x
2
= 3x + 7;
h) x +
3

x 2 = 0;
i) x 4
3

x
2
+
3

x + 6 = 0;
j) (x + 4)(x + 1) 3
_
x
2
+ 5x + 2 = 0;
k)
_
5
_
x + 1 +
_
2x
2
+x + 3 = 1;
l)
_
x
2
x 1+
_
x
2
+x + 3 =
_
2x
2
+ 8;
m)
3

x + 24 +

12 x = 6;
n)

x + 2
3

3x + 2 = 0;
o)

x +
3

x 1 = 1;
p)
3

2 x = 1

x 1;
q)
3

x + 7 +
3

28 x = 5;
r)
3
_
x
2
1 +
3
_
x
2
+ 18 = 5.
6. Encuentre las races reales de la ecuacin:
a)
3

x + 1 +
3

x + 2 +
3

x + 3 = 0;
b)
3

x +
3

x 16 =
3

x 8;
c)
3
_
9

x + 1 +
3
_
7 +

x + 1 = 4;
d)
3
_
54 +

x +
3
_
54

x =
3

18;
e)
_
x
3
+x
2
1 +
_
x
3
+x
2
+ 2 = 3;
f ) x
3
_
35 x
3
(x +
3
_
35 +x
3
) = 30;
g) x +
_
17 x
2
+x
_
17 x
2
= 9;
h)
4

x 2 +
4

6 x =

2;
i)
4

77 +x +
4

20 x = 5;
CAPTULO 6. ECUACIONES Y SISTEMAS DE ECUACIONES 311
j)
4

97 x +
4

x = 5;
k)
5

33 x +
5

x = 3;
l) 4(

1 +x 1)(

1 x + 1) = x;
m) x +

x +

x + 2 +
_
x
2
+ 2x = 3;
n)
_
(x 1)(x 2) +
_
(x 3)(x 4) =

2;
o)

x +

x + 7 + 2
_
x
2
+ 7x = 35 2x.
7. Encuentre las races reales de la ecuacin:
a)

2x + 3 +

x + 1 = 3x + 2
_
2x
2
+ 5x + 3 16;
b)

x 1 +

x + 3 + 2
_
(x 1)(x + 3) = 4 2x;
c)
3
_
4 4x +x
2
+
3
_
49 + 14x +x
2
= 3 +
3
_
14 5x x
2
;
d)
_
x
3
4x
2
+x + 15 +
_
x
3
4x
2
x + 13 = x + 1;
e)
_
x
2
+x + 4 +
_
x
2
+x + 1 =
_
2x
2
+ 2x + 9;
f )
_
x
2
+x + 7 +
_
x
2
+x + 2 =
_
3x
2
+ 3x + 10;
g)
_
x + 2

x 1 +
_
x 2

x 1 = x 1;
h)

x
2
+ 28
2
+x
x

_
x
_
x
2
+ 28
2
x
2
= 3;
i)
_
x + 8 + 2

x + 7 +
_
x + 1

x + 7 = 4;
j)
_
x 2 +

2x 5 +
_
x + 2 + 3

2x 5 = 7

2;
k)
_
x
2
4x + 3 +
_
x
2
+ 3x 2 =
_
x
2
x;
l)
_
x
2
+x 2 +
_
x
2
+ 2x 3 =
_
x
2
3x + 2;
m)
4
_
78 +
3
_
24 +

x
4
_
84
3
_
30

x = 0;
n)

6 x +

x 2 + 2
4
_
(6 x)(x 2) = 2;
o)
5
_
(x 2)(x 32)
5
_
(x 1)(x 33) = 1;
p)

x
2
+ 66
2
+x
x

_
x
_
x
2
+ 66 x
2
= 5.
8. Encuentre las races reales de la ecuacin:
a)

x
2
x + 1

x
2
+ 2x + 1

x
2
x + 1 +

x
2
+ 2x + 1
= 1;
b)

x
2
+ 2x 1 +

x
2
2x + 1

x
2
+ 2x 1

x
2
2x + 1
= 1;
c)
x

x
2
x + 1
=
x
x
2
+x 1
;
d)

x
2
5x + 2

x
2
+ 3x + 1

x
2
5x + 2 +

x
2
+ 3x + 1
= 1;
e)

2x
2
3x + 1

x
2
+ 4x + 1

2x
2
3x + 1 +

x
2
+ 4x + 1
= 1;
f )
x

x
2
x + 1
=

x
2
+x 1
x
.
6.15. Sistemas de ecuaciones no lineales
El sistema de dos ecuaciones con dos incgnitas se llama sistema de segundo grado, si al menos
una de las ecuaciones es de segundo grado.
Resolver el sistema de ecuaciones con dos incgnitas signica hallar todos los pares de valores
de x e y que satisfagan simultneamente ambas ecuaciones. Estos pares de valores de x e y se
CAPTULO 6. ECUACIONES Y SISTEMAS DE ECUACIONES 312
llaman soluciones del sistema.
Para resolver un sistema no lineal, se aconseja despejar una incgnita de la ecuacin lineal y
sustityase en la ecuacin cuadrtica. Como esto lleva a una ecuacin cuadrtica en una incgnita,
el sistema se puede resolver siempre.
La resolucin de un sistema de dos ecuaciones de segundo grado en dos incgnitas lleva a una
ecuacin de cuarto grado en una de las incgnitas.
En ciertos casos los sistemas de ecuaciones se resuelven ms elegantemente que por el mtodo
de sustitucin, si se recurre a procedimientos especiales.
Ejemplo 6.52 Resuelva el sistema de ecuaciones:
a)
_
x +y = 11;
x
2
+xy +y
2
= 91.
b)
_
x + 2y = 1;
x
3
+ 8y
3
= 127.
c)
_
x
2
xy +y
2
= 19;
x
4
+x
2
y
2
+y
4
= 741.
Solucin
a) De la primera ecuacin, despejamos x y reemplazamos en la segunda ecuacin:
_
x = 11 y
(11 y)
2
+ (11 y)y +y
2
= 91

_
x = 11 y
y
2
11y + 30 = 0
Resolviendo la ecuacin cuadrtica, obtenemos y
1
= 6 y y
2
= 5. Sustituyendo estos valores en
la ecuacin x = 11 y, obtenemos x
1
= 5 y x
2
= 6. De esta manera la solucin del sistema de
ecuaciones est dada por (5, 6) y (6, 5).
b) Transformamos la segunda ecuacin del sistema
_
x + 2y = 1
x
3
+ (2y)
3
= 127

_
x + 2y = 1
(x + 2y)(x
2
2xy + 4y
2
) = 127
Reemplazamos la primera ecuacin en la segunda
_
x + 2y = 1
x
2
2xy + 4y
2
= 127
Despejamos x en la primera ecuacin y luego sustituimos en la segunda ecuacin
_
x = 1 2y
(1 2y)
2
2(1 2y)y + 4y
2
= 127

_
x = 1 2y
2y
2
y 21 = 0
Resolviendo la segunda ecuacin, obtenemos que y
1
= 3 y y
2
=
7
2
. Sustituyendo estos valores
en la primera ecuacin, resulta que x
1
= 7 y x
2
= 6. Por lo tanto, la solucin del sistema de
ecuaciones est dada por (7, -3) y
_
6,
7
2
_
.
c) A la segunda ecuacin le sumamos y restamos x
2
y
2
y obtenemos
_
x
2
xy +y
2
= 19
(x
2
+y
2
)
2
x
2
y
2
= 741
Despejamos x
2
+y
2
en la primera ecuacin y luego sustituimos en la segunda ecuacin
_
x
2
+y
2
= 19 +xy
(19 +xy)
2
x
2
y
2
= 741

_
x
2
+y
2
= 19 +xy
xy = 10

_
x
2
+y
2
= 29
xy = 10
CAPTULO 6. ECUACIONES Y SISTEMAS DE ECUACIONES 313
Sumamos y restamos 2xy a la primera ecuacin y obtenemos
_
(x y)
2
9 = 0
xy = 10
Despejamos x en la primera ecuacin y sustituimos en la segunda, obteniendo
_
x = y + 3, x = y 3
x =
10
y

_
x = y + 3, x = y 3
(y + 3)y = 10, (y 3)y = 10
Resolviendo la segunda ecuacin, resulta
_
x = y + 3, x = y 3
y
2
+ 3y 10 = 0, y
2
3y 10 = 0

_
x = y + 3, x = y 3
_
y
1
= 2
y
2
= 5
,
_
y
3
= 5
y
4
= 2
Sustituimos estos valores de y en la primera ecuacin y obtenemos
_

_
_
x
1
= 5
x
2
= 2
,
_
x
3
= 2
x
4
= 5
_
y
1
= 2
y
2
= 5
,
_
y
3
= 5
y
4
= 2
Finalmente, la solucin del sistema est dada por (5, 2), (-2, -5), (2, 5) y (-5, -2).
Ejemplo 6.53 Resuelva el sistema de ecuaciones:
_

_
xy + 2x +y = 7
yz + 3y + 2z = 12
xz +z + 3x = 15
Solucin
Para resolver este sistema, hacemos lo siguiente: despejamos y en la primera y segunda ecuaciones
_

_
y =
72x
x+1
y =
122x
z+3
xz +z + 3x = 15
Igualamos las dos primeras ecuaciones:
7 2x
x + 1
=
12 2x
z + 3
z = 2x 1
Sustituimos z en la tercera ecuacin y resolvemos la ecuacin obtenid:
(2x 1)x + (2x 1) + 3x = 15 x
2
+ 2x 8 = 0
_
x
1
= 4
x
2
= 2
Sustituimos estos valores en z = 2x 1 y obtenemos z
1
= 9 y z
2
= 3. Para encontrar los valores
de y, sustituimos los valores de z en la segunda ecuacin del sistema y entonces y
1
= 5 y 1. La
solucin general del sistema esta dada por (-4, -5, -9) y (2, 1, 3).
CAPTULO 6. ECUACIONES Y SISTEMAS DE ECUACIONES 314
Lo principal que renen los problemas de aplicacin, es que la condicin de un problema se
enuncia solamente en la forma de un texto, sin frmulas ni designaciones algebraicas de las incg-
nitas. Los problemas del tipo habitual, en los cuales todas las condiciones se escriben en forma
de ecuaciones, no presentan, como regla general, grandes dicultades, aunque ciertos elementos de
estos problemas causan a veces situaciones embarazosas. En lo que se reere a los problemas ms
complicados, su dicultad se explica, por lo comn, por el carcter no habitual, y necesita no slo
resolver ciertos sistemas de ecuaciones o desigualdades sino saber razonar.
En este caso resulta a menudo que los razonamientos sencillos, sin componer ecuaciones y
desigualdades, aunque sea posible componerlas, hacen llegar ms fcil y rpidamente a la meta.
Adems, a veces se puede resolver un problema por simples razonamientos y hasta ms rpido que
por los mtodos matemticos corrientes. Sin embargo, la resolucin por razonamientos simples no
siempre es rigurosa y debe completarse con clculos matemticos rigurosos.
Ejemplo 6.54 Una lmina rectangular de estao de permetro 96 cm se usa para hacer una
caja sin tapa. Para ello se recorta un cuadrado de 4 cm de lado en cada esquina y se unen los
bordes. Cules son las dimensiones de la lmina si el volumen de la caja es de 768 cm
3
?
Solucin
Sean:
x : longitud de la lmina en centmetros.
y : ancho de la lmina en centmetros.
Entonces la caja tiene las dimensiones siguientes: longitud=(x8) cm, ancho=(y8) cm, altura=4
cm.
Por hiptesis, volumen y permetro son:
_
4(x 8)(y 8) = 768
2x + 2y = 96
lo cual es equivalente a
_
(x 8)(y 8) = 192
x +y = 48
Despejando y en la segunda ecuacin y reemplazando en la primera, obtenemos
x
2
48x + 512 = 0 (x 32)(x 16) = 0 x = 32 x = 16
Notese que si x = 32, entonces y = 16 y si x = 16, entonces y = 32. Como x es la longitud; las
dimensiones de la lmina son 32 cm x 16 cm.
Ejemplo 6.55 Un obrero hace un cierto nmero de piezas idnticas en un tiempo determinado.
Si hubiera hecho 10 piezas ms cada da, habra terminado el trabajo completo en 4
1
2
das antes de
lo previsto, y si hubiese hecho 5 piezas menos cada da habra tardado 3 das ms de lo previsto.
Cuntas piezas hizo y en cunto tiempo?
Solucin
Supongamos que el obrero hace x piezas en y das. Entonces produce
x
y
piezas por da, por hiptesis,
si hubiera realizado
x
y
+10 piezas, habra completado el trabajo en y 4
1
2
das. Luego como 4
1
2
=
9
2
tenemos
_
x
y
+ 10
__
y
9
2
_
= x
La otra condicin de la ecuacin es
_
x
y
5
_
(y + 3) = x
CAPTULO 6. ECUACIONES Y SISTEMAS DE ECUACIONES 315
As, obtenemos el siguiente sistema
_
10y
9x
2y
= 45
5y +
3x
y
= 15
de donde
x
y
= 50, luego, y = 27 y x = 1350.
Ejemplo 6.56 Dos obreros tienen que hacer un trabajo consistente en mecanizar un lote de
piezas idnticas. Despus que el primero ha trabajado durante 7 horas y el segundo durante 4 horas,
han completado
5
9
del total del trabajo. Si siguieron trabajando los dos a la vez durante 4 horas ms,
les quedara por hacer
1
18
del trabajo. Cunto tardara cada uno en hacer el trabajo completo?
Solucin
Supongamos que el primer operario, trabajando solo, es capaz de completar el trabajo en x horas,
y el segundo en y horas. Entonces en una hora el primero hace
1
x
del trabajo completo y el segundo
1
y
. Por hiptesis
7
1
x
+ 4
1
y
=
5
9
Como despus trabajan juntos otras 4 horas, harn
4
x
+
4
y
del trabajo, que es igual a
1
_
5
9
+
1
18
_
=
7
18
por lo tanto tenemos la ecuacin
4
x
+
4
y
=
7
18
Restndola de la primera, obtenemos
3
x
=
3
18
de donde x = 18 e y = 24. Luego el primero tarda 18 horas y el segundo 24 horas en hacer el
trabajo completo.
Ejemplo 6.57 Se ha de transportar 690 toneladas de mercancas desde un muelle a una
estacin de ferrocarril mediante 5 camiones de 3 toneladas y 10 camiones de una tonelada y
media. En pocas horas, los camiones han transportado
25
46
de las mercancas. Para completar el
transporte a tiempo, se ha de llevar las mercancas restantes en un lapso 2 horas menor que el ya
transcurrido. Se complet el transporte gracias a que los conductores de los camiones comenzaron
a hacer un viaje por hora ms que antes. Determine cuntas horas tardaron en transportar todas
las mercancas y tambin el nmero de viajes por hora que se haca al principio sabiendo que los
camiones de una y media tonelada hacen un viaje ms por hora que los camiones de 3 toneladas.
Solucin
Supongamos que la primera parte de las mercancas que asciende a
25
46
690 = 375 toneladas se trans-
porta en x horas haciendo cada camin de 3 toneladas y viajes por hora. Entonces cada camin de
una y media tonelada har y +1 viajes por hora. Por hiptesis, la parte restante de mercancas (es
decir, 690-375=315 toneladas) se transport en x 2 horas, haciendo los camiones de 3 toneladas
y +1 viajes por hora y los camiones de una y media toneladas (y +1) +1 = y +2 viajes por hora.
Obtenemos as el siguiente sistema de ecuaciones
_
5 3xy + 10
3
2
x(y + 1) = 375
5 3(x 2)(y + 1) + 10
3
2
(x 2)(y + 2) = 315
CAPTULO 6. ECUACIONES Y SISTEMAS DE ECUACIONES 316
simplicando tenemos
_
2xy +x = 25
2xy + 3x 4y = 27
de donde
2x 4y = 2
luego 2y = x 1. Sustituyendo en la primera ecuacin obtenemos x
2
= 25, es decir, x = 5. La
primera parte de las mercancas se transport en 5 horas y la segunda parte en 5-2=3 horas.
Todas las mercancas se transportaron en 8 horas; al principio, los camiones de tres toneladas
hacan 2 viajes por hora y los de una y media toneladas, 3 viajes por hora.
Ejemplo 6.58 Una industria tiene un encargo de 810 artculos y otra de 900 artculos en el
mismo periodo de tiempo. La primera ha completado el pedido 3 das antes del plazo previsto y la
segunda 6 das antes. Cuntos artculos produce al da cada industria, sabiendo que la segunda
produce por da 4 artculos ms que la primera?.
Solucin
Sea x la produccin de artculos diaria de la primera industria, entonces la segunda produce x +4
artculos por da. La primera ha completado su pedido en
800
x
das, luego el tiempo dado para
cumplir el pedido ha sido
800
x
+ 3 das. Anlogamente tenemos que
900
x+4
+ 6 es el tiempo asignado
a la segunda industria, pero el tiempo asignado a ambas industrias es el mismo, luego
800
x
+ 3 =
900
x + 4
+ 6
de donde x = 20. Es decir, la primera industria produce 20 artculos y la segunda 24 artculos por
da.
Ejemplo 6.59 Hallar un nmero de dos cifras sabiendo que el nmero de unidades excede en
cuatro al nmero de las decenas y que el producto del nmero deseado por la suma de sus dgitos
es igual a 576.
Solucin
Si un nmero a tiene n dgitos a
0
, a
1
, a
2
, ..., a
n1
, ordenados de izquierda a derecha, entonces
a = a
n1
10
n1
+... +a
1
10 +a
0
Sea x el dgito de las unidades, y el dgito de las decenas, entonces por hiptesis se tiene la ecuacin
x = y + 4
adems el nmero deseado es 10y +x luego
(10y +x)(x +y) = 576 (11y + 4)(2y + 4) = 576 y = 4 y =
70
11
pero y > 0, luego y = 4. Por lo tanto el nmero buscado es 48.
Ejemplo 6.60 Se tienen tres mezclas compuestas de tres elementos A, B y C. La primera
mezcla consta slo de los elementos A y B en proporcin de peso de 3 : 5, la segunda mezcla
contiene solamente los elementos B y C en proporcin de peso de 1 : 2, en la tercera mezcla entran
slo los elementos A y C en proporcin de peso de 2 : 3. En qu proporcin se han de tomar estas
mezclas para que la mezcla obtenida contenga los ingredientes A, B y C en proporcin de peso de
3 : 5 : 2?
Solucin
CAPTULO 6. ECUACIONES Y SISTEMAS DE ECUACIONES 317
Ya que los elementos A y B componen la primera mezcla en proporcin de 3 : 5, entonces cada
gramo de la primera mezcla contiene
3
8
gr del elemento A y
5
8
gr del elemento B. Anlogamente,
1 gr de la segunda mezcla contiene
1
3
gr del elemento B y
2
3
gr del elemento C; 1 gr de la tercera
mezcla contiene
2
5
gr del elemento A y
3
5
gr del elemento C.
Si tomamos x gr de la primera mezcla, y gr de la segunda y z gr de la tercera y las mezclamos,
obtendremos x+y +z gr de la nueva mezcla, con lo que sta contendr
3
8
x+
2
5
z gr del elemento a,
5
8
x +
1
3
y gr del elemento B y
2
3
y +
3
5
z gr del elemento C. Tenemos que tomar la primera, segunda
y tercera mezclas en tales cantidades que la mezcla obtenida contenga los elementos A, B y C en
proporcin de 3 : 5 : 2, es decir, que 1 gr de la mezcla nueva comprenda
3
10
gr del elemento A,
5
10
gr
del elemento B y
2
10
gr del elemento C. Pues, en x+y +z gr de la mezcla nueva habr
2(x+y+z)
10
gr
del elemento A,
5(x+y+z)
10
gr del elemento B y
2(x+y+z)
10
gr del elemento C. Si igualamos diferentes
expresiones para la misma cantidad de gramos de los elementos A, B y C, obtendremos un sistema
de ecuaciones
_

_
3
8
x +
2
5
z =
3(x +y +z)
10
5
8
x +
1
3
y =
5(x +y +z)
10
2
8
y +
3
5
z =
2(x +y +z)
10
Notemos que aunque se hayan obtenido tres ecuaciones con tres variables, el sistema tiene solamente
dos ecuaciones independientes. Esto es fcil demostrar, por ejemplo, as: sustrayendo de la igualdad
x+y+z = x+y+z la suma de las dos primeras ecuaciones, obtendremos la tercera ecuacin. Por lo
tanto, del sistema de ecuaciones hallaremos no las x, y, z sino la proporcin x : y : z. Eliminando
x, por ejemplo, de las dos primeras ecuaciones del sistema, hallamos que y = 2z. Si colocamos
este valor de y en cualquier ecuacin del sistema, obtendremos que x =
20
3
z. Por consiguiente,
x : y : z = 20 : 6 : 3, es decir, hay que tomar la mezcla en proporcin de peso 20 : 6 : 3.
Ejemplo 6.61 El por ciento (por el peso) de alcohol en tres soluciones forma una progresin
geomtrica. Si se mezclan la primera, segunda y tercera soluciones en proporcin de peso de 2 : 3 : 4,
se obtendr una solucin de un 32 % de alcohol. Si estas se mezclan en proporcin de peso de
3 : 2 : 1, se obtendr una solucin de un 22 % de alcohol. Qu por ciento de alcohol contiene cada
solucin?
Solucin
En la primera solucin hay x%, en la segunda y % y en la tercera, z % de alcohol. Esto signica que
1 g de la primera solucin contiene
x
100
g de alcohol, 1 g de la segunda solucin,
y
100
g de alcohol
y 1 g de la tercera solucin,
z
100
g de alcohol. Si tomamos 2 g de la primera solucin, 3 g de la
segunda y 4 g de la tercera, obtendremos 9 g de una mezcla que contiene 2
x
100
+ 3
y
100
+ 4
z
100
g de alcohol. Segn la condicin del problema, la mezcla obtenida contiene un 32 % de alcohol, es
decir, en 9 g de la mezcla hay 9
32
100
g de alcohol. De esta condicin obtendremos una ecuacin
2x + 3y + 4z
100
= 9
32
100
Por analoga obtendremos una ecuacin ms:
3x + 2y +z
100
= 6
22
100
En n, segn la condicin del problema, los nmeros x, y, z forman una progresin geomtrica por
razn de que y
2
= xz. Ahora nos queda resolver el sistema de ecuaciones
_

_
2x + 3y + 4z = 288
3x + 2y +z = 132
y
2
= xz
CAPTULO 6. ECUACIONES Y SISTEMAS DE ECUACIONES 318
Al resolver las dos primeras ecuaciones con relacin a y y z y al poner las expresiones obtenidas en
la tercera ecuacin, obtenemos la ecuacin x
2
76x +768 = 0, cuyas races son: x
1
= 64, x
2
= 12.
Pero, el valor x
1
= 64 no satisface las condiciones del problema, porque el valor respectivo de
y = 48 2x es negativo. por eso, queda slo x = 12. En este caso se halla fcilmente: y = 24 y
z = 48. de tal modo, la primera solucin contiene el 12 % de alcohol, la segunda, 24 % y la tercera,
48 %.
Ejemplo 6.62 Un auente desemboca en un ro. A cierta distancia de la desembocadura del
auente est situado el punto A. en el ro, a la misma distancia de la desembocadura del auente
se encuentra el punto B. el tiempo necesario para que una lancha a motor navegue, de ida y vuelta,
del punto A a la desembocadura del auente, y el tiempo requerido para que sta cubra la distancia
de ida y vuelta del punto B hasta la desembocadura del auente, se reeren como 32 : 35. Si la
velocidad de la lancha a motor fuera en 2 km/h mayor, esta relacin sera igual a 15 : 16, y si la
velocidad de la lancha a motor fuera en 2 km/h menor, esta relacin sera igual a 7 : 8. Hllese
la velocidad de la corriente del ro. (Las distancias se miden a lo largo del auente y del ro,
respectivamente).
Solucin
Sea u km/h la velocidad de la corriente del ro, v km/h, la velocidad de la lancha en agua muerta
y w km/h, la velocidad de la corriente del auente. Luego, la distancia desde el punto A hasta la
desembocadura del auente es igual a s km. Entonces, para superar la va de ida y vuelta desee el
punto A hasta la desembocadura del auente la lancha necesita
t
1
=
s
v +w
+
s
v w
=
2sv
v
2
w
2
Ya que la distancia desde el punto B hasta la desembocadura del auente es tambin igual a s km,
la lancha, en su navegacin de ida y vuelta desde B hasta la desembocadura del auente, invierte
t
2
=
s
v +u
+
s
v u
=
2sv
v
2
u
2
De la condicin t
1
: t
2
= 32 : 35 obtenemos la primera ecuacin
v
2
u
2
v
2
w2
=
32
35
De modo anlogo se componen las otras dos ecuaciones
_

_
(v + 2)
2
u
2
(v + 2)
2
w
2
=
15
16
(v 2)
2
u
2
(v 2)
2
w
2
=
7
8
Despus de simplicarlo, este sistema de ecuaciones puede escribirse como:
_

_
3v
2
= 35u
2
32w
2
(v + 2)
2
= 16u
2
15w
2
(v 2)
2
= 8u
2
7w
2
De este sistema debemos hallar u. Este sistema se resuelve con facilidad si primero se elimina u,
es decir, aquella incgnita que se busca. Al eliminar u, obtenemos el sistema
_
2(v 2)2 (v + 2)
2
= w
2
35(v 2)
2
24v
2
= 11w
2
CAPTULO 6. ECUACIONES Y SISTEMAS DE ECUACIONES 319
Si de este sistema eliminamos w, obtenemos la ecuacin
13(v 2)
2
+ 11(v + 2)
2
24v
2
= 0
de donde v = 12. Ahora se deduce que w = 2 y que u = 4. de tal modo hemos obtenido la solucin:
la velocidad de la corriente del ro es de 4 km/h.
Ejemplo 6.63 Dos ros desembocan en un lago. Un barco sale del puerto M situado en el
primer ro, navega agua abajo hasta el lago atravesndolo y donde no hay ninguna corriente, y por
el segundo ro, agua arriba, contra la corriente, hasta el puerto N. Seguidamente el barco regresa.
La velocidad del barco, sin tomar en cuenta la corriente es igual a v, la velocidad de la corriente
del primer ro es v
1
; la del segundo ro es v
2
; el tiempo de movimiento del buque desde M hasta
N es igual a t, y la distancia desde M hasta N es igual a S. El tiempo de navegacin de regreso
desde N hasta M, por la misma ruta, es tambin igual a t. Qu distancia recorre el buque por el
lago en una direccin?
Solucin
Designamos por s
1
y s
2
las distancias desde los puertos M y N hasta el lago, y por s, la va que
pasa por el lago. Por la condicin del problema tenemos: s
1
+s+s
2
= S. Es evidente que el tiempo
empleado por el buque para superar la ruta de M a N, es igual a
s
1
v +v
1
+
s
v
+
s
2
v v
2
= t
anlogamente, calculamos el tiempo necesario para superar la ruta de regreso. De este modo obten-
emos el sistema de tres ecuaciones con tres incgnitas s
1
, s
2
, s:
_

_
s
1
+s +s
2
= S
s
1
v +v
1
+
s
v
+
s
2
v +v
2
= t
s
1
v v
1
+
s
v
+
s
2
v +v
2
= t
de estas incgnitas nos interesa la magnitud s.
Este sistema parece bastante complejo, aunque en principio no hay nada de eso: en realidad, si
recordamos que v, v
1
, v
2
, S, t son constantes dadas, resulta claro que este sistema es un sistema de
tres ecuaciones lineales con tres incgnitas. Y tal sistema siempre puede ser resuelto si eliminamos,
sucesivamente las incgnitas.
No obstante, ocurre con frecuencia que lo simple en la teora resulta muy complejo en la prctica. El
mtodo indicado para resolver nuestro problema es muy engorroso y presenta clculos voluminosos
porque los coecientes de este nuevo sistema son bastante complejos.
Por esta razn, vamos a resolver el sistema valindonos de un mtodo un poco articial, pero breve.
La segunda ecuacin de este sistema puede presentarse en forma
v
2
s
1
vv
2
s
1
+v
2
s + (v
1
v
2
)vs v
1
v
2
s +v
2
s
2
+vv
1
s
2
= tv(v
2
+vv
1
vv
2
v
1
v
2
)
Al sustituir la suma del primer miembro v
2
s
1
+v
2
s +v
2
s
2
por v
2
S, hay que referirse a la primera
ecuacin, y al agrupar los trminos obtenemos la ecuacin
v
2
S +v[v
1
s
2
v
2
s
1
+ (v
1
v
2
)s] v
1
v
2
s = tv(v
2
+vv
1
vv
2
v
1
v
2
)
as mismo puede transformarse tambin la tercera ecuacin de nuestro sistema. Pero los clculos
pueden economizarse si notamos que la tercera ecuacin es muy parecida a la segunda: si sustitu-
imos s
1
y v
1
de aqulla por s
2
y v
2
y a la inversa, obtendremos la segunda ecuacin. Por lo tanto,
CAPTULO 6. ECUACIONES Y SISTEMAS DE ECUACIONES 320
al sustituir s
1
y v
1
por s
2
y v
2
de la segunda ecuacin ya transformada, y a la inversa, obtendremos
la tercera ecuacin transformada
v
2
S +v[v
2
s
1
v
1
s
1
+ (v
2
v
1
)s] v
2
v
1
s = tv(v
2
+vv
2
vv
1
v
2
v
1
)
Sumando ahora las igualdades obtenidas, tendremos
2v
2
S 2v
1
v
2
s = tv(2v
2
2v
1
v
2
)
de donde se deduce que la ruta buscada, que pasa por el lago, es:
s = v
vS v
2
t +v
1
v
2
t
v
1
v
2
= vt +v
2

S vt
v
1
v
2
El problema queda completamente resuelto. Sin embargo, algunos estudiantes, al obtener la solucin
del problema con los datos algebraicos, consideran necesario aclarar con cules relaciones entre los
datos esta solucin tiene un sentido real (se superponen requerimientos de que las velocidades, las
rutas, etc., son positivas, se introducen condiciones con las cuales los denominadores son distintos
de cero, etc.) Claro est que una investigacin correcta no empeora la resolucin del problema,
pero esta investigacin no es un elemento lgicamente necesario de la resolucin, porque en la
condicin del problema se sobreentiende que todos los procesos reales descritos tenan lugar y, por
consiguiente, los datos algebraicos ya satisfacen las relaciones adecuadas. sin duda, se debe recurrir
a tal investigacin si lo exige la condicin del problema.
Ejemplo 6.64 Un automvil sale del punto a hacia el punto B. En ese mismo instante del
punto B hacia el punto A sale una motocicleta, pero a menor velocidad. Pasado cierto tiempo se
encuentran; es este momento, del punto B hacia el punto A sale una segunda motocicleta que se
encuentra con el automvil en un punto que dista del punto de encuentro de sta con la primera
motocicleta 2/9 del camino desde A hasta B. Si la velocidad del automvil fuera de 20 km/h menos,
la distancia entre los puntos de encuentro sera igual a 72 km y el primer encuentro tendra lugar
a las 3 horas despus de la partida del automvil desde el punto A. Hllese la distancia entre A y
B. (Las velocidades de las motocicletas son iguales).
Solucin
Sea u km/h la velocidad del automvil y la de la motocicleta, v km/h; sea s km la distancia AB;
el automvil y la primera motocicleta se encuentren despus de t horas. El sistema de ecuaciones
se compone fcilmente
_

_
tu +tv = s
3(u 20) + 3v = s
2
9
s
u
=
vt
2
9
s
v
72
u 20
=
3v 72
v
Si de este sistema eliminamos la incgnita complementaria t y lo simplicamos, obtenemos el
siguiente sistema
_

_
s = 3(u +v 20)
9uv = 2(u +v)
2
v(u 20) = 2(u +v 20)
2
Para hallar s hace falta buscar u y v que guren en las dos ltimas ecuaciones. Al notar que la
segunda ecuacin es la ecuacin homognea de segundo grado respecto a dos variables, hallaremos
CAPTULO 6. ECUACIONES Y SISTEMAS DE ECUACIONES 321
con facilidad la relacin u : v. Ya que nos interesan u y v, distintos de cero, obtendremos, al dividir
la segunda ecuacin entre v
2
, una ecuacin cuadrtica respecto a la nueva variable z =
u
v
:
2z
2
5z + 2 = 0
Las races de esta ecuacin son z
1
= 2 y z
2
=
1
2
, y por eso u = 2v. Poniendo este valor de u en la
tercera ecuacin hallamos que v = 40, de lo que se deduce que u = 80 y s = 300. De tal modo, la
distancia AB queda hallada: s = 300 km.
Ejemplo 6.65 Dos compaeros, al tener una sola bicicleta, partieron en el mismo instante
del punto A hacia B; el primero de ellos se fue en bicicleta y el segundo, a pie. A cierta distancia
de A el primero dej la bicicleta en el camino y lleg caminando a B. El segundo, al llegar donde
estaba la bicicleta, sigui en sta. Ambos amigos llegaron juntos a B. En el camino de regreso del
punto B al punto A procedieron de igual forma, pero el primero de ellos recorri en bicicleta un
kilmetro ms que la vez primera. Por esto, el segundo amigo lleg al punto A 21 minutos ms
tarde que el primero. Determnese la velocidad de marcha de cada uno de los amigos si en bicicleta
van a una velocidad de 20 km/h y caminando, la velocidad del primero en 3 minutos por km es
mayor que la del segundo.
Solucin
Introduzcamos las siguientes designaciones:
s km, la distancia entre los puntos A y B;
v km/h, velocidad de marcha del primer compaero;
w km/h, velocidad de marcha del segundo compaero;
a km, distancia recorrida en bicicleta por el primer compaero desde A hasta B (de tal modo, ste
dej la bicicleta en un punto que dista a km de A y sigui caminando hasta B).
Es evidente que para recorrer todo el camino de A a B, el primer amigo gast
a
20
+
sa
v
horas y el
segundo,
a
w
+
sa
20
horas. Las condiciones de simultaneidad de partida y simultaneidad de llegada
al punto B dan la primera ecuacin
a
20
+
s a
v
=
a
w
+
s a
20
Los datos sobre la marcha de los amigos desde B hasta A permiten componer, en forma anloga,
la segunda ecuacin
a + 1
20
+
s a 1
v
=
a + 1
w
+
s a 1
20

7
20
Por cuanto el primer compaero emplea
1
v
horas y el segundo,
1
w
para 1 km, respectivamente,
entonces de la condicin del problema obtenemos de inmediato la tercera ecuacin
1
w

1
v
=
1
20
As result un sistema de tres ecuaciones con cuatro incgnitas. Es imposible determinar todos los
valores de las incgnitas s, a, v y w de este sistema; en este sentido el sistema es indeterminado.
Y signica esto que no podemos resolver nuestro problema? No. Pues, lo nico que necesitamos,
es hallar dos magnitudes incgnitas: las velocidades v y w. en este sistema ellas pueden hallarse
unvocamente. con este n, restamos la primera ecuacin de la segunda y el resultado obtenido
1
w
+
1
v
=
9
20
lo analizaremos junto con la tercera ecuacin. Despus de un clculo hallamos que v = 5 km/h y
w = 4 km/h.
CAPTULO 6. ECUACIONES Y SISTEMAS DE ECUACIONES 322
Ejemplo 6.66 Un escolar gast cierta suma de dinero para comprar una cartera, un estilgrafo
y un libro. Si la cartera, el estilgrafo y el libro costarn 5, 2 y 2,5 veces ms baratos respectivamente,
la compra costara 8 dlares. Y si, en comparacin con el precio original, la cartera costar 2 veces
ms barata, el estilgrafo 4 veces y el libro 3 veces ms baratos, por la misma compra el escolar
pagara 12 dlares. Cunto vale la compra y por qu cosa se pag ms: por la cartera o por el
estilgrafo?
Solucin
Sea x el precio de la cartera; el precio del estilgrafo y y z, el precio del libro. Hay que aclarar
cuntos dlares pag el escolar por la cartera, el estilgrafo y el libro en conjunto, es decir, hallar
la suma x +y +z.
La primera ecuacin se compone partiendo de la suposicin de que la compra costara 8 dlares:
x
5
+
y
2
+
z
2, 5
= 8
Anlogamente se compone la segunda ecuacin:
x
2
+
y
4
+
z
3
= 12
Es claro que no podremos determinar todas las incgnitas de este sistema obtenido de dos ecua-
ciones con tres incgnitas, pero podemos hallar su suma, que es lo que se exige en el problema.
Para esto escribamos nuestras ecuaciones como:
_
2x + 5y + 4z = 80
6x + 3y + 4z = 144
Si se suman estas dos ecuaciones, se hallar la suma de las incgnitas: x + y + z = 28. De esta
menera se obtiene la respuesta a la primera pregunta del problema: toda la compra cuesta 28
dlares.
Ahora vamos a tratar de esclarecer qu es ms costoso: la cartera o el estilgrafo; en otras palabras,
tenemos que esclarecer cul de las desigualdades tiene lugar: x > y o y > x.
Si de la segunda ecuacin del sistema restamos la primera, obtendremos que
2x y = 32.
Es evidente que x >
y
2
, porque en caso contrario tendramos 32 = 2x y < 0. Sin embargo, la
desigualdad x >
y
2
todava no facilita la resolucin del problema. Y no la facilita porque hemos
usado mal la ecuascin. A saber: hemos utilizado solamente que la diferencia 2x y es positiva.
ahora trataremos de hacer uso del hecho de que sta es igual a 32, tomando en consideracin a la
vez que x +y +z = 28 y que todas las incgnitas son nmeros positivos por su sentido real.
Escribamos la ecuacin como: x+(xy) = 32. Ya que toda la compra cuesta 28 dlares, entonces
es notorio que x < 28, y de la ltima ecuacin se deduce que xy > 0, es deecir, la cartera es ms
cara que el estilgrafo.
Ejemplo 6.67 A las 9 a.m., del punto A hacia el punto C parte un tren rpido. En ese mismo
instante, del punto B, situado entre los puntos A y C, salen dos trenes de pasajeros, el primero
de stos va al punto A y el segundo, al punto C; las velocidades de los trenes son iguales. El tren
rpido encuentra al primer tren de pasajeros a no ms tardar de las 3 horas despus de su partida,
luego pasa por el punto B a no ms tardar de las 14 horas del mismo da, llegando por n al punto
C simultneamente con el tren de pasajeros, 12 horas despus del encuentro con el primer tren de
pasajeros. Hallar la hora de llegada del primer tren de pasajeros al punto A.
Solucin
CAPTULO 6. ECUACIONES Y SISTEMAS DE ECUACIONES 323
Sea v
1
km/h, la velocidad del tren rpido, v
2
km/h, la del de pasajeros, la distancia AB es igual a
s km. De la condicicin de que el tren rpido encuentra al primer tren de pasajeros no ms tardar
de las tres horas despus de su partida, obtenemos que
s
v
1
+v
2
3.
De la condicin de que el tren rpido pas el punto B antes de las 5 horas despus de su partida,
tenemos
s
v
1
5.
Ya que hasta el primer encuentro pasaron
s
v
1
+v
2
horas, entonces, durante el tiempo de 12 +
s
v
1
+v
2
horas el tren rpido alcanzar al segundo tren de pasajeros, por cuya razn resulta que
_
12 +
s
v
1
+v
2
_
(v
1
v
2
) = s
Nos hace falta hallar x =
s
v
2
. De ah s = xv
2
; sustituyendo esta expresin en lugar de s en las
igualdades y desigualdades precedentes y designando
v
1
v
2
por , llegamos al sistema
_

_
x 3( + 1)
x 5
x = 6(
2
1)
Muchos estudiantes no dominan este problema. En realidad, la resolucin no es tan difcil: en
este sistema hay que despejar sea x y pasar al sistema de dos desigualdades respecto a una
incgnita. Por cuanto es ms fcil, a primera vista, eliminar x, emprendemos precisamente este
camino. Sustituyendo x por 6(
2
1) en dos primeras desigualdades, obtenemos el sistema de
desigualdades
_
2
2
3 0
6
2
5 6 0
Las soluciones de la primera desigualdad son: 1
3
2
; las soluciones de la segunda:
3
2
y
2
3
. De tal modo, la solucin del sistema ser: =
3
2
y, adems, todas las dentro del
intervalo 1
2
3
. Como estamos interesados por las positivas, a la condicin del problema
le satisface el valor nico =
3
2
. Ahora hallamos con facilidad que x =
15
2
y obtenemos la solucin:
el primer tren de pasajeros llega al punto A a las 16 horas 30 minutos.
Este problema, as como otros de este tipo, admite una solucin en la que todos los datos se escriben
en forma de ecuaciones. Esto se hace introduciendo incgnitas complementarias y obteniendo un
sistema de ecuaciones, en las cuales el nmero de incgnitas es mayor que el nmero de ecuaciones.
Sin embargo, la solucin de tal sistema de ecuaciones es ms difcil que la del sistema de desigual-
dades.
Resolvamos este problema recurriendo al segundo procedimiento. Conservemos las mismas desig-
naciones. Que el tren rpido encuentre al primer tren de pasajeros despus de 3t
1
horas (t
1
0),
recorre el punto B despus de 5 +t
2
horas (t
2
0) y alcanza al segundo tren de pasajeros despus
de (3 t
1
) + 12 horas. En este caso, las ecuaciones se componen fcilmente
_

_
(v
1
+v
2
)(3 t
1
) = s
v
1
(5 +t
2
) = s
(15 t
1
)(v
1
v
2
) = s
xv
2
= s
CAPTULO 6. ECUACIONES Y SISTEMAS DE ECUACIONES 324
Si eliminamos s de este sistema y designamos
v
1
v
2
por obtenemos el sistema de ecuaciones
_

_
( + 1)(3 t
1
) = x
(5 +t
2
) = x
( 1)(15 t
1
) = x
(6.8)
Este es un sistema de tres ecuaciones con cuatro incgnitas de las cuales hay que hallar slo x.
Vamos a proceder as como lo hicimos antes: eliminamos x obteniendo un sistema
_
t
2
+ ( + 1)t
1
= 3 2
(1 )t
1
t
2
= 15 10
(6.9)
Al notar ahora que el segundo miembro de la segunda ecuacin es 5 veces mayor que el segundo
miembro de la primera, multiplicamos la primera por 5 y, al restar de sta la segunda, obtendremos
6t
2
+ (6 + 4)t
1
= 0. (6.10)
Por cuanto > 0, t
1
0, t
2
0, esta igualdad slo es posible cuando t
1
= 0 y t
2
= 0. Pero,
entonces (7) se halla fcilmente que =
3
2
, y de (6), x =
15
2
resultando la misma solucin.
Ejemplo 6.68 A las 9 a.m., de la ciudad A parti un ciclista a una velocidad constante de 12
km/h. Dos horas despus, siguiendo al primero, parti de la mismas ciudad un motociclista que iba
desplazndose con un movimiento uniformemente retardado a una velocidad inicial de 22 km/h,
de modo que su velocidad disminuia en 2 km/h. Un automovilista que iba al encuentro a ellos, a la
ciudad A, con una velocidad constante de 50 km/h, encontr primeramente al motociclista y luego,
al ciclista. Llegar el automovilista a las 19 horas de este da a la ciudad A?
Solucin
Este problema puede ser resuelto tambin mediante la composicin de ecuaciones y desigualdades.
No obstante, la composicin de tal sistema exigira largos razonamientos. Por esto, es mejor re-
solverlo no por composicin formal del sistema de ecuaciones y desigualdades, sino por un simple
razonamiento.
De la condicin del problema se inere que al principio el motociclista alcanza al ciclista, y luego
el ciclista alcanzar al motociclista. Supongamos que el ciclista demore, hasta el encuentro (no
importa, el primero o el segundo), t horas, mientras que el motociclista demore t 2 horas para
el mismo camino. Ya que hasta el encuentro ambos pasarn un camino igual, entonces, igualando
sus caminos hasta el encuentro, obtendremos que
12t = 22(t 2) 2
(t 2)
2
2
.
Una vez resuelta esta ecuacin, obtenemos que hasta el primer encuentro el ciclista demor 6 horas,
es decir, recorri 72 km, y hasta el segundo pas 96 km en 8 horas. Segn la condicin del problema,
el automovilista encontr al ciclista antes de haber pasado ste 96 km. Por eso, el automovilista
ha de ir hasta el punto A menos de 96 km. Demorar menos de frac9650 horas para recorrer
este camino. Ya que el ciclista demorar menos de 8 horas para encontrarse con el automovilista,
entonces el encuentro tendr lugar antes de las 17 horas. Es decir, despus del encuentro con el
ciclista quedan ms de 2 horas para que el automovilista llegue al punto para las 19 horas. Pero,
para superar este camino se necesita menos de
96
50
horas, es decir, menos de 2 horas. Por lo tanto,
el automovilista llegar al punto A antes de 19 horas.
Con frecuencia se proponen problemas en los cuales se exige hallar una solucin ptima rela-
cionada, por ejemplo, con una suma de dinero que se entrega para la compra de una cantidad
CAPTULO 6. ECUACIONES Y SISTEMAS DE ECUACIONES 325
mayor de piezas, o de unas cuantas variantes posibles de transporte de cargas escoger aquella que
sea ms barata que las dems, etc.
Las resoluciones de los problemas de esta ndole pueden consistir en componer sistemas de
ecuaciones y desigualdades y en resolverlos. sin embargo, los elementos ms necesarios para resolver
estos problemas son los razonamientos que ayudan mucho para elegir la mejor variante.
Ejemplo 6.69 Se requiere edicar cierto nmero de casas de vivienda iguales de un rea til
de 40 mil m
2
. Los gastos para la construccin de una casa de N m
2
de rea habitable se componen
del costo de la superestructura, proporcional a N

N, y del costo de los cimientos, proporcional a

N. La edicacin de una casa de 1600 m


2
cuesta 176,8 mil dlares con que, en este caso el costo
de la superestructura constituye un 36 % del costo de los cimientos. Determinar qu cantidad de
casas hay que construir para que la suma de gastos sea mnima y hallar esta suma.
Solucin
Supongamos que se decidi construir n casas iguales, cada una de las cuales tiene y m
2
de rea
habitable. Entonces es vlida la igualdad yn = 40000. Sea z mil dlares el costo de una casa de y
m
2
de rea habitable; entonces el costo x de toda la obra se calcula por la igualdad x = zn.
El costo de la casa se integra por el costo v de la superestructura de la casa y por el costo w de los
cimientos, es decir, z = v +w. Segn la condicin del problema, el costo de la superestructura de la
casa de y m
2
es proporcional a y

y, es decir, v = y

y, donde es un coeciente. Anlogamente


w =

y, donde es tambin un coeciente adecuado.


En particular, al construir la casa de 1600 m
2
, teniendo en cuenta que el costo de la superestructura
constituye el 36 % del costo de los cimientos, obtenemos que
1600

1600 =
36
100
( 1600)
y tomando en consideracin que la edicacin de la casa de 1600 m
2
cuesta 176,8 mil dlares,
tenemos que
176, 8 = 1600

1600 +

1600.
Tenemos escrito todos los datos del problema; ahora hay que determinar x, como funcin de n, de
las ecuaciones obtenidas y luego hallar para cul valor de n ser mnima la x.
Partiendo de las dos ltimas igualdades se hallan fcilmente y : =
117
160000
, =
13
4
. Poniendo
v y w en la expresin para z, obtenemos que
z =
117
160000
y

y +
13
4

y.
Ahora, al permutar este valor de z y el valor de y =
40000
n
de la primera igualdad a la segunda,
obtenemos que
x = 650
_
9

n
+

n
_
.
De tal manera hemos llegado a la conclusin de que x es el costo de la construccin y la funcin
n recin escrita es la cantidad de casas. Ahora tenemos que determinar el valor mnimo de x. Si
aplicamos al segundo miembro de esta igualdad la desigualdad entre la media aritmtica y la media
geomtrica, obtenemos que
x 2 650

9 = 3900.
donde el signo de igualdad se logra slo cuando
8

n
=

n, es decir, para n = 9. En otras palabras,


el costo de la obra completa ser siempre no menor que 3,9 millones de dlares y exactamente igual
a este nmero si n = 9.
Por eso, al construir las casas, la suma mnima de gastos ser cuando se construyan 9 casas; la
construccin de estas 9 casas costar 3,9 millones de dlares.
CAPTULO 6. ECUACIONES Y SISTEMAS DE ECUACIONES 326
Ejemplo 6.70 Se decici comprar por 100 dlares una cantidad de juguetes para el rbol de
Navidad. Estos adornos se venden por surtidos. El surtido de 20 juguetes cuesta 4 dlares, el de 35
juguetes, 6 dlares; y el surtido compuesto por 50 juguetes, 9 dlares. Cuntos y cules surtidos
hay que comprar para que resulte la cantidad mxima de juguetes?
Solucin
Seanx, y, z el nmero de surtidos de la primera, segunda y tercera especie, respectivamente, para
que la compra de stos asegure la mxima cantidad de juguetes (tal resolucin del problema se
considera, por lo comn, como ptima). Entonc es
4x + 6y + 9z = 100.
Esta es la nica ecuacin que puede ser compuesta segn la condicin del problema. Sin embargo,
es conocido, adems de esto, que x, y y z son nmeros enteros no negativos y que la cantidad
de juguetes de esta compra es mayor que la de cualquier otra. Resulta que estas condiciones son
totalmente sucientes para la determinacin unvoca de todas las incgnitas.
Esta es la nica ecuacin que puede ser compuesta segn la condicin del problema. Sin embargo,
es conocido, adems de esto, que x, y y z son nmeros enteros no negativos y que la cantidad
de juguetes de esta compra es mayor que la de cualquier otra. Resulta que estas condiciones son
totalmente sucientes para la determinacin unvoca de todas las incgnitas.
La primera idea que puede ocurrirse, es decir, resolver la ecuacin dada atacando de frente por se-
leccin de todos los valores posibles de incgnitas, no tiene, evidentemente, perspectivas por razn
de una enorme cantidad de casos. Sin embargo, esta seleccin puede redicirse considerablemente
con ayuda de razonamientos econmicos. En efecto por 12 dlares pueden comprarse 3 surtidos de
la primera especie 2 surtidos de la segunda especie; en el primer caso adquirimos 60 juguetes, y
en el segundo, 70. Por lo tanto, es evidente que el nmero de surtidos de la primera especie, en
cuanto a la solucin ptima, no debe superar a 2. Comparando anlogamente los surtidos de la
segunda y tercera especies, obtenemos que en la resolucin ptima no debe ser ms que un solo
surtido de la tercera especie. De tal modo, hemos obtenido las desigualdades x 2, z 1.
Ahora la seleccin no presenta dicultades. Con x = 0 obtenemos, para determinar y y z, una
ecuacin 6y + 9z = 100 que no tiene soluciones, porque su primer miembro se divide entre 3 y el
segundo no se divide. Luego, para x = 1, obtenemos una ecuacin 2y + 3z = 32 la que (teniendo
en cuenta la desigualdad z 1) tiene la solucin nica y = 16, z = 0. En n, para x = 2, as como
para x = 0, la ecuacin tampoco tiene soluciones.
6.16. Tarea
1. Resolver los sistemas de ecuaciones:
a)
_
x
2
+y
2
= 74
x y = 2
b)
_
x
2
+xy = 12
xy y
2
= 2
c)
_
y x = 2
10x +y = 3xy
d)
_
x +y = 72
3

x +
3

y = 6
e)
_
x
y

y
x
=
16
15
x y = 2
f )
_

y = A

xy = B
g)
_
x
2
+y
2
= 8
1
x
2
+
1
y
2
=
1
2
h)
_
x
y
+
y
x
=
5
2
x +xy +y = 27
i)
_
x 2y = 2
xy = 12
j)
_
x
2
+y
2
= 25
y = x
2
13
k)
_
x
2
+ 2y
2
= 34
x +y = 7
l)
_
x
2
y
+
y
2
x
= 18
x +y = 12
CAPTULO 6. ECUACIONES Y SISTEMAS DE ECUACIONES 327
m)
_
x
A
+
y
B
= 1
A
x
+
B
y
= 4
n)
_
x
2
+y
2
= 58
xy = 21
o)
_
xy(x +y) = 30
x
3
+y
3
= 35
p)
_
x +y = 5
x
3
+y
3
= 35
q)
_
x
2
+ 2y
2
3 = 0
2x
2
y
2
1 = 0
r)
_
xy = 5
2y x 3 + 0
s)
_
_
x
y
+
_
y
x
=
5
2
x +y = 10
t)
_
1
x
+
1
y
= 1
x +y = 4
u)
_

y = 2

xy
x +y = 20
v)
_
x
2
2xy y
2
= 1
x +y = 2
2. Resolver los sistemas de ecuaciones:
a)
_
x
2
+xy +y
2
= 61
x +xy +y = 29
b)
_
(x + 2)
2
+ (y 1)
2
= 25
(x + 2)(y 1) = 12
c)
_
x +y

xy = 7
x
2
+y
2
+xy = 133
d)
_
x
2
+ 3y
2
4x 5y 8 = 0
x y + 1 = 0
e)
_
2(x +y)
2
+x
2
y +xy
2
+ 30 = 0
x +xy +y = 13
f )
_
2x
2
y + 3x 5 = 0
2x y + 5 = 0
g)
_
x
2
+y
2
+x +y 62 = 0
x
2
y
2
+x y 50 = 0
h)
_
(x + 1)
2
(y + 1)
2
27xy = 0
(x
2
+ 1)(y
2
+ 1) 10xy = 0
i)
_
x
2
+xy +y
2
19 = 0
x
4
+x
2
y
2
+y
4
133 = 0
j)
_
x
2
xy + 2y
2
16 = 0
2x
2
3xy y
2
4 = 0
k)
_
x
2
+xy +y
2
3x 3y 6 = 0
xy + 4x + 4y = 0
l)
_
x+y
xy
+
xy
x+y
=
10
3
x
2
+y
2
45 = 0
m)
_
_
_
_
6x
x+y
+
_
x+y
6x
=
5
2
xy x y 9 = 0
n)
_
_
x
y
+
_
y
x

7

xy
= 1
_
x
3
y +
_
y
3
x 78 = 0
3. Resolver los sistemas de ecuaciones:
a)
_
xy 6 =
y
3
x
xy + 24 =
x
3
y
b)
_
x
2
= 13x + 4y
y
2
= 4x + 13y
c)
_
3x
2
+xy 2y
2
= 0
2x
2
3xy +y
2
= 1
d)
_
3x
2
8xy + 4y
2
= 0
5x
2
7xy 6y
2
= 0
e)
_
3x
2
2xy = 160
x
2
3xy 2y
2
= 0
f )
_
x
3
+y
3
= 1
x
3
y + 2xy
2
+y
2
= 2
g)
_
x
4
+x
2
y
2
+y
4
= 91
x
2
xy +y
2
= 7
h)
_
x
3
+x
3
y
3
+y
3
= 17
x +xy +y = 5
i)
_
x
2
+y
2
+ 6x + 2y = 0
x +y + 8 = 0
j)
_
x y = 1
x
2
+y
2
= 41
CAPTULO 6. ECUACIONES Y SISTEMAS DE ECUACIONES 328
k)
_
2x
2
3y = 23
3y
2
8x = 59
l)
_
5x
2
+ 14y = 19
7y
2
+ 10x = 17
m)
_
x
2
(x +y) = 80
x
2
(2x 3y) = 80
n)
_
x y = 2
x
3
y
3
= 8
o)
_
9x
2
+y
2
= 13
xy = 2
p)
_
x
2
+y
2
2x + 3y = 9
2x
2
+ 2y
2
+x 5y = 1
q)
_
x
2
xy y
2
+x 2y = 2
3xy 5y
2
+ 3x 6y = 5
r)
_
x y = 0, 25xy
x
2
+y
2
= 2, 5xy
s)
_
1
x+y
+
1
xy
= 2
3
x+y
+
4
xy
= 7
t)
_
4
x+y
+
4
xy
= 3
(x +y)
2
+ (x y)
2
= 20
u)
_
x+y
xy
+
xy
x+y
=
5
2
x
2
+y
2
= 20
v)
_
(x +y)
2
+ 2x = 35 2y
(x y)
2
2y = 3 2x
w)
_
12(x +y)
2
+x = 2, 5 y
6(x y)
2
+x = 0, 125 +y
x)
_
y
2
(x
2
3) +xy + 1 = 0
y
2
(3x
2
6) +xy + 2 = 0
y)
_
3
x
2
+y
2
1
+
2y
x
= 1
x
2
+y
2
+
4x
y
= 22
z)
_
6x
2
+xy 2y
2
= 0
3x
2
xy 2y
2
= 0
4. Resolver los sistemas de ecuaciones:
a)
_
56x
2
xy y
2
= 0
14x
2
+ 19xy 3y
2
= 0
b)
_
4x
2
3xy y
2
= 0
32x
2
36xy + 9y
2
= 6
c)
_
15x
2
+xy 2y
2
= 0
7x
2
4xy 3y
2
= 32
d)
_
x
2
+xy + 4y
2
= 6
3x
2
+ 8y
2
= 14
e)
_
x
2
3xy +y
2
= 1
3x
2
xy + 3y
2
= 13
f )
_
5x
2
6xy + 5y
2
= 29
7x
2
8xy + 7y
2
= 43
g)
_
x
3
y
3
= 19(x y)
x
3
+y
3
= 7(x +y)
h)
_
x
4
y
4
= 15
x
3
y xy
3
= 6
i)
_
x
4
+ 6x
2
y
2
+y
4
= 136
x
3
y +xy
3
= 30
j)
_
x
2
+xy +y
2
= 19(x y)
2
x
2
xy +y
2
= 7(x y)
k)
_
x
2
+ 4xy 2y
2
= 5(x +y)
5x
2
xy y
2
= 7(x +y)
l)
_
x
2
+y
2
= 34
x +y +xy = 23
m)
_
x +y +x
2
+y
2
= 18
xy +x
2
+y
2
= 19
n)
_
x
3
+y
3
= 19
(x + 8)(x +y) = 2
o)
_
x
2
y
+
y
2
x
= 12
1
x
+
1
y
=
1
3
p)
_
xy(x +y) = 20
1
x
+
1
y
=
5
4
q)
_
x
2
+y
2
= 7 +xy
x
3
+y
3
= 6xy 1
r)
_
x +y = 5
x
4
+y
4
= 97
s)
_
x
5
+y
5
= 33
x +y = 3
CAPTULO 6. ECUACIONES Y SISTEMAS DE ECUACIONES 329
5. Resolver los sistemas de ecuaciones:
a)
_
_
y
x
2
_
x
y
= 1

5x +y +

5x y = 4
b)
_
_
_
_
x
3
y

_
y
3
x
=
65
6
x y = 5
c)
_
y
2
+
_
3y
2
2x + 3 =
2
3
x + 5
3x 2y = 5
d)
_
5
3

x 2y + 3
3

x +y = 13
3
3

x 2y 4
3

x +y = 2
e)
_
3

2x+y
y
+
3

2x+y
2x
=
81
182
3

2xy
y

3

2xy
2x
=
1
182
f )
_

2x y + 11

3x +y 9 = 3
4

2x y + 11 +
4

3x +y 9 = 3
g)
_

x +y +
3

x y = 6
8
_
(x +y)
3
(x y)
2
= 8
h)
_
x +y +

xy = 14
x
2
+y
2
+xy = 84
i)
_
1

x
+
1

y
=
4
3
xy = 9
j)
_
x

y +y

x = 6
x
2
y +y
2
x = 20
k)
_
3

x +
3

y = 3
xy = 8
l)
_
3

x +
3

y = 3
3

x
2

xy +
3
_
y
2
= 3
m)
_
x
2
+x
3
_
xy
2
= 80
y
2
+y
3
_
x
2
y = 5
n)
_
_
_
3
_
y+1
x
2
3
_
x
y+1
= 1

x +y + 1 +

x y + 10 = 5
o)
_

x +

y + 1 = 1

x + 1 +

y = 1
p)
_
_
_
_
20y
x
=

x +y +

x y
_
16x
5y
=

x +y

x y
q)
_
3

x + 2y +
3

x y + 2 = 3
2x +y = 7
r)
_
_
x +

y +
_
x

y = 2
_
y +

x
_
y

x = 1
6. Resuelva los sistemas de ecuaciones:
a)
_

_
(x 1)(y + 5) = 14
(y + 5)(z + 8) = 63
(z + 8)(x 1) = 18
b)
_

_
x
2
+xy +xz = 48
xy +y
2
+yz = 12
xz +yz +z
2
= 84
c)
_

_
x +y +z = 19
x
2
+y
2
+z
2
= 91
y
2
xz = 0
d)
_

_
x
2
+xy +xz = 25
xy +y
2
+yz = 32
xz +yz +z
2
= 8
e)
_

_
(x +y)
2
z
2
= 65
x
2
(y +z)
2
= 13
x +y z = 5
f )
_

_
x y +z = 2
x
2
y
2
+z
2
= 6
2(xy +yz) xz = 13
g)
_

_
xy + 2x +y = 24
yz + 3y + 2z = 15
xz +x + 3z = 9
h)
_

_
4x 2y 7 = 0
x y z = 0
8x y
2
3z
2
+ 4 = 0
7. Resolver los sistemas de ecuaciones:
a)
_

xy +

yz = 9

yz +

xz = 5

xz +

xy = 8
b)
_

x +y +

y +z = 3

y +z +

x +z = 5

x +z +

x +y = 4
CAPTULO 6. ECUACIONES Y SISTEMAS DE ECUACIONES 330
c)
_

x 4 +

y +

z + 4 = 6
2

x 4

y 4

z + 4 = 12
x +y +z = 14
d)
_

4x +y 3z + 7 = 2
3

2y + 5x +z + 25, 5 = 3

y +z

6x = 0
8. La suma de las circunferencias de dos crculos es 88 centmetros y la suma de sus reas es
2200
7
centmetros cuadrados, haciendo
22
7
. Hallar los radios de los crculos.
9. Se organiza un gape que cuesta $30 y resulta que si se agregan tres ms al grupo el costo
por persona se reducira en 50 cntimos. Cuntos son los invitados originalmente?
10. Dos nmeros dieren en 2 y sus cuadrados en 48. Hallarlos.
11. Un camin sale de A a las 05:00 horas en viaje hacia B distante 300 kilmetros. A las 06:00
horas un auto parte de A, pasa al camin, entrega un paquete y vuelve a A. Si el auto hizo
una media de 60 kilmetros por hora y alcanz a llegar a A 2
1
2
horas antes que el camin
llegara a B, hallar la velocidad media del camin y averiguar a qu distancia de A pas el
auto al camin.
12. La suma de los cuadrados de las cifras de un nmero de dos cifras es igual a 10. si del
nmero buscado sustraemos 18, obtenemos un nmero escrito con esas mismas cifras, pero
en orden inverso. Hallar el nmero buscado.
Resp: 31.
13. Qu nmero de dos cifras es 4 veces mayor que la suma de sus cifras y 3 veces mayor que
el producto de ellas?
Resp: 24.
14. Hallar dos nmeros enteros, cuya suma es igual a 1244. Si al primer nmero se escribe a
la derecha la cifra 3 y del segundo se elimina la ltima cifra 2, los nmeros obtenidos sern
iguales.
Resp: 12 y 1232.
15. Un nmero de tres cifras termina con la cifra 3. Si sta se traspasa al comienzo del nmero,
el nuevo ser mayor en 1 que el nmero inicial triplicado. Hallar el nmero inicial.
Resp: 103.
16. Un nmero de seis cifras comienza por la cifra 2. Si traspasamos sta del primer puesto al
ltimo, conservando el orden de las dems, el nmero obtenido ser tres veces mayor que el
inicial. Hallar ste.
CAPTULO 6. ECUACIONES Y SISTEMAS DE ECUACIONES 331
Resp: 285714.
17. La suma de todos los nmeros pares fue dividida sin resto por uno de ellos. Hallar el divisor
si sabemos que la suma de sus cifras es igual a 9 y que el cociente se diferencia del divisor
slo por el orden de las cifras.
Resp: 54.
18. Si dividimos un nmero de dos cifras por la suma de stas, en el cociente obtendremos 7 y
en el resto 6. Si ese mismo nmero de dos cifras se divide por el producto de sus cifras, en el
cociente obtendremos 3 y en el resto, un nmero igual a la suma del nmero inicial. Hallar
el nmero inicial de dos cifras.
Resp: 83.
19. La suma de dos nmeros de tres cifras, escritos con cifras iguales, pero en orden inverso,
es igual a 1252. Hallar dichos nmeros si la suma de sus cifras es igual a 14 y la suma de los
cusdrados de ellas es 84.
Resp: 428 y 824.
20. Un turista que sube a una montaa alcanz en el transcurso de la primera hora la altura
de 800 metros, mientras que durante cada siguiente hora subi a una altura de 25 metros
menor que en la anterior. Cuntas horas pasarn hasta alcanzar la altura de 5700 metros?
Resp: En 8 horas.
21. Al dividir el noveno trmino de una progresin aritmtica por su segundo trmino, en el
cociente se obtiene 5, mientras que al dividir el trmino dcimotercero de la progresin por
su sexto trmino en el cociente tendremos 2 y en el resto 5. Hallar la suma de 20 miembros
de la progresin.
Resp: 820.
22. La suma de una progresin geomtrica ininitamente decreciente es igual a 4, en tanto que
la suma de los cubos de sus trminos, igual a 102. Hallar el primer trmino y la razn de la
progresin.
Resp: 6,
1
2
.
23. Hallar cuatro nmeros de los que los primeros tres forman una progresin aritmtica y
los tres ltimos, una geomtrica; la suma de los nmeros extremos es igual a 60 y la de los
medios, 60.
Resp: 12; 24; 30; 54 52,5; 37,5; 22,5; 13,5.
24. La suma de los tres primeros trminos de una progresin geomtrica es igual a 91. Si a
estos trminos adicionamos 25, 27 y 1, respectivamente, obtenemos tres nmeros que forman
una progresin aritmtica. Hallar el sptimo trmino de la progresin geomtrica.
Resp: 5103
7
81
.
CAPTULO 6. ECUACIONES Y SISTEMAS DE ECUACIONES 332
25. Hallar un nmero de tres cifras, en el cual las cifras forman una progresin geomtrica. Si
de este nmero sustraemos 792, obtenemos un nmero escrito con esas mismas cifras, pero
en orden contrario. Si de la cifra que expresa el nmero de centenas, sustraemos 4 y si de-
jamos las restantes cifras del nmero buscado sin variar, hallamos un nmero, cuyas cifras
componen una progresin aritmtica.
Resp: 931.
26. Hallar un nmero de cuatro cifras, en el que las tres primeras forman una progresin arit-
mtica creciente, si sabemos que l se divide por 225.
Resp: 1350.
27. Tres hermanos, cuyas edades forman una progresin geomtrica, dividen entre s cierta
suma de dinero de modo proporcional a la edad. Si esa misma suma de dinero la dividieran
proporcionalmente a su edad tres aos ms adelante, el menor de los hermanos recibira 105
dlares ms, el mediano, 15 dlares ms que ahora. Cul es la edad de los hermanos si
sabemos que la diferencia de aos entre el mayor y el menor de ellos es igual a 15 aos?
Resp: 12, 18, 27.
28. Hallar el nmero de trminos de una progresin aritmtica, en la que la razn entre la
suma de los 13 primeros trminos y la de los 13 ltimos es igual a
1
2
, en tanto que la razn
entre la suma de todos los trminos sin los tres primeros y la de todos los trminos sin los 3
ltimos es igual a
4
3
.
Resp: 20.
29. La suma de una progresin geomtrica innitamente decreciente es igual a
16
3
. La pro-
gresin contiene un trmino igual a
1
6
. La razn entre la suma de todos los trminos de la
progresin, que preceden al que es igual a
1
6
, y la suma de los trminos que le siguen, es igual
a 30. Determine el nmero del trmino igual a
1
6
.
Resp: 5.
30. Una aleacin pesa 2 kilogramos y consta de plata y cobre, con la particularidad de que la
masa de la plata constituye el 14
2
7
% de la masa del cobre. Qu cantidad de plata hay en la
aleacin?
Resp: 0,25 kilogramos.
31. Se ha comprado 1 metro de tejidos de dos calidades por una suma de 15 dlares y 20
centavos. Si el precio del tejido de la primera calidad fuera ms alto y de la segunda, ms
bajo en un mismo por ciento, 1 metro del tejido de la primera calidad costara 15 dlares y
de la segunda, 2 dlares y 40 centavos. Cunto cuesta 1 metro del tejido de primera calidad?
Resp: 12 dlares 9,5 dlares.
CAPTULO 6. ECUACIONES Y SISTEMAS DE ECUACIONES 333
32. Un nmero dgito fue aumentado en 10. Si el nmero obtenido se aumentara al mismo por
ciento que la primera vez, obtendramos 72. Hallar el nmero dgito inicial.
Resp: 2.
33. De acuerdo con el plan, dos fbricas deberan producir 360 mquinas - herramientas al
mes. La primera de ellas cumpli el plan en el 112 %, la segunda en el 110 % y en conjunto,
las dos fbricas produjeron 400 mquinas - herramientas. Cuntas mquinas produjo cada
fbrica por separado superando el plan?
Resp: 24 y 16.
34. Para producir pan de trigo se han tomado tantos kilogramos de harina, como el por ciento
que constituye el aumento de peso de dicha harina. Para producir pan de centeno se han
tomado 10 kilogramos de harina ms y, precisamente, tantos kilogramos como el por ciento
que constituye el aumento de peso de la harina de centeno. Qu cantidad de harina de uno
y otro tipo se ha tomado si en total se han producido 112,5 kilogramos de pan?
Resp: 35 kilogramos de harina de trigo y 45 kilogramos de cebada.
35. Si disminuyera el da laboral de 8 a 7 horas, en qu por ciento hay que aumentar el
rendimiento del trabajo para que con las mismas tarifas el sueldo aumente el 5 %?
Resp: El 20 %.
36. A principios de ao en la cartilla de ahorros fueron puestos 1600 dlares y a nales de l,
saca 848 dlares. A nales del segundo ao en la cartilla haba 824 dlares. Qu inters pone
en cuenta al ao la cooperativa?
Resp: El 30 %.
37. A nes de ao en la cartilla de ahorros del depositante una cooperativa puso en cuenta el
inters, lo que constituy 6 dlares. El depositante aadi 44 dlares y dej su dinero en la
cooperativa para un ao ms. Al acabar el ao, de nuevo pusieron en cuenta los intereses, y,
ahora, el depsito, junto con los intereses, constitua 257 dlares y 50 centavos. Qu suma
fue depositada en la cuenta de ahorros inicialmente?
Resp: 200 dlares.
38. El precio del artculo fue rebajado el 20 %, a continuacin, el nuevo precio lo rebajaron el
15 %; por n, despus del reclculo se efectu la rebaja al 10 % ms. A qu por ciento total
fue rebajado el precio inicial?
Resp: El 38, 8 %.
39. La cantidad de estudiantes en un centro de enseanza, aumentando el mismo por ciento
anualmente, creci en tres aos de 5000 a 6655 personas. En qu tanto por ciento aument
anualmente el nmero de estudiantes?
Resp: El 10 %.
40. El volumen de la sustancia A constituye la mitad de la suma de los volmenes de las
sustancias B y C, en tanto que el volumen de la sustancia B, el 20 % de la suma de los
CAPTULO 6. ECUACIONES Y SISTEMAS DE ECUACIONES 334
volmenes de las sustancias A y C. Hallar la razn entre el volumen de la sustancia C y la
suma de los volmenes de las sustancias A y B.
Resp: 1.
41. Como resultado de la reconstruccin de un taller el nmero de obreros que quedaron libres
puede encontrarse en los lmites del 1,7 al 2,3 %. Hallar el nmero mnimo de obreros que
puede estar ocupado en el taller antes de la reconstruccin.
Resp: 44 personas.
42. El por ciento de estudiantes del curso que han dado todos los exmenes preliminares se
halla en los lmites del 96,8 al 97,2 %. Hallar el nmero mnimo de estudiantes que puede
haber en dicho curso.
Resp: 32 personas.
43. Un turista tiene que cubrir la distancia desde el pueblo hasta la estacin de ferrocarril.
Despus de pasar 3 kilmetros l comprendi que llegaba tarde al tren y empez a andar a
una velocidad de 4 km/h. El turista lleg a la estacin 45 minutos antes de la partida del
tren. Si l hubiera ido a la velocidad inicial se habra demorado 40 minutos. determine la
distancia desde el pueblo hasta la estacin.
Resp: 20 kilmetros.
44. Un pasajero que viaja en un tren a una velocidad de 40 km/h observ por la ventana que
en sentido opuesto, en el transcurso de 3 segundos, pas un tren de 75 metros de longitud.
Cul era la velocidad del tren que iba en direccin contraria?
Resp: 50 km/h.
45. Un ciclista debera cubrir 48 kilmetros a velocidad media determinada. Pero por ciertas
causas, la primera mitad del recorrido se desplaz a una velocidad el 20 % menor, mientras
que la segunda, a 2 kilmetros mayor que la necesaria. Para cubrir todo el recorrido el ciclista
gast 5 horas. Haalar la velocidad que al principio se prevea.
Resp: 10 km/h.
46. Tras cuerpos se mueven por una recta del punto A al B. El segundo cuerpo comenz a
desplazarse 5 segundos y el tercero, 8 segundos despus que el primero. La velocidad del
primer cuerpo es 6 cm/seg menor que la del segundo. La velocidad del tercero es igual a
30 cm/seg. Hallar la distancia AB y la velocidad del primer cuerpo si sabemos que los tres
cuerpos llegan al punto B en el mismo momento.
Resp: 360 cm y 18 cm/seg 60 cm y 6 cm/seg.
47. Al principio el avin volaba a la velocidad de 220 km/h. Cuando le quedaban por volar
385 kilmetros menos que los ya cubiertos, la velocidad aumenta hasta 330 km/h. En el tran-
scurso de todo el recorrido la velocidad media del avin era igual a 250 km/h. Qu distancia
vol el avin?
Resp: 1375 kilmetros.
CAPTULO 6. ECUACIONES Y SISTEMAS DE ECUACIONES 335
48. De los puntos A y B salieron al encuentro, simultneamente, dos trenes. La velocidad del
primer tren es 10 km/h mayor que la del segundo. Los trenes se encontraron a 28 kilmetros
de la mitad del recorrido AB. Si el primer tren hubiera partido de A 45 minutos ms tarde
que el segundo. Los trenes se encontraran en la mitad del recorrido AB. Hallar la distancia
AB y las velocidades de ambos trenes.
Resp: 840 km/h, 80 km/h y 70 km/h.
49. Dos escolares salieron al mismo tiempo de casa a igual velocidad. Uno de ellos, 3 minutos
despus se acord de que haba dejado en casa un libro que necesitaba y retorn a casa a una
velocidad de 60 mt/min mayor que la inicial. Despus de coger el libro comenz de nuevo su
camino a la misma velocidad y alcanz a su compaero, que iba a velocidad constante, ya
junto a la puerta de la escuela. Hallar las velocidades de los escolares si la distancia entre la
escuela y la casa es 280 metros.
Resp: 40 mt/min.
50. Dos transentes que se hallan en los puntos A y B, entre los que hay una distancia de 27
kilmetros en lnea recta, salen de ellos simultneamente, desplazndose por la recta AB. El-
los se encuentran despus de 3 horas si van al encuentro y uno alcanza al otro 9 horas despus,
si se mueven en una misma direccin. Hallar la velocidad de cada uno de los transentes.
Resp: 6 km/h y 3 km/h.
51. Por los dos lados de un ngulo recto, se mueven dos cuerpos en direccin de su vrtice. En
el momento inicial el cuerpo A estaba distanciado del vrtice del ngulo recto a 60 metros, el
cuerpo B, a 80 metros. Pasados 3 segundos la distancia entre A y B se hizo igual a 70 metros
y despus de 2 segundos ms, 50 metros. Hallar la velocidad de cada uno de los cuerpos.
Resp: 6 mt/seg y 8 mt/seg.
52. Por el ro, la distancia entre dos ciudades es igual a 80 kilmetros. Una lancha pasa esta
distancia dos veces (hacia arriba y abajo) en el transcurso de 8 horas 20 minutos. Determine
la velocidad de la lancha en agua estancada si la velocidad de la corriente del ro es igual a
4 km/h.
Resp: 20 km/h.
53. Una lancha se desplaz por un ro aguas arriba 8 kilmetros, dio la vuelta y se desplaz
aguas abajo 36 kilmetros. Todo el viaje dur 2 horas. Despus, la lancha cubri 6 kilmet-
ros contra corriente y a favor de ella, 33 kilmetros, gastando en el segundo viaje 1 hora 45
minutos. Hallar la velocidad de la lancha en agua estancada.
Resp: 20 km/h.
54. En un lago desembocan dos ros. Una lancha parte del muelle A, situado en el primer ro,
se desplaza 24 kilmetros hacia abajo, hasta el lago despus baja 2 kilmetros por el lago,
y, a continuacin, 32 kilmetros por el segundo ro hasta el muelle B, cubriendo la distancia
desde A hasta B en 8 horas. Si la lancha hubiera navegado por el lago 18 kilmetros ms,
todo el recorrido de A a B consumira 10 horas. Hallar la velocidad de la corriente de cada
ro si sabemos que la velocidad del primer ro es 2 km/h mayor que la del segundo.
CAPTULO 6. ECUACIONES Y SISTEMAS DE ECUACIONES 336
Resp: 3 km/h y 1 km/h.
55. Dos peatones salieron, simultneamente, al encuentro de los puntos A y B. Cuando el
primero pas la mitad del camino, al segundo, hasta el nal del recorrido, le quedaban 24
kilmetros. Cuando el segundo cubri la mitad del recorrido, al primero le quedaban 15
kilmetros ms hasta el nal. Cuntos kilmetros ha de recorrer el segundo peatn hasta A
despus de que el primero cubre el camino desde A hasta B.
Resp: 8 kilmetros.
56. De los puntos A y B salen al encuentro dos trenes, con la particularidad de que el segundo
parti media hora despus que el primero. Al pasar 2 horas despus de la partida del primer
tren, la distancia entre ellos constitua
19
30
de la distancia entre A y B. Los trenes se encon-
traron en la mitad del camino desde A hasta B. Cunto tiempo necesitar cada tren para
cubrir todo el recorrido AB?
Resp: 10 horas y 9 horas.
57. La distancia entre las ciudades A y B es igual a 60 km/h. Dos trenes parten simultnea-
mente, uno de A a B y otro de B a A. Pasados 20 kilmetros el tren que va de A a B se
detiene media hora y, a continuacin, desplazndose 4 minutos, se encuentra con el tren que
viene de B. Los dos trenes llegan al mismo tiempo al punto de destino. Hallar las velocidades
de los trenes.
Resp: 60 km/h y 40 km/h.
58. Dos ciclistas salieron al mismo tiempo de los puntos A y B al encuentro uno de otro. El
ciclista que se desplazaba del punto A lleg a B pasadas 4 horas, en tanto que el que iba
del punto B, lleg a A 9 horas despus del encuentro con el otro. Cuntas horas estuvieron
cada uno de los ciclistas en el camino?
Resp: 15 horas y 10 horas.
59. Tres ciclistas, al arrancar simultneamente de un punto y en la misma direccin, van por
un veldromo circular de 1 kilmetros de longitud. Un tiempo despus el segundo alcanza
al primero al recorrer un crculo ms que ste. Pasados 4 minutos, al mismo punto llega el
tercero, al recorrer una distancia igual a la superada por el primero para el momento de
encuentro con el segundo. Las velocidades de los ciclistas forman en cierta sucesin una pro-
gresin aritmtica con una diferencia de 5 km/h. Hallar estas velocidades.
Resp: 20 km/h, 25 km/h, 15 km/h.
60. Tres hermanos cuyas edades forman una progresin geomtrica, reparten entre s cierta
suma de dinero directamente proporcional a sus edades. Si lo hicieran dentro de tres aos,
cuando el menor sea dos veces ms joven que el mayor, entonces el menor obtendra en 105
y el mediano en 15 dlares ms que ahora. Cuntos aos tiene cada uno de los hermanos?
Resp: 27, 18 y 12 aos.
61. Dos grupos de turistas partieron a la vez del punto A hacia el punto B. El primer grupo
sali en un autobs a una velocidad de 20 km/h y lleg en ste hasta el punto C que se
CAPTULO 6. ECUACIONES Y SISTEMAS DE ECUACIONES 337
encuentra en el centro entre los puntos A y B, y sigui a pie. El segundo grupo al principio
iba caminando pero despus de una hora subi a un vehculo de paso que iba a una velocidad
de 30 km/h, y lleg en ste al punto B. El primer grupo atraves el punto C, 35 minutos
antes que el segundo grupo, y lleg al punto B en 1 hora 25 minutos ms tarde que el segun-
do. Qu distancia hay desde el punto A hasta el punto B, si la velocidad (caminando) del
primer grupo es en 1 km/h mayor que la velocidad del segundo grupo?
Resp: 30 kilmetros.
62. Dos recipientes iguales estn llenos de alcohol. Del primer recipiente se extrayeron a litros
de alcohol y se llen la misma cantidad de litros de agua. Seguidamente, de la mezcla obteni-
da de alcohol y agua se extrayeron a litros y se repuso la misma cantidad de litros de agua.
Del segundo recipiente se vertieron 2a litros de alcohol y se llen con la misma cantidad de
litros de agua. Luego, de la mezcla obtenida de alcohol y agua se extrajeron 2a litros y se
repuso la misma cantidad de litros de agua. Determinar qu parte del volumen del recipiente
constituyen a litros si la fuerza de la mezcla denitiva en el primer recipiente es
25
16
veces
mayor que la fuerza de la mezcla denitiva en el segundo recipiente. (Llmase fuerza de la
mezcla la relacin del volumen del alcohol puro en la mezcla a todo el volumen de la mezcla.
Se supone que el volumen de la mezcla es igual a la suma de volmenes de sus partes com-
ponentes).
Resp: 1/6.
63. Dos cuerpos estn en movimiento uniforme por una circunferencia en el mismo sentido.
Uno de ellos alcanza al otro cada 46 segundos. Si estos cuerpos se mueven a las mismas
velocidades en direcciones contrarias, se encuentran entonces cada 8 segundos. Determnense
las velocidades de movimiento de los cuerpos por la circunferencia sabiendo que su radio es
igual a 184 centmetros.
Resp: 19 cm/s y 27 cm/s.
64. Las ciudades A y B estn situadas a orillas de un ro; la ciudad B se halla aguas abajo. A
las 9 a.m., de la ciudad A hacia la ciudad B zarp una balsa con la velocidad de la corriente
del ro con respecto a las orillas. Al mismo tiempo, de la ciudad B hacia la ciudad A parte
un bote que se encuentra con la balsa despus de 5 horas. Al llegar a la ciudad A, el bote
retorn de instante y arrib a la ciudad B simultneamente con la balsa. Si el bote y la
balsa tenan tiempo de llegar a la ciudad B a las 9 p.m. (del mismo da)?
Resp: No llegarn.
65. Cada uno de tres obreros necesita un tiempo para realizar cierto trabajo; el tercer obrero
lo realiza en una hora ms rpido que el primero. Obrando juntos, realizarn el trabajo en
una hora. Y si el primer obrero trabaja durante una hora y despus va a trabajar las 4 horas
el segundo obrero, los dos realizarn todo el trabajo. En cuntas horas puede cumplir todo
el trabajo cada uno de los obreros?
Resp: 3 horas, 6 horas, 2 horas.
66. Hay dos soluciones de una misma sal en agua. Para obtener una mezcla que contenga 10
gramos de sal y 90 gramos de agua, se toman dos partes de la primera solucin y una de la
segunda. Despus de una semana, de cada kilogramo de la primera y la segunda soluciones se
CAPTULO 6. ECUACIONES Y SISTEMAS DE ECUACIONES 338
evapor 200 gramos de agua y para que resulte la misma mezcla se necesitan cuatro partes
de la primera solucin y una de la segunda. Cuntos gramos de sal contenian en inicio 100
gramos de cada solucin?
Resp: 5 gramos y 20 gramos.
67. Un tren de carga que sali de A hacia B lleg a la estacin C simultneamente con un
tren de pasajeros que iba desde B hacia A a una velocidad m veces mayor que la del tren de
carga. Ambos trenes, despus de permanecer t horas en la estacin C, siguieron su camino
aumentando cada uno de ellos su velocidad en un 25 % en comparacin con su velocidad
inicial (es decir, con la velocidad que tenan antes de la llegada a C). En estas condiciones
el tren de carga lleg a B en t
1
horas ms tarde y el de pasajeros lleg a A en t
2
horas ms
tarde, en caso de que ellos se movieran sin parar y a velocidades iniciales. Con cuntas horas
de anterioridad sali el tren de carga de A respecto del de pasajeros que parti de B?
Resp: En 5m(t t
2
) 5m
1
(t t
1
).
68. A, B y C son tres puntos unidos por caminos rectilneos. Con el segmento del camino AB
linda un campo cuadrado que tiene un lado igual a
1
2
AB; el segmento del camino BC es
contiguo a un lote cuadrado de un lado igual a BC; al segmento de camino CA es adyacente
un bosque de forma rectangular cuya longitud es igual a AC y anchura de 4 km. El rea del
bosque es en 20 km
2
mayor que la suma de las reas de los campos cuadrados. Hallar el rea
del bosque.
Resp: El rea del bosque es 40 km
2
.
69. Un grupo de estudiantes compuesto de 30 personas en un exmen recibi calicaciones de
2, 3, 4 y 5. La suma de las calicaciones obtenidas es igual a 93; las notas de tres fueron
ms que las de cinco y menos que las de cuatro. Por lo dems, el nmero de las de cuatro se
divida por 10 y el nmero de las de cinco fue par. Determine cuntas y cules calicaciones
recibi el grupo.
Resp: El nmero de calicaciones 2, 3, 4 y 5 es igual a 11, 7, 10 y 2, respectivamente.
70. Una motocicleta y un coche X salen simultneamente del punto A hacia el punto B y en
ese mismo instante del punto B hacia el punto A parte un coche Y que 5 horas 50 minutos
despus llega al punto A. Los automviles se encontraron 2 horas 30 minutos despus de
la salida, y la motocicleta y el coche Y , a la distancia de 140 kilmetros del punto A. Si
la velocidad de la motocicleta fuera dos veces mayor, se encontrara con el coche Y a 200
kilmetros del punto A. Hallar las velocidades de la motocicleta, el coche Y y el coche X.
Resp: La velocidad de la motocicleta es de 40 km/h, la del coche Y 60 km/h y la del coche
X, 80 km/h.
71. El agua pura y un cido de concentracin constante empiezan a llegar simultneamente
por dos tubos a un recipiente. Una vez que el recipiente estuvo lleno, result una solucin
de cido al 5 %. Y si se dejara de hacer llegar el agua en el momento cuando el recipiente
est por la mitad, resultara una solucin al 10 %. Determine cul de los tubos proporciona
el lquido ms rpidamente y en cuntas veces.
Resp: El agua se suministra 2 veces ms rpidamente.
CAPTULO 6. ECUACIONES Y SISTEMAS DE ECUACIONES 339
72. Un coche sali del punto A hacia el punto B. Simultneamente, al encuentro de ste, del
punto B parti un ciclista. Tres minutos despus del encuentro, el coche regresa al instante,
sigue al ciclista y, al alcanzarlo, de nuevo vuelve al instante para llegar al punto B. Si el
coche regresara al instante un minuto despus del encuentro y el ciclista aumentar
15
7
veces
la velocidad despus del encuentro, aqul demorara el mismo tiempo para recorrer todo el
camino. Hallar la relacin entre las velocidades del ciclista y del coche.
Resp: 1 : 3.
73. Desde el punto A hacia el punto B que distan uno de otro a 100 kilmetros, al mismo
instante salieron un ciclista y un transeunte. Simultneamente, del punto B parti un au-
tomovilista al encuentro de stos. Una hora despus de la carrera el automovilista encontr
al ciclista y luego, al pasar ms unos 14
2
17
kilmetros, encontr al transeunte y lo subi
al coche; despus de esto echaron a correr detrs del ciclista y lo alcanzaron. Calcular las
velocidades con las cuales se movian el ciclista y el automovilista si es sabido que la velocidad
del transeunte era igual a 5 km/h. El tiempo necesario para la subida del transeunte y el
viraje del automvil se considera igual a cero.
Resp: 20 km/h y 80 km/h.
74. Un laboratorio necesita encargar una cantidad de matraces esfricos iguales de una capaci-
dad total de 100 litros. El valor de un matraz lo componen el costo del trabajo del obrero,
proporcional al cuadrado de la supercie del matraz, y el costo del materjal, proporcional a su
supercie. En estas condiciones, el matraz de 1 litro cuesta 1,25 dlares y el valor del trabajo
constituye un 20 % del costo del matraz (el espesor de las paredes del matraz se considera
despreciativamente pequeo). Son sucientes 100 dlares para realizar el trabajo?
Resp: No.
75. El autobs N
o
1, en el que un estudiante puede llegar de su casa a la universidad, sin
trasbordos, demora 2 horas 1 minuto. En cualesquiera de los autobuses N
o
2, N
o
3, ..., N
o
k se puede llegar tambin a la universidad; sin embargo, el estudiante puede hacer trasbordo
al autobs N
o
p solamente del autobs N
o
(p-1). Las rutas de estos autobuses son tales que
el estudiante, al llegar a la universidad en uno de ellos, demorar un tiempo (sin contar los
trasbordos) inversamente proporcional al nmero de autobuses utilizados. adems de esto,
en cada trasbordo invertir 4 minutos. Es cierto que hay un camino que necesita en total
menos de 40,1 minutos?
Resp: No.
76. Entre el poblado A y la ciudad D se encuentran la gasolinera B y la torre de agua C que
dividen la distancia AD en tres partes iguales (AB = BC = CD). De A hacia D salieron un
coche y un ciclista, y de D hacia A, simultneamente con stos, sali un camin que se cruz
con el coche cerca de la torre de agua, y con el ciclista, cerca de la gasolinera. El ciclista
aument su velocidad en 5 km/h cerca de la gasolinera. El coche, al llegar al punto D, regres
al instante con una velocidad de 8 km/h menos de la que tena antes. Como resultado, en el
momento cuando el camin lleg al punto A, al ciclista le quedaba por recorrer 7,5 kilmet-
ros para llegar a C, y el coche, se encontraba entre B y A a 14 kilmetros de B. Hallar la
distancia entre el poblado y la ciudad y las velocidades de los vehculos y el ciclista.
Resp: La velocidad del ciclista es de 20 km/h, la del camin, 40 km/h y la del coche, 80
CAPTULO 6. ECUACIONES Y SISTEMAS DE ECUACIONES 340
km/h. La distancia AD es igual a 60 km.
77. Un lote rectangular con un rea de 900 m
2
hay que vallar de cerca cuyos lados adyacentes
deben ser de piedra y los otros dos, de madera. Un metro de la cerca de madera cuesta 10
dlares y el de piedra, 25 dlares. Para la construccin se han asignado 2000 dlares. Al-
canzar esta suma?
Resp: No lo alcanzar.
78. El recipiente de una torre de agua se llena por varias bombas. Al principio se pusieron
en accin tres bombas de igual rendimiento y despus de 2,5 horas de trabajo empezaron a
funcionar dos bombas ms de rendimiento distinto de las tres primeras pero igual entre s.
Como resultado, una hora despus de la conexin de las bombas al recipiente le faltaban 15
m
3
para llenarse; despus de una hora el recipiente estaba lleno. Una de las bombas puestas
en accin ms tarde podra llenar el recipiente en 40 horas. Hallar la capacidad del recipiente.
Resp: 60 m
3
.
79. En las competiciones de esqus a la distancia de 10000 metros arranc el primer esquiador y
un tiempo despus sali el segundo con una velocidad en 1 mt/seg mayor que la del primero.
En el instante cuando el segundo alcanz al primero ste aument su velocidad en 2 mt/seg,
mientras que la velocidad del segundo esquiador no vari. Como resultado de esto el segundo
esquiador cruz la meta 7 minutos 8 segundos despus del primero. Si la distancia fuera 500
metros ms larga, el segundo esquiador llegara a la meta 7 minutos 33 segundos ms tarde
que el primero. Hallar qu tiempo pas entre la salida del primero y segundo esquiadores.
Resp: 2 minutos.
80. Tres patinadores, cuyas velocidades en sucesin forman una progresin geomtrica, parten
simultneamente de carrera por un crculo. Despus de un tiempo el segundo patinador
adelanta al primero, recorriendo 400 metros ms que ste. El tercer patinador recorre una
distancia igual a la recorrida por el primero hasta el momento cuando fue adelantado por el
segundo, en espacio de tiempo de
2
3
de un minuto mayor que el primero. Hallar la velocidad
del primer patinador.
Resp: 0,6 km/min.
81. Una hacienda dispone de cuatro marcas de tractores: A, B, C y D. Cuatro tractores (2
tractores de la marca B, un tractor de la marca C y uno de la marca D) realizan la arada
de un campo en dos das. Dos tractores de la marca A y un tractor de la marca C invierten
tres das para el mismo trabajo, y los tres tractores de las marcas respectivamente A, B y
C, demoran cuatro das. En qu tiempo realizarn el trabajo cuatro tractores de distintas
marcas?
Resp: 12/7 das.
82. En tres campos se segaba la hierba durante tres das. En el primer da toda la hierba del
primer campo se seg en 16 horas. En el segundo campo toda la hierba se seg, en el segundo
da, en 11 horas. En el tercer da toda la hierba del tercer campo se seg en 5 horas: 4 horas la
segaban a mano y una hora trabajaba una sola segadora. Durante el segundo y el tercer das
CAPTULO 6. ECUACIONES Y SISTEMAS DE ECUACIONES 341
la hierba se seg 4 veces ms que en el primero. Cuntas horas trabaj la segadora si por
una hora sta segaba 5 veces ms hierba que la que daba la siega a mano? Se sobreentiende
que la segadora no trabajaba, mientras se realizaba la siega a mano y no haba pausas en el
trabajo.
Resp: 12 horas.
83. Una fbrica tiene que mandar a su cliente 1100 piezas. Para el envo las piezas se embalan
en cajones. Los cajones de que se disponen son de tres tipos. En el cajn del primer tipo
caben 70 piezas, en el de segundo tipo, 40 piezas, y en el de tercer tipo, 25 piezas. El costo de
envo de un cajn de primer tipo es de 20 dlares, el costo de env;io de un cajn de segundo
tipo es de 10 dlares, el envo de un cajn de tercer tipo es de 7 dlares. Cules cajones
debe utilizar la fbrica para que el costo de envo sea el mnimo? Los cajones deben estar
completos.
Resp: 4 cajones del tercer tipo y 25 cajones del segundo tipo.
84. Un estudiante encola de nuevo todos sus sellos en otro lbum. Si pega 20 sellos en cada
hoja, entonces no le alcanzar el lbum; si pega 23 sellos, le sobrar, por lo menos, una hoja
vaca. Y si al estudiante se le regala igual lbum con 21 sellos, en cada hoja el estudiante
tendr 500 sellos. Cuntas hojas tiene el lbum?
Resp: 12 hojas.
85. Dos tubos funcionando simultneamente durante una hora llenan de agua
3
4
de un depsito.
Si al principio el primer tubo llena
1
4
del depsito y luego el segundo, estando desconectado
el primero, complete el volumen de agua hasta los
3
4
del depsito, se necesitarn para esto 2,5
horas. Si se pone en funcionamiento el primer tubo durante una hora, y el segundo, media
hora, el depsito se llenar ms all de la mitad. En qu tiempo cada uno de los tubos
llenar el depsito?
Resp: El primer tubo llenar el depsito en 2 horas, el segundo, en 4 horas.
86. Los puntoa A y B se encuentran en un ro de modo que la balsa que va desde A hacia
B a la velocidad de la corriente del ro, recorre el trayecto AB en 24 horas. La lancha a
motor recorre todo el trayecto AB, en ida y regreso, en no menos de 10 horas. Si la velocidad
propia de la lancha (es decir, la velocidad en agua muerta) aumentara en un 40 %, entonces
el trayecto (es decir, el espacio AB) sera recorrido por sta en no ms de 7 horas. Hallar el
tiempo durante el cual la lancha a motor pasa el trayecto AB en caso de que su velocidad
propia no aumente.
Resp: 4 horas.
87. Desde el punto A hacia el punto B, a las 8 a.m. sale un tren rpido. En ese mismo instante,
desde el punto B hacia el punto A salen dos trenes, uno de pasajeros y otro expreso; la ve-
locidad del tren de pasajeros es dos veces menor que la del expreso. El tren rpido encuentra
al tren expreso no antes de las 10:30 a.m., y llega al punto B a las 13:50 p.m. del mismo
da. Hallar la hora de llegada del tren de pasajeros al punto A si se sabe que pasa no menos
de una hora entre los encuentros del tren rpido con el expreso y del tren rpido con el de
pasajeros.
CAPTULO 6. ECUACIONES Y SISTEMAS DE ECUACIONES 342
Resp: 16 horas 45 minutos.
88. A las 9 a.m., desde el punto A parte un ciclista que se dirige al punto B. Dos horas despus
de la salida del ciclista, desde A hacia B parte un automovilista que alcanza al ciclista a no
ms tardar las 12 del da. Siguiendo la marcha, el automovilista llega al punto B y vuelve
al instante desde B hacia A. En este camino el automovilista encuentra al ciclista y llega al
punto A a las 5 p.m. de ese mismo da. Hallar el tiempo de llegada del ciclista al punto B
si se sabe que entre los dos encuentros del automovilista y del ciclista transcurrieron no ms
de 3 horas.
Resp: 18 horas.
89. Del punto A, corriente arriba, parti una canoa y del punto B, situado ms arriba que el
punto A por la corriente, sali, simultneamente, una balsa. Pasadas x horas ellas se encon-
traron y, ms adelante, se desplazaron sin paradas. Al llegar a B, la canoa, sin detenerse, dio
la vuelta y alcanz la balsa en el punto A. Cunto tiempo navegaron la balsa y la canoa
hasta encontrarse en el punto A, si sabemos que la velocidad propia de la canoa es constante?
Resp: x(1 +

2) horas.
90. La distancia entre dos ciudades es cubierta por un tren rpido 4 horas antes que un tren
de mercancas y 1 hora antes que uno ordinario. Sabemos que la velocidad del de mercancas
constituye
5
8
de la del ordinario y es 50 km/h menor que la del rpido. Hallar las velocidades
de los trenes de mercancas y del rpido.
Resp: 50 km/h y 100 km/h.
91. De dos puntos, entre los que hay una distancia igual a 2400 kilmetros, salieron, simultnea-
mente, al encuentro un tren ordinario y un rpido. Cada uno de ellos se desplaza a velocidad
constante y, en cierto momento de tiempo, ellos se encuentran. Si ambos se movieran a la
velocidad del rpido, su encuentro hubiera acontecido 3 horas antes que el momento real del
encuentro. Si ambos trenes marcharan a la velocidad del ordinario su encuentro se hubiera
producido 5 horas despus del momento real del encuentro. Hallar las velocidades de los
trenes.
Resp: 60 km/h y 100 km/h.
92. Por una circunferencia de 360 metros de largura, se mueven dos puntos, con la particulari-
dad de que el primero recorre la circunferencia 4 segundos ms rpido. Hallar la velocidad de
cada punto si sabemos que el primer punto pasa por 1 segundo 4 metros ms que el segundo.
Resp: 40 m/s y 36 m/s.
93. Dos puntos en movimiento por una circunferencia en una misma direccin, se encuentran
despus de cada 20 segundos y estando en movimiento en direcciones opuestas, cada 4 se-
gundos. Hallar la velocidad de cada punto, si se sabe que la longitud de la circunferencia es
igual a 100 metros.
Resp: 15 m/s, 10 m/s.
CAPTULO 6. ECUACIONES Y SISTEMAS DE ECUACIONES 343
94. Dos puntos que se mueven por una circunferencia en la misma direccin se encuentranm
cada 56 minutos y estando en movimiento en direcciones opuestas, cada 8 minutos. Hallar la
velocidad de cada punto y la longitud de la circunferencia, si sabemos que durante 1 segundo
el primer punto cubre una distancia
1
12
metros mayor que el segundo.
Resp: 20 m/min, 15 m/min, 280 metros.
95. Dos puntos en movimiento por una circunferencia en una misma direccin se encuentran
cada 12 minutos, con la particularidad de que el primero da la vuelta a la circunferencia 10
segundos ms rpido que el segundo. Qu parte de la circunferencia cubre en 1 segundo
cada uno de los puntos?
Resp:
1
80
,
1
90
.
96. Una motonave parti del punto A al B y, despus de 7,5 horas, tras ella del punto A sali
una lancha. En la mitad del recorrido de A a B la lancha alcanz a la motonave. Cuando la
primera lleg a B, a la segunda le quedaban navegar
3
10
de todo el recorrido. Cunto tiempo
es necesario para que la motonave pase la distancia de A a B?
Resp: 25 horas.
97. Del punto A al B sali un tren ordinario. Tras l, 3 horas despus, parti de A un rpido.
Este alcanz al ordinario en la mitad del recorrido de A a B. En el momento de la llegada
del rpido a B el ordinario cubri
15
16
de todo el recorrido. Cunto tiempo necesitar el tren
ordinario para cubrir la distancia de A a B?
Resp: 16 horas.
98. Del punto A al B sali un peatn. despus de
3
4
horas, tras el parti un ciclista. Cuando
ste lleg a B al peatn le quedaban por pasar
3
8
de todo el camino. Cunto tiempo necesit
el peatn para cubrir todo el recorrido, si sabemos que el ciclista alcanz al peatn en la
mitad de la distancia de A a B?
Resp: 2 horas.
99. Del punto A al B, entre los que la distancia es igual a 70 kilmetros, sali un ciclista, y,
cierto tiempo despus, un motociclista, cuya velocidad era 50 km/h. Esta alcanz al ciclista
a 20 kilmetros del punto A. tras haber llegado a B, 48 minutos despus, el motociclista sali
en direccin contraria hacia A y se encontr con el ciclista pasadas 2 horas 40 minutos luego
de salir ste de A. Hallar la velocidad del ciclista.
Resp: 25 km/h.
100. Del desembarcadero A, corriente del ro abajo, salieron, simultneamente, una lancha y
una balsa. La primera, despus de llegar al muelle B, situado a 324 kilmetros de A, pasadas
18 horas de escala en l, parti de nuevo en direccin de A. En el momento cuando se encon-
traba a 180 kilmetros del muelle A, la segunda lancha que sali de A 40 horas ms tarde
de la primera, alcanz a la balsa que, hasta entonces, haba cubierto una distancia de 144
kilmetros. Hallar las velocidades de ambas lanchas, si se sabe que son iguales y se conoce la
CAPTULO 6. ECUACIONES Y SISTEMAS DE ECUACIONES 344
velocidad de la corriente del ro.
Resp: La velocidad de los barcos es igual a 15 km/h, la velocidad de la corriente es igual
a 3 km/h.
101. En el ro desemboca un auente. Una lancha parte del muelle A, situado en el auente,
va corriente abajo 60 kilmetros hasta el ro, a continuacin ro abajo 65 kilmetros hasta
el desembarcadero B. Ms adelante, por ese mismo itinerario, la lancha retorna, necesitando
para el recorrido inverso 10 horas. Hallar la velocidad propia de la lancha si sabemos que
para el recorrido por el ro desde A, la lancha gasta 3 horas 45 minutos y la velocidad de la
corriente del ro es 1 km/h menor que la de la corriente del auente.
Resp: 14 km/h.
102. Dos nadadores partieron, uno tras otro, en una piscina de 50 metros para la distancia de
100 metros. El segundo nadador, cuya velocidad es igual a 1,5 mt/seg, alcanz al primero
en la marca 21 metros, despus, al llegar a la pared opuesta de la piscina, dio la vuelta y
se encontr con el primer nadador despus de
2
3
segundos de darla. Hallar el intervalo de
tiempo entre los momentos de partida de los nadadores.
Resp: 1 segundo.
103. Del punto A, en una misma direccin, salieron dos esquiadores, con la particularidad de
que el segundo parti 6 minutos despus que el primero y que alcanz a ste a 2 kilmetros de
la lnea de salida. Al llegar a la marca de 5 kilmetros el segundo esquiador dio la vuelta y se
encontr con el primero a 4 kilmetros de la lnea de salida. Hallar la velocidad del segundo
esquiador.
Resp: 10 km/h.
104. Dos ciclistas partieron, uno tras otro, con un intervalo de 2 minutos. el segundo alcanz al
primero a la distancia de 1 kilmetros de la lnea de salida. Si despus de recorrer 5 kilmetros
desde la lnea de salida, l hubiera dado la vuelta hacia atrs, se encontrara con el primer
ciclista 20 minutos despus de la partida de ste. Hallar la velocidad del segundo ciclista.
Resp: 20 km/h.
105. De A a B, simultneamente, salen un ciclista y un peatn. La velocidad del ciclista es dos
veces mayor que la del peatn. Al mismo tiempo, a su encuentro, de B a A sale el segundo
peatn. El tiempo entre los encuentros de ste con el ciclista y el primer peatn constituye
2
15
de la fraccin del tiempo necesario para su recorrido de B a A. Cul de los peatones
y cuntas veces iba ms rpido, si hasta encontrarse los dos cubrieron ms de
1
4
de toda la
distancia de A a B?
Resp: La velocidad del primer peatn es 2 veces mayor que la del segundo.
106. Del punto A al B sali una motonave. a las 8 horas ella alcanz a una lancha, que iba por
ese mismo recorrido, cuya velocidad era igual a 3 km/h. Al retornar de A a B, en el que tuvo
una parada de 10 minutos, la motonave se encontr con esa misma lancha a las 8 horas 20
minutos. Al punto A la motonave llega cuando la lancha alcanza el punto B. Determine el
CAPTULO 6. ECUACIONES Y SISTEMAS DE ECUACIONES 345
tiempo de llegada de la lancha al punto B, si sabemos que a las 8 horas 10 minutos ella se
encontraba a 1,5 kilmetros del punto A.
Resp: En 8 horas 30 minutos.
107. Si un pasajero sale en tren del punto A, al punto B llegar despus de 20 horas. Si l vuela
en avin, que debe esperar ms de dos horas, llegar a B pasadas 10 horas luego de partir el
tren. Cuntas veces es mayor la velocidad del avin que la del tren, si sabemos que tras
8
9
horas de comenzar el vuelo el avin se encontrar a la misma distancia del punto A que el
tren?
Resp: 10 veces ms.
108. Del punto A al B salen, simultneamente, un peatn y un ciclista. Llegado a B el ciclista
da la vuelta y, tras 1 hora de haber comenzado el movimiento, se encuentra con el peatn.
Despus del encuentro el peatn contina su camino hacia B y el ciclista da la vuelta y
tambin se dirige a B. Habiendo alcanzado B, el ciclista de nuevo retorna y, una vez ms, se
encuentra con el peatn pasados 40 minutos del primer encuentro. Determine cunto tiempo
necesitar el peatn para cubrir la distancia de A a B.
Resp: En 3 horas.
109. Del punto A salieron tres ciclistas. El primero parti 1 hora antes que los otros dos que
comenzaron el movimiento simultneamente. Pasado cierto tiempo, el tercer ciclista alcanz
al primero, mientras que el segundo igual al primero 2 horas despus que el tercero. De-
termine la razn entre las velocidades de los ciclistas primero y tercero, si la razn entre las
velocidades de los ciclistas segundo y tercero es igual a 2 : 3.
Resp:
1
2
.
110. La distancia entre los puntos A y B es igual a 105 kilmetros. De A a B sali un autobs a
una velocidad de v km/h. Despus de 30 minutos, tras l, sali un automvil, cuya velocidad
era igual a 40 km/h. Tras de haber alcanzado al autobs, el automvil da la vuelta y, a la
misma velocidad, retorna hacia A. Con qu valores de la velocidad v el autobs llegar a B
antes que el automvil llegue a A?
Resp: 30 < v < 40.
111. Simultneamente, de los puntos A y B salen dos correos al encuentro uno de otro. Pasado
cierto tiempo ellos se encuentran. Si el primer correo hubiese salido 1 hora antes y el segundo,
0,5 hora ms tarde, ellos se habran encontrado 43 minutos antesw. Si el primero saliera 0,5
hora despus y el segundo, 1 hora antes, el lugar del encuentro se trasladara a 5600 metros.
Cul es la velocidad de cada correo?
Resp: 8 km/h y 7 km/h.
112. Entre los puntos A y B se encuentra C, con la particularidad de que AC = 17 kilmetros,
BC = 3 kilmetros. De A a B parti un automvil que, al recorrer menos de dos kilmetros,
se par. Cierto tiempo despus l sigui su camino hacia B y, en este momento de tiempo,
de C a B partieron un peatn y un ciclista, cada uno de los que al alcanzar B, de inmediato,
comenzaron el camino inverso. Con cul de ellos se igualar antes el automvil, si sabemos
CAPTULO 6. ECUACIONES Y SISTEMAS DE ECUACIONES 346
que la velocidad de ste es 4 veces mayor que la del ciclista y 8 veces mayor que la del peatn?
Resp: Con el ciclista.
113. Del punto A al punto B sali un peatn. Simultneamente, de B a A, a su encuentro,
parti un motociclista. Al encontrarse con el peatn, el motociclista lo subi en su moto, lo
llev a B, all lo dej y, de nuevo, parti hacia A. Como consecuencia, el peatn alcanz B 4
veces ms rpido de lo que plane. Cuntas veces ms rpido hubiera llegado el motociclista
al punto A, si no hubiera tenido que retornar?
Resp: 2,75.
114. Del punto A al B se ha trado una mercanca. De A la llevaron primero en un furgn y, a
continuacin, en un camin. La distancia del lugar del trabordo hasta el punto B es 3 veces
menor que desde el punto de trasbordo al punto A. Para llevar la mercanca de A a B ha
sido necesaria una cantidad de tiempo igual al tiempo requerido para ir de A a B a una
velocidad de 64 km/h. A qu velocidad se desplazaba el camin, si sabemos que la velocidad
del furgn era no ms de 75 km/h, as como que si ste y el camin hubiesen salido de los
puntos A y B al encuentro uno de otro, ellos se habran encontrado despus del intervalo de
tiempo necesario para recorrer la distancia de A a B a una velocidad de 120 km/h?
Resp: 48 km/h.
115. Dos ciclistas salieron, simultneamente, al encuentro de los puntos A y B y, pasadas 2,4
horas, se encontraron. Si el primer ciclista aumentara la velocidad el 50 % y el segundo, el
20 %, para vencer la distancia de A a B al primero le hubiera hecho falta
2
3
horas ms que
al segundo ciclista. Cunto tiempo necesitara cada ciclista para cubrir la distancia entre A
y B?
Resp: En 6 horas y en 4 horas.
116. Del punto A al B parti un motociclista. Pasadas 2 horas sali tras l un automvil que
lleg al punto B al mismo tiempo que el motociclista. Si el automvil y el motociclista hu-
biesen salido simultneamente de A y B al encuentro uno de otro, se habran encontrado
tras pasada 1 hora 20 minutos despus de la partida. Cunto tiempo necesita el motociclista
para vencer la distancia de A a B?
Resp: En 4 horas.
117. Del punto A al B sali un ciclista. Al mismo tiempo, de B a A sali un motociclista y se
encontr con el ciclista 45 minutos despus de su salida. Cunto tiempo necesita el ciclista
para cubrir la distancia entre A y B, si sabemos que el motociclista vence ese mismo recorrido
invirtiendo 2 horas menos? Para recorrer la distancia de A a B una lancha invierte 3 horas
y para la vuelta, 4 horas. Cunto tiempo navegar una balsa de A a B?
Resp: En 3 horas.
118. Para recorrer la distancia de A a B una motonave invierte 3 horas y para la vuelta, 4
horas. Cunto tiempo navegar una balsa de A a B?
Resp: 21 horas.
CAPTULO 6. ECUACIONES Y SISTEMAS DE ECUACIONES 347
119. Un electricista baj por la escalera mecnica en movimiento, empleando para ello 30 se-
gundos. La segunda vez l baj por la escalera mecnica parada invirtiendo 45 segundos.
Cunto tiempo gastara al bajar si estuviera parado en el peldao de la escalera en marcha?
Resp: En 90 segundos.
120. Del punto A al B sali un autobs. Al llegar a B l contina el desplazamiento en la
misma direccin. En el momento cuando al autobus alcanz el punto B, del punto A, en esa
misma direccin, parti un automvil. Para cubrir la distancia desde A hasta B el automvil
invierte 3 horas 20 minutos menos que el autobs en el mismo recorrido. Cuntas horas son
necesarias para que venzan ese recorrido el automvil y el autobs, si sabemos que la suma
de sus velocidades es 1,5 veces mayor que el tiempo necesario para que el automvil alcance
al autobs?
Resp:
20
3
horas y
10
3
horas.
121. Simultneamente, de dos puntos A y B salen, al encuentro uno de otro, un ciclista y un
autobs. Para el recorrido de A a B el ciclista invierte 2 horas 40 minutos ms que el autobs
para recorrer la distancia de B a A, mientras que la suma de dichas horas es
16
3
veces mayor
que el tiempo pasado desde el comienzo del desplazamiento del ciclista y el autobs hasta el
momento de su encuentro. Cunto tiempo invierte el ciclista para ir de A a B y el autobs
para vencer la distancia entre B y A?
Resp: 4 horas y
4
3
horas.
122. Del punto A al B se ha llevado el correo. Primero lo llev un motociclista que, cubriendo
2
3
de la distancia entre dichos puntos, entreg el correo a un ciclista que le esperaba. El correo
fue trasladado de A a B durante el intervalo de tiempo necesario para ir de A a B a una
velocidad de 40 km/h. Sabemos que si el motociclista y el ciclista hubiesen salido de A a B
simultneamente al encuentro, ellos se hubieran encontrado despus del lapso necesario para
cubrir la distancia de A a B a la velocidad de 100 km/h. Hallar la velocidad del motociclista
suponiendo que ella es mayor que la del ciclista.
Resp: 80 km/h.
123. En una mina de carbn trabajaban primero dos secciones y despus de lo cual el rendimien-
to de la mina aument 1,5 veces. Cuntos por ciento del rendimiento de la segunda seccin
constituye el de la primera si durante 4 meses las secciones primera y tercera extraen, con-
juntamente, tanto carbn como arranca la segunda seccin en el transcurso de un ao?
Resp: El 60 %.
124. Dos brigadas comenzaron el trabajo a las 8 horas. Despus de hacer en conjunto 72 piezas,
ellas comenzaron a trabajar por separado. A las 15 horas qued claro que al trabajar por
separado la primera brigada produjo 8 piezas ms que la segunda. Al da siguiente la primera
brigada produca cada 1 hora una pieza ms y la segunda durante 1 hora una pieza menos
que el primer dia. Las brigadas comenzaron a trabajar a las 8 horas en conjunto y, habiendo
hecho 72 piezas, de nuevo pasaron al trabajo por separado. En el transcurso de esta forma
de trabajo, ya hacia las 13 horas, la primera brigada produjo 8 piezas ms que la segunda.
Cuntas piezas por hora produca cada brigada?
CAPTULO 6. ECUACIONES Y SISTEMAS DE ECUACIONES 348
Resp: Primera brigada 13 piezas, segunda brigada 11 piezas.
125. Una piscina se llena de agua por el primer tubo 5 horas antes que por el segundo y 30 horas
antes que por el tercero. Es conocido que la capacidad de paso del tercer tubo es 2,5 veces
menor que la del primer tubo y 24 m
3
/h menor que la del segundo tubo. Hallar la capacidad
de paso de los tubos primero y tercero.
Resp: 60 m
3
por hora y 24 m
3
por hora.
126. Tres obreros deben hacer 80 piezas iguales. Se sabe que, en conjunto, los tres producen
por hora 20 piezas. El primer obrero fue el primero que empez a trabajar. El hizo 20 piezas
invirtiendo para hacerlas ms de 3 horas. La restante parte de piezas fue hecha por el segun-
do y tercero obreros. Para acabar todo el trabajo emplearon 8 horas. Cunto tiempo sera
necesario al primer obrero para producir las 80 piezas?
Resp: 16 horas.
127. Un petrolero se llena de petrleo trabajando dos tubos, con la particularidad de que cada
uno de ellos rellen ms de
1
4
de su volumen. Si la cantidad de petrleo alimentado por hora
por el primer tubo hubiese sido 1,5 veces mayor y la cantidad de petrleo alimentado por
hora por el segundo tubo hubiera sido 4 veces menor, el tiempo necesario para rellenar el
petrolero aumentara
1
6
parte del tiempo que es necesario para llenar el petrolero por slo el
primer tubo. Por qu tubo se alimenta mayor cantidad de petrleo y cuntas veces ms?
Resp: Por el segundo tubo 2 veces ms.
128. Por tres tubos se alimenta petrleo a un depsito y de l se evacua por el cuarto. El primer
da los tubos tercero y cuarto trabajaron 6 horas cada uno, el segundo, 5 horas, el primero
2 horas. Como resultado el nivel del petrleo se elev 4 metros. El segundo da los tubos
primero y segundo funcionaron 3 horas cada uno, el tercero, 9 horas, el cuarto, 4 horas.
Debido a esto, el nivel del petrleo se elev 6 metros ms. El tercer da los tubos segundo y
cuarto funcionaron 6 horas. Subi o baj el nivel de petrleo el tercer da?
Resp: El nivel del petrleo subi.
129. Dos obreros realizaron juntos cierto trabajo en el transcurso de 12 horas. Si al principio el
primer obrero hubiera hecho la mitasd del indicado trabajo y, a continuacin, el segundo la
parte restante, todo el trabajo hubiese sido efectuado durante 25 horas. En el transcurso de
qu tiempo podra realizar este trabajo cada uno de los obreros por separado?
Resp: En 20 horas y en 30 horas.
130. Dos obreros realizan cierto trabajo. Pasados 45 minutos de trabajo conjunto, el primer
obrero fue enviado a realizar otro trabajo y el segundo obrero acab la parte restante del
trabajo en el transcurso de 2 horas 15 minutos. Cunto tiempo necesitara cada uno de los
obreros por separado para realizar todo el trabajo, si sabemos que el segundo necesitara
para ello 1 hora ms que el primero?
Resp: En 3 horas y en 4 horas.
CAPTULO 6. ECUACIONES Y SISTEMAS DE ECUACIONES 349
131. Dos torneros deban producir un determinado nmero de piezas. Despus de trabajar en
conjunto tres horas, continu trabajando slo el segundo tornero que trabaj 4 horas ms.
Despus de esto, la tarea fue sobrecumplida el 12, 5 %. Cunto tiempo sera necesario a cada
tornero por separado para cumplir la tarea, si sabemos que el segundo necesitara 4 horas
menos que el primero?
Resp: En 12 horas y en 8 horas.
132. Una piscina puede llenarse de agua con dos grifos. Si el primero se abre 10 minutos y el
segundo, 20 minutos, la piscina se llenar. Si el primer grifo se abre 5 minutos y el segundo, 15
minutos, se llenar
3
5
de la piscina. En el transcurso de qu tiempo cada grifo por separado
puede llenar toda la piscina?
Resp: En
5
6
horas y en
5
18
horas.
133. Dos brigadas trabajaron juntas 15 das, despus de lo cual a ellas se uni la tercera brigada
y pasados 5 das despus de esto todo el trabajo fue acabado. Sabemos que la segunda briga-
da produce al da el 20 % ms que la primera. Las brigadas segunda y tercera en conjunto
podran realizar todo el trabajo en
9
10
del tiempo necesario para que todo el trabajo sea re-
alizado por las brigadas primera y tercera al trabajar juntas. Si las tres brigadas trabajaran
juntas cunto tiempo necesitaran para ejecutar todo el trabajo?
Resp: En 16 das.
134. Para descargar una barcaza se han destinado dos brigadas de cargadores. Si al tiempo du-
rante el cual puede descargar la barcaza la primera brigada aadimos el tiempo que necesita
la segunda brigada para hacer ese trabajo, resultan 12 horas. En el transcurso de cuntas
horas cada brigada puede descargar la barcaza. Si la diferencia entre esas horas constituye
el 45 % de todo el tiempo necesario para descargar la barcaza trabajando juntas las dos
brigadas?
Resp: En
20
3
horas y en
16
3
horas.
135. Para excavar una zanja se destinan dos excavadoras de diferente tipo. El tiempo necesario
para que la primera excavadora cave la zanja es 3 horas menor que el que precisa la segunda
para realizar ese mismo trabajo. Cuntas horas necesitar cada excavadora para excavar la
zanja, si la suma de dichas horas es
144
35
veces mayor que el tiempo necesario para hacer la
zanja trabajando juntas?
Resp: 7,5 horas y 10,5 horas.
136. Un barco cargero se carga con grus. Primero, durante dos horas, trabajaron 4 gras de
igual potencia, a continuacin, a ellas se unieron dos gras ms, pero de menor potencia;
pasadas 3 horas despus de esto la carga naliz. Si todas las gras hubieran comenzado a
trabajar simultneamente, la carga hibiese acabado en el transcurso de 4,5 horas. Cunto
tiempo necesitan para realizar la carga 1 gra de elevada potencia y 1 gra de menor potencia
al trabajar juntas?
Resp: En 14,4 horas.
CAPTULO 6. ECUACIONES Y SISTEMAS DE ECUACIONES 350
137. A un foso se alimenta agua uniformemente. 10 bombas iguales, funcionando simultnea-
mente, pueden desaguar el agua del foso lleno en el transcurso de 12 horas, en tanto que 15
bombas de ese mismo tipo, en 6 horas. Cunto tiempo es necesario para vaciar el agua del
foso lleno empleando 25 bombas como las indicadas al trabajar ellas conjuntamente?
Resp: En 3 horas.
138. Dos fbricas, trabajando juntas, deben transformar cierta cantidad de materia prima. Si el
rendimiento de la segunda fbrica aumentara el doble el tiempo necesario para que las fbri-
cas realizaran el trabajo disminuira en
2
15
del tiempo que se requerira para que la primera
fbrica cumpliera toda la tarea. En qu fbrica el rendimiento es ms alto y cuntas veces,
si sabemos que cada una de las fbricas transform no menos de
1
3
de todo el volumen de la
materia prima?
Resp: El rendimiento de la segunda fbrica es 2 veces mayor.
139. Dos brigadas, trabajando juntas, cavaron una zanja en 2 das. Despus de esto, ellas comen-
zaron a cavar otra zanja de la misma profundidad y anchura, pero de una longitud 5 veces
mayor. Con esto, comenz a trabajar una brigada y, a continuacin, fue sustituida por la se-
gunda que realiz una vez y media menos trabajo que la primera brigada. La segunda zanja
fue acabada en 21 das. En el transcurso de cuntos das hubiera podido cavar la segunda
brigada la primera zanja, si sabemos que el volumen de trabajo realizado por la primera
brigada por 1 da es mayor que el ejecutado por 1 da por la segunda brigada?
Resp: En 6 das.
140. Un recipiente se llena de agua por 5 tubos. Con el primer tubo el recipiente se llena de agua
en 40 minutos, con el segundo, tercero y cuarto tubos, funcionando al mismo tiempo, en 10
minutos, con el segundo, tercero y quinto tubos, trabajando conjuntamente, en 20 minutos
y, por n, con el quinto y cuarto, en 30 minutos. Cunto tiempo es necesario para llenar el
recipiente si los 5 tubos trabajan juntos?
Resp: En
60
7
minutos.
141. Tres lneas automticas producen iguales artculos, pero tienen diferente rendimiento. El
rendimiento de las tres lneas, al funcionar simultneamente, es 1,5 veces mayor que el de
la primera y segunda lneas al trabajar juntas. La tarea de turno para la primera lnea, la
segunda y la tercera lneas, trabajando conjuntamente, pueden cumplirla 4 horas 48 minutos
antes que la primera lnea; esa misma tarea se cumple por la segunda lnea 2 horas ms rpido
que por la primera. Hallar el tiempo necesario para que la primera lnea cumpla la tarea de
turno.
Resp: 8 horas.
142. Dos tractores aran una parcela dividida en dos partes iguales. Ambos tractores comenzaron
a trabajar en su correspondiente parte al mismo tiempo. Pasadas 5 horas despus del mo-
mento cuando ellos, en conjunto, haban arado la mitad de toda la parcela, se aclar que al
primer tractor le queda por arar
1
10
de su parte y al segundo,
2
5
de la suya. Cunto tiempo
necesitar el segundo tractor para arar el campo?
CAPTULO 6. ECUACIONES Y SISTEMAS DE ECUACIONES 351
Resp: 50 horas.
143. Tres excavadoras estn ocupadas en la excavacin de un foso. La diferencia entre los
rendimientos de la primera y tercera excavadoras es 3 veces mayor que la diferencia en-
tre los rendimientos de la tercera y segunda excavadora. La primera excavadora realiza
4
5
de
todo el trabajo, empleando para ello cierto tiempo. Igual intervalo de tiempo ser necesario
si, primero, la segunda excavadora realiza
1
15
de toda la tarea y, a continuacin, la tercera
excavadora
9
18
del trabajo restante. Cuntas veces es mayor el rendimiento de la primera
excavadora que el de la segunda?
Resp: 3 veces.
144. Un mismo trabajo puede ser realizado por tresw brigadas. En el transcurso de cierto tiem-
po, la primera brigada realiza
2
3
de todo el trabajo. Ese mismo tiempo ser preciso si, primero,
la tercera brigada hace
1
3
de toda la tarea y, a continuacin, la segunda brigada efecta
9
10
del trabajo restante. El rendimiento de la tercera brigada es igual a la semisuma de los
rendimientos de las brigadas primera y segunda. Cuntas veces es mayor el rendimiento de
la segunda brigada que el de la tercera?
Resp:
6
5
veces.
145. Trabajando juntas, dos brigadas de estuquistas estucaron en 6 das una casa de vivienda.
En otra ocasin ellas estucaron un club y realizaron un volumen de trabajo tres veces mayor
que al trabajar en la vivienda. En el club primero trabaj la primera brigada y, despus,
fue sustituida por la segunda que acab el trabajo, con la particularidad de que la primera
brigada realiz un trabajo dos veces mayor que la segunda. El club fue estucado por ella en
35 das. En cuntos das podra haber estucado la primera brigada la casa de vivienda, si
sabemos que la segunda brigada hubiera invertido para ello ms de 14 das?
Resp: en 10 das.
146. Una compra consta de tres objetos: A, B, C. Si A fuera 5, B, 2 y C, 2,5 veces ms barato,
la compra costara 8 dlares. si el objeto A fuera 2, B, 4 y C, 3 veces ms barato, el precio
de la compra sera 12 dlares. Cunto cuesta toda la compra y qu es ms caro, A o B?
Resp: A es 28 dlares ms caro.
147. Al mezclar una disolucin al 40 % de cido con una disolucin al 10 % de cido, se ob-
tuvieron 800 gramos de una disolucin al 20 %. Cuntos gramos de cada disolucin fueron
tomados con este objeto?
Resp: 300 gramos y 500 gramos.
148. Tenemos 735 gramos de una disolucin al 21, 25 % de yodo en alcohol. Hay que obtener
una disolucin de yodo al 10 %. Cuntos gramos de alcohol hay que aadir a la disolucin
que tenamos?
Resp: 441 gramos.
CAPTULO 6. ECUACIONES Y SISTEMAS DE ECUACIONES 352
149. Hay acero de dos marcas, una de las cuales contiene el 5 % de nquel y la otra, el 10 %.
Cuntas toneladas de cada una de estas marcas de acero hay que tomar para producir una
aleacin que contenga el 8 % de nquel, si en el segundo trozo hay 4 toneladas ms de nquel
que en el primero?
Resp: 40 toneladas y 60 toneladas.
150. En 500 kg de mineral hay cierta cantidad de hierro. despus de extraer de la mena 200
kilgramos de impurezas, que contenan, por trmino medio, el 12, 5 % de hierro, el porcentaje
de hieroo en el resto de la mena aument el 20 %. Cunto hierro qued en la mena?
Resp: 187,5 kilogramos.
151. Una mena contiene el 40 % de impurezas, mientras que el metal fundido de ella, el 4 % de
ellas. Qu cantidad de metal se ontendr de 24 toneladas de metal?
Resp: 15 toneladas.
152. De 40 toneladas de mena se funden 20 toneladas de metal con un contenido del 6 % de
impurezas. Cul es el porcentaje de impurezas en la mena?
Resp: El 53 %.
153. De 38 toneladas de materia prima de segunda calidad, que contiene el 25 % de impurezas,
despus de la transformacin se producen 30 toneladas de materia prima de primera calidad.
Cul es el porcentaje de impurezas en la materia prima de primera calidad?
Resp: El 5 %.
154. Los hongos frescos contienen el 90 % de agua, los secos, el 12 %. Cuntos hongos secos se
obtienen de 88 kilogramos de hongos frescos?
Resp: 10 kilogramos.
155. Las abejas que transforman el nctar de las ores en miel, lo liberan en una considerable
parte de agua. ?Cuntos kilogramos de nctar han de transformar las abejas para obtener 1
kilogramo de miel, si sabemos que el nctar contiene un 70 % de agua y la miel que de l se
obtiene, el 17 %?
Resp: Aprximadamente 2,77 kilogramos.
156. Dos aleaciones contienen dos metales. La primera aleacin contiene los metales en una
razn de 1 : 2, la segunda, de 3:2. En qu razn hay que tomar partes de estas aleaciones,
para obtener una nueva aleacin con una razn de los metales de 8 : 7?
Resp: 1 : 3.
157. Hay dos disoluciones de un cido de diferente concentracin. El volumen de una de las
disoluciones es de 4 litros, el de la otra, 6 litros. Si stas se juntan, entonces obtenemos una
disolucin del cido al 35 %. Sin embargo, si se juntan iguales volmenes de dichas disolu-
ciones, obtenemos una disolucin del cido al 36 %. Cuntos litros de cido contiene cada
una de las disoluciones iniciales?
CAPTULO 6. ECUACIONES Y SISTEMAS DE ECUACIONES 353
Resp: 1,64 litros y 1,86 litros.
158. 40 kilogramos de una disolucin de sal se echaron en dos recipientes de forma que en el
segundo recipiente result haber 2 kilogramos ms de sal pura que en el primero. Si aadimos
al segundo recipiente 1 kilogramo de sal, la cantidad de sta en l ser dos veces mayor que
en el primer recipente. Hallar la masa de la disolucin contenida en el primer recipiente.
Resp: 15 kilogramos.
159. Tenemos tres lingotes. La masa del primero es de 5 kilogramos, la del segundo, de 3 kilo-
gramos y cada uno de ellos contiene el 30 % de cobre. Si el primer lingote se funde junto
con el tercero, obtenemos un lingote que contiene el 56 % de cobre, mientras que al fundir
conjuntamente los lingotes segundo y tercero, se obtiene un lingote que contiene el 60 %.
Hallar la masa del tercer lingote y el porcentaje de cobre en l.
Resp: 10 kilogramos, el 69 %.
160. Hay dos lingotes de oro con plata. El porcentaje de oro en el primer lingote es 2,5 veces
mayor que en el segundo. Si fundimos juntos ambos lingotes, se obtiene un lingote en el que
habr el 40 % de oro. Cuntas veces la masa del primer lingote es mayor que la del segundo,
si se conoce que al fundir partes de igual masa de los lingotes primero y segundo se obtiene
un lingote que contiene el 35 % de oro?
Resp: 2 veces.
161. Una aleacin de cobre y plata contiene 2 kilogramos de cobre ms que de plata. Si aadi-
mos a la aleacin
9
16
de la cantidad de plata que ella contiene, el porcentaje de plata en la
nueva aleacin ser igual al porcentaje de cobre en la aleacin inicial. Hallar la masa de sta.
Resp: 18 kilogramos.
162. Hay que tomar varios litros de un lquido a la temperatura a

y otra cantidad de ese mismo


lquido, pero a la temperatura b

, para obtener la temperatura c

de la mezcla. No obstante,
del segundo lquido se tom tanto como se supona tomar del primero y viceversa. Qu
temperatura de la mezcla se obtuvo?
Resp: a +b c.
163. Un recipiente de 12 litros de capacidad est lleno de un cido. De l se vierte cierta can-
tidad de cido al segundo recipiente de la misma capacidad y ste se rellena de agua. A
continuacin, el primer recipiente se llena con la mezcla del segundo. Despus de esto, del
primer recipiente se echan 4 litros al segundo, tras lo cual en ambos recipientes la cantidad
de cido puro (en las disoluciones) resulta ser igual. Cunto cido fue vertido inicialmente
del primer recipiente al segundo?
Resp: 6 litros.
164. En un recipiente con agua se echaron 6 litros de una disolucin de alcohol al 64 % y, a
continuacin, tras realizar el mezclado completo, se vertieron 6 litros de la disolucin obteni-
da. Semejante operacin se efecta 3 veces. Qu cantidad de agua haba inicialmente en el
recipiente, si la concentracin denitiva del alcohol se hizo igual al 37 %
CAPTULO 6. ECUACIONES Y SISTEMAS DE ECUACIONES 354
Resp: 18 litros.
165. Un trozo de 6 kilogramos de masa de una aleacin contiene cobre. El trozo de otra aleacin
de 8 kilogramos de masa contiene cobre en un porcentaje dos veces menor que en el primer
trozo. De ste se ha cortado cierta parte y del segundo trozo se corta una parte que por su
masa es dos veces mayor que la cortada del primer trozo. Cada una de estas partes se funden
con el resto del otro trozo, despus de lo cual se obtuvieron dos nuevas aleaciones con igual
porcentaje de cobre. Cul es la masa de cada una de las partes cortadas inicialmente de los
trozos?
Resp: 2,4 kilogramos y 4,8 kilogramos.
166. De un recipiente lleno de glicerina se han vertido 2 litros de sta y a la glicerina restante
aadieron 2 litros de agua. Despus del mezclado se sacaron 2 litros de la mezcla y aadieron
2 litros de agua. Por n, se realiz de nuevo la agitacin de la mezcla y de ella se sacaron 2
y aadieron 2 litros de agua. Como resultado de estas operaciones el volumen de agua en el
recipiente es 3 litros mayor que el volumen de glicerina que en l queda. Cuntos litros de
glicerina y de agua quedaron en el recipiente a consecuencia de las operaciones realizadas?
Resp: 3,5 litros de glicerina y 0,5 litros de agua.
167. De dos depsitos, uno est lleno de glicerina y el segundo, de agua. Se toman dos cucharones
de tres litros. Con el primer cucharn se saca el contenido del primer depsito y, con el se-
gundo, el contenido del segundo depsito, despus de lo cul el primer cucharn se vierte al
segundo depsito y el segundo cucharn, al primer depsito. A continuacin, tras realizar el
mezclado, esta operacin se realiz una vez ms y, como resultado, la glicerina pura ocup la
mitad del primer depsito. Hallar los volmenes de los depsitos, si se conoce que su volumen
sumario es 10 veces mayor que el del primer depsito.
Resp: 10 litros.
168. Despus de fundir dos trozos de arrabio de igual masa con diferente contenido de cromo,
fue obtenida una aleacin que contena 12 kilogramos de cromo. Si la masa del primer trozo
hubiera sido dos veces mayor, en la aleacin habra 16 kilogramos de cromo. Se sabe que el
contenido de cromo en el primer trozo era el 5 % menor que en el segundo. Hallar el por-
centaje de cromo en cada uno de los trozos de arrabio.
Resp: El 5 % y el 10 %.
169. Tenemos tres aleaciones. La primera contiene el 60 % de aluminio, el 15 % de cobre y el
25 % de magnesio, la segunda, el 30 % de cobre y el 70 % de magnesio, la tercera, el 45 % de
aluminio y el 55 % de magnesio. Es preciso producir de ellas una aleacin con un contenido
del 20 % de cobre. Qu porcentaje mnimo y mximo de aluminio puede haber en la nueva
aleacin?
Resp: El 15 % y el 40 %.
Captulo 7
Desigualdades e inecuaciones
7.1. Desigualdades con una incgnita y de primer grado
7.1.1. La recta real
Suponemos conocidos los nmeros reales, as como su representacin en la recta real. Los
nmeros reales se pueden representar mediante expresiones decimales nitas o innitas. Si la ex-
presin decimal es nita o peridica innita, entonces el nmero real se puede expresar como el
cociente de dos nmeros enteros y se dice que el nmero real es racional. Recprocamente cualquier
nmero racional (cociente de dos enteros) se puede expresar mediante una expresin decimal nita
o innita peridica. Cuando la expresin decimal tiene innitas cifras que no se repiten de manera
peridica se dice que el nmero real es irracional.
Los nmeros reales admiten una representacin geomtri-
ca en una recta. En dicha representacin cada nmero real
se representa por un solo punto de la recta y cada punto de
la recta representa un solo nmero real. En consecuencia, hablaremos indistintamente de nmero o
de punto. Por convenio, los nmeros positivos se representan a la derecha del cero y los negativos
a la izquierda. Se llama recta real a una recta en la que se han representado los nmeros reales.
7.1.2. Segmentos, desigualdades e intervalos
La representacin como puntos sobre una recta revela claramente la ordenacin de los nmeros
reales: cuando a es menor que b (lo que se escribe a < b), entonces a aparece a la izquierda de b.
Si a < b, el conjunto de todos los nmeros que se encuentran entre a y b se llama intervalo
abierto de a a b, y se le representa por (a; b). En trminos precisos, (a; b) es el conjunto de todos
los nmeros x tales que a < x < b, el cual se denomina intervalo. (La notacin a < x < b quiere
decir que ha de tenerse tanto a < x, como x < b.) A los nmeros a y b se les llama extremos del
intervalo (a; b). Ntese que el intervalo abierto (a; b) no incluye a sus puntos extremos.
Si aadimos al intervalo abierto (a; b) sus puntos extremos, obtenemos el intervalo cerrado [a; b],
es decir, el conjunto de todas las x, tales que a x b, el cual se denomina segmento. La notacin
a x se lee, a menor o igual que x, y quiere decir que a < x o a = x.
355
CAPTULO 7. DESIGUALDADES E INECUACIONES 356
Ntese que al expresar un intervalo, un corchete o un punto, indican un extremo que est in-
cluido, mientras que un parntesis o la ausencia de un punto indican un extremo que est excluido.
La denicin de intervalos puede hacerse en forma breve y concisa mediante una notacin de
conjuntos estndar. La frase: El conjunto de todos los nmeros x tales que a < x < b, se escribe
x R / a < x < b. En el interior del parntesis damos primero el smbolo x que representa a
ciertos nmeros, y luego las condiciones de x que caracterizan los nmeros del conjunto.
Con esta notacin, los intervalos abiertos y cerrados pueden denirse como sigue:
Denicin 7.1 Intervalos
Sean a y b dos nmeros reales tales que a b. Se llama intervalo abierto de extremos a y b, al
conjunto de puntos comprendidos entre a y b, excluidos dichos puntos
(a; b) = x R / a < x < b
Se llama intervalo cerrado de extremos a y b, al conjunto de puntos comprendidos entre a y b,
incluidos dichos puntos
[a; b] = x R / a x b
Se llama intervalo semiabierto por la izquierda de extremos a y b, al conjunto de puntos compren-
didos entre a y b, excluido el punto a
(a; b] = x R / a < x b
Se llama intervalo semiabierto por la derecha de extremos a y b, al conjunto de puntos comprendidos
entre a y b, excluido el punto b
[a; b) = x R / a x < b
Denicin 7.2 Intervalos innitos
Se llama intervalo cerrado innito de extremo izquierdo a, al conjunto de puntos, incluido el punto
a
[a; +) = x R / a x
Se llama intervalo cerrado innito de extremo derecho b, al conjunto de puntos, incluido el punto
b
(; b] = x R / x b
Se llama intervalo abierto innito de extremo izquierdo a, al conjunto de puntos, excluido el punto
a
(a; +) = x R / a < x
Se llama intervalo abierto innito de extremo derecho b, al conjunto de puntos, excluido el punto b
(; b) = x R / x < b
Se llama intervalo abierto innito, al conjunto de puntos denido en R
(; ) = R
Denicin 7.3 Desigualdad
Dos nmeros o dos expresiones algebraicas, relacionadas entre s por el signo < (menor), o por el
signo > (mayor), o por el signo ,= (no es igual), se denomina desigualdad.
CAPTULO 7. DESIGUALDADES E INECUACIONES 357
Ejemplo 7.1 Se cumplen las siguientes expresiones:
25 > 3; 3 < 7; 5 ,= 8.
Denicin 7.4 Desigualdades de sentido contrario
Dos desigualdades de tipo a < b, c < d o a > b, c > d se denominan desigualdades del mismo
sentido. Dos desigualdades de tipo a > b y c < d se denominan desigualdades de sentido contrario.
Ejemplo 7.2 Desigualdades del mismo sentido: a
2
+ 2 > a y 4 > 1. Desigualdades de
sentido contrario: 1 < 7 y 3 > 3.
A veces a los signos > o < se une tambin el signo de igualdad o . Tales desigualdades se
denominan no rigurosas.
Exponemos a continuacin las propiedades ms importantes para las desigualdades numricas:
1. Si los nmeros a, b y c son tales, que a > b y b > c, entonces a > c.
2. Si los nmeros a, b, c, d son tales, que a > b, c > d, entonces a +c > b +d.
3. Si los nmeros a, b, c, d son tales, que a > b y c < d, entonces a c > b d.
4. Si a, b, c, d son nmeros positivos y, adems, a > b y c > d, entonces ac > bd.
5. Para cualesquiera nmeros reales a, b y c, las desigualdades a > b y a + c > b + c son
equivalentes, es decir, la validez de la desigualdad a > b predetermina que es vlida la
desigualdad a + c > b + c, y, viceversa, de la validez de la desigualdad a + c > b + c se
desprende la validez de la desigualdad a > b, es decir, a > b si y slo si a +c > b +c.
6. Para cualesquiera nmeros reales a y b y para todo nmero positivo c, las desigualdades
a > b y ac > bc son equivalentes, es decir, si c > 0, entonces a > b si y slo si ac > bc.
7. Para cualesquiera nmeros reales a y b y para todo nmero negativo c, las desigualdades
a > b y ac < bc son equivalentes, es decir, si c < 0, entonces a > b si y slo si ac < bc.
CAPTULO 7. DESIGUALDADES E INECUACIONES 358
7.1.3. Operaciones entre desigualdades
Denicin 7.5 Suma
Dos o varias desigualdades del mismo sentido se pueden sumar miembro a miembro; como resultado
se obtendr una desigualdad del mismo sentido. Es decir
a
1
> b
1
a
2
> b
2
...
a
n
> b
n
a
1
+a
2
+... +a
n
> b
1
+b
2
+... +b
n
Denicin 7.6 Resta
Las desigualdades de sentido contrario se pueden restar miembro a miembro; como resultado ob-
tendremos una desigualdad del mismo sentido que la desigualdad minuendo. Es decir, si a > b y
c < d, y de la primera desigualdad restamos la segunda, entonces a c > b d.
Denicin 7.7 Multiplicacin
Dos o varias desigualdades de igual sentido se pueden multiplicar entre s miembro a miembro si
todos sus miembros son positivos; como resultado se obtiene una desigualdad del mismo sentido.
Es decir, si a
i
> 0, entonces
a
1
< b
1
a
2
< b
2
...
a
n
< b
n
a
1
a
2
... a
n
< b
1
b
2
... b
n
Denicin 7.8 Divisin
Dos desigualdades de sentido contrario se pueden dividir miembro a miembro si todos los miembros
de la desigualdad son nmeros positivos; como resultado se obtiene una desigualdad del mismo
sentido que la desigualdad dividiendo, es decir, la desigualdad que dividimos por la otra. Es decir,
si a > b, b > 0 y c < d, c > 0, entonces
a
c
>
b
d
.
7.1.4. Valor absoluto
Dado un nmero real cualquiera, a mrquesele a [a[ unidades del origen; a la derecha si a es
positivo, y a la izquierda si a es negativo. El smbolo [a[ se emplea aqu para representar el valor
absoluto de a denido por
_

_
[a[ = a, a > 0
[0[ = 0, a = 0
[a[ = a, a < 0
Ahora bien, teniendo en cuenta que para a = 0 es vlida la igualdad [a[ = a, podemos escribir ms
brevemente la siguiente denicin.
CAPTULO 7. DESIGUALDADES E INECUACIONES 359
Denicin 7.9 Valor absoluto
Se llama valor absoluto de un nmero real a, y se denota por el smbolo [a[, a dicho nmero si es
positivo o cero, y a su opuesto si es negativo
_
a, a 0
a, a < 0
De la denicin se deduce que para cualquier nmero a se verica la desigualdad a [a[. La
interpretacin geomtrica del valor absoluto est implcita en las presentaciones que se ha dado
para marcar puntos sobre la recta, a saber: [a[ es la distancia entre a y 0. En general, [a b[ es la
distancia entre a y b.
El valor absoluto tambin se puede denir de la siguiente manera
[a[ = mxa, a
Al valor absoluto de un nmero tambin se le llama su mdulo.
El valor absoluto de un nmero nunca es negativo. Puede sorprender que a sea positivo, sin
embargo, esto no es nada sorprendente, ya que podemos pensar en (5) = +5 que tambin es
positivo, a pesar del signo menos inicial, ya que los dos signos menos se compensan. Igual ocurre
con a donde el signo menos que aparece de manera explcita se compensa con el signo menos que
a tiene implcitamente, ya que hemos supuesto, en el segundo apartado, que a es negativo.
Ejemplo 7.3 Eliminar el valor absoluto en las siguientes expresiones:
a) [2 +

7[; b) [2 +

15[.
Solucin
Tenemos que comprobar si la expresin que hay dentro del valor absoluto da como resultado un
nmero positivo o negativo, si es positivo la dejamos igual, y si es negativo la cambiamos de signo
para convertirlo en positivo. Es decir:
a) [2 +

7[ = 2 +

7;
b) [2 +

15[ = (2 +

15) = 2

3 +

15.
De la denicin se deducen varias propiedades del valor absoluto de un nmero:
1. Para cualquier nmero real a, entonces

a
2
= [a[.
2. Para cualquier nmero real a, entonces [a[ 0.
3. Para cualquier nmero real a, entonces [a[ = [ a[.
4. Para cualquier nmero real a, entonces [a[ a [a[.
5. El valor absoluto del producto es igual al producto de los valores absolutos de los factores.
Es decir, si a y b son nmeros reales, entonces
[ab[ = [a[[b[
6. El valor absoluto del cociente es igual al cociente de dividir el valor absoluto del dividendo
por el del divisor. Es decir, si a y b son nmeros reales, entonces

a
b

=
[a[
[b[
CAPTULO 7. DESIGUALDADES E INECUACIONES 360
7. Si a es un nmero real y n es un entero, entonces
[a
n
[ = [a[
n
8. Para cualquier nmero real a y cualquier nmero positivo c, entonces [a[ < c si y slo si
c < a < c.
9. Para cualesquiera nmeros reales a y b y cualquier nmero positivo c, entonces [a b[ < c
si y slo si b c < a < b +c.
10. Para cualquier nmero real a y cualquier nmero positivo c, entonces [a[ > c si y slo si
a > c o bien a < c.
Teorema 7.1 Sea un nmero positivo. entonces las desigualdades [a[ y a son
equivalentes.
Teorema 7.2 El valor absoluto de la suma algebraica de varios nmeros reales no es mayor que
la suma de los valores absolutos de los sumandos. Es decir, para cualesquiera nmeros reales a y
b, entonces
[a +b[ [a[ +[b[
Ntese que [a b[ [a[ +[b[. Efectivamente
[a b[ = [a + (b)[ [a[ +[ b[ = [a[ +[b[
Teorema 7.3 El valor absoluto de la diferencia de dos nmeros no es menor que la diferencia
de los valores absolutos del minuendo y sustraendo. Es decir, para cualesquiera nmeros reales a y
b, entonces
[a b[ [a[ [b[
Ntese que [a +b[ [a[ [b[. Efectivamente
[a +b[ = [a (b)[ [a[ [ b[ = [a[ [b[
Y en conclusin ntese, adems, que cualesquiera que sean dos nmeros a y b tienen lugar las
relaciones
[a b[ = [a[ [b[ y

a
b

=
[a[
[b[
si b ,= 0
Teorema 7.4 El valor absoluto de la diferencia de dos mdulos no es mayor que el valor absoluto
de la diferencia, es decir
[[a[ [b[[ [a b[
La nocin de valor absoluto surge de una manera natural en problemas de distancia. En una
recta coordenada, sean A y B puntos con coordenadas a y b. Debido a que la distancia es siempre
no negativa, la distancia d, entre A y B es d = b a, cuando B est a la derecha de A, y d = ab,
cuando B est a la izquierda de A.
En el primer caso, b a es positiva, de modo que puede escribirse
d = b a = [b a[
y en el segundo caso, b a es negativa, de modo que puede escribirse
d = a b = (b a) = [b a[
Por tanto, independientemente de si B est a la derecha o a la izquierda de A, la distancia d entre
A y B es d = [b a[. Esta frmula es til cuando se desconocen las posiciones relativas de A y B.
CAPTULO 7. DESIGUALDADES E INECUACIONES 361
7.2. Desigualdades de primer grado con una incgnita
Denicin 7.10 Desigualdad algebraica con una incgnita
Supongamos que se pide resolver la desigualdad
r(x) > q(x) (o r(x) < q(x))
donde r(x) y q(x) son ciertos polinomios enteros, respecto de una incgnita; la desigualdad lleva
el nombre de desigualdad algebraica con una sola incgnita.
Por cuanto el conjunto existencial de los polinomios r(x) y q(x) se compone de todos los
nmeros reales, el problema sobre la resolucin de la desigualdad puede enunciarse de la siguiente
manera: hllense todos los valores numricos de la incgnita x, cada uno de los cuales convierte la
desigualdad, en una desigualdad numrica que es verdadera. Cada valor numrico semejante recibe
el nombre de solucin de la desigualdad. Por eso, resolver la desigualdad signica hallar el conjunto
de todas sus soluciones. Si resulta que el conjunto de todas las soluciones de la desigualdad es un
conjunto vaco, se dice que la desigualdad no tiene soluciones.
Denicin 7.11 Desigualdades equivalentes
Dos desigualdades algebraicas r(x) < q(x) y t(x) < s(x) se denominan equivalentes, si cualquier
solucin de la primera desigualdad es tambin solucin de la segunda y, viceversa, cualquier solu-
cin de la segunda desigualdad es solucin de la primera.
En virtud de esta denicin, son equivalentes cualesquiera dos desigualdades que no tienen
soluciones. La sustitucin de una desigualdad por otra, equivalente a la primera, recibe el nombre
de paso equivalente de una desigualdad a la otra. El paso equivalente suele designarse con una
echa doble . La escritura
r(x) > q(x) t(x) < s(x)
signica que las desigualdades r(x) > q(x) y t(x) < s(x) son equivalentes.
A continuacin damos a conocer algunas propiedades con cuya ayuda se realizarn los pasos
equivalentes:
1. Las desigualdades r(x) > q(x) y r(x) q(x) > 0 son equivalentes.
2. Las desigualdades r(x) > q(x) y r(x)+k > q(x)+k son equivalentes para cualquier nmero
real k.
3. Las desigualdades r(x) > q(x) y kr(x) > kq(x) son equivalentes para cualquier nmero
positivo k.
4. Las desigualdades r(x) > q(x) y kr(x) < kq(x) son equivalentes para cualquier nmero
negativo k.
5. Supongamos que se conoce que para cualquier nmero real x se verica la igualdad r(x) =
t(x), entonces son equivalentes las desigualdades r(x) > q(x) y t(x) < q(x).
Denicin 7.12 Solucin de una desigualdad
Se denomina solucin de una desigualdad, a todo valor de x que satisface a la desigualdad da-
da. Resolver una desigualdad signica hallar todos los valores de la incgnita que verican a la
desigualdad dada. La bsqueda de la solucin de cualquier desigualdad de primer grado con una
incgnita da lugar a desigualdades elementales de la forma x > a o x < a.
CAPTULO 7. DESIGUALDADES E INECUACIONES 362
En el primer caso se dice que el numero a es el limite inferior de los valores de la incgnita, lo
cual quiere decir que cualquier numero mayor que el numero a es solucin de la desigualdad dada.
Si sobre el eje numrico se lleva el punto correspondiente al numero a, los valores de la incgnita x
que verican la desigualdad x > a, se representan por los puntos que se encuentran a la derecha del
punto x = a. En la desigualdad x < a el numero a se denomina limite superior de la incgnita, lo
que signica que, cualquier numero menor que a es una solucin de esta desigualdad. La desigual-
dad x < a se ilustra del siguiente modo: sobre el eje numrico se marca el punto correspondiente
al nmero a; en tal caso, cualquier punto ubicado a la izquierda de a representa al nmero que
verica la desigualdad dada.
Supongamos que se pide resolver la desigualdad ax+b > 0, a ,= 0, la cual se denomina desigual-
dad de primer grado. En virtud de la propiedad 2, sta desigualdad es equivalente a la desigualdad
ax > b, a ,= 0.
Examinemos los casos en que a > 0 y a < 0. Sea a > 0, entonces, teniendo presente la propiedad
3, la desigualdad es equivalente a la desigualdad x >
b
a
, a ,= 0. Es evidente que cualquier x del
intervalo
_

b
a
; +
_
satisface la desigualdad anterior. Por consiguiente, el conjunto de todas las
soluciones de sta desigualdad es el intervalo
_

b
a
; +
_
. Por cuanto la desigualdad ax + b > 0
es equivalente, para a > 0, a la desigualdad x >
b
a
, el conjunto de todas las soluciones de la
desigualdad ax +b > 0 tambin ser el intervalo
_

b
a
; +
_
.
Todos los pasos equivalentes de la desigualdad ax + b > 0 a la desigualdad ax > b y, luego,
a la desigualdad evidente x >
b
a
se escriben ms brevemente en forma de los siguientes pasos
equivalentes:
ax +b > 0 (a > 0) ax > b (a > 0) (a > 0);
ax +b > 0 (a < 0) ax > b (a < 0) (a < 0);
ax +b < 0 (a > 0) ax < b (a > 0) (a > 0);
ax +b < 0 (a < 0) ax < b (a < 0) (a < 0).
A partir de la ltima desigualdad en cada una de estas equivalencias se halla fcilmente el con-
junto de todas las soluciones de la primera desigualdad dada, con la restriccin indicada sobre a.
As, la solucin de la desigualdad ax+b > 0, para a < 0, se representa por el intervalo
_
;
b
a
_
;
la solucin de la desigualdad ax +b < 0, para a > 0, es el intervalo
_
;
b
a
_
; y la solucin de la
desigualdad ax +b < 0, para a < 0, es el intervalo
_

b
a
; +
_
.
Todo lo expuesto anteriormente, concerniente a la resolucin de las desigualdades de primer
grado se enuncia de la siguiente manera: un polinomio de primer grado ax +b (a ,= 0):
1. Es positivo, cuando a > 0, para cualquier x
_

b
a
; +
_
y negativo para cualquier
x
_
;
b
a
_
.
2. Es positivo, cuando a < 0, para cualquier x
_
;
b
a
_
y negativo, para cualquier
x
_

b
a
; +
_
.
En particular, el binomio (x k) es positivo para todos los x que se ubican en el eje numrico
a la derecha respecto del punto que representa el nmero k, y negativo para todo x que se dispone
a la izquierda del punto mencionado. En otras palabras, el punto k divide el eje numrico en dos
partes: en la parte dispuesta a la derecha del punto k el binomio (x k) es positivo, y en la otra
parte, dispuesta a la izquierda del punto k, negativo. En esta propiedad del polinomio (x k) se
CAPTULO 7. DESIGUALDADES E INECUACIONES 363
sabe el mtodo de intervalos y se emplea con frecuencia para resolver las desigualdades algebraicas
de grados superiores.
Ejemplo 7.4 Resuelva las inecuaciones:
a)
x + 1
4
< 2
1
2

1 2x
3
; b)
7x
4
< 0, 3(x + 7) + 2
1
5
.
Solucin
a) Operando, como si se tratara de una ecuacin, resulta:
x + 1
4
<
5
2

1 2x
3

x + 1
4
<
13 + 4x
6
13x + 23 > 0 x >
23
13
.
Por tanto el conjunto solucin es el intervalo
_

23
13
; +
_
.
b) De igual forma que en la desigualdad anterior, operamos como si fuera una ecuacin y resulta:
7x
4
<
3(x + 7)
10
+
11
5

7x
4
<
3x + 43
10
29x 86 < 0 x <
86
29
.
El conjunto solucin es el intervalo
_
;
86
29
_
.
Ejemplo 7.5 Resuelva las inecuaciones:
a) (3x 2)(2x 3) (2x 1)(x 2) + 6x (2x 3)
2
;
b)
(3x 4)(3x + 1)
3

(8x 11)(x + 2)
4

(6x 1)(2x 3)
12
.
Solucin
a) Operando, como si se tratara de una ecuacin, resulta:
6x
2
13x + 6 2x
2
+ 5x + 6x 4x
2
12x + 9 10x 3 0 x >
3
10
.
Por tanto el conjunto solucin es el intervalo
_
3
10
; +
_
.
b) Siguiendo el procedimiento anterior, resulta:
4(9x
2
9x 4) 3(8x
2
+ 5x 22) 12x
2
20x + 3 31x + 47 0 x
47
31
.
El conjunto solucin es el intervalo
_
;
47
31
_
.
Las desigualdades del tipo
_
a
1
x +b
1
> 0
a
2
x +b
2
< 0
o
_
a
1
x +b
1
> 0
a
2
x +b
2
> 0
con respecto a las cuales se buscan sus soluciones generales, forman un sistema de desigualdades
de primer grado con una incgnita.
El mtodo general de resolucin del sistema de dos desigualdades tiene como objeto lo siguiente:
hallamos las soluciones de cada desigualdad por separado y comparndolas establecemos cuales de
las soluciones son comunes para ambas desigualdades; si no existen soluciones generales, el sistema
es incompatible, o contradictorio. La eleccin de las soluciones generales se facilita si las soluciones
de cada desigualdad se representan sobre el eje numrico.
CAPTULO 7. DESIGUALDADES E INECUACIONES 364
Ejemplo 7.6 Resuelva la inecuacin
4[x + 2[ < 2x + 10
Solucin
Aplicamos una propiedad del valor absoluto:
[x + 2[ <
x + 5
2

x + 5
2
< x + 2 <
x + 5
2
de donde obtenemos un sistema de desigualdades
_
x + 2 >
x+5
2
x + 2 <
x+5
2
Se trata de hallar la interseccin de los conjuntos solucin de cada una de las desigualdades. Para
ello, resolvemos ambas inecuaciones por separado
_
2x + 4 > x 5
2x + 4 < x + 5

_
x > 3
x < 1
Intersecando las soluciones, obtenemos el intervalo solucin (-3; 1).
Ejemplo 7.7 Resuelva la inecuacin
[x + 1[ [3x + 7[ > 0
Solucin
Denimos los valores absolutos:
[x + 1[ =
_
x + 1, x 1
x 1, x < 1
y [3x + 7[ =
_
3x + 7, x
7
3
3x 7, x <
7
3
Tenemos cuatro combinaciones posibles:
1.
_
(x + 1) (3x + 7) > 0
x 1, x
7
3

_
2x + 6 < 0
x 1, x
7
3

_
x < 3
x 1, x
7
3
Intersecando las soluciones, obtenemos el intervalo solucin x .
2.
_
(x + 1) (3x 7) > 0
x 1, x <
7
3

_
x + 2 > 0
x 1, x <
7
3

_
x > 2
x 1, x <
7
3
Intersecando las soluciones, obtenemos el intervalo solucin x .
3.
_
(x 1) (3x + 7) > 0
x < 1, x
7
3

_
x + 2 < 0
x < 1, x
7
3

_
x < 2
x < 1, x
7
3
Intersecando las soluciones, obtenemos el intervalo solucin x
_

7
3
; 2
_
.
4.
_
(x 1) (3x 7) > 0
x < 1, x <
7
3

_
x + 3 > 0
x < 1, x <
7
3

_
x > 3
x < 1, x <
7
3
Intersecando las soluciones, obtenemos el intervalo solucin x
_
3;
7
3
_
. La solucin general de
la desigualdad (gura 4.3) se obtiene uniendo las cuatro soluciones parciales, lo cual es evidente
que x (3; 2).
CAPTULO 7. DESIGUALDADES E INECUACIONES 365
En general, el mtodo ms directo de atacar un problema referente a valores absolutos requiere
la consideracin por separado de distintos casos, con objeto de eliminar el valor absoluto. En
particular, siempre habr que considerar si lo que hay dentro del valor absoluto es positivo o es
negativo. Esto hace que cuando aparecen varios valores absolutos, la casustica se complique, ya
que hay que considerar, por separado, todas las posibilidades, en cuanto al signo, de las expresiones
que hay dentro de cada uno de los valores absolutos.
Ejemplo 7.8 Resuelva la inecuacin
[5 x[ < [x 2[ +[7 2x[.
Solucin
Denimos los valores absolutos:
[5 x[ =
_
5 x, x 5
5 +x, x > 5
; [x 2[ =
_
x 2, x 2
x + 2, x < 2
[7 2x[ =
_
7 2x, x
7
2
7 + 2x, x >
7
2
Tenemos ocho combinaciones posibles:
1.
_
5 x < (x 2) + (7 2x)
x 5, x 2, x
7
2

_
0 < 0
x 5, x 2, x
7
2
esta combinacin es falsa, por lo tanto no tiene solucin.
2.
_
5 x < (x 2) + (7 + 2x)
x 5, x 2, x >
7
2

_
2x 7 > 0
x 5, x 2, x >
7
2

_
x >
7
2
x 5, x 2, x >
7
2
Intersecando las soluciones, obtenemos el intervalo solucin x
_
7
2
; 5

.
3.
_
5 x < (x + 2) + (7 2x)
x 5, x < 2, x
7
2

_
x 2 < 0
x 5, x < 2, x
7
2

_
x < 2
x 5, x < 2, x
7
2
Intersecando las soluciones, obtenemos el intervalo solucin x (; 2).
4.
_
5 x < (x + 2) + (7 + 2x)
x 5, x < 2, x >
7
2

_
x 5 > 0
x 5, x < 2, x >
7
2

_
x > 5
x 5, x < 2, x >
7
2
Intersecando las soluciones, obtenemos el intervalo solucin x .
CAPTULO 7. DESIGUALDADES E INECUACIONES 366
5.
_
5 +x < (x 2) + (7 2x)
x > 5, x 2, x
7
2

_
x 5 < 0
x > 5, x 2, x
7
2

_
x < 5
x > 5, x 2, x
7
2
Intersecando las soluciones, obtenemos el intervalo solucin x .
6.
_
5 +x < (x 2) + (7 + 2x)
x > 5, x 2, x >
7
2

_
x 2 > 0
x > 5, x 2, x >
7
2

_
x > 5
x > 5, x 2, x >
7
2
Intersecando las soluciones, obtenemos el intervalo solucin x (5; +).
7.
_
5 +x < (x + 2) + (7 2x)
x > 5, x < 2, x
7
2

_
2x 7 < 0
x > 5, x < 2, x
7
2

_
x <
7
2
x > 5, x < 2, x
7
2
Intersecando las soluciones, obtenemos el intervalo solucin x .
8.
_
5 +x < (x + 2) + (7 + 2x)
x > 5, x < 2, x >
7
2

_
0 < 0
x > 5, x < 2, x >
7
2
esta combinacin es falsa, por lo tanto no tiene solucin. La solucin general de la desigualdad
(gura ) se obtiene uniendo las soluciones parciales:
x (; 2)
_
7
2
; +
_
.
7.3. Tarea
1. Resuelva las inecuaciones:
a) 3x 4 > 5x + 6;
b) 1 5x < 9x + 4;
c) 7 2x > 4x + 5;
d)
x 1
2

2x + 1
3
1;
e)
6 5x
3

x 1
2
+
x
5
;
f )
1 +x
3

x 1
2

2x 1
2
;
g)
x + 2
2
+
2x 3
3

3
2
;
CAPTULO 7. DESIGUALDADES E INECUACIONES 367
h)
5x + 1
3

x + 1
2
< 1;
i)
x 3
3
+
5
4
<
x
12
+
2x + 9
15
;
j)
4x
3
+
1
3
<
7x
6
+
x
2
+
7
18
;
k)
6x 3
2
(2x 6)
x 3
4
;
l)
9x + 1
4
2x 1;
m)
x
2
+
x
3
> x +
x
5
;
n)
3x 5
3

x 1
2
x +
1
2
;
o)
5x 2
3

x 8
4
>
x + 14
2
2;
p)
2x + 1
3
< x
x 3
2
;
q)
x + 1
3

x 2
5
1 +
2x 3
15
;
r)
x + 2
2
3(x + 1) 2
5x
2
;
s)
4 x
5

1 2x
3
x + 1;
t) 3x + 7 5(2x 3)
x 1
2
1;
u) 5(x 2)
1
3
< 3(x 1) + 2x.
Resp: a) x (; 5); b) x
_

3
14
; +
_
; c) x
_
;
1
3
_
;
d) x [1; +); e) x
_
;
75
71
_
; f ) x
_
;
8
7
_
; g) x
_
9
7
; +
_
;
h) x
_
;
5
7
_
; i) x (; 3); j) x
_

5
6
; +
_
; k) x [7; +);
l) x (; 5]; m) x (; 0).
2. Resuelva las inecuaciones:
a)
2(4x + 2)
3
(x 2)
4(4x + 5)
13
;
b)
2 +x
3

2(x 1)
7

7 5x
3

3(x + 1)
7
;
c)
2 +x
3

2(x 1)
7

7 5x
3

3(x + 1)
7
;
d)
2 6x
3
+ 4x
1
2

2 8x
4
.
Resp: a) x [2; +); b) x
_
4
9
; +
_
; c) x
_
4
9
; +
_
; d) x R.
3. Resuelva las inecuaciones:
a)
x + 3
[x 5[
5;
b)
[x + 3[
[x 3[
3;
c)
[3x 2[
x + 5
1;
d)
x 3
[x + 3[
3;
e)
2x 1
[2x + 1[
2;
f )
[5x 3[
[x + 1[ 2x
3;
g)
2x
[3x + 2[ +x
1;
h)
[3x 2[ x
x +[x 1[
2;
i)
[x 1[ + 2x
x + 5
1;
j)
2x [2x 3[
[x + 2[
1;
k)
x 1
[x + 1[ +x
3;
l)
2x + 1
[2x 1[ + 1
1;
m)
[3x 5[ + 2
3x + 1
1;
n)
[2x 4[ 3
[x 1[
2;
o)
[x + 1[
[2x + 3[ 2
3;
p)
2x [3 5x[
[2 x[ 2x
1.
Resp:
CAPTULO 7. DESIGUALDADES E INECUACIONES 368
a) x
_
11
3
; 5
_
;
b) x
_
3
2
; 3
_
(3; 6];
c) x
_
5;
3
4
_

_
7
2
; +
_
;
d) x (; 3) (3; +);
e) x
_
;
1
2
_

1
2
; +
_
;
f ) x
_
3
4
; 1
_

3
2
; 0
_
;
g) x
_
1;
1
2
_
;
h) x (; 0];
i) x (; 5) [3; +);
j) x (; 2) (2; +);
k) x (; 2]
_

4
5
;
1
2
_
;
l) x
_
;
1
4
_
;
m) x
_

1
3
; 1
_
;
n) x (; 1) (1; +);
o) x
_

1
2
;
2
5
_

14
5
;
5
2
_
;
p) x
_
1
2
;
2
3
_

_
5
2
; +
_
.
4. Resuelva las inecuaciones:
a) [2x + 5[ [x + 1[ 3x;
b) [x 5[ [x 1[ +[x 2[;
c) [3x + 2[ 3x +[x + 2[;
d) [2x 7[ [2x 1[ +[3x + 1[;
e) [3x 3[ [x + 3[ 2x + 3;
f ) [2x + 3[ [x 1[ +[x + 1[;
g)
[x + 1[
2

[x + 2[
4
+x;
h) [2x 1[ +x
[x + 3[
2
;
i) [x + 5[ [x 5[
2x + 5
3
.
Resp: a) x [1; +); b) x
_
2;
8
3
_
; c) x (; 0];
d) x
_
;
7
3
_
[1; +); e) x
_

1
2
; +
_
; f ) x
_
;
1
2
_
;
g) x (; 0]; h) x
_
;
1
3
_
[1; +); i) x
_
;
35
2
_

_
5
4
;
25
2
_
.
5. Resuelva las inecuaciones:
a) [x +[x[[ 6;
b) [[x [x + 1[[ 2[ 2;
c) 2 < [5x + 2[ 4;
d) [15 2x[ > 16 15x;
e) [2x 5[ +[x + 6[ > 15;
f ) [x 5[ < [x + 1[;
g) [[x 2[ 3[ > 2;
h) [[x 2[ 3[ < 4;
i) [x + 1[ +[2 x[ > 3;
j) 5x
4x + 2
2
< x.
Resp: a) x (; 3]; b) x
_

5
2
; +
_
; c)
_

6
5
;
4
5
_

_
0;
2
5
_
;
d) x
_
1
3
; +
_
; e) x (; 4)
_
14
3
; +
_
; f ) x (2; +);
g) x (; 3)(1; 3)(7; +); h) x [5; 9]; i) x (; 1)(2; +);
j) x (; 1).
6. Un escolar tena cierta cantidad de sellos. Le regalaron un lbum para selle. Si l pega 20
sellos en cada pgina, el lbum es insuciente, pero si pega 23 sellos en cada pgina, por lo
menos, una pgina quedara vaca. Si al nio le regalaran un lbum absolutamente igual, en
CAPTULO 7. DESIGUALDADES E INECUACIONES 369
cada pgina del cual estuvieran pegados 21 sellos, l tendra un total de 500 sellos. Cuntas
pginas tiene el lbum?
Resp: 12 pginas.
7. El recorrido de A a B lo cubre una balsa en 24 horas y una lancha gasta en el recorrido de
A a B y viceversa no menos de 10 horas. Si la velocidad propia de la lancha se aumenta el
40 %, el recorrido de A a B y viceversa ocupara no ms de 7 horas. Cunto tiempo navega
la lancha de A a B y cunto de B a A?.
Resp: El tiempo de A a B es 4 horas, el tiempo de B a A es 6 horas.
8. En dos cajones hay ms de 20 piezas iguales. El nmero de piezas en el primer cajn, dis-
minuido en 2, ms de 3 veces sobrepasa el nmero de piezas en el segundo cajn. El nmero
triplicado de piezas en el primer cajn supera el nmero doblado de piezas en el segundo
cajn, pero no ms que en 60. Cuntas piezas hay en cada cajn?
Resp: En el primer cajn, 29 piezas y en el segundo, 7 piezas.
9. En dos brigadas, conjuntamente, hay ms de 27 personas. El nmero de miembros de la
primera brigada ms de 2 veces sobrepasa el nmero de miembros de la segunda brigada,
disminuido en 12. El nmero de miembros de la segunda brigada ms de 9 veces sobrepasa
el nmero de miembros de la primera brigada, disminuido en 10. Cuntas personas hay en
cada brigada?
Resp: En la primera brigada, 41 personas y en la segunda, 17 personas.
10. Si los pioneros de un campamento se forman en una columna con 8 personas en cada la,
una de las las quedar incompleta. Si se forman con 7 personas en cada la, habr dos las
ms, pero todas sern completas. Pero, si la formacin se realiza con 5 personas en cada la,
habr 7 las ms, pero una de ellas ser incompleta. Cuntos pioneros hay en el campament?
Resp: 119 pioneros.
11. Hay cierta cantidad de alambre. Si l se enrolla en bobinas que contengan 800 metros de
alambre de cada una, 1 bobina no estar enrollada por completo. Lo mismo pasar si slo
empleamos bobinas que contengan 900 metros de alambre, con la particularidad de que har
falta 3 bobinas menos. Pero, si el alambre se enrolla slo en bobinas de una capacidad de
1100 metros, se necesitarn 6 bobinas menos, pero todas ellas estarn ocupadas por completo.
Cuntos metros de alambre haba?
Resp: 25300 metros.
12. Si un lquido se vierte en botellas de 40 litros de capacidad, con ello una botella quedar
no del todo llena. Si ese mismo lquido se vierte en botellas de 50 litros de capacidad, se
necesitarn 5 botellas menos y todas ellas estarn llenas. Si el lquido se vierte en botellas de
70 litros de capacidad, se necesitarn 4 botellas menos, pero, de nuevo, una botella no estar
llena del todo. Cuntos litros de lquido haba?
Resp: 850 litros.
13. A dos brigadas con un efectivo total de 18 personas fue encargado organizar la guardia
continua de 24 horas, cada vez con una persona, en el transcurso de 3 das. Los primeros dos
CAPTULO 7. DESIGUALDADES E INECUACIONES 370
das llevaron la guardia los miembros de la primera brigada, dividiendo entre s, por partes
iguales, todo ese tiempo. Es conocido que en la segunda brigada haba 3 muchachos y, los
dems muchachos, con la particularidad de que las primeras hicieron guardia 1 hora cada una
y los segundos, dividieron el tiempo restante entre ellos, por partes iguales. Al calcular los
resultados, result que la suma de horas de guardia de cada muchacho de la segunda brigada
y de cualquier miembro de la primera era menor que 9 horas. Cuntas personas haba en
cada brigada?
Resp: Por 9 personas.
14. Al comprar varios libros iguales y cuadernos del mismo tipo, pagaron por los primeros 10
dlares con 56 centavos y por los segundos, 56 centavos. Fueron comprados 6 libros ms que
cuadernos. Cuntos libros compraron si el precio de un libro es 1 dlar mayor que el de un
cuaderno?
Resp: 8 libros.
15. Un grupo de 30 estudiantes daba los exmenes. Con ello se ponan las notas; 2, 3, 4, 5. La
suma de las notas obtenidas era igual a 93, con la particularidad de que 13 hubo ms que 5
y menos que 4. Adems, el nmero de 4 se divida por 10, el nmero de 5 era par. Cuntas
notas de cada tipo recibi el grupo?
Resp: Doses 11, treses 7, cuatros 10, cincos 2.
16. Un grupo de estudiantes decidi comprar una cmara de un precio desde 170 hasta 195
dlares. Pero, en el ltimo momento dos estudiantes se negaron a participar en la compra y,
por ello, cada uno de los restantes tuvo que dar 1 dlar ms. Cunto cost la camara?
Resp: 180 dlares.
17. Un artculo de superior calidad es ms caro que un artculo de primera calidad, en cuanto
ste es ms caro que un asrtculo de segunda calidad, pero esta diferencia en el precio no so-
brepasa el 40 % del precio del artculo de primera calidad. La empresa pag 9600 dlares por
los artculos de superior calidad y esa misma cantidad por los artculos de segunda calidad.
La cantidad total de todos los artculos comprados constitua 1400 unidades. Cunto cuesta
un artculo de primera calidad?
Resp: 14 dlares.
7.4. Desigualdad de segundo grado
Apliquemos el mtodo de intervalos a la resolucin de las desigualdades algebraicas de segundo
grado. Analicemos la desigualdad cuadrtica
ax
2
+bx +c > 0, a ,= 0
Realizando la transformacin de formacin de cuadrado perfecto, obtenemos
ax
2
+bx +c = a
_
_
x +
b
2a
_
2

b
2
4ac
4a
2
_
CAPTULO 7. DESIGUALDADES E INECUACIONES 371
Por eso, la desigualdad ax
2
+bx +c > 0 es equivalente a la desigualdad
a
_
_
x +
b
2a
_
2

b
2
4ac
4a
2
_
> 0, a > 0
Sea a > 0. Entonces, sta desigualdad es equivalente a la desigualdad
_
x +
b
2a
_
2

b
2
4ac
4a
2
> 0, a > 0
CASO 1: Si b
2
4ac < 0, entonces, cualquiera que sea el valor numrico de la incgnita x = x
0
,
en el primer miembro de la desigualdad gura la suma del nmero no negativo
_
x
0
+
b
2a
_
2
con
el nmero positivo
b
2
4ac
4a
2
, es decir, la desigualdad se convierte en una desigualdad numrica
que es verdadera. Por consiguiente, la desigualdad es vlida para cualquier x. En otras palabras, el
conjunto de todas las soluciones de la desigualdad es en este caso el conjunto de todos los nmeros
reales.
CASO 2: Si b
2
4ac = 0, entonces, obviamente, la desigualdad se convierte en una lcita
desigualdad numrica para todo x, a excepcin del nmero x
0
=
b
2a
. Por consiguiente, el conjunto
de todas las soluciones de la desigualdad ser en este caso el conjunto
_
;
b
2a
_

b
2a
; +
_
.
CASO 3: Si b
2
4ac > 0, entonces la desigualdad es equivalente a la desigualdad (x x
1
)(x
x
2
) > 0, a > 0 donde
_

_
x
1
=
b

b
2
4ac
2a
x
2
=
b +

b
2
4ac
2a
Es evidente que x
1
< x
2
, razn por la cual, al aplicar el mtodo de intervalos, llegamos a que el
conjunto de todas las soluciones de sta desigualdad ser el conjunto (; x
1
) (x
2
; +).
Sea a < 0. Entonces, la desigualdad ax
2
+bx +c > 0 es equivalente a la desigualdad
_
x +
b
2a
_
2

b
2
4ac
4a
2
< 0, a < 0
a) Si b
2
4ac < 0, resulta evidente que para todo nmero x esta desigualdad se convierte en
una desigualdad ilcita, por lo cual sta desigualdad no tiene soluciones.
b) Si b
2
4ac = 0, resulta tambin evidente que sta desigualdad no tiene soluciones.
c) Si b
2
4ac > 0, sta desigualdad ser equivalente a la desigualdad (xx
1
)(xx
2
) < 0, a < 0
donde
_

_
x
1
=
b

b
2
4ac
2a
x
2
=
b +

b
2
4ac
2a
Es obvio que x
1
> x
2
, y por ello, al aplicar el mtodo de intervalos, llegamos a que el conjunto de
todas las soluciones de la desigualdad (x x
1
)(x x
2
) < 0 es el intervalo (x
2
; x
1
).
De modo anlogo se efecta la resolucin de la desigualdad ax
2
+bx +c < 0 (a ,= 0).
CAPTULO 7. DESIGUALDADES E INECUACIONES 372
Ejemplo 7.9 Resuelva la inecuacin
x(x 3) 2 < 3x (x
2
+ 2)
Solucin
x
2
3x 2 < 3x x
2
2 2x
2
6x < 0 x(x 3) < 0 x (0; 3).
Ejemplo 7.10 Resuelva la inecuacin
x
x 1

2
x + 1
>
8
x
2
1
Solucin
x(x + 1) 2(x 1)
x
2
1
> 0
x
2
x 6
x
2
1
> 0
(x 3)(x + 2)
(x 1)(x + 1)
> 0
x (; 2) (1; 1) (3; +)
Ejemplo 7.11 Resuelva las inecuaciones:
4
[x + 1[
[x 1[
Solucin
Haciendo la descomposicin del valor absoluto del numerador y denominador, obtenemos:
[x + 1[ =
_
x + 1, x 1 (1)
x 1, x < 1 (2)
, [x 1[ =
_
x 1, x 1 (3)
x + 1, x < 1 (4)
En este caso tenemos cuatro combinaciones posibles.
Primera: (1) y (3)
Esta combinacin nos da el siguiente intervalo de existencia x 1. Resolviendo la desigualdad,
encontramos
4
(x + 1) 2
x 1
4
x 1
x + 1 0
(x + 1)(x 3)
x 1
0
La solucin de esta desigualdad nos da x (; 1] (1 3]. La interseccin de estos dos interva-
los, nos da la solucin de la combinacin: x (1; 3].
Segunda: (1) y (4)
Esta combinacin nos da el siguiente intervalo de existencia 1 x < 1. Resolviendo la desigual-
dad, encontramos
4
(x + 1) 2
x + 1
4
x 1
+x 1 0
x
2
2x + 5
x 1
0
La solucin de esta desigualdad nos da x (1; +). La interseccin de estos dos intervalos, da la
solucin de la combinacin: x = .
Tercera: (2) y (3)
Esta combinacin no est determinada.
Cuarta: (2) y (4)
Esta combinacin nos da el siguiente intervalo de existencia x (; 1). Resolviendo la de-
sigualdad, encontramos
4
(x 1) 2
x + 1
4
x 3
+x 1 0
x
2
+ 2x 5
x + 3
0
CAPTULO 7. DESIGUALDADES E INECUACIONES 373
La solucin de esta desigualdad nos da x
_
2

2 1; 3
_

_
2

2 1; +
_
. La interseccin de
estos dos intervalos, nos da la solucin de la combinacin: x
_
2

2 1; 3
_
. La solucin de la
desigualdad esta dada por la unin de los intervalos encontrados en cada una de las combinaciones,
es decir:
x
_
2

2 1; 3
_
(1; 3).
Ejemplo 7.12 Resuelva las inecuaciones:
a)
x

4x
2
1
x + 1
1; b)

6x
2
1 x
2
x
2
1
1; c)
x

4 9x
2

1 4x
2
1.
Solucin
a) Para resolver esta inecuacin, transformamos la expresin original
x

4x
2
1
x + 1
1 0
x

4x
2
1 x 1
x + 1
0
1 +

4x
2
1
x + 1
0
De la ltima expresin, establecemos las siguientes restricciones
_
4x
2
1 0
x + 1 < 0

_
(2x 1)(2x + 1) 0
x < 1

_
x
_
;
1
2

_
1
2
; +
_
x < 1
Intersecando estas soluciones parciales, obtenemos que la solucin de la inecuacin es x < 1.
b) Resolvemos la inecuacin, transformando la expresin

6x
2
1 x
2
x
2
1
1 0

6x
2
1 x
2
x
2
+ 1
x
2
1
0

6x
2
1 2x
2
+ 1
x
2
1
0
De la ltima expresin, establecemos las siguientes restricciones
_

_
6x
2
1 0
x
2
1 > 0

6x
2
1 2x
2
+ 1 > 0

_
(

6 x 1)(

6 x + 1) 0
(x 1)(x + 1) > 0

6x
2
1 2x
2
+ 1 > 0
La primera restriccin tiene como solucin x
_
;
1

6
_

_
1

6
; +
_
.
La segunda restriccin, tiene como solucin x (; 1) (1; +).
A continuacin solucionamos la tercera restriccin
_
6x
2
1 > 2x
2
1
_
_
6x
2
1
_
2
> (2x
2
1)
2
2x
4
5x
2
+ 1 < 0
x
_

1
2
_
5 +

17;
1
2
_
5

17
_

_
1
2
_
5

17;
1
2
_
5 +

17
_
Intersecando las tres soluciones parciales, obtenemos la solucin general de la inecuacin
x
_

1
2
_
5 +

17; 1
_

_
1;
1
2
_
5 +

17
_
.
c) Para resolver esta inecuacin, transformamos la expresin original
x

4 9x
2

1 4x
2
1 0
x

4 9x
2

1 4x
2

1 4x
2
0
CAPTULO 7. DESIGUALDADES E INECUACIONES 374
De la ltima expresin, establecemos las siguientes restricciones
_

_
4 9x
2
0
1 4x
2
0
x

4 9x
2

1 4x
2
< 0

_
(3x 2)(3x + 2) 0
(2x 1)(2x + 1) 0
x

4 9x
2

1 4x
2
< 0
La primera restriccin tiene como solucin x
_

2
3
;
2
3

.
La segunda restriccin, tiene como solucin x
_

1
2
;
1
2

.
A continuacin solucionamos la tercera restriccin
x <
_
4 9x
2
+
_
1 4x
2
x
2
<
_
_
4 9x
2
+
_
1 4x
2
_
2
52x
4
40x
2
+ 9 < 0
Esta ltima inecuacin no tiene soluciones reales. Por tanto la solucin general de la inecuacin,
esta dada por la interseccin de las soluciones parciales, es decir x
_

1
2
;
1
2

.
7.5. Tarea
1. Resuelva las inecuaciones:
a) 2x
9
x 3
> x
5
x 3
;
b) 1 +
24 4x
x
2
2x 15
> 0;
c)
1
x 1
<
4
(1 x)(x 5)
;
d) 1 +
15 7x
x
2
+x 6
> 0;
e) x + 1
1
1 x
;
f ) 1 +
1 8x
x
2
+ 4x + 3
0;
h)
3x
2
3x + 8
x
2
+x + 1
2;
i)
x
2
x 4
x
2
3x + 4

1
2
;
j)
3x
2
x + 30
x
2
+ 4x + 5
2;
k)
x
x 1

2
x + 1
<
8
x
2
1
;
l)
(2x + 1)
2
(2x 1)
3
(x 1)
4
> 0;
m)
(x 1)(3x 2)
(5 2x)
> 0;
o)
x
2
5x + 6
x
2
12x + 35
> 0;
p)
x
2
4x 2
9 x
2
< 0;
q)
x
3
+x
2
+x
9x
2
25
0;
r)
x
4
+x
2
+ 1
x
2
4x 5
< 0;
s)
x
3
x
2
+x 1
x + 8
0;
t)
x
4
2x
2
8
x
2
+x 1
< 0;
u)
3x 2
2x 3
< 3;
v)
7x 4
x + 2
1;
w)
2x
2
+ 18x 4
x
2
+ 9x + 8
> 2;
x)
1
x + 1
+
2
x + 3
>
3
x + 2
;
y)
x + 1
x 2
>
3
x 2

1
2
;
z)
2
x 1

1
x + 1
> 3.
Resp: a) x (1; 3) (4; +); b) x (; 3) (5; +);
c) x (; 1) (1; 5); d) x (; 3) (2; +)3; e) x (1; +) 0;
f ) x (3; 1) 2; h) x (; 2] [3; +); i) x [4; 3];
j) x [4; 5); k) x (2; 1) (1; 3); l) x
_
1
2
; +
_
1.
2. Resuelva las inecuaciones:
a)
1
3x 2 x
2
>
3
7x 4 3x
2
;
b)
3
6x
2
x 12
<
25x 47
10x 15

3
3x + 4
;
c)
2 x
x
3
+x
2

1 2x
x
3
3x
2
;
d)
1
x + 1

2
x
2
x + 1

1 2x
x
3
+ 1
;
CAPTULO 7. DESIGUALDADES E INECUACIONES 375
e)
10(5 x)
3(x 4)

11(6 x)
3(x 4)

5(6 x)
x 2
;
f )
1
3(x 2)
+
1
(x + 1)(2 x)
> 0;
g)
(x + 1)(x + 2)(x + 3)
(2x 1)(x + 4)(3 x)
> 0.
3. Resuelva las inecuaciones:
a)

2x
x + 1

x + 1
2x

3
2
;
b) x 3 <

2x
2
3
x 1

;
c)
[x
2
4x[ + 3
x
2
+[x 5[
1;
d)

x
2
1
2x 3

x 2;
e)

x
2
x 1

x
2
16
x + 4
;
f )
1
[ x 2[
2x 3;
g) 2x

1 x
1 +x

> 3;
h)
[x
2
x + 4[
x
2
1
0;
i)

3x
2
+ 6
2x + 1

2x + 9;
j)

x + 3
x + 16

>
3
x 3
;
k)

x 2
x + 1

x + 1
x + 2

;
l)
3
[x + 3[ 1
< [x + 2[;
m)

x
2
5x 4
x
2
4

< 1;
n)

2x x
2
+ 1
x
2
+ 3x 2

> 1;
o) [2x 1[ < [14x + 1[;
p) [1 3x[ [2x + 3[ 0;
q) [1 2x[ > 3 x;
r) [x + 8[ 3x 1;
s) [4 3x[ 2 x;
t) [5x
2
2x + 1[ < 1;
u) [6x
2
2x + 1[ 1;
v) [ 2x
2
+ 3x + 5[ > 2;
w) x
2
+ 2[x[ 3 0;
x) x
2
+ 5[x[ 24 > 0;
y) [x
2
+x+10[ 3x
2
+7x+
2;
z) [2x
2
+x+11[ > x
2
5x+6.
Resp: a) x
_
2

109
21
;
8

109
9
_

_
2 +

109
21
;
8 +

109
9
_
; b) x ,= 1;
c) x
_
;
2
3
_

_
1
2
; 2
_
; d) x ,=
3
2

_
;
7 +

21
2
_
; e) x ,= 1;
f ) x ,= 2
_
;

57 1
4
_
; g) x
_

5 3
2
; +
_
;
h) x (; 1) (1; +); i) x
_
; 8

79
_

_
8

85
7
;
8 +

85
7
_
;
j) x (; 3) (9, 2; +) 16; k) x
_
;
5
2
_
[1, 82; 0, 82]1, 2;
l) x 9 ; 5) (4; 2) (

3 2; +); m) x
_
5

89
4
_

_
5 +

89
4
;
_
;
n) x ,= 1

2 x ,= 1 +

2 x ,= 2 x ,= 1.
4. Resuelva las inecuaciones:
a) [[x 2[ x + 3[ < 5;
b) [x 6[ > [x
2
5x + 9[;
c) [x[ +[x 1[ < 5;
d) [x + 1[ +[x 2[ > 5;
e) [2x + 1[ [5x 2[ 1;
f ) [2x [3 x[ 2[ 4;
g) [x 1[ +[2 x[ > 3 +x;
h) [[2x + 1[ 5[ > 2;
i) [[x 3[ + 1[ 2;
j) [[x 1[ +x[ < 3;
k) [4x
2
9x + 6[ > x
2
+x 3;
l) [3x 1[ +[2x 3[ [x + 5[ < 2.
CAPTULO 7. DESIGUALDADES E INECUACIONES 376
5. Resuelva las inecuaciones:
a) x
2
[3x + 2[ +x 0;
b) 2x
2
7[x[ + 3 0;
c) [x 3[ 2x
2
+ 1;
d) [x 2[ 2x
2
9x + 9;
e) 3x
2
[x 3[ > 9x 2;
f ) x
2
+ 4 [3x + 2[ 7x;
g) x
2
[5x 3[ x < 2;
h) [x
2
4[ 4 3x x
2
;
i) [x
2
+2x3[ 33xx
2
;
j) [4+3xx
2
[ x
2
3x4;
k) [x
2
3x + 2[ 2x x
2
.
Resp: a) x (; 2

2] [1 +

3; +);
b) x (; 3]
_

1
2
;
1
2
_
[3; +); c) x
_
;
1 +

17
4
_

17
4
; +
_
;
d) x
_
;
4

2
2
_

_
5 +

3
2
; +
_
; e) x
_
;
4

19
3
_

_
4 +

19
3
; +
_
;
f ) x (; 5

19] [

2 2; +); g) x (5; 3 + 2

2);
h) x
_
;
3 +

73
4
_
[; +); i) x
_
;
5 +

73
4
_
[0; +);
j) x (; 1] [4; +).
6. Resuelva las inecuaciones:
a)

5
x + 2

<

10
x 1

;
b)

x + 2
2x 3

< 3;
c)

2x 3
x
2
1

2;
d)

x
2
3x + 2
x
2
+ 3x + 2

> 1;
e)

x
2
3x 1
x
2
+x + 1

3;
f )

x
2
5x + 4
x
2
4

1;
g)
[x 3[
x
2
5x + 6
2;
h)
x
2
7[x[ + 10
x
2
6x + 9
< 0;
i)
x
2
[x[ 12
x 3
2x.
7. Resuelva la inecuacin:

x
2
2x + 1
x
2
4x + 4

x 1
x 2

< 12.
8. Resuelva la inecuacin:
[2x
_
x
2
2x + 1[ =

x
4
+x
2
+ 1
x
2
+x + 1

.
9. Resuelva la inecuacin:
[x
2
10x + 25[ 2[x 5[ + 35.
10. Resuelva la inecuacin:
3

x 2
x 1 [x[
0.
11. Resuelva la inecuacin:
[x 6[ [x[
[3x 2[ +[4x 3[
0.
12. Resuelva la inecuacin:

2x 5 < [[2x 2[ [3 2x[[.


CAPTULO 7. DESIGUALDADES E INECUACIONES 377
13. Resuelva la inecuacin:
[5x 5[ [5x + 15[
x
2
+x + 1
<
[2x 2[ [4x + 12[
(x + 1)
2
x
.
14. Resuelva la inecuacin:
x
[x
2
+ 4[
>
x 3
[x
2
+x + 4[
.
15. Con qu valores de k la desigualdad
x
2
+kx 1
2x
2
2x + 3
< 1 se verica con toda x?
16. Resuelva las inecuaciones:
a) 2

x + 5 > x + 2;
b) x + 4
_
6 4x x
2
;
c)
_
3x
2
6x + 3 x + 3;
d) x + 1 < 4

x + 6;
e) x 2 <
_
4 + 2x x
2
;
f )
_
2x
2
+ 4x + 2 x + 4;
g)
_
x
2
3x 10 > x + 2;
h)
_
9x
2
+ 6x + 1 < 2 x;
i)
_
5 x
2
> x 1;
j)

x 5

9 x 1;
k)
_
x
2
+ 2x 3 > x;
l)
_
x
2
x 2 > x 1;
m)

2x + 3 < 1

x + 2;
n)

x

x + 1 > 0, 25;
o)
_
2

3 +x <

4 +x;
p)
_
3x x
2
< 4 x;
q)

1 x
4

5 +x;
r)

3 x

x + 1 > 0, 5;
s)
_
4x
2
+ 16x + 16 < 2x + 10;
t)

x + 1

x 1

x 3;
u)

1 6x + 1 3x;
v)
_
24 2x x
2
< x;
w)

x + 5 +

x < 5;
x)

3x + 7

x 2 > 3;
y) x + 4 > 6

x 4;
z)
_
x
2
2x 15 > 3.
Resp: a) x [5; 4); b) x [2

10; 1]; c) x [3 2

3; 3 + 2

3];
d) x [6; 19); e) x [1

5; 3); f ) x (; 2 3

2] [2 + 3

2; +);
g) x (; 2] (14; +); h) x
_

3
2
;
1
4
_
; i) x [

5; 2);
j) x
_
5;
14

7
2
_

_
14 +

7
2
; 9
_
; k) x (; 3]
_
3
2
; +
_
;
l) x (; 1] (3; +); m) x
_

3
2
; 2 2

3
_
; n) x
_
1;
16 +

31
32
_
;
o) x
_

3 +

5
2
; 1
_
; p) x [0; 3]; q) x [1; 1];
r) x
_
1;
8

31
8
_

_
8 +

31
8
; 3
_
; s) x
_

7
2
; +
_
; t) x
_
3 + 4

3
3
; +
_
;
u) x
_

4
3
;
1
6
_
; v) x (3; 4]; w) x [0; 4); x) x [2; 3) (6; +);
y) x [4; 8) (20; +); z) x (; 3] [5; +).
17. Resuelva las inecuaciones:
CAPTULO 7. DESIGUALDADES E INECUACIONES 378
a)

2x 1 < x + 2;
b)
_
x
2
+x 2 2(x + 2);
c)

3x

2x + 1 1;
d)

2x + 5 +

x 1 > 8;
e)

2x + 1 < 5;
f )

3x 2 > 1;
g)
_
x + 3
4 x
2;
h)
_
2x 1
3x 2
3;
i)

2x + 19 < 3x 5;
j)
x 4

x + 2
< x 8;
k)
3

x + 5 + 2 >
3

x 3;
l)
_
x
2
3x 10 < 8 x;
m)
_
x
2
x 12 < x;
n)

17 15x 2x
2
x + 3
> 0;
o)

9x 20 < x;
p)
_
x
2
4x > x 3;
q)
_
3x
2
22x > 2x 7;
r)
_
x
2
5x + 6 x + 4;
s)
_
2x
2
+ 7x + 50 x 3;
t)

x + 1

x 2 1;
u)

x + 3

x 4 2;
v)

x 1 +

x + 2 1;
w) x
2
+
_
x
2
+ 11 < 31;
x)
2
x

1
2
_
4
x
2

3
4
< 0;
y)
_
x
2
2x 3 > 3(x + 1);
z)
_
2x 1
x + 2

_
x + 2
2x 1

7
12
.
18. Resuelva las inecuaciones:
a) x
2
+ 5x + 4 < 5
_
x
2
+ 5x + 28;
b) 0, 25x > (

1 +x 1)(

1 x + 1);
c)

x 2 +

3 x >

x 1

6 x;
d)

3x + 1 +

x 4

4x + 5 < 0.
e) 2

x + 1

x 1 2

x 3;
f )

x 3 +

1 x >

8x 5;
g)

17 4x +

x 5

13x + 1;
h)

x + 6 >

x 1 +

2x 5;
i)

x 2

x + 3 2

x 0;
j)
_
2

7 +x
_
2

7 x >
4

28;
k) x
2
+ 3x 10 + 3
_
x(x + 3) > 0;
l) 2x
2

_
2x
2
13x + 21 < 13x + 9;
m)
_
3x
2
+ 5x + 7
_
3x
2
+ 5x + 2 > 1;
n) (x 3)
_
x
2
4 x
2
9;
o)
6x
x 2

_
12x
x 2
2
4
_
12x
x 2
> 0;
p)
2
2 +

4 x
2
+
1
2

4 x
2
>
1
x
;
q)

x
2
16

x 3
+

x 3 >
5

x 3
;
r)
_
x
2
+ 3x + 4 +

x + 1 > 3.
s)
_
x
2
+ 3x + 2
_
x
2
x + 6 < 1;
t)
_
x
2
3x + 5 +x
2
3x + 7;
u)
_
2x
2
+ 7x 4 < 2(x + 4);
v) (1 +x
2
)
_
x
2
+ 1 > x
2
1;
w)
_
2x +
_
6x
2
+ 1 < x + 1;
x)
3
_
1 +

x < 2
3
_
1

x;
y)
4

x 2 +
4

6 x

2;
z)
_
4 4x
3
+x
6
> x
3

2;
19. Resuelva las inecuaciones:
a)
1

1 8x
2
2x
< 1;
b) 1 +

x
2
1
x
>
3
2
;
c)

24 2x x
2
x
< 1;
d)
_
x
2
+
x 1
2

3 x
3
;
e)
_
3x 9
x + 2
+
_
5 x
x + 1
> 0;
f )
_
x + 4
x 2

x;
g) 1 <
x
2
+ 3x 1
4 x
2
< 1;
h) 1 <
2x
2
7x 29
x
2
2x 15
< 2.
Resp: a) x ,= 0
_

2
4
;
1
3
_
; b) x
_
2

3
3
; +
_
; c) x [6; 0) (3; 4];
CAPTULO 7. DESIGUALDADES E INECUACIONES 379
d) x
_
;
21 + 3

241
32
_

_
3

241 21
32
; +
_
; e) x (2; 1) [3; 5];
f ) x (2; 4]; g) x
_
;
5
2
_
[1; 1); h) x
_
2;
1
3
_
(7; +).
20. Resuelva los sistemas de inecuaciones:
a)
_

_
x
2
3x 10 > 0
x
2
8x + 7 < 0
7x
2
+ 8x 1 > 0
b)
_

_
x
2
+x 0
12x
2
20x + 5 > 0
x
4
2x
2
+ 1 0
c)
_

_
x
2
x 1
x
2
4
0

x 1
3

1
d)
_

x
2
1
x + 2

< 1
4
x 2
< 0
e)
_
_
_
x
2
5x + 6 > 0
3x 21
x
2
+x + 4
< 0
f )
_
x
2
4x + 3 < 0
2x 4 < 0
g)
_
2x
2
+ 2 < 5x
x
2
x
;
h)
_

_
x + 3
x 2
< 1
2x + 3
3x 2
< 2
i)
_
_
_
x
2
+ 9x 20
11x x
2
30
1
x
2
+ 18 > 5x
j) 1 <
3x
2
7x + 8
x
2
+ 1
< 2
k)
_
_
_
2x + 3 > 1
1
x
+
1
3
> 0
l)
_

_
2x 11
4
+
19 2x
2
< 2x
2x + 15
9
>
x 1
5

x
3
m)
_

_
(x + 2)(x
2
3x + 8)
x
2
9
0
1 x
2
x
2
+ 2x 8
0
Resp: a) ; b) x [1; 0]; c) x [4; +); d) x (0, 3; 2).
21. Resuelva los sistemas de inecuaciones:
a)
_

_
3x + 5
7
+
10 3x
5
>
2x + 7
3

148
21
7x
3

11(x + 1)
6
>
3x 1
3

13 x
2
b)
_
_
_
3
3 7x
10
+
x + 1
2
> 4
7 3x
2
7(3x 6) + 4(17 x) > 11 5(x 3)
c)
_
(2x + 3)(2x + 1)(x 1) < 0
(x + 5)(x + 1)(1 2x)(x 3) > 0
d)
_
(x
2
+ 12x + 35)(2x + 1)(3 2x) 0
(x
2
2x 8)(2x 1) 0
e)
5x 7
x 5
< 4
x
5 x
+
3x
x
2
25
< 4
f )
_

_
(x 1)
3
(x
2
4)
2
(x
2
9)
3
(x
2
+ 1)
(1 3x)(x
2
x 6)(x
2
3x + 16)
< 0
2x
2
+x 16
x
2
+x
< 1
22. Resuelva el sistema de inecuaciones:
_
4|4x|
|x|+4
< 4
[[x 1[ +x[ >

x
23. Con qu valores de k la desigualdad
x 2k 1
x k
< 0 se verica con toda x perteneciente
a [1; 2]?
24. Con qu valores de k la desigualdad (x 3k)(x k 3) < 0 se verica con toda x
perteneciente a [1; 3]?
CAPTULO 7. DESIGUALDADES E INECUACIONES 380
25. Con qu valores de k el sistema
_
x
2
+ (5k + 2)x + 4k
2
+ 2k < 0
x
2
+k
2
= 4
tiene, por lo menos, una solucin?
26. Con qu valores de k el sistema
_
x
2
+ (2 3k)x + 2k
2
2k < 0
kx = 1
tiene, por lo menos, una solucin?
27. Con qu valores de k el sistema
_
x
2
(3k + 1)x + 2k
2
+ 2k < 0
x +k
2
= 0
no tiene soluciones?
28. Con qu valores de k el sistema
_
x
2
+
_
1
3
2
k
_
x +
k
2
2

k
2
< 0
x = k
2

1
2
no tiene soluciones?
29. Con qu valores de k el sistema de desigualdades 6 <
2x
2
+kx 4
x
2
x + 1
< 4 se verica con
toda x?
30. Si la temperatura en la escala Fahrenheit es F grados y utilizando la escala Celsius es C,
entonces C =
5
9
(F 32). Cul es el conjunto de valores de F si C est entre 10 y 20?
Resp: F/50 < F < 68
31. Cuando la temperatura del agua es mayor o igual a 100 Celsius, el agua hierve. Utilice la
frmula del problema anterior para determinar la temperatura Fahrenheit a la cual hierve el
agua.
32. Un inversionista tiene invertidos $ 8000 al 9 % y piensa invertir dinero adicional al 16 %
con objeto de lograr un rendimiento de al menos 12 % de la inversin total. Qu cantidad
de dinero deber ser invertida?
Resp: por lo menos $ 6000.
CAPTULO 7. DESIGUALDADES E INECUACIONES 381
33. Parte de $ 20000 son invertidos al 9 % y el resto se invierten al 12 %. Cul es la menor
cantidad de dinero que puede ser invertida al 12 % para tener un rdito anual de al menos $
2250 de las dos inversiones?
34. Un fabricante de lmparas vende nicamente a mayoristas en su sala de exposicin. El
gasto semanal total, incluyendo salarios, costos de planta y renta de la sala de exhibicin, es
de $ 6000. Si cada lmpara se vende por $ 168 y el material usado en su construccin cuesta
$ 44, cuntas lmparas deber hacer y vender cada semana para que el fabricante logre una
ganancia?
Resp: por lo menos 49.
35. Si en un curso particular, un estudiante tiene un promedio de calicaciones, en cuatro
exmenes, de menos de 90 pero no debajo de 80, el estudiante recibir una calicacin de B
en el curso. Si las calicaciones del estudiante en los tres primeros exmenes son 87, 94 y 73,
qu calicacin en el cuarto examen dar como resultado la calicacin B?
36. Un platero piensa obtener una aleacin que contenga al menos 72 % y cuando ms 75 %
de plata. Determine las cantidades mxima y mnima de una aleacin a 80 % que debe ser
combinada con una aleacin de plata de 65 % para obtener 30 gr de la aleacin requerida.
Resp: a lo ms 20 gr y a lo menos 14 gr.
37. Qu cantidad de alcohol puro debe ser agregado a 24 litros de una solucin de alcohol al
20 % para obtener una mezcla que al menos tenga 30 % de alcohol?
38. Una empresa puede vender a $ 100 por unidad todos los artculos de primera necesidad
que produce. Si se fabrican x unidades por da, y el nmero de dlares en el costo total diario
de produccin es x
2
+ 20x + 700. Cuntas unidades debern producirse diariamente de tal
manera que la compaa garantice una ganancia?
Resp: ms que 10 y menos que 70.
39. Una compaa que fabrica escritorios puede vender todos los que produce a $ 400 cada
uno. Si x escritorios se venden cada semana, entonces el nmero de dlares en el costo total
de produccin semanal es 2x
2
+ 80x + 3000. Cuntos escritorios debern construirse sem-
analmente para que el fabricante garantice una ganancia?
40. Un campo rectangular cercado est ubicado en la orilla de un ro; el lado largo del ro no
requiere de cerca. El costo del material para la cerca es de $ 8 por pie lineal para los dos lados
opuestos con cerca y $ 16 por pie lineal para el lado paralelo al ro. Si el rea del campo es
de 12000 pie
2
y el costo de la cerca no debe exceder de $ 3520, cules son las restricciones
en las dimensiones del campo?
Resp: Si x pies es la longitud de cualquier lado, 100 x 120.
41. Una parcela rectangular de terreno ser encerrada por una cerca, luego, dividida a la mitad
por otro tipo de cerca. La cerca que divide a la mitad la parcela cuesta $ 3 por pie lineal
y la otra cerca tiene un costo de $ 6 por pie lineal. Si el rea del terreno es 1800 pie
2
y el
CAPTULO 7. DESIGUALDADES E INECUACIONES 382
costo total de la cerca no debe ser mayor que $ 2310, cules son las restricciones en las
dimensiones del terreno?
7.6. Desigualdades de orden superior
Supongamos que se pide resolver la desigualdad
(x k
1
)(x k
2
)...(x k
n1
)(x k
n
) > 0
donde k
1
, k
2
, ..., k
n1
, k
n
son ciertos nmeros jos, entre los cuales no hay iguales, y, adems,
tales que k
1
< k
2
< ... < k
n1
< k
n
.
Examinemos el polinomio
p(x) = (x k
1
)(x k
2
)...(x k
n1
)(x k
n
)
En virtud de la observacin hecha ms arriba resulta obvio que para cualquier nmero x
0
tal que
x
0
> k
n
, el valor numrico correspondiente de todo factor en el producto (x k
1
)(x k
2
)...(x
k
n1
)(x k
n
) es positivo y, por esta razn, el correspondiente valor numrico p(x
0
) del polinomio
p(x) es tambin positivo. Para cualquier nmero x
1
, elegido del intervalo (k
n1
; k
n
), el valor numri-
co correspondiente del ltimo factor es negativo, y el valor numrico correspondiente de cualquiera
de los factores restantes es positivo, por lo cual el nmero p(x
1
) es negativo; anlogamente, para
todo nmero x
2
, perteneciente al intervalo (k
n2
; k
n1
), el nmero p(x
2
) es positivo, etc.
Precisamente en este razonamiento se basa el mtodo de intervalos que consiste en lo siguiente:
en la recta numrica se marcan los nmeros k
1
, k
2
, ..., k
n1
, k
n
; en el intervalo, que se encuentra
a la derecha del nmero mayor, se pone el signo ms, en el intervalo siguiente, que va de derecha
a izquierda, se pone el signo menos, luego, el signo ms, luego, el signo menos, etc. Entonces el
conjunto de todas las soluciones de la desigualdad dada ser la unin de todos los intervalos que
llevan el signo ms.
El mtodo de intervalos permite resolver aquellas desigualdades algebraicas que pueden re-
ducirse, mediante una sucesin de pasos equivalentes, a las desigualdades del tipo
(x k
1
)(x k
2
)...(x k
n1
)(x k
n
) > 0
Algunas desigualdades algebraicas de grados superiores a dos se reducen, mediante una sucesin
de pasos equivalentes, a la forma
(x a
1
)
k
1
(x a
2
)
k
2
...(x a
n1
)
k
n1
(x a
n
)
k
n
> 0
donde k
1
, k
2
, ..., k
n1
, k
n
son nmeros naturales jos, y a
1
, a
2
, ..., a
n1
, a
n
, nmeros reales jos,
entre los cuales no hay iguales, y tales que a
1
< a
2
< ... < a
n1
< a
n
, indiquemos que si al
CAPTULO 7. DESIGUALDADES E INECUACIONES 383
menos uno de los nmeros k
i
2, entonces el mtodo de intervalos aducido anteriormente no
puede ser aplicado para la resolucin de esta desigualdad. Entonces, las desigualdades de este tipo
se resuelven por el as llamado mtodo de intervalos generalizado. Examinemos el polinomio
p(x) = (x a
1
)
k
1
(x a
2
)
k
2
...(x a
n1
)
k
n1
(x a
n
)
k
n
> 0
Es evidente que para cualquier nmero x
0
tal que x
0
> a
n
, el valor correspondiente de todo
factor en el producto, es positivo, debido a lo cual el valor numrico p(x
0
) del polinomio p(x)
es tambin positivo. Para cualquier nmero x
1
, elegido dentro del intervalo (a
n1
; a
n
), el valor
numrico correspondiente de todo factor, a excepcin del ltimo, es positivo; el valor numrico cor-
respondiente del ltimo factor es positivo, si k
n
es un nmero par, y negativo, si k
n
es un nmero
impar. Por eso, el nmero p(x
1
) es positivo, si k
n
es un nmero par, y el nmero p(x
1
) es negativo,
si k
n
es impar.
En estos casos suele decirse, habitualmente, que el polinomio p(x) cambia de signo, al pasar por
el punto a
n
, si k
n
es un nmero impar, y no cambia de signo, si k
n
es un nmero par. Anlogamente
se muestra que si se conoce el signo del polinomio p(x) en el intervalo (a
i
; a
i+1
), entonces en el
intervalo (a
i1
; a
i
) el signo se determina segn la siguiente regla: el polinomio p(x) cambia de sig-
no, al pasar por el punto a
i
, si k
i
es un nmero impar, y no cambia de signo, si k
i
es un nmero par.
Precisamente en estos razonamientos est basado el mtodo de intervalos generalizado: en el
eje numrico se marcan los nmeros a
1
, a
2
, ..., a
n1
, a
n
; en el intervalo dispuesto a la derecha del
nmero mayor, es decir, a la derecha de a
n
, se pone el signo ms en el intervalo que sigue tras el
primero de derecha a izquierda se pone el signo ms, si k
n
es un nmero par, y el signo menos, si
k
n
es un nmero impar; en el siguiente intervalo de derecha a izquierda se pone el signo, rigindose
por la siguiente regla: el polinomio p(x) cambia de signo, al pasar por el punto a
n1
, si k
n1
es un
nmero impar, y conserva el signo invariable, si k
n1
es un nmero par; a continuacin se examina
el intervalo siguiente que va de derecha a izquierda y se pone en l el signo, rigindose por la misma
regla; de esta manera se analizan todos los intervalos. La solucin de la desigualdad ser la unin
de todos los intervalos en los cuales se ha puesto el signo ms.
Pasemos ahora a la resolucin de las desigualdades no estrictas p(x) 0 o p(x) 0. Si cier-
to nmero x
0
es la solucin de la desigualdad p(x) 0, se vericar la desigualdad numrica
p(x
0
) 0. Entonces, debido a la denicin del signo no estricto de una desigualdad, se verica
o bien la igualdad numrica p(x
0
) = 0, o bien la desigualdad p(x
0
) > 0. En otras palabras, si el
nmero x
0
es la solucin de la desigualdad p(x) 0, entonces dicho nmero es o bien la solucin de
la ecuacin p(x) = 0, o bien, de la desigualdad p(x) > 0. Esto puede decirse sobre cualquier solu-
cin de la desigualdad p(x) 0. Del modo anlogo se muestra que toda solucin de la desigualdad
p(x) > 0 y toda solucin de la ecuacin p(x) = 0 es tambin la solucin de la desigualdad p(x) 0.
De este modo, el conjunto de soluciones de la desigualdad no estricta p(x) 0 representa la
unin de dos conjuntos: el de todas las soluciones de la desigualdad estricta p(x) > 0 y el de todas
las soluciones de la ecuacin p(x) = 0. Anlogamente, el conjunto de todas las soluciones de la
desigualdad no estricta p(x) 0 es la unin de dos conjuntos: el de todas las soluciones de la
desigualdad estricta p(x) < 0 y el de todas las soluciones de la ecuacin p(x) = 0.
En esto precisamente est basado el principio de resolucin de las desigualdades no estrictas.
Se resuelven primeramente la desigualdad estricta y la ecuacin correspondiente despus de lo cual
se renen los conjuntos de soluciones de la desigualdad estricta y de la ecuacin; la unin de dichos
conjuntos es precisamente el conjunto de todas las soluciones de la desigualdad no estricta.
CAPTULO 7. DESIGUALDADES E INECUACIONES 384
Ejemplo 7.13 Resuelva la inecuacin
x
x 1

2
x + 1
>
8
x
2
1
Solucin
x(x + 1) 2(x 1)
x
2
1
>
8
x
2
1

x(x + 1) 2(x 1)
x
2
1

8
x
2
1
> 0
x
2
x 6 > 0 (x + 2)(x 3) > 0
Por tanto la solucin de la inecuacin es: x (; 2) (3; +).
Ejemplo 7.14 Resuelva la inecuacin
(x + 5)(x
2
1)
x(x + 3)
2

49 x
2
> 0
Solucin
Estableciendo las condiciones del problema, tenemos:
_
_
_
(x + 5)(x
2
1)
x(x + 3)
2
> 0
49 x
2
> 0

_
_
_
(x + 5)(x
2
1)
x(x + 3)
2
> 0
x
2
49 < 0
_
_
_
(x + 5)(x 1)(x + 1)
x(x + 3)
2
> 0
(x + 7)(x 7) < 0

_
x (; 5) (1; 0) (1; +)
x (7; 7)
Hacemos la interseccin entre estas dos soluciones y obtenemos la solucin general de la inecuacin:
x (7; 5) (1; 0) (1; 7)
7.7. Tarea
1. Resuelva las inecuaciones:
a) x
4
7x
3
+ 8x
2
+ 28x 48 x
2
5x + 6;
b) x
4
+ 7x
3
+x
2
63x 90 x
2
+ 2x 15;
c) (x
2
16x)
2
63 2(x
2
16x);
d) (2x
2
+ 2x)(x
2
2x + 1)(3x
3
+ 7x 10) > 0;
e) (x
2
4)(x
2
4x + 4)(x
2
x 2) 0;
f ) x
4
x
3
+x 1 x
2
1;
g) x
5
x
3
x
2
+ 1 x
3
+ 1;
h) x
2
+ 2x + 7 (4 + 2x +x
2
)(3 + 2x +x
2
);
i) x(x
2
+ 3x 4) > 7x
3
18x
2
+ 6x + 5;
j) (3x
2
4x + 1)(4x
4
5x
3
+x
2
) 0;
k) (2x
2
3x 14)(2x
2
+ 11x + 14) < 0;
l) (x
2
+ 4x 45)(3x
2
14x 5)(x + 1) 0;
m) (3x x
2
)(x + 4x
2
+ 5) 2(x + 4x
2
+ 5);
n) x
4
+ 5x
3
+ 5x
2
5x 6 x
2
+x 2;
o) (x
2
+ 10x + 25)(25 x
2
) > 0;
p) 2x
4
5x
3
+ 5x 2 2x
2
5x + 2;
CAPTULO 7. DESIGUALDADES E INECUACIONES 385
q) (3x
3
24)(2x
2
+ 6x 20) 0;
r) (4 2x
2
x
4
)(x
2
2x + 1) < 0;
s) (x
2
4x 12)(x
3
7x 6) 0;
t) (x + 4)(x + 2)
3
(x 1)(2 x)
2
(x
2
3x + 5) > 0;
u) (9 x
2
)(x
2
2x 3)(x + 8) 0;
v) (3x 2)(x 1) < 27 + 3(x 3)
2
6(3x + 1);
w) (x 1)(x 3)(x 4)(x 6) + 8 0;
x) (6x
3
x
2
+ 1)(x
3
+x
2
+ 10x + 10) < 0;
y) (x
2
3x + 2)(x
2
5x + 6)(1 x
2
) 0;
z) (5x
2
x 4)(x
3
1)(x 10) > 0.
Resp: a) x [3; 1 +

10] [1

10; 2];
b) x
_
5;

5 + 5
2
_

5
2
; 3
_
;
c) x (; 8

73] [8

57; 8 +

57] [8 +

73; +);
d) x (1; 0) (1; +);
e) x [2; 1] 2;
f ) x (; 1] [0; +);
g) x
_
1

5
2
;
1 +

5
2
_
(; 1];
h) x = 1;
i) x
_
;
15

345
12
_

_
1;
15 +

345
12
_
;
j) x
_
1
4
;
1
3
_
0, 1;
k) x
_

7
2
;
7
2
_
2;
l) x (; 9]
_
1;
1
3
_
5;
m) x (; 1] [2; +);
n) x (; 2

2] [2; 2 +

2] [1; +);
o) x (5; 5);
p) x (;
_
]
_
1
2
;

2
_
[2; +);
q) x [5; +);
r) x (;
_

5 1) (
_

5 1; +);
s) x [1; 3] [6; +) 2;
t) x (4; 2) (1; +);
u) x (; 8] [3; 1] 3;
v) x
_
;
46
31
_
;
w) x
_
;
7

17
2
_
[2; 5]
_
7 +

17
2
; +
_
;
x) x
_
1;
1
2
_
;
y) x (; 1] [3; +) 1, 2;
CAPTULO 7. DESIGUALDADES E INECUACIONES 386
z) x
_
;
4
5
_
(10; +).
2. Resuelva las inecuaciones:
a)
x
2
(x 2)
3
(x + 3)
(x 4)
7
> 0;
b)
(x + 5)(x

3)(x +

2)
(2x 3)(4x + 5)
< 0;
c) 2x
3
5x
2
+ 2x 0;
d)
x
2
3x 18
13x x
2
42
0;
e)
(x 3)(x + 2)
x
2
1
< 1;
f )
(x 1)
3
(x + 2)
4
(x 3)
5
(x + 6)
x
2
(x 7)
3
0;
g)
3x + 4
x
2
3x + 5
< 0;
3. Resuelva el sistema de inecuaciones:
a)
_
5x
2
+ 6x 8 < 0
5x
2
+ 6x + 8 > 0
;
b)
_
3 x 0, 5 + 2x
2 +x > 7x + 1, 5
;
c)
_

_
x
2
+y
2
4
x 1 > 0
y 1 0
;
d)
_
x 1 1 3x
3x + 2 7
;
e)
_
4(1 2x) < 3(1 + 3x)
1 7x 6x
;
f )
_
x
2
4 < 0
3x
2
+ 2x 5 0
;
g)
_
4x
2
9 < 0
x 15 0
;
h)
_
x
2
+y
2
= 0
[x[ +[y[ < 1
;
i)
_

_
9x
2
+ 25y
2
255 < 0
3x + 5y 15 < 0
y + 2 > 0
;
j)
_

_
x
2
+ 4y
2
16 > 0
y + 3 > 0
x +y 2 < 0
;
k)
_
x
2
4y
2
4 > 0
4x + 3y 12 < 0
;
l)
_

_
9x
2
16y
2
+ 144 > 0
2x y 6 < 0
3x +y + 12 > 0
;
m)
_

_
y
2
10x < 0
5x 3y 15 < 0
y 2 < 0
;
n)
_

_
x
2
+ 8y < 0
2x + 3y + 6 < 0
16x
2
9y
2
144
;
o)
_
x
2
+x4
x
< 1
x
2
< 64
;
p)
_
3x6
x+2
0
(x
4
5x
3
+ 6x
2
)(1 x
2
) 0
;
q)
_
x
5
100x
3
(x+9)(5xx
2
18)
x
2
18x+48
0
.
Resp: a) 2 < x <
4
5
; d)
1
2
x
5
3
; e)
1
17
< x 1;
f ) 2 < x
5
2
, 1 x < 2.
4. Resuelva el sistema de inecuaciones:
CAPTULO 7. DESIGUALDADES E INECUACIONES 387
a)
_

_
x +y < 6
y < 5
x > 1
;
b)
_

_
y x 2
x + 5y 10
x + 2y 16
2x +y 20
;
c)
_

_
x +y 9
x y 0
x + 2y 16
x 0
;
d)
_

_
5x + 2y 10 0
x y 2 0
3x + 4y 0
x 0
y 0
;
e)
_

_
x +y 120
3y x 0
x 100
y 100
;
f )
_

_
x + 2y 80
3x + 2y 160
x +y 70
;
g)
_

_
2x +y 18
2x + 3y 26
x +y 16
x 0
y 0
.
5. Resuelva las inecuaciones:
a)
(x + 3)(3x 4)
(x 1)
2
(x + 5)(x + 3)
2
0;
b)

100 x
2
(x + 4)(x + 3)
2
x
2
(x 2)
< 0;
c)
x
3
+ 6x
2
+ 5x 12
(5x
2
8x 13)
3
(x 2)
2
(x 1)
< 0;
d)

x + 7(x + 5)(x 1)
2
(3 x
2
)(x 6)(x 5)
0;
e)
x 2
x + 2

x + 2
x 2
+
x 1
x + 1

x + 1
x 1
0;
f )
(x
2
5x + 2)(x + 3)
2
(x 2)
2
x
4
+ 3x
3
+ 3x
2
+ 3x + 2
> 0;
g)
(x + 2)(x
2
x + 3)
x(2x + 1)(x 5)
0;
h)
3
x + 1
+
7
x + 2

6
x 1
.
Resp: a)
_
x ,= 3 < x
4
3
_
x < 5;
b) x ,= 3 x ,= 0 4 < x < 2;
c)
_
x ,= 2 1 < x <
13
5
_
4 < x < 3;
d)

3 < x <

3 5 < x < 6 7 x 5;
e) x < 2

2 x < 2

2 x < 1 0 x < 1;
f ) (x ,= 3 x < 2) 1 < x <
5

17
2
x >
5 +

17
2
;
g)
1
2
< x < 0 x 2 x > 5;
h) x < 2
5
4
x < 1 1 < x 5.
Captulo 8
Funciones algebraicas
8.1. Funciones
Las funciones juegan un papel muy importante en matemtica. Una precisa denicin es la
siguiente.
Denicin 8.1 Funcin
Sea f una relacin de A en B. Entonces f es una funcin de A en B, denotado f : A B y se
lee (f es una funcin de A en B) si y slo si
a) Dom(f) = A.
b) x A, y, z B, [(x, y) f (x, z) f] y = z.
En palabras, lo anterior dice que si f es una relacin de A en B tal que para cada x A existe
exactamente un y B tal que (x, y) f, entonces f es una funcin. Es decir, una funcin es una
relacin en la cual no hay dos parejas ordenadas que tengan el mismo primer miembro y diferentes
segundos miembros. La condicin a) garantiza que para cada x A existe al menos un tal y y la
condicin b) garantiza que hay a lo ms uno. As, tomados juntos, hay exactamente uno.
Esta denicin requiere que para cualquier valor de x del dominio exista uno y slo un corre-
spondiente valor de y. En contraste, una relacin no tiene esta estipulacin y puede tener ms de
un valor en la imagen correspondiente a un valor del dominio. As vemos que una funcin es una
relacin, aunque una relacin no necesariamente es una funcin.
Si f es una funcin de A en B entonces la propiedad funcional de cada x A relacionado a
exactamente un y B permite el uso de la notacin funcional y = f(x).
Como ejemplos de relaciones que son funciones y algunas que no lo son, consideremos los
siguientes:
A = 1, 2, 3, 4, B = 1, 2, 3, 4, 5, f = (1; 2); (2; 3); (3; 4); (4; 5)
g = (1; 2); (1; 3); (2; 4); (3; 5); (4; 5), h = (1; 1); (2; 2); (3; 3).
Entonces f, g y h son relaciones de A en B, pero slo f es una funcin; g no es funcin ya que
(1, 2) y (1, 3) son elementos de g. Tampoco h es una funcin ya que Dom(h) = 1, 2, 3 , = A. Ob-
servemos que f tiene una simple forma y puede ser descrita por una frmula: x A, f(x) = x+1.
388
CAPTULO 8. FUNCIONES ALGEBRAICAS 389
La mayora de las funciones conocidas en clculo son dadas por una frmula. Sin embargo esto
no es necesario y, en general en matemtica, las funciones no estn dadas por frmulas.
Usaremos las siguientes notaciones y nombres cuando trabajemos con funciones.
Sea f : A B y (x, y) f entonces escribimos y = f(x).
Observe que el nombre de la funcin es f y que f(x) no es el nombre de la funcin sino un
elemento de B.
Si y = f(x) entonces decimos que y es la imagen de x y que x es una preimagen de y.
Observe que se usa la cuando se habla de imagen y se usa una cuando se habla de preimagenes
ya que un elemento de B puede tener varios elementos de A relacionados. Ya que f es una relacin
se puede hablar de su dominio e imagen, componer f con otras relaciones y analizar su inversa.
Note que aunque Dom(f) = A, no necesariamente es Im(f) = B. De esta manera es conveniente
tener tambin un nombre para B. Usualmente se le denomina codominio de f.
El siguiente resultado es til para determinar cuando dos funciones son iguales.
Teorema 8.1 Sean f : A B y g : A B. Entonces f = g si y solo si x A, f(x) = g(x).
Existen ciertas propiedades que las funciones pueden o no tener. Si estas propiedades son usadas
frecuentemente entonces requieren nombres. Algunos de estos son dados en la siguiente denicin.
Denicin 8.2 Inyectividad
Sea f : A B. Entonces:
a) Se dice que f es uno a uno (o f es inyectiva) si y slo si w, z A, f(w) = f(z) implica
w = z.
b) Se dice que f es sobre (o f es sobreyectiva) si y slo si Im(f) = B.
c) Se dice que f es biyectiva (o biunvoca) si y slo si f es a la vez uno a uno y sobre.
Recordemos que ya que funciones son relaciones, ellas tienen inversas que son relaciones. As,
podemos hablar de la inversa de cualquier funcin, pero no hay razn para esperar que esta inversa
sea tambin una funcin. En este sentido las funciones biyectivas son importantes, ya que ellas son
exactamente aquellas funciones cuyas inversas son tambin funciones.
Teorema 8.2 Sea f : A B una funcin. Entonces f
1
: B A es una funcin si y slo si
f es biyectiva.
Se observa que el hecho que f es 1 - 1 implica que f
1
tiene la propiedad de funcin y que
f
1
sobre implica que Dom(f
1
) = B. As, si f : A B es tal que f es 1 - 1 pero no sobre B,
entonces f
1
es una funcin de Im(f) en A pero no es una funcin de B en A.
Si f : A B y g : B C entonces (g f) : A C denota la composicin de f y g.
Si (g f)(x) = z, entonces (x, z) (g f), lo cual signica que existe y B tal que (x, y) f y
(y, z) g. Luego, f(x) = y y g(y) = z. Por lo tanto, z = g(y) = g(f(x)) o (g f)(x) = g(f(x)), que
es la notacin usual. Ya que las funciones son relaciones se pueden componer y, en consecuencia, los
resultados para relaciones valen para funciones. As, si f, g son funciones con dominios e imgenes
apropiadas, entonces (g f)
1
= f
1
g
1
aunque (g f)
1
, f
1
y g
1
pueden no ser funciones.
CAPTULO 8. FUNCIONES ALGEBRAICAS 390
Teorema 8.3 Sean f : A B y g : B C biyectivas. Entonces (g f) : A C es biyectiva.
Para mostrar que f es 1 - 1 se debe probar que distintos elementos en el dominio tienen
distintas imgenes y para mostrar que f es sobre se debe probar que cada elemento de B tiene una
preimagen.
Ejemplo 8.1 Demuestre que f : R R dada por f(x) = ax +b, a ,= 0, es biyectiva.
Solucin
Primero una prueba directa que f es 1-1. Sean x, y R con f(x) = f(y). Entonces ax+b = ay +b,
lo cual implica que ax = ay. Ya que a ,= 0, se tiene x = y y por lo tanto f es 1-1.
Una prueba contrapositiva podra ser: Sean x, y R con x ,= y. Entonces ya que a ,= 0, ax ,= ay.
Luego se tiene ax +b ,= ay +b y as f(x) ,= f(y).
Para mostrar que f es sobre, sea z R. Entonces
zb
a
es tambin un elemento de R, ya que a ,= 0
y
f
_
z b
a
_
= a
_
z b
a
_
+b = z b +b = z
luego f es sobreyectiva. Observe que la eleccin de
zb
a
fue el resultado de resolver la ecuacin
f(x) = ax +b = z para x.
Teorema 8.4 Sean f : A B y g : B C biyectivas. Entonces (g f)
1
: C A y x C
(g f)
1
(x) = (f
1
g
1
)(x) = f
1
(g
1
(x)).
Observemos que la relacin identidad en A, I
A
, es una funcin de A en A que llamaremos
funcin identidad. Usando una notacin funcional, I
A
(x) = x, x A.
Teorema 8.5 Sea f : A B. Entonces
a) f I
A
= f.
b) I
B
f = f.
c) Si f es biyectiva entonces f
1
f = I
A
y f f
1
= I
B
(o x A, f
1
(f(x)) = x y
x B, f(f
1
(x)) = x).
Denicin 8.3 Imagen de una funcin
Sea f : A B. Si C A entonces se dene f(C) = f(x)/x C. Si D B entonces
f
1
(D) = x/f(x) D. f(C) se llama la imagen de C y f
1
(D) la preimagen de D.
Ejemplo 8.2 Sea f : A B donde A = 1, 2, 3, 4, B = 1, 3, 5 y f es dada por f(1) = 1,
f(2) = 1, f(3) = 5, f(4) = 5. Entonces f(1, 3) = 1, 5, f(1, 2) = 1, f
1
(1) = 1, 2,
f
1
(4) = .
Teorema 8.6 Sea f : A B y sean C D B. Entonces f
1
(C) f
1
(D).
Un elemento de una relacin es una pareja ordenada de nmeros reales y las coordenadas de
un punto en un plano coordenado tambin es una pareja ordenada de nmeros reales. Esto nos
conduce a asociar un punto P(x, y) de un plano coordenado con el correspondiente elemento (x, y)
de la relacin.
Denicin 8.4 Grca de una relacin
La grca de una relacin es el conjunto de puntos de un plano coordenado cuyas coordenadas son
parejas ordenadas de una relacin.
CAPTULO 8. FUNCIONES ALGEBRAICAS 391
Ejemplo 8.3 Dibuje la grca de la relacin
f = (2, 1), (3, 3), (1, 2), (2, 1), (1, 2)
Solucin
La grca de la relacin consiste de los puntos marcados en la gura. Esta relacin no es una
funcin, puesto que (-2, -1) y (-2, 1) son dos parejas ordenadas de f que tienen el mismo primer
elemento aunque los segundos elementos son diferentes.
Denicin 8.5 Grca de una ecuacin
La grca de una ecuacin en x y y es el conjunto de todos los puntos del plano coordenado cuyas
coordenadas satisfacen la ecuacin.
Ejemplo 8.4 Graque la relacin
f = (x, y)/y = 2 + 3x x
2
.
Solucin
Para encontrar las coordenadas de los puntos de la graca, usamos la ecuacin dada. Si se asigna
un valor a x, entonces se obtiene el nico valor de y. As si x = 0, entonces y = 2. En consecuencia
(0, 2) son las coordenadas de un punto de la grca deseada.
El trmino funcin suele utilizarse en la vida diaria, como cuando se dice que la conducta de
los precios recientemente ha sido una funcin de los ltimos escrutinios. El concepto de funcin
aparece tambin en las matemticas y es, de hecho, uno de los conceptos ms importantes de esta
ciencia.
Denicin 8.6 Funcin
Una funcin es una regla que asocia con cada elemento de algn conjunto D no vaco uno y slo
un elemento de otro conjunto C.
El conjunto D que aparece en la denicin de funcin recibe el nombre de dominio, y el conjunto
C recibe el nombre de codominio o recorrido. Supngase que x D es un elemento del dominio. El
elemento y C asociado con x por la funcin reciben el nombre de imagen de x. La caracterstica
esencial de una funcin es que cada elemento x D tiene una y slo una imagen en C. De este
modo, no puede haber dos elementos diferentes de C asociados con un solo elemento x del dominio.
No tiene que ser cierto que todo elemento de C es la imagen de algn elemento de D. Puede haber
elementos en C que no sean imagen de ningn elemento de D. Cabe hacer mencin de que no todos
CAPTULO 8. FUNCIONES ALGEBRAICAS 392
los matemticos denen el codominio como se ha hecho aqu. Algunos denen el recorrido como el
subconjunto de C que contiene todas las imgenes de los elementos de D. En esta terminologa, C
no tiene nombre especial. La terminologa anterior y la utilizada en este texto dieren solamente si
C tiene uno o ms elementos que no son imagen de algn elemento de D. En este texto trabajaremos
con un tipo de funciones llamadas funciones reales de una variable real.
Denicin 8.7 Dominio de una funcin
Se llama dominio de una funcin, el conjunto de los valores reales del argumento en los cuales la
funcin toma valores reales.
Denicin 8.8 Funcin real de variable real
Una funcin real de una variable real es una regla que asocia a cada nmero real x de un conjunto
D R, un nico nmero real f(x), llamado imagen de x bajo f. Una funcin tal se denota con
f : D R R.
Si x es cualquier elemento del dominio D, existe entonces un elemento y del codominio C,
asociado con x bajo la funcin f. Para indicar lo anterior puede escribirse y = f(x) que se lee y
es la imagen de x bajo la funcin f. A la derecha se escribe el smbolo de la funcin y luego, entre
parntesis, el smbolo que representa el elemento tpico de D. A la izquierda se escribe el smbolo
que designa la imagen del elemento de C. Los smbolos estn unidos por el smbolo de igualdad.
Este simbolismo suele utilizarse para designar a toda la funcin f, de modo que puede hablarse de
la funcin y = f(x). El smbolo y = f(x) se utiliza entonces con dos propsitos diferentes. Designa
la funcin completa f y tambin que y es la imagen de algn punto x en particular. Este doble uso
normalmente no causa confusin, pero en ciertos casos puede suceder, y por lo tanto debe acudirse
a algn smbolo especial para designar toda la funcin.
Ejemplo 8.5 Escribir la funcin que exprese la dependencia entre el radio r de un cilindro y
su altura h siendo el volumen dado V = 2.
Solucin
Sabemos que el volumen del cilindro es V = r
2
h. Como es volumen es dado y es igual a 2, entonces
2 = r
2
h despejamos r y obtenemos
r
2
=
2
h
r =
_
2
h
.
Ejemplo 8.6 Expresar la dependencia entre la longitud b de un cateto de un tringulo rectn-
gulo y la longitud a de otro lado, siendo la hipotenusa constante e igual a c = 3.
Solucin
Por el graco tenemos que c
2
= a
2
+b
2
, como c = 3, entonces 9 = a
2
+b
2
. Despejando b, obtenemos
b
2
= 9 a
2
b =
_
9 a
2
.
CAPTULO 8. FUNCIONES ALGEBRAICAS 393
Ejemplo 8.7 Una torre tiene la siguiente forma: Un cono circular recto truncado cuyos radios
de base son 2R y R y cuya altura es R, sostiene un cilindro de radio R y de altura 2R. Este ltimo
sostiene, a su vez, una semiesfera de radio R. Exprese el rea S de la seccin transversal de la
torre como funcin de la distancia x que media entre la seccin y la base inferior del cono.
Solucin
Sabemos que
S
1
=
R
2
2
, S
2
= 4R
2
, S
3
= 3R
2
.
Como la altura total es h
T
= 2x = 4R, entonces
R =
x
2
R
2
=
x
2
4
El rea total es
S
t
= S
1
+S
2
+S
3
=
R
2
2
+ 4R
2
+ 3R
2
=
_

2
+ 7
_
R
2
Por tanto el rea total es
S
T
=
_

2
+ 7
_
x
2
4
.
Ejemplo 8.8 Una esfera de radio R lleva inscrito un cilindro. Hallar la dependencia funcional
entre el volumen V del cilindro y su altura x.
Solucin
Sabemos que el volumen del cono es V
C
= r
2
x, haciendo una relacin de tringulos, tenemos que
R
2
= r
2
+
_
x
2
_
2
R
2
= r
2
+
x
2
4
despejando el radio del cilindro, tenemos
r
2
= R
2

x
2
4
De esta manera obtenemos el volumen del cilindro en funcin de la altura x:
V
C
=
_
R
2

x
2
4
_
x.
Ejemplo 8.9 Un cilindro circular recto est inscrito en una esfera si la circunferencia de las
bases del cilindro est sobre la supercie de la esfera. Si la esfera tiene radio R, expresar el volumen
del cilindro en funcin del radio r de su base.
CAPTULO 8. FUNCIONES ALGEBRAICAS 394
Solucin
La gura indica que 2h es la altura del cilindro, entonces
h =
_
R
2
r
2
y el volumen pedido es
V = r
2
2h V = 2r
2
_
R
2
r
2
.
Ejemplo 8.10 Una lata cilndrica cerrada tiene radio r y altura h:
a) Si el rea de la supercie S de la lata es una constante, exprese el volumen V de la lata en
funcin de S y r.
b) Si el volumen de la lata es una constante, exprese el rea de la supercie S en trminos de
V y r.
Solucin
a) Sabemos que la supercie del cilindro se calcula con la frmula S = 2rh. El volumen del
cilindro es V = r
2
h. Para expresar el volumen del cilindro en funcin de r y S, hacemos que
V =
2
3
r
2
h =
1
2
r 2rh
Reemplazando la frmula de la supercie en el volumen, obtenemos
V =
1
2
rS
b) Del inciso anterior, despejamos S
V =
1
2
rS S =
2V
r
.
La dependencia funcional de cierta magnitud y en funcin de otra x signica que a cada valor de
x corresponde un valor determinado de y. En estas condiciones, la magnitud x es llamada variable
independiente e y, funcin de esta variable. En determinadas ocasiones x es llamada argumento de
la funcin.
As pues, el dominio de existencia de una funcin se determina por la propia ley que dene la
funcin, mientras que el dominio de denicin de la misma se preja por las condiciones o por el
sentido del problema a resolver, es decir, el dominio de denicin de una funcin lo puede constituir
cualquier parte del dominio de existencia de la funcin, o bien los dominios mencionados pueden
coincidir completamente. De esta manera, siempre cuando se diga que est dada una funcin
y = f(x), se considera que ya est prejado tambin su dominio de denicin D; este ltimo o
CAPTULO 8. FUNCIONES ALGEBRAICAS 395
bien se indica explcitamente o bien existe el dominio de existencia de dicha funcin. En lo que
se reere al codominio de la funcin y = f(x), ste se calcula en base al dominio de denicin ya
prejado. El dominio de y = f(x) puede visualizarse proyectando la grca sobre el eje de las X;
la proyeccin de la grca sobre el eje de las Y nos da el codominio.
Ejemplo 8.11 Escriba el rea A de un rectngulo de 140 m de permetro como funcin de la
longitud x de la base.
Solucin
Como se indica en la gura, sea y la altura del rectngulo. Entonces, su rea estar dada por
A = xy. Para eliminar y y obtener A como funcin de x sola, usamos el hecho de que el permetro
del rectngulo es 2x + 2y = 140, as que y = 70 x. Por lo tanto, la ecuacin del rea produce
A = x(70 x).
Adems de esta ltima frmula, debemos especicar tambin el dominio de la funcin A. Slo
los valores x > 0 producirn rectngulos efectivos. Por razones similares, tendremos la restriccin
y 0. Puesto que y = 70 x, se sigue que x 70. As es que la denicin completa de nuestra
rea es
A(x) = x(70 x), 0 x 70.
Ejemplo 8.12 A una esfera de radio r se circunscribe un cono. Encuentre la dependencia
entre el volumen V de dicho cono y su altura; indique el dominio de la funcin obtenida.
Solucin
Por el grco podemos ver que
H = 2r +y y = H 2r
Por el teorema de Pitgoras tenemos
(y +r)
2
= x
2
+r
2
x =
_
(y +r)
2
r
2
x =

H
2
2Hr
Haciendo una relacin de tringulos obtenemos
x
2r +y
=
r
R
R =
r(2r +y)
x
Sabemos que el volumen de un cono es V =
1
3
R
2
H. Reemplazando en esta frmula, los valores
obtenidos anteriormente, tenemos
V =
1
3

_
rH

H
2
2Hr
_
2
H V =

3
r
2
H
3
H
2
2Hr
V =

3

r
2
H
2
H 2r
Esta funcin est denida, cuando H 2r ,= 0. De esta forma podemos deducir que el dominio de
la funcin es H R
+
2r.
CAPTULO 8. FUNCIONES ALGEBRAICAS 396
Ejemplo 8.13 Una esfera de radio R lleva inscrito un cono recto. Hallar la dependencia
funcional entre el rea de la supercie lateral S del cono y su generatriz x. Indique el dominio de
esta funcin.
Solucin
Haciendo la relacin de tringulos, obtenemos
x
2
R +h
=
R
x
=
y
r

x
2(R +h)
=
R
x
h =
x
2
2R
R
Aplicando el teorema de Pitgoras, tenemos
x
2
= (R +h)
2
+r
2
x
2
=
_
R +
x
2
2R
R
_
2
+r
2
r =
_
x
2

x
4
4R
2
El rea de la supercie lateral del cono en funcin de su generatriz x es A = rx. Reemplazando
los valores encontrados anteriormente, tenemos
A = x
_
x
2

x
4
4R
2
A = x
_
4R
2
x
2
x
4
4R
2
A =
x
2R
_
4R
2
x
2
Esta funcin est denida si
_
R ,= 0
4R
2
x
2
0

_
R ,= 0
(x 2R)(x + 2R) 0

_
R ,= 0
x [2R; 2R]
Por tanto el dominio de la funcin es x (0; 2R].
Ejemplo 8.14 Un rectngulo cuyo permetro jo es 36 gira en torno a uno de sus lados, S,
para generar un cilindro circular recto. Exprese el volumen V de este cilindro en funcin de la
longitud x del lado S.
Solucin
El permetro del rectngulo est dado por P = 2x +2r. Como el permetro es igual a 36, entonces
36 = 2x + 2r r = 18 x
El volumen del cilindro est dado por V = r
2
x. Reemplazando r en la frmula del volumen,
obtenemos
V = (18 x)
2
x
CAPTULO 8. FUNCIONES ALGEBRAICAS 397
Ejemplo 8.15 Para estudiar la tasa a la que aprenden los animales, un estudiante de psi-
cologa realiz un experimento en el que de modo repetido se enviaba una rata a travs de un
laberinto de laboratorio. Suponga que el tiempo requerido por la rata para atravesar el laberinto en
la n-sima prueba era aproximadamente f(n) = 3 +
12
n
minutos:
a) Cul es el dominio de la funcin?
b) Para qu valores de n tiene signicado f(n) en el contexto del experimento psicolgico?
c) Cunto tiempo se tom la rata para atravesar el laberinto en la tercera prueba?
d) En qu prueba atraves la rata por primera vez el laberinto en 4 minutos o menos?
e) Segn la funcin f, qu le suceder al tiempo requerido para que la rata atraviese el laberinto
a medida que aumenta el nmero de pruebas? Podr la rata atravesar alguna vez el laberinto en
menos de tres minutos?
Solucin
a) Sabemos que
f(n) = 3 +
12
n
f(n) =
3n + 12
n
Para que esta funcin este denida, hacemos que n ,= 0, por tanto el dominio ser n R 0.
b) Como no pueden existir pruebas negativas, n debe ser mayor que cero, es decir n > 0.
c) Haciendo n = 3 en la ecuacin original, obtenemos
f(3) = 3 +
12
3
= 7 minutos
d) Haciendo que f(n) 4, entonces
3n + 12
n
4
3n + 12
n
4 0
n 12
n
0
De aqu deducimos que n 12. Es decir a partir de la prueba 12, la rata atravesara el laberinto
en 4 minutos o menos.
e) A medida que el nmero de pruebas aumenta, la rata disminuye el tiempo en que atraviesa el
laberinto. Es imposible que la rata pueda atravesar el laberinto en menos de 3 minutos, por cuanto
la desigualdad obtenida tendra la forma
3n + 12
n
< 3
3n + 12
n
3 < 0
12
n
< 0
lo cul es un absurdo, porque n tendra que ser negativa, lo cul no es posible por el inciso b).
Ejemplo 8.16 Suponga que durante un programa nacional para inmunizar a la poblacin con-
tra cierto tipo de gripe, los funcionarios de salud pblica encontraron que el costo de vacunar al
x % de la poblacin era aproximadamente f(x) =
150x
200x
millones de dlares:
a) Cul es el dominio de la funcin f?
b) Para qu valores de x tiene f(x) una interpretacin prctica en este contexto?
c) Cul fue el costo de vacunacin del primer 50 % de la poblacin?
d) Cul fue el costo de vacunacin del segundo 50 % de la poblacin?
e) Qu porcentaje de la poblacin se haba vacunado despus de una inversin de 15 millones
de dlares?
Solucin
a) Para que esta funcin este denida, hacemos que
200 x ,= 0 x ,= 200
CAPTULO 8. FUNCIONES ALGEBRAICAS 398
por tanto el dominio ser x R 200.
b) Como f(x) no puede ser negativa, entonces 0 x < 200.
c) El costo de vacunacin del primer 50 % de la poblacin est dado por
f(50) =
150 50
200 50
=
150 50
150
= 50
Por tanto, cuando se ha vacunado el 50 % de la poblacin el costo es aproximadamente 50 millones
de dlares.
d) Para calcular el costo de vacunacin del segundo 50 % de la poblacin, hacemos f(100)f(50),
es decir
f(100) f(50) =
150 100
200 100
50 =
150 100
100
50 = 150 50 = 100
es decir, se invierten 100 millones de dlares.
e) Para calcular el porcentaje de vacunados, con una inversin de 15 millones de dlares, hacemos
lo siguiente
15 =
150x
200 x
3000 15x = 150x 165x = 3000 x 18, 18
Es decir, se vacunaron aproximadamente el 18.18 %.
Ejemplo 8.17 De acuerdo con la ley de Boyle, la presin p (libras por pulgada cuadrada) y
el volumen v (pulgadas cbicas) de cierto gas satisfacen la condicin pv = 100. Supngase que
50 v 150. Cul es el rango de los valores posibles de la presin?
Solucin
Si sustituimos v =
100
p
en la desigualdad dada 50 v 150, obtenemos
50
100
p
150
Se sigue que tanto
50
100
p

100
p
150
es decir, que tanto p 2 como p
2
3
. Entonces, la presin p debe pertenecer al intervalo cerrado
[2/3; 2].
Ejemplo 8.18 El gerente de una tienda de muebles compra refrigeradores al precio de mayoreo
de $ 250 cada uno. Sobre la base de experiencias pasadas, el gerente sabe que puede vender 20
refrigeradores al mes a $ 400 cada uno y un refrigerador adicional al mes por cada reduccin de $
3 en el precio de venta. Exprese la utilidad mensual P como funcin del nmero x de refrigeradores
mensualmente vendidos.
Solucin
Interpretemos el enunciado del problema con el signicado de que el precio de venta p de cada
refrigerador es impuesto al principio de cada mes y que todos los refrigeradores se venden al mismo
precio. Puesto que el precio de mayoreo es de $ 250 cada uno, la utilidad en la venta de cada
refrigerador es p 250, y, por lo tanto, la utilidad mensual total P de la venta de x refrigeradores
estar dada por P = x(p 259). Para expresar P como funcin slo de x, tenemos que eliminar la
variable p. Sea n el nmero de reducciones de $ 3 hechas al precio de venta original, de modo que
p = 4003n. Entonces, se pueden vender n refrigeradores ms (es decir, ms que los 20 originales)
y por lo tanto x = n + 20; es decir n = x 20. Por lo tanto, deducimos que
P = 400 3(x 20) = 460 3x.
CAPTULO 8. FUNCIONES ALGEBRAICAS 399
Mediante la sustitucin de este valor de p en la ecuacin P = x(p 250), obtenemos la frmula
P = x(210 3x) = 3x(70 x)
para la utilidad mensual total P como funcin del nmero x de refrigeradores vendidos al mes.
Para concluir, debemos encontrar el dominio relevante de valores de x.
Por cierto, x 0. Por otra parte, sera inaceptable una utilidad negativa, por lo que x 70. En
consecuencia, la descripcin completa de nuestra funcin utilidad es
P(x) = 3x(70 x), 0 x 70.
Ejemplo 8.19 Determine el dominio de la expresin:
a) f(x) =
x + 2
x
2
5x + 4
+
x 4
x
2
3x + 2
; b) f(x) =
_
6
6 x

_
1 x
1 2x x
2
;
c) f(x) =
6

x
2
1
x

x
5

x
2
2x + 2
.
Solucin
a) La expresin est denida si se cumplen las siguientes condiciones
_
x
2
5x + 4 ,= 0
x
2
3x + 2 ,= 0

_
(x 4)(x 1) ,= 0
(x 2)(x 1) ,= 0

_
x ,= 1, x ,= 4
x ,= 1, x ,= 2
Por tanto, el dominio es x R 1, 2, 4.
b) La expresin est denida si se cumplen las siguientes condiciones:
_

_
x
6x
0
6 x ,= 0
1x
12xx
2
0
1 2x x
2
,= 0

_
x
x6
0
x 6 ,= 0
x1
x
2
+2x1
0
x
2
+ 2x 1 ,= 0

_
x [0; 6]
x ,= 6
x [

2 1;

2 1] [1; +)
x ,=

2 1, x ,=

2 1
Por lo tanto el dominio de la expresin es x [0;

2 1) [1; 6).
c) La expresin est denida si se cumplen las siguientes condiciones:
_

_
x
2
1 0
x ,= 0
x
2
2x + 2 ,= 0

_
(x 1)(x + 1) 0
x ,= 0
x
2
2x + 2 ,= 0

_
x (; 1] [1; +)
x ,= 0
x R
Por lo tanto el dominio de la expresin es: x (; 1] [1; +).
Ejemplo 8.20 Determine el codominio de la expresin:
f(x) =
_
x 3
1 3x + 2x
2
Solucin
Para determinar el codominio de la funcin, debemos despejar la variable x:
y =
_
x 3
1 3x + 2x
2
y
2
=
x 3
1 3x + 2x
2
2y
2
x
2
(3y
2
+ 1)x + (y
2
+ 3) = 0 x =
3y
2
+ 1
_
y
4
18y
2
+ 1
4y
2
CAPTULO 8. FUNCIONES ALGEBRAICAS 400
Esta nueva expresin esta denida si y
4
18y
2
+ 1 0 y y ,= 0. Es decir
_
y (; 2

5] [2

5; 1 +

5] [2 +

5; +)
y ,= 0
Como y 0, entonces el codominio buscado es (0; 2 +

5] [2 +

5; +).
8.2. Tarea
1. Considere las siguientes relaciones:
1
1
= (a, b) A x A/a = b con A = 1, 2, 3; 1
2
= (a, b) N x N/2a +b = 9.
1
3
= (a, b) A x A/a divide a b si A = 1, 2, 3, 4, 5.
1
4
= (a, b) A x A/ab 0 si A = 2, 1, 0, 1, 2, 3.
1
5
= (a, b) A x A/a
2
+b
2
> 3 si A = 1, 0, 1, 2, 3.
a) Determine por extensin 1
i
, i = 1, 2, 3, 4, 5; b) Determine Dom(1
i
), Rec(1
i
);
c) Determine por extensin 1
1
.
2. Considere las siguientes relaciones denidas en Z:
1
1
= (a, b)/a = b
2
; 1
2
= (a, b)/a+a
2
= b +b
2
; 1
3
= (a, b)/ab; es mltiplo de 3
1
4
= (a, b)/ c Z tal que a = cb; 1
5
= (a, b)/ c Z tal que a b = 2c
Determine cuales de las relaciones son: reejas, simtricas, transitiva, antisimetricas.
3. Sean las relaciones denidas en R:
1 = (x, y)/y = 2x y = (x, y)/y = 2x
3
.
Determine 1 y 1 .
4. Determine el dominio y la imagen de cada relacin. Diga cules son funciones:
a) (x, y)/y = 2x
2
3;
b)
_
(x, y)/y =
2x
3x + 1
_
;
c)
_
(x, y)/y =
5x
x + 3
_
;
d) (x, y)/y = 5

x;
e)
_
(x, y)/y =
3
2x 1
_
;
f )
_
(x, y)/y =
x
5 3x
_
.
5. Encuentre el dominio, rango y gracar cada una de las relaciones:
CAPTULO 8. FUNCIONES ALGEBRAICAS 401
a) xy(x +y 2) = x y + 1;
b) (x +y)
2
+ 2x + 2y = 0;
c) x
2
2xy + 4x y = 0;
d) xy
2
+ 5x
2
y + 4x
2
+xy 9 = 0;
e) y
2
(x
2
+ 1) 6x
2
y +x
4
= 0;
f ) x
4
+y
4
7x
2
3y
2
+ 14 = 0;
g) x
4
+x
2
y
2
2x
2
y xy
2
+y
2
= 0;
h) y
2
(x 1)
2
(x 2) = 0;
i) y
2
(4x 1) = x
2
(x 1);
j) x
2
+y
2
6x + 4y + 4 = 0;
k) x
2
y
2
12x + 8y + 7 = 0;
l) 9x
2
+ 4y
2
18x + 24y + 45 = 0;
m) 4y
2
3x
2
+ 8y 12x 16 = 0;
n) 4x
2
9y
2
+ 16x + 18y + 7 = 0.
6. Sea f = (x, 3), (2, x
2
), (1, 4), (2, 1) una funcin, encuentre el valor de x.
7. Determine el valor de ab si el conjunto de pares ordenados f = (2, 5), (1, 3), (2, 2a
b), (1, b a), (a +b
2
, a) representa una funcin.
8. Si el conjunto de pares ordenados f =
_
(1, a),
_
2,
a
2
bc
+
b
2
ac
+
c
2
ab
_
, (3, a +b), (3, c)
_
rep-
resenta una funcin, determine el valor de f(2).
9. Sea f(x) =

5 x +
1

x 2
una funcin real de variable real, determine su dominio.
10. Sean las funciones f(x) =
4
_
9 x
2
, g(x) =
x + 5
x 2
y h(x) = x
5
+4x
3
+3x
2
x+2. Calcule
Dom(f) Dom(g) Dom(h).
11. Hallar el dominio de la funcin f(x) =
x
2
+ 2x + 1

9 4x
2
.
12. Encuentre el dominio de la funcin f(x) =
_
1

1 x.
13. Encuentre el dominio de la funcin f(x) =

x 1
6

x 1
+
3

2 x
5 x
+ 3x + 1.
14. Sea f(x) =

4x x
2
, encuentre Ran(f) Dom(f).
15. Dada la funcin f : D
f
R R tal que f(x) =
1
x
2
+x
2
1. Determine Dom(f)Ran(f).
16. Determine el menor valor que toma la funcin f(x) = x(x + 2) + 2 si su dominio es
Dom(f) :
_
x R/ 1 <
1
x 1
< 1
_
.
17. Calcule Dom(f) Ran(f) de la funcin f(x) =
_
3x x [2; 3),
x
2
, 3 x < 5
18. Determine el rango de la funcin f =
__
x,
x
x 2
_
/

x(x
2
4) > 0
_
.
CAPTULO 8. FUNCIONES ALGEBRAICAS 402
19. Encuentre el rango de la funcin
f(x) =
_

x
2
9 5 x 3
[x + 3[ 2, 0 < x 5
3x16
x5
, x > 6
20. Si f es una funcin lineal de pendiente 8 e intercepto con el eje Y, 5, encuentre el valor de
f(0) +f(1) +f(1).
21. Sea f una funcin lineal de pendiente negativa, tal que Dom(f) : (1; 2) y Ran(f) : (3; 4).
Hallar f(x).
22. Si la grca de g(x) = 4(x 2)(x + 3) intercepta al eje X en los puntos P y Q, entonces
encuentre la longitud del segmento PQ.
23. Las grcas de x
2
+ y = 5 y x + y = 5 se cortan en 2 puntos, segn ello determine la
distancia entre estos 2 puntos.
24. Determine el rango de f(x) = [x + 2[ [x 2[.
25. Determine el dominio de la funcin f(x) =
_
1

x
2
x[[2x 1[] 2x
.
26. Determine el rango de la funcin f(x) =
_

2 x
2

_
, x (2; 1].
27. Dada la relacin 1 = (x, y) R
2
/x y R
+
, hallar su grca.
28. Dada la relacin 1 = (x, y) R
2
/y < x
2
+ 1 y 1/2, hacer su grca.
29. Dada la relacin 1 = (x, y) R
2
/

y < x y > x, hacer su grca.


30. Graque la relacin 1 = (x, y) R
2
/[2x +y[ < 3.
31. Dadas las relaciones 1
1
= (x, y) R
2
/y
2
> x
2
y 1
2
= (x, y) R
2
/[y[ x
2
. Graque
1
1
1
2
.
32. Graque la relacin 1
1
= (x, y) R
2
/[3x y[ +[x +y[ [4x[.
33. Dada la relacin 1 = (x, y) R
2
/[y[ < [x[, [x[ < 3. Encuentre el nmero de elementos
del conjunto P= (x, y) 1/x, y Z.
CAPTULO 8. FUNCIONES ALGEBRAICAS 403
34. Dada la funcin f(x) = mx y la circunferencia (x 2)
2
+ (y + 1)
2
= 1. Determine los
valores de m para que la frca de f tenga puntos comunes en la circunferencia.
35. Dadas las funciones f = (3, 2), (1, 0), (2, 3), (4, 1) y g = (6, 3), (1, 2), (4, 0), (3, 1).
Determine la funcin f
2
+
f
g
.
36. Si f +g = (1, 2), (2, 4), (3, 6) y f g = (1, 2), (2, 2), (3, 2), determine Ran(f
2
g
2
).
37. Dadas las funciones f = (1, 2), (2, 3), (3, 4), (4, 5) y g = (2, 3), (3, 4), (4, 5), (5, 1). En-
cuentre f g e indique la suma de los elementos del dominio.
38. Dadas las funciones f = (2, 0), (1, 4), (3, 1), (5, 2) y g = (2, 1), (0, 3), (1, 4), (2, 0), (4, 5).
Encuentre f g.
39. Sean las funciones g = (1, 2), (2, 0), (4, 5), (3, 1) y f g = (1, 4), (4, 7), (3, 1). Encuen-
tre la suma de elementos del dominio de la funcin f.
40. Dadas las funciones f = (2, 1), (2, 3), (1, 5), (3, 4), (7, 8) y g = (3, 2), (3, 1), (7, 2), (2, 4).
Encuentre el valor de
_
(f g)(3) + 3g
2
(3) (f +g)(2)
f(7)g(7)
_
1/2
41. Un estudio ambiental de una cierta ciudad sugiere que el nivel diario medio de monxido
de carbono en el aire ser de c(p) = 0, 4p + 1 partes por milln cuando la poblacin sea p
miles. Se estima que dentro de t aos la poblacin de la comunidad ser de p(t) = 8 + 0, 2t
2
miles. Determine:
a) El nivel futuro de monxido de carbono en la comunidad, como funcin del tiempo.
b) El nivel de monxido de carbono dentro de 10 aos.
42. Los bilogos han hallado que la velocidad de la sangre en una arteria es una funcin de
la distancia de la sangre al eje central de la arteria. De acuerdo con la ley de Poiseuille, la
velocidad de la sangre medida en centmetros por segundo, que est a r centmetros del eje
central de una arteria viene dada por la funcin S(r) = c(R
2
r
2
), donde c es una constante
y R es el radio de la arteria. Suponga que para una cierta arteria, c = 1, 76x105 centmetros
y R = 1, 2x10
2
centmetros. Calcule:
a) La velocidad de la sangre en el eje central de esta arteria.
b) La velocidad de la sangre equidistante entre la pared de la arteria y el eje central.
43. Dos rayos, entre los que el ngulo es igual a 60

, tienen origen comn. Desde ste, por


uno de los rayos sali una partcula a una velocidad v y, pasada una hora, por el otro rayo,
la segunda partcula a velocidad 3v. Determine la dependencia entre la distancia entre las
partculas y el tiempo de movimiento de la primera. A qu distancia mnima se aproximarn
las partculas despus de la salida de la segunda de ellas?
CAPTULO 8. FUNCIONES ALGEBRAICAS 404
44. Dado que 0

C es lo mismo que 32

F y que un cambio de 1

C equivale a un cambio de
1, 8 F, exprese la temperatura Celsius C en funcin de la temperatura Fahrenheit F.
45. Una caja rectangular tiene 125 cm
3
de volumen y una base cuadrada de longitud x cm. en
su arista. Exprese el rea A del rectngulo como funcin de x.
46. Un rectngulo cuya base tiene longitud x est inscrita en un crculo de radio 2. Exprese el
rea A del rectngulo en funcin de x.
47. Un campo petrolero que contiene 20 pozos ha estado produciendo 4000 barriles diarios de
petrleo. Por cada nuevo pozo que es perforado, suponga que la produccin diaria de cada
uno disminuye 5 barriles. Escriba la produccin diaria del campo petrolero en funcin del
nmero x de pozos nuevos que se perforan.
48. Un cilindro circular recto tiene un volumen de 1000 cm
3
y el radio de su base x cm. Exprese
la supercie total A del cilindro como funcin de x.
49. A y B parten del mismo sitio. A camina 4 kilmetros por hora y B 5 kilmetros por hora:
a) Cunto caminan en x horas?
b) Cmo estn de alejados entre s al cabo de x horas si saliendo al mismo tiempo han
caminado en direccin opuestas?
c) Qu distancia los separa cuando A ha caminado x > 2 horas si van en la misma direc-
cin, pero B sale 2 horas despus que A?
d) Por cuntas horas tiene que caminar B para alcanzar a A?
50. La tasa a la cual la temperatura de un objeto cambia es proporcional a la diferencia entre
su propia temperatura y la del medio que lo rodea. Exprese esta tasa como una funcin de
la temperatura del objeto.
51. Se consideran las secciones del tetraedro regular ABCD paralelas a la arista AB y a la
altura DO del tetraedro. Encuentre la dependencia entre el rea S de la seccin y la distancia
x entre el plano de la seccin y la arista AB si la altura de la cara del tetraedro es igual a b.
Hallar el valor mximo de S.
52. Un almacn de discos ofrece la siguiente oferta: si se compran 5 discos compactos, a $ 10
cada uno, pueden obtenerse discos adicionales a mitad de precio. Hay un lmite de 10 discos
por cliente. Exprese el costo de los discos como una funcin de la cantidad comprada.
53. Un tren parte de la estacin a medioda y viaja hacia el este a 30 kilmetros por hora. A
las 2 p.m. del mismo da un segundo tren deja la estacin y viaja hacia el sur a 25 kilmetros
por hora. Exprese la distancia y entre ambos trenes en funcin de t, tiempo que ha estado
rodando el segundo tren.
CAPTULO 8. FUNCIONES ALGEBRAICAS 405
54. Un tren parte de la estacin a medioda y viaja hacia el este a 30 kilmetros por hora.
A las 14:00 horas del mismo da un segundo tren deja la estacin y viaja hacia el sur a 25
kilmetros por hora. Expresar la distancia d entre ambos trenes en funcin de t, tiempo que
ha estado rodando el segundo tren.
55. La tasa a la que se propaga una epidemia en una comunidad es conjuntamente proporcional
a la cantidad de personas que han contrado la enfermedad y al nmero de personas sanas. Ex-
prese esta tasa como una funcin de la cantidad de personas que han contrado la enfermedad.
56. La tasa a la que las personas resultan implicadas en un escndalo gubernamental es con-
juntamente proporcional a la cantidad de personas ya implicadas y a la cantidad de personas
involucradas que an no han sido implicadas. Exprese esta tasa como una funcin de la can-
tidad de personas que han sido implicadas.
57. En determinada fbrica, el costo de instalacin es directamente proporcional al nmero
de mquinas utilizadas y el costo de operacin es inversamente proporcional al nmero de
mquinas empleadas. Exprese el costo total como una funcin del nmero de mquinas uti-
lizadas.
58. Se tiende un cable desde una planta de energa, a un lado de un ro de 900 metros de ancho,
hasta una fbrica en el otro lado, 3000 metros ro abajo. El cable ir en lnea recta desde la
planta de energa a algn punto P en la orilla opuesta, y luego a lo largo de la orilla hasta
la fbrica. El costo de tender el cable por el agua es $ 5 por metro, mientras que el costo
sobre tierra es $ 4 por metro. Si x es la distancia desde P al punto del otro lado del ro en-
frente de la planta de energa, exprese el costo de instalacin del cable como una funcin de x.
59. Un automvil que viaja hacia el Este a 80 Km. por hora y un camin que viaja hacia el
Sur a 60 Km. por hora parten de la misma interseccin. Exprese la distancia entre ellos como
una funcin del tiempo.
60. Se va a construir una caja sin tapa con una hoja cuadrada de cartn cuyo lado tiene una
longitud de 50 cm. Primero, se recortan cuatro pequeos cuadrados, cada uno de los cuales
tiene lados de x cm. de longitud, de las cuatro esquinas de la hoja de cartn. Despus, los
cuatro faldones resultantes se doblan hacia arriba para formar los cuatro lados de la caja, que
tendr una base cuadrada y una profundidad de x cm. Exprese el volumen V como funcin
de x.
61. Una escalera de 25 metros de largo se apoya contra una pared vertical, estando su pie a
7 metros de la base de la pared. Si el pie de la escalera se aleja de la pared a razn de 2
metros por segundo, expresar la distancia y del extremo superior de la escalera sobre el nivel
del suelo como funcin del tiempo t durante el movimiento.
62. Una lancha de motor que navega x km/h en aguas tranquilas, se encuentra en un ri cuya
corriente es de y < x km/h:
a) Cul es la velocidad de la lancha subiendo el ro?
CAPTULO 8. FUNCIONES ALGEBRAICAS 406
b) Cul es la velocidad de la lancha bajando el ro?
c) Cunto sube la ancha en 8 horas?
d) Qu tiempo tarda la lancha para bajar 20 km. si el motor se para a los 15 km. del
punto de partida?
63. Una pelota se deja caer desde el tejado de un edicio. Si su altura respecto del suelo medida
en metros, despus de t segundos viene dada por la funcin H(t) = 16t
2
+256. Determine:
a) A qu altura estar la pelota despus de 2 segundos?
b) Qu distancia recorrer la pelota durante el tercer segundo?
c) Cul es la altura del edicio?
d) Cundo llegar al suelo la pelota?
64. Dada f : R R, determine el dominio de las expresiones:
a) f(x) =

13 +x

10 + 2x

19 + 2x 5
;
b) f(x) =
[x + 2[ + 1 2x 2x
2
[2x + 2[ 1
;
c) f(x) =

x
2
5x
x
2
+ 2x 5
;
d) f(x) =
3

15 + 6x
3

25 +x
x
4
+ 2x 20
;
e) f(x) =
x

x 1
x + 2

x + 1
;
f ) f(x) =
x
3
+ 2x

x
2
3x
;
g) f(x) =
x +

x + 5
x

x + 1
;
h) f(x) =
1 x
2

x + 3x
;
i) f(x) =
x
x
2
1
+
2
x
;
j) f(x) =
1 +

x + 1
1

x 1
;
k) f(x) =

x + 2

6 x
x 2
;
l) f(x) =

15 +x

17 x

3 +x 2
;
m) f(x) =
x

x + 1

x
2
1
x
;
n) f(x) =

x
2
1

x + 1

x 1
;
o) f(x) =
3

x + 3

x 1
3

x + 1
;
p) f(x) =

x + 11 2

x 1
x
2
25
;
q) f(x) =

x
2
1 +

x 1

x
2
+ 2x 1
;
r) f(x) =

x
2
+ 1

x
2
1

x
2
+x x 1
;
s) f(x) =

x
2
x + 1
x
2
+x 3
;
t) f(x) =

x + 2 +x

x 1 +

x
;
u) f(x) =
1

x
2
1 x
;
v) f(x) =

x + 1

x 1 +

x
;
w) f(x) =

x
2
+ 1 x

x + 1

x
;
x) f(x) =
2x

4x
2
1

x
2
+ 3 x
;
y) f(x) =

x
2
16 +x

x
2
4 +x
;
z) f(x) =

x
2
+ 1 x

x
3
+ 1 x

x
.
65. Dada f : R R, determine el dominio de las expresiones:
CAPTULO 8. FUNCIONES ALGEBRAICAS 407
a) f(x) =
_
9x
2

x
3
x
4

x
2
3
;
b) f(x) =
x
2
+ 3x 1

x
2
+ 5x 1
;
c) f(x) =

x + 1

x
2
+ 1
;
d) f(x) =

x
2
3x
3

1 x +
3

1 +x
;
e) f(x) =
_

x 1
3
_

x + 1
;
f ) f(x) =
3

3 x x + 2
3

1 + 2x 1
;
g) f(x) =

x
2
+x + 1 2 x
x + 3
;
h) f(x) =

4 x + 1
2 9

9 +x
;
i) f(x) =

x 1 +

x + 1
x 1
;
j) f(x) =

x
4
+ 8x
2
+ 3

x
4
+x2
;
k) f(x) =

x + 1

x
2
1 +

x 1
;
l) f(x) =

x
2
+ 3x 2
3

x
2
3x 2
;
m) f(x) =
_
x
6 x
;
n) f(x) =
4

x
3
+x

x
x + 2 +

x + 1
;
o) f(x) =
1
_
(8 2x x
2
)
3
;
p) f(x) =
3

x
3

x + 1
4

x + 1
4

x
.
66. Dada f : R R, determine el codominio de las expresiones:
a) f(x) = x +
1
x
;
b) f(x) =

x
2
+ 1;
c) f(x) =
_
9x
2
+ 1
x
;
d) f(x) =
x
2
+ 2x 2
x
2
x + 1
;
e) f(x) =

x
2
+ 2x + 2;
f ) f(x) = x
1
x
2
;
g) f(x) =
x
2
+ 4
x
;
h) f(x) =
4

x
2
1;
i) f(x) =
2x
x
2
+ 9
;
j) f(x) =

x
2
+ 2x 1;
k) f(x) =
x
2
x
2
+ 1
;
l) f(x) =
3 x
2
3 +x
2
;
m) f(x) =

4x x
2
;
n) f(x) =

8 2x x
2
;
o) f(x) =
2x 3
2x
2
+ 3x 2
;
p) f(x) =
3x 2
2x2 + 3x 9
;
q) f(x) =
2x
2
5x + 2
3x
2
+ 7x 6
;
r) f(x) =
_
x + 1
x 1
_
4
;
s) f(x) = x +
_
x
2
1;
t) f(x) = x
_
x
2
2x;
u) f(x) =
3
_
x
2
1 +x
.
67. De acuerdo con la ley de Boyle, la presin p (libras por pulgada cuadrada) y el volumen
v (pulgadas cbicas) de cierto gas satisfacen la condicin pv = 800. Cul es el rango de los
valores posibles de la presin, dado que 100 v 200?
68. La relacin entre la temperatura Fahrenheit F y la temperatura Celsius C est dada por
F = 32 +
9
5
C, Si el rango de temperaturas en cierto da va de la mnima 70

F a la mxima
de 90

F, cul es el rango de la temperatura en gados Celsius?.


CAPTULO 8. FUNCIONES ALGEBRAICAS 408
69. El periodo T (en segundos) de un pndulo simple de longitud L (en pies) est dado por
T = 2
_
L
32
. Si 3 < L < 4, cul es el rango de valores posibles de T?
8.3. Funcin inversa
Puesto que x no est determinada unvocamente cuando se conoce y, no nos atreveremos a
usar la notacin x = g(y) y nos abstendremos de llamarla funcin. Tal correspondencia en la que
a cada y puede corresponder ninguno, uno o varios valores de x se llama relacin. El conjunto de
pares (y, x) se denomina relacin inversa de y = f(x) si contiene el par (y
0
, x
0
) si y solamente si
y
0
= f(x
0
). La situacin se simplica ms cuando se seleccionan algunos de los pares a partir de
la relacin, en forma tal que se da origen a una funcin inversa x = g(y).
Denicin 8.9 Funcin inversa
Se llama funcin inversa de y = f(x) para y en un dominio D, a una funcin x = g(y), denida
para y en D, si f(g(y)) = y para cada y D.
Como para una f(x) ja puede haber muchas funciones inversas, a cada una de stas se la
llama rama de la funcin inversa.
Toda funcin y = f(x) aplica el dominio de existencia de la funcin sobre el codominio de tal
modo que a cada x del dominio de existencia le corresponde el nico valor y del codominio. As
pues, las funciones pueden dividirse en dos grupos:
1. Funciones que realizan una aplicacin biunvoca del dominio de existencia sobre el codo-
minio.
2. Funciones que no poseen esta propiedad.
Si las funciones del segundo grupo se analizan no en todo el dominio de existencia, se logra fre-
cuentemente elegir tal dominio de denicin (una parte del dominio de existencia) que la funcin
aplicar dicho dominio de denicin sobre el correspondiente codominio ya de manera biunvo-
ca. Cabe indicar que cualquier funcin y = f(x) en aquella parte del dominio de denicin D
perteneciente al dominio de existencia de la funcin, donde ella es estrictamente montona, es
decir, creciente o decreciente, pertenece al primer grupo. Supongamos que el dominio de la funcin
y = f(x) es tal que la funcin realiza una aplicacin biunvoca del dominio D sobre el codominio C.
Entonces, a partir de cualquier y, perteneciente al codominio C, se puede establecer unvocamente
el valor de x de dominio D, procediendo de la manera siguiente: en la igualdad f(x) y = 0 se
considera jo cualquier y C y se busca x D que satisfaga la igualdad citada. Cada x D
encontrado se denota con f
1
(y). La igualdad x = f
1
(y) lleva el nombre de regla inversa.
Denicin 8.10 Funcin inversa
Se denomina funcin inversa de la funcin y = f(x), x D, y C, aquella que se obtiene a partir
de la regla inversa x = f
1
(y), sustituyendo x por y, e y por x con la sustitucin simultnea del
dominio por el codominio y del codominio por el dominio.
Realizada la sustitucin mencionada, el codominio de la funcin y = f(x) se convierte en el
dominio de la funcin inversa y = f
1
(x), mientras que el dominio de la funcin y = f(x) se
hace el codominio de la funcin inversa y = f
1
(x). As pues, dos funciones, a saber y = f(x)
con el dominio D y el codominio C, y la funcin y = f
1
(x) con C y D que intervienen como el
dominio y el codominio, respectivamente, donde f(f
1
(x)) = x para todo x C, y f
1
(f(x)) = x
CAPTULO 8. FUNCIONES ALGEBRAICAS 409
para todo x D, son tales que una de ellas es inversa de la otra. No siempre se logra encontrar
para cada funcin tal dominio, que se aplique por ella de manera biunvoca sobre el codominio
correspondiente.
Para que una funcin f tenga asociada una funcin g con las caractersticas anteriormente
mencionadas, es necesario que tenga una propiedad importante: no puede enviar a dos elementos
diferentes de su dominio a la misma imagen. Es decir, no pueden haber x
1
y x
2
en el dominio de
f tales que f(x
1
) = f(x
2
). As pues, debemos considerar funciones f : I R R que tengan la
propiedad de los elementos de su dominio nunca comparten imgenes.
Denicin 8.11 Funcin inyectiva
Una funcin uno a uno es una funcin en la que a cada elemento en el codominio slo se corre-
sponde con un elemento en el dominio. Ms precisamente, tales que:
x
1
, x
2
I, x
1
,= x
2
f(x
1
) ,= f(x
2
).
Una funcin que tiene esta propiedad tambin se le conoce con el nombre de inyectiva.
Debemos observar que la grca de una funcin inyectiva tiene una propiedad importante: Si
trazamos cualquier recta horizontal, sta puede cortar la grca de la funcin cuando mucho en un
punto. En efecto, tal recta horizontal es del tipo y = y
0
(constante), de modo que si cruza la grca
de la funcin es en un punto cuya abscisa x tiene por imagen justamente a y
0
. Siendo inyectiva la
funcin, solamente puede haber un valor de x que tenga por imagen a y
0
.
Sea entonces f : I R R una funcin inyectiva. La funcin g que deshace las imgenes de f
se llama funcin inversa de f, y se denota con f
1
. Observe que esta ltima funcin tendr como
dominio las imgenes de f. Es decir, el dominio de f
1
es el rango de f. Y recprocamente, el
rango de f
1
es el dominio de f. La funcin f
1
tiene entonces la propiedad fundamental:
(f
1
f)(x) = f
1
(f(x)) = x para toda x del dominio de f
(f f
1
)(y) = f(f
1
(y)) = y para toda y en el dominio de f
1
Pensemos cmo podemos hacer para encontrar la inversa de una funcin inyectiva dada y =
f(x). Si en esta frmula se nos est diciendo qu es lo que hace f con x para obtener su imagen
y = f(x), y nosotros buscamos una funcin f
1
que a y la regrese a x, lo que debemos bus-
car es qu debemos hacer con la y para obtener nuevamente el valor x. As que una buena idea
CAPTULO 8. FUNCIONES ALGEBRAICAS 410
para conseguir esta funcin es, de la frmula y = f(x), despejar a x en trminos de y. La frmu-
la en la que se muestra x en trminos de y es la expresin x = f
1
(y) de la funcin inversa buscada.
Algunas funciones importantes no son inyectivas,
de modo que de ellas no podemos obtener una in-
versa. Sin embargo, es comn que se restrinja el do-
minio de la funcin para que sta quede inyectiva, y
entonces podamos denir su inversa. Una propiedad
interesante que tienen las grcas de una funcin in-
yectiva y de su inversa es la siguiente: supongamos
que (a, b) es un punto de la grca de la funcin
y = f(x). Esto signica que b = f(a). La funcin
inversa f
1
debe ser tal que f
1
(b) = a, lo cual sig-
nica que el punto (b, a) es un punto de la grca de
f
1
. As entonces, si (a, b) es un punto de la grca
de y = f(x), entonces (b, a) debe ser un punto de la
grca de y = f
1
(x). Observe que los puntos (a, b)
y (b, a) estn situados simtricamente respecto de la
recta y = x.
Podemos concluir entonces que la grca de la funcin y = f
1
(x) es un reejo, respecto de la
recta y = x, de la grca de la funcin y = f(x).
Ejemplo 8.21 Dada f : R R, encuentre la inversa:
a) f(x) = 2 +x x
2
; b) f(x) =
x
2
+ 3x
x
2
5x
; c) f(x) = x

x
2
1, x (; 1].
Solucin
a) Completando cuadrados, obtenemos
y =
9
4

_
x
1
2
_
2
Con y <
9
4
hay dos valores diferentes del argumento, es decir, corta el grco de la funcin en dos
puntos. Esto signica que la funcin f denida para todo R es no invertible.
b) Para probar que la funcin es inyectiva, hacemos:
f(x
1
) = f(x
2
)
x
2
1
+ 3x
1
x
2
1
5x
1
=
x
2
2
+ 3x
2
x
2
2
5x
2
(x
2
1
+ 3x
1
)(x
2
2
5x
2
) = (x
2
2
+ 3x
2
)(x
2
1
5x
1
) x
1
= x
2
Por tanto la funcin es inyectiva.
Como y =
x
2
+3x
x
2
5x
, entonces x =
5y+3
y1
, reemplazamos esta nueva expresin en la original y obtenemos
y =
_
5y + 3
y 1
_
2
+ 3
_
5y + 3
y 1
_
_
5y + 3
y 1
_
2
5
_
5y + 3
y 1
_
=
(5y + 3)
2
+ 3(5y + 3)(y 1)
(y 1)
2
(5y + 3)
2
5(5y + 3)(y 1)
(y 1)
2
y =
(5y + 3)(5y + 3 + 3y 3)
(5y + 3)(5y + 3 5y + 5)
=
8y
8
= y
CAPTULO 8. FUNCIONES ALGEBRAICAS 411
De esta manera probamos que la funcin es sobreyectiva. Como la funcin es biyectiva, entonces
la funcin inversa est dada por
g(x) =
5x + 3
x 1
c) Probamos que la funcin es inyectiva:
f(x
1
) = f(x
2
) x
1

_
x
2
1
1 = x
2

_
x
2
2
1
(x
1
x
2
)
2
=
_
_
x
2
1
1
_
x
2
2
1
_
2
1 x
1
x
2
=
_
(x
2
1
1)(x
2
2
1)
(x
1
x
2
)
2
= 0; x
1
= x
2
Por tanto la funcin es inyectiva.
Como y = x

x
2
1, entonces x =
1+y
2
2y
, reemplazamos esta nueva expresin en la original y
obtenemos
y =
1 +y
2
2y

_
1 +y
2
2y
_
2
1 =
1 +y
2
2y

(1 +y
2
)
2
4y
2
4y
2
y =
1 +y
2
2y

(1 y
2
)
2
4y
2
=
1 +y
2
2y

1 y
2
2y
y = y
De esta manera probamos que la funcin es sobreyectiva. Como la funcin es biyectiva, entonces
la funcin inversa est dada por
g(x) =
1 +x
2
2x
8.4. Paridad de una funcin
Denicin 8.12 Funcin par
La funcin y = f(x) se denomina par, si el dominio es un conjunto simtrico respecto de las
ordenadas y si f(x) = f(x) para cualquier x D.
De cualquier funcin par y = f(x), con dominio D, se dice que es simtrica respecto del eje de
ordenadas, puesto que, cualquiera que sea x D, los puntos del plano (x, f(x)) y (x, f(x)) son
simtricos con relacin al eje de ordenadas.
Denicin 8.13 Funcin impar
La funcin y = f(x) se denomina impar, si el dominio D es un conjunto simtrico respecto del
origen de coordenadas y si f(x) = f(x) para cualquier x D.
De cualquier funcin impar y = f(x), que dispone del dominio D, se dice que es simtrica
respecto del origen de coordenadas, puesto que, cualquiera que sea x D, los puntos del plano
(x, f(x)) y (x, f(x)) son simtricos con relacin al origen de coordenadas. A la par con las
funciones pares e impares existen tambin funciones que no son ni unas ni otras.
Teorema 8.7 Toda funcin denida en un conjunto D, simtrico respecto del origen de coorde-
nadas, puede ser representada en forma de la suma de dos funciones, cada una de las cuales est
denida en el mismo conjunto D, y una de las cuales es par y la otra, impar.
CAPTULO 8. FUNCIONES ALGEBRAICAS 412
A la par con el concepto de funcin par, es decir, de funcin simtrica respecto del eje de
ordenadas, se puede introducir una notacin ms general de una funcin, simtrica respecto de una
recta vertical que pasa por el punto (a, 0). Suele decirse que el conjunto D es simtrico respecto
del punto (a, 0), si dicho conjunto es tal que el punto 2a x D para cualquier x D.
Denicin 8.14 Funcin simtrica respecto a una recta
Una funcin y = f(x) es simtrica respecto de la recta vertical que pasa por el punto de coordenadas
(a, 0), si el dominio es un conjunto simtrico respecto del punto (a, 0) y si para todo x perteneciente
al dominio se verica que f(2a x) = f(x).
La grca de una funcin par es obviamente, simtrica respecto del eje de ordenadas, y la
grca de una funcin impar es simtrica respecto del origen de coordenadas. Para construir la
grca de una funcin impar es suciente construirla para x 0. Para x < 0, la grca resulta una
representacin simtrica de la parte de la grca construida respecto al origen de coordenadas.
El producto de dos funciones pares o de dos funciones impares ser una funcin par, y el
producto de una funcin par por otra impar ser una funcin impar. Desde luego, la mayora de
las funciones son no pares y no impares.
Ejemplo 8.22 Determinar la paridad de la funcin:
a) f(x) = 6x
2
+ 8 + (x 2)
2
; b) f(x) = [x + 1[ +[x 1[;
c) f(x) = [10 x[ [10 +x[.
Solucin
a) f(x) = 6(x)
2
+ 8 + (x 2)
2
= 6x
2
+ 8 (x + 2)
2
,= f(x).
La funcin no es par ni impar.
b) f(x) = [ x + 1[ +[ x 1[ = [ (x 1)[ +[ x(x + 1)[ = [x 1[ +[x + 1[ = f(x).
La funcin es par.
c) f(x) = [10 (x)[ [10 + (x)[ = [10 +x[ [10 x[ = ([10 x[ [10 +x[) = f(x).
La funcin es impar.
8.5. Tarea
1. Demuestre que el producto de dos funciones pares o dos impares es una funcin par, mien-
tras que el producto de una funcin par por una impar es una funcin impar.
CAPTULO 8. FUNCIONES ALGEBRAICAS 413
2. Demuestre que toda funcin denida sobre un conjunto simtrico al origen de coordenadas
es representable en forma de la suma de funciones par e impar.
3. La funcin f es ni par ni impar, la g es par, la h es impar. Puede la suma:
a) f +g se par; b) f +g ser impar; c) f +h ser par; d) f +h ser impar.
4. La funcin f es ni par ni impar, la g es par, la h es impar y tiene sentido la composicin
de cualesquiera dos de estas funciones. Indique todas las composiciones que son:
a) funciones pares; b) funciones impares.
5. Escriba la funcin f(x) = x
3
+ 2x
2
+ 3x 4 como la suma de una funcin par ms una
funcin impar.
6. Demuestre que si f y g son funciones pares, entonces su suma h(x) = f(x) + g(x) es una
funcin par.
7. Demuestre que si f y g son funciones impares, entonces su suma h(x) = f(x) +g(x) es una
funcin impar.
8. Demuestre que si f y g son funciones pares, entonces su producto h(x) = f(x)g(x) es una
funcin par.
9. Demuestre que si f y g son funciones impares, entonces su producto h(x) = f(x)g(x) es
una funcin par.
10. Demuestre que si f es una funcin par y g es una funcin impar, entonces su producto
h(x) = f(x)g(x) es una funcin impar.
11. Demuestre que si f y g son funciones pares, entonces su cociente h(x) =
_
f
g
_
(x) es una
funcin par.
12. Demuestre que si f y g son funciones impares, entonces su cociente h(x) =
_
f
g
_
(x) es una
funcin par.
13. Represente la funcin f en forma de la suma de funciones par e impar:
a) f(x) = (x + 1)
2
; b) f(x) =
x 3
x
4
; c) f(x) =
1
x 1
, [x[ < 1;
d) f(x) = [x 1[.
14. f : I R R una funcin cualquiera, denida en el conjunto I de R, simtrico respecto
del origen:
CAPTULO 8. FUNCIONES ALGEBRAICAS 414
a) Demuestre que la funcin g(x) =
1
2
[f(x) +f(x)] es par.
b) Demuestre que la funcin g(x) =
1
2
[f(x) f(x)] es impar.
c) Verique que f(x) = g(x) +h(x). Concluya que toda funcin se puede escribir como la
suma de una funcin par ms una funcin impar.
8.6. Monotona de una funcin
En la presente seccin estudiaremos una serie de propiedades importantes de las funciones
continuas y que encuentran muchas aplicaciones.
Denicin 8.15 Funcin continua
La funcin f : D R, D R se llama continua sobre el conjunto D, si es continua por D en
cada uno de sus puntos.
Una clase importante de funciones continuas es la clase de funciones continuas sobre los inter-
valos del eje numrico. Comencemos el estudio por las funciones continuas sobre los segmentos.
Si la funcin f es continua sobre el segmento [a; b] entonces su continuidad en el punto x = a es
equivalente a la continuidad por la derecha y su continuidad en el punto x = b, a la continuidad
por la izquierda en este punto.
Denicin 8.16 Funcin acotada superiormente
La funcin f recibe el nombre de acotada superiormente sobre el conjunto D(f) si existe un
nmero C tal que para cualquiera x es cierta la desigualdad f(x) C. Simblicamente esta
denicin la podemos escribir de la siguiente forma:
C x [(x ) (f(x) C)]
Anlogamente, la funcin f es acotada inferiormente sobre el conjunto
D(f)si : C x [(x ) (f(x) C)]
La funcin acotada tanto superior como inferiormente sobre el conjunto lleva el nombre de
acotada sobre el conjunto . Esta denicin es equivalente a la siguiente:
Denicin 8.17 Funcin acotada
La funcin f es acotada sobre el conjunto D(f) si existe un nmero C > 0 tal que para
cualquier x es cierta la desigualdad [f(x)[ C; para abreviar:
C > 0 x [(x ) ([f(x)[ C)]
Si en estas deniciones = D(f), la funcin se denomina superiormente acotada, inferiormente
acotada, acotada, respectivamente.
Teorema 8.8 Weierstrass
Cualquier funcin continua sobre un segmento est acotada y alcanza sobre l su cota superior y
su cota inferior.
CAPTULO 8. FUNCIONES ALGEBRAICAS 415
Ejemplo 8.23 Demuestre que la funcin
f(x) =
x
x
2
+ 1
, x R
es acotada.
Solucin
De la desigualdad para la madia proporcional y la aritmtica se desprende que
[x[
x
2
+ 1
2
De aqu obtenemos

x
x
2
+ 1

=
[x[
x
2
+ 1

1
2
Para toda x R, es decir, se verica con C =
1
2
y, por lo tanto, la funcin dada es acotada. c _
Ejemplo 8.24 Demuestre que la funcin
f(x) =
1
x
2
, x R, x ,= 0
Es no acotada.
Solucin
Sea C un nmero arbitrario positivo. La desigualdad
1
x
2
> C es equivalente a la desigualdad
[x[ <
1

C
con x ,= 0. Si tomamos x =
1
2

C
obtenemos que
1
x
2
= 4C > C, lo que en correspondencia
con la denicin, precisamente, signique que la funcin dada es no acotada. c _
Denicin 8.18 Mximos y mnimos
Supongamos que una funcin y = f(x) est denida en el conjunto D. Si existe tal k D, que
para cualquier x D se verica la desigualdad f(x
1,2
) f(k), se dice que la funcin y = f(x),
denida en el conjunto D, toma para x = k, el valor mnimo r = f(k).
Supongamos que una funcin y = f(x) est denida en el conjunto D. Si existe tal k D, que
para cualquier x D se verica la desigualdad f(x
1,2
) f(k), se dice que la funcin y = f(x),
denida en el conjunto D, toma para x = k, el valor mximo r = f(k).
Ejemplo 8.25 Encuentre el radio de la base y la altura del cilindro inscrito en una esfera de
radio R, si el rea de la supercie lateral del cilindro tiene el valor mximo de los posibles.
Solucin
Por el teorema de Pitgoras tenemos
R
2
=
_
h
2
_
2
+r
2
h = 2
_
R
2
r
2
El rea del cilindro est dada por A = 2rh. Reemplazamos h en la ecuacin del rea del cilindro
A(r) = 4r
_
R
2
r
2
Para encontrar las dimensiones del cilindro, hacemos A(r) = A(k):
4r
_
R
2
r
2
= 4k
_
R
2
k
2
r
2
(R
2
r
2
) = k
2
(R
2
k
2
) = 0
(r
2
k
2
)(R
2
r
2
k
2
) = 0 k =
R

2
CAPTULO 8. FUNCIONES ALGEBRAICAS 416
De esta manera obtenemos que el radio del cilindro es r =
R

2
. Reemplazamos r en la ecuacin h,
obtenemos la altura del cilindro
h = 2
_
R
2

R
2
2
=

2R.
por tanto r =
R

2
y h =

2R dan el rea mxima.


Ejemplo 8.26 Se desea construir un pequeo recipiente cilndrico sin tapa que tenga un volu-
men de 24 centmetros cbicos. El material que se usa para la base cuesta tres veces ms que el que
se emplea para la parte cilndrica. Suponiendo que en la construccin no se desperdicia material,
evaluar las dimensiones para las que es mnimo el costo del material de fabricacin.
Solucin
El volumen del cilindro est dado por
V = r
2
h 24 = r
2
h h =
24
r
2
El costo total de fabricar el recipiente es
C(r) = A
1
p + 3A
2
p C(r) = 2rhp + 3r
2
p
Reemplazamos h en C(r)
C(r) = 2r
24
r
2
p + 3r
2
p C(r) =
48p
r
+ 3r
2
p C(r) =
3p
r
(16 +r
3
)
Para encontrar las dimensiones del recipiente, hacemos C(r) = C(k):
3p
r
(16 r
3
) =
3p
k
(16 +k
3
) 16(r k) rk(r k)(r +k) = 0
(r k)(16 r
2
k rk
2
) = 0 k = 2
Las dimensiones para las que el costo del material sea mnimo sern, el radio r = 2 centmetros y
la altura h = 6 centmetros.
Examinemos la curva representada en la gura (a). Trazando una tangente AB, por ejem-
plo, vemos que los puntos de la curva contiguos al punto de tangencia A y situados a ambos
lados del mismo se hallan ms abajo que la tangente. En este caso se dice que la curva tiene
convexidad en el punto A; si se verica esta condicin para la parte de la curva comprendida entre
CAPTULO 8. FUNCIONES ALGEBRAICAS 417
los puntos M y N, esta parte recibe el nombre de convexa.
Tomemos la curva representada en la gura (b). Aqu observamos otro fenmeno, a saber: los
puntos de la curva prximos al punto de tangencia C y situados a ambos lados del mismo se hallan
ms arriba que la tangente CD. En este caso se dice que la curva tiene concavidad en el punto
C, y se llama cncava la parte de la curva comprendida entre los puntos P y Q, que satisface esta
condicin.
Existen casos en que una parte de la curva es
convexa y otra, cncava; por ejemplo, la gura (c)
presenta convexidad (encima del eje 0X) y con-
vexidad (debajo del eje 0X); adems, el punto A
sirve de frontera entre ellas. La tangente trazada
a la curva en este punto es comn para la parte
convexa y la cncava. Al mismo tiempo, esta tan-
gente corta la curva en el punto de tangencia; por
este motivo, la grca no es convexa ni cncava en el punto A. Este punto recibe el nombre de
punto de inexin.
Denicin 8.19 Punto de inexin
Supongamos que una funcin y = f(x) est denida en el conjunto D. Si existe tal k D, que
para cualquier x D alcanza la inexin de la funcin, si existe un cambio en el sentido de la
concavidad de la curva. Es decir hay un cambio de concavo a convexo o de convexo a concavo.
El tmino concavidad y convexidad, quiere decir que la graca de la funcin tiene direcciones
diferentes de convexidad a la izquierda y a la derecha del punto k.
En el intervalo (a; b) la grca de la funcin f es concava, si la grca de esta funcin se
encuentra no por debajo de cualquiera de sus tangentes entre los lmtes de dicho intervalo. Es decir
f(x) >
f(b) f(a)
b a
(x a) +f(a) concavo
En el intervalo (a; b) la grca de la funcin f es convexa, si la grca de esta funcin se encuentra
no por encima de cualquiera de sus tangentes entre los lmtes de dicho intervalo. Es decir
f(x) <
f(b) f(a)
b a
(x a) +f(a) convexo
CAPTULO 8. FUNCIONES ALGEBRAICAS 418
La grca de una funcin puede tener varios puntos de mximo, uno de los cuales puede estar
ms alto que todos los dems. Con objeto de distinguir dicho punto de entre los dems se le llama
mximo absoluto, mientras que a cada uno de los otros se le denomina mximo relativo.
Denicin 8.20 Mximo relativo
Se dice que la funcin f(x) tiene un mximo relativo en k si existe un intervalo con k en su
interior, tal que ese intervalo f(k) es el valor mximo de la funcin.
La expresin funcin creciente o decreciente, signica obviamente lo que dice, pero debemos
establecer con precisin dnde crece o decrece la funcin.
Denicin 8.21 Funcin creciente y decreciente
Una funcin y = f(x), denida en el conjunto D, se denomina creciente en este conjunto, si
para cualquier par de nmeros x
1
y x
2
de dicho conjunto de la desigualdad x
1
< x
2
proviene que
f(x
1
) < f(x
2
). Una funcin y = f(x), denida en el conjunto D, se denomina decreciente en este
conjunto, si para cualquier par de nmeros x
1
y x
2
de dicho conjunto, de la desigualdad x
1
< x
2
se deduce que f(x
1
) > f(x
2
).
Denicin 8.22 Funcin no creciente y no decreciente
Una funcin y = f(x), denida en el conjunto D, se denomina no decreciente en este conjunto, si
para cualquier par de nmeros x
1
y x
2
pertenecientes a dicho conjunto, de la desigualdad x
1
< x
2
se deduce que f(x
1
) f(x
2
). Una funcin y = f(x), denida en el conjunto D, se denomina
no creciente en este conjunto, si para cualquier par de nmeros x
1
y x
2
, pertenecientes a dicho
conjunto, de la desigualdad x
1
< x
2
se deduce que f(x
1
) f(x
2
).
Las funciones crecientes, decrecientes, no crecientes y no decrecientes llevan el nombre de fun-
ciones montonas. Las funciones crecientes y decrecientes se llaman estrictamente montonas.
Ejemplo 8.27 Dada la funcin f : R R denida explcitamente como
f(x) =
x 1
x
2
+ 1
determine los intervalos de monotona.
Solucin
El dominio de esta funcin son todos los reales. Para encontrar los puntos de mximos y mnimos,
hacemos f(x) = f(k):
x 1
x
2
+ 1
=
k 1
k
2
+ 1

x 1
x
2
+ 1

k 1
k
2
+ 1
= 0

(x k)[x(k 1) k 1]
(x
2
+ 1)(k
2
+ 1)
= 0
k
2
2k 1
(k
2
+ 1)
2
= 0
Resolviendo esta ecuacin, obtenemos: k
1
= 1

2 y k
2
= 1 +

2. Haciendo el anlisis correspondi-


ente, establecemos que k
1
es punto de mnimo y k
2
es punto de mximo. Para establecer los intervalos de
monotona, procedemos de la siguiente manera: El intervalo (; k
1
) es decreciente, (k
1
; k
2
) es
creciente y (k
2
; +) es estrictamente decreciente.
CAPTULO 8. FUNCIONES ALGEBRAICAS 419
Ejemplo 8.28 Dada la funcin f : R R denida explcitamente como
f(x) =
x
3
(x + 1)
2
determine los intervalos de monotona.
Solucin
El dominio de esta funcin son todos los x ,= 1. Para encontrar los puntos de mximos y mnimos,
hacemos f(x) = f(k):
x
3
(x + 1)
2
=
k
3
(k + 1)
2

x
3
(x + 1)
2

k
3
(k + 1)
2
= 0
(x k)[x
2
(k
2
+ 2k + 1) +x(2k
2
+k) +k
2
]
(k + 1)
2
(x + 1)
2
= 0
_
_
_
x k = 0
x
2
(k
2
+ 2k + 1) +x(2k
2
+k) +k
2
(k + 1)
2
(x + 1)
2
= 0

_
_
_
x = k
k
2
(k + 3)
(k + 1)
3
= 0
Resolviendo esta ecuacin, obtenemos: k
1
= 3 y
k
2
= 0. Haciendo el anlisis correspondiente, estale-
cemos que k
1
es punto de mximo y k
2
es punto de
inexin. Para establecer los intervalos de monotona,
procedemos de la siguiente manera: El intervalo (; k
1
) es creciente, (k
1
; 1) es estrictamente
decreciente y (1; +) es estrictamente creciente.
8.7. Tarea
1. Demuestre que la funcin cuadrtica f(x) = ax
2
+bx +c:
a) Con a > 0 estrictamente decrece sobre (; b/2a] y estrictamente crece sobre
[b/2a; +);
b) Con a < 0 estrictamente crece sobre (; b/2a] y estrictamente decrece sobre
[b/2a; +).
2. Demuestre que la funcin f(x) = x
3
+x crece.
3. Demuestre que la funcin f(x) =
1x
2
x
decrece en cualquier intervalo que no contiene cero.
4. Demuestre que la funcin f(x) =
1+x
2
x
:
a) Estrictamente crece sobre (; 1] y sobre [1; +);
b) Estrictamente decrece sobre [1; 0) y sobre (0; 1].
5. Encuentre los mximos intervalos sobre los que la funcin f(x) = x
4
2x
2
2:
a) Crece; b) Decrece.
CAPTULO 8. FUNCIONES ALGEBRAICAS 420
6. Demuestre que la funcin f(x) = x
3
+x
2
:
a) Crece sobre (0; +); b) No es montona sobre [1; 0].
7. Encuentre la distancia desde la parbola f(x) =
x
2
4
hasta la recta y = x 2.
8. Demuestre que la funcin y = x
3
3a
2
x crece en los intervalos (; a] y [a; +) y
decrece en [a; a], (a > 0).
9. Demuestre que la funcin y = x
3
3bx
2
(b > 0) crece en (; 0] y [2b; +) y decrece
en [0; 2b].
10. La velocidad de la sangre que est a r centmetros del eje central de una arteria de radio
R es S(r) = c(R
2
r
2
), donde c es una constante positiva. Dnde es mayor la velocidad de
la sangre?
11. Un punto luminoso est situado en la lnea de los centros de dos esferas y se encuentra
fuera de ellas. Con qu posicin del punto luminoso ser mxima la suma de las reas de
las partes iluminadas de las supercies de las esferas?
12. Un embudo cnico, de radio de base R y altura H est lleno de agua. Una esfera pesada
est sumergida en el embudo. Cul ha de ser el radio de la esfera para que el volumen de
agua expulsada del embudo por la parte sumergida de la esfera, sea el mayor posible?
13. Dos cuerpos se mueven por rectas en el sentido hacia su punto de interseccin A. Las ve-
locidades de los cuerpos son constantes e iguales a v
1
y v
2
, en el momento inicial los cuerpos
se hallan a las distancias a y b del punto A, respectivamente. El ngulo entre las direcciones
de movimiento de los cuerpos es igual a . Encuentre la distancia mnima entre ellos.
14. Se desea construir un almacn con un volumen de 100 metros cbicos que tenga techo
plano y base rectangular cuya anchura sea tres cuartas partes de su longitud. El costo por
metro cbico de los materiales es de 36 dlares para el piso, 54 dlares para los lados y 27
dlares para el techo. Qu dimensiones minimizan el costo?
15. Una pista de 400 metros de longitud est formada por dos semicrculos iguales y dos partes
rectas tambin iguales Cules son las dimensiones de la pista que encierra la mayor rea?
La pista encierra tres reas, un rectngulo y dos semicrculos. Cules son las dimensiones
de la pista que encierra el rectngulo de mayor rea?
16. Hallar el rea total mxima de un cilindro inscrito en una esfera de radio R.
17. Una pila elctrica que tiene un voltaje jo V y una resistencia interna ja r se conecta a
un circuito que tiene resistencia variable R. Por la ley de Ohm, la corriente I en el circuito
es I =
V
R+r
. La potencia de salida P est dada por P = I
2
R. Demuestre que la potencia
CAPTULO 8. FUNCIONES ALGEBRAICAS 421
mxima se alcanza cuando R = r.
18. A qu altura sobre el centro de una mesa redonda de radio R hay que situar una bombilla
elctrica para que la iluminacin del borde de la mesa sea la mxima?
19. En un tringulo est inscrito un rectngulo de forma que uno de sus lados yace en uno
de los lados del tringulo y dos vrtices, en otros dos. Encuentre el rea mxima posible del
rectngulo si la del tringulo es igual a A.
20. A las 13:00 horas el barco A se encuentra 30 millas al sur del barco B y viaja hacia el norte
a 15 millas por hora. El barco B navega hacia el oeste a 10 millas por hora. A qu hora se
alcanza la distancia mnima entre las dos embarcaciones?
21. Una parbola tiene su vrtice situado sobre una circunferencia de radio R, y el eje de la
parbola sigue la direccin del dimetro. Cul ha de ser el parmetro de la parbola para
que el rea del segmento limitado por la parbola y la cuerda comn para sta y la circunfer-
encia, sea la mayor posible? El rea del segmento parablico simtrico es igual a dos tercios
del producto de su base por la altura.
22. Se desea construir un recipiente cilndrico de metal sin tapa que tenga la capacidad de 1
metro cbico. Encuentre las dimensiones que debe tener para que la cantidad de material sea
mnima, suponiendo que no se desperdicia nada en la construccin. La base circular del recipi-
ente se corta de una hoja cuadrada y el metal restante se desperdicia. Calcule las dimensiones
del recipiente para las cuales la cantidad de material necesario en la construccin sea mnima.
23. Dado un cierto punto A en una circunferencia, trazar una cuerda BC paralela a la tangente
en el punto A de modo que el rea del tringulo ABC sea la mayor posible.
24. Encuentre el radio de la base y la altura de un cono circunscrito a una esfera si el volumen
del cono tiene el valor mnimo de los posibles y el radio de la esfera es igual a R.
25. Una carretera que va de norte a sur y otra que va de este a oeste se cruzan en un punto P.
Un vehculo que viaja hacia el este a 20 kilmetros por hora, pasa por P a las 10:00 horas.
En ese mismo momento un automvil que viaja hacia el sur a 50 kilmetros por hora se
encuentra 2 kilmetros al norte de P. Calcular cuando se encuentran los dos vehculos ms
cerca uno del otro y cul es la distancia mnima entre ellos.
26. Un hombre que navega en una barca de remos a 2 millas del punto ms cercano de una
costa recta, desea llegar a su casa, la cual est en la citada costa a 6 millas de dicho punto. El
hombre puede remar a razn de 3 millas por hora y caminar a 5 millas por hora. Qu debe
hacer para llegar a su casa en el menor tiempo posible? Si el hombre tiene una lancha a motor
que puede viajar a 15 millas por hora, qu debe hacer para llegar en el menor tiempo posible?
CAPTULO 8. FUNCIONES ALGEBRAICAS 422
27. El volumen de un prisma triangular regular es igual a V . Cunto debe medir el lado de
la base para que su supercie total sea la menor posible?
28. Hallar la relacin entre el radio R y la altura H de un cilindro que tiene la menor supercie
total posible, conociendo su volumen.
29. El permetro de un tringulo issceles es 2p. Cunto deben medir sus lados para que el
volumen del cuerpo engendrado por la rotacin del tringulo en torno a su base sea el mayor
posible.
30. El permetro de un tringulo issceles es 2p. Cunto deben medir sus lados para que el
volumen del cono engendrado por la rotacin del tringulo en torno a su altura bajada sobre
la base sea el mayor posible?
31. Un torpedero est anclado a 9 km del punto ms prximo de la orilla. Se necesita enviar
a un mensajero al campamento situado en la orilla. La distancia entre ste y el punto ms
prximo referido, es igual a 15 km. Teniendo en cuenta que el mensajero recorre a pie 5 km/h,
y en una barca, remando, 4 km/h, en qu punto de orilla debe desembarcar para llegar al
campamento lo ms pronto posible.
32. Se va a construir una armazn para embalaje con un trozo de madera con seccin cuadrada
de 2 por 2 pulgadas y 24 pie de largo. El embalaje va a tener extremos cuadrados. Calcule
las dimensiones que producen el mximo volumen exterior.
33. Tres puntos A, B y C se hallan situados de modo que < (ABC) = /3. Un automvil
sale del punto A, en el mismo momento del punto B parte un tren. El auto avanza hacia el
punto B a 80 kilmetros por hora, el tren se dirige hacia el punto C a 50 kilmetros por hora.
Teniendo en cuenta que la distancia AB = 200 kilmetros, en qu momento, al comenzar el
movimiento, ser mnima la distancia entre el automvil y el tren?
34. Un veterinario cuenta con 30 metros de malla de metal y quiere construir 6 jaulas para
perros levantando primero una cerca alrededor de una regin rectangular, y dividiendo luego
la regin en seis rectngulos iguales mediante cinco rejas paralelas a uno de los lados. Cules
son las dimensiones de la zona rectangular para las que el rea total es mxima?
35. Se tender un cable desde una central elctrica situada al lado de un ro de 900 metros de
ancho hasta una fbrica en el otro lado, 3000 metros ro abajo. El costo de tender el cable
bajo el agua es $ 5 por metro, y el costo sobre tierra es $ 4 por metro. Cul es la ruta ms
econmica sobre la cual tender el cable?
36. Una banda de hierro, de anchura a, ha de ser encorvada de modo que tome la forma de
canaln cilndrico abierto (la seccin del canaln ha de semejarse a un arco de segmento
circular). Cul ha de ser la abertura del ngulo central que se apoya en este arco para que
la capacidad del canaln sea la mayor posible?
CAPTULO 8. FUNCIONES ALGEBRAICAS 423
37. Hallar el cilindro con el volumen mximo entre todos los cilindros inscritos en un cubo con
arista a, de forma que el eje de cada cilindro coincida con la diagonal del cubo, en tanto que
las circunferencias de las bases hagan contacto con las caras del cubo.
38. Trazar una recta de modo que pase por un punto dado P(1, 4) y que la suma de las longi-
tudes de los segmentos positivos cortados por dicha recta en los ejes de coordenadas, sea la
menor posible.
39. Cul ha de ser la abertura del ngulo en el vrtice de un tringulo issceles, de rea dada,
para que el radio de un crculo inscrito en dicho tringulo sea el mayor posible?
40. Sean dados dos puntos A(1, 4) y B(3, 0) en la elipse 2x
2
+y
2
= 18. Hallar el tercer punto
C tal que el rea del tringulo ABC sea la mayor posible.
41. Dados los dos puntos F
1
(1, 0) y F
2
(1, 0), y el crculo C = (x, y)/x
2
+y
2
= 4:
a) Encuntrense los puntos P sobre C donde PF
1
+PF
2
es un mnimo.
b) Hllense los puntos P sobre C tales que PF
1
+PF
2
es un mximo.
42. Una cerca de 8 pie de alto al nivel del suelo va paralela a un edicio alto. La cerca dista
1 pie del edicio. Calcule la longitud de la escalera ms corta que se puede apoyar entre el
suelo y el edicio por encima de la reja.
43. Se desea construir un tanque de acero con la forma de un cilindro circular recto y semies-
feras en los extremos para almacenar gas propano. El costo por pie cuadrado de los extremos
es el doble del de la parte cilndrica. Qu dimensiones minimizan el costo si la capacidad
deseada es de 10 pie cbicos?
44. Un recipiente con pared vertical de altura h se encuentra sobre un plano horizontal. De
un oricio en la pared del recipiente uye un chorro. Determine la posicin del oricio con la
que el alcance del chorro ser el mximo si la velocidad del lquido que uye es igual a

2gx,
donde x es la profundidad del oricio (ley de Torricelli).
45. La fbrica A debe unirse mediante una carretera con la lnea frrea rectilnea en la que se
encuentra el poblado B. La distancia AC desde la fbrica hasta el ferrocarril es igual a a,
en tanto que la distancia BC por el ferrocarril es igual a b. El costo del transporte de las
mercancas por la carretera es k veces (k > 1) mayor que por el ferrocarril. En qu punto D
del segmento BC hay que trazar la carretera desde la fbrica para que el costo del transporte
de las mercancas desde la fbrica A hasta el poblado B sea el mnimo?
46. La cubierta de un escritorio de longitud L se est deslizando de costado alrededor de la
esquina rectangular de un pasillo de una habitacin de ancho x a otra de ancho y. Cul es
el ancho mnimo y para el que es posible esta maniobra?
CAPTULO 8. FUNCIONES ALGEBRAICAS 424
47. Se han de fabricar envases cilndricos de hojalata de volumen prejado. No se desperdicia
material al cortar la hoja que constituye la pared cilndrica, pero las bases se forman con tro-
zos cuadrados, desperdicindose los recortes. Hllese la relacin entre la altura y el dimetro
de los envases, de manera que resulten lo ms econmicos posible.
48. Se desea que las pginas de un libro tengan un rea de 900 centmetros cuadrados con
mrgenes de 2,5 centmetros abajo y a los lados, y de 1,5 centmetros arriba. Determine las
dimensiones de la pgina que darn la mayor rea posible para el texto.
49. Se desea construir un oleoducto de un punto A a otro punto B que distan 10 kilmetros
y se encuentran en riberas opuestas de un ro de cauce recto de 1 kilmetro de ancho. El
oleoducto ir bajo el agua de A a un punto C en la ribera opuesta y luego sobre el suelo de
C a B. El costo por kilmetro de tubera bajo el agua es cuatro veces ms del costo sobre
tierra. Calcule la posicin de C que minimizar el costo. Desprecie la pendiente del lecho del
ro.
50. Determine las dimensiones del rectngulo que se puede inscribir en un semicrculo de radio
r de manera que dos de sus vrtices estn sobre el dimetro.
51. Cada lado de un cuadrado tiene una longitud L. Hallar el tamao del cuadrado de mxima
rea que puede circunscribirse al cuadrado dado.
52. Se va a construir un vaso de papel en forma de cono circular recto quitando un sector
circular a una hoja de papel con forma de crculo y radio r, y uniendo despus las dos orillas
rectas del papel restante. Calcule el volumen del vaso ms grande que se pueda construir.
53. Una isla situada a 20 kilmetros de una costa prcticamente recta, tiene que disponer per-
manentemente el servicio trasbordador para los carros de una ciudad situada a 50 kilmetros
costa abajo:
a) Si el trasbordador va a 15 kilmetros por hora y los automviles a un promedio de 80
kilmetros por hora. Dnde debe localizarse la terminal, en tierra, del trasbordador para
que el viaje sea lo ms rpido posible?
b) Si el trasbordador va a F kilmetros por hora y los automviles promedian los C
kilmetros por hora. Para qu valores de F/C debe localizarse la terminal exactamente en
la ciudad sobre tierra rme para que el viaje sea lo ms rpido posible?
54. Calcule el volumen del cono circular recto ms grande que se puede inscribir en una esfera
de radio r.
55. Demostrar que entre todos los rectngulos de rea dada, el cuadrado tiene el crculo cir-
cunscrito mnimo.
56. Encuentre el punto de la grca de y = x
2
+ 1 ms cercano al punto P(3, 1).
CAPTULO 8. FUNCIONES ALGEBRAICAS 425
57. Dada una esfera de radio R. Hallar el radio r y la altura h del cilindro circular recto de
mayor supercie lateral 2rh que puede inscribirse en la esfera.
58. La resistencia de una viga rectangular es directamente proporcional al producto del ancho
y el cuadrado de la altura de su seccin transversal. Halle las dimensiones de la viga ms
resistente que se pueda obtener de un tronco circular de radio r.
59. Un trozo de madera de 12 decmetros de largo tiene forma de un tronco de cono circular
recto de dimetros 4 y 4 +h decmetros en sus bases, donde h 0. Determinar en funcin de
h el volumen del mayor cilindro circular recto que se puede cortar de este trozo de madera,
de manera que su eje coincida con el del tronco de cono.
60. Una carretera A que va de norte a sur y otra carretera B que va de este a oeste se cruzan
en un punto P. A las 10:00 horas un automvil pasa por P viajando hacia el norte sobre A
a 80 kilmetros por hora. En ese mismo momento, un avin que vuela hacia el este a 320
kilmetros por hora y a una altura de 8500 metros, pasa exactamente por arriba del punto de
la carretera B que se encuentra 160 kilmetros al oeste de P. Suponiendo que el automvil
y el avin mantienen la misma velocidad y direccin, a qu hora se encontrarn ms cerca
uno del otro?
61. Un tanque de peso W es movido a lo largo de un plano por una fuerza que forma un
ngulo con la recta de la direccin del movimiento, siendo 0 /2. Supongamos que
la resistencia por friccin es proporcional a la fuerza normal con la que el bloque presiona
perpendicularmente contra el plano. Hallar el ngulo para el que la fuerza de propulsin
necesaria para vencer la friccin sea lo ms pequea posible.
62. Hay que construir un silo de forma cilndrica rematado por una bveda semiesfrica. El
coste de construccin por metro cuadrado es doble en la bveda que en la parte cilndrica.
Hllense las dimensiones, si el volumen se ja de antemano, para que los costes de produccin
sean mnimos. Desprciese el espesor de la pared y los desperdicios de material.
63. Si la suma de las supercies de un cubo y de una esfera es constante, determnese la relacin
del dimetro de la esfera a la arista del cubo en los casos de que:
a) Sea mnima la suma de volmenes; b) Sea mxima esta suma.
64. Un alambre de 36 centmetros de largo se va a partir en dos trozos. Una de las partes se ha
de doblar en forma de tringulo equiltero y la otra en forma de un rectngulo cuya longitud
es el doble de su anchura. Cmo se debe partir el alambre para que la suma de las reas del
tringulo y el rectngulo sea mxima.
65. Dos pasillos de 3 y 4 metros de ancho se encuentran formando un ngulo recto. Evale la
longitud de la barra rgida ms larga que puede transportarse horizontalmente dando vuelta
a la esquina.
CAPTULO 8. FUNCIONES ALGEBRAICAS 426
66. Encuentre la ecuacin de la recta tangente a la elipse b
2
x
2
+ a
2
y
2
= a
2
b
2
en el primer
cuadrante y que forma con los ejes coordenados el tringulo de menor rea posible.
67. Dos ciudades, situadas a un mismo lado de un ro rectilneo, acuerdan construir en la orilla
una estacin de bombeo y ltrado para el suministro de agua potable a las mismas. Si son A
y B las distancias de las ciudades al ro, y es C la distancia que las separa, prubese que la
suma de las longitudes de tubera necesaria para unirlas con la estacin de bombeo es igual
o mayor que

C
2
+ 4AB.
68. La luz que emana de un foco luminoso A se reeja sobre un espejo plano e incide sobre un
punto B. Si es mnimo el tiempo necesario para que la luz vaya desde A al espejo y desde
aqu a B. Prubese que son iguales los ngulos de incidencia y de reexin.
69. Dos fbricas A y B que se encuentran a 4 millas una de la otra, emiten humo con partculas
que contaminan el aire de la regin. Suponga que el nmero de partculas provenientes de
cada fbrica es directamente proporcional a la cantidad de humo e inversamente proporcional
al cubo de la distancia desde la fbrica. Qu punto entre A y B tendr la menor contami-
nacin si la fabrica A emite el doble de humo que la fbrica B?
70. Una pequea isla est a 2 millas, en lnea recta del punto ms cercano P de la ribera de un
gran lago. Si un hombre puede remar en su bote a 3 millas por hora y caminar 4 millas por
hora, dnde debe desembarcar para llegar a un pueblo que est 10 millas playa abajo del
punto P, en el tiempo mas corto? Suponga que el hombre usa su bote de motor que avanza
a 20 millas por hora, dnde debe desembarcar?
71. Si a y b son los catetos de un tringulo rectngulo cuya hipotenusa es 1. Hallar el mayor
valor de 2a +b.
72. Un tringulo issceles tiene base b y lados iguales de longitud a. Encuentre las dimensiones
del rectngulo de mayor rea que se puede inscribir en el tringulo de manera que uno de sus
lados coinciden con la base del tringulo?
73. Una ventana tiene forma de un rectngulo coronado por un tringulo equiltero. Encuentre
las dimensiones del rectngulo para el cual el rea de la ventana es mxima, si el permetro
de la misma debe ser de 12 pie.
74. La intensidad de iluminacin que produce un foco en cualquier punto es proporcional a la
intensidad del mismo e inversamente proporcional al cuadrado de la distancia. Si dos focos
de intensidades a y b se encuentran a una distancia c, en qu punto de la recta que les une
existe un mnimo de intensidad? Supngase que la intensidad en cualquier punto es la suma
de las intensidades debidas a ambos focos.
75. Se desea construir un cilindro juntando los lados AD y BC de un rectngulo de material
elstico. Para hacer ms resistente el cilindro, se colocar un alambre de longitud ja L segn
CAPTULO 8. FUNCIONES ALGEBRAICAS 427
la diagonal del rectngulo. Calcule el ngulo para el cual el volumen del cilindro es mximo.
76. Un barco debe navegar 100 millas ro arriba contra una corriente de 10 millas por hora. Sea
v la velocidad del barco en millas por hora. El nmero de galones de gasolina que consume
la nave es directamente proporcional a v
2
:
a) Demuestre que si se mantiene la velocidad constante de v millas por hora, entonces el
nmero total y de galones de combustible que se consumen est dado por y =
100kv
2
v10
, donde
v > 10 y k una constante positiva.
b) Calcule la velocidad que minimiza el nmero de galones de gasolina que se consumen
durante el viaje.
77. Se va a inscribir un cono circular recto dentro de otro cono circular recto de volumen dado,
con el mismo eje y con el vrtice del cono interior tocando la base del exterior. Cul debe
ser la razn de sus alturas para que el cono inscrito tenga el mximo volumen?
78. Se desea construir una tienda de campaa con forma de pirmide de base cuadrada. Un
poste de metal colocado en el centro ser el soporte de la tienda. Se cuenta con s pie cuadra-
dos de lona para los cuatro lados del albergue y x es la longitud de la base. Demuestre que:
a) El volumen V de la tienda es V =
1
6
x

s
2
x
4
;
b) V alcanza un valor mximo cuando x =

2 veces la longitud del poste.


79. Girando un rectngulo de permetro p alrededor de uno de sus lados, se genera un cilindro
circular recto. Calcule las dimensiones del rectngulo que producen el cilindro de mayor vol-
umen.
80. Cules son las dimensiones relativas de un cilindro circular recto, con la mxima supercie
curva, que se puede inscribir en una esfera dada?
8.8. Operaciones con funciones
A continuacin vamos a estudiar cmo se pueden efectuar operaciones entre funciones para
producir otras nuevas. Veremos que estos objetos matemticos pueden sumarse, multiplicarse,
dividirse, y producir as nuevos objetos de esta naturaleza.
Denicin 8.23 Suma
Sean f : D
1
R R y g : D
2
R R dos funciones denidas en los subconjuntos D
1
y D
2
de
R, respectivamente. Se dene la suma de f y g, como la funcin f + g : D R R denida en
D = D
1
D
2
y dada por (f +g)(x) = f(x) +g(x).
As pues, la suma de las funciones f y g es una nueva funcin f +g cuya imagen en un punto
x es la suma de las imgenes de f y g en x. Observe que para poder suma funciones f y g en un
punto x, es necesario poder evaluar las funciones f y g en x, de tal modo que x debe pertenecer
tanto al dominio de f como al de g. Es por eso que el dominio de la funcin suma de f y g es la
interseccin de los dominios de estas dos funciones.
CAPTULO 8. FUNCIONES ALGEBRAICAS 428
Denicin 8.24 Producto
Sean f : D
1
R R y g : D
2
R R dos funciones denidas en los subconjuntos D
1
y D
2
de
R, respectivamente. Se dene el producto de f y g, como la funcin fg : D R R denida en
D = D
1
D
2
y dada por (fg)(x) = f(x)g(x).
Como en el caso de la suma, la funcin producto de f y g se dene como la funcin cuya
imagen en el punto x es el producto de las imgenes de f y g en ese punto. Para poder calcular
estas ltimas, es necesario que la x se encuentre en el dominio de f y en el de g. Es decir, al igual
que la funcin suma, la funcin producto tiene por dominio a la interseccin de los dominios de las
funciones involucradas.
Denicin 8.25 Cociente
Sean f : D
1
R R y g : D
2
R R dos funciones denidas en los subconjuntos D
1
y D
2
de
R, respectivamente. Se dene el cociente de f entre g, como la funcin
f
g
: D R R denida
en D = D
1
D
2
x D
2
: g(x) = 0 y dada por
_
f
g
_
(x) =
f(x)
g(x)
.
En el caso de cociente, la denicin es similar a las dadas anteriormente para la suma y para el
producto, slo que en este caso el dominio de la funcin cociente presenta un a restriccin adicional:
debemos eliminar la posibilidad de que el denominador de la nueva funcin sea cero.
Denicin 8.26 Composicin
Sean f : D
1
R R y g : D
2
R R dos funciones denidas en los subconjuntos de R,
D
1
y D
2
, respectivamente, tales que g(D
2
) = y R/y = g(x), x D
2
D
1
. Se dene la
composicin de f con g, como la funcin f g : D R R dada por (f g)(x) = f(g(x)).
Obsrvese la restriccin que se establece en la denicin sobre los dominios de las funciones
f y g. Para poder evaluar la funcin compuesta f g en un punto x, debemos evaluar primero
la funcin g en x, lo cual nos impone de inmediato la restriccin de que tal x debe pertenecer
al dominio de g. Sin embargo, esta restriccin no es suciente, pues en la siguiente etapa, para
obtener la imagen (f g)(x), debemos evaluar la funcin f en g(x), lo cual es posible solamente si
y = g(x) pertenece al dominio de f.
As pues, el dominio de la funcin compuesta f g est formado por aquellas x que pertenezcan
al dominio de g, tales que y = g(x) pertenezca al dominio de f. Es por eso que en la denicin se
pide que las imgenes de g (el conjunto g(D
2
)) pertenezcan al dominio de la funcin f (es decir,
que el conjunto g(D
2
) sea un subconjunto de D
1
).
Ejemplo 8.29 Un estudio ambiental en una determinada comunidad seala que el nivel medio
diario de monxido de carbono en el aire ser c(p) = 0, 5p+1 partes por milln cuando la poblacin
sea de p miles. Se estima que dentro de t aos la poblacin de la comunidad ser p(t) = 10 +0, 1t
2
miles:
a) Exprese el nivel de monxido de carbono en el aire como una funcin del tiempo.
b) Cundo alcanzar el nivel de monxido de carbono 6,8 partes por milln?
Solucin
a) Puesto que el monxido de carbono esta relacionado con la variable p por la ecuacin c(p) =
0, 5p + 1 y la variable p est relacionada con la variable t por la ecuacin p(t) = 10 + 0, 1t
2
se
desprende que la funcin compuesta
c(p(t)) = c(10 + 0, 1t
2
) = 0, 5(10 + 0, 1t
2
) + 1 = 6 + 0, 05t
2
CAPTULO 8. FUNCIONES ALGEBRAICAS 429
expresa el nivel de monxido de carbono en el aire como una funcin de la variable t.
b) Sea c(p(t)) igual a 6,8 y despjese t para obtener
6 + 0, 05t
2
= 6, 8 0, 05t
2
= 0, 8 t
2
=
0, 8
0, 05
= 16 t = 4
es decir, dentro de 4 aos el nivel de monxido de carbono ser 6,8 partes por milln. c _
Ejemplo 8.30 En cierta industria, el costo total de produccin de q unidades durante el peri-
odo diario de produccin es c(q) = q
2
+q + 900 dlares. En un da normal de trabajo, se fabrican
q(t) = 25t unidades durante las primeras t horas de un periodo de produccin:
a) Exprese el costo total de produccin como una funcin de t.
b) Cunto se habr gastado en produccin al nal de la tercera hora?
c) Cundo alcanzar el costo total de produccin $ 10000?
Solucin
a) Puesto que el costo de produccin esta relacionado con la variable q por la ecuacin c(q) =
q
2
+q +900 y la variable q est relacionada con la variable t por la ecuacin q(t) = 25t se desprende
que la funcin compuesta
c(q(t)) = c(25t) = (25t)
2
+ 25t + 900 = 625t
2
+ 25t + 900 = c(t)
expresa el costo total de produccin como una funcin de la variable t.
b) Al nal de la tercera hora, t = 3, se habr gastado
c(3) = 625(3)
2
+ 25(3) + 900 = $6600
c) Para calcular t cuando el costo total de produccin c(t) = 10000, hacemos
10000 = 625t
2
+ 25t + 900 t 3, 8 horas. c _
Ejemplo 8.31 Considere las funciones f y g. Describa las funciones f g y g f, as como
los dominios de stas:
a) f(x) = x
2
+ 3x 5, g(x) = 3x + 4; b) f(x) =

x
2
+ 2, g(x) = 2x + 3;
c) f(x) =

x + 1, g(x) =

x 1.
Solucin
a) Tanto f como g tienen por dominio a todos los reales. En este caso no hay restriccin alguna
para los dominios de las funciones compuestas: Ser tambin el conjunto de los reales. Tenemos
entonces que:
(f g)(x) = f(g(x)) = f(3x + 4) = (3x + 4)
2
+ 3(3x + 4) 5 = 9x
2
+ 33x + 23
(g f)(x) = g(f(x)) = g(x
2
+ 3x 5) = 3(x
2
+ 3x 5) + 4 = 3x
2
+ 9x 11
b) Tanto f como g tienen por dominio a todos los reales. En este caso no hay restriccin alguna
para los dominios de las funciones compuestas: Ser tambin el conjunto de los reales. Tenemos
entonces que:
(f g)(x) = f(g(x)) = f(2x + 3) =
_
(2x + 3)
2
+ 2 =
_
4x
2
+ 12x + 11
(g f)(x) = g(f(x)) = g(
_
x
2
+ 2) = 2
_
x
2
+ 2 + 3
c) El dominio de la funcin f esta dada por x 1 y el de g es x 1. Entonces el dominio de
g se encuentra en f. Por tanto
(f g)(x) = f(g(x)) = f(

x 1) =
_

x 1 + 1
CAPTULO 8. FUNCIONES ALGEBRAICAS 430
El dominio de esta nueva funcin esta denido para todas las x 1. Para el caso g f, tenemos
(g f)(x) = g(f(x)) = g(

x + 1) =
_

x + 1 1
cuyo dominio es el conjunto x R/x 0. c _
8.9. Tarea
1. Suponga que f, g y h son funciones tales que f(1) = 4, g(1) = 2 y h(1) = 3. Determine
en cada caso la imagen indicada:
a) (3f 4g)(1);
b) g(1)[2f(x) + 3h(1)];
c) (f
2
+g
2
+h
2
)(1);
d) f(1)[1 + 5g(1)];
e) (f +g)(1)(f +g)(x);
f )
_
(f 5g)(1)
3h(1)
_
f(x);
g)
_
2f
g 3h
_
(1);
h)
_
3f
2
g
2g h
2
_
(1).
2. Sean f(x) = x+6, g(x) =
6
x+1
. Encuentre todos los valores de x para los que se cumple la
ecuacin:
a) [f(x) +g(x)[ = f(x) +g(x);
b) [f(x) +g(x)[ = [f(x)[ [g(x)[;
c) [f(x) +g(x)[ = [f(x)[ +[g(x)[;
d) [f(x) +g(x)[ = f(x) g(x).
3. Resolver la ecuacin [f(x) g(x)[ = [f(x)[ [g(x)[, dado f(x) = 2x + 3, g(x) = 3x 2.
4. Dada la expresin f(x) = ax
2
bx+3, hallar los valores de a y b para los cuales se cumpla
la ecuacin f(x 2) +f(x + 2) = 4x 2.
5. Dadas las funciones
f(x) = 5x 2, g(x) = 3x
2
2x + 1, h(x) =
_
2x, x < 0
x
2
+ 1, x 0
Determine en cada caso la imagen indicada:
a) (2g + 3h)(0);
b) (f
2
g
2
)(1)(h f)(3x);
c) h(1 +g(1));
d) (3g
2
+h)(1);
e) (fgh)(2)h(2 x
2
);
f ) f(1 + 2g(1 + 2h(1)));
g) g(1 f(1 h(1)));
h) (2f 2g)(3g 3h)(f(1 +f(1)));
i) f(2 h(2 x2));
j) (f g + 3h)(2);
k)
_
h
g
_
(f(f(1)));
l) g(3 +f(2));
CAPTULO 8. FUNCIONES ALGEBRAICAS 431
6. Considere las funciones
f(x)
_

_
x
2
, x < 1
2, x = 1
5, 1 < x 1
x + 6, x > 1
y g(x) =
_

_
1
x
2
+ 1
, x < 2
3x + 2, 2 x < 2
1, x = 2
2x 1, x > 2
Calcule:
a) (f +g)(1); b) (fg)(2).
7. Para cada una de las funciones f y g, determine: f +g, fg,
f
g
, f g, g f:
a) f(x) = 3x
2
+ 4x 1, g(x) = 5x
2
+ 3x + 1;
b) f(x) = x +

x 1, g(x) = x
2
1;
c) f(x) =

x
2
x + 1, g(x) = x
2
+x;
d) f(x) = x
3
x, g(x) = x
4
3x
2
1;
e) f(x) = Sgn(x
2
1), g(x) =

x
2
1;
f ) f(x) =

x 4, g(x) =

x 5;
g) f(x) = [x
2
1], g(x) =
x + 1
x
;
h) f(x) =
_
x
2
x
2
1
_
, g(x) = [x
2
1[.
8. Para cada una de las funciones f y g, determine: f +g, fg, f g, g f:
a) f(x) =
_
4x + 3, x < 1
x
3
, x 1
, g(x) = x + 2;
b) f(x) =
_
x 1, x < 0
x + 1, x 1
, g(x) = x
2
1;
c) f(x) =
_

_
3x + 2, x < 1
5x, 1 x 1
4x 3, x > 1
, g(x) = 5 2x;
d) f(x) =
_

_
x, x < 0
x
2
, 0 x 1
x
3
, x > 1
, g(x) = x
2
+ 2;
e) f(x) =
_

_
x, x < 0
x
2
, 0 x 1
x
3
, x > 1
, g(x) = x
2
2;
f ) f(x) =
_
x + 3, x < 0
2x
2
+ 3, x 0
, g(x) = x + 1;
g) f(x) =
_
4x + 3, x < 1
x
3
, x 1
, g(x) = x + 2;
h) f(x) =
_
x
2
, x < 0
x
2
, x 0
, g(x) =
_
1 x
2
, x < 0
x
2
1, x 0
;
CAPTULO 8. FUNCIONES ALGEBRAICAS 432
i) f(x) =
_

_
x, x < 1
x
3
, 1 x 1
2x 1, x > 1
, g(x) =
_
1 x
2
, x < 0
x
2
1, x 0
;
j) f(x) =
_

_
x, x < 1
x
4
, 1 x 1
x, x > 1
, g(x) =
_

_
x, x < 1
x
5
, 1 x 1
2x 1, x > 1
;
k) f(x) =
_
x, x < 0
x
2
, x 0
, g(x) =
_
4x 2, x < 1
5x + 1, x 1
;
l) f(x) =
_
5x + 4, x < 0
3x
2
+x, x 0
, g(x) =
_
x 1, x < 0
x
2
1, x 0
;
m) f(x) =
_
3x + 2, x 0
5x + 4, x > 0
, g(x) =
_
2x + 1, x 0
2 x, x > 0
;
n) f(x) =
_
x, x < 0
2x, x 0
, g(x) =
_
x + 1, x < 1
4x + 4, x 1
.
9. Considere las funciones f, g y h. Describa las funciones f f, g g, f g h:
a) f(x) =
x
2
x + 1
, g(x) =

1 x
2
, h(x) = x
2
3x + 2;
b) f(x) =
x

x + 1
, g(x) =

x 1
x
, h(x) =
x + 1
x
.
10. Sea f(x) =
1
x
. Determine la funcin compuesta (f f)(x). En dnde est denida esta
funcin?
11. Si f(x) =
1
1 x
, determine (f f f)(x). En dnde est denida esta funcin?
8.10. Grca de una funcin
Una de las caractersticas importantes que tienen las funciones reales de una variable real es
que podemos tener representaciones geomtricas de ellas, por medio de una curva en el plano
cartesiano, que llamaremos grca de la funcin.
Denicin 8.27 Grca de una funcin
Sea f : D R R una funcin denida en D. Se denomina grca de la funcin real y = f(x)
un conjunto de puntos (x
0
, y
0
) en el plano que satisface las siguientes condiciones:
a) Todo punto con las coordenadas (x
0
, y
0
), donde y
0
= f(x
0
), pertenece a este conjunto;
b) Todo punto perteneciente a dicho conjunto de puntos tiene tales coordenadas (x
1
, y
1
), que
y
1
= f(x
1
).
Es decir, la grca de la funcin y = f(x) es el conjunto de todos los puntos del plano cuyas
coordenadas satisfacen la condicin y = f(x), y no contiene otros puntos.
Si para una funcin dada y = f(x) se han estudiado todas las propiedades mencionadas ante-
riormente, suele decirse que se ha realizado el anlisis de la funcin y = f(x). As pues, al analizar
una funcin, se debe responder a las siguientes preguntas:
CAPTULO 8. FUNCIONES ALGEBRAICAS 433
1. Cul es el dominio e la funcin?
2. Cul es el codominio de la funcin?
3. Est acotada o no la funcin?
4. Toma la funcin los valores mximo y mnimo?
5. Es peridica?
6. Es la funcin par o impar o ni una ni otra?
7. Tiene la funcin intervalos, donde es montona?
8. Hay puntos de interseccin de la grca con los ejes de coordenadas?
9. Cul es la grca de la funcin?
Se propone el siguiente mtodo de construccin de las grcas de funciones, que se basa en el
empleo de algunas reglas de construccin, valindose de las grcas de funciones ya conocidas.
Supongamos que se da la grca de la funcin y = f(x). Construyamos la grca de la funcin
y = f(xa). Esta grca puede ser obtenida del modo siguiente: partiendo del punto arbitrario x,
en el que la ordenada f(x) se conoce, determinaremos el punto x
1
en el cual la ordenada f(x
1
a)
tiene el mismo valor, es decir, se cumple la igualdad
f(x
1
a) = f(x).
Para que se cumpla esta igualdad basta, evidentemente, que se cumpla la igualdad
x
1
a = x
de donde encontramos que x
1
= x +a.
Regla 1. Para obtener la grca de la funcin y = f(x a) a partir de la grca de la
funcin y = f(x) es necesario la grca de la funcin y = f(x) desplazarla a lo largo del eje 0X en
a a la derecha, si a > 0, o bien en [a[ a la izquierda, si a < 0.
Se da la grca de la funcin y = f(x). Vamos a construir la grca de la funcin y = f(x) +c.
Regla 2. Para obtener la ordenada de la grca de la funcin y = f(x) +c en el punto x a
partir de la ordenada de la grca de la funcin y = f(x) en el mismo punto, es necesario desplazar
la grca de la funcin y = f(x) a lo largo del eje 0Y hacia arriba en c, si c > 0, o bien en [c[ hacia
abajo, si c < 0.
Se da la grca de y = f(x). Constryase la grca de la funcin y = f(x).
Regla 3. Para obtener la ordenada de la grca de la funcin y = f(x) en el punto x a
partir de la ordenada de la grca de la funcin y = f(x) en el mismo punto, es necesario en la
ordenada de la grca de la funcin y = f(x) cambiar el signo por el opuesto. As pues, la grca de
la funcin y = f(x) se obtiene a partir de la grca de la funcin y = f(x) mediante la reexin
directa respecto al eje 0X.
CAPTULO 8. FUNCIONES ALGEBRAICAS 434
Se da la grca de la funcin y = f(x). Constryase la grca de la funcin y = f(x).
Regla 4. Para obtener la ordenada de la grca de la funcin y = f(x) en el punto x a
partir de la ordenada de la grca y = f(x) en el mismo punto, es necesario multiplicar el valor x
por -1. As pues, la grca de la funcin y = f(x) se obtiene a partir de la grca de la funcin
y = f(x) mediante la reexin directa respecto al eje 0Y.
Se da la grca de la funcin y = f(x). Constryase la grca de la funcin y = kf(x).
Regla 5. Para obtener la ordenada de la grca de la funcin y = kf(x) en el punto x a par-
tir de la ordenada de la grca de la funcin y = f(x) en el mismo punto, es necesario multiplicar
el valor de la ordenada f(x) por el nmero k.
En este caso debido a la multiplicacin de todos los valores de la funcin f(x) por k > 1 las
ordenadas de la grca de la funcin aumentan k veces y la grca de la funcin y = f(x) se
estira a partir del eje 0X k veces, mientras que debido a la multiplicacin por k para 0 < k < 1
las ordenadas de la grca de la funcin disminuyen k veces y la grca de la funcin y = f(x) se
contrae k veces hacia el eje 0X.
Se da la grca de la funcin y = f(x). Constryase la grca de la funcin y = f(kx). Partiendo
de un punto arbitrario x en el cual se conoce la ordenada f(x) encontraremos el punto x
1
en el
cual la grca de la funcin y = f(kx
1
) tiene la misma ordenada, es decir, se cumple la igualdad
f(x) = f(kx
1
).
Para que esta igualdad se cumpla es, evidentemente, suciente el cumplimiento de la igualdad
x = kx
1
, de donde encontramos x
1
=
1
k
x.
Regla 6. Para construir la grca y = f(kx) basta dividir el valor de x por el nmero k.
En este caso debido a la divisin de todos los valores del argumento de la funcin y = f(x) por
k > 1 la grca de la funcin se contrae hacia el eje 0Y,
1
k
veces y debido a la divisin por k para
0 < k < 1 la grca de la funcin se estira a partir del eje 0Y,
1
k
veces.
Se da la grca de la funcin y = f(x). Constryanse la grca de la funcin y = [f(x)[.
Tenemos
[f(x)[ =
_
f(x) si x 0
f(x) si x < 0
Regla 7. Para obtener la grca de la funcin y = [f(x)[ a partir de la funcin y = f(x)
es necesario dejar sin cambios los trozos de la grca y = f(x) que estn por encima del eje 0X y
reejar en forma especular respecto al eje 0X los trozos inferiores a este eje.
Se da la grca de la funcin y = f(x). Constryase la grca de la funcin y = f([x[). Puesto
que f([ x[) = f([x[), la funcin y = f([x[) es par, por lo tanto, su grca es simtrica respecto al
eje 0Y. Adems, para x 0, f([x[) = f(x).
Regla 8. Para obtener la grca de la funcin y = f([x[) a partir de la grca de la ecuacin
y = f(x) es necesario construir la grca de la funcin y = f(x) para x 0 y reejarla en forma
especular respecto al eje 0Y.
CAPTULO 8. FUNCIONES ALGEBRAICAS 435
Denicin 8.28 Asntota
Si la distancia entre un punto variable sobre una curva y una recta ja se hace y permanece menor
que cualquier nmero preasignado, arbitrariamente pequeo y positivo cuando el punto se aleja
innitamente sobre la curva, se dice que la recta es una asntota de la curva.
Si una ecuacin en x e y se resuelve para y en trminos de x, puede ocurrir que un valor de x,
digamos a, haga el cero el denominador del miembro derecho sin hacer cero al numerador. Si hay
un valor tal de x, no puede usarse, porque ningn valor de y corresponde a l, ya que la divisin
entre cero no es una operacin posible. No obstante, si x est sucientemente cerca de a, y es
numricamente mayor que cualquier nmero preasignado.
Ahora vericaremos que la distancia de la recta xa = 0 a un punto cualquiera P
1
(x
1
, y
1
) sobre
una curva, e innitamente alejada del origen, es tan pequea como deseemos bajo las condiciones
enunciadas anteriormente. La ecuacin de la recta, x a = 0, est ya en forma normal si a > 0:
por consiguiente, sustituimos en ella las coordenadas de P
1
con objeto de obtener la distancia d
y el sentido de la recta a P
1
. As, tenemos d = x
1
a; sin embargo, x
1
a es tan pequea como
deseemos porque x
1
est sucientemente cerca de a. El anlisis es anlogo si a es negativa.
Se puede deducir de manera anloga que y = b es una asntota de la curva si x llega a ser y
permanece mayor que cualquier nmero preasignado cuando y est sucientemente cerca de b.
Denicin 8.29 Asntotas horizontales y verticales
La recta x a = 0 es una asntota vertical de una curva si x a es un factor del denominador
despus que en la ecuacin se ha despejado y en trminos de x, y se han eliminado todos los
factores comunes en el numerador y el denominador.
La recta y b = 0 es una asntota horizontal de una curva si y b es un factor del denominador
despus que en la ecuacin se ha despejado x en trminos de y, y se han eliminado todos los
factores comunes en el numerador y el denominador.
Para encontrar las asntotas horizontales de una curva, hgase el coeciente de la mayor po-
tencia de x igual a cero y despjese y. Para encontrar las asntotas verticales de una curva, hgase
el coeciente de la mayor potencia de y igual a cero y despjese x.
Para encontrar cualquier interseccin de y = f(x) con el eje Y , se hace x igual a cero y se
calcula y. Para encontrar cualesquiera intersecciones de y = f(x) con el eje X se hace y igual a
cero y se despeja x.
El carcter ilustrativo de la grca hace de ella un medio auxiliar insustituible del anlisis
de una funcin, pero la grca slo ilustra las propiedades de la funcin y no las demuestra. A
continuacin analizaremos los diversos tipos de funciones:
Funcin constante y = k: A cada nmero real x dicha funcin pone en correspondencia
un mismo nmero k. La grca de la funcin representa una recta, paralela al eje de abscisas, que
dista de este eje a una magnitud [k[ y pasa por encima de l, si k > 0, y por debajo, si k < 0. Es
una funcin continua en todo el eje real.
Funcin lineal y = x: Si y es funcin de x a cada valor de x le corresponde un valor
determinado de y. Por lo tanto, dando muchos valores diferentes de x hallamos diferentes y cor-
respondientes a ellos y estos pares de valores (x, y) proporcionarn muchos puntos en el plano. Si
aumentamos el nmero de algunos valores de x, tomndolos ms cercanos entre s, al n y al cabo,
CAPTULO 8. FUNCIONES ALGEBRAICAS 436
estos puntos formarn una curva continua. Esta curva se denomina grca de la funcin.
Consideremos la llamada dependencia lineal y = mx + b. Esta ecuacin, es llamada ecuacin
de una recta. El coeciente m determina el ngulo entre la recta y el eje X. Sustituyendo en la
ecuacin x = 0 obtenemos y = b. Esto signica que uno de los puntos de la recta es el punto (0, b);
este punto est situado en el eje Y a la altura b sobre el origen de coordenadas. Si b < 0, el punto
estar situado debajo del origen de coordenadas. As pues, b es la ordenada del punto de intersec-
cin de la recta con el eje Y , [b[ es la longitud del segmento cortado por la recta en el eje de las
ordenadas. Para construir una recta correspondiente a una ecuacin dada, no es necesario calcular
las coordenadas de un gran nmero de puntos y marcarlas en la grca: est claro que si se con-
struyen dos puntos, as mismo queda determinada por completo la recta que pasa a travs de stos.
La dependencia y = x se denomina directamente proporcional. Se comprueba con facilidad las
siguientes propiedades de esta funcin:
1. El dominio es (; +);
2. El codominio es (; +);
3. La funcin no est acotada ni inferior ni superiormente;
4. La funcin no toma ni el valor mximo, ni tampoco el mnimo;
5. La funcin no es peridica;
6. La funcin es impar;
7. La funcin es creciente en todo el intervalo (; +);
8. El punto (0, 0) es el nico punto de interseccin con los ejes coordenados.
Funcin potencial y = x
k
: Las funciones estudiadas anteriormente, representan casos
particulares de la funcin potencial. A continuacin vamos estudiar otros casos:
1. y = x
2k
, k N:
a) El dominio es (; +);
b) El codominio es [0; +);
c) La funcin est acotada inferiormente: y 0;
d) La funcin toma su valor mnimo y = 0 cuando x = 0;
e) La funcin no es peridica;
f ) La funcin es par;
g) La funcin no es montona en todo el dominio, pero es decreciente en el intervalo
(; 0] y creciente en el intervalo [0; +);
h) El punto (0, 0) es el nico punto de interseccin con los ejes coordenados.
2. y = x
2k1
, k N:
a) El dominio es (; +);
b) El codominio es (; +);
c) La funcin no est acotada ni superior ni inferiormente;
CAPTULO 8. FUNCIONES ALGEBRAICAS 437
d) La funcin no toma el valor mximo ni tampoco el mnimo;
e) La funcin no es peridica;
f ) La funcin es impar;
g) La funcin es creciente en todo el dominio;
h) El punto (0, 0) es el nico punto de interseccin con los ejes coordenados.
3. y = x
2k
, k N:
a) El dominio es (; 0) (0; +);
b) El codominio es (0; +);
c) La funcin est acotada inferiormente: y > 0;
d) La funcin no toma el valor mximo ni tampoco el mnimo;
e) La funcin no es peridica;
f ) La funcin es par;
g) La funcin no es montona en todo el dominio, pero crece en el intervalo (; 0) y
decrece en el intervalo (0; +);
h) No hay puntos de interseccin con los ejes coordenados.
4. y = x
2k+1
, k N:
a) El dominio es (; 0) (0; +);
b) El codominio (; 0) (0; +);
c) La funcin no est acotada ni superior ni inferiormente;
d) La funcin no toma el valor mximo ni tampoco el mnimo;
e) La funcin no es peridica;
f ) La funcin es impar;
g) La funcin no es montona en todo el dominio, pero decrece en el intervalo (; 0)
y, adems, en el intervalo (0; +);
h) No hay puntos de interseccin con los ejes coordenados.
5. y = x
k
, k > 0, k / Z:
a) El dominio es [0; +);
b) El codominio es [0; +);
c) La funcin est acotada inferiormente: y 0;
d) La funcin toma el valor mnimo y = 0 para x = 0;
e) La funcin no es peridica;
f ) La funcin no es par ni tampoco impar;
g) La funcin es creciente en todo el dominio;
h) El punto (0, 0) es el nico punto de interseccin con los ejes coordenados.
6. y = x
k
, k > 0, k / Z:
a) El dominio es (0; +);
CAPTULO 8. FUNCIONES ALGEBRAICAS 438
b) El codominio es (0; +);
c) La funcin est acotada inferiormente: y > 0;
d) La funcin no toma el valor mximo ni tampoco el mnimo;
e) La funcin no es peridica;
f ) La funcin no es par ni tampoco impar;
g) La funcin es decreciente en todo el dominio;
h) No hay puntos de interseccin con los ejes coordenados.
Algunas funciones poseen caractersticas especiales comunes que permiten agruparlas y llamar-
las de algn modo especco.
Denicin 8.30 Funcin algebraica
Una funcin f : D R R se dice que es algebraica, si las operaciones que la funcin hace con
la variable x, para obtener su imagen f(x) son solamente algebraicas (sumar, restar, multiplicar,
dividir, elevar a potencias, extraer races). En caso contrario se dice que la funcin es trascendente.
Denicin 8.31 Funcin polinomial
Sea n un nmero entero no negativo. Una funcin polinomial de grado n es una funcin f : R R
del tipo f(x) = a
n
x
n
+a
n1
x
n1
+... +a
1
x+a
0
, en donde a
0
, a
1
, ..., a
n
son nmeros reales dados
(llamados coecientes de la funcin) y a
n
,= 0.
Denicin 8.32 Funcin racional
Una funcin racional es un cociente de dos funciones polinomiales. Es decir, es una funcin del
tipo f(x) =
g(x)
h(x)
, en donde g(x) y h(x) son funciones polinomiales.
Funcin parte entera y = [x]: La funcin parte entera, se dene en el dominio de los
nmeros reales. A cualquier x de este dominio, la funcin asocia el mximo entero algebraicamente
menor o igual que x. En entero asociado con x se designa escribiendo este smbolo como [x]. Tiene
la propiedad de ser menor o igual que x, mientras que el entero siguiente es mayor que x; es decir:
[x] x [x] + 1. Es decir [x] es el nmero entero mayor que no sobrepasa x. En cada intervalo
[n; n+1), donde n Z, la funcin dada es constante e igual a n. Es evidente la razn de denominar
a toda funcin de este tipo funcin escalonada. De acuerdo con esto se ha representado su grca.
Funcin signo y = Sign(x): Por denicin
f(x) =
_

_
1, x > 0
0, x = 0
1, x < 0
La funcin no es par ni impar.
Construccin de la grca de la funcin y = [f(x)[ segn la grca de la funcin y = f(x).
Recordemos ante todo la denicin:
[f(x)[ =
_
f(x), f(x) 0
f(x), f(x) < 0
Supongamos que el punto P(x
0
, y
0
) pertenece a la grca de la funcin y = f(x), es decir, sea
y
0
= f(x
0
). Analicemos dos casos:
CAPTULO 8. FUNCIONES ALGEBRAICAS 439
1. y
0
0. Entonces, por cuanto [f(x
0
)[ = f(x
0
) = y
0
, el punto P(x
0
, y
0
) pertenece a la
grca de la funcin y = [f(x)[.
2. y
0
< 0. Entonces, por cuanto [f(x
0
)[ = f(x
0
) = y
0
, el punto Q(x
1
, y
0
) pertenece a la
grca de la funcin y = [f(x)[.
Por consiguiente, la grca de la funcin y = [f(x)[ se obtiene a partir de la grca para la
funcin y = f(x) del modo siguiente: Todos los puntos de la grca y = f(x), dispuestos en el eje
0X y por arriba de ste, quedan en su lugar. Todos los puntos de la grca y = f(x), dispuestos
por debajo del eje 0X, se aplican simtricamente respecto del eje 0X. Observemos que la grca
de la funcin y = [f(x)[ no tiene puntos por debajo del eje 0X.
Adicin de grcas: Sean dadas las funciones y = f(x) e y = g(x). En la parte comn
de sus dominios queda denida la funcin y = f(x) = g(x). Supongamos que el punto P(x
0
, y
1
)
pertenece a la grca de la funcin y = f(x), y el punto Q(x
0
, y
2
) pertenece a la grca de la
funcin y = g(x), con la particularidad de que el nmero x
0
pertenece a la parte comn de los
dominios de las funciones y = f(x) e y = g(x). En este caso el punto R(x
0
, y
1
+ y
2
) pertenece
a la grca de la funcin y = f(x) + g(x). Quiere decir, para construir la grca de la funcin
y = f(x) +g(x) es necesario:
a) Dejar aquellos puntos de las grcas y = f(x) e y = g(x) en los que x integra la parte comn
de los dominios de estas funciones.
b) Para cada tal x realizar la adicin algebraica de las ordenadas de estas dos grcas.
Multiplicacin de grcas: Sean dadas las funciones y = f(x) e y = g(x). Entonces, en
la parte comn de sus dominios queda denida la funcin y = f(x)g(x). Supongamos que el punto
P(x
0
, y
1
) pertenece a la grca de la funcin y = f(x), y el punto Q(x
0
, y
2
), a la grca de la
funcin y = g(x). Est claro que el nmero x
0
pertenece a la parte comn de los dominios de la
funcin y = f(x) e y = g(x). En este caso el punto R(x
0
, y
1
y
2
) pertenece a la grca de la funcin
y = f(x)g(x). Quiere decir, para construir la grca de la funcin y = f(x)g(x) es necesario:
a) Dejar aquellos puntos de las grcas y = f(x) e y = g(x), en los cuales x integra la parte
comn de los dominios de estas funciones.
b) Para cada tal x realizar la multiplicacin de las ordenadas de estas dos grcas.
Divisin de grcas: Sean dadas las funciones y = f(x) e y = g(x). Para obtener la grca
de la funcin y =
f(x)
g(x)
a partit de las grcas de las funciones f(x) y g(x) es necesario dividir los
valores correspondientes de las ordenadas de las grcas de las funciones f(x) y g(x) en los puntos
donde g(x) ,= 0.
Composicin de grcas: Se da la grca de la funcin u = g(x). Constryase la grca de
la funcin y = f[g(x)]. Para construir la grca de la funcin y = f[g(x)] es necesario primero con-
struir la grca de la funcin u = g(x) y luego conociendo las propiedades de la funcin y = f(u),
construir la grca de la funcin compuesta y = f[g(x)].
Ejemplo 8.32 Un fabricante puede producir grabadoras a un costo de $ 20 cada una. Se estima
que si stas se venden a x dlares cada una, los usuarios comprarn 120 x grabadoras al mes.
Exprese la utilidad mensual del fabricante como una funcin del precio, elabore la grca de esta
funcin y calcule el precio ptimo de venta.
Solucin
CAPTULO 8. FUNCIONES ALGEBRAICAS 440
Expresamos en palabras la relacin deseada
Utilidad = (cantidad de grabadoras vendidas)(utilidad por grabadora)
A continuacin reemplazamos las palabras por expresiones algebraicas
Cantidad de grabadoras vendidas = 120 x
Y puesto que las grabadoras se producen a un costo de $ 20 cada una y se venden a x dlares cada
una, se desprende que
Utilidad por grabadora = x 20
Si U(x) es la utilidad, se concluye que
U(x) = (120 x)(x 20) = x
2
+ 140x 2400
Para gracar esta funcin, debemos hacer el siguiente anlisis: Encontramos los puntos de corte
con el eje X, es decir (20, 0) y (120, 0).
Calculamos los puntos de mximo de la siguiente manera
x
2
+ 140x 2400 = k
2
+ 140k 2400 (x k)(x +k 140) = 0
Resolviendo esta ecuacin, obtenemos k = 70. Por tanto, tenemos que (70, 2500). Con estos datos,
procedemos a elaborar la grca de la funcin dada. El precio ptimo para vender las grabadoras
es de $ 70.
Ejemplo 8.33 Si un objeto se lanza verticalmente hacia arriba desde el suelo con una velocidad
de 160 pies por segundo, su altura (en pies), t segundos despus, est dada por la funcin H(t) =
16t
2
+ 160t:
a) Elabore la grca de la funcin H(t).
b) Cundo llegar al suelo el objeto?
c) Calcule qu altura alcanzar el objeto.
Solucin
a) El dominio de la funcin son todos los reales, t R. La funcin no es par ni impar. Los puntos
de corte con el eje t son (0, 0) y (10, 0). El punto de corte con el eje H(t) es (0, 0). A continuacin
procedemos a calcular los puntos de mximo y mnimo:
16t
2
+ 160t = 16k
2
+ 160k (t k)(t +k 10) = 0
CAPTULO 8. FUNCIONES ALGEBRAICAS 441
resolviendo esta ecuacin, obtenemos k = 5. Haciendo las comprobaciones, obtenemos que (5, 400)
es un punto de mximo.
b) El objeto llegara al suelo cuando H(t) = 0, es decir:
0 = 16t
2
+ 160t 0 = t
2
10t t(t 10) = 0
resolviendo esta ecuacin, tenemos que t = 10 segundos.
c) Con el punto de mximo obtenido en el literal a), podemos asegurar que cuando t = 5, la
altura alcanzada por el objeto es
H(5) = 16 5
2
+ 160 5 = 400 pies.
Ejemplo 8.34 Suponga que durante un programa nacional para inmunizar a la poblacin con-
tra cierto tipo de gripe, los funcionarios de salud pblica encontraron que el costo de vacunacin
del x % de la poblacin era aproximadamente f(x) =
150x
200 x
millones de dlares. Represente la
funcin de manera grca y especique qu segmento de sta es pertinente para la situacin prc-
tica en consideracin.
Solucin
Para representar grcamente esta funcin, debemos hac-
er el anlisis completo. El dominio de la funcin est dado
por x R200. El codominio de la funcin est dado por
f(x) R150. La curva no tiene puntos de mximo ni
mnimo.
La funcin no es par ni impar. Tiene una asntota vertical
en x = 200 y una asntota horizontal en f(x) = 150. Con
todos estos datos, ya podemos trazar la grca de la fun-
cin.
Para la situacin prctica en consideracin, debemos con-
siderar el tramo de curva que est determinada en el primer cuadrante, por cuanto, el rango de
vacunacin es 0 < x < 200.
8.11. Tarea
1. Encuentre la funcin lineal si:
a) y(1) = 0, y(0) = 2; b) y(1) = 2, y(1) = 1; c) y(5) = 3, y(2) = 1.
2. Encuentre la funcin cuadrtica si:
a) y(1) = 0, y(0) = 5, y(6) = 7; b) y(2) = 2, y(1) = 1, y(3) = 7;
c) y(6) = 7, y(3) = 8, y(2) = 7.
3. Encuentre el polinomio p(x) de un grado no mayor que tres que satisfaga las condiciones
p(2) = 1, p(1) = 6, p(0) = 5, p(1) = 10.
4. Gracar las siguientes expresiones:
a) f(x) =
x
3
+ 1
x
2
x 2
;
b) f(x) =
(x 1)
5
(x 2)
4
;
c) f(x) =
_
x + 1
x 1
_
4
;
d) f(x) =
x
3
2x
2
x
2
x 3
;
e) f(x) =
3x 2
2x
2
+ 3x 9
;
f ) f(x) =
x
3
+ 2x
2
(x 1)
2
;
CAPTULO 8. FUNCIONES ALGEBRAICAS 442
g) f(x) = (x+1)
_
x 1
x 2
_
2
;
h) f(x) =
3x
2
+x 4
2x
2
+x 6
;
i) f(x) =
x
5
x
4
1
;
j) f(x) =
_
x
2
+x 6
x 3
;
k) f(x) =
_
x
2
+ 3x + 2
x
2
4x + 3
;
l) f(x) = (x
2
1)

x + 1;
m) f(x) =
x
3
+ 2x
2
(x 1)
2
;
n) f(x) =
3x 2
2x
2
+ 3x 9
;
o) f(x) =
x + 8

x
2
+ 4x + 16
;
p) f(x) =
2x
2
5x + 2
3x
2
+ 7x 6
;
q) f(x) =
x
2
+x 6
x 3
;
r) f(x) =
x
2
x
2
+ 3x 4
;
s) f(x) =
x
2
+ 2x 3
x
2
x 2
;
t) f(x) =
x
3
x
x
2
4
;
u) f(x) =
x
2
+ 3x + 2
x
2
4x + 3
;
v) f(x) =
2x
2
x 15
2x
2
3x 5
;
w) f(x) =
x
3
+x
2
2x
x
2
+x 6
;
x) f(x) =
(x 1)
3
(x 2)
2
;
y) f(x) =
x
2
+ 4x + 3
x
3
2x
2
5x + 6
;
z) f(x) =
x
2
x
x + 2
.
5. Gracar las siguientes expresiones:
a) f(x) =
x
2
+ 3x + 2

x
2
1
;
b) f(x) =

x
2
1
2x 1
;
c) f(x) =
x
3

x + 1
;
d) f(x) =
x + 8

x
2
+ 4x + 16
;
e) f(x) =
_
x
4
x
2
1
;
f ) f(x) =
4x

x
2
+ 1

x
2
;
g) f(x) =
_
3x
2
4
x
3
;
h) f(x) =
3
_
(3x 2)
2
x 1
;
i) f(x) =
_
x
3
2x
2
x
2
x 3
;
j) f(x) =
x
3
_
(x 2)
2
;
k) f(x) =
_
x
2
+ 2x 3
x
2
x 2
;
l) f(x) =

_
x + 1
x + 2
_
2
;
m) f(x) =

_
1 +[x 2[
1 +[x[
;
n) f(x) =
4
_
[x 1[
x 2
;
o) f(x) =
_
[x[ 1
x 2
;
p) f(x) =

3
(3x 2)
2
x 1
;
q) f(x) =
x
3

x + 1
;
r) f(x) =
x + 2

x
2
+ 2
;
s) f(x) =
_
3x
2
4
x
2
;
t) f(x) =
3x 2

x
2
1
;
u) f(x) =

4x
2
1
x
;
v) f(x) =

2x
2
+ 9
x + 1
;
w) f(x) =

x
2
4x
2 x
.
Captulo 9
Funciones exponenciales y
logartmicas
9.1. Expresiones exponenciales y logartmicas
Analicemos los problemas principales que surgen al estudiar las potencias:
1. Sean dados los nmeros reales a y k. Hllese un nmero real x tal, que x = a
k
. Este es
un problema de elevacin de un nmero real a potencia. Es resoluble para cualquier nmero
positivo a y cualquier nmero real k. Si a = 0 y k > 0, entonces x = 0.
2. Sean dados los nmeros reales b y k. Hllese un nmero real x tal, que se verique x
k
= b.
Si b es nmero positivo cualquiera y k es cualquier nmero real distinto de cero, el problema
se reduce al anterior, pues la respuesta la da el nmero x = b
1
k
. En efecto,
x
k
=
_
b
1
a
_
a
= b
1
k
k
= b
1
= b.
Si k = 0 y b = 1, entonces la solucin de este problema es un nmero real x distinto de cero.
Si k = 0 y b ,= 1, este problema no tiene solucin.
3. Sean dados los nmeros reales a y b. Hllese un nmero real x tal, que se verique a
x
= b.
Se estudiar este problema slo para a y b reales y positivos. Si a = 1 y b = 1, a ttulo de
solucin de este problema interviene cualquier nmero real x. Si a = 1 y b ,= 1, el problema
no tiene solucin. Analicemos el caso en que a ,= 1.
Teorema 9.1 Para todo par de nmeros reales a y b tales, que a > 0, a ,= 1, y b > 0, existe un
nmero real, y slo uno, x tal, que a
x
= b.
Demostracin
Supongamos que existen unos nmeros reales x
1
y x
2
tales, que a
x
1
= b y a
x
2
= b. Segn la
propiedad de transitividad de las igualdades tenemos a
x
1
= a
x
2
, implica que x
1
= x
2
, lo que se
trataba de demostrar.
Denicin 9.1 Logartmo de un nmero
Si a > 0, a ,= 1 y b > 0, un nmero real k recibe el nombre de logaritmo del nmero b de base a y
se denota k = log
a
b, si a
k
= b.
443
CAPTULO 9. FUNCIONES EXPONENCIALES Y LOGARTMICAS 444
El logaritmo se dene solamente para un nmero positivo de base positiva y distinta de la uni-
cidad, es decir, para cualquier a 0, a = 1 y para todo b 0 el concepto de logaritmo est privado
de sentido. As pues, en la denicin de logaritmo log
a
b tenemos siempre a > 0, a ,= 1, b > 0. De
la denicin de logaritmo se deduce la identidad logartmica fundamental a
log
a
b
. Haciendo uso de
la denicin de logaritmo, obtenemos log
a
a = 1, y log
a
1 = 0. Teniendo en cuenta la unicidad del
logaritmo podemos constatar que si c > 0 y c ,= 1, entonces siempre log
a
c ,= 0.
Procedemos a considerar las propiedades ms importantes del logaritmo:
Teorema 9.2 Suponga que los nmeros M, N y a son tales que M > 0, N > 0, a > 0 y a ,= 1.
Entonces
log
a
MN = log
a
M + log
a
N.
Demostracin
Examinemos
a
log
a
MN
= MN
= a
log
a
M
a
log
a
N
= a
log
a
M+log
a
N
.
As pues,
a
log
a
MN
= a
log
a
M+log
a
N
.
Al aplicar a la ltima igualdad, las propiedades de las potencias, obtenemos
log
a
MN = log
a
M + log
a
N.
Teorema 9.3 Suponga que los nmeros M, N y a son tales que M > 0, N > 0, a > 0 y a ,= 1.
Entonces
log
a
M
N
= log
a
M log
a
N.
Teorema 9.4 Suponga que los nmeros M, a, k son tales que M > 0, a > 0 y a ,= 1, mientras
que k es un nmero real cualquiera. Entonces
log
a
M
k
= k log
a
M.
Demostracin
Examinemos
a
log
a
M
k
= M
k
=
_
a
log
a
M
_
k
= a
k log
a
M
.
As pues,
a
log
a
M
k
= a
k log
a
M
.
Al aplicar a la ltima igualdad, las propiedades de las potencias, obtenemos
log
a
M
k
= k log
a
M.
CAPTULO 9. FUNCIONES EXPONENCIALES Y LOGARTMICAS 445
Teorema 9.5 Suponga que los nmeros M, a, k y r son tales que M > 0, a > 0 y a ,= 1,
mientras que k y r son nmeros reales cualesquiera (r ,= 0). Entonces
log
a
r M
k
=
k
r
log
a
M.
Demostracin
Examinemos
(a
r
)
log
a
r M
k
= M
k
=
_
a
log
a
M
_
k
= a
k log
a
M
=
_
(a
r
)
1
r
_
k log
a
M
= (a
r
)
k
r
log
a
M
.
As pues,
(a
r
)
log
a
r M
k
= (a
r
)
k
r
log
a
M
.
Aplicando a la ltima ecuacin las propiedades de las potencias, obtenemos
log
a
r M
k
=
k
r
log
a
M.
Teorema 9.6 Suponga que los nmeros M, a, b son tales que M > 0, a > 0, b > 0, a ,= 1,
b ,= 1. Entonces
log
b
M =
log
a
M
log
a
b
.
Demostracin
Examinemos
a
log
a
M
= M
= b
log
b
M
=
_
a
log
a
b
_
log
b
M
= a
log
a
b log
b
M
.
As pues,
a
log
a
M
= a
log
a
b log
b
M
.
Aplicando a la ltima ecuacin las propiedades de las potencias, obtenemos
log
a
M = log
a
b log
b
M.
Conforme a la propiedad de las igualdades, ambos miembros de esta igualdad podemos multi-
plicarlos por
1
log
a
b
(puesto que b ,= 1, tenemos log
a
b ,= 0) y convencernos de que es vlida la
igualdad
log
b
M =
log
a
M
log
a
b
.
CAPTULO 9. FUNCIONES EXPONENCIALES Y LOGARTMICAS 446
Teorema 9.7 Suponga que los nmeros M, N, a, son tales que M > 0, N > 0, a > 0, a ,= 1.
Entonces
log
a
M = log
a
N M = N.
Demostracin
De acuerdo con la identidad logartmica fundamental, tenemos
M = a
log
a
M
y N = a
log
N
por consiguiente
M = N a
log
a
M
= a
log
a
N
(1)
Segn las propiedades de las potencias, tenemos
a
log
a
M
= a
log
a
N
log
a
M = log
a
N (2)
De (1) y (2) se deduce que
M = N log
a
M = log
a
N.
Ejemplo 9.1 Simplique la expresin:
_

2
_
3log

2
52log

2
25log

2
10+2log

5
.
Solucin
A =
_

2
_
3log

2
52log

2
5
2
log

2
10+2log

2
5
1
2
=
_

2
_
3log

2
54log

2
5log

2
10+log

2
5
=
_

2
_
log

2
10
=
1
10
.
Ejemplo 9.2 Simplique la expresin:
4
3log
4
2
(1, 5)
log3
2
31
.
Solucin
A = 4
log
4
2
3

_
3
2
_
log3
2
3log3
2
3
2
= 2
3

_
3
2
_
log3
2
2
= 2
3
2 = 8 2 = 6.
Ejemplo 9.3 Simplique la expresin:
log
2
3

16 + log
8
4

2 log
3
27

3 log
5
_
5

5
Solucin
A = log
2
2
4
3
+ log
2
32
1
4
log
3
3
7
2
log
5
5
3
4
=
4
3
log
2
2 +
1
4

1
3
log
2
2
7
2
log
3
3
3
4
log
5
5
=
4
3
1 +
1
4

1
3
1
7
2
1
3
4
1
=
4
3
+
1
4

1
3

7
2

3
4
=
17
6
.
CAPTULO 9. FUNCIONES EXPONENCIALES Y LOGARTMICAS 447
Ejemplo 9.4 Simplique la expresin:
9
2log
3
2+4log
81
2

_
3
2+
1
2
log
3
16
Solucin
A = [9
2log
3
2+4log
3
42

_
3
2+
1
2
log
3
2
4
= 9
2log
3
2+log
3
2

_
3
2+2log
3
2
=
_
3
2
_
3log
3
2
3
1+log
3
2
= 3
6log
3
2
3
1+log
3
2
= 3
7log
3
2
3 = 3
log
3
2
7
3 = 2
7
3 = 384.
Ejemplo 9.5 Simplique la expresin:
72 log
2
_
1
5
log
25
3

2 + 10 log
2
_
5

8
2
_
.
Solucin
A = 72 log
2
5

1
2
log
5
22
1
3
+ 10 log
2
2

2
5
= 72
_

1
2
_
log
2
5
1
3

1
2
log
5
2 + 10
_

2
5
_
log
2
2
= 36log
2
5
1
6
log
5
2 + 10
_

2
5
_
log
2
2
= 6 log
2
5 log
5
2 4 = 6 4 = 10.
Ejemplo 9.6 Simplique la expresin:
log
2
_
3

4
_
2
5

16

2
_
log1
2
3

2
+ log 1

3
9
3

3.
Solucin
A = log
2
_
2
2
3
2
9
10
2
1
2
_
+ log
2
1
_
2
2
2
1
2
_
1
3
+ log
3

1
2
_
3
2
3
1
3
_
= log
2
2
16
15
+ log
2
2
1
2
2
7
3
log
3
3 =
16
15
log
2
2 +
1
2
log
2
2
14
3
=
16
15
+
1
2

14
3
=
31
10
.
Ejemplo 9.7 Simplique la expresin:
72
_
49
1
2
log
7
9log
7
6
+ 5
log

5
4
_
.
Solucin
A = 72
_
_
7
2
_
log
7

9log
7
6
+ 5
log
5
1
2
2
2
_
= 72
_
7
2log
7
1
2
+ 5
4log
5
2
_
= 72
_
7
log
7
(
1
2
)
2
+ 5
log
5
2
4
_
= 72
_
1
4
+ 2
4
_
= 72
5
16
=
45
2
.
CAPTULO 9. FUNCIONES EXPONENCIALES Y LOGARTMICAS 448
Teorema 9.8 Suponga que los nmeros M, N, a, son tales que M > 0, N > 0, a > 1. Entonces,
si la base es mayor que la unidad, al menor de dos nmeros positivos le corresponde el logaritmo
menor y al menor logaritmo le corresponde el nmero menor. Es decir:
log
a
M < log
a
N M < N.
Demostracin
De acuerdo con la identidad logartmica fundamental, tenemos
M = a
log
a
M
y N = a
log
N
por consiguiente
M < N a
log
a
M
< a
log
a
N
(3)
Segn las propiedades de las potencias, tenemos
a
log
a
M
< a
log
a
N
log
a
M < log
a
N (4)
De (3) y (4) se deduce que
M < N log
a
M < log
a
N.
Teorema 9.9 Suponga que los nmeros M, a, r son tales que M > 0, 0 < a < 1. Entonces,
si la base es mayor que la unidad, al menor de dos nmeros positivos, le corresponde el logaritmo
mayor y al logaritmo mayor le corresponde el nmero menor. Es decir:
log
a
M < log
a
N M > N.
Teorema 9.10 Suponga que los nmeros M, a, r son tales que M > 0, a > 0, a ,= 1, mientras
que r es un nmero real cualquiera (r ,= 0). Entonces
log
a
r M
r
= log M.
Teorema 9.11 Suponga que los nmeros a, b son tales que, a > 0, b > 0, a ,= 1, b ,= 1. Entonces
log
a
b = log
b
a = 1.
Denicin 9.2 Logartmos de base 10 y base e
Los logaritmos de base 10 se denominan decimales y en lugar de la designacin log
10
M se es-
cribe logM. Los logaritmos de base e (e es un nmero irracional, cuyo valor aproximado es
2,718281828459045...) se denominan naturales, y en lugar de la designacin log
e
N se escribe lnN.
Ejemplo 9.8 Demustrese que si a, b y c son nmeros reales que satisfagan la condicin
0 < b c < a 1, entonces se verica la desigualdad
log
a
(a +b) < log
(ac)
a.
Solucin
Por cuanto a > 0 y c b > 0, resulta evidente la validez de la desigualdad
a
2
(c b)a bc < a
2
,
la cual puede ser escrita de la manera siguiente:
(a +b)(a c) < a
2
. (5)
CAPTULO 9. FUNCIONES EXPONENCIALES Y LOGARTMICAS 449
Como a > 1, podemos aprovechar una de las propiedades y obtener la desigualdad
log
a
(a +b)(a c) < 2, (6)
que es equivalente a la desigualdad (5).
Haciendo uso de la propiedad 1, obtenemos la desigualdad
log
a
(a +b) + log
a
(a c) < 2, (7)
que es equivalente a la desigualdad (6).
Cada sumando en el primer miembro de la desigualdad (5) es positivo, puesto que a + b > 1 y
ac > 1. Por consiguiente, elevando al cuadrado los miembros primero y segundo de la desigualdad
(7), obtenemos una desigualdad equivalente.
Por eso, la desigualdad (7) es equivalente a la desigualdad
[log
a
(a +b) + log
a
(a c)]
2
< 4,
la cual es equivalente a la desigualdad siguiente
[log
a
(a +b) log
a
(a c)]
2
< 4 4 log
a
(a +b) log
a
(a c). (8)
La desigualdad (8) es equivalente a la desigualdad (5), la cual es verdadera, por consiguiente, ser
verdadera tambin la desigualdad (8).
Dado que, para b > 0, c > 0, se verica la desigualdad
0 < [log
a
(a +b) log
a
(a c)]
2
, (9)
podemos valernos de la propiedad de transitividad de las desigualdades.
En este caso la validez de las desigualdades (8) y (9) predetermina la validez de la desigualdad
0 < 4 4 log
a
(a +b) log
a
(a c),
que puede ser escrita en la forma
log
a
(a +b) log
a
(a c) < 1.
Al aplicar la propiedad correspondiente y al tener presente que a c > 1 y a > 1, concluimos que
la desigualdad de partida es verdadera.
9.2. Tarea
1. Utilizando las propiedades de los logaritmos, simplique la expresin 49
1
1
4
log
7
25
.
2. Calcular log 25, si log 2 = a.
3. Calcular log
3
18, si log
3
12 = a.
4. Calcular log
49
16, si log
14
28 = a.
CAPTULO 9. FUNCIONES EXPONENCIALES Y LOGARTMICAS 450
5. Calcular log
12
60, si log
6
30 = a, log
15
24 = b.
6. Calcular log 1250, si log 2 = 0, 3010.
7. Calcular log
100
40, si log
2
5 = a.
8. Calcular log
6
16, si log
12
27 = a.
9. Calcular log
3
5, si log
6
2 = a, log
6
5 = b.
10. Calcular log
35
28, si log
14
7 = a, log
14
5 = b.
11. Calcular log
6
3, 38, si log 2 = a, log
13
= b.
12. Calcular log
2
360, si log
3
20 = a, log
3
15 = b.
13. Calcular log
12
60, si log
6
30 = a, log
15
24 = b.
14. Demuestre las identidades:
a) log
ab
n =
log
a
nlog
b
n
log
a
n + log
b
n
; b)
log
a
n
log
ab
n
= 1 + log
a
b; c) log
bn
an =
log
b
a + log
b
n
1 + log
b
n
.
15. Utilizando las propiedades de los logaritmos, simplique las expresiones:
a) log1
3

9 + log
3

1
3
9 log1
8
4

32 + log 1

2
3
_
128

2;
b) log
3
27 log

3
27 log1
3
27 log

3
2
_
64
27
_
;
c) 2 log
5
4

5 +
1
2
log

5
25 log
2
5

5 2;
d) log
2
5

5 log 5

5
_
5

5
_
+ log
(1+

3)
_
4 + 2

3
_
;
e)
_
log

5
_
1
5
__ _
log1
5
_
5

5
_
+ log

5
_
5

5
_
;
f ) log
0,4
_
1
5
3

50
_
+ log
0,6
_

15
5
_
+ log
0,32
_
2

2
5
_
;
g)
_
log

5
125 log
2
5
25
_
_
log1
5

5 log
0,2
3

25
_
;
h) (0, 1)
2 log0,11,5 log0,1
(0, 1)
[log(
8
3
)+2log20]
;
i)
_
log1
2
_
1
4
+ 6 log1
4
_
1
2
_
2 log 1
16
_
1
4
_
_
log

2
5

8;
j)
log
2
3
_
2
3
log
2
2

2 log

7
3
_
2
3
log

7
log 6

7
3
_
2
3
log

7
3
_
2
3
;
CAPTULO 9. FUNCIONES EXPONENCIALES Y LOGARTMICAS 451
k)
_
log
8
27 log
0,5
_
1
3
__ _
log
3
12
log
36
3

log
3
4
log
108
3
_
;
l)
log

2
16
log
4

2
_
log

2
_
2
4

2
_
+ 100
1
2
log82 log2
_
.
16. Utilizando las propiedades de los logaritmos, simplique la siguiente expresin:
a) 10
1
2
log9log5+log2
7
log
3

3
27
;
b) 3
1+log
3
4
+ 2
log
2
32
;
c) 2
log
5
3

_
1
3
_
1log
5
2,5
log
9
2 log
4
81;
d) 3
2
5
log
3
32
1
3
log
3
64+log
3
10
;
e) 2
3log
4
3
+ 7
1+2 log
7
2
;
f ) (0, 2)
1
2
(9 log
0,2
23 log
0,2
4)
.
17. Utilizando las propiedades de los logaritmos, simplique la siguiente expresin:
a)

_
log

3
4

3
+ log 3

2
;
b) log
2
_
1
4

4
_
+ log
3
_
3
_
3

3
27
_
+ log
4
_
3

8
128

2
_
log
7
_

7
3

49
_
;
c)
log
3
81
log
3
9
_
36
1log
6
2
+ 49
log
7
6
_
;
d) 3
1
5 log
5
3
(log2 + log5 + log300 log3);
e)
1
2
_
9
1+log
25
5
3
2(
1
4
+log
16
2)
_
log

2
_
2

2
_
;
f )

_
log

2
4

_
2
_

2 + log

2
4
_
_
2

2.
9.3. Ecuaciones exponenciales y logartmicas
Denicin 9.3 Ecuacin exponencial
Sea a un nmero positivo jo distinto de la unidad, entonces, la ecuacin a
x
= b se denomina
ecuacin exponencial elemental.
El dominio natural de denicin para la funcin y = a
x
es el conjunto de todos los nmeros
reales. Es decir, es el intervalo (; +). La funcin y = a
x
es estrictamente montona en el
conjunto (; +) y el codominio esta representado por el intervalo (0; +). Por consiguiente,
la ecuacin a
x
= b no tiene races, cuando cada b es no positivo, mientras que para cada b positivo
la ecuacin a
x
= b tiene una raz nica que se denotar x
1
. Por cuanto x
1
es la raz de la ecuacin
a
x
= b, es vlida la igualdad numrica a
x
1
= b, la cual es equivalente a la igualdad numrica
x
1
= log
a
b.
As pues, para cada b no positivo la ecuacin a
x
= b no tiene races, y para cada b positivo
tiene una raz nica x
1
= log
a
b.
En la siguiente tabla se exponen los resultados de la resolucin de la ecuacin a
x
= b.
CAPTULO 9. FUNCIONES EXPONENCIALES Y LOGARTMICAS 452
b > 0 b = 0 b < 0
a
x
= b x
1
= log
a
b No hay solucin No hay solucin
Sea a un nmero positivo jo distinto de la unidad, entonces la ecuacin log
a
x = b se denomina
ecuacin logartmica elemental.
El dominio natural de la funcin y = log
a
x es el conjunto de todos los nmeros positivos. Es
decir, es el intervalo (0; +). La funcin y = log
a
x es estrictamente montona en el conjunto
(0; +) y el codominio est representado por toda la recta numrica (; +).
Por eso, para todo b la ecuacin log
a
x = b tiene una raz nica que se denotar con x
1
. Por
cuanto x
1
es la raz de la ecuacin log
a
x = b, es vlida la igualdad numrica log
a
x = b, la cual es
equivalente a la igualdad numrica x
1
= a
b
. Por consiguiente para cada b la ecuacin log
a
x = b
tiene la raz nica x
1
= a
b
.
En la siguiente tabla se exponen los resultados de la resolucin de la ecuacin log
a
x = b.
b > 0 b = 0 b < 0
log
a
x = b x
1
= a
b
x
1
= 1 x
1
= a
b
Existen ecuaciones que no se resuelven aplicando solamente transformaciones equivalentes; al
resolver las ecuaciones mucho ms a menudo tenemos que aplicar transformaciones no equivalentes.
Sea a un nmero jo tal, que a > 0 y a ,= 1:
1. f(x) = a
log
a
f(x)
;
2. log
a
f
2
(x) = 2 log
a
f(x);
3. log
a
f
2
(x) = 2 log
a
[f(x)];
4. log
a
[f(x)g(x)] = log
a
f(x) + log
a
g(x);
5. log
a
f(x)g(x) = log
a
[f(x)] + log
a
[g(x)];
6. log
a
_
f(x)
g(x)
_
= log
a
f(x) log
a
g(x);
7. log
a
_
f(x)
g(x)
_
= log
a
[f(x)] log
a
[g(x)].
Sea a un nmero jo positivo cualquiera, distinto de la unidad. Sea dada la ecuacin log
a
f(x) =
log
a
g(x). La sustitucin de esta ecuacin por la ecuacin f(x) = g(x) se denomina potenciacin
de la ecuacin.
Al realizar la potenciacin de una ecuacin, no se pueden perder races, sino slo adquirir ex-
traas. Por esta razn, si al resolver una ecuacin, se realiza la potenciacin, resulta necesaria la
comprobacin al nal de la resolucin.
CAPTULO 9. FUNCIONES EXPONENCIALES Y LOGARTMICAS 453
Ejemplo 9.9 Resuelva las siguientes ecuaciones:
a) 9
x

_
1
3
_
23x
=

27
x

81
x+3
; b) 5
x+3
2 5
x+2
= 375;
c) 3 4
x2
= 2
_
256 16
x+1
2
_
.
Solucin
a) 3
2x

_
1
3
_
23x
= 3
3x
2
3
4(x+3)
3
3
2x
3
3x2
= 3
3x
2
3
4(x+3)
3
3
5x2
= 3
17x+24
6
5x 2 =
17 + 24
6
13x 36 = 0 x =
36
13
.
b) 5 5
x+2
2 5
x+2
= 375 (5 2)5
x+2
= 375 5
x+2
= 125
5
x+2
= 5
3
x + 2 = 3 x = 1
c) 3 4
x

1
16
= 512 2 4
x
4 4
x
_
3
16
+ 8
_
= 512 4
x
=
16 512
131
xln 4 = ln
_
8192
131
_
x = ln
4
_
8192
131
_
.
Ejemplo 9.10 Resuelva las siguientes ecuaciones:
a) log
0,5
(x + 1) log
0,5
(x 3) = 1; b) log
4
(x
2
1) log
4
(x 1)
2
= log
4
_
(2 x)
2
.
Solucin
a) log1
2
_
x + 1
x 3
_
= 1
x + 1
x 3
=
1
2
2x + 2 = x 3 x = 5.
La raz obtenida no es solucin de la ecuacin.
b) log
4
x
2
1
(x 1)
2
= log
4
_
(2 x)
2

x
2
1
(x 1)
2
=
_
(2 x)
2
x
2
1
(x 1)
2
= (2 x)
(x 1)(x + 1)
(x 1)
2
= (2 x)
x + 1
x 1
= (2 x)
x + 1 = (2 x)(x 1)
_
x + 1 = (2 x)(x 1)
x + 1 = (2 x)(x 1)

_
x
2
2x + 3 = 0
x
2
4x + 1 = 0
Este sistema de ecuaciones tiene por solucin x = 2 +

3.
9.4. Tarea
1. Resuelva las ecuaciones:
a) 4
2x
= 32
x2
;
b) 4
x
+ 64 = 16 2
x
;
c) 3
x1
+ 3
x
+ 3
x+1
= 39;
d) 3
2x+1
= 81;
e) 2
x+1
2
x
= 64;
f ) 2
x+1
+ 2
x
= 15;
g) 3
2x
5 3
x
= 6;
h) 3
x+4
+ 2 3
x1
= 2205;
i) 7
x+2
7
x+1
+ 7
x
= 43;
j) 5
x+1
+ 5
x
+ 5
x2
151 = 0;
k) 2
x+2
+ 2
x1
2
x3
= 70;
l) 5
2x+1
+ 9 5
x
2 = 0;
m) 9
x
2 3
x+2
+ 81 = 0;
n) 7
2x+3
8 7
x+1
= 1;
o) 4
x
3 2
x+1
+ 8 = 0;
p) 4
2x1
= 13 4
x2
21;
q) 2
2x+6
+ 4(4
2x
8
x+1
) = 0;
r) 3 13
x
+ 13
x+1
2
x+2
= 5 2
x+1
;
CAPTULO 9. FUNCIONES EXPONENCIALES Y LOGARTMICAS 454
s)
_
4
7
_
x
_
7
4
_
3x1

16
49
= 0;
t) 5
x
32 =
108 139 5x
25
x
;
u)
_
5
9
_
2x7
=
3

_
5
9
_
13x
;
v)
4

512 = 8
3x
2
1x
;
w) 6
x
2
x1
3
x2
= 2

5
log
5
289
;
x) 2
x
(2
3x
2x + 2) = 2 8
x
1;
y) 4
3x
2
+x
8 = 2 8
x
2
+
x
3
;
z) 3

2
x31
5

2
x35
32 = 0.
2. Resuelva las ecuaciones:
a) 3
x
2
4
= 5
2x
;
b) 5
1+2x
+ 6
1+x
= 30 + 150
x
;
c) 4
x
+ 2
x+1
24 = 0;
d) 2
x
+ 0, 5
2x3
6 0, 5
x
= 1;
e) 6 3
2x
13 6
x
+ 6 2
2x
= 0;
f )
_
3
5
_
x
+
7
5
= 2
x
;
g) (x
2
+x 57)
3x
2
+3
= (x
2
+x 57)
10
x
;
h)
_
2
3
_
x
_
9
8
_
x
=
27
64
;
i) 2
x
2
5
x
2
= 0, 001(10
3x
)
2
;
j)
_
3
4
_
x1
_
4
3
_
1/x
=
9
16
;
k) 0, 6
x
_
25
9
_
x
2
12
=
_
27
125
_
3
;
l)
_
2x
3
_
4
x
0, 125
1/x
= 4
3

2;
m) 10
x
5
x1
2
x2
= 950;
n) 2
3x
3
x
2
3x1
3
x+1
= 288;
o) 2 7
3x
5 49
3x
+ 3 = 0;
p) 3 5
2x1
2 5
x1
= 0, 2;
q) 9
x
2
1
36 3
x
2
3
+ 3 = 0;
r)
2
x
+ 10
4
=
9
2
x2
;
s) 3
3x+1
4 27
x1
+ 9
1,5x1
80 = 0;
t) 4

x+1
2

x+1+2
= 0;
u) 2
x+

x
2
4
5 (

2)
x2+

x
2
4
6 = 0;
v) 5
2x
7
x
35 5
2x
35 7
x
= 0;
w) 4
x
3
x0,5
= 3
x+0,5
2
2x1
;
x) (2 +

3)
x
+ (2

3)
x
= 4;
y) 4
x
+ 6
x
= 2 9
x
;
z) 2 4
x
+ 25
x+1
= 15 10
x
.
3. Resuelva las ecuaciones:
a) 16
x
+ 36
x
= 2 81
x
;
b) 56 4
x1
53 14
x
+ 2 49
x+0,5
= 0;
c) 2
2x
9
x
2 6
3x1
+ 4
2x1
3
4x2
= 0;
d) 2
x
2 0, 5 2
x
0, 5
x
+ 1 = 0;
e) 27 2
3x
+ 9 2
x
2
3x
27 2
x
= 8;
f ) 3
x
8
x
x+2
= 6;
g) 5
x2
2
3x
x+1
= 4;
h) (x
2
x 1)
x
2
1
= 1;
i) [x[
x
2
2x
= 1;
j) (x 2)
x
2
x
= (x 2)
12
;
k) (3x 4)
2x
2
+2
= (3x 4)
5x
;
l) 3
x
+ 4
x
= 5
x
;
m) 8 x 2
x
+ 2
3x
x = 0.
4. Resuelva las ecuaciones:
a) 3

x
2
+2
3

x
2
+1
3

x
2
1
= 68;
b) 2 5
x+1

1
5
4
x+2

1
3
5
x+2
= 3 4
x1
;
c) 3(10
x
6
x+2
) + 4 10
x+1
= 5(10
x1
+ 6
x1
);
d) (2 +

3)
x
2
2x+1
+ (2

3)
x
2
2x1
=
101
10(2

3)
;
e) (2 +

3)
x
2
2x+1
+ (2

3)
x
2
2x1
=
4
2

3
;
CAPTULO 9. FUNCIONES EXPONENCIALES Y LOGARTMICAS 455
f )

x(9

x
2
3
3

x
2
3
) = 3
2

x
2
3+1
3

x
2
3+1
+ 6

x 18;
g) 2

x 4
x
+ 5 2
x+1
+ 2

x = 2
2x+2
+ 5

x 2
x
+ 4;
5. Resuelva las ecuaciones:
a) log(5 x) + log(2x 3) = log 5;
b) 3 log 125 = (x
2
5x + 9) log 2;
c) log x + log 50 = log 1000;
d) log x + log(x + 20) = 2;
e) log x = 1 + log(22 x);
f ) log(54 x
3
) = 3 log x;
g) log
x
(2x
2
7x + 12) = 2;
h) log
x
(2x
2
4x + 3) = 2;
i) log
3
(3
x
1) log
3
(3
x+1
3) = 6;
j) x + log(1 + 2x) = xlog 5 + log 6;
k) log
2
(4x + 4) = x + log
2
(2
x+1
3);
l) log
(x6)
2(x
2
5x + 9) =
1
2
;
m) log(2x 3)
2
log(3x 1)
2
= 2;
n) log
2
(x + 4) = (log
2
7 log
2
5) log
2
4;
o) log1
2
(4 x) = log1
2
2 log1
2
(x 1);
p) log
9
(x
2
+ 2x 3) = log
9
x 1
x + 3
;
q) log
4
_
(x 1)
log
4
(x1)
2
_
= 2;
r)
log 2 + log(4 5x 6x
2
)
log(2x 1)
= 3;
s)
_
1 + log
2
x +
_
4 log
4
x 2 = 4;
t) log

x
2
x 1
x
2
+x 2

= 0;
u)
_
log
2
x
4
+ 4 log
2
4
_
2
x
= 2;
v)
log(3x 5)
3x
2
+ 25
=
1
2
;
w)
log(2x 5)
x
2
8
=
1
2
;
x) log
3
(x
2
3x 5) = log
3
(7 2x);
y) log(x + 4) + log(2x + 3) = log(1 2x);
z) log
2
(x
2
1) = log
1/2
(x 1).
6. Resuelva las ecuaciones:
a) log
x+4
(x
2
1) = log
x+4
(5 x);
b) log
2
x + log x + 1 =
7
log
x
10
;
c) log
2
x
3
log(0, 1x
10
) = 0;
d) (1 log 2) log
5
x = log 3 log(x 2);
e) log(20 x) = log
3
x;
f ) x
1log x
= 0, 01;
g) log
x
(3x
log
6
x
+ 4) = 2 log
5
x;
h) log
5
(5
1/x
+ 125) = log
5
6 + 1 +
1
2x
;
i) log
4
2
x 1
= log
4
(4 x);
j) log
3
[(x 1)(2x 1)] = 0;
k) log

2
x
2
4x + 3
4
= 2;
l) log(x + 1, 5) = log x;
m) log(4, 5 x) = log 4, 5 log x;
n) log
x
2(x + 2) = 1;
o) log

x 5 + log

2x 3 + 1 = log 30;
p) log
x2
(2x 9) = log
x2
(23 6x);
q) log
5x2
2 + 2 log
5x2
x = log
5x2
(x + 1);
r) log
5
(3x 11) + log
5
(x 27) = 3 + log
5
8;
s)
1 log x
x
=
log
2
14 log
2
4
log 3, 5
x
;
t) log
2
(x + 1)
2
+ log
2
_
x
2
+ 2x + 1 = 6;
u)
log(35 x
3
)
log(5 x)
= 3;
v)
log 2 + log(4 5x 6x
3
)
log
3

2x 1
= 3;
w) log
1/5
log
5

5x = 0;
x) log
1/2
log
8
x
2
2x
x 3
= 0;
y) log
4
log
2
log
3
(2x 1) =
1
2
;
z) log
4
(x + 12) log
x
2 = 1.
7. Resuelva las ecuaciones:
CAPTULO 9. FUNCIONES EXPONENCIALES Y LOGARTMICAS 456
a) x
2
log
x
27 log
9
x = x + 4;
b) 1 + log
x
4 x
10
= (log x
2
1) log
x
10;
c) 1 + 2 log
x
2 log
4
(10 x) =
2
log
4
x
;
d) log
x+
1
8
2 = log
x
4;
e) log
3
(x
2
8x 14) log
x
2
+4x+4
9 = 1;
f ) 0, 1 log
4
x log
2
x + 0, 9 = 0;
g)
1
5 4 log(x + 1)
+
5
1 + 4 log(x + 1)
= 2;
h) 4 log x = 3
_
log x;
i) log
2
(100x) + log
2
(10x) = 14 log x + 15;
j)
1 log
2
x
log x 2 log
2
x
= log x
4
+ 5;
k) log
x
5

5
5
4
= log
2
x

5;
l) log(log x) + log(log x
3
2) = 0;
m) log
x
2 + log
2
x = 2, 5;
n) log
2
1/2
4x + log
2
x
2
8
= 8;
o) log(x
2
8) log(2 x) =
log
5
(x
2
8)
log
5
(2 x)
;
p) (

x)
log
5
x1
= 5;
q) x
log
3
x+1
= 9x;
r) 2 log
x
3 + log
3x
3 + 3 log
9x
3 = 0;
s) log
x+1
_
x
1
2
_
= log
x
1
2
(x + 1);
t) log
3x+7
(5x + 3) + log
3x+3
(3x + 7) = 2;
u) 0, 4
log
2
x+1
= 6, 25
2log x
3
;
v) 1, 25
1log
2
2
x
= 0, 64
2 log
2
2x
;
w) x
log x
= 1000xr;
x)
_
x
log

x
= 10;
y) x
log

x
2x
= 4;
z) x
log x+7
4
= 10
log x+1
.
8. Resuelva las ecuaciones:
a) x
log
2
x
3
log
2
2
x+3

1
x
= 0;
b) log
x
(2x
x2
1) + 4 = 2x;
c) 15
log
5
3
x
log
5
9x+1
= 1;
d) 5
log x
3
log x1
= 3
log x+1
5
log x1
;
e) 2x
log x
+ 3x
log x
= 5;
f ) log
2
(9 2
x
) = 25
log
5

3x
;
g) [x 1[
log
2
xlog x
2
= [x 1[
3
;
h) 4
log
3
(1x)
= (2x
2
+ 2x + 5)
log
3
2
.
9. Resuelva las ecuaciones:
a) log
(x1)
2(x
2
4x + 4) = 2 + log
(x1)
2(x + 5)
2
;
b) log

1 +x + 3 log

1 x = log
_
1 x
2
+ 2;
c) log
_
3

4x+12
4

4x+1
_
= 2
_
x +
1
4
log 4 +
log 16
4
;
d) log
6
(x + 1) = log
6
(1 x) + log
6
(2x + 3);
e)
1
5 4 log(x + 1)
= 3
1
1 + log(x + 1)
;
f )

2
_
log
2
x
2
64
1
_
(2 + log
4
8x) = log
2
2x;
g) log
2x1
(6x
2
5x + 1) log
3x1
(4x
2
4x + 1) = 2;
h) log
5x1
(10x
2
7x + 1)
4
= 2 + log
2x1
(25x
2
10x + 1);
i) log(x
2
7x + 3) log(2x + 1) = log(x
2
+ 7x 3) log(2x 1);
j) log(x 2) + log(x 3) = 1 log 5;
k) log

5x 4 + log

x + 1 = 2 + log 0, 18.
l) log
0,5
x
2
14 log
16x
x
2
+ 40 log
4x

x = 0;
m) log

5x 4 + log

x + 1 = 2 + log 0, 18;
n) log(x
3
+ 27) 0, 5 log(x
2
+ 6x + 9) = 3 log
3

7;
o) log 5 + log(x + 10) = 1 log(2x 1) + log(21x 20);
CAPTULO 9. FUNCIONES EXPONENCIALES Y LOGARTMICAS 457
p) ) log
1/2

1 +x + 3 log
1/4
(1 x) = log
1/16
(1 x
2
)
2
+ 2;
q) log
2
log
3
(x
2
16) log
1/2
log
1/3
x
2
16 = 2;
r) 3 log
16
(
_
x
2
+ 1 +x) + log
2
(
_
x
2
+ 1 x) = log
16
(4x 1) 0, 5;
s) x
log
2
2
x
2
log
2
2x2
+ (x + 2)
log
(x+2)
2 4=3
= 3;
t) log(3
x
2
4x
) = 2 + 0, 25 log 16 0, 5xlog 4;
u) 3 log 2 + log(2

x11
1) = log(0, 4
_
2

x1
+ 4) + 1.
10. Resuelva los sistemas de ecuaciones:
a)
_
4
x
= 16y
2
x+1
= 4y
b)
_
log

2
(y x) = 4
3
x
2
y
= 576
c)
_
64
2x
+ 64
2y
= 40
64
x+y
= 12
d)
_
2

x+

y
= 512
log

xy = 1 + log 2
e)
_
2
x
2
y
= 24
x +y = 8
f )
_
3
x
2
y
2
= 77
3
x
2
2
y
2
2
= 7
g)
_
xy

x +y =
1
2

3
2
yx
(x +y) = 48
h)
_
y + log x =
2

ArcSen1
x
y
= 2
2 log
0,5
10
i)
_
2x
2
+y = 75
2 log x log y = 2 log 2 + log 3
j)
_
x
x+y
= y
xy
x
2
y =
2

_
ArcSen
1

2
+ArcCos
1

2
_
k)
_
4

x
=
_
1
2
_
2y
3
log
9
x
=
y
3
l)
_
25
2x
+ 25
2y
= 30
25
x+y
= 5

5
m)
_
2
x
3
y
= 12
2
y
3
x
= 18
n)
_
x
y2
= 4
x
2y3
= 64
o)
_
x
log
y
x
y = x
5/2
log
4
y
y
(y 3x) = 1
p)
_
_
3
2
_
xy

_
2
3
_
xy
=
65
36
xy x +y = 118
q)
_
2
x
+ 2
y
= 12
x +y = 5
r)
_
64
2x
+ 64
2y
= 12
64
x+y
= 4

2
s)
_
8
x
= 10y
2
x
= 5y
t)
_
2
x
9
y
= 648
3
x
4
y
= 432
u)
_
3
x
2
2y
= 77
3
x/2
2
y
= 7
v)
_
x
y+1
= 27
x
2y5
=
1
3
w)
_
x
x+y
= y
12
y
x+y
= x
3
x)
_
x

y
= y
y

y
= x
4
y)
_
log x + log y = log 2
x
2
+y
2
= 5
z)
_
log
y
x log
x
y =
8
3
xy = 16
11. Resuelva los sistemas de ecuaciones:
CAPTULO 9. FUNCIONES EXPONENCIALES Y LOGARTMICAS 458
a)
_
log(x
2
+y
2
) 1 = log 13
log(x +y) log(x y) = 3 log 2
b)
_
5(log
y
x + log
x
y) = 26
xy = 64
c)
_
2
x
4
y
= 32
log(x y)
2
= 2 log 2
d)
_
10
2log(xy)
= 25
log(x y) + log(x +y) = 1 + 2 log 2
e)
_
2
xy
2
(
4

2)
xy
= 12
3
log(2yx)
= 1
f )
_
3
x
2
y
= 576
log

2
(y x) = 4
g)
_
log
5
x + 3
log
3
y
= 7
x
y
= 5
12
h)
_
3(2 log
y
2 x log
1/x
y) = 10
xy = 81
i)
_
log
0,5
(y x) + log
2
1
y
= 2
x
2
+y
2
= 25
j)
_
3
yx
(x +y) =
5
27
3 log
3
(x +y) = x y
k)
_
x
y
= y
x
x
x
= y
9y
l)
_
log
4
xy +
3 log
4
x
log
4
y
= 0
log
4
x
y
log
4
x log
4
y = 0
m)
_
log
2
(x +y) + 2 log
3
(x y) = 5
2
x
5 2
0,5(x+y1)
+ 2
y+1
= 0
n)
_
log
2
(10 2
y
) = 4 y
log
2
x+3y1
8yx
= log
2
(x 1) log
2
(3 x)
o)
_
log x log(x +y) = log y log(x 1)
log y log(x +y) = log x log(x y)
p)
_
4
x+y
= 27 + 9
xy
8
x+y
21 2
x+y
= 27
xy
+ 7 3
xy+1
q)
_
x 2
x+1
2 2
y
= 3y 4
x+y
2x 2
2x+y
+ 3y 8
x+y
= 1
9.5. Desigualdades exponenciales y logartmicas
Denicin 9.4 Desigualdades potenciales
Sea a un nmero positivo jo distinto de la unidad, entonces las desigualdades
_
a
x
> b
a
x
< b
se denominan desigualdades potenciales elementales.
Si b es un nmero no positivo, entonces, tomando en consideracin que en el intervalo (; +)
la funcin es positiva, concluimos que el intervalo (; +) es el conjunto de todas las soluciones
de la desigualdad a
x
> b, mientras que la desigualdad a
x
< b no tiene soluciones. Si b es un nmero
positivo, se analizan dos casos:
CASO 1: Sea a > 1. En toda la recta numrica, es decir, en el intervalo (; +) la funcin
y = a
x
es creciente, por lo cual cada valor numrico de (0; +) ella lo toma una sola vez. Quiere
decir, si para x = x
0
(; +) la funcin toma el valor b, entonces para cada x > x
0
toma
un valor superior a b, y para cada x < x
0
, un valor inferior a b. Por consiguiente, en este caso el
conjunto de todas las soluciones de la desigualdad a
x
> b ser el intervalo (x
0
; +), y el conjunto
de todas las soluciones de la desigualdad a
x
< b, el intervalo (; x
0
), donde x
0
= log
a
b.
CASO 2: Sea 0 < a < 1. El intervalo (; +), es decir, en toda la recta numrica la
funcin y = a
x
decrece. Por eso, razonando de forma semejante, concluimos que en este caso el
conjunto de todas las soluciones de la desigualdad a
x
> b es el intervalo (; x
0
), y el conjunto
CAPTULO 9. FUNCIONES EXPONENCIALES Y LOGARTMICAS 459
de todas las soluciones de la desigualdad a
x
< b es el intervalo (x
0
; +), donde x
0
= log
a
b.
As pues, si a > 1, el conjunto de todas las soluciones de la desigualdad a
x
> b es:
1. El conjunto (log
a
b; +), para cada b positivo.
2. El conjunto (; +), para cada b no positivo; el conjunto de todas las soluciones de la
desigualdad a
x
< b es:
a) El conjunto (; log
a
b), para cada b positivo.
b) Un conjunto vaco, para cada b no positivo.
Si 0 < a < 1, el conjunto de todas las soluciones de la desigualdad a
x
> b es:
1. El conjunto (; log
a
b), para cada b positivo.
2. El conjunto (; +), para cada b no positivo; el conjunto de todas las solucio-nes de la
desigualdad a
x
< b es:
a) El conjunto (log
a
b; +), para cada b positivo.
b) Un conjunto vaco, para cada b no positivo.
En la siguiente tabla se dan los resultados que se obtienen al resolver las desigualdades a
x
> b
y a
x
< b.
b > 0 b = 0 b < 0
a
x
> b, a > 1 (log
a
b; +) (; +) (; +)
a
x
< b, a > 1 (; log
a
b) No hay soluciones No hay soluciones
a
x
> b, 0 < a < 1 (; log
a
b) (; +) (; +)
a
x
< b, 0 < a < 1 (log
a
b; +) No hay soluciones No hay soluciones
Ejemplo 9.11 Resuelva las inecuaciones:
a) 5
x
2
+3x
125 5
x
; b) 3

x+1
81
_
_
1
9
_
5
x
4
; c) 5

7x

7x+1

7x1
125

5.
Solucin
a) 5
x
2
+3x
5
3
5
x
5
x
2
+3x
5
x+3
x
2
+ 3x x + 3
x
2
+ 2x 3 0 (x + 3)(x 1) 0 x [3; 1].
b) 3

x+1
3
4

_
_
1
3
2
_
20x
4
3

x+1
3
4

_
1
3
_
20x
4
3

x+1
3
4
20x
4
3

x+1
3
x4
4

x + 1
x 4
4

_
x + 1 0
16(x + 1) (x 4)
2
_
x + 1 0
x(x 24) 0

_
x 1
x [0; 24]
La solucin de esta inecuacin se obtiene intersecando ambas soluciones parciales. Es decir: x
[0; 24].
c) 5
7x2

7x1

7x1
5
3
5
1
2
5
7x2

7x1

7x1
5
7
2

7x2

7x1

7x1

7
2
7x 2

7x + 5

7x 1
0
_

7
14
__

x
5

7
7
_

7
7
0
Por lo tanto, la solucin esta dada por: x
_
1
28
;
1
7
_

_
25
7
; +
_
.
CAPTULO 9. FUNCIONES EXPONENCIALES Y LOGARTMICAS 460
Denicin 9.5 Desigualdades logartmicas
Sea a un nmero positivo jo distinto de la unidad, entonces las desigualdades
_
log
a
x > b
log
a
x < b
se denominan desigualdades logartmicas elementales.
Como las propiedades de la funcin y = log
a
x, que se emplean al resolver estas desigualdades,
son diferentes para a > 1 y para 0 < a < 1, entonces tenemos dos casos:
CASO 1: Sea a > 1. En el intervalo (0; +) la funcin y = log
a
x es creciente, por lo cual
cada valor numrico ella lo toma una sola vez. Quiere decir, si para x = x
0
(0; +) la funcin
toma el valor b, entonces para cada x > x
0
tal, que x (0; +) la funcin toma un valor mayor que
b, y para cada x < x
0
tal, que x (0; +), ella toma un valor menor que b. Por consiguiente, en
este caso el conjunto de todas las soluciones de la desigualdad log
a
x > b es el intervalo (x
0
; +),
y el de todas las soluciones de la desigualdad log
a
x < b, el intervalo (0; x
0
), donde x + 0 = a
b
.
CASO 2: Sea 0 < a < 1. En el intervalo (0; +) la funcin y = log
a
x decrece. Por eso, ra-
zonando de forma semejante, concluimos que en este caso el conjunto de todas las soluciones de la
desigualdad log
a
x > b es el intervalo (0; x
0
) y el conjunto de todas las soluciones de la desigualdad
log
a
x < b, el intervalo (x
0
, +), donde x
0
= a
b
.
As pues, si a > 1, entonces para cada b el conjunto de todas las soluciones de la desigualdad
log
a
x > b se representa por el intervalo (a
b
; +), y el conjunto de todas las soluciones de la
desigualdad log
a
x < b, es el intervalo (; a
b
); si 0 < a < 1, entonces para cada b el conjunto de
todas las soluciones de la desigualdad log
a
x > b ser el intervalo (0; a
b
), y el conjunto de todas
las soluciones de la desigualdad log
a
x < b, es el intervalo (a
b
; +).
En la siguiente tabla se dan los resultados que se obtienen al resolver las desigualdades log
a
x > b
y log
a
x < b.
< b < +
log
a
x > b, a > 1 (a
b
; +)
log
a
x < b, a > 1 (0; a
b
)
log
a
x > b, 0 < a < 1 (0; a
b
)
log
a
x < b, 0 < a < 1 (a
b
; +)
Ejemplo 9.12 Resuelva las inecuaciones:
a) log
0,3
(x
2
+ 1) < log
0,3
(2x 5); b) 2 log
2
(x 1) log
2
(2x 4) > 1.
Solucin
a) Aplicando las propiedades de los logartmos, tenemos
log
0,3
(x
2
+ 1) log
0,3
(2x 5) < 0 log
0,3
x
2
+ 1
2x 5
< 0
x
2
+ 1
2x 5
< 1
x
2
+ 1
2x 5
1 < 0
x
2
2x + 6
2x 5
< 0 x
_
;
5
2
_
Otra condicin est dada por 2x 5 > 0, de donde x >
5
2
. Por tanto, la solucin de la inecuacin
es x = .
CAPTULO 9. FUNCIONES EXPONENCIALES Y LOGARTMICAS 461
b) Anlogamente al inciso anterior, tenemos
log
2
(x 1)
2
2x 4
> 1
(x 1)
2
2x 4
> 2
x
2
6x + 9
2x 4
> 0
(x 3)
2
2x 4
> 0 x (2; 3) (3; +)
Otras condiciones son las suientes:
x 1 > 0 x > 1, 2x 4 > 0 x > 2
Por tanto, la solucin est dada por: x (2; 3) (3; +).
9.6. Tarea
1. Resuelva las inecuaciones:
a) 5
|4x6|
25
3x4
;
b)
2
x+3
+ 11
2
2x+1
+ 2
x
5
< 3;
c)
4 7 5
x
5
2x+1
12 5
x
+ 4

2
3
;
d)
11 3
x1
31
4 9
x
11 3
x1
5
5;
e)
15 2 13
x+1
6 13
2x
13
x+1
+ 6
> 2;
f )
_
1
3
_

x+4
>
_
1
3
_

x
2
+3x+4
;
g)
1
3
x
+ 2

1
3
x+1
1
;
h) 4
x
< 3 2

x+x
+ 4
1+

x
;
i) 35
_
1
3
_
23x
< 6 + 3
43x
;
j) 5
2x103

x2
4 5
x5
< 5
1+3

x2
;
k) 5
2x+1
+ 6
x+1
> 30 + 5
x
30
x
;
l) 3 2
1x2

x
> 4
3x
2
+ 2
x

x
;
m) 2
2x+1
21
_
1
2
_
2x+3
+ 2 > 0;
n) 18 3
x+2

x+1
> 3
3x2

x+1
7 3
2x
;
o) 11
3x2
+ 13
3x2
13
3x1
11
3x1
.
2. Resuelva las inecuaciones:
a)
3
_
2x 1
2
x1
< 8
x3
3x7
;
b)
_
1
3
_
|x+2|
81;
c) 8
x
+ 18
x
2 27
x
> 0;
d) (x
2
+x + 1)
x
< 1;
e) 2
x
11 x;
f ) 6
3x
< 216;
g) 2
x
5
x
> 0, 1(10
x1
)
5
;
h)
1
3
x
+ 5
<
1
3
x+1
1
;
i) 0, 04
5xx
2
8
< 625;
j) 0, 5
x2
> 6;
k) 0, 4
x
2
1
> 0, 6
x
2
+6
;
l) 4
x+1,5
+ 9
x
< 9
x+1
;
m) 1 3
|x
2
x|
< 9;
n) [x 3[
2x
2
7x
> 1;
o) 8
x+1
8
2x1
> 30;
p) 2
2+x
2
2x
> 15;
q) 5
2x+1
> 5
x
+ 4;
r) 2
x
2
6x2,5
> 16

2;
s)
_
9
x
3
x+2
> 3
x
9;
t) 36
x
2 18
x
8 9
x
> 1;
u) 4
2x+1
+ 2
2x+6
< 4 8
x+1
;
v) 0, 3
2+4+6+...+2x
> 0, 3
72
;
w) 5
3

x
+ 5 < 5

x+1
+ 5

x
;
x) 25 2
x
10
x
+ 5
x
> 25;
y)

3
x64
7

3
x58
162;
z) (4x
2
+ 2x + 1)
x
2
x
> 1.
3. Resuelva las inecuaciones:
a) 2
x+2
2
x+3
2
x+4
> 5
x+1
5
x+2
;
b)
_
3
5
_
13x
2

_
3
5
_
x
4
+36
<
_
25
9
_
6x
2
;
c)
1
0, 5
x
1

1
1 0, 5
x+1
0;
d) 2
4x
2
3x+1
2
2x
2
x+1
2 0;
CAPTULO 9. FUNCIONES EXPONENCIALES Y LOGARTMICAS 462
e) 0, 008
x
+ 5
13x
+ 0, 04
3
2
(x+1)
< 30, 04;
f )
_
2(5
x
+ 24)

5
x
7

5
x
+ 7;
g)

13
x
5
_
2(13
x
+ 12)

13
x
+ 5;
h)
_
3
2
_
2x
+3
_
3
2
_
x1

1
9
_
2
3
_
x2
+1, 25 >
0;
i) ) log 3)
3x7
> (log
3
10)
7x+3
;
j) 4
x
2
2(x1)
+ 8
2
3
(x2)
> 52;
k) 2
2x+2
+ 6
x
2 3
2x+2
> 0;
l)
6 3
x+1
x
>
10
2x 1
;
m)
2
x+1
7
x 1
<
10
3 2x
.
4. Resuelva las inecuaciones:
a) log
0,5
2x
2
4x 6
4x 11
Cot
3
4
;
b) log

2
x
2
4x + 3
4
< 2Cot
3
4
;
c)
1
log
2
x

1
log
2

x + 2
;
d)
log
8
x
log
2
(1 + 2x)
<
log
2
3

1 + 2x
log
2
x
;
e) log
(
4 + 2x x
2
)
_
1 x
2
_

1
2
;
f ) log
8
(x
2
4x + 3) > Tan

4
;
g) log
2
2
x 3 log
2
x + 2 0;
h) log
4
(2x
2
+ 3x + 1) log
2
(2x + 2);
i)
[3 5x[ 4
_
log1
3
3[x[
0;
j) log
(
x + 1)(x
2
+x 6)
2
4;
k) log
1/2
2x
2
4x 6
4x 11
1;
l) log
2
4
x + 3
> log
2
(2 x);
m) log
x2
(2x 3) > log x 2(24 6x);
n) log
x+
5
2
_
x 5
2x 3
_
2
< 0;
o) log
2
2
(x 1)
2
log
0,5
(x 1) > 5;
p) x
log x
> 10;
q) (8 x)
log
2
2
(8x)
2
3x4
;
r) log
3
3
x 1
> log
3
(5 x);
s) log
1/4
(2 x) > log
1/4
2
x + 1
;
t) log
1/2
(5 + 4x x
2
) > 3;
u) log
0,1
(x
2
+ 75) log
0,1
(x 4) 2;
v) log
1/5
(2x + 5) < log
1/5
(16 x
2
) 1;
w) log
5
(x + 27) log
5
(16 2x) > log
5
x;
x) 2 log
8
(x 2) log
8
(x 3) >
2
3
.
5. Resuelva las inecuaciones:
a) log
|x1|
0, 5 > 0, 5;
b) log
2
0,2
(x 1) > 4;
c)
_
2
5
_
log
0,25
(x
2
5x+8)
2, 5;
d)
_
1
2
_
log
1/9
(x
2
3x+1)
< 1;
e) log
x
(x 1) 2;
f ) log
x

21 4x > 1;
g) log
x
x + 3
x 1
> 1;
h) log
x
(16 6x x
2
) 1;
i) log
x
2
3
729 > 3;
j) logx1
x+5
0, 3 > 0;
k) log
5

3x + 4 log
x
5 > 1;
l) 2
log
8
(x
2
6x+9)
3
2 log
x

x1
;
m)
1
2
+ log
9
x log
3
5x > log
1/3
(x + 3);
n) log
x
(x
3
+ 1) log
x+1
x > 2;
o) log
x
(x + 1) < log
1/x
(2 x);
p) log
|x4|
(2x
2
9x + 4) > 1;
q) log
|x+6|
2 log
2
(x
2
x 2) 1;
r) log
2
0,5
x + log
0,5
x 2 0;
s)
1 log
4
x
1 + log
2
x

1
2
;
t) log
2
(x + 1)
2
+ log
2
_
x
2
+ 2x + 1 > 6;
u) log
1/5
x + log
4
x > 1;
v) log
x
5

5 1, 25 > (log
x

5)
2
.
6. Resuelva las inecuaciones:
CAPTULO 9. FUNCIONES EXPONENCIALES Y LOGARTMICAS 463
a) log

2
(5
x
1) log

2
2

2
5
x
1
> 2;
b) 2
log
0,4
xlog
0,4
2,5x
> 1;
c)
_
x
log
2

x
> 2;
d) 0, 2
6
3
log
4
x
>
3
_
0, 008
2 log
4
x
1;
e) 0, 4
log
3
3
x
log
3
3x
> 6, 25
log
3
x
2
+2
;
f ) 2
log
2
0,5
x
+x
log
0,5
x
> 2, 5;
g) 3
log x+2
< 3
log x
2
+5
2;
h) x
log
2
x
+ 16x
log
2
x
< 17;
i) log
3
(4
x
+ 1) + log
4
x
+1
3 > 2, 5;
j) log
3
(3
x
1) log
1/3
(3
x+2
9) > 3;
k) log
2
[log
3
(2 log
4
x)] < 1;
l) x + log(1 + 2
x
) > xlog 5 + log 6;
m) log
2
(9
x
+ 3
2x1
2
x+1/2
) < x + 3, 5;
n) log
1/2
x +
_
1 4 log
2
1/2
x < 1;
o)
_
1 9 log
2
1/8
x > 1 4 log
1/8
x;
p) log
x/2
8 + log
x/4
8 <
log
2
x
4
log
2
x
2
4
;
q) log
x
2 log
2x
2
2
4x > 1;
r) log
2
log
1/2
(x
2
2) < 1;
s)
_
1
2
_
log
2
log
1/5
(x
2
4/5)
1;
t) 0, 3
log
1/3
log
2
3x+6
x
2
+2
> 1;
u) log
3
[log
2
(2 log
4
x) 1] < 1;
v) log
5
log
3
log
2
(2
2x
3 2
x
+ 10) > 0;
w) log
2
(1 + log
1/9
x log
9
x) < 1;
x) log
1/2
log
2
log
x1
9 > 0.
7. Resuelva las inecuaciones:
a) log
3
log
x
2 log
x
2 x
4
> 0;
b) log
x
log
2
(4
x
12) 1;
c)
log
5
(x
2
+ 3)
4x
2
16x
< 0;
d)
3x
2
16x + 21
log
0,3
(x
2
+ 4)
< 0;
e)
(x 0, 5)(3 x)
log
2
[x 1[
> 0;
f )
log
0,3
[x 2[
x
2
4x
< 0;
g)
log 7 log(8 x
2
)
log(x + 3)
> 0;
h)
log
2
(

4x + 5 1)
log
3
(

4x + 5 + 11)
>
1
2
;
i)
log
0,5
(

x + 3 1)
log
0,5
(

x + 3 + 5
<
1
2
;
j)
log

x + 7 log 2
log 8 log(x 5)
< 1;
k)
log(

x + 1 + 1)
log
3

x 40
< 3;
l) log
5
(x + 3) log
x+3
625;
m) log
2
x log
3
2x + log
3
x log
2
3x 0;
n) log
1/

5
(6
x+1
36
x
) 2;
o) log

3/3
(2
x+2
4
x
) 2;
p) 25
log
2
5
x
+x
log
5
x
30;
q) (2
x
+ 3 2
x
)
2 log
2
xlog
2
(x+6)
> 1;
r)
1
log
0,5

x + 3

1
log
0,5
(x + 1)
;
s)
1
log
2
x

1
log
2

x + 2
;
t)
_
log
2
0,5
x 81 + 2
log
0,5
x 1
< 1;
u) [x 1[
log
2
(4x)
> [x 1[
log
2
(1+x)
;
v)
x 1
log
3
(9 3
x
) 3
1;
w)
2 + log
3
x
x 1
<
6
2x 1
;
x)
6
2x + 1
>
1 + log
2
(2 +x)
x
.
8. Resuelva las inecuaciones:
a)
_
log
4
2x
2
3x + 3
2
+ 1 > log
2
2x
2
3x + 3
2
;
b) log
0,2
(x
3
+ 8) 0, 5 log
0,2
(x
2
+ 4x + 4) log
0,2
(x + 58);
c) log
2
(x
2
x 6) + log
1/2
(x 3) < log
1/

2
3;
d) log

2
7 3x
x + 2
log
1/

2
(x + 2) > log
1/2
4;
CAPTULO 9. FUNCIONES EXPONENCIALES Y LOGARTMICAS 464
e) 2, 25
log
2
(x
2
3x10)
>
_
2
3
_
log
1/2
(x
2
+4x+4)
;
f ) (log
2
x)
4

_
log
1/2
x
3
8
_
2
+ 9 log
2
32
x
2
< 4(log
1/2
x)
2
;
g) 9
log
2
(x1)1
8 5
log
2
(x1)2
> 9
log
2
(x1)
16 5
log
2
(x1)
;
h) log
2
(x 1) log
2
(x + 1) + logx+1
x1
2 > 0;
i) log
0,5
(x + 2) log
2
(x + 1) + log
x+1
(x + 2) > 0.
9. Resuelva los sistemas de inecuaciones:
a)
_
log
x
(x + 2) > 2
(x
2
8x + 13)
4x6
< 1
b)
_
(x 1) log 2 + log(2
x+1
+ 1) < log(7 2
x
+ 12)
log
x
(x + 2) > 2
9.7. Funciones exponenciales y logartmicas
Denicin 9.6 Funcin exponencial
Una funcin exponencial es una funcin de la forma f(x) = a
x
, donde a es una constante positiva.
En una funcin exponencial, la variable independiente x es el exponente de una constante positiva
conocida como la base de la funcin.
As, una funcin exponencial es fundamentalmente diferente de una funcin potencial donde la
base es la variable y el exponente es una constante.
El hecho de trabajar con funciones exponenciales requiere el uso de la notacin exponencial y
las leyes algebraicas de exponentes.
Corrientemente, en lgebra elemental, a
x
(a > 0) tiene signicado slo cuando x es un nmero
racional. En clculo es importante denir a
x
para valores irracionales de x.
Denicin 9.7 Logartmo de un nmero
Sean a > 0 y a ,= 1. El nmero k se llama logaritmo del nmero b > 0 en el sistema de base a si
a
k
= b.
El logaritmo del nmero b en el sistema de base a se designa por log
a
b. Por la misma denicin
a
log
a
b
.
Denicin 9.8 Funcin exponencial
Una funcin de la forma h(x) = ka
x
(a > 0, a ,= 1) se llama funcin exponencial con base a, y la
curva correspondiente se conoce como curva exponencial.
Sea a > 0 el nmero positivo dado, a ,= 1. La funcin exponencial f(x) = a
x
est denida en
R, el intervalo (0; +) es el conjunto de sus valores. Con a > 1 la funcin estrictamente crece, con
0 < a < 1, estrictamente decrece, los valores negativos de x producen valores positivos de f(x).
La funcin exponencial f(x) = a
x
, x R es inversible. La funcin inversa recibe el nombre de
logartmica y se designa con f(x) = log
a
x, ella est denida en el intervalo (0; +), el conjunto
R es el conjunto de sus valores.
CAPTULO 9. FUNCIONES EXPONENCIALES Y LOGARTMICAS 465
Con a > 1 la funcin logartmica estrictamente crece y la tasa de crecimiento para x > 1 es
lenta, con 0 < a < 1 estrictamente decrece y la tasa de decrecimiento para x > 1 es lenta. Si a > 1,
entonces, cuando x tiende a 0, la funcin decrece rpidamente. Si 0 < a < 1, entonces, cuando x
tiende a 0, la funcin crece rpidamente. Si a > 1, la funcin es negativa para 0 < x < 1 y positiva
para x > 1. Si 0 < a < 1, la funcin es positiva para 0 < x < 1 y negativa para x > 1.
Los grcos de las funciones
f(x) = a
x
, x R y f(x) = log
a
x, x (0; +)
son simtricos entre s con relacin a la recta y = x.
Sea a ,= 1 un nmero positivo. Decimos que y es el logaritmo de x en base a si a
y
= x. Es decir
log
a
x = y.
Sea x un nmero positivo. El logaritmo natural de x es log
e
x = lnx. Ntese que lnx solamente
se calcula para valores de x entre 1 y 10.
Sean x y a nmeros positivos, a ,= 1. Entonces
lnx
lna
= log
a
x.
Sea a un nmero positivo. Entonces, a
x
= e
xlna
para cada nmero real x.
Las grcas de algunas funciones exponenciales para a > 1 se exponen en la gura (a) y en (b)
para 0 < a < 1.
La funcin exponencial posee las siguientes caractersticas:
El dominio es (; +);
El codominio es (0; +);
La funcin est acotada inferiormente: y > 0;
La funcin no toma el valor mximo ni tampoco el mnimo;
La funcin no es peridica;
CAPTULO 9. FUNCIONES EXPONENCIALES Y LOGARTMICAS 466
La funcin no es par ni tampoco impar;
Si a > 1, la funcin y = a
x
crece en todo el dominio; si 0 < a < 1, la funcin y = a
x
decrece en todo el dominio;
El punto (0, 1) es el nico punto de interseccin con los ejes coordenados.
Ejemplo 9.13 Hallar el dominio de la funcin:
a) f(x) =
1
16
x
2
2
x
; b) f(x) =

2
x
3
x
.
Solucin
a) La funcin est denida si se cumple que 16
x
2
2
x
,= 0, lo cual implica que
16
x
2
,= 2
x
2
4x
2
,= 2
x
4x
2
,= x
x(4x 1) ,= 0 x ,= 0 y x ,=
1
4
Por lo tanto el dominio de la funcin es
x (; 0)
_
0;
1
4
_

_
1
4
; +
_
.
b) La funcin est denida si 2
x
3
x
0, es decir:
3
x
2
x
xlog3 xlog2 x(log3 log2) 0 x 0.
Por lo tanto el dominio de la funcin es x (; 0].
Ejemplo 9.14 Hallar el conjunto imagen de la funcin:
f(x) = 4
x
2
x
+ 1.
Solucin
Transformamos la ecuacin de la siguiente manera:
y = 2
2x
2
x
+ 1 y 1 = 2
2x
2
x
y 1 =
_
2
x

1
2
_
2

1
4
1
2
+
_
y
3
4
= 2
x
x = log
2
_
1
2
+
_
y
3
4
_
La funcin est denida si
1
2
+
_
y
3
4
> 0, lo cual implica que y
3
4
0, de donde y
3
4
. Por lo
tanto el conjunto imagen est dado por y
_
3
4
; +
_
.
Ejemplo 9.15 Determinar la paridad de las funciones:
a) f(x) =
1
3
x

1
3
x
; b) f(x) =
2
x
+ 2
x
3
x
3
x
.
Solucin
a) f(x) =
1
3
x

1
3
(x)
=
1
3
x

1
3
x
=
_
1
3
x

1
3
x
_
= f(x).
Por tanto, la funcin es impar.
b) f(x) =
2
x
+ 2
(x)
3
x
3
(x)
=
2
x
+ 2
x
3
x
3
x
=
2
x
+ 2
x
3
x
3
x
= f(x)
Por tanto, la funcin es par.
CAPTULO 9. FUNCIONES EXPONENCIALES Y LOGARTMICAS 467
Ejemplo 9.16 Hallar de ser posible la inversa de la funcin dada, indique su dominio:
f(x) =
3
x
2
x
3
x
+ 2
x
.
Solucin
y =
3
x
2
x
3
x
+ 2
x
y(3
x
+ 2
x
) = 3
x
2
x
(y + 1)2
x
= (1 y)3
x
log(y + 1) +xlog2 = log(1 y) +xlog3 x = log3
2
1 +y
1 y
.
La funcin est denida si
1 +y
1 y
> 0
y + 1
y 1
< 0
De esta desigualdad, obtenemos y (1; 1).
Una funcin y = log
a
x, donde a es un nmero jo tal, que a > 0 y a ,= 1, se denomina funcin
logartmica. La funcin logartmica posee las siguientes caractersticas:
El dominio es (0; +);
El codominio es (; +);
La funcin no est acotada ni superior ni inferiormente;
La funcin no toma el valor mximo ni tampoco el mnimo;
La funcin no es peridica;
La funcin no es par ni tampoco impar;
Si a > 1, la funcin y = log
a
x crece en todo el dominio; si 0 < a < 1, la funcin y = log
a
x
decrece en todo el dominio;
El punto (1, 0) es el nico punto de interseccin con los ejes coordenados.
De nuestra experiencia con funciones inversas es intuitivamente posible que si la funcin f(x)
es biunvoca y continua sobre su dominio, entonces su inversa f
1
(x) es continua sobre su dominio.
Como se dijo anteriormente, la funcin a
x
es continua sobre su dominio R. Consecuentemente su
inversa log
a
x es continua sobre su dominio (0; +). De y = log
a
x se sigue que si a > 0, a ,= 1
y x > 0, entonces x = a
log
a
x
. Tambin, ya que a
0
= 1 y a
1
= a, tendremos que log
a
1 = 0 y
log
a
a = 1.
Ejemplo 9.17 Hallar el dominio de la funcin:
a) f(x) = log
3+x
(x
2
1); b) f(x) =
_
log
3
2x3
x1
.
Solucin
a) Transformamos la ecuacin haciendo uso de las propiedades de los logaritmos:
f(x) = log
3+x
(x
2
1) =
log(x
2
1)
log(3 +x)
.
La funcin est denida si
_

_
x
2
1 > 0
3 +x > 0
log(3 +x) ,= 0

_
(x 1)(x + 1) > 0
3 +x > 0
3 +x ,= 1

_
x (; 1) (1; +)
x (3; +)
x (; 2) (2; +)
CAPTULO 9. FUNCIONES EXPONENCIALES Y LOGARTMICAS 468
Por lo tanto el dominio de la funcin es x (3; 2) (2; 1) (1; +).
b) Transformamos la ecuacin haciendo uso de las propiedades de los logaritmos:
f(x) =
_
log
3
2x 3
x 1
=

log
2x3
x1
log3
.
La funcin est denida si log
2x3
x1
0, lo cual implica que
2x3
x1
1, es decir
2x 3
x 1
1 0
x 2
x 1
0 x (; 1) [2; +).
Por lo tanto el dominio de la funcin es x (; 1) [2; +).
Ejemplo 9.18 Hallar el conjunto imagen de la funcin:
a) f(x) = log
3
x + log
x
3; b) f(x) =
_
2log
2
x log
2
2
x.
Solucin
a) Transformamos la ecuacin de la siguiente manera:
y = log
3
x + log
x
3 =
logx
log3
+
log3
logx
=
log
2
x + log
2
3
log3 logx
ylog3logx log
2
x = log
2
3 log
2
x ylog3logx = log
2
3
_
logx
1
2
ylog3
_
2 =
1
4
y
2
log
2
3 log
2
3 logx
1
2
ylog3 =
_
1
4
y
2
1 log3
logx =
_
_
1
4
y
2
1 +
1
2
y
_
log3 x = 3

1
4
y
2
1+
1
2
y
.
La funcin est denida si
_
1
4
y
2
1 0, es decir
y
2
4 0 (y + 2)(y 2) 0.
Por lo tanto el conjunto imagen est dado por y (; 2] [2; +).
b) Transformamos la ecuacin de la siguiente manera:
y
2
= 2log
2
x log
2
2
x log
2
2
x 2log
2
x = y
2
(log
2
x 1)
2
1 = y
2
log
2
x = 1 +
_
1 y
2
x = 2
1+

1y
2
La funcin est denida si 1 y
2
0, es decir:
y
2
1 0 (y + 1)(y 1) 0.
Por lo tanto el conjunto imagen est dado por y [1; 1].
Ejemplo 9.19 Determinar la paridad de la funcin:
f(x) = ln
2
_
x +
_
x
2
+ 1
_
.
CAPTULO 9. FUNCIONES EXPONENCIALES Y LOGARTMICAS 469
Solucin
f(x) = ln
2
_
(x) +
_
(x)
2
+ 1
_
= ln
2
_
x +
_
x
2
+ 1
_
= ln
2
_
_
x
2
+ 1 x
_ _
x
2
+ 1 +x
_

x
2
+ 1 +x
_
= ln
2
_
1

x
2
+ 1 +x
_
= ln
2
_
_
x
2
+ 1 +x
_
= f(x).
Por lo tanto, la funcin es impar.
Ejemplo 9.20 Hallar de ser posible la inversa de la funcin dada, indique su dominio:
f(x) = log
a
_
x +
_
x
2
+ 1
_
.
Solucin
y = log
a
_
x +
_
x
2
+ 1
_
yloga = log
_
x +
_
x
2
+ 1
_
a
y
= x +
_
x
2
+ 1 (a
y
x)
2
= x
2
+ 1
a
2y
2a
y
x = 1 x =
1
2
_
a
y
a
y
_
.
Esta funcin est denida para todo y R.
Las funciones exponenciales y logartmicas desempean un papel especial en las matemticas
aplicadas. A continuacin se presenta una muestra de situaciones prcticas provenientes de las
ciencias que pueden describirse matemticamente en trminos de tales funciones.
CURVAS LOGISTICAS
La grca de una funcin de la forma Q(t) =
B
1 +Ae
Bkt
, donde B, A y k son constantes positi-
vas, es una curva en forma de S. El trmino curva logstica tambin se utiliza para referirse a una
grca de este tipo.
Para representar la funcin logstica Q(t) =
B
1 +Ae
Bkt
, observe que la interseccin con el eje
vertical es
Q(0) =
B
1 +Ae
0
=
B
1 +A
.
Las curvas logsticas son modelos bastante pre-
cisos del crecimiento de la poblacin cuando los factores ambientales imponen un lmite superior
al tamao posible de la poblacin. Tambin describen la propagacin de epidemias y rumores en
una comunidad.
DATACION MEDIANTE CARBONO 14
El dixido de carbono en el aire contiene el istopo radiactivo
14
C (carbono 14) as como el istopo
estable
12
C (carbono 12). Las plantas vivas absorben dixido de carbono del aire, lo que implica
que la razn de
14
C a
12
C en una planta viva (o en un animal que se alimenta de plantas) es la
misma que hay en el aire. Cuando una planta o un animal mueren, la absorcin de dixido de
CAPTULO 9. FUNCIONES EXPONENCIALES Y LOGARTMICAS 470
carbono cesa. El
12
C que ya est en la planta o el animal permanece igual que en el momento
de la muerte, mientras que el
14
C decrece, y la razn de
14
C a
12
C decrece exponencialmente.
Es razonable suponer que la razn R
0
de
14
C a
12
C en la atmsfera es el mismo hoy que en el
pasado, de manera que la razn de
14
C a
12
C en una muestra est dada por una funcin de la
forma R(t) = R
0
e
kt
. El promedio de vida del
14
C es 5730 aos. Al comparar R(t) con R
0
, los
arquelogos pueden estimar la edad de la muestra.
CURVAS DE APRENDIZAJE
La grca de una funcin de la forma Q(t) = B
Ae
kt
, donde B, A y k son constantes positivas, se lla-
ma curva de aprendizaje. El nombre surge cuando los
psiclogos descubrieron que funciones de esta forma de-
scriben con frecuencia, la relacin entre la eciencia con
que un individuo realiza una tarea y la cantidad de ca-
pacitacin o experiencia que ste ha tenido.
En la gura se muestra una grca con estas carac-
tersticas. El comportamiento de la grca cuando t crece sin lmite, reeja el hecho de que al nal
un individuo se aproximar a una eciencia mxima, y que la capacitacin adicional tendr poco
efecto sobre el desempeo.
CRECIMIENTO EXPONENCIAL
Si una cantidad Q(t) crece de acuerdo con una ley de la forma Q(t) =
Q
0
e
kt
, donde Q
0
y k son constantes positivas, se dice que experimenta un
crecimiento exponencial. Por ejemplo, en ausencia de restricciones ambien-
tales, la poblacin crece en forma exponencial.
Las cantidades que aumentan exponencialmente se caracterizan por el
hecho de que su ritmo de crecimiento es proporcional a su tamao y que
su razn porcentual de cambio es constante. Q(t) crece exponencialmente si
Q(t) = Q
0
e
kt
donde k es una constante positiva y Q
0
es el valor inicial Q(0).
Para representar grcamente la funcin Q(t) = Q
0
e
kt
, observe que Q(t) es
siempre positiva, que Q(0) = Q
0
, que Q(t) crece sin lmite a medida que t
aumenta sin lmite y que Q(t) se aproxima a cero a medida que t decrece sin
lmite.
Ejemplo 9.21 Se proyecta que dentro de t aos la poblacin de cierto pas ser P(t) = 50e
0,02t
millones:
a) Cul es la poblacin actual?
b) Cul ser la poblacin dentro de 30 aos?
Solucin
a) La poblacin actual se calcula haciendo t = 0
P(0) = 50e
(0,02)(0)
= 50 millones.
b) La poblacin dentro de 30 aos ser
P(30) = 50e
(0,02)(30)
91, 1 millones.
CAPTULO 9. FUNCIONES EXPONENCIALES Y LOGARTMICAS 471
Ejemplo 9.22 El nmero total de hamburguesas vendidas por una cadena nacional de comida
rpida crece exponencialmente. Si se vendieron 5 millones en 2008 y 8 millones en 2009, cuntas
se vendern en 2010?
Solucin
Sea Q(t) el nmero de hamburguesas vendidas despus de t aos. Como el nmero de hamburguesas
crece exponencialmente, y puesto que al comienzo (2008) se vendieron 5 millones, Q es una funcin
de la forma Q(t) = 5e
kt
. Ya que pasado 1 ao (2009) se vendieron 8 millones, se obtiene que
8 = 5e
k
e
k
=
8
5
.
Para hallar cuntas hamburguesas se vendern en el segundo ao (2010), calculamos Q(2)
Q(2) = 5e
2k
= 5
_
e
k
_
2
= 5
_
8
5
_
2
=
64
5
= 12, 8
Es decir, se vendern 12.8 millones de hamburguesas en el 2010.
Ejemplo 9.23 La densidad de poblacin a x kilmetros del centro de una ciudad es D(x) =
12e
0,07x
miles de personas por kilmetro cuadrado:
a) Cul es la densidad de poblacin en el centro de la ciudad?
b) Cul es la densidad de poblacin a 5 kilmetros del centro de la ciudad?
Solucin
a) La densidad de poblacin en el centro de la ciudad es
D(0) = 12e
(0,07)(0)
= 12
es decir habr doce mil personas.
b) La densidad de poblacin a 5 kilmetros del centro de la ciudad es
D(5) = 12e
(0,07)(5)
es decir, la densidad de poblacin ser de ocho mil quinientas personas.
Ejemplo 9.24 La produccin diaria de un trabajador que ha estado en el trabajo t semanas
est dada por una funcin de la forma Q(t) = 40Ae
kt
. Al comienzo el trabajador poda producir
20 unidades por da, y despus de una semana puede producir 30 unidades por da. Cuntas
unidades por da producir el trabajador despus de 3 semanas?
Solucin
Al comienzo, es decir t = 0, Q(0) = 20, entonces
20 = 40 Ae
0k
= 40 A A = 20.
Despus de 1 semana, t = 1, Q(1) = 30, entonces
30 = 40 Ae
k
Ae
k
= 10 e
k
=
1
2
.
Cuando el trabajador tiene 3 semanas, t = 3, tenemos
Q(t) = 40 Ae
3k
= 40 A
_
e
k
_
3
= 40 20
_
1
2
_
3
= 37, 5.
Es decir, el trabajador producir aproximadamente 38 unidades por da.
CAPTULO 9. FUNCIONES EXPONENCIALES Y LOGARTMICAS 472
Ejemplo 9.25 Un arquelogo ha encontrado un fsil en el que la razn de
14
C a
12
C es
1
5
de
la razn encontrada en la atmsfera. Aproximadamente, cul es la edad del fsil?
Solucin
La edad del fsil es el valor de t para el que R(t) =
1
5
R
0
, es decir, para el cual
1
5
R
0
= R
0
e
kt

1
5
= e
kt
ln
1
5
= kt t =
ln5
k
Para hallar k, sabemos que el promedio de vida del
14
C es 5730 aos, entonces
R(5730) =
1
2
R
0
= R
0
e
5730k

1
2
= e
5730k
ln
1
2
= 5730k
Por lo tanto k =
ln 2
5730
. De esta manera, la edad del fsil es
t =
ln5
k
=
5730 ln5
ln2
13305
Es decir, el fsil tiene aproximadamente 13305 aos.
Ejemplo 9.26 Segn un modelo logstico basado en el supuesto de que la Tierra no puede
soportar ms de 40000 millones de personas, la poblacin mundial (en miles de millones) t aos
despus de 1980 est dada por una funcin de la forma P(t) =
40
1+Ce
kt
, donde C y k son constantes
positivas. Halle la funcin de esta forma que concuerde con el hecho de que la poblacin mundial era
aproximadamente de 4000 millones en 1980 y de 6000 millones en 2000. Qu predice su modelo
con respecto a cul ser la poblacin en el ao 2010?
Solucin
Sabemos que cuando t = 0 (1980), P(0) = 4000, es decir
400 =
40
1 +Ce
0k
400 =
40
1 +C
C =
99
100
.
Cuando t = 20 (2000), P(20) = 6000, es decir
6000 =
40
1 +Ce
20k
Ce
20k
=
149
150
k =
1
20
ln
298
297
.
Cuando t = 30 (2010), entonces
P(30) =
40
1
99
100
e
30k
=
40
1
99
100
e
30
1
20
ln
298
297
=
40
1
99
100
_
298
297
_3
2
Es decir, la poblacin mundial en el ao 2010 ser aproximadamente de 8007 millones de personas.
9.8. Tarea
1. Determine el valor de x, para que se cumpla:
a) f(x + 2) g
_
x
2
_
si f(x) = 5
3x+3
2
3x+1
y g(x) = 2
x+3
+ 4 5
x+4
;
b) f(x 2) g
_
x
3
_
si f(x) = 3
x4
+ 2
x
y g(x) = 3
2x
+ 2
2x
.
2. Con qu valores de a el dominio de la funcin f(x) contiene el dominio de la funcin g(x)?
f(x) = log
_
x
2
+a
_
, g(x) =
x
2
+x a
x
.
3. Determine el dominio de las siguientes expresiones:
CAPTULO 9. FUNCIONES EXPONENCIALES Y LOGARTMICAS 473
a) f(x) = 2

x1
+ 3

x+1
;
b) f(x) = e

x
2
1
x
2
+1
;
c) f(x) =
e

x
2
2x3
x
2
1
;
d) f(x) = 10
x
2
+x6

x
2
x6
;
e) f(x) =
_
x+1
x1
e

x1
x+1
;
f ) f(x) =
3

x
2
1
2

x
2
+2x15
;
g) f(x) = ln
_
x
2
5
x21
;
h) f(x) = log
x

x
2
4;
i) f(x) =

x
2
4x 5
10
x
2
1
10
x
2
+1
;
j) f(x) = ln
_
x

x
2
+ 1
_
;
k) f(x) =
ln
_
x
2
4x 5
_
ln

x
2
2x 3
;
l) f(x) = log

x
2
x6
_
x
2
3
_
;
m) f(x) =
ln
_
x
2
+ 3x 4
_
x
2
5x + 6
.
4. Determine el dominio de las siguientes expresiones:
a) f(x) =
1
2
x
2
5x+6
2
x
2
+3x4
;
b) f(x) = 5

x
2
+2x3
3

x
2
+5x+6
;
c) f(x) = e

5x+3
ln
_
x
2
+ 5x + 6
_
;
d) f(x) =

x
2
+ 2x 15 ln
_
x
2
1
_
.
5. Investigar la monotona de la funcin y construya su grca:
f(x) = ln
_
3x x
2
x 1
_
+ ln
_
x
3
x
2
3x + 2
x
5
+ 1
_
6. Investigar la monotona de la funcin y construya su grca:
a) f(x) = 2
1x
2
x1
;
b) f(x) = log
_
1 +x
3
_
;
c) f(x) = log1
2
_
x
x + 1
_
;
d) f(x) = ln
_
x +

x
2
+ 1
_
;
e) f(x) = log
2
_
8x x
2
_
;
f ) f(x) = 2 3
1x
9
x
.
7. Demuestre que el grco de la funcin f(x) = ln (1 e
x
) es simtrico con relacin a la
recta f(x) = x.
8. Los registros de salud pblica indican que t semanas despus del brote de la gripe AH1N1,
aproximadamente f(t) =
2
1+3e
0,8t
miles de personas han contrado la enfermedad:
a) Trace la grca de f(t).
b) Cuntas personas tenan la enfermedad al comienzo?
c) Cuntas haban contrado la enfermedad al nal de 3 semanas?
d) Si la tendencia contina, aproximadamente cuntas personas en total contraern la
enfermedad?
9. Demuestre que las funciones f(x) y g(x) son recprocamente inversas:
f(x) = e
1x
2
2
, x [0; +); g(x) =
_
1 2ln(x), x
_

e; 0
_
.
10. Cuando cierta maquinaria industrial tenga t aos, su valor de reventa ser V (t) = 4800e

t
5
dlares:
a) Dibuje la grca de V (t). Qu le sucede al valor de la maquinaria cuando t crece sin
lmite?
b) Cul era el valor de la maquinaria cuando estaba nueva?
c) Cul ser el valor de la maquinaria despus de 5 aos?
Captulo 10
Funciones hiperblicas
10.1. Funciones hiperblicas directas e inversas
A causa de la semejanza que existe entre la circunferencia y la hiprbola, se plantea la cuestin
de si habr un conjunto de magnitudes o funciones que se correspondan con la hiprbola de la misma
manera que las funciones circulares se corresponden con la circunferencia. Esas funciones existen
y se denominan funciones hiperblicas, es decir, seno hiperblico, coseno hiperblico, tangente
hiperblico, etc. Se representan por Senhx, Coshx, Tanhx, etc, aludiendo la letra h a la hiprbola.
En la gura, se ha dibujado un cuadrante MNP de la circunferencia x
2
+y
2
= a
2
y de la hiprbola
x
2
y
2
= a
2
, y para un punto cualquiera P de ambas curvas la abscisa es x = 0Q, la ordenada es
y = QP y el radio es a = 0M. En el caso de la circunferencia, cuando es el ngulo circular Q0P,
las funciones circulares son:
Sen =
y
a
, Cos =
x
a
, etc.
Anlogamente, una vez denido convenientemente el ngulo hiperblico , las funciones hiperbli-
cas son:
Senh =
y
a
, Cosh =
x
a
, etc.
Sin embargo, como el ngulo hiperblico no es el ngulo ordinario Q0P deberemos proceder a
su denicin. Con este objeto comenzaremos por desarrollar una importante propiedad de la cir-
cunferencia. Designemos por u el rea del sector circular M0P. Puesto que el rea de un cr-
culo es igual a
1
2
(radio longitud de la circunferencia), el rea de un sector circular ser igual a
474
CAPTULO 10. FUNCIONES HIPERBLICAS 475
1
2
(radio longitud del arco), siendo el arco aquella parte de la circunferencia que limita al sector.
Por lo tanto, en la gura
reaA =
1
2
a(arcoMP).
Pero cuando = M0P, se verica que arcoMP = a. Por consiguiente,
A =
1
2
a(a), donde =
2A
a
2
(10.1)
Es decir, que en toda frmula cuando aparece un ngulo circular se puede sustituir por el rea
del sector correspondiente al ngulo multiplicada por
1
a
2
. Por este motivo se llama a veces a A
ngulo sectorial, y la magnitud theta expresada en funcin de A por medio de la relacin =
2A
a
2
es el correspondiente ngulo circular.
Utilizando el ngulo circular as expresado, las funciones circulares de la circunferencia sern,
pues,
_
y
a
= Sen
2A
a
2
x
a
= Cos
2A
a
2
En el caso de la hiprbola, no se usa el ngulo ordinario M0P, y el ngulo hiperblico se dene
como
2A
a
2
, en que A es el rea del sector hiperblico M0P de la gura y a = 0M. Las funciones
hiperblicas quedan entonces denidas por las frmulas
_
y
a
= Senh
2A
a
2
x
a
= Cosh
2A
a
2
(10.2)
en las que x y y son las coordenadas de un punto P de la hiprbola equiltera. Las dems funciones
hiperblicas se denen como sus anlogas de trigonometra circular y entre ellas existen las mismas
relaciones como, por ejemplo,
Tanh =
Senh
Cosh
, Cot =
Cosh
Senh
, etc.
Si recordamos que al hablar del ngulo hiperblico correspondiente a un determinado punto P de la
hiprbola equiltera, no nos referimos al ngulo ordinario M0P como en el caso de la circunferencia,
sino el ngulo hiperblico, podremos escribir, como en (14.1) para la circunferencia, para el ngulo
hiperblico correspondiente al rea A del sector:
=
2A
a
2
(10.3)
y las frmulas (10.2) se pueden escribir
_
y
a
= Senh
x
a
= Cosh
(10.4)
que corresponden a las frmulas corrientes de las funciones circulares. El resto de las funciones
hiperblicas se expresan en funcin del radio a y de las coordenadas x y y, por medio de las rela-
ciones ya conocidas.
Existen muchas, interesantes y tiles relaciones entre las funciones hiperblicas, cuyo conjunto
forman lo que a veces se llama trigonometra hiperblica. Las funciones exponenciales e hiperbli-
cas, estn estrechamente relacionadas, tienen enorme importancia en electricidad, principalmente
CAPTULO 10. FUNCIONES HIPERBLICAS 476
en telefona, telegrafa, cables de transmisin, y tambin en la teora de la mquina de vapor, mo-
tores de gasolina, compresores de aire, y en muchas otras ramas de la fsica y de la fsico-qumica.
Como vamos a ver ahora, las funciones hiperblicas estn estrechamente relacionadas con el
nmero e. El rea A = M0PM en el caso de la hiprbola equiltera, est dada por
A =
1
2
a
2
log
e
_
x +y
a
_
De aqu,
log
e
_
x +y
a
_
=
2A
a
2

x +y
a
= e
2A
a
2
y segn la frmula (10.2), resulta
x +y
a
= e

(10.5)
Ahora bien, la ecuacin de la hiprbola es
x
2
y
2
= a
2

_
x +y
a
__
x y
a
_
= 1
Si dividimos miembro a miembro esta ecuacin y la (10.5), se obtiene
x y
a
=
1
e


x y
a
= e

Esta ecuacin y la (10.5) se pueden escribir:


x
a
+
y
a
= e

(10.6)
x
a

y
a
= e

(10.7)
Restando miembro a miembro (10.7) de (10.6), los trminos
x
a
se reducen, y se obtiene
2y
a
= e

y
a
=
1
2
(e

) (10.8)
Anlogamente, sumando miembro a miembro las ecuaciones (10.6) y (10.7) se obtiene
x
a
=
1
2
(e

+e

) (10.9)
Ahora bien, en las ecuaciones (10.8) y (10.9) y en las ecuaciones (10.4), x y y son las mismas
coordenadas de un punto P de la hiprbola y a es el radio hiperblico. Comparando esas ecuaciones,
tendremos
_
Senh =
1
2
(e

)
Cosh =
1
2
(e

+e

)
(10.10)
y mediante estas ecuaciones podremos, gracias a las relaciones ya conocidas, expresar tambin la
tangente, cotangente, secante y cosecante hiperblicas en funcin de las funciones exponenciales.
Estos son los resultados que buscbamos al investigar las relaciones que existen entre las funciones
hiperblicas y el nmero e.
CAPTULO 10. FUNCIONES HIPERBLICAS 477
Gracias a estas ecuaciones podremos expresar directamente las funciones hiperblicas de un
nmero cualquiera, en funcin de las funciones exponenciales, sin hacer ninguna referencia a la
hiprbola, y eso es lo que se suele hacer frecuentemente. Hay que sobrentender, sin embargo, que
la relacin hiperblica, se use explcitamente o no, es la base de las ecuaciones.
Despejando en las ecuaciones (10.10) e

y e

, se pueden expresar tambin las exponenciales


en funcin de las funciones hiperblicas. En efecto, sumando primero las dos ecuaciones se eliminan
las exponenciales negativas, y restando la primera de la segunda, se eliminan los trminos positivos,
teniendo as los resultados
_
e

= Cosh +Senh
e

= Cosh Senh
(10.11)
Esas dos notables frmulas dan la funcin exponencial e

en funcin de las funciones hiperblicas.


10.1.1. Funcin seno hiperblico
El seno hiperblico se dene en R, con la frmula
f(x) =
1
2
(e
x
e
x
)
Dado que
f(x) =
1
2
[e
(x)
e
(x)
] =
1
2
(e
x
e
x
) =
1
2
(e
x
e
x
) = f(x)
la funcin f(x) = Senhx es impar, montona creciente desde hasta +. El origen de coorde-
nadas es un punto de inexin y el centro de simetra de la curva. No tiene asntotas.
La inversa de f(x) = Senhx, se establece de la siguiente manera:
Figura 10.1: f(x)=Senhx y f(x)=AreaSenhx
y =
1
2
(e
x
e
x
) e
2x
2ye
x
1 = 0 x = ln
_
y
_
1 +y
2
_
de donde
AreaSenhx = ln
_
x +
_
1 +x
2
_
, x R
Dado que
f(x) = AreaSenh(x) = AreaSenhx = f(x)
la funcin f(x) = AreaSenhx es impar, montona creciente desde hasta +. El origen de
coordenadas es un punto de inexin y es el centro de la simetra de la curva. Carece de asntotas.
CAPTULO 10. FUNCIONES HIPERBLICAS 478
Ejemplo 10.1 Determine el dominio de la siguiente expresin:
f(x) = Senh
2x
2
1
4x
2
1
Sen
x + 1
6x
2
x 1
Solucin
La expresin est determinada si se cumple lo siguiente:
4x
2
1 ,= 0 (2x 1)(2x + 1) ,= 0 x ,=
1
2
y x ,=
1
2
6x
2
x 1 ,= 0 (2x 1)(3x + 1) ,= 0 x ,=
1
3
y x ,=
1
2
Por lo tanto, el dominio de la funcin es: x R
_

1
2
,
1
3
,
1
2
_
.
10.1.2. Funcin coseno hiperblico
El coseno hiperblico se dene en R, con la frmula
f(x) =
1
2
(e
x
+e
x
)
Dado que
f(x) =
1
2
(e
x
+e
(x)
) =
1
2
(e
x
+e
x
) =
1
2
(e
x
+e
x
)
la funcin f(x) = Coshx es par; para x < 0 decrece desde + hasta 1, para x > 0 crece desde 1
hasta +.
Tiene un mnimo en el punto (0, 1): no tiene asntotas. La curva est situada simtricamente
con respecto al eje Y .
La inversa de f(x) = Coshx, se establece de la siguiente manera:
Figura 10.2: f(x)=Coshx y f(x)=AreaCoshx
y =
1
2
(e
x
+e
x
) e
2x
2ye
x
+ 1 = 0 x = ln
_
y
_
y
2
1
_
de donde
AreaCoshx = ln
_
x +
_
x
2
1
_
, x 1. (AreaCoshx > 0es valor principal)
La expresin f(x) = AreaCoshx no es par ni impar, es biforme y existe slo para los valores de
x 1. La curva es simtrica con respecto al eje X; en el punto (1, 0) es tangente a la recta vertical
x = 1, despus y crece en valor absoluto.
CAPTULO 10. FUNCIONES HIPERBLICAS 479
Ejemplo 10.2 Demuestre la siguiente propiedad
Cosh
2
x Senh
2
x = 1
Solucin
Cosh
2
x Senh
2
x =
_
e
x
+e
x
2
_
2

_
e
x
e
x
2
_
2
=
e
2x
+ 2e
x
e
x
+e
2x
4

e
2x
2e
x
e
x
+e
2x
4
=
e
2x
+ 2e
x
e
x
+e
2x
e
2x
+ 2e
x
e
x
e
2x
4
=
4e
x
e
x
4
= 1.
Ejemplo 10.3 Demuestre la siguiente propiedad
Senh(x +y) = SenhxCoshy +CoshxSenhy
Solucin
Para demostrar esta propiedad, utilizaremos las identidades establecidas anteriormente para el
Senhx y Coshx.
SenhxCoshy +CoshxSenhy =
e
x
e
x
2

e
y
+e
y
2
+
e
x
+e
x
2

e
y
e
y
2
=
e
x
e
y
+e
x
e
y
e
x
e
y
e
x
e
y
4
+
e
x
e
y
e
x
e
y
+e
x
e
y
e
x
e
y
4
=
e
x
e
y
+e
x
e
y
e
x
e
y
e
x
e
y
+e
x
e
y
e
x
e
y
+e
x
e
y
e
x
e
y
4
=
2e
x
e
y
2e
x
e
y
4
=
e
x+y
e
x+y
4
= Senh(x +y).
Ejemplo 10.4 Demuestre la siguiente propiedad
Senh2x = 2SenhxCoshx
Solucin
Para demostrar esta propiedad, utilizaremos las identidades establecidas anteriormente para el
Senhx y Coshx:
2SenhxCoshx = 2
e
x
e
x
2

e
x
+e
x
2
=
e
x
e
x
+e
x
e
x
e
x
e
x
e
x
e
x
2
=
e
2x
e
2x
2
= Senh2x.
10.1.3. Funcin tangente hiperblica
La tangente hiperblica se dene en R, de la siguiente manera:
f(x) =
e
x
e
x
e
x
+e
x
CAPTULO 10. FUNCIONES HIPERBLICAS 480
Dado que
f(x) =
e
x
e
(x)
e
x
+e
(x)
=
e
x
e
x
e
x
+e
x
=
e
x
e
x
e
x
+e
x
= f(x)
la funcin f(x) = Tanhx es impar, montona creciente desde -1 hasta + 1. El origen de coordenadas
es un punto de inexin y es el centro de simetra de la curva. Tiene dos asntotas: y = 1.
La inversa de f(x) = Tanhx, se establece de la siguiente manera:
Figura 10.3: f(x)=Tanhx y f(x)=AreaTanhx
y =
e
x
e
x
e
x
+e
x
e
2x
=
1 +y
1 y
x =
1
2
ln
_
1 +y
1 y
_
de donde
AreaTanhx =
1
2
ln
_
1 +x
1 x
_
, 1 < x < 1.
La expresin f(x) = AreaTanhx es impar y existe slo para los valores de [x[ < 1; desde
hasta + es montona creciente. El origen de coordenadas es un punto de inexin y es el centro
de simetra de la curva. Tiene dos asntotas: x = 1.
Ejemplo 10.5 Demuestre la siguiente propiedad
Tanh(x +y) =
Tanhx +Tanhy
1 +TanhxTanhy
Solucin
Para probar esta identidad, utilizaremos las frmulas deducidas anteriormente para Senhx y
Coshx:
Tanh(x +y) =
Sen(x +y)
Cosh(x +y)
=
SenhxCoshy +CoshxSenhy
CoshxCoshy +SenhxSenhy
=
SenhxCoshy+CoshxSenhy
CoshxCoshy
CoshxCoshy+SenhxSenhy
CoshxCoshy
=
Senhx
Coshx
+
Senhy
Coshy
1 +
Senhx
Coshx

Senhy
Coshy
=
Tanhx +Tanhy
1 +TanhxTanhy
.
CAPTULO 10. FUNCIONES HIPERBLICAS 481
10.1.4. Funcin cotangente hiperblica
La Cotangente hiperblica se dene en R0, de la siguiente manera:
f(x) =
e
x
+e
x
e
x
e
x
Dado que
f(x) =
e
x
+e
(x)
e
x
e
(x)
=
e
x
+e
x
e
x
e
x
=
e
x
+e
x
e
x
e
x
= f(x)
la funcin f(x) = Cothx es impar, para x = 0 tiene una discontinuidad. Para x < 0 decrece desde
-1 hasta , para x > 0 decrece desde + hasta +1. No tiene extremos ni puntos de inexin.
Tiene tres asntotas: x = 0, y = 1.
La inversa de f(x) = Cothx, se establece de la siguiente manera:
Figura 10.4: f(x)=Cothx y f(x)=AreaCothx
y =
e
x
+e
x
e
x
e
x
e
2x
=
y + 1
y 1
x =
1
2
ln
_
y + 1
y 1
_
de donde
AreaCotx =
1
2
ln
_
x + 1
x 1
_
, x > 1 x < 1.
La funcin f(x) = AreaCothx es impar y existe slo para los valores de [x[ > 1. Para < x < 1
decrece desde 0 hasta , para +1 < x < + decrece desde + hasta 0. No tiene extremos ni
puntos de inexin. Tiene tres asntotas: y = 0, x = 1.
Ejemplo 10.6 Demuestre la siguiente propiedad
AreaCothx = AreaTanh
1
x
Solucin
Para probar esta identidad, se procede de la siguiente manera:
AreaCothx =
1
2
ln
_
x + 1
x 1
_
=
1
2
ln
_
1 +
1
x
1
1
x
_
= AreaTanh
1
x
.
CAPTULO 10. FUNCIONES HIPERBLICAS 482
10.1.5. Funcin secante hiperblica
La Secante hiperblica se dene en R, de la siguiente manera:
f(x) =
2
e
x
+e
x
Dado que
f(x) =
2
e
x
+e
(x)
=
2
e
x
+e
x
=
2
e
x
+e
x
= f(x)
la funcin f(x) = Sechx es par; para x < 0 crece desde 0 hasta 1, para x > 0 decrece desde 1 hasta
0. Tiene un mximo en el punto (0, 1). No tiene extremos, la curva est situada simtricamente
con respecto al eje Y . Tiene una asntota: y = 0.
La inversa de f(x) = Sechx, se establece de la siguiente manera:
Figura 10.5: f(x)=Sechx y f(x)=AreaSechx
y =
2
e
x
+e
x
ye
2x
2e
x
+y = 0 x = ln
_
1
y

_
1
y
2
1
_
de donde
AreaSechx = ln
_
1
x
+
_
1
x
2
1
_
, 0 < x 1 (AreaSechx > 0es valor principal)
la funcin f(x) = AreaSechx no es par ni impar y existe slo para los valores de 0 < x 1. Para
0 < x 1 decrece desde + hasta 0. No tiene asntotas.
10.1.6. Funcin cosecante hiperblica
La Cosecante hiperblica se dene en R0, de la siguiente manera:
f(x) =
2
e
x
e
x
Dado que
f(x) =
2
e
x
e
(x)
=
2
e
x
e
x
=
2
e
x
e
x
= f(x)
CAPTULO 10. FUNCIONES HIPERBLICAS 483
Figura 10.6: f(x)=Cschx y f(x)=AreaCschx
la funcin f(x) = Cschx es impar; para x < 0 decrece desde 0 hasta , para x > 0 decrece desde
+hasta 0. No tiene extremos, la curva est situada simtricamente con respecto al origen. Tiene
dos asntotas: y = 0, x = 0. La inversa de f(x) = AreaCschx, se establece de la siguiente manera:
y =
2
e
x
e
x
ye
2x
2e
x
y = 0 x = ln
_
1
y

_
1
y
2
+ 1
_
de donde
AreaCschx = ln
_
1
x
+
_
1
x
2
+ 1
_
, x ,= 0.
la funcin f(x) = AreaCschx es impar; para x < 0 decrece desde 0 hasta , para x > 0 decrece
desde +hasta 0. No tiene extremos, la curva est situada simtricamente con respecto al origen.
Tiene dos asntotas: y = 0, x = 0.
10.2. Tarea
1. Demuestre las identidades:
a) Sech
2
x +Tanh
2
x = 1;
b) Coth
2
x Coth
2
x = 1;
c) SenhxSenhy =
1
2
[Cosh(x +y) Cosh(x y)];
d) SenhxCoshy =
1
2
[Senh(x +y) +Senh(x y)];
e) CoshxCoshy =
1
2
[Cosh(x +y) +Cosh(x y)];
f ) Senh(x y) = SenhxCoshy CoshxSenhy;
g) Cosh(x +y) = CoshxCoshy +SenhxSenhy;
h) Cosh(x y) = CoshxCoshy SenhxSenhy.
2. Demuestre las identidades:
a) (Coshx +Senhx)
n
= Coshnx +Senhnx;
b) Coshnx =
1
2
[(Coshx +Senhx)
n
+ (Coshx Senhx)
n
];
c) Senhnx =
1
2
[(Coshx +Senhx)
n
(Coshx Senhx)
n
].
3. Utilizando las igualdades
Senh
n
x =
1
2
n
(e
x
e
x
)
n
; Cosh
n
x =
1
2
n
(e
x
+e
x
)
n
.
CAPTULO 10. FUNCIONES HIPERBLICAS 484
Demuestre:
a) Cosh
3
x =
1
4
Cosh3x +
3
4
Coshx;
b) Senh
5
x =
1
16
Senh5x
5
16
Senh3x +
5
8
Senhx.
4. Simplique las expresiones:
a) (CosxCoshy +iSenxSenhy)
2
(CosxSenhy +iSenxCoshy)
2
;
b) (xCosht +ySenht)
2
(xSenht +yCosht)
2
.
5. Demuestre las identidades:
a) AreaCschx = AreaSenh
1
x
; b) AreaSechx = AreaCosh
1
x
.
6. Demuestre la identidad:
Cosh2x = Cosh
2
x +Senh
2
x = 2Cosh
2
x 1 = 1 + 2Senh
2
x
7. Sea f(x) = AreaCoshx, x 1 una funcin inversa a f(x) = Coshx, x 0. Demuestre que
la funcin
f(x) =
_
2Cosh
AreaCoshx
3
, x 1,
2Cos
ArcCosx
3
, 1 x < 1.
es inversa a la funcin f(x) =
1
2
(x
3
3x), x 1.
8. Determine el dominio de las funciones:
a) f(x) =
x +Senhx
x
2
+Cosh
2
x
;
b) f(x) = AreaTanh
_
1 x
1 +x
;
c) f(x) = Cosh(x +Senhx);
d) f(x) = Tanh(AreaTanhx);
e) f(x) =
_
1 Senhx
1 +Senhx
;
f ) f(x) =
Tanhx

1 +Senhx
;
g) f(x) =
x +Senhx
x +Coshx
;
h) f(x) =
Cosh2x 1
Cosx 1
;
i) f(x) =
_
1 +Tanhx
1 Tanhx
;
j) f(x) =
_
1 +Senhx
Senhx
;
k) f(x) =

Senhx +Coshx
1 Coshx
;
l) f(x) =
1 +xAreaTanhx

1 x
2
;
m) f(x) =

1 Sechx
Coshx
;
n) f(x) =
AreaSenhx

Senhx +Coshx
.
9. Determine la paridad de las funciones:
a) f(x) =
(1 +Senhx)(1 +Coshx)
3 +Tanh
2
x
;
b) f(x) =
1
1 +Senhx
+
1
1 +Cosh
2
x
;
c) f(x) = x
3
Coshx + 3x
2
Senhx;
d) f(x) = SenhxCoshx +x(Senh
2
x +Cosh
2
x);
e) f(x) =
CoshxSenx +CosxSenhx
Cosh
2
x
;
f ) f(x) = SenxSenhx +CosxCoshx;
g) f(x) = SenxSenhxCosxCoshx;
h) f(x) = Senh
2
(ln x) +Cosh
2
(ln x);
i) f(x) = SenhxCosh
2
xTanh
3
x;
CAPTULO 10. FUNCIONES HIPERBLICAS 485
j) f(x) = x3Senh
2
x +x2Cosh
2
x;
k) f(x) = (cosx +Coshx)(Senx +Senhx);
l) f(x) = (2 +Cosh
2
x)
2
Senhx;
m) f(x) = (x
2
+ 1)(Senh2x + 1)(Cosh2x + 1).
10. Construir el grco de las funciones:
a) f(x) = Tanhx

x;
b) f(x) = ln Cothx;
c) f(x) =
Coshx
1 Coshx
;
d) f(x) = Tanh(x +Coshx);
e) f(x) =
1 Senhx
1 +Senhx
;
f ) f(x) = Tanh(AreaTanhx);
g) f(x)

1 +AreaTanhx;
h) f(x) =
Senhx
e
x
1
;
i) f(x) =
Senhx Coshx
Senhx +Coshx
;
j) f(x) =

1 Senhx;
k) f(x) = AreaSenh
_
1 x
2
1 +x
2
_
.
Captulo 11
Funciones trigonomtricas
11.1. Angulos
Sean dados dos lados coincidentes, el lado 0A y el lado 0B. Supongamos que el lado 0A realiza
cierto giro, dando vueltas en un plano en torno al punto 0. Entonces para cualquier giro semejante
el lado 0B se considera como lado inicial de giro, mientras que el lado 0A, el lado nal, que realiza
el giro dado.
Cualquier giro del lado nal 0A con relacin al lado jo
0B puede realizarse en dos direcciones opuestas, en el sen-
tido horario y en el sentido antihorario. Si en el lado nal
0A montamos un dispositivo trazador que se aleje uniformemente del punto 0, desplazndose a lo
largo del lado 0A, entonces, a medida que gira el lado 0A, el dispositivo dejar en el plano cierta
huella. Al realizar el lado 0A cierto giro, la huella representar una curva en desenrollamiento en
torno del punto de giro 0. Dicha curva tiene por origen el lado inicial 0B y termina junto al lado
nal 0A. Con ayuda de tal curva se muestran en las grcas los giros, con la particularidad de que
junto al lado nal la curva termina con una echa que indica el sentido del giro realizado.
Si el lado nal 0A realiza el giro en el sentido horario de modo tal que el lado 0A ha coincidido
por primera vez con el lado jo 0B. Este giro suele llamarse vuelta completa en el sentido horario.
Si el lado nal 0A realiza tal giro en el sentido antihorario de modo tal que el lado 0A coincide por
primera vez con el lado 0B. Este giro suele llamarse vuelta completa en el sentido antihorario.
Si el lado nal 0A realiza un giro en el plano en torno del punto 0 respecto del lado jo 0B. En
este caso suele considerarse que de esta manera se forma un ngulo y se dice que el lado nal
0A describe el ngulo , correspondiente al giro citado. El punto 0 se denomina vrtice del ngulo
, el lado jo 0B es el lado de referencia del ngulo , y 0A, el lado nal que describe el ngulo
. El lado jo 0B suele disponerse en los dibujos horizontalmente orientado a la derecha. Se ha
convenido en considerar que si el lado nal realiza cierto giro en el sentido antihorario, se describe
de este modo el correspondiente ngulo positivo: si el lado nal realiza cierto giro en el sentido
horario, l describe el correspondiente ngulo negativo; si el lado nal no realiza ningn giro, l
preja el ngulo nulo.
Por ejemplo, si el lado nal 0A da una vuelta completa en el sentido antihorario, dicho lado
describe el ngulo positivo completo: si el lado 0A da una vuelta completa en el sentido horario,
486
CAPTULO 11. FUNCIONES TRIGONOMTRICAS 487
l describe el ngulo negativo completo.
11.1.1. Medicin del ngulo en grados
Si el lado nal 0A realiza un giro igual a
1
360
parte de vuelta completa en el sentido antihorario.
En este caso se dice que el lado nal 0A describe un ngulo cuya medida en grados es igual a
un grado. Por consiguiente, el ngulo positivo completo y el de 360

es el mismo ngulo, descrito


por el lado nal 0A que realiza una vuelta completa en el sentido antihorario. Para las partes del
ngulo de un grado se han aceptado denominaciones especiales que son minuto y segundo.
Si el lado nal 0A realiza en el sentido antihorario un giro igual a
1
60
parte de vuelta, corre-
spondiente al ngulo de un grado. En este caso se dice que el lado nal 0A describe un ngulo de
un minuto. Por consiguiente, un ngulo de 60 y un ngulo de 1

son un mismo ngulo.


Si el lado nal 0A realiza en el sentido antihorario un giro igual a
1
60
parte de vuelta, corre-
spondiente al ngulo de un minuto. En este caso se dice que el lado nal 0A describe un ngulo de
un segundo. Por consiguiente, un ngulo de 60 y un ngulo de 1 son un mismo ngulo.
Si el lado nal 0A realiza en el sentido antihorario un giro igual a
1
4
parte de una vuelta comple-
ta. En este caso se dice que el lado nal 0A describe un ngulo recto positivo o bien un ngulo de 90

.
Si el lado nal 0A realiza en el sentido antihorario un giro igual a
1
2
parte de una vuelta com-
pleta. En este caso se dice que el lado nal 0A describe un ngulo positivo, o bien un ngulo de 180

.
Si el lado nal 0A no realiza ningn giro, en este caso se dice que el lado nal 0A describe un
ngulo nulo, o bien un ngulo de 0

. En los casos como el citado suele decirse a veces que el lado


nal 0A ha realizado una vuelta nula.
Si el lado nal 0A realiza un giro igual a
1
2
parte de una vuelta completa en el sentido horario.
En este caso se dice que el lado nal 0A describe un ngulo negativo, o bien un ngulo de 180

.
Si el lado nal 0A realiza un giro igual a
1
4
parte de una vuelta completa en el sentido horario.
Entonces, el lado nal 0A describe un ngulo recto negativo, o bien un ngulo de 90

.
11.1.2. Medida radial del ngulo
Todo ngulo se puede considerar como resulta-
do de la rotacin de un vector en el plano alrededor
de un punto inicial. La nocin acerca de la medi-
cin de los ngulos se conoce de la geometra. Al
medir los ngulos se toma un ngulo determina-
do por unidad de medida y con su ayuda se mi-
den otros ngulos. Por unidad de medida se puede
tomar un ngulo cualquiera.
Supongamos que el lado nal 0A coincide con el lado jo 0B sin dar ninguna vuelta. Elijamos
arbitrariamente un punto P en el lado jo 0B y un punto Q del lado nal 0A que coincide con el
punto P. Tracemos una circunferencia con centro en el punto 0 y de radio R, igual a la longitud
del segmento 0Q.
CAPTULO 11. FUNCIONES TRIGONOMTRICAS 488
Si el lado nal 0A empieza a girar alrededor del punto 0, el punto Q se desplazar a lo largo
de esta circunferencia.
Supongamos que el lado nal 0A realiza tal giro en el sentido antihorario que el punto Q, de-
splazndose por la circunferencia, pase una distancia igual al radio de sta. En este caso se dice que
el lado nal 0A describe un ngulo cuya medida radial es igual a un radin, o, ms brevemente,
un ngulo de un radin.
Sea dado un nmero positivo k. Supongamos que el la-
do nal 0A realiza tal giro en el sentido antihorario que el
punto Q, desplazndose por la circunferencia, pase una dis-
tancia D, igual a kR; entonces se dice que el lado nal 0A
preja un ngulo de k radianes.
Sea dado un nmero negativo k, y supongamos que el
lado nal 0A realiza tal giro en el sentido horario, que el
punto Q, desplazndose por la circunferencia, pase una dis-
tancia D, igual a [k[R; entonces se dice que el lado nal 0A
describe un ngulo de k radianes. As pues, la medida radial
de cualquier ngulo se dene del modo siguiente.
Denicin 11.1 Medida radial
Sea dado cierto ngulo , descrito por el lado nal 0A. Se denomina medida radial del ngulo
tal nmero, cuyo valor absoluto es igual a la razn entre la distancia D, recorrida a lo largo de la
circunferencia de radio R por el punto Q del lado nal 0A, y el radio R, y cuyo signo se dene
por el sentido del giro realizado, en otras palabras, se llama medida radial del ngulo un nmero
positivo
D
R
, si el giro se realiza en el sentido antihorario o bien un nmero negativo
D
R
, si el
giro se realiza en el sentido horario. Si el ngulo viene descrito por el lado nal 0A que no realiza
ningn giro, entonces el ngulo ser nulo y la medida radial de este ngulo se considera igual a
cero.
De esta denicin se deduce que el ngulo cuya medida circular vale 1 es un ngulo que es
congruente al ngulo central del circulo unitario el cual se apoya en el arco de la longitud unitaria.
Si el lado nal 0A realiza una vuelta completa en el sentido antihorario, entonces, el punto Q
del lado nal 0A, desplazndose por la circunferencia de radio R, recorre una distancia igual a
2R. Quiere decir, en este caso el lado nal 0A describe un ngulo, cuya medida radial es igual a
2 radianes, es decir, el ngulo de 360

y el de 2 radianes son un mismo ngulo.


Si el lado nal 0A da una vuelta completa en el sentido horario, se describe un ngulo de 2
radianes, es decir, el ngulo de 360

y el ngulo de 2 radianes son un mismo ngulo.


Supongamos que el lado nal 0A realiza
1
4
parte de vuelta completa en el sentido antihorario.
En este caso el punto Q del lado nal, desplazndose por la circunferencia de radio R, recorre una
distancia igual a

2
R. Por consiguiente, si el lado nal 0A realiza
1
4
parte de vuelta completa en el
sentido antihorario, l describe un ngulo de

2
radianes, es decir, un ngulo de 90

y un ngulo
de

2
radianes son un mismo ngulo.
Si el lado nal 0A realiza
1
4
parte de vuelta completa en el sentido horario, l describe un n-
gulo de

2
radianes, es decir, un ngulo de 90

y un ngulo de

2
radianes son un mismo ngulo.
CAPTULO 11. FUNCIONES TRIGONOMTRICAS 489
Supongamos que el lado nal 0A realiza
1
2
parte de vuelta completa en el sentido antihorario.
Entonces, el punto Q del lado nal 0A, desplazndose a lo largo de la circunferencia de radio R,
recorre una distancia igual a R, por consiguiente, en este caso, el ngulo que se describe por el
lado nal 0A medir radianes, es decir, el ngulo de 180

y el ngulo de radianes representa


un mismo ngulo.
Anlogamente, el ngulo de 180

y el ngulo de radianes representan un mismo ngulo


que se preja por el lado nal 0A que realiza
1
2
parte de vuelta completa en el sentido horario.
Si la medida radial de cierto ngulo constituye k radianes, mientras que la medida por grados
del mismo ngulo es igual a grados, los nmeros mencionados estarn ligados entre si mediante
la siguiente proporcin:

: 360

= k : 2
Haciendo uso de esta proporcin, se puede convertir la medida radial en medida en grados, y
viceversa, la medida en grados a la radial.
Ejemplo 11.1 1. El ngulo de 60

y el de

3
radianes representan un mismo ngulo, lo que
se deduce de la validez de la proporcin 60

: 360

=

3
: 2.
2. El ngulo de 90

y el de

2
radianes representan un mismo ngulo, lo que se deduce de la
validez de la proporcin 90

: 360

=

2
: 2.
3. El ngulo de 270

y el de
3
2
radianes representan un mismo ngulo, lo que se deduce de
la validez de la proporcin 270

: 360

=
3
2
: 2.
En lo sucesivo siempre se emplear slo la medida radial del ngulo. En las designaciones las
medidas de un ngulo en radianes casi siempre se omite la palabra radin. Por esta razn, en
adelante por ngulo se entiende un ngulo de radianes, es decir, un ngulo cuya medida radial
es igual a radianes; por ngulo
2
3
se entiende un ngulo de
2
3
radianes, es decir, un ngulo cuya
medida radial es igual a
2
3
radianes; por ngulo , donde es cierto nmero jo, se entiende un
ngulo de radianes, es decir, un ngulo cuya medida radial es igual a radianes; por ngulo
se entiende un ngulo, cuya medida radial es igual a radianes. Ntese, adems, que
por las palabras un ngulo tal que ,= +n, n Z se entiende que es un ngulo tal que su
medida radial no es igual al nmero +n, cualquiera que sea el nmero entero n.
A continuacin, citaremos la tabla para los ngulos y arcos que se encuentran con mayor
frecuencia.
Grados 360

180

90

60

45

30

18

15

10

Radianes 2

2

10

12

18

180

180
11.2. Crculo unitario
Denicin 11.2 Crculo unitario
Supongamos que en un plano se ha introducido un sistema rectangular de coordenadas X0Y con el
semieje positivo de abscisas 0X orientado a la derecha y el semieje positivo de ordenadas 0Y, hacia
arriba. Sea dada una circunferencia cuyo radio es igual a la unidad de medicin de longitudes con
centro en el origen de coordenadas. Tal circunferencia suele llamarse crculo unitario.
CAPTULO 11. FUNCIONES TRIGONOMTRICAS 490
Tomemos como vrtice de cualquier ngulo el origen de coordenadas, es decir, el punto 0(0, 0).
Consideramos como lado inicial el semieje positivo de abscisas, es decir, como punto de referencia
para cualquier ngulo .
Sea dado un ngulo cualquiera : es obvio que el lado
nal 0A, que describe este ngulo , cortar si falta el cr-
culo unitario en cierto punto P(a, b). No es menos evidente
que para cualquier punto Q(c, d) del crculo unitario existe
obligatoriamente un ngulo tal, que el lado nal 0A, que
describe dicho ngulo , corte el crculo unitario precisa-
mente en este punto Q(c, d). Queda claro, ante todo que: el
lado nal 0A, que describe el ngulo nulo, corta el crculo
unitario en el punto (1, 0); el lado nal 0A que describe el
ngulo , corta el crculo unitario en el punto (-1, 0); el lado
nal 0A que describe el ngulo

2
interseca el crculo unitario en el punto (0, 1); el lado nal 0A
que describe el ngulo

2
interseca el crculo unitario en el punto (0, -1).
Ejemplo 11.2 Supongamos que el lado nal 0A, que describe el ngulo

4
, corta el crculo
unitario en un punto P. Calcule las coordenadas de este punto.
Solucin
Tracemos por el punto P una recta paralela al eje 0Y, y supongamos que corta el eje 0X en el punto
Q. Por cuanto ambas coordenadas del punto P son positivas, sern iguales, respectivamente, a las
longitudes de los catetos del tringulo rectngulo issceles 0QP. Conforme al teorema de Pitgoras,
[0P[
2
= [0Q[
2
+ [PQ[
2
, como [0Q[ = [QP[, obtenemos de aqu que [0Q[ = [PQ[ =

2
2
. Por eso, la
abscisa del punto P es igual a la ordenada del punto P e igual al nmero

2
2
. Quiere decir, el lado
nal 0A que describe el ngulo

4
corta el crculo unitario en el punto P
_

2
2
,

2
2
_
.
Ejemplo 11.3 Supongamos que el lado nal 0A, que describe el ngulo de

6
, corta el crculo
unitario en el punto P. Calcule las coordenadas de este punto.
Solucin
Tracemos por el punto P una recta paralela al eje 0Y que corte el eje 0X en el punto Q. Por cuanto
ambas coordenadas del punto P son positivas, sern iguales, respectivamente, a las longitudes de
los catetos del tringulo rectngulo 0QP. Por geometra se sabe que en un tringulo rectngulo la
longitud del cateto opuesto al ngulo de

6
, es igual a la mitad de la longitud de la hipotenusa. Por
consiguiente, [PQ[ =
1
2
. De acuerdo con el teorema de Pitgoras, [0Q[
2
= [0P[
2
[PQ[
2
. De aqu
tenemos [0Q[ =

3
2
. Por eso la abscisa del punto P es igual a

3
2
, y su ordenada, a
1
2
. Quiere decir
el lado nal 0A, que describe el ngulo de

3
, corta el crculo unitario en el punto R. Calculemos
las coordenadas de dicho punto. Tracemos por el punto R una recta paralela al eje 0Y, que corta
el eje 0X en el punto S. Por cuanto ambas coordenadas del punto R son positivas, sern iguales,
respectivamente, a las longitudes de los catetos del tringulo rectngulo. Empleando la armacin
enunciada ms arriba sobre la longitud del cateto opuesto al ngulo de

6
, llegamos a que [0S[ =
1
2
,
mas, en este caso, al aplicar el teorema de Pitgoras, encontramos que [SR[ =

3
2
. Por eso, la
abscisa del punto R es igual a
1
2
, y su ordenada, a

3
2
. Quiere decir, el lado nal 0A que describe
el ngulo de

3
corta el crculo unitario en el punto R
_
1
3
,

3
2
_
.
Supongamos que el lado nal 0A, que describe el ngulo , corta el crculo unitario en cierto
punto P(a, b). En este caso es fcil ver la validez de las siguientes armaciones:
1. El lado nal 0A, que preja el ngulo + 2, corta el crculo unitario en el mismo punto
P.
CAPTULO 11. FUNCIONES TRIGONOMTRICAS 491
2. El lado nal 0A que describe el ngulo 2 corta el crculo unitario en el mismo punto
P.
3. El lado nal que describe el ngulo + corta el crculo unitario en el punto Q(a, b),
simtrico al punto P con relacin al origen de coordenadas, es decir, al punto 0(0, 0).
4. El lado nal que describe el ngulo corta el crculo unitario en un punto Q(a, b),
simtrico al punto P(a, b) respecto del eje 0X.
5. El lado nal que describe el ngulo corta el crculo unitario en un punto Q(a, b),
simtrico al punto P(a, b) respecto al eje 0Y.
CAPTULO 11. FUNCIONES TRIGONOMTRICAS 492
11.3. Funciones trigonomtricas de un ngulo
Sea introducido en un plano el sistema rectangular de coordenadas X0Y con el semieje positivo
de abscisas 0X orientado a la derecha y el semieje positivo de ordenadas 0Y, orientado hacia arriba.
Sea dada, adems un crculo unitario. Elijamos como vrtice de cualquier ngulo el origen de coor-
denadas, es decir, el punto 0(0, 0). El semieje positivo de abscisas se considera como el lado inicial
0B, es decir, como el punto de referencia en la medicin de cualquier ngulo . Supongamos que el
punto P es un punto comn del lado inicial 0B y del crculo unitario. Entonces, una parte del lado
inicial 0B, a saber, el segmento 0P se denominar radio unitario inicial, o bien punto de referencia
de los ngulos. Supongamos que el lado nal 0A coincide con el lado inicial 0B sin realizar ninguna
vuelta. Denotemos con Q el punto del lado nal 0A que coincide con el punto P del lado inicial
0B. Entonces, una parte del lado nal 0A, es decir, el segmento 0Q se denominar radio unitario
nal, y el punto Q, extremo del radio unitario nal.
Si el lado nal 0A realiza cierto giro, entonces junto
con l realizar tambin el mismo giro el radio unitario
nal 0Q. Por eso se puede considerar que el ngulo lo
describe no slo el lado nal 0A, sino tambin el radio
unitario nal 0Q.
Convengamos en decir en lo sucesivo: el radio unitario
nal 0Q describe un ngulo , sobreentendiendo por ello
que el lado nal correspondiente 0A describe el mismo
ngulo . Supongamos que el extremo del radio unitario
nal 0Q, que describe el ngulo , coincide con el punto R(a, b) del crculo unitario; entonces, las
coordenadas del punto R se llamarn coordenadas del extremo del radio unitario nal que describe
el ngulo y se notar: Q(a, b).
Denicin 11.3 Seno del ngulo
Sea dado un ngulo cualquiera . El nmero igual a la ordenada del extremo del radio unitario
nal que describe lleva el nombre de seno del ngulo y se designa Sen.
De la denicin proviene que para cualquier ngulo existe el seno de este ngulo y, adems,
es nico.
Ejemplo 11.4
1. La ordenada del extremo del radio unitario nal, que describe el ngulo nulo, es igual a
cero, por consiguiente, Sen0 = 0.
CAPTULO 11. FUNCIONES TRIGONOMTRICAS 493
2. La ordenada del extremo del radio unitario nal, que describe el ngulo es igual a cero.
Por consiguiente, Sen = 0.
3. La ordenada del extremo del radio unitario nal, que describe el ngulo de

2
, es igual a la
unidad, por consiguiente Sen

2
= 1.
4. La ordenada del extremo del radio unitario nal, que describe el ngulo

2
es igual a -1,
por consiguiente, Sen
_

2
_
= 1.
5. La ordenada del extremo del radio unitario nal, que describe el ngulo de

6
, es igual a
1
2
,
por consiguiente, Sen

6
=
1
2
.
6. La ordenada del extremo del radio unitario nal, que describe el ngulo de

4
, es igual a

2
2
, por consiguiente, Sen

4
=

2
2
.
7. La ordenada del extremo del radio unitario nal, que describe el ngulo de

3
, es igual a

3
2
, por consiguiente, Sen

3
=

3
2
.
A continuacin damos a conocer algunas propiedades del seno de un ngulo. Por cuanto, para
cualquier ngulo , la ordenada del extremo del radio unitario nal, que describe dicho ngulo
, no puede ser menor de -1 y mayor de 1, encontrndose encerrada entre los valores aducidos,
incluidos -1 y 1, entonces, cualquiera que sea el ngulo , se verica la desigualdad doble
1 Sen 1.
Si la ordenada del extremo del radio unitario nal, que describe el ngulo , es el numero b,
entonces, segn lo expuesto antes, la ordenada del extremo del radio unitario nal, que describe el
ngulo , ser el numero b. Por eso, para cualquier ngulo se verica la igualdad
Sen() = Sen.
Esta propiedad del seno de un ngulo puede enunciarse como: el signo menos puede sacarse del
signo del seno o introducirse bajo el signo del seno, es decir:
Sen() = Sen = Sen().
Segn lo indicado anteriormente, la ordenada del extremo del radio unitario nal, que describe
el ngulo , es igual a la ordenada del extremo del radio unitario nal que preja el ngulo .
Por eso, para cualquier ngulo se verica la igualdad
Sen( ) = Sen.
Ejemplo 11.5 Simplique las expresiones:
a) Sen
_

2
3
_
; b) Sen
5
6
.
Solucin
a) Sen
_

2
3
_
= Sen
2
3
= Sen
_


3
_
= Sen

3
=

3
2
;
b) Sen
5
6
= Sen
_


6
_
= Sen

6
=
1
2
.
Si la ordenada del extremo del radio unitario nal, que describe el ngulo , es el numero b,
entonces, segn lo expuesto anteriormente, la ordenada del extremo del radio unitario nal, que
describe el ngulo +, ser el numero b. Por eso, para cualquier ngulo se verica la igualdad
Sen( +) = Sen
CAPTULO 11. FUNCIONES TRIGONOMTRICAS 494
Ejemplo 11.6 Simplique las expresiones:
a) Sen
7
6
; b) Sen
11
6
.
Solucin
a) Sen
7
6
= Sen
_
+

6
_
= Sen

6
=
1
2
;
b) Sen
11
6
= Sen
_
2

4
_
= Sen
_

4
_
=

2
2
.
Segn lo indicado ms arriba, la ordenada del extremo del radio unitario nal, que describe el
ngulo , es igual a la ordenada del extremo del radio unitario nal que describe el ngulo +2
e igual a la ordenada del extremo del radio unitario nal que describe el ngulo 2. Por eso,
para cualquier ngulo se verican las igualdades
_
Sen = Sen( + 2)
Sen = Sen( 2)
Haciendo uso de estas igualdades y aplicando el mtodo de induccin matemtica, se puede
mostrar que para cualquier nmero entero n y todo ngulo se verican las igualdades
_
Sen = Sen( + 2n)
Sen = Sen( 2n)
Esta propiedad del seno de un ngulo puede enunciarse de la siguiente manera: el seno de
cualquier ngulo se repite, al variar el ngulo en la magnitud de 2n, donde n es un numero
entero cualquiera.
Ejemplo 11.7 Simplique las expresiones:
a) Sen
7
4
; b) Sen
25
6
.
Solucin
a) Sen
7
4
= Sen
_
2

4
_
= Sen
_

4
_
= sen

4
=

2
2
;
b) Sen
25
6
= Sen
_
4 +

6
_
= Sen

6
=
1
2
.
Sea dado un nmero (0; ). Examinemos un ngulo cuya medida radial es el nmero . El
extremo del radio unitario nal, que describe dicho ngulo, coincide con cierto punto del crculo
unitario dispuesto en el primero o en el segundo cuadrantes, o bien en el semieje positivo de orde-
nadas. Por eso, la ordenada del extremo del radio unitario nal, que describe el ngulo mencionado,
es positiva. Es decir, el seno de este ngulo es positivo.
Tomando en consideracin que Sen = Sen( +2n) para cualquier nmero entero n, se puede
armar que Sen es positivo para cualquier ngulo tal, que su medida radial, el nmero ,
pertenece, para cierto n entero, al intervalo correspondiente (2n; + 2n).
De modo anlogo es tambin vlida la siguiente armacin: Sen es negativo para cualquier
ngulo tal, que su medida radial, el nmero , pertenece, con cierto n entero, al intervalo
( +2n; 2 +2n). En la recta numrica, estn mostrados tales intervalos, que para cada nmero
, perteneciente a cualquiera de ellos, Sen es negativo.
CAPTULO 11. FUNCIONES TRIGONOMTRICAS 495
En n, teniendo presente que Sen = Sen(+2n) para todo nmero n entero y que Sen0 =
Sen = 0, resulta que Sen es igual a cero para cualquier ngulo tal, que su medida radial, el
nmero , es igual al nmero m, siendo m Z.
Denicin 11.4 Coseno del ngulo
Sea dado un ngulo cualquiera . El nmero igual a la abscisa del extremo del radio unitario nal,
que describe dicho ngulo se denomina coseno del ngulo y se designa Cos.
De la denicin se deduce que para cualquier ngulo existe el coseno de este ngulo y es,
adems, nico.
Ejemplo 11.8 1.- La abscisa del extremo del radio unitario nal, que describe el ngulo

2
,
es igual a cero, por consiguiente, Cos

2
= 0.
2.- La abscisa del extremo del radio unitario nal, que describe el ngulo

2
, es igual a cero, por
consiguiente, Cos
_

2
_
= 0.
3.- La abscisa del extremo del radio unitario nal, que describe el ngulo nulo, es igual a la
unidad, por consiguiente Cos0 = 1.
4.- La abscisa de los extremos del radio unitario nal, que describe el ngulo , es igual a -1, por
consiguiente, Cos = 1.
5.- La abscisa del extremo del radio unitario nal, que describe el ngulo

6
, es igual a

3
2
, por
consiguiente, Cos

6
=

3
2
.
6.- La abscisa del extremo del radio unitario nal, que describe el ngulo

4
es igual a

2
2
, por
consiguiente, Cos

4
=

2
2
.
7.- La abscisa del extremo del radio unitario nal, que describe el ngulo

3
, es igual a
1
2
, por
consiguiente, Cos

3
=
1
2
.
CAPTULO 11. FUNCIONES TRIGONOMTRICAS 496
Por cuanto para cualquier ngulo la abscisa del extremo del radio unitario nal, que describe
el ngulo , no puede ser menor que -1 y mayor que 1, encontrndose encerrada entre dichos
valores, incluidos -1 y 1, entonces para todo ngulo se verica la siguiente desigualdad doble
1 Cos 1
Segn lo visto anteriormente, la abscisa correspondiente al extremo del radio unitario nal, que
describe el ngulo , es igual a la abscisa del extremo del radio unitario nal que describe el ngulo
. Por eso, para todo ngulo se verica la igualdad
Cos() = Cos
Esta propiedad del coseno de un ngulo puede enunciarse de la siguiente manera: el signo
delante de un ngulo que est bajo el signo del coseno se puede cambiar sin variar el valor del
coseno del ngulo, es decir,
Cos() = Cos = Cos()
Supongamos que la abscisa correspondiente al extremo del radio unitario nal, que describe
un ngulo A, es el nmero k; entonces, de acuerdo con lo mostrado anteriormente, la abscisa del
extremo del radio unitario nal, que describe el ngulo A, es el nmero k. Por eso, para todo
ngulo A se verica la igualdad
Cos( ) = Cos
Ejemplo 11.9 Simplique las expresiones:
a) Cos
5
6
; b) Cos
_

2
3
_
.
Solucin
a) Cos
5
6
= Cos
_


6
_
= Cos

6
=

3
2
;
b) Cos
_

2
3
_
= Cos
2
3
= Cos
_


3
_
= Cos

3
=
1
2
.
Sea el nmero k la abscisa correspondiente al extremo del vector unitario nal que describe
el ngulo ; entonces, segn lo mostrado anteriormente, la abscisa del extremo del radio unitario
nal, que describe el ngulo + , es el nmero k. Por eso, para cualquier ngulo se verica
la igualdad
Cos( +) = Cos
Ejemplo 11.10 Simplique las expresiones:
a) Cos
_

7
6
_
; b) Cos
5
4
.
Solucin
a) Cos
_

7
6
_
= Cos
7
6
= Cos
_
+

6
_
= Cos

6
=

3
2
;
b) Cos
5
4
= Cos
_
+

4
_
= Cos

4
=

2
2
.
Como se ha indicado anteriormente, la abscisa del extremo del radio unitario nal, que describe
un ngulo , es igual a la abscisa del extremo del radio unitario nal que describe el ngulo +2
CAPTULO 11. FUNCIONES TRIGONOMTRICAS 497
y es igual a la abscisa del extremo del radio unitario nal que describe el ngulo 2. Por eso,
para cualquier ngulo se verican las igualdades
_
Cos = Cos( + 2)
Cos = Cos( 2)
Haciendo uso de estas igualdades y aplicando el mtodo de induccin matemtica, podemos
mostrar que para todo nmero entero n y para cualquier ngulo se verican las igualdades
_
Cos = Cos( + 2n)
Cos = Cos( 2n)
Esta propiedad del coseno puede enunciarse de la siguiente manera: el coseno de cualquier ngu-
lo se repite, al cambiar el ngulo en la magnitud de 2n, donde n es un nmero entero cualquiera.
Ejemplo 11.11 Simplique las expresiones:
a) Cos
7
4
; b) Cos
25
6
.
Solucin
a) Cos
7
4
= Cos
_
2

4
_
= Cos
_

4
_
= Cos

4
=

2
2
;
b) Cos
25
6
= Cos
_
4 +

6
_
= Cos

6
=

3
2
.
Sea dado un nmero
_

2
;

2
_
. Examinemos un
ngulo cuya medida radial es el nmero . El extremo
del radio unitario nal, que describe este ngulo coincide
con cierto punto del circulo unitario dispuesto o bien en
el cuadrante I o bien en el cuadrante IV, o bien en el
semieje positivo de abscisas. Por eso, la abscisa corre-
spondiente al extremo del radio unitario nal, que de-
scribe el ngulo dado, es positiva. Con otras palabras, el
coseno de este ngulo es positivo. Teniendo presente que
Cos = Cos(+2n) para cualquier nmero n positivo,
podemos armar que Cos es positivo para todo ngulo
tal, que la medida radial de ste pertenece, para cierto
n entero, al intervalo
_

2
+ 2n;

2
+ 2n
_
.
Tomando en consideracin que Cos = Cos( + 2n) para toda n entera y que Cos

2
=
Cos
_

2
_
= 0, resulta que Cos es igual a cero para cualquier ngulo tal, que la medida radial
de ste es igual, con cierto n entero, al nmero

2
+n.
Ejemplo 11.12 Simplique la expresin:
Sen

3
Cos
_

6
_
Cos
3
4
Sen
_

4
_
CAPTULO 11. FUNCIONES TRIGONOMTRICAS 498
Solucin
F = Sen

3
Cos

6

_
Cos

4
_ _
Sen

4
_
=

3
2

3
2

1

2
=
3
4

1
2
=
1
4
.
Denicin 11.5 Tangente del ngulo
Sea dado un ngulo cualquiera tal que ,=

2
+ n, n Z. Se denomina tangente de dicho
ngulo un nmero igual a la razn entre el seno del ngulo y el coseno del mismo y se designa
Tan.
De la denicin se deduce que para cualquier ngulo tal que ,=

2
+n, n Z, la tangente
de este ngulo existe y es, adems, nica.
Teorema 11.1 Para cualquier ngulo tal que ,=

2
+n, n Z, se verica la igualdad
Tan() = Tan
Demostracin
Para cualquier ngulo se verican las igualdades Sen() = Sen y Cos() = Cos, por lo
cual para cualquier ngulo tal que ,=

2
+ n, n Z, tendremos, de acuerdo con la denicin
de tangente:
Tan() =
Sen()
Cos()
=
Sen
Cos
=
Sen
Cos
= Tan.
Teorema 11.2 Para todo ngulo tal que ,=

2
+n, n Z, se verican las igualdades
_
Tan = Tan( +)
Tan = Tan( )
Demostracin
Para todo ngulo de esta ndole resulta verdadera la igualdad
Tan( +) =
Sen( +)
Cos( +)
=
Sen
Cos
=
Sen( )
Cos(
= Tan( ).
CAPTULO 11. FUNCIONES TRIGONOMTRICAS 499
Ejemplo 11.13 Simplique las expresiones:
a) Tan
5
6
; b) Tan
_

2
3
_
; c) Tan
_

7
6
_
; d) Tan
5
4
.
Solucin
a) Tan
5
6
= Tan
_


6
_
= Tan
_

6
_
= Tan

6
=
1

3
;
b) Tan
_

2
3
_
= Tan
2
3
= Tan
_


3
_
= Tan
_

3
_
= Tan

3
=

3;
c) Tan
_

7
6
_
= Tan
7
6
= Tan
_
+

6
_
= Tan

6
=
1

3
;
d) Tan
5
4
= Tan
_
+

4
_
= Tan

4
= 1.
Aprovechando las igualdades Tan = Tan( + ) = Tan( ) y aplicando el mtodo de
induccin matemtica, se puede mostrar que para todo numero entero n y para cualquier ngulo
tal que ,=

2
+n, n Z, se verican las igualdades
_
Tan = Tan( +n)
Tan = Tan( n)
Ejemplo 11.14 Simplique las expresiones:
a) Tan
7
4
; b) Tan
25
6
; c) Tan
4
3
; d) Tan
5
6
.
Solucin
a) Tan
7
4
= Tan
_
2

4
_
= Tan
_

4
_
= Tan

4
= 1;
b) Tan
25
6
= Tan
_
4 +

6
_
= Tan

6
=
1

3
;
c) Tan
4
3
= Tan
_
+

3
_
= Tan

3
=

3;
d) Tan
5
6
= Tan
_


6
_
= Tan
_

6
_
= Tan

6
=
1

3
.
Para cualquier ngulo cuyos seno y coseno son de un mismo signo, la tangente del ngulo
es positiva, es decir, Tan es positiva para todo ngulo que se determina por el radio unitario
nal, cuyo extremo coincide con un punto del crculo unitario dispuesto en los cuadrantes I o III,
es decir, para todo nmero que interviene como medida radial del ngulo correspondiente ,
perteneciente, para cierto n entero, al intervalo
_
n;

2
+n
_
.
Para cualquier ngulo , cuyos seno y coseno son de
signos opuestos, la tangente del ngulo es negativa, es
decir, Tan es negativa para cualquier ngulo que se
determina por el radio unitario nal cuyo extremo coin-
cide con un punto del crculo unitario dispuestos en los
cuadrantes II o IV, es decir, para cualquier nmero
que interviene como medida radial del ngulo correspon-
diente , perteneciente, para cierto n entero, al intervalo
_

2
+n; n
_
.
Para cualquier ngulo , cuyo seno es igual a cero, la
tangente del ngulo es tambin nula, es decir Tan = 0 para todo ngulo determinado por el
CAPTULO 11. FUNCIONES TRIGONOMTRICAS 500
radio unitario nal cuyo extremo coincide o bien con el punto P(1, 0) o bien con el punto Q(1, 0),
es decir, para cualquier nmero que interviene como medida radial del ngulo correspondiente
, igual, para cierto n entero, al nmero n.
Denicin 11.6 Tangente del ngulo
Sea dado un ngulo cualquiera tal que ,=

2
+ n, n Z, y supongamos que el extremo del
radio unitario mvil, que determina dicho ngulo , es el punto R(a, b), se denomina tangente del
ngulo el nmero igual a la razn de la ordenada del punto R a la abscisa del mismo punto R,
es decir Tan =
b
a
.
Es fcil darse cuenta que la recta que pasa por el origen de coordenadas y el punto Q(a, b)
corta la recta x = 1 en el punto R(1, b/a). Es decir, la recta que pasa por el origen de coordenadas
y el extremo del radio unitario nal, que describe el ngulo , corta la recta x = 1 en el punto
R(1, Tan). Por esta razn la recta x = 1 se llama lnea de las tangentes.
Ejemplo 11.15 Calcular el valor de la fraccin
Sen Cos
Sen +Cos
para Tan =
2
5
.
Solucin
Dividamos el numerador y el denominador de dicha fraccin por Cos ,= 0, por lo que la magnitud
de la fraccin no vara
F =
SenCos
Cos
Sen+Cos
Cos
=
Tan 1
tan + 1
=
2
5
1
2
5
+ 1
=
3
7
.
CAPTULO 11. FUNCIONES TRIGONOMTRICAS 501
Ejemplo 11.16 Determine el valor de la expresin
Cos
_
100 +

3
_
Sen
_
20
5
6
_
Tan
_
11 +

4
_
Solucin
E = Cos

3
Sen
_

5
6
_
Tan

4
= Cos

3
Sen
5
6
Tan

4
= Cos

3
Sen

6
Tan

4
=
1
2

1
2
1
=
1
4
.
Ejemplo 11.17 Determine el valor de la expresin
Tan
_

3
_
Tan
_

113
2


6
_
_
Cos
_

2
_
+Sen
_

4
_
Cos
3
4
Solucin
F =
Tan

3
Tan
_
56
2
3
_
_
Cos

2
Sen

4
_
Cos

4
_
=

3 Tan
2
3
_

2
__

2
_
=

3 Tan

3
1
2
=

3
1
2
= 6.
Ejemplo 11.18 Determine el valor de la expresin
Sen
13
2
Cos113 +Tan
7
4
Cos
_

15
2
_
Sen
_

49
6
_
Cos
51
3
+Tan
_

113
4
_
Solucin
CAPTULO 11. FUNCIONES TRIGONOMTRICAS 502
F =
Sen
_
6 +

2
_
Cos(112 +) +Tan
_
+
3
4
_
Cos
_
7

2
_
Sen
_
8

6
_
Cos(16 +) +Tan
_
28

4
_
=
Sen

2
Cos +Tan
3
4
Cos
3
2
Sen

6
Cos Tan

4
=
1 (1) +
_
Tan

4
_ _
Cos

2
_

1
2
(1) 1
=
1 + 1 0
1
2
1
= 2.
Denicin 11.7 Cotangente del ngulo
Sea dado un ngulo cualquiera tal que ,= m, m Z. Se denomina cotangente del ngulo el
nmero igual a la razn entre el coseno de este ngulo y el seno del mismo y se designa Cot.
De la denicin se desprende que para cualquier ngulo tal que ,= m, m Z, la cotangente
de dicho ngulo existe y, adems, es nica.
Teorema 11.3 Para cualquier ngulo tal que ,= m, m Z, se verica la igualdad
Cot() = Cot
Demostracin
Para todo ngulo de este gnero es vlida la igualdad
Cot() =
Cos()
Sen()
=
Cos
Sen
=
Cos
Sen
= Cot.
Teorema 11.4 Para cualquier ngulo tal que ,= m, m Z, son vlidas las igualdades
Cot = Cot( +) = Cot( )
Demostracin
Para cualquier ngulo de ste gnero son vlidas las igualdades
Cot( +) =
Cos( +)
Sen( +)
=
Cos
Sen
=
Cos( )
Sen( )
= Cot( ).
CAPTULO 11. FUNCIONES TRIGONOMTRICAS 503
Para cualquier ngulo A tal que A ,= m, m Z, son vlidas las igualdades
_
CotA = Cot(A+)
CotA = Cot(A)
Ejemplo 11.19 Simplique las expresiones:
a) Cot
5
6
; b) Cot
_

2
3
_
; c) Cot
_

7
6
_
; d) Cot
5
4
.
Solucin
a) Cot
5
6
= Cot
_


6
_
= Cot
_

6
_
= Cot

6
=

3;
b) Cot
_

2
3
_
= Cot
2
3
= Cot
_


3
_
= Cot
_

3
_
= Cot

3
=
1

3
;
c) Cot
_

7
6
_
= Cot
7
6
= Cot
_
+

6
_
= Cot

6
=

3;
d) Cot
5
4
= Cot
_
+

4
_
= Cot

4
= 1.
Aprovechando las igualdades Cot = Cot( + ) = Cot( ) y aplicando el mtodo de
induccin matemtica, se puede mostrar que para todo nmero entero n y todo ngulo tal que
,= m, m Z, se verican las igualdades
_
Cot = Cot( +n)
Cot = Cot( n)
Ejemplo 11.20 Simplique las expresiones:
a) Cot
7
4
; b) Cot
25
6
; c) Cot
4
3
; d) Cot
5
6
.
Solucin
a) Cot
7
4
= Cot
_
2

4
_
= Cot
_

4
_
= Cot

4
= 1;
b) Cot
25
6
= Cot
_
4 +

6
_
= Cot

6
= Cot

6
=

3;
c) Cot
4
3
= Cot
_
+

3
_
= Cot

3
=
1

3
;
d) Cot
5
6
= Cot
_


6
_
= Cot
_

6
_
= Cot

6
=

3.
Para cualquier ngulo , cuyos coseno y seno son de
un mismo signo, la cotangente del ngulo es positi-
va, es decir, Cot es positiva para cualquier ngulo
determinado por el radio unitario nal cuyo extremo co-
incide con el punto del crculo unitario dispuesto en los
cuadrantes I o III, es decir, para cualquier nmero que
interviene como medida radial del ngulo correspondi-
ente , perteneciente, para cierto r entero, al intervalo
_
r;

2
+r
_
.
Para cualquier ngulo , cuyos coseno y seno son de
signos opuestos, la cotangente del ngulo es negativa,
es decir, Cot es negativa para cualquier ngulo prejado por el radio unitario nal cuyo ex-
tremo coincide con el punto del crculo unitario dispuesto en los cuadrantes I o IV, es decir, para
CAPTULO 11. FUNCIONES TRIGONOMTRICAS 504
cualquier nmero que interviene como medida radial del ngulo correspondiente , perteneciente,
para cierto k entero, al intervalo
_

2
+r; +k
_
.
Para cualquier ngulo A, cuyo coseno es igual a cero, la cotangente del ngulo A es tambin
nula, es decir, CotA = 0 para todo ngulo A determinado por el radio unitaria nal cuyo extremo
coincide o bien con el punto P(0, 1), o bien con el punto Q(0, 1), es decir, para todo nmero
A que interviene como medida radial del ngulo correspondiente A igual, para cierto nmero n
entero, al nmero

2
+n.
Denicin 11.8 Cotangente del ngulo
Sea dado un ngulo cualquiera tal que ,= m, m Z, y supongamos que el extremo del radio
unitario nal, que determina este ngulo, es el punto Q(a, b), con la particularidad de que b ,= 0,
a consecuencia de que ,= m, m Z; se denomina cotangente del ngulo un nmero igual a
la razn de la abscisa del punto Q a la ordenada del mismo punto Q, es decir, Cot =
a
b
.
Es fcil ver que la recta que pasa por el origen de coordenadas y el punto Q(a, b) interseca la
recta y = 1 en el punto R(a/b, 1). Es decir, la recta que pasa por el origen de coordenadas y el
extremo del radio unitario nal, que describe el ngulo , interseca la recta y = 1 en el punto
R(Cot, 1). Por este motivo, la recta y = 1 se denomina lnea de cotangentes.
Ejemplo 11.21 Determine el valor de la expresin
Sen
_
112 +

2
_
Cos
_
17
2
+

3
_
Tan

4
Cot
3
4
Solucin
F =
Sen

2
Cos
_
8 +
5
6
_
Tan

4
+Cot

4
=
1 Cos
5
6
1 + 1
=
Cos

6
2
=

3
4
.
CAPTULO 11. FUNCIONES TRIGONOMTRICAS 505
Ejemplo 11.22 Determine el valor de la expresin
Cot
_

4
_

_
Tan() Cos
_

5
6
_
Tan
_

3
10
_
Cos
_

2
3
+ 10
_
Solucin
F =
Cot

4

_
Tan Cos
5
6
_
Tan
_

3
+ 10
_
Cos
_

2
3
+ 10
_
=
1 Cos
5
6
Tan

3
Cos
_

2
3
_
=

3
2

3 Cos

3
=

3
2

3
1
2
= 1.
Ejemplo 11.23 Dado Tan +Cot = 3. Calcular Tan
2
+Cot
2
.
Solucin
Tan
2
+Cot
2
= (Tan +Cot)
2
2
= 3
2
2
= 7.
Para facilitar la comprensin de las frmulas de reduccin, damos las siguientes tablas:
Argumento Frmulas reducidas del seno Frmulas reducidas del coseno

2
Sen
_

2

_
= Cos Cos
_

2

_
= Sen

2
+ Sen
_

2
+
_
= Cos Cos
_

2
+
_
= Sen
Sen( ) = Sen Cos( ) = Cos
+ Sen( +) = Sen Cos( +) = Cos
3
2
Sen
_
3
2

_
= Cos Cos
_
3
2

_
= Sen
3
2
+ Sen
_
3
2
+
_
= Cos Cos
_
3
2
+
_
= Sen
2k Sen( 2k) = Sen Cos( 2k) = Cos
+ 2k Sen( + 2k) = Sen Cos( + 2k) = Cos
Argumento Frmulas reducidas de la tangente Frmulas reducidas de la cotangente

2
Tan
_

2

_
= Cot Cot
_

2

_
= Tan

2
+ Tan
_

2
+
_
= Cot Cot
_

2
+
_
= Tan
Tan( ) = Tan Cot( ) = Cot
+ Tan( +) = Tan Cot( +) = Cot
3
2
Tan
_
3
2

_
= Cot Cot
_
3
2

_
= Tan
3
2
+ Tan
_
3
2
+
_
= Cot Cot
_
3
2
+
_
= Tan
k Tan( k) = Tan Cot( k) = Cot
+k Tan( +k) = Tan Cot( +k) = Cot
CAPTULO 11. FUNCIONES TRIGONOMTRICAS 506
Sen Cos Tan Cot Sec Csc
0 0 1 0 1

12

2
4

6 +

2
4
2

3 2 +

6 +

6
1
2

3
2

3
3

3
2

3
3
2

2
2

2
2
1 1

3
2
1
2

3
3
2
2

3
3
5
12

6 +

2
4

2
4
2 +

3 2

6 +

2
1 0 0 1
7
12

6 +

2
4

6
4
2

3 2 +

2
2
3

3
2

1
2

3
3
2
2

3
3
3
4

2
2

2
2
1 1

2
5
6
1
2

3
2

3
3

3
2

3
3
2
11
12

6
4

6 +

2
4
2 +

3 2

6 +

6 +

2
0 1 0 1
13
12

6
4

6 +

2
4
2

3 2 +

6 +

2
7
6

1
2

3
2

3
3

3
2

3
3
2
5
4

2
2

2
2
1 1

2
4
3

3
2

1
2

3
3
2
2

3
3
17
12

6 +

2
4

6
4
2 +

3 2

6 +

2
3
2
1 0 0 1
19
12

6 +

2
4

2
4
2

3 2 +

6 +

6 +

2
5
3

3
2
1
2

3
3
2
2

3
3
7
4

2
2

2
2
1 1

2
11
6

1
2

3
2

3
3

3
2

3
3
2
23
12

2
4

6 +

2
4
2 +

3 2

2
2 0 1 0 1
CAPTULO 11. FUNCIONES TRIGONOMTRICAS 507
11.4. Identidades trigonomtricas
Teorema 11.5 El cuadrado del seno de cualquier ngulo ms el cuadrado del coseno del mismo
ngulo es igual a la unidad. Es decir, Para cualquier ngulo A se verica la igualdad
Sen
2
A+Cos
2
A = 1
Demostracin
Sea dado cierto ngulo . Entonces las coordenadas
del extremo del radio unitario nal que describe el n-
gulo sern (Cos, Sen). Por cuanto el cuadrado de
la distancia entre dos puntos cualesquiera de un plano
determinados por sus coordenadas es igual a la suma
de los cuadrados de la diferencia entre las coordenadas
homnimas, entonces para los puntos (Cos, Sen) y (0,
0) tenemos
(Cos 0)
2
+ (Sen 0)
2
= 1
2
o bien
Sen
2
+Cos
2
= 1.
Ejemplo 11.24 Demuestre la identidad
(Sen +Cos)
2
1
Cot SenCos
= 2Tan
2

Solucin
F =
Sen
2
+ 2SenCos +Cos
2
1
Cos
Sen
SenCos
=
2SenCos
_
1
Sen
Sen
_
Cos
=
2Sen
1Sen
2

Sen
=
2Sen

Cos
2

= 2Tan
2
.
Ejemplo 11.25 Hallar Cos, si Tan =
1
3
, donde
_
;
3
2
_
.
Solucin
Sabemos que
Sen
Cos
=
1
3
Sen =
1
3
Cos
Reemplazando, obtenemos
Sen
2
+Cos
2
= 1
_
1
3
Cos
_
2
+Cos
2
= 1
CAPTULO 11. FUNCIONES TRIGONOMTRICAS 508
1
9
Cos
2
+Cos
2
= 1 Cos
2
=
9
10
De donde Cos =
3

10
. Como
_
;
3
2
_
, entonces tomamos Cos =
3

10
.
Ejemplo 11.26 Dado Sen +Cos =
1
2
. Determine:
a) SenCos; b) Sen Cos; c) Sen
3
+Cos
3
; d) Sen
4
+Cos
4
.
Solucin
a) Tomando la hiptesis, tenemos
Sen +Cos =
1
2
(Sen +Cos)
2
=
1
4
Sen
2
+Cos
2
+ 2SenCos =
1
4
1 + 2SenCos =
1
4
SenCos =
3
8
b) Tomando la hiptesis, tenemos
(Sen +Cos)
2
=
1
4
(Sen +Cos)
2
2SenCos =
1
4
2SenCos
(Sen Cos)
2
+ 2SenCos =
1
4
2SenCos (Sen Cos)
2
=
1
4
4SenCos
(Sen Cos)
2
=
1
4
4
_

3
8
_
(Sen Cos)
2
=
7
4
Sen Cos =

7
2
c) Haciendo
Sen
3
+Cos
3
= (Sen +Cos)(Sen
2
SenCos +Cos
2
)
=
1
2

_
1 +
3
8
_
=
11
16
.
d) Haciendo
Sen
4
+Cos
4
= (Sen
2
+Cos
2
)
2
2Sen
2
Cos
2
+Cos
2

= 1 2(SenCos)
2
= 1 2
9
64
=
23
32
.
La identidad trigonomtrica fundamental muestra en qu dependencia se encuentran el seno
y el coseno de un mismo ngulo. Conociendo una de las magnitudes que guran en la identidad
trigonomtrica fundamental para cierto ngulo , se puede hallar la otra magnitud del mismo
ngulo . En efecto, la identidad trigonomtrica fundamental es equivalente a la igualdad
Cos
2
= 1 Sen
2

la cual es equivalente, a su vez, a la siguiente:


[Cos[ =
_
1 Sen
2

CAPTULO 11. FUNCIONES TRIGONOMTRICAS 509


De esta igualdad tenemos que
Cos =
_
1 Sen
2

para cualquier ngulo con el que Cos es no negativo, es decir, para cualquier perteneciente,
con cierto m Z, al intervalo
_
2m

2
;

2
+ 2m

. En el otro caso
Cos =
_
1 Sen
2

para cualquier ngulo con el que Cos es no positivo, es decir, para cualquier , perteneciente,
con cierto m Z, al intervalo
_
2m +

2
;
3
2
+ 2m

.
Luego, la identidad trigonomtrica fundamental es equivalente a la igualdad
Sen
2
= 1 Cos
2

la cual es equivalente a la siguiente:


[Sen[ =
_
1 Cos
2

De esta igualdad tenemos que


Sen =
_
1 Cos
2

para cualquier ngulo con el que Sen es no negativo, es decir, para cualquier perteneciente,
con cierto m Z, al intervalo [2m; + 2m]. En el otro caso
Sen =
_
1 Cos
2

para cualquier ngulo con el que Sen es no positivo, es decir, para cualquier perteneciente,
con cierto m Z, al intervalo [ + 2m; 2 + 2m].
Para los valores de frontera del ngulo , es decir, cuando =

2
+ m, donde m Z, las
frmulas
Cos =
_
1 Sen
2
y Cos =
_
1 Sen
2

dan un mismo valor de Cos = 0; las frmulas


Sen =
_
1 Cos
2
y Sen =
_
1 Cos
2

dan en las mismas condiciones, cuando = m, donde m Z, un mismo valor de Sen = 0.


Teorema 11.6 Para cualquier ngulo tal que ,=

2
+m, m Z, se verica la igualdad
1 +Tan
2
= Sec
2

Demostracin
Por cuanto ,=

2
+m, m Z, entonces Cos ,= 0, y por esta razn la identidad trigonomtrica
Sen
2
+Cos
2
= 1 puede dividirse trmino a trmino por Cos
2
. En este caso para cualquier
tenemos
1
Cos
2

=
Cos
2
+Sen
2

Cos
2

=
Cos
2

Cos
2

+
Sen
2

Cos
2

= 1 +Tan
2
.
CAPTULO 11. FUNCIONES TRIGONOMTRICAS 510
Esta igualdad, muestra en qu dependencia se encuentran la tangente y el coseno de un mismo
ngulo . Si se conoce una de las magnitudes que guran en la igualdad, se puede hallar, para
cierto ngulo de esta ndole, la otra magnitud del mismo ngulo. Efectivamente, por cuanto
,=

2
+m, donde m Z, la igualdad es equivalente a la igualdad
Cos
2
=
1
1 +Tan
2

la cual es equivalente a su vez a la siguiente:


[Cos[ =
1
_
1 +Tan
2

De esta igualdad, tenemos


Cos =
1
_
1 +Tan
2

para cualquier ngulo con el que Cos es positivo, es decir, para cualquier perteneciente, con
cierto m Z, al intervalo
_
2m

2
;

2
+ 2m

. En el otro caso
Cos =
1
_
1 +Tan
2

para cualquier ngulo con el que Cos es negativo, es decir, para cualquier perteneciente, con
cierto m Z, al intervalo
_
2m +

2
;
3
2
+ 2m

.
Luego, la igualdad original es equivalente a la igualdad
Tan
2
=
1 Cos
2

Cos
2

la cual es equivalente a la siguiente


[Tan[ =
_
1 Cos
2

[Cos[
De esta igualdad, tenemos
Tan =
_
1 Cos
2

Cos
para cualquier ngulo con el que Tan y Cos son de un mismo signo, es decir, para cualquier
, perteneciente, con cierto m Z, al conjunto
_
2m;

2
+ 2m

2
+ 2m; + 2m

. Por otro
lado
Tan =
_
1 Cos
2

Cos
para cualquier ngulo con el que Tan y Cos son de signos opuestos, es decir, para cualquier
, perteneciente, con cierto m Z, al conjunto
_
2m; ;
pi
2
+ 2m

2
+ 2m; + 2m

.
Para los valores de frontera del ngulo , es decir, cuando = m, m Z, las frmulas
Tan =
_
1 Cos
2

Cos
y Tan =
_
1 Cos
2

Cos
dan el mismo valor de Tan = 0.
CAPTULO 11. FUNCIONES TRIGONOMTRICAS 511
Ejemplo 11.27 Demuestre la identidad
Tan
2
Sen
2
= Tan
2
Sen
2

Solucin
Partiendo del lado derecho de la identidad, tenemos
Tan
2
Sen
2
=
Sen
2

Cos
2

Sen
2

=
1 Cos
2

Cos
2

Sen
2

=
_
1
Cos
2

1
_
Sen
2

=
Sen
2

Cos
2

Sen
2

= Tan
2
Sen
2
.
Ejemplo 11.28 Demuestre la identidad
Tan +
1
Cos
3


1
Sec Tan
=
Sen
2

Cos
3

Solucin
Partiendo del lado izquierdo de la identidad, tenemos
E =
Sen
Cos
+
1
Cos
3


1
1
Cos

Sen
Cos
=
Sen
Cos
+
1
Cos
3


Cos
1 Sen
=
Sen
Cos
+
1
Cos
3


(1 +Sen)Cos
(1 Sen)(1 +Sen)
=
Sen
Cos
+
1
Cos
3


(1 +Sen)Cos
Cos
2

=
SenCos
2
+ 1 Cos
2
SenCos
2

Cos
3

=
1 Cos
2

Cos
3

=
Sen
2

Cos
3

.
Teorema 11.7 Para cualquier ngulo tal que ,= m, m Z, se verica la igualdad
1 +Cot
2
=
1
Sen
2

Demostracin
Por cuanto ,= m, m Z, entonces Sen ,= 0, y por esta razn la identidad trigonomtrica
Sen
2
+Cos
2
= 1 puede dividirse trmino a trmino por Sen
2
. En este caso para cualquier
CAPTULO 11. FUNCIONES TRIGONOMTRICAS 512
tenemos
1
Sen
2

=
Cos
2
+Sen
2

Sen
2

=
Cos
2

Sen
2

+
Sen
2

Sen
2

= 1 +Cot
2
.
Esta igualdad, muestra en qu dependencia se encuentran la cotangente y el seno de un mismo
ngulo . Al conocer una de las magnitudes que guran en la igualdad, para cierto ngulo , se
puede hallar la otra magnitud del mismo ngulo . En efecto, puesto que ,= m, m Z, entonces
la igualdad es equivalente a la igualdad
Sen
2
=
1
1 +Cot
2

la cual es equivalente, a su vez, a la siguiente:


[Sen[ =
1
_
1 +Cot
2

De esta igualdad, tenemos


Sen =
1
_
1 +Cot
2

para cualquier ngulo con el que Sen es positivo, es decir, para cualquier perteneciente, con
cierto m Z, al intervalo (2m; + 2m). Por otro lado
Sen =
1
_
1 +Cot
2

para cualquier ngulo con el que Sen es negativo, es decir, para cualquier perteneciente, con
cierto m Z, al intervalo ( + 2m; 2 + 2m).
Luego, por cuanto ,= m, m Z, entonces la igualdad original es equivalente a la igualdad
Cot
2
=
1 Sen
2

Sen
2

la cual es equivalente, a su vez, a la siguiente


[Cot[ =
_
1 Sen
2

[Sen[
De esta igualdad, tenemos
Cot =
_
1 Sen
2

Sen
para cualquier ngulo con el que Cot y Sen son de un mismo signo, es decir, para cualquier
perteneciente, con cierto m Z, al conjunto
_
2m

2
; 2m
_

_
2m;

2
+ 2m

. Por otro lado


Cot =
_
1 Sen
2

Sen
CAPTULO 11. FUNCIONES TRIGONOMTRICAS 513
para cualquier ngulo con el que Cot y Sen son de signos diferentes, es decir, para cualquier
perteneciente, con cierto m Z, al conjunto
_
2m +

2
; 2m +
_

_
+ 2m;
3
2
+ 2m

.
Para los valores de frontera del ngulo , es decir, cuando =

2
+m, m Z, las frmulas
Cot =
_
1 Sen
2

Sen
y Cot =
_
1 Sen
2

Sen
dan el mismo valor de Cot = 0.
Para cualquier ngulo tal que ,=
m
2
, m Z, se verican las igualdades
TanCot = 1, Tan =
1
Cot
, Cot =
1
Tan
Las dos ultimas igualdades, muestran en qu dependencia se encuentran la tangente y la cotangente
de un mismo ngulo . Si se conoce una de las magnitudes que guran en estas igualdades, para
cierto ngulo , se puede hallar la otra magnitud del mismo ngulo .
Ejemplo 11.29 Demuestre la identidad
2 Csc
2

Tan 1
Csc
2
+ 1 = Cot
Solucin
Partiendo del lado izquierdo de la identidad, tenemos
E =
1 (Csc
2
1)
1
Cot
1
(Csc
2
1)
=
1 Cot
2

1Cot
Cot
Cot
2

=
(1 Cot)(1 +Cot)Cot
1 Cot
Cot
2

=
Cot Cot
3
Cot
2
+Cot
3

1 Cot
=
Cot Cot
2

1 Cot
=
(1 Cot)Cot
1 Cot
= Cot.
Ejemplo 11.30 Demuestre la identidad
(1 +Cot)Sen
3
+ (1 +Tan)Cos
3
= Sen +Cos
Solucin
Partiendo del lado izquierdo de la identidad, tenemos
E =
_
1 +
Cos
Sen
_
Sen
3
+
_
1 +
Sen
Cos
_
Cos
3

=
Sen +Cos
Sen
Sen
3
+
Cos +Sen
Cos
Cos
3

= (Sen +Cos)(Sen
2
+Cos
2
)
= Sen +Cos.
CAPTULO 11. FUNCIONES TRIGONOMTRICAS 514
11.5. Frmulas de adicin
Denicin 11.9 Adicin y sustraccin de ngulos
Sean dados un ngulo y otro ngulo , es decir, supongamos que estn dados un nmero ,
que representa la medida radial del ngulo , y otro nmero que representa la medida radial
del ngulo . Entonces, por ngulo se entiende un ngulo cuya medida radial es el nmero
; el ngulo recibe el nombre de diferencia de dos ngulos dados. Por ngulo + se
entiende un ngulo cuya medida radial es el nmero +; el ngulo + se denomina suma de
dos ngulos dados.
Teorema 11.8 El coseno de la suma entre dos ngulos cualesquiera es igual al producto del
coseno del primer ngulo por el coseno del segundo ngulo menos el producto del seno del primer
ngulo por el seno del segundo ngulo. Es decir, para cualesquiera ngulos y se verica
Cos( +) = CosCos SenSen.
Demostracin
Cuando y son ngulos agudos y su suma + tambin lo es. Sean A0B y B0C los ngulos
y , respectivamente. Entonces, A0C = + . Por un punto cualquiera C, sobre 0C, tracemos
CA y CB respectivamente perpendiculares a 0A y 0B; y tracemos BD y BE perpendiculares a 0A
y AC, respectivamente.
Por ser EC y BC perpendiculares a 0A y 0B, los ngulos BCE y A0B son iguales; esto es, BCE =
. Ahora
Cos( +) =
0A
0C
=
0D BE
0C
=
0D
0C

BE
0C
.
pero
0D
0C
=
0D
0B

0B
0C
= CosCos
y
BE
0C
=
BE
BC

BC
0C
= SenSen
de donde,
Cos( +) = CosCos SenSen.
Cuando y son ngulos agudos y su suma + es obtuso.
Sean D0B y B0C los ngulos y , respectivamente. Entonces, D0C = + . Por un punto
cualquiera C, sobre 0C, tracemos CB perpendicular a 0B, y CA perpendicular a 0D prolongada; y
CAPTULO 11. FUNCIONES TRIGONOMTRICAS 515
tracemos adems BD y BE perpendiculares a 0D y AC, respectivamente.
Por ser EC y BC perpendiculares a 0D y 0B, los ngulos BCE y D0B son iguales; esto es, BCE =
. Ahora
Cos( +) =
0A
0C
=
0D BE
0C
=
0D
0C

BE
0C
.
pero
0D
0C
=
0D
0B

0B
0C
= CosCos
y
BE
0C
=
BE
BC

BC
0C
= SenSen
de donde,
Cos( +) = CosCos SenSen.
Ejemplo 11.31 Calcular Cos75
o
.
Solucin
Descomponiendo, tenemos
Cos75
o
= Cos(45
o
+ 30
o
)
= Cos45
o
Cos30
o
Sen45
o
Sen30
o
=

2
2

3
2

2
2

1
2
=

2
4
.
Ejemplo 11.32 Demuestre que
Cos3 = 4Cos
3
3Cos.
Solucin
Descomponiendo el ngulo, tenemos
Cos3 = Cos(2 +)
= Cos2Cos Sen2Sen
= (2Cos
2
1)Cos (2SenCos)Sen
= 2Cos
3
Cos 2(1 Cos
2
)Cos
= 4Cos
3
3Cos.
Ejemplo 11.33 Dado: Sen = 0, 6 y Sen = 0, 8. Es sabido que

2
< < y

2
< < .
Hallar Cos( +).
Solucin
Cos =
_
1 0, 62 = 0, 8 y Cos =
_
1 0, 8
2
= 0, 6
Entonces
Cos( +) = (0, 8)(0, 6) (0, 6)(0, 8) = 0
Teorema 11.9 El coseno de la diferencia entre dos ngulos cualesquiera es igual al producto
del coseno del primer ngulo por el coseno del segundo ngulo ms el producto del seno del primer
ngulo por el seno del segundo ngulo. Es decir, para cualesquiera ngulos y se verica
Cos( ) = CosCos +SenSen
CAPTULO 11. FUNCIONES TRIGONOMTRICAS 516
Demostracin
Como
Cos( ) = Cos[ + ()]
= CosCos() SenSen()
= CosCos Sen(Sen)
= CosCos +SenSen.
Ejemplo 11.34 Demuestre la identidad
Cos( +)Cos( ) = Cos
2
Sen
2
.
Solucin
Partiendo del lado izquierdo, tenemos
Cos( +)Cos( ) = (CosCos SenSen)(CosCos +SenSen)
= Cos
2
Cos
2
Sen
2
Sen
2

= (1 Sen
2
)Cos
2
Sen
2
Sen
2

= Cos
2
Cos
2
Sen
2
Sen
2
Sen
2

= Cos
2
(Cos
2
+Sen
2
)Sen
2

= Cos
2
Sen
2
.
Ejemplo 11.35 Demuestre la identidad
Cos( +)
Cos( )
=
1 TanTan
1 +TanTan
.
Solucin
Partiendo del lado izquierdo, tenemos
Cos( +)
Cos( )
=
CosCos SenSen
CosCos +SenSen
=
CosCosSenSen
CosCos
CosCos+SenSen
CosCos
=
CosCos
CosCos

SenSen
CosCos
CosCos
CosCos
+
SenSen
CosCos
=
1
Sen
Cos

Sen
Cos
1 +
Sen
Cos

Sen
Cos
=
1 TanTan
1 +TanTan
.
Denicin 11.10 Angulos complementarios
Dos ngulos y , cuya suma es igual a

2
, es decir, + =

2
, se denominan complementarios
uno de otro.
Teorema 11.10 El seno de cualquier ngulo es igual al coseno del ngulo complementario y,
el coseno de cualquier ngulo es igual al seno del ngulo complementario. Es decir, para cualquier
CAPTULO 11. FUNCIONES TRIGONOMTRICAS 517
ngulo se verican las igualdades
_
Cos
_

2

_
= Sen
Sen
_

2

_
= Cos
Demostracin
Al aplicar la igualdad
Cos( ) = CosCos +SenSen
tenemos que
Cos
_

2

_
= Cos

2
Cos +Sen

2
Sen
= Sen.
Es decir, queda demostrada la validez de la primera frmula.
Anlogamente, tenemos que
Sen
_

2

_
= Sen

2
Cos Cos

2
Sen
= Cos.
Teorema 11.11 Para cualquier ngulo tal que ,= m, m Z, se verica la igualdad
Tan
_

2

_
= Cot
Demostracin
Por la denicin de la tangente en funcin del seno y coseno, obtenemos
Tan
_

2

_
=
Sen
_

2

_
Cos
_

2

_
=
Cos
Sen
= Cot.
Teorema 11.12 Para cualquier ngulo tal que ,=

2
+m, m Z, se verica la igualdad
Cot
_

2

_
= Tan
Demostracin
Por la denicin de la cotangente en funcin de coseno y seno, obtenemos
Cot
_

2

_
=
Cos
_

2

_
Sen
_

2

_
=
Sen
Cos
= Tan.
Teorema 11.13 El seno de la suma de dos ngulos cualesquiera es igual al producto del seno
del primer ngulo por el coseno del segundo ms el producto del coseno del primer ngulo por el
seno del segundo. Es decir, para cualesquiera ngulos A y B se verica la igualdad
Sen( +) = SenCos +CosSen
CAPTULO 11. FUNCIONES TRIGONOMTRICAS 518
Demostracin
Cuando y son ngulos agudos y su suma + tambin lo es. Sean A0B y B0C los ngulos
y , respectivamente. Entonces, A0C = + . Por un punto cualquiera C, sobre 0C, tracemos
CA y CB respectivamente perpendiculares a 0A y 0B; y tracemos BD y BE perpendiculares a 0A
y AC, respectivamente.
Por ser EC y BC perpendiculares a 0A y 0B, los ngulos BCE y A0B son iguales; esto es, BCE =
. Ahora
Sen( +) =
AC
0C
=
BD +CE
0C
=
BD
0C
+
CE
0C
.
Pero
BD
0C
=
BD
0B

0B
0C
= SenCos.
y
CE
0C
=
CE
BC

BC
0C
= CosSen.
De donde
Sen( +) = SenCos +CosSen.
Cuando y son ngulos agudos y su suma + es obtuso.
Sean D0B y B0C los ngulos y , respectivamente. Entonces, D0C = + . Por un punto
cualquiera C, sobre 0C, tracemos CB perpendicular a 0B, y CA perpendicular a 0D prolongada; y
tracemos adems BD y BE perpendiculares a 0D y AC, respectivamente.
Por ser EC y BC perpendiculares a 0D y 0B, los ngulos BCE y D0B son iguales; esto es, BCE =
CAPTULO 11. FUNCIONES TRIGONOMTRICAS 519
. Ahora
Sen( +) =
AC
0C
=
BD +CE
0C
=
BD
0C
+
CE
0C
.
Pero
BD
0C
=
BD
0B

0B
0C
= SenCos.
y
CE
0C
=
CE
BC

BC
0C
= CosSen.
De donde
Sen( +) = SenCos +CosSen.
Ejemplo 11.36 Calcular el valor de Sen75

.
Solucin
Sen75

= Sen(45

+ 30

)
= Sen45

Cos30

+Cos45

Sen30

2
2

3
2
+

2
2

1
2
=

6 +

2
4
.
Ejemplo 11.37 Dado: Sen = 0, 6 y Sen = 0, 8. Es sabido que

2
< < y

2
< < .
Hallar Sen( +) y Cos( +).
Solucin
Cos =
_
1 0, 6
2
= 0, 8 y Cos =
_
1 0, 8
2
= 0, 6
Entonces
Sen( +) = 0, 6 (0, 6) + (0, 8) 0, 8 = 1
y
Cos( +) = (0, 8) (0, 6) 0, 6 0, 8 = 0.
Teorema 11.14 El seno de la diferencia entre dos ngulos es igual al producto del seno del
primer ngulo por el coseno del segundo ngulo menos el producto del coseno del primer ngulo
por el seno del segundo ngulo. Es decir, para cualesquiera ngulos A y B se verica la igualdad
Sen( ) = SenCos CosSen
Demostracin
Como
Sen( ) = Sen[ + ()]
= SenCos() +CosSen()
= SenCos +Cos(Sen)
= SenCos CosSen.
CAPTULO 11. FUNCIONES TRIGONOMTRICAS 520
Ejemplo 11.38 Calcular el valor de Sen15

.
Solucin
Sen15

= Sen(45

30

)
= Sen45

Cos30

Cos45

Sen30

2
2

3
2

2
2

1
2
=

2
4
.
Ejemplo 11.39 Dados: SenA =
3
4
, CosB =
1
3
, donde A es del segundo cuadrante, B es un
ngulo agudo. Hallar Sen(AB).
Solucin
Tenemos que
CosA =
_
1
9
16
=

7
4
y SenB =
_
1
1
9
=

8
3
entonces
Sen(AB) =
3
4

1
3

7
4

8
3
=
3 2

14
12
.
Ejemplo 11.40 Demuestre la identidad
Sen2
Sen

Cos2
Cos
= Sec.
Solucin
Sen2
Sen

Cos2
Cos
=
Sen( +)
Sen

Cos( +)
Cos
=
SenCos +CosSen
Sen

CosCos SenSen
Cos
=
SenCos
2
+Cos
2
Sen SenCos
2
+Sen
3

SenCos
=
SenCos
2
+Sen
3

SenCos
=
(Sen
2
+Cos
2
)Sen
SenCos
=
1
Cos
= Sec.
Teorema 11.15 La tangente de la suma de dos ngulos es igual a una fraccin, cuyo numerador
es la suma de las tangentes, y el denominador, la diferencia entre la unidad y el producto de las
tangentes de estos ngulos. Es decir, para cualesquiera ngulos y tales que ,=

2
+ m,
m Z, ,=

2
+m, m Z, + ,=

2
+m, m Z se verica la igualdad
Tan( +) =
Tan +Tan
1 TanTan
CAPTULO 11. FUNCIONES TRIGONOMTRICAS 521
Demostracin
Para cualesquiera dos ngulos y tales que ,=

2
+ m, m Z, ,=

2
+ m, m Z,
+ ,=

2
+m, m Z, resulta valida la igualdad
Tan( +) =
Sen( +)
Cos( +)
=
SenCos +CosSen
CosCos SenSen
=
SenCos+CosSen
CosCos
CosCosSenSen
CosCos
=
SenCos
CosCos
+
CosSen
CosCos
CosCos
CosCos

SenSen
CosCos
=
Sen
Cos
+
Sen
Cos
1
Sen
Cos

Sen
Cos
=
Tan +Tan
1 TanTan
.
Ejemplo 11.41 Demuestre la identidad
Tan3 =
3Tan Tan
3

1 3Tan
2

.
Solucin
Tan3 = Tan(2 +)
=
Tan2 +Tan
1 Tan2Tan
=
2Tan
1Tan
2

+Tan
1
2Tan
1Tan
2

Tan
=
2Tan + (1 Tan
2
)Tan
1 Tan
2
2Tan
3

=
3Tan Tan
3

1 3Tan
2

.
Teorema 11.16 La tangente de la diferencia de dos ngulos es igual a una fraccin, cuyo
numerador es la diferencia de las tangentes, y el denominador, la suma de la unidad y el producto
de las tangentes de estos ngulos. Es decir, para cualesquiera ngulos y tales que ,=

2
+m,
m Z, ,=

2
+m, m Z, ,=

2
+m, m Z se verica la igualdad
Tan( ) =
Tan Tan
1 +TanTan
Demostracin
Para cualesquiera dos ngulos y tales que ,=

2
+ m, m Z, ,=

2
+ m, m Z,
CAPTULO 11. FUNCIONES TRIGONOMTRICAS 522
+ ,=

2
+m, m Z, resulta valida la igualdad
Tan( ) =
Sen( )
Cos( )
=
SenCos CosSen
CosCos +SenSen
=
SenCosCosSen
CosCos
CosCos+SenSen
CosCos
=
SenCos
CosCos

CosSen
CosCos
CosCos
CosCos
+
SenSen
CosCos
=
Sen
Cos

Sen
Cos
1 +
Sen
Cos

Sen
Cos
=
Tan Tan
1 +TanTan
.
Ejemplo 11.42 Demuestre la identidad
Tan( +) Tan
1 +Tan( +)Tan
= Tan.
Solucin
Tan( +) Tan
1 +Tan( +)Tan
= Tan[( +) ]
= Tan( + )
= Tan.
Ejemplo 11.43 Determine el valor de Tan15

. Solucin
Tan15

= Tan(45

30

)
=
Tan45

Tan30

1 +Tan45

Tan30

=
1

3
3
1 + 1

3
3
=
3

3
3 +

3
= 2

3.
Teorema 11.17 La cotangente de la suma de dos ngulos es igual a una fraccin, cuyo numer-
ador es la diferencia entre el producto de las cotangentes de estos dos ngulos y la unidad, y el
denominador, es la suma de las cotangentes. Es decir, para cualesquiera ngulos y tales que
,= m, m Z, ,= m, m Z, + ,= m, m Z se verica la igualdad
Cot( +) =
CotCot 1
Cot +Cot
CAPTULO 11. FUNCIONES TRIGONOMTRICAS 523
Demostracin
Para cualesquiera dos ngulos y tales que ,= m, m Z, ,= m, m Z, + ,= m,
m Z, resulta valida la igualdad
Cot( +) =
Cos( +)
Sen( +)
=
CosCos SenSen
SenCos +CosSen
=
CosCosSenSen
SenSen
SenCos+CosSen
SenSen
=
CosCos
SenSen

SenSen
SenSen
SenCos
SenSen
+
CosSen
SenSen
=
CotCot 1
Cot +Cot
.
Teorema 11.18 La cotangente de la diferencia de dos ngulos es igual a una fraccin, cuyo
numerador es la suma del producto de las cotangentes de estos ngulos y la unidad, y el denomi-
nador, la diferencia de las cotangentes. Es decir, para cualesquiera ngulos y tales que ,= m,
m Z, ,= m, m Z, + ,= m, m Z se verica la igualdad
Cot( ) =
CotCot + 1
Cot Cot
Demostracin
Para cualesquiera dos ngulos y tales que ,= m, m Z, ,= m, m Z, + ,= m,
m Z, resulta valida la igualdad
Cot( ) =
Cos( )
Sen( )
=
CosCos +SenSen
SenCos CosSen
=
CosCos+SenSen
SenSen
SenCosCosSen
SenSen
=
CosCos
SenSen
+
SenSen
SenSen
SenCos
SenSen

CosSen
SenSen
=
CotCot + 1
Cot Cot
.
Teorema 11.19 El producto del coseno de cualquier ngulo por el coseno de cualquier ngulo
B es igual a la semisuma del coseno de la diferencia entre estos ngulos con el coseno de la suma
de los mismos. Es decir, para cualesquiera dos ngulos y se verica la igualdad
CosCos =
Cos( ) +Cos( +)
2
Demostracin
Anteriormente se demostr que para cualesquiera ngulos y se verican las igualdades
Cos( ) = CosCos +SenSen
CAPTULO 11. FUNCIONES TRIGONOMTRICAS 524
y
Cos( +) = CosCos SenSen.
Al sumar estas dos identidades, obtenemos
2CosCos = Cos( ) +Cos( +)
de donde
CosCos =
Cos( ) +Cos( +)
2
.
Teorema 11.20 El producto del seno de cualquier ngulo A por el seno de cualquier ngulo B
es igual a la semidiferencia entre el coseno de la diferencia de estos ngulos y el coseno de la suma
de los mismos. Es decir, para cualesquiera dos ngulos y se verica la igualdad
SenSen =
Cos( ) Cos( +)
2
Demostracin
Anteriormente se demostr que para cualesquiera ngulos y se verican las igualdades
Cos( ) = CosCos +SenSen
y
Cos( +) = CosCos SenSen.
Al sumar estas dos identidades, obtenemos
2SenSen = Cos( ) Cos( +)
de donde
SenSen =
Cos( ) Cos( +)
2
.
En virtud de que Cos() = Cos para cualquier ngulo , al determinar el coseno de la
diferencia entre dos ngulos, podemos tomar en los dos ltimos teoremas, tanto el coseno del
ngulo como el del ngulo .
Teorema 11.21 El producto del seno de cualquier ngulo por el coseno de cualquier ngulo
es igual a la semisuma del seno de la suma de los ngulos y con el seno de la diferencia
entre los ngulos y , con la particularidad de que la diferencia se toma de tal modo que del
ngulo que se encuentra bajo el signo del seno se resta el ngulo que se encuentra bajo el signo del
coseno. Es decir, para cualesquiera ngulos y se verica la igualdad
SenCos =
Sen( +) +Sen( )
2
Demostracin
Anteriormente se demostr que para cualesquiera ngulos y se verican las igualdades
Sen( +) = SenCos +CosSen
y
Sen( ) = SenCos CosSen.
Al sumar estas dos identidades, obtenemos la frmula para calcular el producto del seno de un
ngulo por el coseno de otro ngulo:
SenCos =
Sen( +) +Sen( )
2
.
CAPTULO 11. FUNCIONES TRIGONOMTRICAS 525
Teorema 11.22 La suma de los cosenos de dos ngulos cualesquiera es igual al producto dupli-
cado del coseno de la semisuma de dichos ngulos por el coseno de la semidiferencia de los mismos.
Es decir, para cualesquiera dos ngulos y se verica la igualdad
Cos +Cos = 2Cos
+
2
Cos

2
Demostracin
Al designar
_
+ = x
= y
(1)
y al sumar estas igualdades, obtendremos
_
=
x+y
2
=
xy
2
(2)
De la igualdad (2) se deduce que para todo par x e y siempre existe un par y tal, que se
verican las igualdades (1).
Si en la igualdad
CosCos =
Cos( ) +Cos( +)
2
sustituimos y por x e y, entonces segn las frmulas (1) y (2) obtendremos, como resultado,
la validez de la frmula:
Cosx +Cosy = 2Cos
x +y
2
Cos
x y
2
.
Ejemplo 11.44 Calcular el valor de la expresin:
Cos35

+Cos25

Solucin
Cos35

+Cos25

= 2Cos
35

+ 25

2
Cos
35

25

2
= 2Cos30

Cos5

3Cos5

Teorema 11.23 La diferencia entre los cosenos de dos ngulos cualesquiera es igual al producto
duplicado del seno de la semisuma de dichos ngulos por el seno de la diferencia inversa de estos
ngulos. Es decir, para cualesquiera dos ngulos y se verica la igualdad
Cos Cos = 2Sen
+
2
Cos

2
Demostracin
Al designar
_
+ = x
= y
(1)
y al sumar estas igualdades, obtendremos
_
=
x+y
2
=
xy
2
(2)
CAPTULO 11. FUNCIONES TRIGONOMTRICAS 526
De la igualdad (2) se deduce que para todo par x e y siempre existe un par y tal, que se
verican las igualdades (1).
Si en la igualdad
SenSen =
Cos( ) Cos( +)
2
sustituimos y por x e y, entonces segn las frmulas (1) y (2) obtendremos, como resultado,
la validez de la frmula:
Cosx Cosy = 2Sen
x +y
2
Sen
x y
2
.
Por diferencia inversa entre los ngulos se entiende la diferencia que se forma al sustraer el
ngulo que se encuentra bajo el signo del coseno minuendo del ngulo que se encuentra bajo el
signo de coseno sustrayendo.
Ejemplo 11.45 Calcular el valor de la expresin:
Cos
5
12
+Cos

12
Solucin
Cos
5
12
+Cos

12
= 2Sen
5
2
+

2
2
Sen
5
2


2
2
= 2Sen

4
Sen

6
= 2

2
2

1
2
=

2
2
.
Teorema 11.24 La suma de los senos de dos ngulos cualesquiera es igual al producto duplicado
del seno de la semisuma de dichos ngulos por el coseno de la semidiferencia entre los mismos. Es
decir, para cualesquiera dos ngulos y se verica la igualdad
Sen +Sen = 2Sen
+
2
Cos

2
Demostracin
Al designar
_
+ = x
= y
(1)
y al sumar estas igualdades, obtendremos
_
=
x+y
2
=
xy
2
(2)
De la igualdad (2) se deduce que para todo par x e y siempre existe un par y tal, que se
verican las igualdades (1).
Si en la igualdad
SenCos =
Sen( +) +Sen( )
2
sustituimos y por x e y, entonces segn las frmulas (1) y (2) obtendremos, como resultado,
la validez de la frmula:
Senx +Seny = 2Sen
x +y
2
Cos
x y
2
.
CAPTULO 11. FUNCIONES TRIGONOMTRICAS 527
Ejemplo 11.46 Calcular el valor de la expresin:
Sen75

+Sen15

Solucin
Sen75

+Sen15

= 2Sen
75

+ 15

2
Cos
75

15

2
= 2Sen45

Cos30

= 2

2
2

3
2
=

6
2
.
Teorema 11.25 La diferencia entre los senos de dos ngulos cualesquiera es igual al producto
duplicado del seno de la semidiferencia de dichos ngulos por el coseno de la semisuma de estos
ngulos, con la particularidad de que el seno de la semidiferencia se toma de tal modo que el ngulo
que se encuentra bajo el signo del seno sustrayendo se resta del ngulo que se encuentra bajo el
signo de seno minuendo. Es decir, para cualesquiera dos ngulos y se verica la igualdad
Sen Sen = 2Sen

2
Cos
+
2
Demostracin
Al designar
_
+ = x
= y
(1)
y al sumar estas igualdades, obtendremos
_
=
x+y
2
=
xy
2
(2)
De la igualdad (2) se deduce que para todo par x e y siempre existe un par y tal, que se
verican las igualdades (1).
Si en las igualdades
CosCos =
Cos( ) +Cos( +)
2
SenSen =
Cos( ) Cos( +)
2
SenCos =
Sen( +) +Sen( )
2
sustituimos y por x e y, entonces segn las frmulas (1) y (2) obtendremos, como resultado,
la validez de las frmulas:
Cosx +Cosy = 2Cos
x +y
2
Cos
x y
2
. (3)
Cosx Cosy = 2Sen
x +y
2
Sen
x y
2
. (4)
Senx +Seny = 2Sen
x +y
2
Cos
x y
2
. (5)
Haciendo uso de que Sen(y) = Seny para cualquier ngulo y, de la frmula (5) obtenemos
Senx Seny = Senx +Sen(y) = 2Sen
x y
2
Cos
x +y
2
.
CAPTULO 11. FUNCIONES TRIGONOMTRICAS 528
es decir, es vlida la frmula:
Senx Seny = 2Sen
x y
2
Cos
x +y
2
.
Ejemplo 11.47 Calcular el valor de la expresin:
Sen
5
12
Sen

12
Solucin
Sen
5
12
Sen

12
= 2Sen
5
12


12
2
Cos
5
12
+

12
2
= 2Sen

6
Cos

4
= 2
1
2

2
2
=

2
2
.
Ejemplo 11.48 Verique la identidad:
Sen +Sen
Sen Sen
=
Tan
+
2
Tan

2
.
Solucin
Sen +Sen
Sen Sen
=
2Sen
+
2
Cos

2
2Cos
+
2
Sen

2
=
Sen
+
2
Cos
+
2

Cos

2
Sen

2
= Tan
+
2
Cot

2
=
Tan
+
2
Tan

2
.
Ejemplo 11.49 Verique la identidad:
Sen3 Sen
Cos3 +Cos
= Tan.
Solucin
Sen3 Sen
Cos3 +Cos
=
2Cos
3+
2
Sen
3
2
2Cos
3+
2
Cos
3
2
=
Cos2
Cos2

Sen
Cos
= Tan.
CAPTULO 11. FUNCIONES TRIGONOMTRICAS 529
11.6. Frmulas de arcos dobles y mitad
Denicin 11.11 Angulo de arco doble
Sea dado un ngulo , es decir, cierto nmero que representa la medida radial del ngulo citado.
Entonces, por ngulo 2 se entiende aquel cuya medida radial es el nmero 2; el ngulo 2 se
denomina con frecuencia ngulo de arco doble.
Teorema 11.26 El seno de un ngulo de arco doble 2 es igual al producto duplicado del seno
del ngulo por el coseno del ngulo . Es decir, para cualquier ngulo se verica la igualdad
Sen2 = 2SenCos
Demostracin
Suponiendo que = en la frmula para el seno de la suma de dos ngulos
Sen2 = Sen( +)
= SenCos +CosSen
= 2SenCos
obtenemos lo que se quera demostrar.
Ejemplo 11.50 Dado Sen = 0, 8, 0 < <

2
. Hallar Sen2.
Solucin
Sen2 = 2SenCos = 2Sen
_
1 Sen
2
= 2(0, 8)
_
1 0, 8
2
= 0, 96
Ejemplo 11.51 Demuestre la identidad
1 +Sen2
1 Sen2
=
_
Tan + 1
Tan 1
_
2
.
Solucin
1 +Sen2
1 Sen2
=
1 + 2SenCos
1 2SenCos
=
Sen
2
+Cos
2
+ 2SenCos
Sen
2
+Cos
2
2SenCos
=
(Sen +Cos)
2
(Sen Cos)
2
=
_
Sen+Cos
Cos
_
2
_
SenCos
Cos
_
2
=
_
Tan + 1
Tan 1
_
2
.
Teorema 11.27 El coseno de un ngulo de arco doble 2 es igual al cuadrado del coseno del
ngulo menos el cuadrado del seno del ngulo . Es decir, para cualquier ngulo 2 se verica
la igualdad
Cos2 = Cos
2
Sen
2

CAPTULO 11. FUNCIONES TRIGONOMTRICAS 530


Demostracin
Suponiendo que = en la frmula para el coseno de la suma de dos ngulos
Cos2 = Cos( +)
= CosCos SenSen
= Cos
2
Cos
2
.
Con lo cual queda demostrada la igualdad.
Teorema 11.28 La tangente de un ngulo doble es igual a la tangente doblada del ngulo dado
dividido por la diferencia entre la unidad y el cuadrado de la tangente de dicho ngulo. Es decir,
para cualquier ngulo tal que ,=

2
+k, k Z, y ,=

4
+
n
2
, n Z, se verica la igualdad
Tan2 =
2Tan
1 Tan
2

Demostracin
Representando 2 como + y aplicando la frmula de tangentes de la suma de dos ngulos,
tenemos
Tan2 = Tan( +)
=
Tan +Tan
1 TanTan
=
2Tan
1 Tan
2

.
lo que se queria demostrar.
Teorema 11.29 La cotangente de un ngulo doble es igual a la diferencia entre el cuadrado de
la cotangente de dicho ngulo y la unidad dividida por la cotangente doblada del ngulo dado. Es
decir, para cualquier ngulo tal que ,=
k
2
, k Z, se verica la igualdad
Cot2 =
Cot
2
1
2Cot
Demostracin
Representando 2 como + y aplicando la frmula de Cotangente de la suma de dos ngulos,
tenemos
Cot2 = Cot( +)
=
CotCot 1
Cot +Cot
=
Cot
2
1
2Cot
.
lo que se queria demostrar.
Examinemos un ngulo n, donde n es un nmero natural cualquiera. Por ngulo n se entiende
aquel cuya medida radial es el nmero n. Se pueden deducir las frmulas que expresan Senn y
Cosn, n N, en trminos de Sen y Cos.
CAPTULO 11. FUNCIONES TRIGONOMTRICAS 531
Denicin 11.12 Angulo de arco mitad
Sea dado un ngulo , es decir, cierto nmero que representa la medida radial de dicho ngulo.
Entonces, por ngulo

2
se entiende aquel cuya medida radial es el nmero

2
; el ngulo

2
se
denomina a menudo, ngulo de arco mitad. Para cualquier ngulo se verican las igualdades:
Sen
2

2
=
1 Cos
2
y Cos
2

2
=
1 +Cos
2
Teorema 11.30 Para cualquier ngulo se verica la igualdad
Cos
2

2
=
1 +Cos
2
.
Demostracin
Es evidente que el ngulo puede considerarse como un ngulo de arco doble con relacin al ngulo

2
. Por eso, para cualquier ngulo se verica la igualdad:
Cos = Cos
2

2
Sen
2

2
adems, para todo ngulo resulta vlida la identidad trigonomtrica:
1 = Cos
2

2
+Sen
2

2
.
Sumando estas dos igualdades, obtenemos la igualdad
Cos
2

2
Sen
2

2
Esta igualdad es equivalente a la igualdad

Cos

=
_
1 +Cos
2
.
De la ltima igualdad tenemos:
Cos

2
=
_
1 +Cos
2
para todo ngulo , para el cual Cos

2
es no negativo, es decir, para todo , perteneciente, con
cierto k Z, al intervalo [ + 4k; + 4k]. Por otro lado, tenemos
Cos

2
=
_
1 +Cos
2
para todo ngulo , para el cual Cos

2
es no positivo, es decir, para todo , perteneciente, con
cierto k Z, al intervalo [ + 4k; 3 + 4k].
Para los valores de frontera del ngulo , es decir, cuando = + 2m, donde m Z, las
frmulas
Cos

2
=
_
1 +Cos
2
y Cos

2
=
_
1 +Cos
2
dan un mismo valor, Cos

2
= 0.
CAPTULO 11. FUNCIONES TRIGONOMTRICAS 532
Ejemplo 11.52 Dado Sen =
1
3
, < <
3
2
. Hallar Cos

2
.
Solucin Primero calculamos Cos:
Cos =
_
1
1
9
=
2

2
3
Se ha tomado el signo menos porque el ngulo termina en el tercer cuadrante, donde el coseno
es negativo; el semingulo

2
termina en el segundo cuadrante, por lo tanto
Cos

2
=
_
1 +Cos
2

1
2

2
3
2
=

3 2

2
6
Teorema 11.31 Para cualquier ngulo se verica la igualdad
Sen
2

2
=
1 Cos
2
.
Demostracin
Se ha observado anteriormente que para todo ngulo se verica la igualdad
1 = Cos
2

2
+Sen
2

2
y Cos = Cos
2

2
Sen
2

2
Al restar la segunda igualdad de la primera, obtenemos la igualdad
Sen
2

2
=
1 Cos
2
.
Esta igualdad es equivalente a la igualdad

Sen

=
_
1 Cos
2
.
De la ltima igualdad tenemos:
Sen

2
=
_
1 Cos
2
para todo ngulo , para el cual Sen

2
es no negativo, es decir, para todo , perteneciente, con
cierto k Z, al intervalo [4k; 2 + 4k]. Por otro lado, tenemos
Sen

2
=
_
1 Cos
2
para todo ngulo , para el cual Sen

2
es no positivo, es decir, para todo , perteneciente, con
cierto k Z, al intervalo [2 + 4k; 4k].
Para los valores de frontera del ngulo , es decir, cuando = 2m, donde m Z, las frmulas
Sen

2
=
_
1 Cos
2
y Sen

2
=
_
1 Cos
2
dan un mismo valor, Sen

2
= 0.
CAPTULO 11. FUNCIONES TRIGONOMTRICAS 533
Ejemplo 11.53 Calcular el valor de Sen

8
.
Solucin Sabemos que
Sen

2
=
_
1 Cos
2
.
Hemos tomado ante el radical el signo positivo, y por ser

8
un ngulo agudo. Por tanto
Sen

8
=
_
1 Cos

4
2
=

2
2
2
=

2
2
=
_
2

2
2
.
Teorema 11.32 Para cualquier ngulo A tal que A ,= + 2k, k Z, se verica la igualdad
Tan
2
A
2
=
1 CosA
1 +CosA
Demostracin
Haciendo uso de la denicin de tangente de un ngulo y las igualdades
Cos
2

2
=
1 +Cos
2
y Sen
2

2
=
1 Cos
2
obtenemos la igualdad
Tan
2
A
2
=
1 CosA
1 +CosA
Esta igualdad es equivalente a la igualdad

Tan

1 Cos
1 +Cos
.
De la ltima igualdad tenemos:
Tan

2
=

1 Cos
1 +Cos
para todo ngulo , para el cual Tan

2
es no negativo, es decir, para todo , perteneciente, con
cierto n Z, al intervalo [2n; + 2n). Por otro lado, tenemos
Tan

2
=

1 Cos
1 +Cos
para todo ngulo , para el cual Tan

2
es no positivo, es decir, para todo , perteneciente, con
cierto n Z, al intervalo [ + 2n; 2n].
Para los valores de frontera del ngulo , es decir, cuando = 2n, donde n Z, las frmulas
Tan

2
=

1 Cos
1 +Cos
y Tan

2
=

1 Cos
1 +Cos
dan un mismo valor, Tan

2
= 0.
CAPTULO 11. FUNCIONES TRIGONOMTRICAS 534
Ejemplo 11.54 Calcular Tan15

.
Solucin
Tan15

=
_
1 Cos30

1 +Cos30

_
1

3
2
1 +

3
2
=

3
2 +

3
= 2

3.
Teorema 11.33 Para cualquier ngulo tal que ,= k, k Z, se verica la igualdad
Tan

2
=
1 Cos
Sen
Demostracin
Por cuanto ,= k, k Z, entonces Sen

2
,= 0 y Cos

2
,= 0. Quiere decir, se verica la igualdad
Tan

2
=
Sen

2
Cos

2
=
Sen

2
Sen

2
Cos

2
Sen

2
=
2Sen
2
2
2Sen

2
Cos

2
.
Aplicando ahora las frmulas
Sen
2

2
=
1 Cos
2
y Sen2 = 2SenCos
obtenemos la validez de la igualdad
Tan

2
=
1 Cos
Sen
Ejemplo 11.55 Calcular Tan

8
.
Solucin
Tan

8
=

1 Cos

4
Sen

4
=

_
1

2
2

2
4
=
2

2
=

2 1.
CAPTULO 11. FUNCIONES TRIGONOMTRICAS 535
Teorema 11.34 Para cualquier ngulo tal que ,= + 2k, k Z, se verica la igualdad
Tan

2
=
Sen
1 +Cos
Demostracin
Por cuanto ,= k, k Z, entonces Sen

2
,= 0 y Cos

2
,= 0. Quiere decir, se verica la igualdad
Tan

2
=
Sen

2
Cos

2
=
Sen

2
2Cos

2
Cos

2
2Cos

2
=
2Sen
2
2
Cos
2
2
2Cos
2

2
.
Aplicando ahora las frmulas
Sen
2

2
=
1 Cos
2
y Sen2 = 2SenCos
obtenemos la validez de la igualdad
Tan

2
=
Sen
1 +Cos
Demos a conocer algunas frmulas ms que expresan el seno, el coseno, la tangente y la cotan-
gente de un ngulo en trminos de la tangente del ngulo de arco mitad.
Teorema 11.35 El coseno de un ngulo es igual a la diferencia entre la unidad y el cuadrado
de la tangente de la mitad de este ngulo, dividida por la suma de la unidad y el cuadrado de la
tangente del semingulo. Es decir, para cualquier ngulo tal que ,= +2k, k Z, se verica
la igualdad
Cos =
1 Tan
2
2
1 +Tan
2

2
Demostracin
Para cualquier ngulo tal que ,= +k, k Z, entonces
Cos =
Cos
1
=
Cos
2
2
Sen
2
2
Cos
2

2
+Sen
2

2
=
Cos
2
2
Sen
2
2
Cos
2

2
Cos
2

2
+Sen
2

2
Cos
2

2
=
1
_
Sen

2
Cos

2
_
2
1 +
_
Sen

2
Cos

2
_
2
=
1 Tan
2
2
1 +Tan
2

2
.
CAPTULO 11. FUNCIONES TRIGONOMTRICAS 536
Teorema 11.36 El seno de un ngulo es igual a la tangente doblada de la mitad de este ngulo
dividida por la suma de la unidad y el cuadrado de la tangente del semingulo. Es decir, para
cualquier ngulo tal que ,= + 2k, k Z, se verica la igualdad
Sen =
2Tan

2
1 +Tan
2

2
Demostracin
Para cualquier ngulo tal que ,= + 2k, k Z, entonces
Sen =
Sen
1
=
2Sen

2
Cos

2
Cos
2

2
+Sen
2

2
=
2Sen

2
Cos

2
Cos
2

2
Cos
2

2
+Sen
2

2
Cos
2

2
=
2Tan

2
1 +
_
Sen

2
Cos

2
_
2
=
2Tan

2
1 +Tan
2

2
.
Ejemplo 11.56 Calcular Sen, si Tan

2
= 2.
Solucin
Sen =
2Tan

2
1 +Tan
2

2
=
2 2
1 + 2
2
=
4
5
.
Ejemplo 11.57 Dada Tan =
3
2
, 0 < <

2
, hallar Sen4.
Solucin
En primer lugar encontramos Sen2 y Cos2:
Sen2 =
2Tan
1 +Tan
2

=
2
3
2
1 +
_
3
2
_
2
=
12
13
.
Cos2 =
1 Tan
2

1 +Tan
2

=
1
_
3
2
_
2
1 +
_
3
2
_
2
=
5
13
.
CAPTULO 11. FUNCIONES TRIGONOMTRICAS 537
El ngulo 4 es el doble con respecto al ngulo 2, y por tanto
Sen4 = 2Sen2Cos2
= 2
12
13

_

5
13
_
=
120
169
.
Ejemplo 11.58 Calclese la suma
S
n
(x) =
n

k=1
Senkx
Solucin
Analicemos la igualdad
2Sen
x
2
S
n
(x) = 2Sen
x
2

n

k=1
Senkx
Ya que
2Sen
x
2
Senkx = Cos
_
k
1
2
_
x Cos
_
k +
1
2
_
x
de
n

k=1
(a
k+1
a
k
) = a
n+1
a
1
hallamos
2Sen
x
2
S
n
(x) = Cos
x
2
Cos
_
n +
1
2
_
x
= 2Sen
n + 1
2
xSen
n
2
x.
de donde
S
n
(x) =
Sen
n+1
2
xSen
n
2
x
Sen
x
2
si Sen
x
2
,= 0, si Sen
x
2
= 0, S
n
(x) = 0.
Ejemplo 11.59 Utilizando el principio de induccin matemtica, vericar la siguiente iden-
tidad:
1
2
+Cosx +Cos2x +... +Cosnx =
Sen
_
2n+1
2
_
x
2Sen
x
2
Solucin
P(n):
1
2
+

n
i=1
Cosix =
Sen(
2n+1
2
)x
2Sen
x
2
entonces
CAPTULO 11. FUNCIONES TRIGONOMTRICAS 538
i) P(1):
1
2
+Cosx =
Sen
3x
2
2Sen
x
2
Sen
3x
2
2Sen
x
2
=
Sen
_
x +
x
2
_
2Sen
x
2
=
SenxCos
x
2
+CosxSen
x
2
2Sen
x
2
=
2Sen
x
2
Cos
2 x
2
+CosxSen
x
2
2Sen
x
2
=
_
2Cos
2 x
2
+Cosx
_
Sen
x
2
2Sen
x
2
=
1
2
_
2Cos
2
x
2
+Cosx
_
=
1
2
+Cosx.
ii) P(k):
1
2
+

n
i=1
Cosix =
Sen
2k+1
2
2Sen
x
2
lo cual es verdadero por hiptesis inductiva.
P(k+1):
1
2
+

k+1
i=1
Cosix =
Sen(
2k+3
2
)x
2Sen
x
2
Sen
_
2k+3
2
_
x
2Sen
x
2
=
_
1
2
+
k

i=1
Cosix
_
+Cos(k + 1)x
=
Sen
2k+1
2
2Sen
x
2
+Cos(k + 1)x
=
Sen
_
(k + 1)
1
2
_
x
2Sen
x
2
+Cos(k + 1)x
=
Sen(k + 1)xCos
x
2
Cos(k + 1)xSen
x
2
+ 2Sen
x
2
Cos(k + 1)x
2Sen
x
2
=
Sen(k + 1)xCos
x
2
+Cos(k + 1)xSen
x
2
2Sen
x
2
=
Sen
_
(k + 1) +
1
2
_
x
2Sen
x
2
=
Sen
_
2k+3
2
_
x
2Sen
x
2
.
Por lo tanto queda demostrado que P(n) es verdadero.
Teorema 11.37 Para cualquier ngulo tal que ,= + 2k, k Z y ,=

2
+n, n Z, se
verica la igualdad
Tan =
2Tan

2
1 Tan
2

2
Demostracin
CAPTULO 11. FUNCIONES TRIGONOMTRICAS 539
Para cualquier ngulo tal que ,= + 2k, y ,=

2
+k, k Z, se verica la igualdad
Tan =
Sen
Cos
=
2Sen

2
Cos

2
Cos
2

2
Sen
2

2
=
2Sen

2
Cos

2
Cos
2

2
Cos
2

2
Sen
2

2
Cos
2

2
=
2Tan

2
1
_
Sen

2
Cos

2
_
2
=
2Tan

2
1 Tan
2

2
.
Teorema 11.38 Para cualquier ngulo tal que ,= k, k Z, se verica la igualdad
Cot =
1 Tan
2
2
2Tan

2
Demostracin
Para cualquier ngulo tal que ,= k, k Z, se verica la igualdad
Cot =
Cos
Sen
=
Cos
2
2
Sen
2
2
2Sen

2
Cos

2
=
Cos
2
2
Sen
2
2
Cos
2

2
2Sen

2
Cos

2
Cos
2

2
=
1
_
Sen

2
Cos

2
_
2
2Tan

2
=
1 Tan
2
2
2Tan

2
.
11.7. Tarea
1. Comprobar las igualdades:
a) 4Sen

6
+

2 Cos

4
+Cos = 2; b) 2

3 Sen
2
3
+ 4Sen

6
2Sen

2
= 3.
2. Simplicar las siguientes expresiones:
a)
Cos
_

6
_
Cos
_

3
+

6
_
+Sen
2

6
Sen
2
2

6 Sen
_
2

6

6
_
Cos
_
2

6

3
_;
b)
Sen
2
_
3
2
+

3
_
Sec
2
_
3

2
_
1
+
Cos
2
_
3
2

3
_
Csc
2
_
3
2

3
_
1
;
CAPTULO 11. FUNCIONES TRIGONOMTRICAS 540
c)
Sen(4 )Cos(4 2)Sen(2 4)
Sen
_

2
4
_
Cot( 4)Cot
_
3
2
+ 4
_;
d)
Sen(2

2 + 1) +Sen(2

2 1) Cos
_
3
2
2

2
_
Cos(2

2 + 1) +Cos(2

2 1) Sen
_
3
2
+ 2

2
_;
e)
Cos
_
1 +
3
2
_
+ 2Cos
_
1
11
6
_
2Sen
_

3
+ 1
_
+

3Sen
_
3
2
1
_;
f )
Sen(5)
Cos
_

2
+ 5
_ +Cot
_
3
2
+ 5
_
Tan
_

2
5
_
;
g) Cos

65
Cos
2
65
Cos
4
65
Cos
8
65
Cos
16
65
Cos
32
65
;
h)
2Cos
_

2

_
Sen
_

2
+
_
Tan( )
Cot
_

2
+
_
Sen( )
;
i)
Sen
3
( 270
o
)Cos(360
o
)
Tan
3
( 90
o
)Cos
3
( 270
o
)
.
Resp: a) ; b) ; c) ; d) ; e) ; f ) ; g)
1
64
; h) 3Cos; i) Cos, ,=
n
2
.
3. Simplicar las siguientes expresiones:
a)
_
1
Sen

1
Cos
_
(Sen +Cos);
b)
2Sen
2
Tan
2

Tan
2
Sen
2

;
c)
Sen
2
Sen
2

SenCos SenCos
;
d) (Tan
2
Sen
2
)
Cot
2

2Sen
2

;
e) 1
Sen
2

1 +Cot

Cos
2

1 +Tan
;
f ) Sen7Tan
7
2
+Cos7;
g) Sen +Sen
_
+
2
3
_
+Sen
_
+
4
3
_
.
Resp: a) Cot Tan; b) 2; c)
Sen
2
Sen
2

Sen( )
; d)
1
2
; e)
1
2
Sen2; f ) 1;
g) 0.
4. Comprobar las identidades:
a) 4SenSen(60
o
)Sen(60
o
+) = Sen3;
b) Sen47
o
+Sen61
o
Sen11
o
Sen25
o
= Cos7
o
;
c) 3(Sen
4
+Cos
4
) 2(Sen
6
+Cos
6
) = 1;
d)
Tan
3

Sen
2


1
SenCos
+
Cot
3

Cos
2

= Tan
3
+Cot
3
;
e)
Cos
3
Cos3
Cos
+
Sen +Sen3
Sen
= 3;
f ) Tan
2
_
45
o
+

2
_
=
Cos
1
+Tan
Cos
1
Tan
;
g) (Cos +Sen)
2
+ (Sen Cos)
2
= 4Cos
2
_
45
o
+

2
_
;
CAPTULO 11. FUNCIONES TRIGONOMTRICAS 541
h) 2
_
1
Sen2
+Cot2
_
= Cot

2
Tan

2
;
i)
_
1 +Sen
_
1 Sen = 2Sen

2
;
j) 4Sen
_
+

3
_
Sen
_


3
_
= 4Sen
2
3;
k) 2CosCosCos( +) = Cos
2
+Cos
2
Sen
2
( +);
l) Cos +Cos +Cos +Cos( + +) = 4Cos
+
2
Cos
+
2
Cos
+
2
;
m)
Sean +Sen
Tan
+
2
+Cot

2
=
Sen( +)Sen( )
2Cos
;
n) Cos
1
2
Cos3
1
2
Cos5 = 8Sen
2
Cos
3
;
o)
2Sen Sen3 +Sen5
Cos 2Cos2 +Cos3
=
2Cos2
Tan

2
;
p) Cos +Cos(120
o
) +Cos(120
o
+) = 0;
q) Tan(35
o
+)Tan(25
o
) =
2Cos(10
o
+ 2) 1
2Cos(10
o
+ 2) + 1
;
r)

3
2
Cos2
1
2
Sen2
1
1
2
Cos2

3
2
Sen2A
= Cot
_
+

3
_
.
5. Comprobar las identidades:
a)
Cos2
1 +Sen2
=
1 Tan
1 +Tan
;
b) 1 Sen8 = 2Cos
(
45
o
+ 4);
c)
Cos

2
Sen

2
Cos

2
+Sen

2
=
1
Cos
Tan;
d)
Cot +Sen
1

Sen +Tan
=
2Cos
1 Cos2
;
e)
1 2Cos
2

SenCos
= Tan Cot;
f ) 3 4Cos2 +Cos4 = 8Sen
4
.
6. Sin emplear tablas, simplicar las expresiones:
a)
Sen10
o
Cos20
o
+Cos10
o
Sen20
o
Cos19
o
Cos11
o
Sen19
o
Sen11
o
;
b)
Sen9
o
Cos39
o
Cos9
o
Sen30
o
Cos
3
7
Cos
5
28
+Sen
3
7
Sen
5
28
;
c) Cos10
o
Cos30
o
Cos50
o
Cos70
o
;
d) 16Sen10
o
Sen30
o
Sen50
o
Sen70
o
Sen90
o
;
e) Tan9
o
Tan27
o
Tan63
o
+Tan81
o
.
Resp: a)

3
3
; b)

2
2
; c)
3
16
; d) 1; e) 4.
7. Sin emplear tablas, simplicar las expresiones:
2Sen40
o
+ 2Cos130
o
3Sen160
o
3Cos(110
o
)
Resp: 0.
CAPTULO 11. FUNCIONES TRIGONOMTRICAS 542
8. Calcular Sen, Cos, Tan si Cot = 2 y

2
< < .
Resp: Sen =

5
5
, Cos =
2

5
5
, TanA =
1
2
.
9. Calcular Sen, Tan, Cot si Cos =
3
5
y < <
3
2
.
Resp: Sen =
1
5
, Tan =
4
3
, Cot =
3
4
.
10. Calcular Cos, Tan, Cot si Sen =
12
13
y
3
2
< < 2.
Resp: Cos =
5
13
, Tan =
12
5
, Cot =
5
12
.
11. Calcular Sen2, Cos2, Tan2, Cot2 si Cos =
5
13
y 0 < <

2
.
Resp: Sen2 =
120
169
, Cos2 =
119
169
, Tan2 =
120
119
, Cot2 =
119
120
.
12. Calcular
5Sen + 7Cos
6Cos 3Sen
si Tan =
4
15
.
Resp:
125
78
.
13. Calcular Cos
_

3

_
si Sen =
12
13
y
3
2
< < 2.
Resp:
5 12

3
26
.
14. Demuestre que + =

4
si Sen =
1

5
, Sen =
1

10
.
15. Si Tan +Cot = k, hallar:
a) Tan
2
+Cot
2
; b) Tan
2
+Cot
3
; c) TanACotA.
Resp: a) m
2
2; b)
m
m
2
3
; c)
_
m
2
4.
16. Calcular Sen

2
, Cos

2
, Tan

2
si:
a) Cos = 0, 8 y 0 < <

2
; b) Tan =
24
7
y 180
o
< < 270
o
.
Resp: a) Sen

2
=

10
10
, Cos

2
=
3

10
10
, Tan

2
=
1
3
; b) Sen

2
=
4
5
, Cos

2
=

3
5
, Tan

2
=
4
3
.
17. Calcular Sen

4
si Sen =
336
625
y 450
o
< < 540
o
.
Resp:
4
5
.
CAPTULO 11. FUNCIONES TRIGONOMTRICAS 543
18. Demuestre que Sen =
2a
1 +a
2
, Cos =
1 a
2
1 +a
2
, Tan =
2a
1 a
2
, Cot =
1 a
2
2a
si
Tan

2
= a.
19. Calcular Tan

2
si Sen +Cos =

7
2
y 0 < <

6
.
Resp:

7 2
3
.
20. Compruebe las identidades:
a) 1 +Cot

2
+Cot
_
45
o


2
_
= Cot

2
Cot
_
45
o


2
_
;
b) Tan3 = TanTan(60
o
+)Tan(60
o
);
c) CosCos2Cos4Cos8Cos16 =
Sen32
32Sen
;
d) 9Cos15 + 3Cos7 + 3Cos19 + 9Cos11 = 24Cos
3
2Cos13;
e)
1 +SenCos
Sen
1
Cos
1
Sen +Cos
=

2Sen2
4Sen
_

4

_;
f )
_
1
1 +Cos
+
1
1 Cos
Sen =

2 si 0 < < ;
g)
_
1
Sen
2

+
1
Cos
2

=
2
Sen2
si

2
< < 0;
h)
_
Sen
2
(1 +Cot) +Cos
2
(1 +Tan) =

2Cos
_
+

4
_
si

4

3
4
, ,= 0;
i)
_
Cot +Cos +
_
Cot Cos = 2Cos

2
_
Cot si 0 <

2
;
j)

1 +Sen
1 Sen

1 Sen
1 +Sen
=
_
2Tan si

2
+ 2k < <

2
+ 2k
2Tan si

2
+ 2k < <
3
2
+ 2k
;
k)
_
Tan
2
+Cot
2
+ 2 =
_
2
Sen2
si k < <

2
+k

2
Sen2
si

2
+k < < k
;
l)
_
1 +Cos2 +
_
1 Cos2 +

2(SenA+CosA) =
_

_
2

2(Sen +Cos) si 2k

2
+ 2k
2

2Sen si

2
+ 2k < < + 2k
0 si + 2k
3
2
+ 2k
2

2Cos si
3
2
+ 2k < < 2 + 2k
;
m)
_
1 + 2SenCos =
_

2Cos
_


4
_
si

4
+ 2k
3
4
+ 2k

2Cos
_


4
_
si
3
4
+ 2k < <
7
4
+ 2k
;
n) Tan2Tan(30
o
) +Tan2Tan(60
o
) +Tan(60
o
)Tan(30
o
) = 1;
o) Sen
3
Sen
3
( ) +Sen
3
Sen
3
( ) +Sen
3
Sen
3
( ) =
3SenSenSenSen( )Sen )Sen( );
p) Cos +Cos +Cos = 1 + 4Sen

2
Sen

2
Sen

2
, si + + = ;
q) Sen +Sen +Sen = 4Cos

2
Cos

2
Cos

2
, si + + = ;
r) Tan +Tan +Tan = TanTanTan, si + + = ;
s) Sen +Sen +Sen = 4Cos

2
Cos

2
Sen

2
, si + = .
CAPTULO 11. FUNCIONES TRIGONOMTRICAS 544
21. Sabemos que Tan =
3
4
y

2
< < . Hallar los valores de las dems funciones
trigonomtricas del argumento .
Resp: Sen =
3
5
, Cos =
4
5
, Cot =
4
3
.
22. Calcular Sen, Cos, Tan, Cot si =112
o
30.
Resp: Sen =
_
2 +

2
2
, Cos =
_
2 +

2
2
, Tan = (1 +

2), Cot = 1

2.
23. Calcular Tan

4
si Cos = 0, 6 y 180
o
< < 270
o
.
Resp: Tan

4
=
1 +

5
2
.
24. Calcular 16Sen

2
Sen
3
2
si Cos =
3
4
.
Resp: 5.
25. Demuestre que si > 0, > 0, > 0 y + + =

2
, entonces
TanTan +TanTan +TanTan = 1.
26. Demuestre que si
3
4
< < , entonces
_
2Cot +
1
Sen
2

= 1 Cot.
27. Demuestre que si Sen +Sen = 2Sen( +) donde + ,= k, entonces
Tan

2
Tan

2
=
1
3
.
28. Demuestre que si Tan =
1
7
, Sen =
1

10
, 0 < <

2
y 0 < <

2
, entonces +2 =

4
.
29. Demuestre la identidad
Tan
2
2 Tan
2

1 Tan
2
2Tan
2

= Tan3Tan
30. Demuestre las siguientes identidades:
a)
2Sen
2
1
Cos
+
(2Sen + 1)Cos
1 +Sen
= Tan;
b)
Sen
2

1 +Cot
+
Cos
2

1 +Tan
= 1
1
2
Sen2;
c)
2(Sen2 + 2Cos
2
1)
Cos Sen Cos3 +Sen3
= Csc;
d)
1 +Cos +Cos2 +Cos3
Cos + 2Cos
2
1
= 2Cos;
CAPTULO 11. FUNCIONES TRIGONOMTRICAS 545
e)
Sen
_


6
_
+Cos
_

3

_
Cos
=

3Tan;
f )
Cos
2
Sen
2

4Cos
2

=
1
4
(1 4Sen
2
);
g)
Sen(2 +)
Sen
2Cos( +) =
Sen
Sen
;
h) 2(Sen
6
+Cos
6
) 3(Sen
4
+Cos
4
) = 1;
i) Tan +
1
Cos
3


1
Sec Tan
=
Sen
2

Cos
3

;
j)
1 +Sen Cos
Sen

2
= 2
_
Sen

2
+Cos

2
_
;
k)
Tan
(1 +Tan
2
)
2
+
Cot
(1 +Cot
2
)
2
=
1
2
Sen2;
l) 3Cos2 (Sen +Cos)

1 Sen2 = 4Cos2;
m) Cos
2
+Cos
2
( +) 2CosCosCos( +) = Sen
2
;
n) Sen( +)Sen( )Sec
2
Sec
2
=
Cos
2
Cos
2

Cos
2
Cos
2

;
o) (1 +Cot)Sen
3
+ (1 +Tan)Cos
3
= Sen +Cos;
p) SenCosTanCotSecCsc = 1;
q) (Sen +Cos)
2
+ (Sen Cos)
2
= 2.
31. Demuestre las siguientes identidades:
a) Tan3 =
3Tan Tan
3

1 3Tan
2

;
b)
Sen2
Sen

Cos2
Cos
= Sec;
c) Tan
2
Sen
2
= Tan
2
Sen
2
;
d)
Cos2
1 Sen2
=
1 +Tan
1 Tan
;
e)
Tan2Tan
Tan2 Tan
= Sen2;
f )
Csc( )
Cot2 Cot
=
Cos2
Sen
;
g)
Sec Tan
2Sen
=
1
2
Tan;
h)
2Sen
2

Sec
2
1
= 2Cos
2
;
i)
Sen3 Sen
Cos3 +Cos
= Tan;
j)
Sen2Cos
1 +Cos2
= Sen;
k)
2Sen Sen2
2Sen +Sen2
= Tan
2

2
;
l)
4Cos2
Tan

2
Cot

2
= Sen2;
m)
1 +Tan2Tan
Cot +Tan
=
1
2
Tan2;
n)
1 +Sen2
1 Sen2
=
_
Tan + 1
Tan 1
_
2
;
o)
Sen +Sen
Cos +Cos
= Tan
+
2
;
p)
Tan( +) Tan
1 +Tan( +)Tan
= Tan.
q)
1 + 2Tan Tan
2

Cos2 +Sen2
= Sec
2
;
r)
2 Csc
2

Tan 1
Csc
2
+ 1 = Cot;
s)
(Sen +Cos)
2
1
Cot SenCos
= 2Tan
2
;
t)
(1 +Sen)Tan
_

4


2
_
Sen
= Cot;
u)
Tan
2

1 +Tan
2


1 +Cot
2

Cot
2

= Tan
2
;
v)
2Cos
2
1
2Tan
_

4

_
Sen
2
_

4
+
_ = 1;
w)
1 +Tan
2

Cot
=
Tan
Cos
2

;
x)
Tan +Tan
Cot +Cot
= TanTan;
y) Sen =
2Tan

2
1 +Tan
2

2
;
z)
1 Tan
2
2
1 +Tan
2

2
= Cos.
CAPTULO 11. FUNCIONES TRIGONOMTRICAS 546
32. Demuestre las siguientes identidades:
a)
Cos
2

Cot

2
Tan

2
=
1
4
Sen2;
b) Sen
2
Cos
2
= Sen
2
Cos
2
;
c)
Tan
2
2 Tan
2

1 Tan
2
2Tan
2

= Tan3Tan;
d)
SenCos
Cos
2
Sen
2

=
Tan
1 Tan
2

;
e) Cot
Cot
2
1
Cot
= Tan;
f )
Sen +Cot
Tan +Csc
= Cos;
g) Tan +Cot =
1
SenCos
;
h)
Cot +Tan
Cot Tan
=
1
Cos
2
Sen
2

;
i) Cot
2
Sen
2
= Cot
2
Sen
2
;
j)
1 Sen
Cos
=
Cos
1 +Sen
;
k)
1 +Tan
1 Tan
=
Cos +Sen
Cos Sen
.
33. Demuestre las siguientes identidades:
a) Cos
2
Cos
2
Sen
2
Sen
2
= Cos
2
Sen
2
;
b) Sen47
o
+Sen61
o
Sen11
o
Sen25
o
= Cos7
o
;
c) 4SenSen(60
o
)Sen(60
o
+) = Sen3.
34. Hallar las siguientes sumas:
a)
n

i=1
Sen(2i 1)x;
b)
n

i=1
Cos(2i 1)x;
c)
n

i=1
Sen
2
ix;
d)
n

i=1
Cos
2
ix;
e)
n

i=1
Sen
3
ix.
35. Utilizando el principio de induccin matemtica, demostrar la identidad:
a) Senk =
_
(2Cos)
k1

_
k 2
1
_
(2Cos)
k3
+
_
k 3
2
_
(2Cos)
k5
...
_
Sen;
b) Cosk =
1
2
_
(2Cos)
k

k
1
(2Cos)
k2
+
k
2
_
k 3
1
_
(2Cos)
k4

k
3
_
k 4
2
_
(2Cos)
k6
...
_
;
c) Sen
2k1
=
(1)
k1
2
2k2
_
Sen(2k 1)
_
2k 1
1
_
Sen(2k 3) +...(1)
k1
_
2k 1
k 1
_
Sen
_
;
d) Cos
2k1
=
1
2
2k2
_
Cos(2k 1) +
_
2k 1
1
_
Cos(2k 3) +... +
_
2k 1
k 1
_
Cos
_
;
e) Sen
2k
=
1
2
2k
_
2k
k
_
+
(1)
k
2
2k1
_
Cos2k
_
2k
1
_
Cos(2k 2) +...(1)
k1
_
2k
k 1
_
Cos2
_
;
f ) Cos
2k
=
1
2
2k
_
2k
k
_
+
1
2
2k1
_
Cos2k +
_
2k
1
_
Cos(2k 2) +... +
_
2k
k 1
_
Cos2
_
.
36. Haciendo uso del principio de induccin matemtica, vericar si se cumplen las siguientes
identidades:
a) (Cosx +iSenx)
k
= Coskx +iSenkx;
b) Senx + 2Sen2x +... +kSenkx =
(k + 1)Senkx kSen(k + 1)x
4Sen
2
x
2
;
CAPTULO 11. FUNCIONES TRIGONOMTRICAS 547
c) Cosx + 2Cos2x +... +kCoskx =
(k + 1)Coskx kCos(k + 1)x 1
4Sen
2
x
2
;
d)
1
2
Tan
x
2
+
1
2
2
Tan
x
2
2
+... +
1
2
k
Tan
x
2
k
=
1
2
k
Cot
x
2
k
Cotx.
37. Una escalera tiene el pie en la calle y forma un ngulo de 30

con el suelo cuando su


extremo superior descansa contra un edicio, y de 40

con el edicio frontero. Si la escalera


tiene 15 metros de largo, qu ancho tiene la calle?
38. Un rbol quebrado por el viento forma un tringulo rectngulo con el suelo. Si la parte
quebrada forma un ngulo de 50

con el suelo y si la copa del rbol est ahora a 6 metros de


su base, qu altura tena el rbol?
39. Hallar la base y la altura de un tringulo issceles de ngulo en el vrtice igual a 65
o
y de
lado a iguales de 415 metros.
40. La base de un tringulo issceles es de 15,90 centmetros y los ngulos de la base tienen
54

28

. Hallar los lados iguales y la altura.


41. Cul es el permetro de un tringulo issceles de 40 centmetros de base y cuyos ngulos
de base son de 70

?
42. Un hombre conduce durante 150 m a lo largo de una va inclinada 20
o
sobre la horizontal.
A qu altura se encuentra sobre su punto de partida?
43. Dos caminos rectos se cortan bajo un ngulo de 75
o
. Hallar la mnima distancia de uno de
ellos a una estacin de gasolina que est sobre el otro a 300 metros de la encrucijada.
44. Dos edicios de cubierta plana distan 18 metros. Del techo del ms bajo, de 12 metros de
alto, el ngulo de elevacin del borde del techo del ms alto es de 40

. Cul es la altura del


edicio ms alto?
45. El radio de un crculo es de 21,4 metros. Hallar:
a) La longitud de la cuerda subtendida por el ngulo central de 110

40

.
b) La distancia entre dos cuerdas paralelas del mismo lado del centro subtendidas por
ngulos centrales de 118

40

y 52

20

.
46. Mostrar que la base b de un tringulo issceles de lados iguales a y de ngulo en el vrtice
es b = 2aSen

2
.
47. Mostrar que el permetro P de un polgono regular de n lados inscrito en un crculo de
radio es P = 2nrSen
180

n
.
CAPTULO 11. FUNCIONES TRIGONOMTRICAS 548
48. Una rueda de 1,50 metros de dimetro sube por un plano inclinado 18

20

. Cul es la
altura del centro de la rueda sobre la base del plano cuando ha rodado 1,50 por l?
49. Una pared tiene 4,5 metros de altura y est a 3 metros de una casa. Hallar la longitud
de la escalera ms corta que toque precisamente el borde superior de la pared y alcance una
ventana que est a 6,15 metros sobre el suelo.
11.8. Ecuaciones trigonomtricas
Las ecuaciones Cosx = b, Senx = b, Tanx = b, Cotx = b suelen llamarse ecuaciones trigono-
metricas elementales. Suponga que se pide resolver la ecuacin elemental f(x) = A, donde f(x)
es una funcin trigonomtrica elemental. Diremos que la ecuacin elemental f(x) = A tiene p
por perodo principal, si p es el perodo principal de la funcin f(x). Si para cierta ecuacin
trigonomtrica elemental de perodo principal p se ha hallado una solucin x
0
, entonces cualquier
nmero x
k
= x
0
+k
p
ser tambin solucin de esta ecuacin para cualquier k entero. En este caso
el conjunto de todas las soluciones de la forma x
k
= x
0
+ k
p
, donde k Z, lleva el nombre de
serie de soluciones de la ecuacin. Para encontrar el conjunto de todas las soluciones de la ecuacin
trigonomtrica elemental dada, cuyo perodo principal de p, se deben hallar todas las soluciones de
esta ecuacin en el intervalo de longitud p, y luego escribir la serie de soluciones correspondiente
para cada una de las soluciones halladas.
Denicin 11.13 Ecuacin trigonomtrica
Una ecuacin se llama trigonometrica si ella contiene la incgnita slo bajo los signos de las
funciones trigonometricas.
Al resolver una ecuacin trigonomtrica elemental, el intervalo de longitud igual al perodo
principal p debe elegirse de un modo tal, que contenga un intervalo, sobre el cual para la funcin
f(x) quede denida la funcin trigonomtrica inversa, y, adems, que todas las soluciones de la
ecuacin en dicho intervalo puedan ser fcilmente determinadas. Al resolver una ecuacin que no
es elemental, nos vemos obligados, por regla general a realizar varias transformaciones, a veces bas-
tante numerosas. En este caso, la ecuacin se sustituye cada vez por alguna otra nueva y esta nueva
ecuacin puede tener, otras races. Una ecuacin dada se resolver correctamente, si, al realizar la
transformacin de las ecuaciones, una ecuacin se sustituye cada vez por otra ecuacin nueva que
tenga las races de la ecuacin anterior, es decir, que no haya prdida de races. Si una ecuacin se
sustituye cada vez por la ecuacin equivalente, las races de la ltima sern precisamente las races
de la ecuacin original.
Denicin 11.14 Solucin de una ecuacin trigonomtrica
Resolver una ecuacin trigonometrica signica hallar todos los ngulos que satisfacen dicha ecuacin,
es decir, que reducen la ecuacin a una igualdad despus de la sustitucin de la incgnita.
Sea dada la ecuacin f(x) = g(x) y supongamos que para todo x real se verica la igualdad
g(x) = h(x), entonces la ecuacin original es equivalente a la ecuacin f(x) = h(x).
Sea la ecuacin ASenx +BCosx = C. En el caso en que A = 0, o bien B = 0, esta ecuacin se
reduce a:
CAPTULO 11. FUNCIONES TRIGONOMTRICAS 549
a) La ecuacin elemental Cosx =
C
B
, si A = 0, B ,= 0.
b) La ecuacin elemental Senx =
C
A
, si A ,= 0, B = 0.
Suponga que A ,= 0 y B ,= 0. Esto signica que A
2
+B
2
,= 0. Segn esto llegamos a la ecuacin
dada es equivalente a la ecuacin
A

A
2
+B
2
Senx +
B

A
2
+B
2
Cosx =
C

A
2
+B
2
1. Sea A un nmero positivo. Examinamos dos casos: Sea B > 0. Construyamos un tringulo
rectngulo cuyos catetos son de longitud A y B. El ngulo opuesto al cateto de longitud B
se designar con . Entonces tenemos las igualdades numricas
Sen =
B

A
2
+B
2
y Cos =
A

A
2
+B
2
de las cuales se deduce que
= ArcSen
B

A
2
+B
2
y = ArcCos
A

A
2
+B
2
ArcSen
B

A
2
+B
2
= ArcCos
A

A
2
+B
2
Ahora la ecuacin tendr la forma
CosSenx +SenCosx =
C

A
2
+B
2
Sen(x +) =
C

A
2
+B
2
que es una ecuacin elemental. Sea B < 0. Construyamos un tringulo rectngulo con los
catetos A y [B[. El ngulo opuesto al cateto de longitud [B[ se designar con . En este caso
tenemos las igualdades
Sen =
[B[

A
2
+B
2
y Cos =
A

A
2
+B
2
de las cuales se deduce que
= ArcSen
[B[

A
2
+B
2
y = ArcCos
A

A
2
+B
2
ArcSen
[B[

A
2
+B
2
= ArcCos
A

A
2
+B
2
Por cuanto B = [B[, la ecuacin toma la forma
CosSenx SenCosx =
C

A
2
+B
2
Sen(x ) =
C

A
2
+B
2
que es una ecuacin elemental.
Si hacemos que = ArcTan
B
A
, veremos que = , cuando B > 0, y = , para B < 0.
Por eso podemos escribir que para A > 0 la ecuacin es equivalente a
Sen
_
x +ArcTan
B
A
_
=
C

A
2
+B
2
que es tambin una ecuacin elemental.
CAPTULO 11. FUNCIONES TRIGONOMTRICAS 550
2. El caso de A < 0 se reduce a lo analizado anteriormente, multiplicando ambos miembros
de la ecuacin por -1.
Ejemplo 11.60 Resuelva las ecuaciones:
a) Senx+Sen3x+4Cos
3
x = 0; b) Sen
4
xCos
4
x = Sen2x; c)

1 +Sen2x

1 Sen2x =
1.
Solucin
a) Senx + (3SenxCos
2
x Sen
3
x) + 4Cos
3
x = 0
Senx + 3(1 Sen
2
x)Senx Sen
3
x + 4(1 Sen
2
x)Cosx = 0
4Senx 4Sen
3
x + 4Cosx 4CosxSen
2
x = 0
(Senx +Cosx) (Senx +Cosx)Sen
2
x = 0
(Senx +Cosx)(1 Senx)(1 +Senx) = 0
_

_
1 Senx = 0
1 +Senx = 0
Senx +Cosx = 0
_

_
Senx = 1
Senx = 1
Senx = Cosx

_
_
_
x =

2
+k, k Z
x =

4
+k, k Z
b) (Sen
2
x Cos
2
x)(Sen
2
x +Cos
2
x) = Sen2x Sen
2
x Cos
2
x = Sen2x
Cos2x = Sen2x
Sen2x
Cos2x
= 1 Tan2x = 1
x =
1
2
ArcTan(1) x =
1
2
_
3
4
+k
_
, k Z
c)
_
1 +Sen2x

1 Sen2x
_
2
= 1
1 +Sen2x 2
_
1 Sen
2
2x + 1 Sen2x = 1 2 2

Cos
2
2x = 1
1 = 2

Cos
2
2x 1 = 4
_

Cos
2
2x
_
2
1 4Cos
2
2x = 0
(1 2Cos2x)(1 + 2Cos2x) = 0
_
1 2Cos2x = 0
1 + 2Cos2x = 0
_

_
Cos2x =
1
2
Cos2x =
1
2

_
_
_
x =

6
+k, k Z
x =

3
+k, k Z
Los mtodos de resolucin de las ecuaciones trigonometricas son variados y no existe una regla
general de resolucin de cada ecuacin. Por eso, nos limitaremos a mostrar en ejemplos algunos de
los mtodos de resolucin frecuentemente utilizados:
1.- Si la ecuacin contiene varias funciones trigonometricas diferentes de igual argumento, to-
das las funciones se pueden expresar mediante una de ellas, despus de lo cual obtendremos una
ecuacin algebraica con respecto a la incgnita, que designa la funcin por la cual se expresan
todas las dems.
2.- Si la ecuacin contiene funciones trigonometricas de distintos argumentos, en los que se en-
cuentra la incgnita, frecuentemente lo conveniente es reducir las funciones a un argumento.
CAPTULO 11. FUNCIONES TRIGONOMTRICAS 551
3.- Los ejemplos que expondremos demuestran que uno de los mtodos ms ecientes de resolu-
cin de las ecuaciones es la descomposicin del primer miembro de la ecuacin en factores despus
de pasar todos los trminos a ese miembro. Por eso, a veces se hace necesario recurrir a mtodos
articiosos de descomposicin.
4.- Si la ecuacin contiene senos y cosernos cuadrados del argumento incgnito, generalmente
se utilizan las formulas de reduccin de la potencia, sustituyendo Sen2x y Cos2x en funcin del
ngulo doble.
5.- La ecuacin que contiene trminos con productos de senos o cosenos, puede ser conveniente
reducirla a la forma en la que los productos sean sustituidos por sumas algebraicas.
Ejemplo 11.61 Resuelva las ecuaciones:
a) 2Cos
2
x + 5CosxSenx 3Sen
2
x = 0; b) 5Cosx + 4Senx = 3; c) Cosx Cos2x = 1.
Solucin
a) Despus de dividir por Cos
2
x, obtendremos:
3Tan
2
x 5Tanx 2 = 0
Resolviendo esta ecuacin cuadrtica, obtenemos:
Tanx =
5

25 + 24
6
=
5 7
6
Tanx
1
=
1
3
y Tanx
2
= 2
de donde
x
1
= ArcTan
1
3
+k y x
2
= ArcTan2 +k
donde k = 0, , 1, 2, ...
b) Haciendo z = Tan
x
2
, entonces:
Cosx =
Cosx
1
=
Cos
2 x
2
Sen
2 x
2
Cos
2
x
2
+Sen
2
x
2
=
1 Tan
2 x
2
1 +Tan
2
x
2
=
1 z
2
1 +z
2
Senx =
Senx
1
=
2Sen
x
2
Cos
x
2
Cos
2
x
2
+Sen
2
x
2
=
2Tan
x
2
1 +Tan
2
x
2
=
2z
1 +z
2
Reemplazando estas frmulas en la ecuacin planteada, tenemos:
5
2z
1 +z
2
+ 4
1 z
2
1 +z
2
= 3
4z
2
4z 1
1 +z
2
= 0
como el discriminante de la ecuacin cuadrtica es 32 0, entonces la ecuacin tiene races reales:
z =
4

16 + 16
8
=
1

2
2
z
1
=
1

2
2
y z
1
=
1 +

2
2
de donde
Tan
x
1
2
=
1

2
2

x
1
2
= ArcTan

2 1
2
+k x
1
= 2ArcTan

2 1
2
+ 2k
Tan
x
2
2
=
1 +

2
2

x
2
2
= ArcTan

2 + 1
2
+k x
2
= 2ArcTan

2 + 1
2
+ 2k
El menor ngulo positivo que satisface a la ecuacin dada es
x = 2ArcTan

2 + 1
2
CAPTULO 11. FUNCIONES TRIGONOMTRICAS 552
c) Representemos la ecuacin en la siguiente forma
Cosx (1 +Cos2x) = 0 Cosx 2Cos
2
x = 0 (1 2Cosx)Cosx = 0.
Si el producto es igual a cero, debe ser igual a cero aunque sea uno de los factores: o bien
Cosx = 0 x
1
=

2
(2k + 1)
o bien
1 2Cosx = 0 Cosx =
1
2
x
2
=

3
+ 2k, k = 0, 1, 2, ...
Ejemplo 11.62 Resuelva las ecuaciones:
a) Senx +Sen3x +Sen5x = 0; b) 3Senx +Cos
2
x = 2; c) 3Senx +Cosx = 1.
Solucin
a) Transformemos la suma Senx +Sen5x en producto:
Senx +Sen5x = 2Sen3xCos2x.
La ecuacin toma la forma
2Sen3xCos2x +Sen3x = 0 (2Cos2x + 1)Sen3x = 0.
Igualando a cero cada factor por separado, obtendremos:
Sen3x = 0 3x = k x
1
=
k
3
, k = 0, 1, 2, ...
2Cos2x + 1 = 0 Cos2x =
1
2
2x =
2
3
+ 2k
x
2
=

3
+k, k = 0, 1, 2, ...
b) Aqu es conveniente expresar Cos
2
x por el seno, despus de lo cual obtendremos una ecuacin
cuadrtica respecto a Senx:
Sen
2
x 3Senx + 1 = 0.
Resolvindola, obtendremos:
Senx =
3

5
2
x = (1)
k
22

30 + 180

k, k = 0, 1, ...
la segunda raz, Senx =
3+

5
2
se desprecia, puesto que
3+

5
2
> 1.
c) En este caso no conviene sustituir Cosx por

1 Sen
2
x, puesto que obtendramos una
ecuacin irracional respecto a Senx, y despus de librarnos del radical podran aparecer races
impropias. Lo sencillo es resolver esta ecuacin del siguiente modo:
3Senx (1 Cosx) = 0 6Sen
x
2
Cos
x
2
2Sen
2
x
2
= 0.
Despus de simplicar por 2 y descomponer el primer miembro en factores, tendremos:
_
3Cos
x
2
Sen
x
2
_
Sen
x
2
= 0
de donde
Sen
x
2
= 0 x
1
= 2k
y
Tan
x
2
= 3 x
2
= 2ArcTan3 + 2k
CAPTULO 11. FUNCIONES TRIGONOMTRICAS 553
Ejemplo 11.63 Resuelva las ecuaciones:
a) Senx+2Cos2x =
3
2
; b) SenxTanx+1 = Senx+Tanx; c) Cosx+Cos2x+Cos3x = 0.
Solucin
a) Aqu Cos2x se puede expresar solamente por Senx, puesto que Cos2x = 1 2Sen2x. Despus
de lo cual obtendremos una ecuacin cuadrtica respecto a Senx:
8Sen2x 2Senx 1 = 0,
Senx
1
=
1
2
x
1
=
(1)
k

6
k, k = 0, 1, 2, ...
Senx
2
=
1
4
x
2
= (1)
k+1
ArcSen
1
4
+k, k = 0, 1, 2, ...
b) Traslademos todos los trminos de la ecuacin al primer miembro:
SenxTanx + 1 Senx Tanx = 0
y descompongamos en factores el primer miembro de la ecuacin obtenida:
(Senx 1)(Tanx 1) = 0.
Por consiguiente, Senx 1 = 0 o bien Tanx 1 = 0. Como las soluciones de la ecuacin sern
x =

2
+ 2k y x =

4
+k, k Z.
c) Al resolver esta ecuacin no es necesario reducir todas las funciones a un argumento. Trans-
formemos el primer miembro de la ecuacin en un producto:
(Cosx +Cos3x) +Cos2x = 0 2Cos2xCosx +Cos2x = 0 (2Cosx + 1)Cos2x = 0,
de donde
x
1
=

4
(2k + 1), x
2
=
2
3
+ 2k, k = 0, 1, 2, ...
Ejemplo 11.64 Resuelva las ecuaciones:
a)
1+Tanx
1Tanx
= 1 +Sen2x; b)
Sen2x
1Cos2x
=
1+Cos2x
2Cosx
; c) Sen
4
x +Cos
4
x =
5
8
.
d) Sen5xCos3x Sen8xCos6x = 0.
Solucin
a) Representemos la ecuacin en la siguiente forma:
1 +Tanx
1 Tanx
1 = 2SenxCosx
2Tanx
1 Tanx
2SenxCosx = 0
_
1
(1 Tanx)Cosx
Cosx
_
Senx = 0
_
1
Cosx Senx
Cosx
_
Senx = 0
de donde
Senx = 0, x = k k = 0, 1, 2, ...
1
Cosx Senx
Cosx = 0
1 Cos
2
x +SenxCosx
Cosx Senx
= 0
Suponiendo que Cosx Senx ,= 0, hallamos
1 Cos
2
x +SenxCosx = 0 Sen
2
x +SenxCosx = 0 (Senx +Cosx)Senx = 0
CAPTULO 11. FUNCIONES TRIGONOMTRICAS 554
de donde o bien Senx = 0, entonces tendremos la primera raz, o bien
Senx +Cosx = 0 Tanx = 1, x
2
=

4
+k, k = 0, 1, ...
b) Como
Sen2x
1 Cos2x
=
2SenxCosx
2Sen
2
x
=
Cosx
Senx
y
1 +Cos2x
2Cos2x
=
2Cos
2
x
2Cosx
= Cosx
obtenemos la ecuacin
Cosx =
Cosx
Senx
(Senx 1)Cosx = 0
Sus soluciones sern x =

2
+k, k Z.
Puesto que el segundo miembro de la ecuacin dada pierde el signicado cuando x =

2
+ k,
entonces todos los valores encontrados para x no son las soluciones. No podramos perder ninguna
solucin, ya que al pasar de la ecuacin dada a la obtenida, el conjunto de los valores admisibles
para x se ha ampliado. Por consiguiente, la ecuacin dada no tiene soluciones.
c) Haciendo
_
1 Cos2x
2
_
2
+
_
1 Cos2x
2
_
2
=
5
8
(1 Cos2x)
2
+ (1 +Cos2x)
2
=
5
2
2 + 2Cos
2
2x =
5
2
o bien
1 +Cos4x =
1
2
Cos4x =
1
2
, x =

6
+
k
2
, k = 0, 1, 2, ...
d) Como
1
2
(Sen8x+Sen2x)
1
2
(Sen14x+Sen2x) = 0 Sen8xSen14x = 0 2Sen3xCos11x = 0
Sen3x = 0, x
1
=
k
3
, k = 0, 1, 2, ...
Cos11x = 0, x
2
=

22
(2k + 1), k = 0, 1, 2, ...
11.9. Tarea
1. Resuelva las ecuaciones:
a) 3Senx +Cos
2
x = 2;
b) 3Senx +Cosx = 1;
c) 5Cosx + 4Senx = 3;
d)
Sen2x
1 Cos2x
=
1 +Cos2x
2Cosx
;
e) Senx + 2Cos2x =
3
2
;
f ) Sen
4
x +Cos
4
x =
5
8
;
g) Cosx Cos2x = 1;
h) Tan2xTan3x = 1;
i) Sen2x +Sen3x = 2;
j) 1 Tan2x = 4Sen
2
2x;
k) 8Cos
4
x = 3 + 5Cos4x;
l) Sen
4
x + 5Cos2x + 4 = 0;
m) Cos
x
2
Cos
3x
2
= 3Sen
2
x
2
;
n)
1 +Tanx
1 Tanx
= 1 +Sen2x;
o) Cosx +Cos2x +Cos3x = 0;
CAPTULO 11. FUNCIONES TRIGONOMTRICAS 555
p) Senx +Sen3x +Sen5x = 0;
q) Cosx = Cos3x + 2Sen2x;
r) Sen5xSen4x = Cos6xCos3x;
s) 2

3Cos2x +Sen2x = 4Cos


2
3x;
t) 2Cos
2
x + 5CosxSenx 3Sen
2
x = 0;
u) Tan
_

2
Cosx
_
cot(Senx) = 0;
v) Tan
2
x + 8Cos2xCot2x = Cot
2
x;
w) SenxTanx + 1 = Senx +Tanx;
x) 2(Cotx Tanx) = Sen4x;
y) Sen5x +Senx + 2Sen
2
x = 1;
z) Cos15x = Sen5x.
2. Resuelva las ecuaciones:
a) 2Sen
_
3x +

4
_
=

1 + 8Sen2xCos
2
2x;
b) Cos7x Sen5x =

3(Cos5x Sen7x);
c) Tanx +Cotx =
_
1 +TanxTan
x
2
_
Senx;
d) Cos4xCos8x Cos5xCos9x = 0;
e) Senx + 7Cosx = 5;
f ) 5Senx 12Cosx = 13Sen3x;
g)
Sen2x
Sen
2x+
3
= 0;
h) 8Senx 7Cosx = 0;
i) Sen
2
x + 2SenxCosx 3Cos
2
x = 0;
j) 5Sen
2
x + 3SenxCosx 3Cos
2
x = 2;
k) 5Sen
2
x +

3SenxCosx + 6Cos
2
x = 5;
l) 2Cos
2
x + 4Cosx = 3Sen
2
x;
m) Sen2x + 5Senx + 5Cosx + 1 = 0;
n) 3Tan2x 4Tan3x = Tan
2
3xTan2x;
o) Senx + 2Sen2x = 3 +Sen3x;
p)
_
3 Cos
2
x + 3Sen5x = 1 Senx;
q)

1 +Sen2x =

2Cos2x;
r) Sen
4
x +Cos
4
y + 2 = 4SenxCosy;
s)
Cosx
1 +Cos2x
= 0;
t)
Senx +Cosx
Cos2x
= 0;
u) (1 +Cosx)
_
1
Senx
1
_
= 0;
v) CosxTan3x = 0;
w) Sen4xCosxTan2x = 0;
x) Sen
2
3x 5Sen3x + 4 = 0;
y) (1 +Cosx)Tan
x
2
= 0;
z) Tan
3
x +Tan
2
x 3Tanx = 3.
3. Resuelva las ecuaciones:
a) 8Cos
4
x 8Cos
2
x Cosx + 1 = 0;
b) 2Sen
3
x Cos2x Senx = 0;
c) 2Cos
2
x + 5Senx 4 = 0;
d) 3Sen
2
2x + 7Cos2x = 3;
e) 2Cos
2
x +Senx = 2;
f )

2Sen
2
x +Cosx = 0;
g) Sen2x +Cos2x = Senx +Cosx;
h)

2Cos2x = Cosx +Senx;
i) Sen3x = Cos2x;
j) Cos5x = Sen15x;
k) Sen(5 x) = Cos(2x + 7);
l) 4Sen
2
x +Sen
2
2x = 3;
m) 4Cos
2
2x + 8Cos
2
x = 7;
n) 8Sen
6
x + 3Cos2x + 2Cos4x + 1 = 0;
o) 3(1 Senx) = 1 +Cos2x;
p) Senx =
3
4
Cosx;
q) 3Senx = 2Cosx;
r) 3Sen
2
x + 3SenxCosx 6Cos
2
x = 0;
s) Sen
2
x + 3Cos
2
x 2Sen2x = 0;
t) 3Sen
2
x + 2SenxCosx = 2;
u) 2Cos
2
x 3SenxCosx + 5Sen
2
x = 3;
v) Sen5xCos3x = Sen9xCos7x;
w) Sen
6
x +Cos
6
x =
7
16
;
x) 2Cos
2
x +Cos5x = 1;
y) Senx +Sen2x +Sen3x = 0;
z) Senx +Sen3x +Cosx +Cos3x = 0.
4. Resuelva las ecuaciones:
a)
1 +Senx
1 +Cosx
=
1
2
;
b) Sen
3
x +Cos
3
x = 1;
c) 4Sen
4
3x 3Cosx + 5 = 0;
d) Sen2x +Tanx = 2;
e) Cos4x + 2Sen4x = 1;
CAPTULO 11. FUNCIONES TRIGONOMTRICAS 556
f )

3Sen2x +Cos2x =

2;
g)
1
2
Sen3x +

3
2
Cos3x = Sen5x;
h) 2Cos3x +

3Senx +Cosx = 0;
i) Sen5x +Cos5x =

2Cos13x;
j) Sen
2
x Cos2x = 2 Sen2x;
k) 8Sen
2
x
2
3Senx 4 = 0;
l) Sen
4
x +Cos
4
x = Cos4x;
m) 3Tan
x
2
+Cotx =
5
Senx
;
n) Cosx =
Tanx
1 +Tan
2
x
;
o) Cotx +
Senx
1 +Cosx
= 2;
p) 2Senx 3Cosx = 3;
q) 3Sen2x +Cos2x = 2;
r) Sen6xCos2x = Sen5xCos3x Sen2x;
s) Cos
4
x+Sen
4
xSen2x+
3
4
Sen
2
2x = 0;
t)
_
2Sen
4
x
2
1
_
1
Cos
4
x
2
= 2;
u) CosxCos2xCos4xCos8x =
1
16
;
v) 2Sen17x +

2Cos5x +Sen5x = 0;
w) 4Cos
2
x
2
+ 3

2Senx = 8Cos
x
2
;
x)
7
4
Cos
x
4
= Cos
3
x
4
+Sen
x
2
;
y) 4Sen2x Tan
2
_
x
x
4
_
= 4;
z) Cos
4x
3
= Cos
2
x.
5. Resuelva las ecuaciones:
a) SenxCosx 6Senx + 6Cosx + 6 = 0;
b) 4 4(Cosx Senx) Sen2x = 0;
c) 5Sen2x 11(Senx +Cosx) + 7 = 0;
d) Senx + 2Cosx = Cos2x Sen2x;
e) 32Cos
6
x Cos6x = 1;
f ) Tanx +Cotx Cos4x = 3;
g) Sen
5
x Cos
5
x =
1
Cosx

1
Senx
;
h) Sen
6
2x +Cos
6
2x =
41
128
;
i) Sen
10
x +Cos
10
x =
29
64
;
j) Sen
10
x +Cos
10
x =
29
16
Cos
4
2x;
k) [Cosx[ = Cosx 2Senx;
l) [Cotx[ = Cotx +
1
Senx
;
m)

5 2Senx = 6Senx 1;
n)

2 + 4Cosx =
1
2
+ 3Cosx;
o)
_
3 + 2Tanx Tan
2
x =
1 + 3Tanx
2
;
p)
_
3Sen5x Cos
2
x 3 +Senx = 1;
q) Tanx +
1
9
Cotx =
_
1
Cos
2
x
1 1;
r)
_
Cos
2
x +
1
2
+
_
Sen
2
x +
1
2
= 2;
s) 1 +Cos2xCos3x =
1
2
Sen
2
3x;
t) Sen5x +Senx = 2 +Cos
2
x;
u) 3Sen
2
x
3
+ 5Sen
2
x = 8;
v) (Senx +

3Cosx)Sen3x = 2;
w)
_
1
3
4
Sen
2
2x
_
Cos2x = 1;
x) Senx +Cosx =

2 +Sen
4
4x;
y) Cos
6
2x = 1 +Sen
4
x;
z) Cot
_

3
Cos2x
_
=

3.
6. Resuelva las ecuaciones:
a)

1 2Tanx

1 + 2Cotx = 2;
b) 2Cot2x 3Cot3x = Tan2x;
c) 6Tanx + 5Cot3x = Tan2x;
d) x
2
+ 2xSenxy + 1 = 0;
e) Sen
x
4
+ 2Cos
x 2
3
= 3;
f ) Sen
2
(x) + log
2
2
(y
2
2y + 1) = 0;
g) 2Sen
_

2
Cos
2
x
_
= 2 Cos(Sen2x);
h) Cos(x y) 2Senx + 2Seny = 3;
i) Sen18x+Sen10x+Sen2x = 3+Cos
2
2x;
j) 2Sen
_
2x
3


6
_
3Cos
_
2x +

3
_
= 5;
k) 4 +Sen
2
x +Cos
2
2x = 5Sen
2
xSen
2
y.
CAPTULO 11. FUNCIONES TRIGONOMTRICAS 557
7. Resuelva las ecuaciones:
a) Sen
6
x +Sen
4
x +Cos
6
x +Cos
4
x +Sen
x
2
= 3;
b) 1 2x x
2
= Tan
2
(x +y) +Cot
2
(x +y);
c) Tan
2
2x + 2

3Tan2x + 3 = Cot
2
_
4y

6
_
;
d)
_
Cos
2
x +
1
Cos
2
x
_
(1 +Tan
2
2y)(3 +Sen3x) = 4;
e) (Cosx Senx)
_
1 +
1
2
Senx
_
+Senx = 2Cos
2
x;
f ) Senx 2Sen2x +Sen3x = [1 2Cosx +Cos2x[;
g) 4 +Sen
2
x = (3 +

3)Sen2x 2(2

3)Cos
2
x;
h) Sen
_
2x +
5
2
_
3Cos
_
x
7
2
_
= 1 + 2Senx;
i) Sen
_
x +

6
_
+Cos
_
x +

3
_
= 1 +Cos2x;
j) Sen
6
x +Sen
4
xCos
2
x = Sen
3
xCos
3
x +SenxCos
3
x;
k) Sen
2
xCos
2
x 10SenxCos
3
x + 21Cos
4
x = 0;
l) Cos2x 3Cosx + 1 =
1
(Cot2x Cotx)Sen(x )
;
m) (Sen2x +

3Cos2x)
2
5 = Cos
_

6
2x
_
;
n) 2(1 Senx Cosx) +Tanx +Cotx = 0;
o) (1 +Cosx)
_
Tan
x
2
2 +Senx = 2Cosx;
p)

3Senx
_
2Sen
2
x Sen2x + 3Cos
2
x = 0;
q) Cosx +
_
Sen
2
x 2Sen2x + 4Cos
2
x = 0;
r)

Cos2x +

1 +Sen2x = 2

Senx +Cosx;
s) Tan
_
x

4
_
TanxTan
_
x +

4
_
=
4Cos
2
x
Tan
x
2
Cot
x
2
;
t)
_
Sen
2
x +
1
Sen
2
x
_
+
_
Cos
2
x +
1
Cos
2
x
_
2
= 12 +
1
2
Seny;
u)
_
Sen7xCosx Sen
x
2
Cos7x
_
Sen
2
5x =
Sen
3x
2
Cos
x
2
+SenxCos7x
1 +Cot
2
5x
.
8. Resuelva los sistemas de ecuaciones:
a)
_
SenxSeny = 0, 75
TanxTany = 3
;
b)
_
Sen
3
x =
1
2
Seny
Cos
3
x =
1
2
Cosy
;
c)
_

_
x +y +z =
TanxTanz = 2
TanyTanz = 18
;
d)
_

_
Senx = Cosy

6Seny = Tanz
2Senz =

3Cotx
;
e)
_
Sen(x +y) = 0
Sen(x y) = 0
;
f )
_
SenxCosy = 0, 25
SenyCosx = 0, 75
;
g)
_
Senx +Cosy = 0
Sen
2
x +Cos
2
y =
1
2
;
h)
_
SenxSeny = 0, 25
x +y =

3
;
i)
_
Senx +
1
Cosy
= 2
Senx
Seny
= 0, 5
;
CAPTULO 11. FUNCIONES TRIGONOMTRICAS 558
j)
_
Cosx +Cosy = 0, 5
Sen
2
x +Sen
2
y = 1, 75
;
k)
_
Senx +Seny = 0
Cosx +Cosy = 0
;
l)
_
x y =
1
3
Cos
2
x Sen
2
y = 0, 5
;
m)
_
Sen
2
x +Sen
2
y = 0, 75
x +y =

3
;
n)
_
Cos
2
x +Cos
2
y = 0, 25
x +y =
5
6
;
o)
_
Sen
2
x +Cos
2
y = 0, 5
x +y =

4
;
p)
_
Cosx +Cosy = 1
Cos
x
2
+Cos
y
2
=

22
2
;
q)
_
CosxSeny =

2
2
x +y =
3
4
;
r)
_
1Tanx
1+Tanx
= Tany
x y =

6
;
s)
_
Tanx +Tany = 1
x +y =

3
;
t)
_
SenxCoty =

5
2
TanxCosy =

3
2
;
u)
_
Cos(x y) = 2Cos(x +y)
CosxCosy = 0, 75
;
v)
_
Sen(x y) = 3SenxCosy 1
Sen(x +y) = 2CosxSeny
;
w)
_
Cos
x+y
2
Cos
xy
2
=
1
2
CosxCosy =
1
4
;
x)
_
SenxSeny =
1
4

2
TanxTany =
1
3
;
y)
_
Senx = 3Seny
Tanx = 5Tany
;
z)
_
x +y =

4
Tanx
Tany
=
3
4
.
9. Resuelva los sistemas de ecuaciones:
a)
_
x y =
5
3
Senx = 2Seny
;
b)
_
x +y =

4
TanxTany =
1
6
;
c)
_

2Senx = Seny

2Cosx =

3Cosy
;
d)
_
Cotx +Sen2y = Sen2x
2SenySen(x +y) = Cosx
;
e)
_
4Tan3y = 3Tan2x
2SenxCos(x y) = Seny
;
f )
_
Tanx +Coty = 3
[x y[ =

3
;
g)
_
Senx = Sen2y
Cosx = Seny
;
h)
_
x +y =
2
3
Senx
Seny
= 2
;
i)
_
Senx Seny =
1
2
Cosx +Cosy =

3
2
;
j)
_
Seny = 5Senx
3Cosx +Cosy = 2
;
k)
_
CosxCosy =
1+

2
4
CotxCoty = 3 + 2

2
;
l)
_
Sen
2
x = CosxCosy
Cos
2
x = SenxSeny
;
m)
_
Cos
2
y + 3SenxSeny = 0
2 +Cos2x Cos2y = 10
;
n)
_
Sen
2
x = CosxCosy
Cos
2
x = SenxSeny
;
o)
_
Sen
2
x = Seny
Cos
4
x = Cosy
;
p)
_

_
x +y +z =
TanxTanz = 3
TanyTanz = 6
;
q)
_

_
x +y +z =
TanxTany = 2
Tanx +Tany +Tanz = 6
;
CAPTULO 11. FUNCIONES TRIGONOMTRICAS 559
r)
_

_
x +y +z =
Senx = 2Seny

3Seny = Senx
;
s)
_

_
Sen
2
x +Sen
2
y +Sen
2
z = 1
Cos
2
x +Cos
2
y Cos
2
z = 1
Tan
2
x Tan
2
y +Tan
2
z = 1
.
10. Resuelva los sistemas de ecuaciones:
a)
_
SenxCos(x +y) +Sen(x +y) = 3Cos(x +y)
4Senx = 5Cot(x +y)
;
b)
_
Cos
2
4x +

262
2
Tan(2y) =

261
4
Tan
2
(2y)

262
2
Cos4x =

261
4
;
c)
_
x +y =

6
5(Sen2x +Sen2y) = 2[1 +Cos
2
(x y)]
.
11.10. Desigualdades trigonomtricas
Sea f(x) una funcin trigonomtrica elemental de perodo principal igual a p, y sea dada la
desigualdad
f(x) > A o f(x) < A.
Elegimos un intervalo de longitud igual a p y hallamos en dicho intervalo la solucin de la desigual-
dad dada. Supongamos que el conjunto de todas las soluciones de la desigualdad en el intervalo
citado est representado por el intervalo (a; b), donde a < b y b a p. Entonces, haciendo uso
de la periodicidad de la f(x), llegamos a que el conjunto de todas las soluciones de la desigualdad
es la unin de una innidad de todos los intervalos (a +kp; b +kp), donde k es un nmero entero
cualquiera.
Ntese, adems, que el intervalo de longitud igual al perodo principal p puede ser cualquiera,
mas se elige, corrientemente, de modo tal, que satisfaga dos condiciones:
1. Ha de contener un trozo en el que para la funcin dada f(x) est denida una funcin
trigonomtrica inversa.
2. El conjunto de todas las soluciones de la desigualdad dada en dicho trozo represente en s
un intervalo.
Ejemplo 11.65 Resuelva las inecuaciones:
a) Sen
4
x > Cos
4
x; b) 4Cos
2
2x + 8Cos2x 5 < 0; c) Tan
1
1 +x
2
> 1.
Solucin
a) Sabemos que
Cos
4
x Sen
4
x < 0 (Cos
2
x +Sen
2
x)(Cos
2
x Sen
2
x) < 0 Cos2x < 0
CAPTULO 11. FUNCIONES TRIGONOMTRICAS 560
Quiere decir, que la desigualdad Cos
4
xSen
4
x < 0 es equivalente a la desigualdad Cos2x < 0. El
conjunto de todas las soluciones de la ultima desigualdad es la serie de intervalos
_

4
+k;
3
4
+k
_
,
k Z.
b) Esta desigualdad es una desigualdad cuadrtica, haciendo t = Cos2x. Resolviendo la desigual-
dad 4t
2
+8t 5 < 0, obtenemos que el conjunto de todas sus soluciones es el intervalo
_

5
2
;
5
2
_
.
Por consiguiente, la desigualdad original, es equivalente al sistema de desigualdades
Cos2x >
5
2
y Cos2x <
1
2
El conjunto de todas las soluciones de la primera desigualdad es toda la recta real. El conjunto
de todas las soluciones de la segunda desigualdad es la serie de intervalos
_

6
+k;
5
6
+k
_
,
k Z. Quiere decir, el conjunto de todas las soluciones de la desigualdad es la serie de intervalos
_

6
+k;
5
6
+k
_
, k Z.
c) La funcin y = Tan
1
1 +x
2
es la superposicin de dos funciones: la funcin elemental mas sim-
ple y = Tanz y la funcin y =
1
1 +x
2
. Resolvamos primero la desigualdad elemental Tanz > 1. El
conjunto de todas las soluciones de esta desigualdad es una serie de intervalos
_

4
+k;

2
+k
_
,
k Z. Quiere decir, la desigualdad de partida es equivalente al conjunto innito de sistemas de
desigualdades
1
1 +x
2
<

2
+k y
1
1 +x
2
>

4
+k
donde k es un numero entero cualquiera. Examinemos todos los sistemas en el conjunto innito.
Cualquiera que sea k positivo, ninguno de estos sistemas tiene solucin, puesto que

4
+ k > 1
para cualquier k natural, y
1
1 +x
2
1 para cualquier x real, a consecuencia de lo cual la segunda
desigualdad en el sistema de desigualdades no tiene soluciones. Siendo k negativo, ninguno de los
sistemas dados tiene solucin, puesto que
1
1 +x
2
> 0 para todo x real, y

2
+k < 0 para cualquier
k entero y negativo, a consecuencia de lo cual la primera desigualdad del sistema de desigualdades
no tiene soluciones. Cuando k = 0, tenemos el sistema
1
1 +x
2
<

2
y
1
1 +x
2
>

4
Por cuanto
1
1 +x
2
1 <

2
para cualquier x real, el conjunto de todas las soluciones de la primera
desigualdad de este sistema es toda la recta numrica. Para todos los x reales la funcin y = 1+x
2
positiva, por lo cual, suprimiendo el denominador, obtenemos la desigualdad 1 + x
2
<

4
que
es equivalente a la segunda desigualdad del sistema ltimo. El conjunto de todas las soluciones
de esta desigualdad elemental est representado por el intervalo
_

_
4

1;
_
4

1
_
. Quiere
decir, el conjunto de todas las soluciones del sistema es precisamente este intervalo. Al resumir,
concluimos que el conjunto de todas las soluciones de la desigualdad de partida es el intervalo
_

_
(4 )

;
_
(4 )

_
.
CAPTULO 11. FUNCIONES TRIGONOMTRICAS 561
11.11. Tarea
1. Resuelva las inecuaciones:
a) Senx >
1
2
;
b) Cosx <
1
3
;
c) Tanx
1
2
;
d) Cotx <

3
3
;
e) Senx >
1
2
;
f ) Cosx <

3
2
;
g) Tanx

3
3
;
h) Cosx
7
10
;
i) Senx <
1
5
;
j) Cotx 1;
k) Tanx 5;
l) Cotx >

3
4
.
2. Resuelva las inecuaciones:
a) 2Sen
2
_
x +

4
_
+

3Cos2x > 0;
b) 6Sen
2
x SenxCosx Cos
2
x > 2;
c) Senx +Cosx <
1
Senx
;
d)

3Sen2x +Cos2x < 1;
e) Cos3x +

Sen2x <

2;
f ) Cos2x +Cosx > 0;
g)
Cosx
1 +Cos2x
< 0;
h) Sen3x > Cos3x;
i) Tanx + 3Cotx 4 > 0;
j) Sen
2
x Cos
2
x 3Senx + 2 < 0;
k) 2Sen
2
x
2
+Cos2x < 0;
l) Tan
3
x + 3 > 3Tanx +Tan
2
x;
m)
Sen3x Cos3x
Sen3x +Cos3x
< 0;
n) 5Sen
2
x 3SenxCosx 30Cos
2
x > 0;
o) 2Sen
2
x 4SenxCosx + 9Cos
2
x > 0;
p) Cos
2
x + 3Sen
2
x + 2

3SenxCosx < 1;
q) 3Sen
2
x +Sen2x Cos
2
x 2;
r)

3Cos
2
x < 4Tanx;
s) Sen4x +Cos4xCot2x > 1;
t) 2 +Tan2x Cot2x < 0;
u) 2(Cosx

8Tanx)Cosx < 5;
v) Senx +Cosx <
1
Cosx
;
w) Sen
6
x +Cos
6
x <
7
16
;
x) Cotx +
Senx
Cosx 2
0;
y) Cos
2
2x +Cos
2
x 1;
z) 8Sen
2
x
2
+ 3Senx 4 > 0.
3. Resuelva las inecuaciones:
a) Senx +Cosx >

2Cos2x;
b) Tanx +Tan2x +Tan3x > 0;
c) Cos2xCos5x < Cos3x;
d) Sen2xSen3x Cos2xCos3x > Sen10;
e) Cotx+Cot
_
x +

2
_
+2Cot
_
x +

3
_
> 0;
f ) 2Sen
2
x Senx +Sen3x < 1;
g) 4SenxSen2xSen3x > Sen4x;
h)
Cos
2
2x
Cos
2
x
3Tanx;
i) 3Cos
2
xSenx Sen
2
x <
1
2
;
j)
Cosx + 2Cos
2
x +Cos3x
Cosx + 2Cos
2
x 1
> 1.
4. Resuelva los sistemas de inecuaciones:
a)
_
Senx <

3
2
Cosx >

2
2
;
b)
_
Senx <
1
2
Cosx <
1
2
;
c)
_
Senx >

3
2
Tanx 0
;
d)
_
Cosx

2
2
Cotx >

3
;
e)
_
Tanx < 1
Cotx

3
3
;
f )
_
Senx >
1
5
Cosx <
1
5
;
CAPTULO 11. FUNCIONES TRIGONOMTRICAS 562
g)
_
Cosx
3
5
Tanx < 3
;
h)
_
Senx <
4
7
Cotx < 2
;
i)
_
Tanx > 0, 23
Cotx 0, 3
;
j)
_
Cosx < 0
Sen
3x
5
> 0
;
k)
_
Sen
x
2
<
1
2
Cos2x >
1
2
.
11.12. Funciones trigonomtricas
Las funciones trigonomtricas son importantes, no slo por su relacin con los lados y los n-
gulos de un tringulo, sino ms bien por las propiedades que poseen como funciones. Las seis
funciones trigonomtricas tienen en comn una propiedad importante llamada periodicidad.
No debe sorprendernos que estudiemos funciones del tipo que a continuacin denimos, ya que
sabemos que las estaciones del ao, las mareas y otros fenmenos ocurren y vuelven a ocurrir a
intervalos regulares.
Denicin 11.15 Funcin trigonomtrica
Se denomina funcin trigonomtrica de argumento numrico x la funcin de un ngulo que contiene
x radianes.
Denicin 11.16 Funcin peridica
Una funcin y = f(x) se llama peridica, si existe tal nmero p ,= 0 que para cualquier x,
perteneciente al dominio de la funcin y = f(x), los nmeros x + p y x p tambin integran
el dominio y para todo x del dominio se verica f(x +p) = f(x). Se llama perodo de la funcin,
el menor nmero positivo, cuya suma a un valor cualquiera del argumento no vara el valor de la
funcin. Si una funcin peridica tiene un rango de 2A en sus valores, entonces se dice que A es
la amplitud de la funcin.
Para una funcin peridica tiene lugar la igualdad f(x p) = f(x). En efecto, la funcin
y = f(x) en el punto x p esta denida y f(x) = f[(x p) +p] = f(x p).
Teorema 11.39 Si un nmero p es el perodo de la funcin y = f(x), entonces el nmero q = mp,
donde m es cualquier nmero entero y jo distinto de cero, tambin ser perodo de dicha funcin.
Todas las funciones trigonomtricas son peridicas, adems el perodo del seno, del coseno, de
la secante y la cosecante es igual a 2, en tanto que para la tangente y la cotangente el perodo
es igual a , lo que se aprecia de las frmulas de reduccin. La propiedad de periodicidad de la
funcin f(x) se escribe como f(x) = f(x +p), donde p es el perodo de la funcin.
El perodo no slo se puede sumar al argumento, sino tambin se le puede restar; adems, se
puede sumar y restar del argumento cualquier nmero entero de perodos. Ntese que por perodo
de una funcin suele entenderse su perodo mnimo positivo.
Muchos problemas de Fsica e Ingeniera tratan fenmenos peridicos, tales como vibraciones,
movimiento planetario y de ondas y las funciones seno y coseno constituyen la base para el anlisis
de las dems funciones.
Las funciones seno y coseno pueden introducirse de varias maneras. Por ejemplo, hay deni-
ciones que relacionan las funciones seno y coseno a los ngulos y hay otras de carcter analtico
CAPTULO 11. FUNCIONES TRIGONOMTRICAS 563
que introducen esas funciones sin referencia alguna a la Geometra. Unas y otras son equivalentes,
en el sentido de que todas ellas conducen a las mismas funciones.
Para construir la grca de una funcin cuyo perodo principal es p, basta construirla en el
segmento de longitud p y prolongarla luego peridicamente.
11.12.1. Funcin seno
Representemos grcamente la variacin de la funcin y = Senx al variar el argumento x desde
x = 0 hasta x = 2, o en radianes, desde 0

hasta 360

. Esto se puede realizar sencillamente del


modo siguiente:
Trazamos una circunferencia de radio unitario y la dividimos en 16 partes iguales. A cada di-
visin de arco corresponde un ngulo central de 22, 30

, o en radianes,

8
. Por el eje 0X llevamos
los ngulos 0,

8
,

4
,
3
8
,

2
,
5
8
, ..., representndolos en forma de segmentos en la escala elegida. En
los puntos de divisin trazamos perpendiculares al eje 0X y en ellas llevamos los valores del seno
de los correspondientes ngulos. Los valores del seno los hallamos por construccin, proyectando
los puntos de divisin de la circunferencia sobre el eje 0Y y transportando las proyecciones sobre
las correspondientes perpendiculares. Por los extremos de las perpendiculares trazamos una lnea
suave.
La curva obtenida se lla-
ma sinusoide o senoide. Hemos
construido slo una onda de la
sinusoide, correspondiente a la
variacin del argumento de 0 a
2. Debido a la periodicidad
de la funcin Senx, la ulterior
variacin del argumento x en
el intervalo de 2 a 4 da lu-
gar a la formacin de la segun-
da onda de la sinusoide, igual
a la primera. Lo mismo ocurrir si quisisemos construir la parte de la curva que corresponde a
la variacin del argumento x desde 0 hasta 2. La grca reeja la marcha de variacin de la
funcin. De la grca se establecen fcilmente las propiedades de la funcin y = Senx:
La funcin Senx est denida para cualquier valor real del argumento x, es decir, su
dominio son todos los nmeros reales, admitidos como medida en radianes del ngulo;
Todos los valores de la funcin Senx abarcan el segmento [-1; 1], es decir 1 Senx 1;
La funcin est acotada inferior y superiormente;
La funcin toma su valor mnimo y = 1 para cada x
k
=

2
+2k, donde k es un nmero
entero cualquiera, como tambin su valor mximo y = 1 para cada x
m
=

2
+2m, donde m
es un nmero entero cualquiera;
La funcin es peridica, de perodo principal igual a 2;
La funcin es impar, puesto que la curva es simtrica con respecto al origen de coordenadas;
CAPTULO 11. FUNCIONES TRIGONOMTRICAS 564
La funcin Senx no es montona en todo el dominio pero, crece en cada intervalo
_

2
+ 2k;

2
+ 2k
_
,
donde k es un nmero entero cualquiera, variando desde -1 hasta +1, y decrece en todo in-
tervalo
_

2
+ 2k;
3
2
+ 2k
_
, donde k es un nmero entero cualquiera, desde +1 hasta -1;
Los puntos de interseccin con los ejes coordenados son aquellos que tienen las coordenadas
(k, 0), donde k es un nmero entero cualquiera.
Mostremos, que en el segmento
_

2
;

2

la funcin y = Senx es creciente, es decir, que para


cualquier par de nmeros x
1
y x
2
es tal, que

2
x
1
< x
2


2
se verica la desigualdad
Senx
1
< Senx
2
. Para cualquier par de nmeros x
1
y x
2
tenemos, segn la frmula para la diferencia
de los senos:
Senx
1
Senx
2
= 2Sen
x
1
x
2
2
Cos
x
1
+x
2
2
Demostremos que el segundo miembro de esta igualdad es negativo, si

2
x
1
< x
2


2
. La
condicin x
2


2
es equivalente a la condicin

2
x
2
. Al sumar esta igualdad con la igualdad

2
x
1
, obtendremos x
1
x
2
. Tomando en consideracin que la desigualdad x
1
< x
2
es
equivalente a la desigualdad x
1
x
2
< 0, tenemos x
1
x
2
< 0, o bien

2

x
1
x
2
2
< 0. Por
consiguiente, Sen
x
1
x
2
2
< 0. Al sumarlas desigualdades

2
x
1
<

2
y

2
< x
2


2
obtenemos
< x
1
+x
2
< , o bien

2
<
x
1
x
2
2
<

2
. Por consiguiente, Cos
x
1
+x
2
2
> 0. As pues, el segundo
miembro de la igualdad es inferior a cero, por consiguiente, Senx
1
< Senx
2
.
Mostremos que en el segmento
_

2
;
3
2

la funcin y = Senx es decreciente, es decir, que para


cualquier par de nmeros x
1
y x
2
tal, que

2
x
1
< x
2


2
se verica la desigualdad Senx
1
>
Senx
2
. Adicionando a las desigualdades

2
x
1
< x
2

3
2
, tenemos

2
x
1
< x
2

2
. En virtud de que en el segmento
_

2
;

2

la funcin y = Senx es montona creciente, tenemos


para x
1
y x
2
que Sen(x
1
) < Sen(x
2
). Ahora, es vlida la cadena de desigualdades
equivalentes:
Sen(x
1
) < Sen(x
2
) Sen[( x
1
)] < Sen[( x
2
)]
Sen( x
1
) < Sen( x
2
) Sen( x
1
) > Sen( x
2
) Senx
1
> Senx
2
.
Quiere decir que es vlida la desigualdad Senx
1
> Senx
2
, lo que se trataba de demostrar. De
modo anlogo se demuestra que la funcin y = Senx es creciente en cada intervalo
_

2
+ 2k;

2
+ 2k

,
donde k es un nmero entero cualquiera, y decreciente en cada intervalo
_

2
+ 2k;
3
2
+ 2k

, donde
k es un nmero entero cualquiera.
Teniendo presente el carcter peridico de la funcin, se puede construir la grca de la fun-
cin y = Senx, que se llama sinusoide. La periodicidad, la simetra con respecto al origen, y la
amplitud de Senx se aplicarn para trazar la grca de y = Senx. Vemos que Senx es simtrico
con respecto al origen, ya que Sen(x) = Senx. Como Senx es de perodo 2, podemos obtener
CAPTULO 11. FUNCIONES TRIGONOMTRICAS 565
tanta expresin de la grca de y = Senx como deseemos, usando una faja de longitud horizontal
2 como patrn. Adems, debido a la simetra con respecto al origen, es posible obtener la parte de
la grca desde x = 0 hasta x = a, empleando esa forma desde x = 0 hasta x = a como un patrn.
En particular si trazamos la parte de la grca de x = 0 a x = , puede emplearse como patrn
para obtener la parte de x = 0 a x = . Tenemos entonces una faja de longitud horizontal 2, y
podemos usarla como patrn para obtener la grca en la extensin que se desee. Haciendo una
tabla, se indican los valores correspondientes de x e y = Senx para varios valores de x desde 0
hasta . Los valores correspondientes de x y y se usan como las coordenadas de un punto, estando
localizados varios de esos puntos, por los que se traz la curva. La parte llena de la curva de
la gura se obtiene con los datos de una tabla, y la otra parte se dibuj mediante la propiedad
Sen(x) = Senx. Ahora tenemos una faja de la curva de longitud horizontal 2, y podemos
obtener la grca en la extensin que deseemos usando esto como un patrn.
Grca de la funcin y = Sen(x +k):
Veamos como estn relacionadas entre s las grcas de las funciones y = Senx e y = Sen(x+k).
Supongamos para certeza que k > 0. Para iguales valores de la variable independiente x los
argumentos de estas dos funciones se diferencian en la magnitud constante (x+k) x = a. Debido
a esto, a todo punto P de la grca de y = Senx le corresponder un punto Q de la segunda grca,
de igual ordenada pero la abscisa del punto Q, es menor que la del punto P en la magnitud k. De
este modo, cualquier punto de la primera grca puede transformarse en el punto correspondiente
de la segunda grca transportndolo paralelamente al eje 0X en la magnitud k en sentido negativo.
Si se desplaza la sinusoide y = Senx a lo largo del eje 0X a la magnitud k, en sentido nega-
tivo, se producir la unin (coincidencia) de estas dos grcas, y puede decirse que la grca de
y = Sen(x +k) es la sinusoide y = Senx desplazada a la magnitud k a lo largo del eje 0X hacia la
izquierda. En general, la grca de la funcin y = Sen(x+k) es la sinusoide y = Senx, desplazada
a la magnitud [k[ a lo largo del eje 0X hacia la derecha cuando k < 0, y hacia la izquierda cuando
k > 0.
Grca de la funcin y = kSenx:
La grca de la funcin y = kSenx se obtiene de la grca de y = Senx alargando las ordenadas
k veces en direccin del eje 0Y cuando k > 1 y comprimindolas
1
k
veces, si 0 < k < 1. El numero
k se llama amplitud de la sinusoide y = kSenx y denota la desviacin mxima de los puntos de la
grca del eje 0X, es decir, la ordenada mayor en valor absoluto de la curva.
La grca de la funcin y = kSenx para k < 0 se obtiene de la grca de y = Senx alargando las
CAPTULO 11. FUNCIONES TRIGONOMTRICAS 566
ordenadas, si [k[ > 1, respectivamente comprimiendo, si [k[ < 1, con la ulterior reexin respecto
al eje 0X.
Grca de la funcin y = Sen(kx +a), k > 0:
El argumento kx + a se puede representar en la forma k(x + b), donde b =
a
k
. En tal caso,
y = Senk(x + b). Desplazando paralelamente (trasladando) la sinusoide y = Senx la magnitud
b en direccin al eje 0X (en [b[ hacia la derecha, si b < 0, y hacia la izquierda cuando b > 0)
logrando que a la nueva posicin de la sinusoide corresponde la nueva ecuacin y = Sen(x +b). Si
ahora reducimos la longitud de la onda k veces, si k > 1 (correspondientemente alargamos
1
k
veces,
si k < 1), a tal transformacin geomtrica secundaria de la sinusoide le corresponde la ecuacin
y = Senk(x + b), o y = Sen(kx + a). As, pues, la grca de la funcin y = Sen(kx + a) es una
sinusoide transformada o deformada.
Grca de la funcin y = bSen(kx +a):
Esta grca es una deformacin de la grca de y = Sen(kx +a), es decir, el alargamiento en
b veces de todas las ordenadas de la grca en direccin al eje 0Y, si b > 1, o la compresin en
1
b
veces, si 0 < k < 1 (si k < 0, la respectiva compresin o alargamiento se realiza con la reexin
ulterior respecto al eje 0X).
Grca de la funcin y = aCoskx +bSenkx:
Demostremos que
y = aCoskx +bSenkx (1)
CAPTULO 11. FUNCIONES TRIGONOMTRICAS 567
puede ser reducida a la forma
y = cSen(kx +).
Multiplicamos y dividimos el segundo miembro de la igualdad (1) por :
y =
_
a
2
+b
2
_
a

a
2
+b
2
Coskx +
b

a
2
+b
2
Senkx
_
.
Pongamos
a

a
2
+b
2
= Sen y
b

a
2
+b
2
= Cos
lo que siempre es posible, puesto que, en valor absoluto, cada una de las fracciones
a

a
2
+b
2
y
b

a
2
+b
2
no es mayor que la unidad y la suma de sus cuadrados es igual a la unidad:

a
2
+b
2

1,

a
2
+b
2

1,
_
a

a
2
+b
2
_
2
+
_
b

a
2
+b
2
_
2
= 1
En tal caso tendremos
y =
_
a
2
+b
2
(SenCoskx +CosSenkx)
=
_
a
2
+b
2
Sen(kx +)
= cSen(kx +)
donde c =

a
2
+b
2
.
Ejemplo 11.66 Encuentre el dominio de las funciones:
a) f(x) =
1 +Senx
Senx
; b) f(x) =
1
(Senx 2Sen
2
x)
3
4
; c) f(x) =
_
1
2
Senx.
Solucin
a) Esta funcin est determinada si Senx ,= 0, con lo que obtenemos que x ,= 0, , 2. De esta
manera podemos observar que el dominio de la funcin es el conjunto de valores en los cuales
x ,= n, donde n es un nmero entero.
b) Esta funcin est determinada si Senx2Sen2x > 0, lo cual implica que (12Senx)Senx > 0.
CAPTULO 11. FUNCIONES TRIGONOMTRICAS 568
Encontrando los valores para Senx = 0 y 1 2Senx = 0, obtenemos que x = 0, , 2, ... y
x =

6
,
5
6
,
3
2
, ..., respectivamente. Relacionando estos valores con la desigualdad antes mencionada,
tenemos que el dominio parcial es x
_
0;

6
_

_
5
6
;
_
, pero como la funcin seno tiene periodo
2, entonces de forma general el dominio de la funcin es x
_
2;

6
+ 2
_

_
5
6
+ 2n; + 2n
_
,
donde n es un nmero entero.
c) Esta funcin est determinada si
1
2
Senx 0, de donde 2Senx 1 0. Resolviendo esta
ecuacin trigonomtrica, obtenemos las races x =
7
6
,

6
,
5
6
. Por lo tanto el dominio de la funcin
es x
_

7
6
;

6

. Como la funcin es peridica de perodo 2, entonces el dominio general de la


funcin es x
_

7
6
+ 2n;

6
+ 2n

, donde n es un nmero entero.


Ejemplo 11.67 Encuentre el rango de las funciones:
a) f(x) =
1 +Senx
Senx
; b) f(x) =
_
1
2
Senx.
Solucin
Para encontrar el rango de la funcin trigonomtrica, debemos expresar la variable independiente
en funcin de la variable dependiente:
a) ySenx Senx = 1 (y 1)Senx = 1 x = ArcSen
1
y 1
Para que esta expresin tenga sentido, debe cumplirse la siguiente desigualdad:
1
1
y 1
1
_

_
1
y 1
1
1
y 1
1

_
y
y 1
0
y 2
y 1
0
Para encontrar la solucin general de la desigualdad, hacemos la interseccin de las soluciones
parciales: y (; 0] [2; +).
b) y
2
=
1
2
Senx Senx =
1
2
y
2
x = ArcSen
_
1
2
y
2
_
Para que esta expresin tenga sentido, debe cumplirse la siguiente desigualdad:
1
1
2
y
2
1
_

_
1
y 1
0
2y 1
y 1
0

_
_
_
y (; 1)
y
_
;
1
2
_
(1; +)
Para encontrar la solucin general de la desigualdad, hacemos la interseccin de las soluciones
parciales. Es decir la solucin es: y
_
;
1
2
_
.
Ejemplo 11.68 Encuentre el perodo positivo mnimo de las funciones:
a) f(x) =
1 +Senx
Senx
; b) f(x) = Sen
4
x +Cos
4
x; c) f(x) = Senx +Sen
_
x +

3
_
.
Solucin
a)
1 +Sen(x +p)
Sen(x +p)
=
1 +Sen(x + 2)
Sen(x + 2
Sen(x + 2) +Sen(x +p)Sen(x + 2) = Sen(x +p) +Sen(x + 2)Sen(x +p)
Sen(x + 2) = Sen(x +p) x + 2 = x +p p = 2.
b) y =
_
1
2

1
2
Cos2x
_
2
+
_
1
2
+
1
2
Cos2x
_
2
y =
3
4
+
1
4
Cos4x
3
4
+
1
4
Cos4(x +p) =
3
4
+
1
4
Cos(4x + 2) Cos(4x + 4p) = Cos(4x + 2)
CAPTULO 11. FUNCIONES TRIGONOMTRICAS 569
4x + 4p = 4x + 2 4p = 2 p =

2
c) y = Senx +SenxCos

3
+CosxSen

3
y =
3
2
Senx +

3
2
Cosx
3
2
Sen(x +p) +

3
2
Cos(x +p) =
3
2
Sen(x + 2) +

3
2
Cos(x + 2)
_
x +p = x + 2
x +p = x + 2
p = 2
.
Ejemplo 11.69 Determine la paridad de las funciones:
a) f(x) =
Cosx
1 Senx
; b) f(x) =
Senx
1 Tan
2
x
.
Solucin
a) f(x) =
Cos(x)
1 Sen(x)
=
Cosx
1 +Senx
,= f(x).
La funcin no es par ni impar.
b) f(x) =
Sen(x)
1 Tan
2
(x)
=
Senx
1 Tan
2
x
=
Senx
1 Tan
2
x
= f(x).
La funcin es impar.
Ejemplo 11.70 Gracar la funcin:
f(x) =
_
1 Sen
2
x
Solucin
Para poder gracar esta curva, debemos hacer un anlisis completo. La expresin esta denida si
se cumple la siguiente desigualdad
1 Sen
2
x 0 Cos
2
x 0
Esta ecuacin se satisface para cualquier valor real, es decir el dominio es x R.
Para encontrar el rango, hacemos:
y
2
= 1 Sen
2
x Senx =
_
1 y
2
x = ArcSen
_
1 y
2
Esta expresin esta denida si cumple las siguientes condiciones:
1
_
1 y
2
1
_
y [1; 1]
y [0; +)
Intersecando todos estos intervalos, encontramos la solucin al sistema de desigualdades: y [0; 1].
El periodo se establece de la siguiente manera:
f(x) =
_
1
1
2
+
1
2
Cos2x =
_
1
2
+
1
2
Cos2x
_
1
2
+
1
2
Cos2(x +p) =
_
1
2
+
1
2
Cos(2x + 2)
2x + 2p = 2x + 2 p =
CAPTULO 11. FUNCIONES TRIGONOMTRICAS 570
Aplicando el teorema de mximos y mnimos, tenemos:
_
1 Sen
2
x =
_
1 Sen
2
k Sen
2
x = Sen
2
k Sen
2
x Sen
2
k = 0
(Senx Senk)(Senx +Senk) = 0
_
Senx = Senk
2Senk = 0
_
(Senx 1)
2
= (Senk 1)
2
k = n

_
(Senx Senk)(Senx +Senk 2) = 0
2Senk = 0
_

_
Senx Senk = 0
Senx +Senk 2 = 0
k = n

_
Senx = Senk
Senk = 2
k = n

_
k =

2
+n
k = n
Haciendo los reemplazos, obtenemos que k = n, n Z son puntos de mximo y k =

2
+ n,
n Z son puntos de mnimo.
Como
f(x) =
_
1 Sen
2
(x) =
_
1 Sen
2
x = f(x)
la funcin es par. De acuerdo a los puntos de mximos y mnimos, podemos establecer los intervalos
de monotona:
_
n;

2
+n
_
, n Z son intervalos de decrecimiento y
_

2
+n; n
_
, n Z son
intervalos de crecimiento.
11.12.2. Funcin Coseno
Haciendo uso de las propiedades del coseno de un ngulo, obtenemos las siguientes caracters-
ticas de la funcin y = Cosx:
La funcin Cosx est denida para cualquier valor real del argumento x, es decir, su
dominio son todos los nmeros reales, admitidos como medida en radianes del ngulo;
Todos los valores de la funcin Cosx abarcan el segmento [-1; 1], es decir 1 Cosx 1;
La funcin est acotada inferior y superiormente;
La funcin toma su valor mnimo y = 1 para todo x
k
= + 2k, donde k es un nmero
entero cualquiera, y el mximo y = 1, para cada x
m
= 2m, donde m es un nmero entero
cualquiera;
La funcin es peridica, de perodo principal igual a 2;
CAPTULO 11. FUNCIONES TRIGONOMTRICAS 571
La funcin es par, puesto que la curva es simtrica con respecto al eje de ordenadas;
La funcin no es montona en todo el dominio, pero la funcin es creciente en cada intervalo
[2k ; 2k], donde k es un nmero entero cualquiera, y decreciente en cada intervalo
[2k; 2k +], donde k es un nmero entero cualquiera;
El punto de interseccin con el eje 0Y tiene las coordenadas (0, 1); hay una innidad de
puntos de interseccin con el eje 0X; cada uno de los puntos
_

2
+k; 0
_
, donde k es un
nmero entero cualquiera, es el punto de interseccin con el eje 0X.
Teniendo presente el carcter peridico de la funcin, se puede construir la grca de la fun-
cin y = Cosx, que se llama cosinusoide. La grca es simtrica con respecto al eje Y, puesto
que Cos(x) = Cosx; adems, Cosx es de perodo 2. Estos dos hechos y la tabla de datos que
se construya, se aplican para trazar la parte de la curva mostrada en la gura. La parte llena se
obtuvo con los datos de la tabla, y el resto se traz por medio de la propiedad Cos(x) = Cosx.
Ahora tenemos una faja de longitud horizontal 2, y podemos usarla como patrn para asegurar
la grca en la extensin que deseemos.
Ejemplo 11.71 Hallar el campo de denicin de la funcin:
y = log
x
Cosx.
Solucin
El campo de denicin de esta funcin abarca slo aquellos valores de x para los cuales se cumplen
simultneamente las siguientes condiciones:
a) x > 0, x ,= 1 (porque la base de los logaritmos tiene que ser positiva y no igual a 1);
b) Cosx > 0 (ya que los nmeros negativos y el cero no tienen logaritmos). Al resolver este sistema
de desigualdades, obtenemos que el recinto de denicin de la funcin considerada lo presenta el
conjunto de nmeros siguiente:
0 < x < 1, 1 < x <

2
,

2
+ 2k < x <

2
+ 2k
donde k=1, 2, 3, ...
Ejemplo 11.72 Encuentre el dominio de la funcin:
a) f(x) =
Cosx
1 Senx
; b) f(x) =
Cosx
1 +Cosx
; c) f(x) =
_
Senx Cosx
Senx +Cosx
.
Solucin
a) Esta funcin est determinada si 1 Senx ,= 0, de donde Senx ,= 1, de donde x ,=

2
,
5
2
, ...
De esta manera, el dominio de la funcin esta dada por todos los valores de x ,=
(4n+1)
2
, donde n
CAPTULO 11. FUNCIONES TRIGONOMTRICAS 572
es un nmero entero.
b) Esta funcin est denida si cumple la siguiente condicin:
1 +Cosx ,= 0 Cosx ,= 1
Esta ecuacin se satisface para todos los reales excepto los x = (2n 1), n Z. Por tanto el
dominio de la funcin est dado por x R(2n 1).
c) Esta funcin est denida si cumple la siguiente condicin:
_
_
_
Senx Cosx
Senx +Cosx
0
Senx +Cosx ,= 0

_
x
_

4
+n;
3
4
+n

x ,=
3
4
+n
Por tanto el dominio de la funcin se obtiene haciendo la interseccin de las dos condiciones. Es
decir x
_

4
+n;
3
4
+n
_
, n Z.
Ejemplo 11.73 Encuentre el rango de las funciones:
a) f(x) =
_
Senx Cosx
Senx +Cosx
; b) f(x) =
Cosx
1 +Cosx
; c) f(x) = log(Cosx + Sen
2
x).
Solucin
Para encontrar el rango de la funcin trigonomtrica, debemos expresar la variable independiente
en funcin de la variable dependiente:
a) y
2
=
Senx Cosx
Senx +Cosx
y
2
=
Tanx 1
Tanx + 1
y
2
Tanx +y
2
= Tanx 1
(y
2
1)Tanx = 1 y
2
x = ArcTan
_
1 +y
2
1 y
2
_
En este caso la solucin general son todos los nmeros reales.
b) y yCosx Cosx = 1 (y + 1)Cosx = y 1 x = ArcCos
_
y 1
y + 1
_
Para que esta expresin tenga sentido, debe cumplirse la siguiente desigualdad:
1
y 1
y + 1
1
_

_
2y
y + 1
0
2
y + 1
0

_
y (; 1) [0; +)
y (1; +)
Para encontrar la solucin general de la desigualdad, hacemos la interseccin de las soluciones
parciales. Es decir la solucin es: y [0; +).
c) 10
y
= Cosx + Sen
2
x 10
y
= Cosx + 1 Cos
2
x
_
Cosx
1
2
_
2

5
4
= 10
y
Cosx
1
2
=
_
5
4
10
y
x = ArcCos
_
1
2
+
_
5
4
10
y
_
Para que esta expresin tenga sentido, debe cumplirse la siguiente desigualdad:
1
1
2
+
_
5
4
10
y
Resolviendo esta inecuacin, obtenemos y ln
5
4
. Por tanto el rango de la funcin est dado por:
y
_
;
5
4

.
CAPTULO 11. FUNCIONES TRIGONOMTRICAS 573
11.12.3. Funcin Tangente
Haciendo uso de las propiedades de la tangente de un ngulo, obtenemos las siguientes carac-
tersticas de la funcin y = Tanx:
El dominio es cualquier x, salvo x
k
=

2
+k, donde k es un nmero entero positivo;
El codominio es (; +);
La funcin no est acotada;
La funcin no toma el valor mximo ni tampoco el mnimo;
La funcin es peridica, de perodo principal igual a ;
La funcin es impar, ya que es simtrica con respecto al origen de coordenadas;
La funcin no es montona en todo el dominio, pero es creciente en cada uno de los
intervalos
_
k

2
; k +

2
_
, donde k es un nmero entero cualquiera;
Los puntos de interseccin con los ejes coordenados son aquellos que tienen las coordenadas
(m, 0), donde m es un nmero entero cualquiera.
Teniendo presente el carcter peridico de la funcin, podemos construir la grca de la funcin
y = Tanx, que se llama curva de tangente. Para trazar la grca de y = Tanx, aplicamos el hecho
de que Tanx es peridica con periodo , y que Tan(x) = Tanx, mostrando que es simtrica
con respecto al origen. La parte de

2
a 0 puede obtenerse con la porcin de

2
a 0, aplicando
la relacin Tan(x) = Tanx. Entonces tenemos una faja de longitud horizontal , y podemos
obtener cualquier extensin de la curva que deseemos usando la seccin de

2
a

2
como patrn.
El estudiante debe observar que las coordenadas de la curva crecen cuando x crece de

2
a

2
.
Las asntotas son x =

2
,
3
2
,
5
2
, ...,
(2n + 1)
2
, ..., puesto que y = Tanx se acerca al innito
cuando x se acerca a cualquiera de estos valores.
CAPTULO 11. FUNCIONES TRIGONOMTRICAS 574
Ejemplo 11.74 Encuentre el dominio de la funcin:
f(x) =
Senx
1 +Tan
2
x
Solucin
Esta funcin est determinada si 1 + Tan
2
x ,= 0, entonces el dominio de la funcin es x R.
Ejemplo 11.75 Exprese el rea de un trapecio issceles de bases a y b como funcin del ngulo
de base a.
Solucin
Por el graco tenemos que
Tan =
h
x
h = xTan
como
2x = a b x =
a b
2
de la ecuacin inicial, tenemos
h =
a b
2
Tan
El rea del trapecio es
A =
a +b
2
h A =
a +b
2

a b
2
Tan A =
a
2
b
2
4
Tan
Ejemplo 11.76 Un observador se encuentra frente a un cuadro colgado de una pared vertical.
El borde inferior del cuadro est situado a una distancia a sobre el nivel de los ojos del observador,
el borde superior, a una distancia b. A qu distancia de la pared debe hallarse el observador para
que el ngulo bajo el que ve el cuadro sea el mximo?
Solucin
Del tringulo pequeo tenemos Tan =
a
x
. Del tringulo grande tenemos
Tan( +) =
b
x
Descomponemos esta identidad
Tan +Tan
1 TanTan
=
b
x
Tan +
a
x
1
a
x
Tan
=
b
x
Tan =
(b a)x
x
2
+ab
(x) = ArcTan
(b a)x
x
2
+ab
CAPTULO 11. FUNCIONES TRIGONOMTRICAS 575
Para obtener x, la distancia del observador a la pared, hacemos (x) = (k):
ArcTan
(b a)x
x
2
+ab
= ArcTan
(b a)k
k
2
+ab
x
x
2
+ab
=
k
k
2
+ab
k =

ab
Por tanto la distancia a la que debe estar el observador de la pared es x =

ab.
11.12.4. Funcin Cotangente
Haciendo uso de las propiedades de la cotangente de un ngulo, obtenemos las siguientes car-
actersticas de la funcin y = Cotx:
El dominio es cualquier x, salvo x
m
= m, donde m es un nmero entero cualquiera;
El codominio es (; +);
La funcin no est acotada;
La funcin no toma el valor mximo ni tampoco el mnimo;
La funcin es peridica, de perodo principal igual a ;
La funcin es impar, ya que es simtrica con respecto al origen de coordenadas;
La funcin no es montona en todo el dominio, pero es decreciente en cada uno de los
intervalos (m; +m), donde m es un nmero entero cualquiera;
Los puntos de interseccin con los ejes coordenados son aquellos que tienen por coordenadas
_

2
m; 0
_
, donde m es un nmero entero cualquiera.
CAPTULO 11. FUNCIONES TRIGONOMTRICAS 576
Tomando en consideracin el carcter peridico de la funcin, podemos construir la grca de
la funcin y = Cotx, que se llama curva de cotangente. La funcin y = Ctgx, es peridica con el
perodo positivo mnimo . El grco de la funcin y = Ctgx, que de modo evidente muestra sus
propiedades, se ofrece a continuacin: La funcin y = Ctgx est denida para toda x R, x ,= n,
n Z. De la frmula Cotx = Tan
_
x

2
_
se desprende que el grco de la funcin y = Ctgx se
obtiene del grco de la funcin y = Tanx desplazndolo a

2
a la derecha por el eje de abscisas y
con simetra con relacin a dicho eje.
Ejemplo 11.77 El radio de la circunferencia circunscrita a un tringulo issceles es igual a
R. Determine la dependencia entre el radio de la circunferencia inscrita y el ngulo en el vrtice
del tringulo.
Solucin
Del tringulo T0P, obtenemos
Tan

2
=
r
x
x = rCot

2
(11.1)
Del tringulo T0S, obtenemos
R
2
= (y +r)
2
+x
2
y +r =
_
R
2
x
2
(11.2)
Del tringulo T0Q, tenemos
Tan =
R +y +r
x
x = (R +y +r)Cot (11.3)
Igualando la primera y tercera ecuaciones, tenemos
rCot

2
= (R +y +r)Cot (11.4)
Reemplazamos (15) en (17) y simplicamos
rCot

2
=
_
R +
_
R
2
x
2
_
Cot rTan =
_
R +
_
R
2
x
2
_
Tan

2
r =
_
R +

R
2
x
2
_
Tan

2
Tan
r =
_
R +
_
R
2
r
2
Cot
2
2
_
Tan

2
Tan
r =
2RTanTan

2
Sec
2

.
Ejemplo 11.78 Hallar el campo de denicin de la funcin
y =
Cotx

Senx Cosx
.
Solucin
Esta funcin es indenida para aquellos valores de x para los cuales Senx Cosx = 0 (el de-
nominador de la fraccin debe ser distinto de cero), y adems, para aquellas x, para las cuales
Senx Cosx < 0 (porque para estos valores de x, el denominador toma valores imaginarios). Por
consiguiente, el recinto de denicin de la funcin consta solamente de aquellos valores de x para
los cuales se cumple la desigualdad Senx Cosx > 0; resolviendo esta desigualdad, hallamos que

4
+ 2k < x <
5
4
+ 2k, k = 0, 1, 2, ...
CAPTULO 11. FUNCIONES TRIGONOMTRICAS 577
Sin embargo, hay que notar que Cotx es indenido para x = n, donde n es un nmero entero
cualquiera. Por eso, todos los valores de x = n, n = 0, 1, 2, ..., tampoco pertenecen al recinto de
denicin de la funcin considerada y deben ser excluidos del sistema de intervalos obtenido anteri-
ormente. De tal modo, en calidad de recinto de denicin de la funcin, obtenemos denitivamente
el siguiente conjunto de nmeros reales:

4
+ 2k < x < + 2k, + 2k < x <
5
4
+ 2k, k = 0, 1, 2, ...
11.13. Tarea
1. Determine el dominio de la funcin:
a) f(x) =
1 Cos(8x 3
Tan2x Cot2x
;
b) f(x) =
Cos
2
x Sen
2
x
4Cos
2
x
;
c) f(x) =
4Cos
2
x
Tan
x
2
Cot
x
2
;
d) f(x) =
1 +Tan2xTanx
Cotx +Tanx
;
e) f(x) =
Cos
x
2
Sen
x
2
+Cos
x
2
;
f ) f(x) =
1 Sen2x
1 +Sen2x
;
g) f(x) =
x

1 Tan
2
x
;
h) f(x) =
1
2 +Cosx
;
i) f(x) =
Cos2x
1 Sen2x
;
j) f(x) =
Secx Cosx
2Senx
;
k) f(x) =
2Sen
2
x
Sec
2
x 1
;
l) f(x) =
1 +Sen2x
Cos2x
;
m) f(x) =
Csc( x)
Cot2x Cotx
;
n) f(x) =
Senx +Tanx
Cosx +Cotx
;
o) f(x) =
1 +Tanx
1 Tanx
;
p) f(x) =
1 +Cos2x
1 +Senx
;
q) f(x) =
_
Senx
1 +Cosx
_
2
;
r) f(x) =
Tan
2
x
1 Senx
;
s) f(x) =
2 senx
2 Cosx
;
t) f(x) =
Cos2x
Cotx 1
;
u) f(x) =
1
Cos
2
x
+Cot
2
x + 1;
v) f(x) = Sen
x

1 x
2
;
w) f(x) = log(Cosx +Sen
2
x);
x) f(x) =
Sen
2
x
1 +Cotx
+
Cos
2
x
1 +Tanx
;
y) f(x) = Cos
x
2
1 x
2
;
z) f(x) = Cotx
1
Senx
.
2. Determine el rango de la funcin:
a) f(x) =
1 Cos(8x 3)
Tan2x Cot2x
;
b) f(x) =
Senx +Tanx
Cosx +Cotx
;
c) f(x) =
Secx Cosx
2Senx
;
d) f(x) =
Sen
2
x
1 +Cotx
+
Cos
2
x
1 +Tanx
;
e) f(x) =
2Senx Sen2x
2Senx +Sen2x
;
f ) f(x) =
Cos
2
x Sen
2
2x
4Cos
2
x
;
g) f(x) =
2Sen
2
x 1
Cosx
+
(2Senx + 1)Cosx
1 +Senx
;
h) f(x) =
Tanx
(1 +Tan
2
x)
2
+
Cotx
(1 +Cot
2
x)
2
;
i) f(x) =
1 +Cosx +Cos2x +Cos3x
Cosx + 2Cos
2
x 1
;
j) f(x) =
2Sen
2
x
Sec
2
x 1
;
CAPTULO 11. FUNCIONES TRIGONOMTRICAS 578
k) f(x) =
1 Sen2x
1 +Sen2x
.
3. Determine la paridad de la funcin:
a) f(x) = Sen(Cos
2
x)Cos(Sen
2
x);
b) f(x) = Tan
3
(Tanx) + 3Tan(Tanx);
c) f(x) = (2 x
2
)Cosx + 2xSenx;
d) f(x) = xSenxCosx +
1
2
Cos
2
x;
e) f(x) = Tan2x +
2
3
Tan
3
2x +
1
5
Tan
5
2x;
f ) f(x) =
1 +Tan2xTanx
Cotx +Tanx
;
g) f(x) =
Sen
2
x
1 +Cotx
+
Cos
2
x
1 +Tanx
;
h) f(x) =
1 Senx
Cosx
Tan
_

4
+
x
2
_
;
i) f(x) = Cotx
2

1
3
Tan
3
x;
j) f(x) =
2Senx Sen2x
2Senx +Sen2x
.
4. Encuentre el perodo mnimo positivo de las funciones:
a) f(x) = Sen2x +Sen
2
3x;
b) f(x) = Sen4x + 5Cos6x;
c) f(x) = 3Sen4x + 2Tan5x;
d) f(x) = Tan(x +Senx);
e) f(x) = Sen
3
x +Cos
3
x;
f ) f(x) = 8Sen
9x
8
+ 2Cos
3x
2
;
g) f(x) =
1 +Senx Cosx
1 Senx Cosx
;
h) f(x) = Sen
3x
4
+Sen
9x
8
;
i) f(x) = SenxTanx;
j) f(x) = (1 +Cosx)Cotx;
k) f(x) = Sen5x +Cos3x;
l) f(x) = Tan
2
x +Cot
2
x;
m) f(x) = 4Sen
2
x 12Senx + 5;
n) f(x) = log(Cosx +Sen
2
x).
5. Graque la funcin:
a) f(x) = Senx +Sen
_
x +
2
3
_
+Sen
_
x +
4
3
_
;
b) f(x) =
2Cos
2
x 1
2Tan
_

4
x
_
Sen
2
_

4
+x
_;
c) f(x) = TanxTan
_

3
x
_
Tan
_

3
+x
_
;
d) f(x) = Tanx 1 + (1 Tanx)Senx +
1
1 +Tan
2
x
;
e) f(x) = Sen
2
_

8
+x
_
Sen
2
_

8
x
_
;
f ) f(x) =
Tanx
(1 +Tan
2
x)
2
+
Cotx
(1 +Cot
2
x)
2
;
g) f(x) =
2Sen
2
x 1
Cosx
+
(2Senx + 1)Cosx
1 +Senx
;
h) f(x) =
Sen
2
x
1 +Cotx
+
Cos
2
x
1 = Tanx
.
6. Graque la funcin:
CAPTULO 11. FUNCIONES TRIGONOMTRICAS 579
a) f(x) =
Cos2x
1 Sen2x
;
b) f(x) =
1 +Tanx
1 Tanx
;
c) f(x) =
Senx +Tanx
Cosx +Cotx
;
d) f(x) =
2Senx Sen2x
2Senx +Sen2x
;
e) f(x) =
1 Sen2x
1 +Sen2x
;
f ) f(x) =
1 +Sen2x
Cos2x
;
g) f(x) =
Csc( x)
Cot2x Cotx
;
h) f(x) =
_
Senx
1 +Cosx
_
2
;
i) f(x) =
2Sen
2
x
Sec
2
x 1
;
j) f(x) =
4Cos
2
x
Tan
x
2
Cot
x
2
;
k) f(x) =
Cos
2
x Sen
2
x
4Cos
2
x
;
l) f(x) =
1
2 +Cosx
;
m) f(x) = Tan3x +
1
Cos3x
;
n) f(x) =
Secx Cosx
2Senx
;
o) f(x) =
Tan2xTanx
Tan2x Tanx
;
p) f(x) =
1 +Cos2x
1 +Senx
;
q) f(x) =
2 Senx
2 Cosx
.
7. Se tiene una pieza de metal de 20 metros de largo y 6 metros de ancho, con la cual va a
construirse un abrevadero. A qu ngulo deben juntarse los lados para que el volumen del
abrevadero sea el mximo posible?
8. Los dos lados y la base de un trapecio issceles tienen 5 pulg de largo cada uno. A qu
ngulo deben juntarse los lados con el techo horizontal para maximizar el rea del trapecio?
9. Pruebe que, de todos los tringulos issceles cuyos lados iguales tienen una longitud es-
pecicada, el tringulo de mayor rea es el tringulo rectngulo.
10. Halle el largo del tubo de mayor longitud que puede transportarse horizontalmente por
una esquina que une dos pasillos que tienen 2

2 pies de ancho.
11. Hay que hacer una artesa con un fondo plano y lados igualmente inclinados doblando una
pieza de hoja metlica de ancho x:
a) Si los dos lados y el fondo tienen, cada uno, un ancho de igual a x/3, cul es el ngulo
de los lados que da la seccin transversal de mayor rea?
b) Si el ngulo entre el lado y el fondo es un ngulo dado , 0

2
, cul es el ancho
que debe tener el fondo?
12. Una lmpara de altura ajustable cuelga directamente encima del centro de una mesa cir-
cular que tiene 8 pies de dimetro. La iluminacin en el borde de la mesa es directamente
proporcional al coseno del ngulo e inversamente proporcional al cuadrado de la distancia
d, donde y d son como se muestra en la gura. Qu tan cerca de la mesa debe situarse la
lmpara para maximizar la iluminacin en el borde de aquella?
13. Dos pozos petrolferos estn, respectivamente, a a y a b millas mar adentro. Un bote de
motor que viaja a una velocidad constante s transporta trabajadores desde el primer pozo a
CAPTULO 11. FUNCIONES TRIGONOMTRICAS 580
la orilla y luego prosigue hacia el segundo pozo. Demuestre que el tiempo total de viaje es
mnimo si el ngulo entre la trayectoria de partida del bote y la orilla es igual al ngulo
entre la orilla y la trayectoria de salida del bote.
11.14. Expresiones trigonometricas inversas
Surge con frecuencia el problema en el que se requiere hallar, para cualquier nmero real k, tal
ngulo que el seno de ste sea igual al nmero . Si k > 1 y si k < 1, entonces este problema
no tiene solucin, pues, por denicin de seno de un ngulo, no existe tal ngulo cuyo seno sea
mayor que 1, o menor que -1. En cambio, si k [1; 1], se puede mostrar que existe una innidad
de ngulos tales, que el seno de cada uno de ellos es igual al nmero k. En efecto, la recta y = k
corta al circulo unitario o bien en dos puntos, o bien en un solo punto.
Mas, segn lo expuesto anteriormente, para todo punto de este tipo en el circulo unitario existe
un ngulo tal, que el seno de dicho ngulo es igual a la ordenada del punto citado, es decir, igual
a k.
Ahora, de acuerdo con la propiedad del seno tenemos Sen = Sen( + 2n) para cualquier
ngulo y para todo nmero entero n. Por eso, el seno del ngulo + 2n es igual al nmero k,
cualquiera que sea el nmero entero n.
Denicin 11.17 Angulo principal
El ngulo cuyo seno es igual al nmero k y que forma parte del segmento
_

2
;

2
_
, recibe el nombre
de ngulo principal y se designa ArcSenk (se lee: arco seno del nmero k).
De este modo, por denicin, ArcSenk es el ngulo que satisface simultneamente dos condi-
ciones:
_

2
ArcSenk

2
Sen(ArcSenk) = k
Es fcil ver que para cualquier nmero k [1; 1] el arco seno de este nmero existe y es, adems,
nico. Para todo nmero k (; 1) (1; +) el arco seno de l no existe.
Ejemplo 11.79 Simplique las expresiones:
a) ArcSen
_
Sen
_

2
3
__
; b) ArcSen
_
Sen
7
6
_
; c) ArcSen
_
Sen
11
6
_
.
CAPTULO 11. FUNCIONES TRIGONOMTRICAS 581
Solucin
a) ArcSen
_
Sen
2
3
_
= ArcSen
_
Sen
2
3
_
=
2
3
b) ArcSen
_
Sen
_
+

6
__
= ArcSen
_
Sen

6
_
=

6
c) ArcSen
_
Sen
_
+
5
6
__
= ArcSen
_
Sen
5
6
_
=
5
6
A continuacin se indican algunas propiedades del arco seno de un nmero, que se desprenden
de su denicin.
Teorema 11.40 Para todo nmero k, mayor que 1, y tambin para todo nmero k menor que
-1, la notacin ArcSenk est privada de sentido.
Teorema 11.41 Para cualquier nmero k [1; 1] se verica la siguiente desigualdad doble

2
ArcSenk

2
Teorema 11.42 Para todo nmero k [1; 1] es vlida la igualdad
Sen(ArcSenk) = k
Teorema 11.43 Para todo nmero k
_

2
;

2
_
es vlida la igualdad
ArcSen(Sen) =
Teorema 11.44 Para cualquier nmero k [1; 1] se verica la igualdad
ArcSen(k) = ArcSenk.
Demostracin
Efectivamente, por denicin, ArcSenk = , con la particularidad de que Sen = k y k
_

2
;

2

,
ArcSen(k) = , con la particularidad de que Sen = k y
_

2
;

2

. De aqu se hace evidente


que = , es decir
ArcSen(k) = ArcSenk.
Surge con frecuencia el problema en el que se requiere hallar, para cualquier nmero real k, tal
ngulo que el coseno de ste es igual al nmero k.
Notemos aqu mismo que si k > 1, y tambin si k < 1, este problema no tiene solucin, puesto
que, por denicin de coseno de un ngulo, no existe un ngulo, cuyo coseno sea mayor que 1, o
menor que -1. En cambio, si k [1; 1], podernos mostrar que existe una innidad de tales ngulos
que el coseno de cada uno de ellos es igual al nmero k.
CAPTULO 11. FUNCIONES TRIGONOMTRICAS 582
En efecto, la recta x = k interseca, para k [1; 1], del crculo unitario o bien en dos puntos,
o bien en un punto. Mas, segn lo expuesto anteriormente, para cada tal punto existe un ngulo
tal, que el coseno de l es igual a la abscisa del punto citado, es decir, igual a k. Ahora, de acuerdo
con la propiedad del coseno Cos = Cos( + 2n) para cualquier ngulo y cualquier nmero
n entero. Por eso, para cualquier nmero entero n el coseno del ngulo +2n es igual al nmero k.
Denicin 11.18 Angulo principal
El ngulo cuyo coseno es igual al nmero k y que forma parte del segmento [0; ] recibe el nombre
de ngulo principal y se designa ArcCosk.
De este modo, por denicin, ArcCosk es un ngulo que satisface simultneamente dos condi-
ciones:
_
0 ArcCosk
Cos(ArcCosk) = k
Es fcil ver que para cualquier nmero k [1; 1] el arco coseno de este nmero existe y es, adems,
nico. Para todo nmero k (; 1) (1; +) el arco coseno de ste no existe.
Ejemplo 11.80 Simplique las expresiones:
a) ArcCos
_
Cos
7
4
_
; b) ArcCos
_
Cos
7
6
_
; c) ArcCos
_
Cos
_

7
6
__
.
Solucin
a) ArcCos
_
Cos
_
+
3
4
__
= ArcCos
_
Cos
3
4
_
= ArcCos (Cos34) =
3
4
=

4
.
b) ArcCos
_
Cos
_
+

6
__
= ArcCos
_
Cos

6
_
= ArcCos
_
Cos

6
_
=

6
=
5
6
.
c) ArcCos
_
Cos
7
6
_
= ArcCos
_
Cos
_
+

6
__
= ArcCos
_
Cos

6
_
= ArcCos
_
Cos

6
_
=

6
=
5
6
Teorema 11.45 Para cualquier nmero k inferior a -1 y tambin para cualquier nmero k
superior a 1, la notacin ArcCosk est privada de sentido.
Teorema 11.46 Para cualquier nmero k [1; 1] es valida la desigual doble
0 ArcCosk
Teorema 11.47 Para cualquier nmero k [1; 1] es valida la igualdad
Cos(ArcCosk) = k
Teorema 11.48 Para cualquier ngulo [0; ] es valida la igualdad
ArcCos(Cos) =
Teorema 11.49 Para todo nmero k [1; 1] se verica la igualdad
ArcCos(k) = ArcCosk
Demostracin
Por denicin, ArcCosk = , con la particularidad de que Cos = k y [0; ], ArcCos(k) = ,
con la particularidad de que Cos = k y [0; ]. De aqu se ve que = , es decir
ArcCos(k) = ArcCosk.
CAPTULO 11. FUNCIONES TRIGONOMTRICAS 583
Surge con frecuencia el problema en el que se requiere hallar, para cualquier nmero real k, un
ngulo tal que su tangente sea igual al nmero k.
Teorema 11.50 Existe una innidad de ngulos tales que la tangente de cada uno de ellos es
igual a k.
Demostracin
Efectivamente, en la gura se ve que la recta que pasa
por el origen de coordenadas y el punto S(1, k), dispuesto
en la lnea de tangente, interseca la circunferencia uni-
taria en dos puntos
_
1

1 +k
2
;
k

1 +k
2
_
y
_

1 +k
2
;
k

1 +k
2
_
.
Pero, segn lo indicado anteriormente, para cada uno
de estos puntos de la circunferencia unitaria existe un
ngulo tal que la tangente de dicho ngulo es igual
a la razn de la ordenada de este punto a la abscisa del
mismo, es decir, a k. Ahora, de acuerdo con la propiedad
de la tangente tenemos Tan = Tan(+r), para cualquier ngulo tal que ,=

2
+m, m Z,
y para todo numero entero r. Por eso, para todo numero entero r la tangente del ngulo +r es
igual al numero k.
Teorema 11.51 El ngulo cuya tangente es igual al nmero k y que pertenece al intervalo
_

2
;

2
_
recibe el nombre de ngulo principal y se designa ArcTank.
De este modo, por denicin, ArcTank es un ngulo que satisface simultneamente dos condi-
ciones
_

2
< ArcTank <

2
Tan(ArcTank) = k
El arco tangente de este nmero k existe y adems es nico.
Ejemplo 11.81 Simplique las expresiones:
a) ArcTan
_
Tan
7
4
_
; b) ArcTan
_
Tan
_

7
6
__
; c) ArcTan
_
Tan
_

7
4
__
.
Solucin
a) ArcTan
_
Tan
_
+
3
4
__
= ArcTan
_
Tan
3
4
_
=
3
4
.
b) ArcTan
_
Tan
2
3
_
= ArcTan
_
Tan
2
3
_
=
2
3
.
c) ArcTan
_
Tan
_

4
2
__
= ArcTan
_
Tan

4
_
=

4
Teorema 11.52 Para cualquier nmero real k se verica la desigualdad doble

2
< ArcTan <

2
Teorema 11.53 Para cualquier nmero real k se verica la igualdad
Tan(ArcTank) = k
CAPTULO 11. FUNCIONES TRIGONOMTRICAS 584
Teorema 11.54 Para cualquier ngulo
_

2
;

2
_
se verica la igualdad
ArcTan(Tan) =
Teorema 11.55 Para cualquier nmero real k se verica la igualdad
ArcTan(k) = ArcTank
Demostracin
Por denicin ArcTank = , con la particularidad de que Tan = k y
_

2
;

2

, ArcTan(k) =
, con la particularidad de que Tan = k y
_

2
;

2

, de aqu es evidente que = , es


decir
ArcTan(k) = ArcTank.
Surge frecuentemente el problema en el que se requiere hallar, para cualquier nmero real k,
un ngulo tal que la cotangente de l es igual al nmero k. Se puede mostrar que existe una
innidad de ngulos tales que la cotangente de cada uno de ellos es igual al nmero k. Es fcil ver
que una recta que pasa por el origen de coordenadas y el punto P(k, 1) dispuesto en la lnea de
cotangentes interseca la circunferencia unidad en dos puntos
_
k

1+k
2
;
1

1+k
2
_
y
_
k

1+k
2
;
1

1+k
2
_
,
Pero segn lo indicado anteriormente, para cada pun-
to de esta ndole existe un ngulo tal que la cotangente
de l es igual a la razn de la abscisa de dicho punto a
su ordenada, es decir, igual a k. Luego, de acuerdo con
la propiedad de la cotangente, Cot = Cot(+n) para
cualquier ngulo tal que ,= m, m Z, y para cada
nmero entero n. Por eso, para cualquier nmero entero
n la cotangente del ngulo + n es igual al nmero
k.
Un ngulo cuya cotangente es igual al nmero k y que
pertenece al intervalo (0; ) recibe el nombre de ngulo
principal y se designa ArcCotk. De este modo, ArcCotk
es, por denicin, un ngulo que satisface simultneamente dos condiciones:
_
0 < ArcCotk <
Cot(ArcCotk) = k
Es fcil ver que para todo nmero real k el arco cotangente de dicho nmero existe y es, adems,
nico.
Ejemplo 11.82 Simplique las expresiones:
a) ArcCot
_
Cot
5
4
_
; b) ArcCot
_
Cot
7
4
_
; c) ArcCot
_
Cot
25
6
_
.
Solucin
a) ArcCot
_
Cot
_
+

4
__
= Arccot
_
Cot

4
_
=

4
.
b) ArcCot
_
Cot
_
+
3
4
__
= ArcCot
_
Cot
3
4
_
=
3
4
.
c) ArcCot
_
Cot
_

6
+ 4
__
= ArcCot
_
Cot

6
_
=

6
CAPTULO 11. FUNCIONES TRIGONOMTRICAS 585
Teorema 11.56 Para cualquier nmero real k se verica la desigualdad doble
0 < ArcCotk <
Teorema 11.57 Para cualquier nmero real k se verica la igualdad
Cot(ArcCotk) = k
Teorema 11.58 Para todo ngulo (0, ) se verica la igualdad
ArcCot(Cot) =
Teorema 11.59 Para cualquier nmero real k se verica la igualdad
ArcCot(k) = ArcCotk
Demostracin
Por denicin, tenemos ArcCotk = , con la particularidad de que Cot = k y (0; ),
ArcCot(k) = , con la particularidad de que Cot = k y (0; ). De aqu se deduce que
= , es decir,
ArcCot(k) = ArcCotk
El concepto de funciones inversas se puede aplicar a las funciones trigonomtricas. La funcin
peridica no es inversible, en particular, tampoco son inversibles las funciones trigonomtricas.
Pero sobre ciertos subconjuntos de su campo de denicin estas funciones son inversibles.
Ejemplo 11.83 Determine el valor de la siguiente expresin:
a)
4ArcTan(1) +ArcTan1
5
2
Sen
_
ArcSen
1
2
_
+Cos[ ArcTan(1)]
b)
Cos
_
4ArcSen
1
2
3ArcTan(

3)
_
Tan
_

11
2
2ArcCos

3
2
_ .
Solucin
a) Transformando y simplicando, obtenemos
4ArcTan1 +ArcTan1
5
2
1
2
+Cos[ +ArcTan1]
=
3ArcTan1
5
2
1
2
+Cos
_
+

4
_
=
3

4

5
2
1
2
Cos

4
=
3

4

5
2
1
2

2
2
=
13(1 +

2)
2
.
CAPTULO 11. FUNCIONES TRIGONOMTRICAS 586
b) Transformando y simplicando, obtenemos
Cos
_
4

6
+ 3ArcTan

3
_
Tan
_

11
2
2

6
_ =
Cos
_
4

6
+ 3

3
_
Tan
_

11
2
2

6
_
=
Cos
_
2
3
+
_
Tan
_

11
2


3
_
=
Cos
2
3
Tan
5
6
=
Cos
_


3
_
Tan
_


6
_
=
Cos

3
Tan

6
=

3
2
.
11.15. Tarea
1. Simplique las expresiones:
a) Cot
_
3
2
5ArcCot(1)
_
;
b) Cos
_
2 2ArcCos
_

3
2
__
;
c) Cot
_
3ArcSen(1) +ArcCos

2
2
_
;
d) Tan
_

2
ArcCos
_

1
3
__
;
e) Sen
_

149
2
+ 2ArcCot(

3)
_
;
f ) Cot
_
2ArcSen
_

2
2
_
+ArcCos
1
2
_
;
g)
ArcSen
_
Sen
_

2
__
+
ArcTan
_
Tan

4
_ ;
h) Sen
_
ArcCos
2
3
+ArcCos
1
3
_
;
i) Cos
_
ArcCos
5
13
ArcSen
3
5
_
;
j) Tan
_
3
2
+ 4ArcTan

3
3
_
;
k) Sen
_

137
2
2ArcCos
_

1
2
__
.
2. Simplique las expresiones:
a)
ArcCos(Cos) ArcSen
_
Sen

2
_
2ArcCot
_
Cot
3
4
_
+ArcCot
_
Tan
_

4
__;
b)
Sen
_
2ArcCot
_

3
3
__
+Cos(5ArcCot0)

3Tan
_
10 +

4
_ ;
c)
Sen
_
15
2
ArcCot3
_
+Cos
_
2ArcCot

3
2
_
Tan(115 ArcCot1)
;
d) Sen
_
ArcSen
1
2
_
+Cos( arcTan(1));
e) ArcCos
_

1
2
_
+ArcSen
1
2
+ArcTan

3;
CAPTULO 11. FUNCIONES TRIGONOMTRICAS 587
f ) 2ArcCos
_

2
2
_
+ArcCot(1) ;
g) 3ArcSen
_

1
2
_
2ArcCos
_

3
2
_
+

2
.
3. Simplique la expresin:
a) Sen
_
1
2
ArcCot
_

3
4
__
;
b) ArcCos
_
Cos
_

17
5
__
;
c) Tan
_
5ArcTan

3
3

1
4
ArcSen

3
2
_
;
d) Sen
_
3ArcTan

3 + 2ArcCos
1
2
_
;
e) Cos
_
3ArcSen

3
2
+ArcCos
_

1
2
_
_
;
f ) ArcCos
_
Cos

4
_
;
g) ArcTan(Tan0, 3);
h) ArcSen
_
Sen
7
3
_
;
i) ArcCos
_
Cos
3
4
_
;
j) ArcTan
_
Tan
2
3
_
;
k) Sen
_
1
2
ArcSen
_

2
3
__
;
l) Tan
_
1
2
ArcSen
5
13
_
;
m) Cot
_
1
2
ArcCos
_

4
7
__
;
n) Sen
_
ArcTan
8
15
ArcSen
8
17
_
;
o) Sen
_
2
_
ArcSen

5
3
ArcCos

5
3
__
.
4. Simplique la expresin:
a) ArcSen
_
Sen
33
7
_
+ArcCos
_
Cos
46
7
_
;
b) ArcTan
_
Tan
13
8
_
+ArcCot
_
Cot
_

19
8
__
.
5. Simplicar la expresin
2ArcSen
_

3
2
_
+ArcCot(1) +ArcCos
1

2
+
1
2
ArcCos(1)
6. Demuestre la expresin
ArcCos
1
2
+ArcCos
_

1
7
_
= ArcCos
_

13
14
_
7. Simplique la expresin:
a) Sen(2ArcSenx);
b) Tan(2ArcTanx);
c) Cos(2ArcTanx);
d) Sen(2ArcCotx);
e) Cos(2ArcCotx).
8. Simplique la expresin:
CAPTULO 11. FUNCIONES TRIGONOMTRICAS 588
a) Cos(ArcCosx +ArcCosy);
b) Sen(ArcCosx +ArcSeny);
c) Tan(ArcTanx +ArcTany);
d) Tan(ArcSenx +ArcSeny).
9. Compruebe las identidades:
a) ArcTan
2
3
+ArcTan
1
5
=

4
;
b) ArcCot
1
9
+ArcCot
4
5
=
3
4
;
c) ArcCot
1
7
+ 2ArcCot
1
3
=
5
4
;
d) ArcSen
4
5
ArcCos
2

5
= ArcTan
1
2
.
10. Compruebe las identidades:
a) ArcSen
7
25
+
1
2
ArcCos
7
25
= ArcCos
3
5
;
b) ArcTan

2
2
+ArcSen

2
2
= ArcTan(3 + 2

2);
c) ArcTan
1
3
+ArcTan
1
4
+ArcTan
2
9
=

4
;
d) ArcSen
4
5
+ArcSen
5
13
+ArcSen
16
65
=

2
.
11.16. Ecuaciones trigonomtricas inversas
Sea dada la ecuacin elemental ArcSenx = b. El dominio de la funcin y = ArcSenx es el
intervalo [-1; 1]. En el intervalo [-1; 1] la funcin y = ArcSenx es estrictamente creciente y el
codominio est dado por el intervalo
_

2
;

2
_
. Por consiguiente, la ecuacin ArcSenx = b no tiene
soluciones, cuando cada b es tal, que b <

2
b >

2
; si en cambio, b es tal, que

2
b

2
,
entonces la ecuacin ArcSenx = b tendr una raz nica que se representar por x
1
. Puesto que
x
1
es la raz de la ecuacin, x
1
[1; 1] y b
_

2
;

2
_
, es vlida la siguiente equivalencia:
ArcSenx
1
= b Sen(ArcSenx
1
) = Senb x
1
= Senb
De esta forma, la ecuacin ArcSenx = b tiene la raz nica x
1
= Senb, cuando cada b es tal, que

2
b

2
, y no tiene races, cuando cada b es tal, que b <

2
b >

2
.
b <

2
b =

2
< b <

2
b =

2
b >

2
ArcSenx = b Sin solucin x
1
= 1 x
1
= Senb x
1
= 1 Sin solucin
Sea dada la ecuacin elemental ArcCosx = b. El dominio de la funcin y = ArcCosx es el
intervalo [-1; 1]. En el intervalo [-1; 1] la funcin y = ArcCosx es estrictamente decreciente y el
codominio est dado por el intervalo [0; ]. Por consiguiente, para cada b tal que b < 0 b > , la
ecuacin ArcCosx = b no tiene races; si, en cambio, b es tal, que 0 b , entonces la ecuacin
tiene una raz nica que se representar con x
1
. Por cuanto x
1
es la raz de la ecuacin, x
1
[1; 1]
y b [0; ], entonces es vlida la siguiente equivalencia:
ArcCosx
1
= b Cos(ArcCosx
1
) = Cosb x
1
= Cosb
CAPTULO 11. FUNCIONES TRIGONOMTRICAS 589
De esta forma, la ecuacin ArcCosx = b tiene la raz nica x
1
= Cosb, cuando cada b es tal, que
0 b , y no tiene races, cuando cada b es tal, que b < 0 y b > .
b < 0 b = 0 0 < b < b = b >
ArcCosx = b Sin solucin x
1
= 1 x
1
= Cosb x
1
= 1 Sin solucin
Sea dada la ecuacin elemental ArcTanx = b. El dominio de la funcin y = ArcTanx es el
conjunto de todos los nmeros reales, es decir, (; +). En este conjunto (; +) la funcin
y = ArcTanx es estrictamente creciente y el codominio est representado por el intervalo

2
;

2
.
Por consiguiente, la ecuacin ArcTanx = b no tiene soluciones, cuando cada b es tal, que b

2
b

2
; si, en cambio, b es tal, que

2
< b <

2
, la ecuacin ArcTanx = b tendr una raz nica
que se representar con x
1
.Por cuanto x
1
es la raz de la ecuacin ArcTanx = b, x
1
(; +)
y b
_

2
;

2
_
, entonces es vlida la siguiente equivalencia:
ArcTanx
1
= b Tan(ArcTanx
1
) = Tanb x
1
= Tanb
As pues, la ecuacin ArcTanx = b tiene la nica raz x
1
= Tanb, cuando cada b es tal, que

2
< b <

2
; y para cada b tal, que b

2
b

2
la ecuacin ArcTanx = b no tiene races.
b

2
< b <

2
b

2
i
ArcTanx = b Sin solucin x
1
= Tanb Sin solucin
Sea dada la ecuacin elemental ArcCotx = b. El dominio de la funcin y = ArcCotx es el conjunto
de todos los nmeros reales, es decir, (; +). En este conjunto (; +) la funcin y =
ArcCotx es estrictamente decreciente y el codominio est representado por el intervalo (0; ). Por
consiguiente, la ecuacin ArcCotx = b no tiene soluciones, cuando cada b es tal, que b 0
b ; en cambio, si b es tal, que 0 < b < , la ecuacin ArcCotx = b tendr una raz nica que
se representar con x
1
. Por cuanto x
1
es la raz de la ecuacin ArcCotx = b, x
1
(; +) y
b (0; ), entonces es vlida la siguiente equivalencia:
ArcCotx
1
= b Cot(ArcCotx
1
) = Cotb x
1
= cotb
As pues, la ecuacin ArcCotx = b tiene la nica raz x
1
= Cotb, cuando cada b es tal, que
0 < b < , y para cada b tal, que b 0 b , la ecuacin ArcCotx = b no tiene races.
b 0 0 < b < b
ArcCotx = b Sin solucin x
1
= Cotb Sin solucin
11.17. Tarea
1. Demuestre que si 1 < x < 1, entonces
ArcSenx = ArcTan
x

1 x
2
2. Demuestre las identidades:
CAPTULO 11. FUNCIONES TRIGONOMTRICAS 590
a) ArcTanx = ArcSen
x

1 +x
2
;
b) ArcSenx =
_
ArcCos

1 x
2
si 0 x 1
ArcCos

1 x
2
si 1 x 0
;
c) ArcCosx =
_
ArcSen

1 x
2
si 0 x 1
ArcSen

1 x
2
si 1 x 0
;
d) ArcTanx =
_
ArcCos
1

1+x
2
si x > 0
ArcCos
1

1+x
2
si x 0
;
e) ArcCosx =
_
ArcTan

1x
2
x
si 0 < x 1
+ArcTan

1x
2
x
si 1 x < 0
;
f ) ArcTanx =
_
ArcCot
1
x
si x > 0
ArcCot
1
x
si x < 0
;
g) ArcSenx =
_
ArcCot

1x
2
x
si 0 < x 1
ArcCot

1x
2
x
si 1 x < 0
;
h) ArcCotx =
_
ArcSen
1

1+x
2
si x 0
ArcSen
1

1+x
2
si x < 0
;
i) ArcCotx =
_
ArcTan
1
x
si x < 0
+ArcTan
1
x
si x < 0
;
j) 2ArcCos
_
1 +x
2
= ArcCosx;
k)
1
2
ArcCos(2x
2
1) = ArcCosx si x 0.
3. Demostrar la identidad
ArcSen(x) = ArcSenx, si 1 x 1
4. Demostrar la identidad
ArcCos(x) = ArcCosx, si 1 x 1
5. Demostrar la identidad
ArcSenx +ArcCosx =

2
, siendo 1 x 1
6. Demostrar las identidades:
a) ArcTan(x) = ArcTanx;
b) ArcCot(x) = ArcCotx;
c) ArcTanx +ArcCotx =

2
.
7. Resolver las ecuaciones:
a) Sen
_
1
5
ArcCosx
_
= 1;
b) ArcCos

3x =

2
ArcCosx;
CAPTULO 11. FUNCIONES TRIGONOMTRICAS 591
c) ArcSen
3x
5
+ArcSen
4x
5
= ArcSenx;
d) ArcSen(1 x) 2ArcSenx =

2
;
e) ArcSenx = ;
f ) ArcSen
1

x
ArcSen

1 x =

2
;
g) ArcCotx = ArcCosx;
h) ArcSenx ArcCosx = ArcCos

3
2
;
i) ArcCosx ArcSenx =

6
;
j) ArcSen2x +ArcSenx =

3
;
k) 3ArcSenx +x = 0;
l) 4ArcTan(x
2
3x + 3) =

4
;
m) ArcTan3x ArcCot3x =

4
;
n) 2ArcSen
2
x 5ArcSenx + 2 = 0;
o) 4ArcTanx 6ArcCotx = ;
p) 2ArcSenx = ArcCos2x;
q) ArcSen
x
2
+ArcCos
_
x +

3
3
_
=

6
;
r) ArcCosx = ArcTanx;
s) ArcSen
2
3

x
ArcSen

1 x = ArcSen
1
3
;
t) 3ArcCosx x

2
= 0;
u) ArcSen
_
Tan

4
_
ArcSen
_
3
x


6
= 0;
v) ArcTanx +ArcTan
1
y
= ArcTan3.
11.18. Funciones trigonomtricas inversas
A menudo surge una situacin en que la forma de la funcin inversa no es del todo simple. Para
cualquiera de las funciones trigonomtricas fundamentales, se pueden escoger muchos dominios,
cada uno de los cuales se aplique biunvocamente por la funcin trigonomtrica correspondiente
sobre el correspondiente codominio. Adems, si una funcin trigonomtrica fundamental se anal-
iza en su dominio, especialmente elegido, como funcin inversa de ella intervendr una funcin
trigonomtrica.
Es evidente que cualquier funcin trigonomtrica fundamental inversa aplica biunvocamente
su dominio sobre su codominio. Por eso, cada una de estas funciones cuenta con su funcin inversa,
que es una funcin trigonomtrica fundamental correspondiente, pero analizada solamente en el
dominio correspondiente.
CAPTULO 11. FUNCIONES TRIGONOMTRICAS 592
11.18.1. Funcin arco seno
Vamos a partir de la grca de la funcin y = Senx. A cada valor del ngulo x corresponde un
valor determinado y nico del seno de este ngulo; en la interpretacin geomtrica esto signica
que la perpendicular trazada desde cualquier punto del eje 0X corta a la curva de la funcin slo
en un punto. Empero, se podra decir lo contrario, es decir, que a cada valor admisible del seno,
es decir, al nmero y, corresponde un valor nico del ngulo x? Evidentemente que no, puesto que
sabemos que al valor dado del seno corresponde un conjunto innito de ngulos.
Geomtricamente estos ngulos los obtenemos si trazamos una recta paralela al eje 0X a una
distancia d y sobre el eje 0X. Esta paralela corta a la sinusoide innitas veces, puesto que la
grca puede continuarse indenidamente a ambos lados. De este modo, por ahora no se puede
establecer la correspondencia inversa entre los valores del seno y los valores de x, de manera que esta
correspondencia sea unvoca. No obstante, si el ngulo x se considera variable slo en el segmento
_

2
;

2
_
, a cada valor de y le corresponder un nico valor de x. En otras palabras, existe una
funcin inversa de simple valuacin que se designa del modo siguiente: Si
y = Senx,

2
x

2
tendremos que
x = ArcSeny, [y[ 1.
La ltima igualdad se lee as: x es un ngulo medido en radianes, cuyo seno es igual a y, o abrevi-
adamente: x es igual al arco seno de y.
El argumento de la funcin inversa tambin se admite en des-
ignar por la letra x, y la funcin, por la letra y, de manera que en
lugar de x = ArcSeny en adelante escribiremos:
y = ArcSenx
donde
[x[ 1,

2
x

2
.
La propiedad de que las funciones Senx y ArcSenx sean inversas se escribe como:
_
Sen(ArcSenx) = x, si [x[ 1;
ArcSen(Senx) = x, si [x[

2
.
Es decir, los signos de las operaciones ArcSen, y Sen si se suceden una a otra, se anulan mutua-
mente y queda el numero x, con el cual se realizaron sucesivamente estas dos operaciones.
Para trazar la grca de la funcin y = Senx puede servirse de que de la correlacin y =
ArcSenx se deduce x = Seny por denicin de la funcin ArcSen. Si construimos la parte de la
sinusoide x = Seny, que corresponde a la variacin del argumento y en el segmento
_

2
;

2
_
, sta
es precisamente la grca de la funcin y = ArcSenx. Toda la sinusoide x = Seny es la grca de
la funcin de valuacin mltiple y = ArcSenx.
Sealemos las propiedades de la funcin ArcSenx, que se maniestan mediante la grca:
La funcin est denida solamente en el segmento [-1; 1];
CAPTULO 11. FUNCIONES TRIGONOMTRICAS 593
El conjunto de todos los valores de la funcin compone el segmento
_

2
;

2
_
, es decir

2
ArcSenx

2
;
La funcin est acotada inferior y superiormente;
La funcin toma su valor mnimo y =

2
cuando x = 1, y el valor mximo y =

2
, cuando
x = 1;
La funcin no es peridica;
La funcin es impar, es decir es simtrica con respecto al origen de coordenadas;
La funcin es creciente en todo el dominio;
El nico punto de interseccin con los ejes coordenados es el (0, 0).
Ejemplo 11.84 Hallar el recinto de denicin de la funcin
y =
_
Cos(Cosx) +ArcSen
1 +x
2
2x
Solucin
Examinemos por separado cada uno de los sumandos. Al recinto de denicin de esta funcin
pueden pertenecer slo aquellos valores del argumento para los cuales el primer sumando toma los
valores reales, es decir, aquellos valores de x para los cuales la expresin subradical Cos(Cosx) no
es negativa: Cos(Cosx) 0. Es fcil convencerse de que esta desigualdad es vlida para todos los
valores reales de x.
Vamos a referirnos al segundo sumando. Segn la denicin, la expresin ArcSena tiene sentido
slo para [a[ 1; es decir, al recinto de denicin de la funcin original pertenecen solamente
aquellos valores de x para los cuales

1 +x
2
2x

1. Sin embargo, se demuestra directamente que


para todos los valores reales no nulos de x es vlida la desigualdad

1 +x
2
2x

1, con la cual el
signo de igualdad se obtiene slo para x = 1 y x = 1. Por consiguiente, el recinto de denicin
de la funcin consta de dos puntos: x = 1 y x = 1.
Ejemplo 11.85 Encuentre el dominio de la funcin:
a) f(x) = ArcSenx
2
; b) f(x) = ArcSen
_
1 x
2
; c) y = ArcSen
x
2
1
x
.
Solucin
a) Para que esta funcin est denida, debe cumplir las siguientes condiciones:
1 [x[ 1
_
[x[ 1
[x[ 1

_
[x[ 1
1 x 1
Por tanto el dominio de la funcin, esta dado por la interseccin de ambos intervalos. Es decir
x [1; 1].
b) Para que esta funcin est denida, debe cumplir la siguiente condicin:
1
_
1 x
2
1
_

1 x
2
1

1 x
2
1

_
1 x
2
0
1 x
2
0, 1 x
2
1
CAPTULO 11. FUNCIONES TRIGONOMTRICAS 594
_
(x 1)(x + 1) 0
(x 1)(x + 1) 0, x 0

_
1 x 1
1 x 1, x 0
Por tanto el dominio de la funcin, esta dado por la interseccin de ambos intervalos. Es decir
x [0; 1].
c) Para que esta funcin est denida, debe cumplir la siguiente condicin:
1
x
2
1
x
1
_

_
x
2
1
x
1
x
2
1
x
0

_
x
2
+x 1
x
0
x
2
x 1
x
0
_

_
x
_

1 +

5
2
; 0
_

5
2
; +
_
x
_
;
1

5
2
_

_
0;
1 +

5
2
_
Por tanto el dominio de la funcin, esta dado por la interseccin de ambos intervalos. Es decir
x
_

1 +

5
2
;
1

5
2
_

5
2
;
1 +

5
2
_
Ejemplo 11.86 Encuentre el rango de la funcin
f(x) = Sen(3ArcSenx)
Solucin
ArcSeny = 3ArcSenx
_

_
Sen
_
ArcSeny
3
_
Sen
_
ArcSeny
3
+

3
_
Cos
_
ArcSeny
3
+

6
_
En este caso tenemos tres opciones para analizar:

2

ArcSeny
3


2

_

_
ArcSeny
3

2
ArcSeny
3


2

_
ArcSeny
3
2
ArcSeny
3
2
Estas inecuaciones estn fuera del rango de denicin.

2

ArcSeny
3
+

3


2

_

_
ArcSeny
3
+

3

2
ArcSeny
3
+

3


2

_
_
_
ArcSeny
5
2
ArcSeny

2
Estas inecuaciones tienen la solucin y 1.
0
ArcSeny
3
+

6

_

_
ArcSeny
3
+

6
0
ArcSeny
3
+

6

_
_
_
ArcSeny

2
ArcSeny
5
2
Estas inecuaciones tienen la solucin y 1. Por tanto el rango es la interseccin de estas condi-
ciones: y [1; 1].
CAPTULO 11. FUNCIONES TRIGONOMTRICAS 595
Ejemplo 11.87 Determine la paridad de la funcin:
a) f(x) =

4 x
2
ArcSen(2 x)
; b) f(x) = ArcSen(x [x]).
Solucin
a) f(x) =
_
4 (x)
2
ArcSen(2 (x))
=

4 x
2
ArcSen(2 +x)
,= f(x)
La funcin no es par ni impar.
b) f(x) = ArcSen((x) [(x)]) = ArcSen(x [x]) ,= f(x)
La funcin no es par ni impar.
11.18.2. Funcin arco coseno
La funcin inversa al coseno se llama arco coseno. Si y = Cosx, tendremos que x = ArcCosy,
lo que se debe interpretar del siguiente modo: x es un ngulo cuyo coseno es igual a y. Designando
el argumento de la funcin inversa tambin por la letra x, y la funcin por la letra y, obtendremos
la notacin
y = ArcCosx.
La funcin arco coseno ser de simple valuacin si el con-
junto de sus valores estn comprendidos en el segmento [0; ].
En tal caso, a cada valor de [x[ 1 corresponde un nico valor
de y (0 y ).
La propiedad de que las funciones Cosx y ArcCosx sean inver-
sas se escribe como:
_
Cos(ArcCosx) = x, si 1 x 1;
ArcCos(Cosx) = x, si 0 x .
La grca de la funcin y = ArcCosx coincide con la parte de
la grca de la funcin x = Cosy, que corresponde a la variacin de y desde 0 hasta .
Haciendo uso de las propiedades del arco coseno de un nmero, obtenemos las siguientes car-
actersticas de la funcin y = ArcCosx:
El dominio es [-1; 1];
El codominio es [0; ];
La funcin est acotada inferior y superiormente;
La funcin toma su valor mximo y = cuando x = 1, y el valor mnimo y = 0, cuando
x = 1;
La funcin no es peridica;
La funcin no es par ni tampoco impar;
La funcin es decreciente en todo el dominio;
Los puntos
_
0,

2
_
y (1, 0) son puntos de interseccin con los ejes de coordenadas.
CAPTULO 11. FUNCIONES TRIGONOMTRICAS 596
Ejemplo 11.88 Encuentre el dominio de la funcin
f(x) =

ArcSenx ArcCosx
Solucin
Para que esta funcin est denida, debe cumplir la siguiente condicin:
_
ArcSenx ArcCosx 0
1 x 1

_
_
_
x

2
2
1 x 1
Por tanto el dominio de la funcin, esta dado por la interseccin de ambos intervalos. Es decir
x
_

2
2
; 1
_
.
Ejemplo 11.89 Encuentre el rango de las funciones:
f(x) = ArcCos
1 x
2
1 +x
2
Solucin
y =

2
ArcSen
_
1 x
2
1 +x
2
_
x
2
=
1 Sen
_

2
y
_
1 +Sen
_

2
y
_ x =

1 Sen
_

2
y
_
1 +Sen
_

2
y
_
Esta expresin est denida si cumple la siguiente desigualdad:
_

_
1 Sen
_

2
y
_
1 +Sen
_

2
y
_ 0
1 +Sen
_

2
y
_
,= 0

_
Sen
_

2
y
_
1
Sen
_

2
y
_
+ 1
0
1 +Sen
_

2
y
_
,= 0

_
y [0; ]
y ,=
Por tanto el rango es la interseccin de estas condiciones: y [0; ).
Ejemplo 11.90 Determine la paridad de la funcin:
a) f(x) = ArcCosx [ArcCosx]; b) f(x) = ArcCos(Cosx) x.
Solucin
a) f(x) = ArcCos(x) [ArcCos(x)] ,= f(x)
La funcin no es par ni impar.
b) f(x) = ArcCos(Cos(x)) (x) = ArcCos(Cosx) +x ,= f(x)
La funcin no es par ni impar.
11.18.3. Funcin arco tangente
La funcin y = Tanx pone a cada valor del argumento x, del dominio x ,=

2
+ n, en corre-
spondencia un valor determinado de y, es decir, la tangente de este ngulo. Se puede establecer
tambin la correspondencia univoca inversa entre los valores de y y x, si a la funcin y = Tanx la
vamos a considerar slo para los valores de x que se encuentran en el intervalo
_

2
;

2
_
. En tal
caso, a cada nmero real y, tomado como valor de la tangente, se puede poner en correspondencia el
nico numero x, es decir, el correspondiente ngulo en radianes: cualquier recta paralela al eje 0X,
trazada a una distancia nita cualquiera del eje 0X, interseca a la grca de la funcin y = Tanx
CAPTULO 11. FUNCIONES TRIGONOMTRICAS 597
slo en un punto, cuya abscisa se encuentra entre

2
y

2
.
La funcin inversa a la tangente se llama arco tangente. Si y = Tanx es la funcin directa,
tendremos que x = ArcTany es la funcin inversa. Esta notacin hay que entenderla as: x es un
ngulo tal, medido en radianes, tomado en el intervalo
_

2
;

2
_
, cuya tangente es igual al nmero
y. Trasladando las designaciones del argumento y de la funcin, escribimos la funcin inversa en
la forma y = ArcTanx. En esta notacin el argumento x es un numero real cualquiera; la funcin
y (ngulo en radianes) es un numero cualquiera del intervalo
_

2
;

2
_
.
La propiedad de que las operaciones Tan
y ArcTan sean inversas se escribe del sigu-
iente modo:
_
Tan(ArcTanx) = x, si x R;
ArcTan(Tanx) = x, si

2
< x <

2
.
En la gura se muestra la grca de la fun-
cin y = ArcTanx. Esta curva coincide con
la curva de la funcin x = Tany, cuando el
argumento y vara en el intervalo
_

2
;

2
_
.
Haciendo uso de las propiedades del arco tangente de un nmero, obtenemos las siguientes
caractersticas de la funcin y = ArcTanx:
El argumento x puede ser un numero real cualquiera, es decir, la funcin est denida en
todo el eje numrico;
El conjunto de valores de la funcin forma el intervalo
_

2
;

2
_
;
La funcin est acotada inferior y superiormente;
La funcin no tiene valores mximo ni mnimo, si se la considera en todo el eje numrico;
La funcin no es peridica;
La funcin es impar, puesto que la grca es simtrica respecto al origen de coordenadas;
La funcin crece en todo el dominio; cuando x, al crecer, recorre el eje numrico de izquierda
a derecha, los valores de la funcin aumentan sucesivamente;
El punto (0, 0) es el nico punto de interseccin con los ejes coordenados.
Ejemplo 11.91 Encuentre el dominio de la funcin
f(x) =
1
ArcTanx
Solucin
Para que esta funcin est denida, debe cumplir la siguiente condicin:
ArcTanx ,= 0 x ,= 0.
Por tanto el dominio de la funcin est dado por x R0.
CAPTULO 11. FUNCIONES TRIGONOMTRICAS 598
11.18.4. Funcin arco cotangente
La funcin inversa a la cotangente se llama arco cotangente. De la igualdad y = Cotx se de-
duce que x = ArcCoty, o en las notaciones ya acostumbradas y = ArcCotx. En la igualdad x es
un numero real cualquiera, tomado como valor de la cotangente, y es el ngulo correspondiente
tomado del intervalo 0 < y < . En la gura se muestra la grca de la funcin.
La propiedad de que las funciones ArcCotx
y Cotx sean inversas se puede apreciar de
la siguiente manera:
_
Cot(ArcCotx) = x, si x R;
ArcCot(Cotx) = x, si 0 < x < .
Haciendo uso de las propiedades del ar-
co cotangente de un nmero, obtenemos las
siguientes caractersticas de la funcin y =
ArcCotx:
El dominio es (; +);
El codominio es (0; );
La funcin est acotada superior e inferiormente;
La funcin no toma el valor mximo ni tampoco el mnimo;
La funcin no es peridica;
La funcin no es par ni impar;
La funcin es decreciente en todo el dominio;
El punto
_
0,

2
_
es el nico punto de interseccin con los ejes coordenados.
Ejemplo 11.92 Graque la funcin:
f(x) = log(1 2ArcCotx)
Solucin
Para gracar una funcin trigonomtrica inversa, tenemos que hacer el siguiente anlisis:
Para que esta funcin est denida, debe cumplir la siguiente condicin:
1 2ArcCotx > 0 1 + 2ArcTanx > 0 ArcTanx >
1
2
Esta inecuacin se satisface para los valores de x > Cot
1
2
. Por tanto el dominio de la funcin est
dado por x
_
Cot
1
2
; +
_
. Para calcular el rango, hacemos:
10
y
= 1 2ArcCotx ArcCotx =
1 10
y
2
x = Cot
_
1 10
y
2
_
Esta expresin est denida si cumple la siguiente desigualdad:
0 <
1 10
y
2
<
_
_
_
1 10
y
2
> 0
110
y
2
<

_
1 10
y
> 0
1 10
y
< 2
y < 0
CAPTULO 11. FUNCIONES TRIGONOMTRICAS 599
Por tanto el rango es y (; 0). Aplicando el teorema de mximos y mnimos, tenemos
log(1 2ArcCotx) = log(1 2ArcCotk) 1 2ArcCotx = 1 2ArcCotk
ArcCotx = ArcCotk

2
ArcTanx =

2
ArcTank
ArcTan
_
x +k
1 xk
_
= 0
x +k
1 xk
= 0 x = k
Como contradice el principio del teorema, entonces la funcin no tiene puntos de mximo ni mnimo.
La paridad de la funcin se establece como:
f(x) = log(1 2ArcCot(x)) = log(1 2ArcTanx ) ,= f(x)
Por lo tanto la funcin no es par ni impar. Esta curva tiene una asntota horizontal en y = 0. Segn
el dominio y el rango de la funcin, el intervalo
_
Cot
1
2
; +
_
es de crecimiento.
Las relaciones trigonomtricas inversas no son funciones, ya que hay muchos valores de f(x)
que estn asociados con un valor dado de x. Para obtener las funciones trigonomtricas inversas,
se restringen las relaciones trigonomtricas a ciertos valores, llamados valores principales. A con-
tinuacin se da una tabla en la cual se indica el dominio e imagen de las funciones trigonomtricas
inversas:
Funcin Valores principales x 0 Valores principales x < 0
y = ArcSenx 0 y

2

2
y < 0
y = ArcCosx 0 y

2

2
< y
y = ArcTanx 0 y

2

2
y < 0
y = ArcCotx 0 y

2

2
< y <
Funcin Dominio Imagen
y = ArcSenx 1 x 1

2
y

2
y = ArcCosx 1 x 1 0 y
y = ArcTanx < x < +

2
< y <

2
y = ArcCotx < x < + 0 < y <
11.19. Tarea
1. Determine el dominio de la siguiente funcin:
CAPTULO 11. FUNCIONES TRIGONOMTRICAS 600
a) f(x) = xArcCos
x
2

_
4 x
2
;
b) f(x) = ArcTan
_
x +
_
1 +x
2
_
;
c) f(x) = ArcCos(2x
_
1 x
2
);
d) f(x) =
1

2
ArcTan
x
2
1

2x
;
e) f(x) = ArcTan
1

1 x
2
x
;
f ) f(x) = xArcSenx +
_
1 x
2
;
g) f(x) =

xArcSen

x +

1 x;
h) f(x) = Cosx ArcSenx +x;
i) f(x) =

1 x ArcSen

x;
j) f(x) = ArcCos(Senx
4
Cosx
4
);
k) f(x) =

x
2
+ 2x
(x + 1)
2
ArcSen
1
x + 1
.
2. Determine el dominio de la siguiente funcin:
a) f(x) = ArcTan(x + 1) +
x + 1
x
2
+ 2x + 2
;
b) f(x) = xArcCos
_
x
x + 1

x ArcTan

x;
c) f(x) = x(ArcSenx)
2
2x + 2
_
1 x
2
ArcSenx.
3. Determine el codominio de la siguiente funcin:
a) f(x) = ArcSen
Senx

1 +Sen
2
x
;
b) f(x) = ArcTan
3x x
3
1 3x
2
;
c) f(x) =
1 +x
2
ArcTanx
2

1 +x4
;
d) f(x) =
1

2
ArcSen
_
_
2
3
Senx
_
;
e) f(x) =
1

2
ArcTan
x
2
1

2x
;
f ) f(x) = ArcSec
3x x
2
1 3x
2
;
g) f(x) = ArcSen
2x
3
1 +x
6
;
h) f(x) = ArcSen
2x
1 +x
2
.
4. Determine la paridad de la siguiente funcin:
a) f(x) = ArcTan
3x x
3
1 3x
2
;
b) f(x) = ArcTan
3x x
2
1 3x
2
;
c) f(x) = ArcTan
1

1 x
2
x
;
d) f(x) = (x + 1)ArcTan

x;
e) f(x) = xArcSenx +
_
1 x
2
;
f ) f(x) =
1

2
ArcTan
x
2
1

2x
;
g) f(x) = Cosx ArcSenx +x;
h) f(x) =
1 +x
2
ArcTanx
2

1 +x4
.
5. Determine la paridad de la siguiente funcin:
a) f(x) =
(x
2
+ 1)ArcTanx

x
;
b) f(x) = ArcTan(x + 1) +
x + 1
x
2
+ 2x + 2
;
c) f(x) =

x
2
+ 2x
(x + 1)
2
ArcSen
1
x + 1
;
d) f(x) = (2x
2
1)ArcSenx +x
_
1 x
2
;
CAPTULO 11. FUNCIONES TRIGONOMTRICAS 601
e) f(x) = (ArcSenx)
2
+ 2x
_
1 x
2
ArcSenx x
2
;
f ) f(x) = xArcCos
_
x
x + 1

x ArcTan

x;
g) f(x) = 3x
4
ArcSen
1
x
+ (x
2
+ 2)

x2 1.
6. Graque las funciones:
a) f(x) = ArcTan
3x x
3
1 3x
2
;
b) f(x) = ArcSen
2x
1 +x
2
;
c) f(x) = ArcTan
_
1 x
1 +x
;
d) f(x) = ArcTan(Tan
2
x);
e) f(x) = ArcSec
3x x
2
1 3x
2
;
f ) f(x) = ArcTan
1

1 x2
x
;
g) f(x) = ArcCos(2x
_
1 x
2
).
Captulo 12
Curvas dadas implcitamente
12.1. Deniciones generales
Una funcin de una sola variable puede denirse implcitamente con ayuda de la igualdad
F(x, y) = 0, donde F es una funcin de dos variables x e y.
Supongamos que en cierto conjunto A de puntos (x, y) viene dada una funcin F. La igualdad
F(x, y) = 0 determina cierto subconjunto B del conjunto A, en el cual la funcin F es igual a cero.
Desde luego, en particular B puede representar un conjunto vaco. Sea B un conjunto no vaco y
supongamos que C es un conjunto no vaco, evidentemente de aquellos valores de x, a los cuales
corresponde al menos un valor de y de modo tal que el par (x, y) pertenezca a B. As pues, C es
un conjunto de todos los nmeros x, a cada uno de los cuales corresponde un conjunto no vaco de
nmeros y de tal modo que (x, y) B o bien, que es lo mismo, para el par citado (x, y) se verique
la igualdad F(x, y) = 0. Con ello queda denida en el conjunto C cierta funcin de x: y = (x) que
es, en general, multiforme. En este caso suele decirse que la funcin est denida implcitamente
con ayuda de la igualdad F(x, y). Para ella se cumple, obviamente, la identidad F(x, (x)) = 0
para todo x C.
Anlogamente, podemos denir tambin una funcin x = (y) de la variable y, que se dene
implcitamente con ayuda de la igualdad F(x, y) = 0. Para ella se cumple la identidad F((y), y) =
0 para todo y C
1
, donde C
1
es un conjunto de nmeros. Adems, la funcin y = (x) x = (y)
satisface la ecuacin F(x, y) = 0. La funcin x = (y) se denomina inversa respecto de la funcin
y = (x).
Denicin 12.1 Funcin implcita
Se dice que una funcin est denida implcitamente con la ecuacin F(x, y) = 0 (funcin implci-
ta) si cada valor de su argumento x y el valor de la funcin y, correspondiente a l, son la solucin
de la mencionada ecuacin F(x, y) = 0.
Lleva el nombre de grco de la ecuacin F(x, y) = 0 en el sistema cartesiano de coordenadas
X0Y el conjunto de todos los puntos del plano, cuyas coordenadas (x, y) satisfacen dicha ecuacin.
El grco de toda funcin denida implcitamente con la ecuacin F(x, y) = 0 est contenido en el
grco de sta. Mediante la ecuacin F(x, y) = 0 es posible prejar no una, sino todo un conjunto
de funciones.
602
CAPTULO 12. CURVAS DADAS IMPLCITAMENTE 603
En ocasiones, se consigue pasar del procedimiento implcito de denicin al explcito, es decir,
dar la funcin con la formula y = f(x).
Ejemplo 12.1
Demuestre que con la siguiente ecuacin se preja una funcin:
a)
_
y
3
+xy x

y x
2
= 0; b)
xy1
yx
= 0; c) log(xy 1) = log[(1 x)(1 y)].
Solucin
a) Pasamos la ecuacin de la forma implcita a la explcita, es decir:
_
y
3
x

y = x
2
xy

y(y x) x(x y) = 0 (

y +x)(y x) = 0
_

y +x = 0
y x = 0

_
x =

y, y 0
x = y
El grco de la ecuacin dada esta formado por una parbola y una recta.
b) Pasamos la ecuacin de la forma implcita a la explcita, es decir:
xy 1
y x
= 0
_
xy 1 = 0
y x ,= 0

_
y =
1
x
y ,= x
El grco de la ecuacin dada esta formado por una hiprbola con dos huecos por donde pasa la
recta y = x.
c) Pasamos la ecuacin de la forma implcita a la explcita, es decir:
log(xy 1) log[(1 x)(1 y)] log
xy 1
(1 x)(1 y)
= 0
xy 1
(1 x)(1 y)
= 1 xy 1 = (1 x)(1 y) y = 2 x
El grco de la ecuacin dada esta formado por una recta con pendiente negativa con un intervalo
en el cual no existe la expresin.
12.2. Curvas implcitas
Durante el anlisis y la construccin de una curva, prejada con la ecuacin F(x, y) = 0, en
ocasiones, se consigue representar la curva o bien parte de ella como el grco de la funcin y = f(x)
(ella satisface la igualdad F(x, f(x)) = 0).
Denicin 12.2 Curva algebraica
Recibe el nombre de curva algebraica de n-simo orden aquella que en el sistema cartesiano de
coordenadas se puede prejar con la ecuacin de la forma

a
kp
x
k
y
p
= 0 (12.1)
CAPTULO 12. CURVAS DADAS IMPLCITAMENTE 604
donde la suma est compuesta de todos los enteros k y p tales que 0 k n, 0 p n, k +p n
y hay al menos un sumando no nulo para el que k +p = n. Si la recta y = kx +b es la asntota de
una curva algebraica, los coecientes k y b pueden ser hallados de la forma siguiente: pongamos
en la ecuacin (14.1) y = kx + b y en el polinomio obtenido respecto de x, igualamos a cero los
coecientes de las dos mayores potencias de x, los coecientes k y p son las soluciones de este
sistema de dos ecuaciones. Si la recta x = c es la asntota vertical de una curva algebraica, c es la
raz del polinomio respecto de x y que es el coeciente de la potencia mayor de y en la ecuacin de
la curva.
Ejemplo 12.2 Analizar y trazar el grco de las curvas:
a) x
2
(y 2)
2
+ 2xy y
2
= 0; b) x
3
+y
3
= 3x
2
.
Solucin
a) El coeciente de la potencia mayor de y
2
es igual a x
2
1. Por consiguiente, las asntotas
verticales slo pueden ser las rectas x = 1 y x = 1. Con el n de hallar las asntotas oblicuas,
pongamos en la ecuacin dada y = kx +b. Obtenemos
k
2
x
4
+ 2k(b 2)x
3
+ (b
2
4b k
2
+ 2k + 4)x
2
+ 2b(1 k)x b
2
= 0
e igualando a cero los coecientes de las potencias de x
4
, x
3
y x
2
, llegamos al sistema
_

_
k
2
= 0
2k(b 2) = 0
b
2
4b k
2
+ 2k + 4 = 0
Este sistema nos proporciona la solucin siguiente: k = 0 y b = 2. Esto quiere decir que la curva
no tiene asntotas oblicuas, pero si una asntota horizontal y = 2.
b) El coeciente de la potencia mayor de y
3
es igual a 1. Por consiguiente, no existen asntotas
verticales. Con el n de hallar las asntotas oblicuas, pongamos en la ecuacin dada y = kx + b.
Obtenemos
(k
3
+ 1)x
3
+ (3bk
2
3)x
2
+ 3b
2
kx +b
3
= 0
e igualando a cero los coecientes de las potencias de x
3
y x
2
, llegamos al sistema
_
k
3
+ 1 = 0
3bk
2
3 = 0

_
(k + 1)(k
2
k + 1) = 0
3bk
2
3 = 0
Este sistema nos proporciona la solucin siguiente: k = 1 y b = 1. Esto quiere decir que la curva
tiene como asntota oblicua y = x + 1. No hay asntotas horizontales.
CAPTULO 12. CURVAS DADAS IMPLCITAMENTE 605
12.3. Tarea
1. Construya la curva dada implcitamente:
a) x
4
y
4
= xy;
b) x
2
y
2
+y = 1;
c) x
2
y +xy
2
= 1;
d) (y x
2
)
2
= x
5
;
e) y
2
= 2x
3
x
4
;
f ) y
2
x
4
= (x
2
1)
3
;
g) x
3
y
3
= 1;
h) x
3
+y
3
= 6x
2
;
i) 9y
2
= 4x
3
x
4
;
j) x
4
+ 2y
3
= 4x
2
y;
k) x
4
+y
4
= 1;
l) x
2
y
2
= (x 1)(x 2);
m) x
2
y
2
= x
3
y
3
;
n) y
2
(x
2
1) = x
4
4x
2
;
o) x
4
y
4
= x
2
2y
2
;
p) y
2
(1 x) = x
2
(1 +x);
q) x
4
2ax
3
+a
2
y
2
= 0;
r) (xy +1)(x+y 1) = 1;
s) (x +y + 1)
2
= x
2
+ 1;
t) x
3
+xy
2
+ax
2
= ay
2
;
u) xy(x y) +x +y = 0;
v) x
3
+xy
2
+ay
2
= 3ax
2
.
2. Construya la curva dada implcitamente:
a) x
4
+y
4
6y
3
+ 8x
2
y = 0;
b) y
2
(x
2
+ 1) = x
2
(x
2
1);
c) (x
2
+y
2
2x)
2
= x
2
+y
2
;
d) (x 2y)
2
+ (4x + 2y)
2
= 4;
e) (x
2
+y
2
x)
2
= 4(x
2
+y
2
);
f ) (x
2
+y
2
)
3
= 27x
2
y
2
;
g) (x
2
y
2
)(x y) = 4x
2
;
h) x
2
y
2
+y
4
= 4x
2
;
i) x
3
+y
3
= x
2
+y
2
;
j) x
4
+y
4
= x
2
+y
2
;
k) y
2
= 9(x
4
x
6
);
l) y
2
x
2
= 4(x 1);
m) y
2
(2 x) = x
3
;
n) x
4
+y
4
= 2xy;
o) (x
2
+y
2
)
2
= xy;
p) 3y
2
x = x
3
2.
3. Construya la curva dada implcitamente:
a) x
3
2x
2
x
2
y 3x + 2xy +xy
2
+ 3y y
3
= 0;
b) 2x
3
x
2
y 16x 4xy + 2xy
2
+ 8y + 2y
2
y
3
= 0;
c) 3x
3
2x
2
y 12x
2
+ 8xy + 3xy
2
2y
3
= 0;
d) 3x
3
12x
2
2x
2
y + 11xy 2y
2
= 0.
Captulo 13
Curvas dadas paramtricamente
13.1. Ecuaciones parametricas
Cuando la trayectoria de una partcula que se mueve en el plano se ve como la curva de la
gura, no podemos describirla con una frmula cartesiana que exprese y directamente en funcin
de x o x directamente en funcin de y.
Si en una ecuacin f(x, y) = 0, x y y son expresadas en trminos de una sola variable en tal
forma que la relacin dada entre x y y sea vlida, la tercera variable se llama parmetro. Las
ecuaciones entre x y la tercera variable y y y la tercera variable se conocen como representacin
paramtrica de f(x, y) = 0 o de una parte de ella.
Expresamos cada una de las coordenadas de la partcula en funcin del tiempo t y describimos
la trayectoria con un par de ecuaciones x = f(t) e y = g(t). Para estudiar el movimiento, son
preferibles ecuaciones como stas a una frmula cartesiana, porque nos dan la posicin de la
partcula en cualquier tiempo t.
Denicin 13.1 Curva paramtrica
Sea que en el conjunto I son dadas dos funciones x = f(t) e y = g(t). El conjunto de todos los
puntos del plano de coordenadas con coordenadas (f(t), g(t)), t I, recibe el nombre de curva plana
C. Las ecuaciones x = f(t) e y = g(t) se denominan ecuaciones paramtricas C, donde a t se le
conoce como parmetro.
Sean P y Q los conjuntos de los valores de las funciones x = f(t) e y = g(t), respectivamente,
denidas sobre I. Para cada t I al valor de x = f(t) contraponemos el valor de y = g(t).
Con ello, puede suceder que al valor x P se ha contrapuesto ms de un valor de y Q. Sea
dada una regla de acuerdo con la cual, del conjunto de los valores de y, contrapuestos del modo
indicado ms arriba al valor de x, slo se elige un valor. Las funciones x = f(t) e y = g(t), t I,
junto con la mencionada regla, denen la funcin y = f(x), x P, de la que se dice que est
prejada paramtricamente.
Debemos entender, sin embargo, que puede ser difcil o imposible despejar la segunda variable.
Este procedimiento puede conducir a una complicada funcin del parmetro, a menos que se eli-
ja cuidadosa y juiciosamente la funcin arbitraria del parmetro que se pone en ecuacin con la
primera variable. Una representacin conveniente debe ser simple y debe dar la curva entera, a
606
CAPTULO 13. CURVAS DADAS PARAMTRICAMENTE 607
menos que se aclare de otro modo.
Podemos obtener una representacin paramtrica de una ecuacin con dos variables en muchos
casos, del siguiente modo:
Igualando una de las variables a una funcin arbitraria del parmetro.
Sustituyendo esta igualdad en la ecuacin dada por la variable.
Despejando la segunda variable en trminos del parmetro.
Ejemplo 13.1 Eliminando el parmetro t, obtenga la ecuacin cuyo grco coincide con la
curva dada:
a)
_
x = 2 + 3Cost
y = 3 + 4Sent
; b)
_
x = Sent
y = (1 +Sent)Tant
; c)
_
x = (t + 1)
2
y = (t 1)
2
.
Solucin
a) Sabemos que Sen
2
t +Cos
2
t = 1, entonces
x = 2 + 3Cost Cost =
x 2
3
; y = 3 + 4Sent Sent =
y + 3
4
Sen
2
t +Cos
2
t = 1
_
x 2
3
_
2
+
_
y + 3
4
_
2
= 1.
b) Reemplazando x en y, obtenemos
y = (1 +Sent)Tant = (1 +x)
x
Cost
Cost =
x(1 +x)
y
Sabemos que Sen
2
t +Cos
2
t = 1, reemplazando Sent y Cost en esta ecuacin, obtenemos:
x
2
+
x
2
(1 +x)
2
y
2
= 1 x
2
y
2
+x
2
(1 +x)
2
= y
2
x
2
y
2
y
2
+x
2
(1 +x)
2
= 0 x
2
(1 +x)
2
+y
2
(x
2
1) = 0
c) Despejando el parmetro t en ambas ecuaciones, obtenemos:
x = (t + 1)
2
t =

x 1; y = (t 1)
2
t =

y + 1
Igualando ambas ecuaciones, tenemos

y 2 = 0
13.2. Tarea
1. Eliminando el parmetro t, obtenga la ecuacin cuyo grco coincide con la curva dada:
a)
_
x = e
t
,
y = t
3
.
b)
_
x = aCos
2
t,
y = bSen
2
t.
c)
_
x = te
t
,
y = te
t
.
d)
_
x = e
t
Cost,
y = e
t
Sent.
e)
_
x = e
t
,
y = e
2t
.
f )
_
x = 2t + 3t
2
,
y = t
2
+ 2t
3
.
CAPTULO 13. CURVAS DADAS PARAMTRICAMENTE 608
g)
_

_
x =
1 + ln t
t
2
,
y =
3 + 2 ln t
t
.
h)
_
x = t
3
+ 3t + 1,
y = t
3
3t + 1.
i)
_
_
_
x =

t
2
+ 1,
y =
t 1

t
2
+ 1
.
j)
_
x = ln Sen
t
2
,
y = ln Sent.
k)
_

_
x =
t 8
t
2
4
,
y =
3
t(t
2
4)
.
l)
_
_
_
x = t
2
+ 6t + 5,
y =
t
2
54
t
.
m)
_
_
_
x = t ln t,
y =
ln t
t
.
n)
_
x = a(t Sent),
y = a(1 Cost).
o)
_

_
x =
1
t + 1
,
y =
_
t
t + 1
_
2
.
p)
_
_
_
x = Cot2t,
y =
2Cos2t 1
2Cost
.
q)
_

_
x =
3at
1 +t
3
,
y =
3at
2
1 +t
3
.
r)
_
x = Sen
2
t,
y = Cos
2
t.
s)
_

_
x =
2e
t
t 1
,
y =
tet
t 1
.
t)
_
x = (t 1)
2
(t 2),
y = (t 1)
2
(t 3).
u)
_
x = t 3,
y = t bArcTant.
v)
_
x = t + 2t
2
+t
3
,
y = 2 + 3t t
3
.
w)
_

_
x =
2at
1 +t
2
,
y =
a(1 t
2
)
1 +t
2
.
x)
_
x = t 1,
y = t
2
2t + 2.
y)
_
x = 2 3Cost,
y = 1 + 3Sent.
z)
_
x = Tant,
y = Sen2t + 2Cos2t.
13.3. Curvas paramtricas
Al dibujar una curva, dada por un par de ecuaciones paramtricas, seguiremos marcando los
puntos en un plano XY . Cada par de coordenadas (x, y) queda determinado por un valor escogi-
do para el parmetro t. Marcando los puntos en orden de t creciente, trazamos la curva en una
direccin determinada. Nos referimos a ella como la orientacin de la curva. El mtodo simple de
dibujo punto a punto para trazar la curva dada, es un proceso tedioso y puede simplicarse, a
veces, hallando una ecuacin rectangular que tenga la misma grca. Llamaremos a este proceso
eliminacin del parmetro. Si la funcin de t es una funcin trigonomtrica, la eliminacin de t se
facilita mediante el uso de identidades trigonomtricas.
Los rangos de x e y involucrados en las ecuaciones paramtricas pueden verse alterados al
cambiar a forma rectangular. En tal circunstancia, hay que ajustar el dominio de la ecuacin
rectangular de modo tal que su grca se adapte a la grca de las ecuaciones paramtricas.
Si las ecuaciones paramtricas representan la trayectoria de un objeto en movimiento, la grca
por s sola no es suciente para describir el movimiento del objeto. Necesitaremos las ecuaciones
paramtricas para saber la posicin, direccin y velocidad en un instante dado.
1. En este punto del esquema se pueden hallar las asntotas de la curva. Para esto es necesario
tener en cuenta lo siguiente:
si para t t
p
, x x
0
e y , entonces x = x
0
es la asntota vertical de la curva;
si para t t
p
, x e y y
0
, entonces y = y
0
es la asntota horizontal de la curva;
si para t t
p
, x e y , resulta que puede existir una asntota oblicua.
2. Se determina si la curva posee simetra, lo que permitir reducir los clculos. Al estudiar
la simetra hace falta tomar en consideracin cuatro casos, cuando en vez de todo el dominio
D es suciente examinar slo su parte no negativa:
CAPTULO 13. CURVAS DADAS PARAMTRICAMENTE 609
para todo t D: x(t) = x(t), y(t) = y(t) simetra con respecto al eje 0X;
para todo t D: x(t) = x(t), y(t) = y(t) simetra respecto al eje 0Y ;
para todo t D: x(t) = x(t), y(t) = y(t) simetra respecto al origen de
coordenadas;
para todo t D: x(t) = x(t), y(t) = y(t) superposicin.
3. Se hallan los valores nulos de las funciones x(t), y(t) y los intervalos donde estas funciones
poseen signos constantes.
4. Cuando a dos valores distintos t
1
y t
2
del parmetro t corresponde el mismo punto, a dicho
punto se le denomina punto doble, pues pertenece a dos ramas de la curva. Estos puntos son
los correspondientes a valores a y b distintos que satisfacen x(a) = x(b) y y(a) = y(b). De
forma anloga se denira un punto de multiplicidad n.
Ejemplo 13.2 Gracar las curvas:
a)
_
x = t
3
+ 3t + 1,
y = t
3
3t + 1.
b)
_
x =
t
2
t1
,
y =
t
2
1
t
.
c)
_
x = 2t
3
3t
2
+ 1,
y = t
3
4t 1.
Solucin
a) Segn la expresin, podemos darnos cuenta que el dominio es t R. Haciendo x(t) = x(k) y
y(t) = y(k), obtenemos
t
3
+ 3t + 1 = k
3
+ 3k + 1
_
t = k
k
2
+ 1 = 0
Como la primera ecuacin no tiene races reales, entonces no tenemos valores de k.
t
3
3t + 1 = k
3
3k + 1
_
t = k
k
2
1 = 0
Resolviendo la segunda ecuacin, tenemos que k = 1 y k = 1. Con estos nuevos valores y el
dominio de la curva, construimos una tabla de la siguiente manera:
t (; 1) (1; 1) (1; +)
x (; 3) (3; 5) (5; +)
y (; 3) (3; 1) (1; +)
De aqu podemos deducir que P(3, 3) es un punto de mximo y Q(5, 1) es un punto de mnimo.
El intervalo (; 3) es creciente, (-3; 5) es decreciente y (5; +) es estrictamente creciente.
b) Segn la expresin, podemos darnos cuenta que el dominio es t R0, 1.
Haciendo x(t) = x(k) y y(t) = y(k), obtenemos
t
2
t 1
=
k
2
k 1

_
t = k
k(k 2) = 0
CAPTULO 13. CURVAS DADAS PARAMTRICAMENTE 610
De la primera ecuacin obtenemos k = 0 y k = 2.
t
2
1
t
=
k
2
1
k

_
t = k
k
2
+ 1 = 0
Resolviendo la segunda ecuacin, obtenemos races imaginarias, por tanto no existen valores de k
reales. Con estos nuevos valores y el dominio de la curva, construimos una tabla de la siguiente
manera: El intervalo (; 0) es estrictamente decreciente, (0; ) es estrictamente decreciente,
(+; 4) es creciente y (4; +) es estrictamente creciente. Podemos observar en el cuadro que y = 0
es asntota horizontal.
t (; 0) (0; 1) (1; 2) (2; +)
x (; 0) (0; ) (+; 4) (4; +)
y (; +) (; 0) (0; 3/2) (3/2; +)
c) Segn la expresin, podemos darnos cuenta que el dominio es t R. Haciendo x(t) = x(k) y
y(t) = y(k), obtenemos
2t
3
3t
2
+ 1 = 2k
3
3k
2
+ 1
_
t = k
k(k 1) = 0
De la primera ecuacin obtenemos k = 0 y k = 1.
t
3
4t 1 = k
3
4k 1
_
t = k
3k
2
4 = 0
Resolviendo la segunda ecuacin, obtenemos k =
2

3
3
y k =
2

3
3
. Con estos nuevos valores y el
dominio de la curva, construimos una tabla de la siguiente manera:
CAPTULO 13. CURVAS DADAS PARAMTRICAMENTE 611
El intervalo
_
;
27+16

3
9
_
es decreciente,
_

27+16

3
9
; 1
_
es decreciente, (1; 0) es decreciente,
_
0;
2716

3
9
_
es decreciente y
_

2716

3
9
; +
_
es estrictamente creciente.
t (; 1, 15) (1, 15; 0) (0; 1) (1; 1, 15) (1, 15; +)
x (; 6, 08) (6, 08; 1) (1; 0) (0; 0, 08) (0, 08; +)
y (; 2, 08) (2, 08; 1) (1; 4) (4; 4, 08) (4, 08; +)
13.4. Tarea
1. Gracar las curvas:
a)
_

_
x =
t
2
1 +t
3
,
y =
t
3
1 +t
3
.
b)
_

_
x =
1
t(t + 1)
,
y =
(t + 1)
2
t
.
c)
_

_
x =
(t + 1)
2
t
,
y =
t + 1
t + 2
.
d)
_

_
x =
t
3
1 +t
2
,
y =
t
3
2t
2
1 +t
2
.
e)
_

_
x = t
3
3t,
y =
_
t 1
t
_
2
.
f )
_

_
x =
1
t
3
t
2
,
y =
1
t
2
t
.
g)
_

_
x =
t
2
1 2t
,
y =
t
3
1 2t
.
h)
_

_
x =
1
t t
2
,
y =
1
t t
3
.
i)
_

_
x =
t +t
3
1 +t
4
,
y =
t t
3
1 +t
4
.
j)
_

_
x =
t
2
t
2
1
,
y =
t
2
+ 1
t + 2
.
k)
_

_
x =
t
2
+ 1
t
,
y =
t
3
+ 1
t
2
.
l)
_

_
x =
t
2
+ 6t + 5
3
,
y =
t
3
54
2t
.
m)
_

_
x =
1
t t
5
,
y =
t
4
1 t
4
.
n)
_

_
x =
5at
2
1 +t5
,
y =
5at
3
1 +t
5
.
o)
_

_
x =
3at
1 +t
3
,
y =
3at
2
1 +t
3
.
p)
_

_
x =
at
4
1 t
3
,
y =
at
3
1 t
3
.
q)
_

_
x =
2t
1 t
2
,
y =
t
2
1 t
2
.
r)
_

_
x =
t
3
t
2
+ 1
,
y =
t
3
2t
2
t
2
+ 1
.
2. Si una pelota es lanzada horizontalmente a 120 pies por segundo desde un punto a 4 pies
sobre el terreno horizontal, cunto tiempo tardar la pelota para estar a slo 1,44 pies sobre
el terreno? qu distancia recorrer horizontalmente?
3. Si una pelota es lanzada horizontalmente a 120 pies por segundo desde un punto a 4,5 pies
sobre el terreno horizontal, a qu distancia del terreno estar despus de recorrer 60 pies?
4. Una pelota es lanzada a 120 pies por segundo a un ngulo de 45

con la horizontal. Cun-


to tiempo tardar para estar a la misma altura sobre el terreno que al ser lanzada? Qu
distancia ha recorrido horizontalmente en este tiempo?
CAPTULO 13. CURVAS DADAS PARAMTRICAMENTE 612
5. Una pelota es lanzada a 96 pies por segundo a un ngulo de 30

sobre la horizontal. De-


spus de cuntos segundos estar a su distancia original sobre el nivel del terreno?
6. Se dispara un cuerpo desde el origen con velocidad inicial v
0
metros por segundo formando
ngulo con el eje positivo X. Suponiendo que solo acta la fuerza de gravedad sobre el
cuerpo una vez disparado, obtener las ecuaciones paramtricas de su trayectoria tomando
como parmetro el tiempo t, t mide los segundos transcurridos despus del disparo.
7. Se alcanza un pjaro de un tiro cuando va volando horizontalmente a 40 metros por encima
del cazador. Si la velocidad que lleva es de 45 kilmetros por hora, hallar el tiempo que tarda
en caer y la distancia a que caer del cazador.
8. Se lanza una bola hacia arriba desde el suelo con un ngulo de 60

con la horizontal y con


velocidad inicial de 20 metros por segundo. Hallar:
a) El tiempo que estar en el aire.
b) Su alcance, es decir, la distancia horizontal que cubre.
c) La mxima altura que alcanza.
9. Una escalera de 10 metros se apoya sobre un suelo horizontal y est recostada en una
casa. Un hombre se encuentra a 2/3 arriba de la escalera cuando el pie de sta comienza a
deslizarse. Hallar la trayectoria del hombre.
10. Hallar el conjunto de los vrtices de los tringulos rectngulos de hipotenusa 2a. Tmese
la hipotenusa sobre el eje X con su punto medio en el origen y llmese un ngulo agudo
del tringulo.
11. Se arroja una piedra hacia arriba con velocidad inicial de 16 metros por segundo y forman-
do 60

con la horizontal desde un montculo a 30 metros sobre el nivel de la supercie de un


lago. Hallar:
a) Su mxima altura sobre el lago.
b) Cundo caer a la supercie del lago.
c) Su distancia horizontal desde el punto en que se lanz cuanto toca la supercie.
Captulo 14
Curvas dadas en coordenadas polares
14.1. Transformaciones
En secciones anteriores se especic la posicin de un punto en un plano por medio del sistema
de coordenadas rectangulares, se localizaba el punto dando las distancias perpendiculares del punto
a dos rectas perpendiculares jas, los ejes. Este sistema no es el nico sistema de coordenadas del
que se dispone; se pueden usar otros sistemas, siempre que cada conjunto de coordenadas determine
solamente un punto.
Comenzamos por determinar un punto, O, llamado polo, y una semirrecta, O, llamada eje
polar. El eje polar tiene su origen en el polo, y se extiende indenidamente en una direccin, que,
para nosotros, ser horizontalmente hacia la derecha.
Sea P un punto del plano. Trcese el segmento OP. Dentese
por el ngulo OP cuyo lado inicial es el eje polar, y cuyo lado
nal es OP. Desgnese por r la distancia (no orientada) entre O
y P; es decir, hgase [OP[ = r. Entonces (r, ), tomadas siempre
en este orden, es un conjunto de coordenadas polares de P. Al
determinar las coordenadas polares de P es obvio que r > 0, y
0 < 2.
Exceptuando el origen, cada punto del plano tiene un par de coordenadas nicas (r, ) tal que
r > 0 y 0 < 2. Recprocamente para cada par de nmeros (r, ) que satisfacen estas condi-
ciones hay un solo punto P en el plano que tiene las coordenadas polares (r, ).
Segn se dijo antes, r tiene que ser positivo o cero.
Se pueden denir otros sistemas de coordenadas polares
donde r sea negativo, como sigue: El lado nal de es una
semirrecta cuyo origen coincide con el polo. Por prolon-
gacin del lado nal de entendemos la semirrecta cuyo
origen es el polo, pero cuyo sentido es opuesto al del la-
do nal de . En nuestros diagramas la prolongacin del
lado nal de se ha trazado con lneas de puntos.
Sea P un punto cualquiera del plano. Constryase un ngulo tal que P se encuentre sobre
613
CAPTULO 14. CURVAS DADAS EN COORDENADAS POLARES 614
la prolongacin de su lado nal. Sea r = [OP[. Entonces, (r, ) se dene como un conjunto de
coordenadas polares de P. Las palabras hacia arriba o hacia abajo no determinan por s mismas
el signo de r. Siempre que P se encuentre sobre el lado nal de , r es positivo; siempre que P se
encuentre sobre la prolongacin del lado nal de , r es negativo. Obsrvese que, en la gura, r es
negativo porque P pertenece a la prolongacin del lado nal de , aun cuando la direccin de O
hacia P es hacia arriba.
Para determinar el punto P(r, ), constryase primero
el ngulo con el polo como origen, y el eje polar como
lado inicial. Si r es positiva, sitese P sobre el lado nal
de , de modo que [OP[ = r. Si r es negativa, sitese
P sobre la prolongacin del lado nal de de modo que
[OP[ = r.
Si el origen y el semieje positivo X de un sistema de
coordenadas cartesianas coinciden, respectivamente, con el polo y el eje polar de un sistema de
coordenadas polares, entonces las coordenadas cartesianas y polares de un punto cualquiera P
estn relacionadas por medio de las siguientes frmulas:
x = rCos, y = rSen, r =
_
x2 +y
2
= ArcTan
y
x
, Sen =
y

_
x
2
+y
2
, Cos =
x

_
x
2
+y
2
Con este procedimiento es claro que a cada par de nmeros reales (r, ) corresponde un punto
unvocamente determinado, P, en el plano. Los nmeros (r, ) pueden seguir siendo considerados
como coordenadas polares de P, pero ya no podemos hablar de las coordenadas polares de P,
dado que para cualquier punto jo habr una innidad de pares (r, ) que correspondan a P. Por
ejemplo, si P se encuentra en el origen, entonces r = 0, pero puede ser cualquier nmero real. Si
r ,= 0, entonces puede variar en cualquier mltiplo de 2. Tambin podemos reemplazar r por
r y sumar a para obtener otro conjunto de coordenadas polares para el mismo punto P.
Ejemplo 14.1 Expresar la ecuacin en coordenadas polares:
a) (x
2
+y
2
)
2
= 2(x
2
y
2
); b) 3x
2
+x y
2
= 0; c) 4x
2
+ 9y
2
= 16x.
Solucin
Haciendo los reemplazos correspondientes, obtenemos:
a) (r
2
Cos
2
+r
2
Sen
2
)
2
= 2(r
2
Cos
2
r
2
Sen
2
)
(Cos
2
+Sen
2
)
2
r
4
= 2(Cos
2
Sen
2
)r
2
r
2
= 2Cos2.
b) 3(rCos)
2
+ (rCos) (rSen)
2
= 0
3r
2
Cos
2
r
2
Sen
2
+rCos = 0 3r
2
Cos
2
r
2
(1 Cos
2
) +rCos = 0
4r
2
Cos
2
r
2
+rCos = 0 r[r(4Cos
2
1) +Cos] = 0
r = 0 y r(4Cos
2
1) +Cos = 0.
Por lo tanto
r =
Cos
1 4Cos
2

CAPTULO 14. CURVAS DADAS EN COORDENADAS POLARES 615


es la ecuacin buscada. Ninguna parte de la curva se perdi al omitir el polo r = 0, ya que esta
ecuacin pasa por el polo.
c) 4(rCos)
2
+ 9(rSen)
2
= 36rCos 4r
2
Cos
2
+ 9r
2
Sen
2
= 36rCos
4r
2
Cos
2
+ 9r
2
(1 Cos
2
) = 36rCos 9r
2
5r
2
Cos
2
= 36rCos
r[r(9 5Cos
2
) 36Cos] = 0 r = 0, y r(9 5Cos
2
) 36Cos = 0.
Por lo tanto
r =
36Cos
9 5Cos
2

es la ecuacin buscada. Ninguna parte de la curva se perdi al omitir el polo r = 0, ya que esta
ecuacin pasa por el polo.
Ejemplo 14.2 Encontrar la ecuacin cartesiana correspondiente a la curva polar:
a) r =
2
3Cos +Sen
; b) r =
1
2 Sen
; c) r
2
= Sen3.
Solucin
Haciendo los reemplazos correspondientes, obtenemos:
a)
_
x
2
+y
2
=
2
3x

x
2
+y
2
+
y

x
2
+y
2

_
x
2
+y
2
=
2
_
x
2
+y
2
3x +y
3x +y + 2 = 0.
b)
_
x
2
+y
2
=
1
2
y

x
2
+y
2

_
x
2
+y
2
=

_
x
2
+y
2
2
_
x
2
+y
2
y
2
_
x
2
+y
2
y = 1 4x
2
+ 3y
2
2y 1 = 0
c) Primero transformamos Sen3 en ngulos simples r
2
= (4Cos
2
1)Sen. Utilizando las
ecuaciones establecidas anteriormente, obtenemos:
x
2
+y
2
=
_
4x
2
x
2
+y
2
1
_

_
x
2
+y
2
(x
2
+y
2
)
2
=
y(3x
2
y
2
)

_
x
2
+y
2
(x
2
+y
2
)
5
2
= 3x
2
y y
3
.
14.2. Tarea
1. En los siguientes problemas se da un punto por un par de coordenadas polares. Site el
punto y determine sus coordenadas rectangulares:
a)
_
2,

2
_
; b)
_
4,
3
2
_
; c)
_
2,

6
_
; d)
_
8,
3
4
_
; e)
_
8,
5
4
_
.
2. En los problemas siguientes se dan las coordenadas rectangulares de un punto. En cada
caso encuentre todos los conjuntos posibles de coordenadas polares para el punto dado:
a) (1, 1); b) (

3,

3); c) (2, 2

3); d) (0, 5); e) (3, 3); f ) (4, 0);


g) (3, 4); h) (5, 12); i) (2, 1); j) (6, 3); k) (2, 5); l) (

2,

3).
3. Demuestre que los puntos (r, ) y (r, ) son simtricos con respecto al eje X.
CAPTULO 14. CURVAS DADAS EN COORDENADAS POLARES 616
4. Demuestre que los puntos (r, ) y (r, ) son simtricos con respecto al eje Y .
5. Qu puede decir acerca de la simetra del par de puntos (r, ) y (r, )?
6. Demuestre que la recta vertical x = A tiene a rCos = A como ecuacin en coordenadas
polares.
7. Demuestre que la recta horizontal y = B tiene a rSen = B como una ecuacin en coor-
denadas polares.
8. Obtenga una ecuacin apropiada en coordenadas polares para la circunferencia x
2
+y
2
=
2B.
9. Una ciudad B est localizada 60 kilmetros al este y 90 kilmetros al sur de A. Una estacin
meteorolgica en A detecta que una fuerte tormenta se ha desatado en B. Qu distancia
hay entre la tormenta y la estacin meteorolgica?
10. En un experimento sobre orientacin y navegacin, se liberan algunas palomas mensajeras
a 85 km de su jaula. Cul es la distancia oeste y norte desde el punto de liberacin?
11. Una abeja exploradora descubre una fuente de miel a medioda que es cuando estos ani-
males usan coordenadas polares para orientarse. La fuente se localiza 800 m al este y 1250 m
al sur de la colmena. Cules sern las coordenadas polares que la abeja proporcione a sus
compaeras al llegar a la colmena?
12. Expresar la ecuacin en coordenadas polares:
a) 4x
2
+ 3y
2
+ 6y = 9;
b) x
2
+y
2
= 2x;
c) y
2
(1 x) = x
3
;
d) 2xy = x
2
y
2
;
e) x
2
+y
2
4y = 0;
f ) (x
2
+y
2
)x = y
2
;
g) x
4
y
4
= 2xy;
h) (x
2
+y
2
)
2
= 2a
2
xy;
i) 4y y
2
= x
2
;
j) y
2
3x
2
= x;
k) x(x
2
+y
2
) = a(3x
2
y
2
);
l) (x
2
+y
2
)
2
= 2(x
2
y
2
);
m) (2a x)y
2
= x(x a)
2
.
13. Expresar la ecuacin en coordenadas rectangulares:
a) 10rCos
2
r = 9sen;
b) r = a(4Cos Sec);
c) r(3Cos +Sen) = 2;
d) r = 9TanSec;
e) r(2 Sen) = 1;
f ) r
2
= Tan2;
g) r = SecTan
2
;
h) r = 4CotCsc;
i) 2ar r
2
Cos = r
2
Cos;
j) 4r + 5rSen
2
= 36Cos;
k) r = 2aCos + 2bSen.
CAPTULO 14. CURVAS DADAS EN COORDENADAS POLARES 617
14.3. Grca en coordenadas polares
Como sucede en las coordenadas rectangulares, la grca de una ecuacin F(r, ) = 0, es por
denicin el conjunto de todos los puntos P(r, ) cuyas coordenadas polares satisfacen la ecuacin.
Aqu el punto P tiene varios pares distintos de coordenadas, pero P estar en la grca slo si
uno de sus diferentes pares de coordenadas satisface la ecuacin. En muchos casos la ecuacin
F(r, ) = 0 puede resolverse explcitamente para determinar r o , dando r = f() o bien = g(r).
En cualquiera de estos casos es fcil trazar la grca. Por ejemplo, en el de la ecuacin r = f(),
simplemente seleccionamos una sucesin de valores para y se determina el valor correspondiente
de r.
Denicin 14.1 Grca polar
Una grca polar de una ecuacin que incluye coordenadas polares r y es el conjunto de todos los
puntos (r, ) en el plano que tiene por lo menos un conjunto de coordenadas polares que satisfacen
la ecuacin dada.
El trabajo necesario para trazar una curva cuya ecuacin se da en coordenadas polares se reduce
a menudo si empleamos la simetra y las intersecciones de la curva. Daremos pruebas de simetra
con respecto al eje polar, el polo, y a la recta = /2. La simetra puede ser a menudo una ayuda
para hacer la grca de una ecuacin polar. La grca ilustra tres tipos de simetra:
Simetra con respecto a la recta = /2;
Simetra con respecto al eje polar;
Simetra con respecto al polo.
La grca de la ecuacin polar es:
1. Simtrica con respecto a la recta = /2 si reemplazando (r, ) por (r, ) se produce
una ecuacin equivalente.
2. Simtrica con respecto al eje polar si reemplazando (r, ) por (r, ) se produce una
ecuacin equivalente.
3. Simtrica con respecto al polo si reemplazando (r, ) por (r, ) o cuando es reemplazada
por +, resulta una ecuacin equivalente.
Consideremos una recta que pasa por el punto A(a, 0) y que es perpendicular al eje polar o a
su prolongacin. Su ecuacin cartesiana viene dada por x = a. Por tanto, rCos = a es la ecuacin
polar de una recta perpendicular al eje polar. Cuando a es positivo, la recta est a la derecha del
polo; cuando a es negativo, la recta se encuentra a la izquierda del polo.
CAPTULO 14. CURVAS DADAS EN COORDENADAS POLARES 618
Consideremos ahora una recta l que pasa por el punto A(a, /2) y que es paralela al eje polar.
Su ecuacin cartesiana es y = a. Por tanto, su ecuacin polar es rSen = a. As, esta es la ecuacin
polar de una recta paralela al eje polar. Si a es positivo, la recta se encuentra por arriba del eje
polar; si a es negativo, la recta se encuentra por debajo del eje polar. Cualquier recta que pasa
por el polo tiene una ecuacin polar muy sencilla = k donde la constante k es la medida del
ngulo que forma la recta con el eje polar; un punto pertenece a esta recta si y slo si el valor de
su segunda componente polar es k.
Consideremos una circunferencia con centro en el polo y radio k. Entonces, un punto P(r, )
pertenece a la circunferencia si, y slo si [OP[ = k. Ahora bien, [OP[ = r. Por lo tanto r = k
son las ecuaciones polares de una circunferencia con centro en el polo y radio igual a k.
Consideremos ahora una circunferencia con centro en el
punto A(a, 0) y radio igual a [a[. La curva pasa por el polo.
Denotemos por Q su segunda interseccin con el eje po-
lar. Sea P(r, ) un punto cualquiera de la circunferencia.
Entonces, OPQ es un ngulo rectngulo por estar inscrito
en una semicircunferencia. Por lo tanto r = 2aCos es la
ecuacin de una circunferencia de radio [a[ que tiene su cen-
tro sobre el eje polar o sobre la prolongacin de ste, y que pasa por el polo. Cuando a es positivo
en la ecuacin, la circunferencia se encuentra a la derecha del polo. Cuando a es negativo, la cir-
cunferencia se halla a la izquierda del polo. Del mismo modo r = 2aSen es la ecuacin polar de
una circunferencia de radio [a[ que tiene su centro sobre el eje /2 o sobre su prolongacin, y que
pasa por el polo.
Recordemos que, conforme a la denicin de cni-
ca dada en el curso de geometra analtica, la parbo-
la, la elipse y la hiprbola tienen la propiedad de
que la razn existente entre la distancia de un pun-
to cualquiera de la curva al foco y la distancia entre
dicho punto y la directriz es siempre igual a la excent-
ricidad. Las ecuaciones ms sencillas de las cnicas se
obtienen cuando se coloca el foco y se toma por direc-
triz una perpendicular al eje polar, como se muestra
en la gura. Supongamos que la directriz est a la
izquierda del foco, y sea D su interseccin con la prolongacin del eje polar. Denotemos por d la
distancia entre el foco y la directriz; es decir, [DO[ = d. Sea P(r, ) un punto cualquiera de la
cnica. Trcense PQ y PR perpendiculares, respectivamente, al eje polar (o a su prolongacin)
y a la directriz. Por denicin, P pertenece a la cnica si y slo si [OP[ = e[RP[ donde e es la
excentricidad. Supongamos, por el momento, que P est a la derecha de la directriz y sobre el
lado nal de . Entonces, tanto RP como r son positivos y, por tanto, [RP[ = RP y [OP[ = r.
Sustituyendo en [OP[ = e[RP[ tenemos r = e(RP).
Ahora bien
RP = DQ y DO +OQ = DQ
y tambin
DO = d y OQ = rCos
De donde
RP = d +rCos.
CAPTULO 14. CURVAS DADAS EN COORDENADAS POLARES 619
Sustituyendo en r = e(RP) llegamos a
r = e(d +rCos)
con lo cual
r =
ed
1 eCos
(14.1)
La grca de (14.1), es una parbola si e = 1, y una elipse si 0 < e < 1. Ambas curvas caen, por
completo, a la derecha de la directriz dada, pero, supuesto que P est a la derecha de la directriz
y que r es positivo, la grca de (14.1), para e > 1 corresponder a slo una de las ramas de una
hiprbola. Puede probarse, sin embargo, que, si r tomara valores negativos y positivos, la grca
de (14.1), para e > 1, corresponder a toda la hiprbola. Del mismo modo, si la directriz est a la
derecha del correspondiente foco, la ecuacin polar de la cnica sera
r =
ed
1 +eCos
La ecuacin polar de una cnica con foco en el polo y cuya directriz es perpendicular al eje polar
es
r =
ed
1 eCos
donde e es la excentricidad, y d es la distancia entre el foco y la correspondiente directriz. Se toma
el signo positivo cuando la directriz dada se encuentra a la derecha del foco, y el signo negativo
cuando la directriz se encuentra a la izquierda del foco.
Ecuaciones polares igualmente sencillas se obtienen si un foco coincide con el polo y la directriz
es paralela al eje polar. La ecuacin polar de una cnica con foco en el polo y cuya directriz es
paralela al eje polar es
r =
ed
1 eSen
donde e es la excentricidad, y d es la distancia entre el foco y la correspondiente directriz. Se toma
el signo positivo cuando la directriz dada est por arriba del eje polar, y el signo negativo cuando
la directriz se encuentra por debajo del eje polar.
Ejemplo 14.3 Despus de hacer un anlisis general, trazar la grca de la curva:
r = 3Cos2
Solucin
La ecuacin r = 3Cos2, podemos expresar f(, r) = r 3Cos2 = 0. Como
0

15

30

45

60

75

90

2 0

30

60

90

120

150

180

Cos2 1

3
2
1
2
0
1
2

3
2
1
r 3
3

3
2
3
2
0
3
2

3

3
2
3
f(, r) = r 3Cos2()
= r 3Cos2
= f(, r).
CAPTULO 14. CURVAS DADAS EN COORDENADAS POLARES 620
la curva es simtrica con respecto al eje polar. Como
f( +, r) = r 3Cos2( +)
= r 3Cos(2 + 2)
= r 3Cos2Cos2 + 3Sen2Sen2
= r 3Cos2
= f(, r).
La curva es simtrica con respecto al polo. Como
f( , r) = r 3Cos2( )
= r 3Cos(2 2)
= r 3Cos2Cos2 3Sen2Sen2
= r 3Cos2
= f(, r).
La curva es simtrica con respecto a la recta = /2. Las intersecciones se calculan de la siguiente
manera:
f(0

, r) = r 3Cos0

= r 3 = 0 para r = 3 y (0

, 3)
es una interseccin
f(90, r) = r 3Cos180

= r + 3 = 0 para r = 3 y
(90

, 3) es una interseccin
f(180

, r) = r3Cos360

= r3 = 0 para r = 3 y (180

, 3)
es una interseccin
f(270

, r) = r 3Cos540 = r + 3 = 0 para r = 3 y
(270, 3) es una interseccin
Como podemos darnos cuenta, la curva es simtrica con
respecto al polo, al eje polar y a la recta = /2 y sus
intersecciones son (0

, 3), (90, 3), (180, 3) y (270

, 3).
Dado que Cos2 es una funcin par, basta elaborar una
tabla para 0, que muestre los valores correspondientes
de , 2, Cos2 y r, y localizaremos los puntos (, r). Esta
curva se llama rosa de cuatro ptalos.
14.4. Tarea
1. Despus de hacer un anlisis general, trazar la grca de la curva:
a) r = 2(Sen 1);
b) r = 2Sen + 3;
c) r = 3Sen + 2;
d) r = 3Cos 2;
e) r
2
= 4Sen;
f ) r2 = 9Cos;
g) r = 2a(Cos + 1);
h) r = 2a(Sen + 1);
i) r = 2Cos 4;
j) r = Csc;
k) r = Cot;
l) r = Cos(2n + 1);
m) r = Sen(2n + 1);
n) r = 2aSen +b;
o) r
2
= Sen;
p) r = Sen(2n);
q) r = Cos
3
2
;
r) r = Cos
2
3
;
s) r = Sen
2
4
;
t) r
2
= Cos.
2. Una circunferencia tiene su centro sobre el eje o lnea polar y pasa por el origen. Encuentre
una ecuacin apropiada para el lugar geomtrico de los puntos medios de las cuerdas que
CAPTULO 14. CURVAS DADAS EN COORDENADAS POLARES 621
pasan por el origen. Identique la curva.
3. Pruebe que la grca de r = 2ASen +2BCos puede ser una circunferencia que pasa por
el origen o un solo punto. Encuentre el centro y el radio de la circunferencia.
4. Un segmento rectilneo de longitud ja 2a se desliza de manera que un extremo se encuen-
tra siempre sobre el eje X, y el otro se halla siempre sobre el eje Y . Obtenga una ecuacin en
coordenadas polares para el lugar geomtrico de los puntos P que son las intersecciones del
segmento mvil con una recta que pasa por el origen y es perpendicular al segmento. Trace
la curva.
5. Hallar la ecuacin de cada conjunto de puntos descritos a continuacin. Comprobar cada
lnea, circunferencia y ecuacin cnica aplicando la ecuacin general apropiada:
a) Una recta paralela al eje polar y 5 unidades arriba; 5 unidades abajo de l.
b) Una recta cuya normal desde el polo tiene 4 unidades de longitud y forma un ngulo de
60

con el eje polar.


c) Una circunferencia con centro en (90

, 6) y radio 6.
d) Una circunferencia con centro en (240

, 2) y radio 7.
e) Una parbola con foco en el polo y directriz perpendicular al eje polar y 3 unidades a la
derecha del eje normal.
f ) Una parbola con foco en el polo y directriz paralela y 4 unidades abajo del eje polar.
g) La curva tal que la distancia de cada punto sobre ella desde el polo es igual al triple del
seno del doble del ngulo vectorial.
h) La parbola con vrtice en el polo y foco en (0

, a). Comprobar la ecuacin transforman-


do y
2
= 4ax a forma polar.
6. Demuestre que la grca de r(ACos +BSen) = C es siempre una recta en tanto que A
y B no sean cero.
7. Una cuerda focal de una cnica es una cuerda que pasa por uno de los focos de la curva.
Considere que el foco F divide a la cuerda en dos segmentos de longitudes d
1
y d
2
. Demuestre
que para una elipse o una parbola jas, 1/d
1
+1/d
2
es una constante. Qu es tal constante?
8. Trace la curva r = aSenk para 0 /k. A partir de esta parte de la grca deduzca
el hecho de que si k es un entero la grca completa es una rosa. Si k es un entero par la
rosa tiene 2k ptalos, pero si k es un entero impar la rosa slo tendr k ptalos. Demuestre
la misma aseveracin para la grca de r = aCosk, comprobando que es congruente con la
grca de r = aSenk. Sugerencia: Sustituya por + 3k en r = aCosk.
9. Obtenga una ecuacin en forma polar para cada una de las grcas descritas a contin-
uacin:
a) Una lnea que pasa por el punto O y tiene pendiente 1;
b) Una lnea que pasa por el punto (1, /2), con pendiente -1;
c) Una lnea paralela a la del literal a) y que pasa por el punto polar (-1, 0);
d) Una lnea perpendicular a la lnea del literal b) y que pasa por el punto (2, /3);
CAPTULO 14. CURVAS DADAS EN COORDENADAS POLARES 622
e) Un crculo con centro en O y radio 5;
f ) Un crculo con centro en (1, /4), con radio 4;
g) Una parbola, cuya ecuacin rectangular es y = x
2
;
h) Una parbola, cuya ecuacin rectangular es x
2
1 = 2y.
10. Si C
1
es la curva r = f() y C
2
la curva r = kf(), donde k es una constante diferente de
cero, se dice que la curva C
2
es semejante a la C
1
con el origen como centro de semejanza:
a) Demuestre que si C
1
es una recta, entonces tambin C
2
lo es.
b) Pruebe que si C
1
es una circunferencia que pasa por el origen, tambin lo ser C
2
.
11. Demuestre que en el caso de una hiprbola la aseveracin del problema anterior es falsa a
menos que slo se considere una rama. Por ejemplo, considere la hiprbola r =
15
1 4Cos
haciendo primero = 0 y despus = /3.
12. Clasicar y trazar la curva:
a) r =
6
1 +Cos
;
b) r =
5
1 +Sen
;
c) r =
3
2 +Sen
;
d) r =
4
3 +Cos
;
e) r =
5
4 Sen
;
f ) r =
3
1 + 2Cos
;
g) r =
2
1 3Cos
;
h) r =
4
1 3Sen
;
i) r =
1
1 Sen
.
14.5. Interseccin de curvas
Al resolver un par de ecuaciones en coordenadas rectangulares, encontramos aquellos puntos
que tienen iguales valores de x e iguales valores de y. Como las coordenadas rectangulares tienen
exactamente un par de coordenadas para cada punto, y exactamente un punto para cada par de co-
ordenadas, obtendremos todos los puntos de interseccin de dos curvas si resolvemos sus ecuaciones
simultneamente, obtendremos aquellos puntos que tienen iguales valores de r e iguales valores de
, pero no obtendremos necesariamente todos los puntos de interseccin de las curvas.
Esto es cierto, porque un punto de interseccin de dos curvas puede tener un par de coorde-
nadas respecto a una de las curvas, y otro par relativo a la otra curva. Hay modos de encontrar
todos los puntos de interseccin de dos curvas cuyas ecuaciones estn en forma polar, pero no los
presentaremos aqu.
Para resolver dos ecuaciones polares simultneamente y obtener los puntos que tengan los
mismos valores de r y los mismos valores de en ambas curvas, podemos empezar resolviendo
cada ecuacin para r, continuar resolviendo la ecuacin en obtenida poniendo en ecuacin las dos
expresiones de r, y nalmente, sustituyendo los valores obtenidos para en cualquier expresin de
r para obtener cada valor correspondiente de r.
Ejemplo 14.4 Resolver el sistema de ecuaciones:
_
r
2
= 4Sen
r
2
= 4Cos
CAPTULO 14. CURVAS DADAS EN COORDENADAS POLARES 623
Solucin
Dividiendo cada miembro de la primera ecuacin por el miembro correspondiente de la segunda
ecuacin, tenemos
1 = Tan = ArcTan1 = 45

, 225

.
Obtenemos cada valor correspondiente de r por medio de la primera ecuacin. As,
r
2
= 4Sen45

= 2

2 r = 1, 68
si = 45

y r es imaginaria si = 225

. Por consiguiente, los puntos sobre las dos curvas que tienen
valores iguales de r y valores iguales de son P(1, 68, 45

) y Q(1, 68, 45

) y estas son soluciones


del par de ecuaciones.
No obstante, puede haber otros puntos en que las curvas
se intersequen. Evidentemente, las curvas se cruzan entre s en
un punto de cada cuadrante y en el origen, a pesar del he-
cho de que solamente hemos encontrado dos polinomios para
el par de ecuaciones. Estos cruces no son soluciones del par de
ecuaciones, puesto que ninguno de ellos tiene las mismas co-
ordenadas de la primera ecuacin que de la segunda ecuacin.
Es decir, las coordenadas del cruce en el segundo cuadrante
son (1, 68, 135

) y (1, 68, 315

) de la primera ecuacin y de la
segunda ecuacin. Finalmente, las curvas se interceptan en el
origen, ya que r
2
= 4Sen es cero si = 0

y r
2
= 4Cos es
cero si = 90

.
14.6. Tarea
1. Resolver el sistema de ecuaciones y trazar sus grcas en el mismo conjunto de ejes:
a)
_
r
2
= 4Sen,
r
2
= 4Cos.
b)
_
r = 3Cos,
r = 3Sen.
c)
_
r = Sen,
r
2
= Sen2.
d)
_
rCos = 2,
rSen = 2

3.
e)
_
r
2
= 2Cos,
r = 2(1 +Cos).
f )
_
r = aCos,
rCos = a.
g)
_
r = 2Sen2,
r = 2Cos2.
h)
_
r = a(1 +Cos),
r =
a
1 Cos
.
i)
_
r = a,
r = 4aCos.
j)
_
r = aCos2,
4rCos = a

3.
k)
_
r = aSen,
rSen = a.
l)
_
r = 3Cos,
r = 1 +Cos.
m)
_
r = 2Cos,
r
2
= 4Cos.
n)
_
_
_
r = Cos,
r =
2
3 + 2Cos
.
o)
_
r = 1 Cos,
r = 3Cos

2
.
p)
_
r = 5Sen,
r = 2 +Sen.
q)
_
rSen = 2,
r = 4Sen.
r)
_
rCos = 1,
r = 2Cos + 1.
Captulo 15
Nmeros complejos
15.1. Deniciones
Desde tiempo inmemorial, los nmeros han sido asociados con longitudes, y debi haber sido
una terrible sorpresa el descubrir que algunas longitudes no estn dadas por nmeros racionales.
Lo anterior no se puede evitar, ya que la diagonal d de un cuadrado de lado igual al que satisface
d
2
= 2, pero no existe nmero racional x que satisfaga x
2
= 2. La tenaz intuicin nos haca sentir
que cada longitud debera estar expresada por un nmero; esto condujo con el tiempo a una exten-
sin de los nmeros racionales para formar los nmeros reales, los cuales nos permiten resolver la
ecuacin x
2
= 2, y en donde en realidad obtengamos todas las longitudes que la geometra requiere.
Otro problema de extensin se presenta por la ecuacin cuadrtica ax
2
+bx +c = 0, la cual se
resuelve mediante la expresin
x =
b

b
2
4ac
2a
En vista de que un nmero negativo no tiene raz cuadrada real, esta frmula no nos da un nmero
real x cuando b
2
4ac < 0. No obstante, asignaramos un signicado a

b
2
4ac si, por ejemplo,
b
2
4ac = 1? o, simplemente diramos que no existen races en este caso? La respuesta es, una
vez ms ampliar el sistema numrico e incluir expresiones semejantes a

1. Esta expresin es
llamada un nmero imaginario, y la ampliacin del sistema contiene combinaciones de stos con
los nmeros reales; dicha combinacin se llama sistema de los nmeros complejos.
Los nombres nmero complejo y nmero imaginario sugieren un cierto aire de misterio, en
notorio contraste con los conocidos nmeros reales. El misterio se debe, muy probablemente, al
hecho de que los nmeros reales satisfacen una necesidad geomtrica muy clara, mientras que los
nmeros complejos e imaginarios son introducidos por razones puramente algebraicas.
De lo anterior, nos sera muy natural concluir que los nmeros complejos no son muy utilizados
en aplicaciones de las matemticas a situaciones fsicas reales. No obstante, la realidad es que estos
nmeros constituyen una extraordinaria y potente herramienta tanto en las matemticas puras
como en sus aplicaciones. Este capitulo dar solamente una visualizacin limitada de tales aplica-
ciones y de los resultados que de ellas se deriven.
A los estudiantes se les ensea cmo realizar operaciones con estos nmeros imaginarios por un
procedimiento puramente formal; pero no se da ninguna explicacin adecuada de los fundamentos
624
CAPTULO 15. NMEROS COMPLEJOS 625
de estas operaciones con smbolos que, por s solos, no tienen ningn signicado. Probablemente este
procedimiento se justica, por cuanto a la edad en que los estudiantes se encuentran por primera
vez estos nmeros imaginarios, no han desarrollado an una suciente facultad de abstraccin como
para entender lo que realmente estn tratando, y slo puede esperarse que adquieran una cierta
destreza en las manipulaciones formales.
En vista de la insuciencia del conjunto de los nmeros reales, lo ampliaremos creando nuevos
entes, cada uno se compone de nuevos entes, cada uno de los cuales se compone de un par de
nmeros reales, de igual modo que todo nmero racional se compone con un par de nmeros nat-
urales.
Introduciremos el nuevo nmero i, la unidad imaginaria, que posee la propiedad de que su
cuadrado sea igual a -1: i
2
= 1. Admitiremos sin demostracin que se pueden introducir los
nuevos nmeros, llamados nmeros complejos, de manera que unindolos con los ya conocidos
nmeros reales, obtendremos un conjunto de nmeros con los que se pueden realizar las opera-
ciones aritmticas segn las reglas ordinarias, y, adems, entre los nuevos nmeros se tendr el
nmero i. La denicin lgica de los nmeros complejos y de las reglas para las operaciones con
stos, es una de las dicultades fundamentales de este tema.
Por ejemplo, los nmeros complejos se denen a menudo como: al nmero complejo se le llama
nmero de la expresin a + bi, donde a y b son nmeros reales e i =

1. En realidad, esta
denicin es incomprensible, porque el signo del radical se utiliza para designar la raz cuadrada
aritmtica de un nmero real positivo.
Los estudiantes pueden formular las deniciones principales de los nmeros complejos de la
siguiente manera:
Denicin 15.1 Nmero complejo
Llmase nmero complejo a un ente abstracto representado z = a + bi, donde a y b son nmeros
reales e i el smbolo especial. En este caso a se llama parte real del nmero z; bi, parte imaginaria
de este nmero; i unidad imaginaria denida por i
2
= 1.
Toda la teora de los nmeros complejos puede ser desarrollada aritmticamente sin utilizar
representacin geomtrica alguna; pero es til mostrar que la creacin de estos nuevos nmeros
ha sido en parte motivada por la necesidad de poder representar numricamente los puntos de
un plano, de igual modo que los nmeros reales surgieron en la mente de los matemticos para
poder representar todos los puntos de una recta. A cada nmero complejo a + bi se puede poner
en correspondencia el punto P(a, b) del plano de coordenadas y, a la inversa, a cada punto P(a, b)
del plano, el nmero complejo a +bi.
La correspondencia establecida de semejante forma es biunvoca. Ella proporciona la posibil-
idad de considerar los nmeros complejos como puntos del plano de coordenadas. Este recibe el
nombre de plano complejo. El eje de abscisas lleva el nombre de eje real (en l se disponen los
puntos que corresponden a los nmeros reales), el de ordenadas, eje imaginario (en l yacen los
puntos correspondientes a los nmeros imaginarios).
El nmero complejo z = a + bi se admite representarlo por un punto P en el plano; la abscisa
de este punto es igual a la parte real a, la ordenada es igual a b, es decir, al coeciente de la unidad
imaginaria. A todo nmero complejo corresponde un punto determinado del plano, y, viceversa, a
cada punto del plano corresponde un nmero complejo determinado. De este modo, se establece
CAPTULO 15. NMEROS COMPLEJOS 626
una correspondencia biunvoca entre los puntos del plano de coordenadas X0Y y el conjunto de
nmeros complejos. A los puntos del eje 0X corresponden nmeros reales b = 0; a los puntos del
eje de coordenadas 0Y corresponden los nmeros imaginarios.
Con frecuencia, es cmodo interpretar el nmero complejo a+bi
como el vector OP. A cada vector del plano con origen en el punto
O(0, 0) y extremo en el punto P(a, b) corresponde el nmero com-
plejo a +bi y viceversa. Al punto O(0, 0) le corresponde el vector
nulo. La correspondencia establecida entre un conjunto de nmeros
complejos, por un lado, y los conjuntos de puntos o vectores del
plano, por otro lado, permite denominar los nmeros complejos
puntos o vectores. Es decir, reciben el nombre de nmeros com-
plejos las expresiones de la forma a +bi, donde a y b son nmeros
reales, i, cierto smbolo, para las cuales las nociones de igualdad y operaciones de adicin y multi-
plicacin se introducen ms adelante.
Ejemplo 15.1 Obtenga polinomios cuyas races sean:
a) 1

2 i y 1 +

2 i; b) 3 2i y 1 3i; c)

2 3i y

2 + 3i.
Solucin
a) Formamos los correspondientes factores:
[x (1

2 i)][x (1 +

2 i)] = [(x 1) +

2 i][(x 1)

2 i]
= (x 1)
2
(

2 i)
2
= x
2
2x + 1 2i
2
= x
2
2x + 1 + 2
= x
2
2x + 3.
b) Formamos los correspondientes factores:
[x (3 2i)][x (1 3i)] = [(x 3) + 2i][(x 1) + 3i]
= (x 3)(x 1) + 3(x 3)i + 2(x 1)i + 6i
2
= x
2
4x + 3 + 3xi 9i + 2xi 2i 6
= x
2
(4 5i)x (3 + 11i).
c) Formamos los correspondientes factores:
[x (

2 3i)][x (

2 + 3i)] = [(x

2) + 3i][(x

2) 3i]
= (x

2)
2
(3i)
2
= x
2
2

2 x + 2 9i
2
= x
2
2

2 x + 2 + 9
= x
2
2

2 x + 11.
Ejemplo 15.2 Escriba en forma binmica el nmero complejo z = 3(Cos210

+iSen210

).
Solucin
Haciendo las transformaciones del caso, obtenemos
3(Cos210

+iSen210

) = 3
_

3
2

1
2
i
_
=
3

3
2

3
2
i.
CAPTULO 15. NMEROS COMPLEJOS 627
Denicin 15.2 Nmeros complejos iguales
Dos nmeros complejos z
1
= a +bi y z
2
= c +di se consideran iguales, si y slo si a = c y b = d.
Por denicin se supone tambin que a + 0i = a, 0 + bi = bi, 0 + 0i = 0. Los nmeros complejos
que no satisfacen esta condicin de igualdad se llaman desiguales.
Si a = 0, b ,= 0, el nmero complejo a + bi se convierte en un nmero imaginario puro bi; b se
llama coeciente de la unidad imaginaria. Si b = 0, el nmero complejo a +bi deviene un nmero
real igual a a. El conjunto de nmeros complejos contiene, como parte, tanto todos los nmeros
reales como todos los nmeros imaginarios puros; en otras palabras, los nmeros reales, as como
los nmeros imaginarios son casos particulares de nmeros complejos.
Teniendo en cuenta que la igualdad de nmeros reales es reexiva, simtrica y transitiva; de
acuerdo a esta denicin, resulta inmediato que la igualdad de nmeros complejos tambin posee
estas propiedades.
Ejemplo 15.3 Cunto debe valer x, real, para que (3 xi)
2
sea imaginario puro?
Solucin
Descomponemos la expresin
(3 xi)
2
= 9 6xi +x
2
i
2
= 9 6xi x
2
.
Para que sea imaginario puro, tenemos que hacer
9 x
2
= 0 (3 x)(3 +x) = 0 x = 3 y x = 3.
Ejemplo 15.4 Resolver la ecuacin
x
2
+ 2x + 2 = 0
Solucin
aplicando a la ecuacin dada la regla conocida de determinacin de las races de una ecuacin
cuadrtica, obtenemos
x
1,2
= 1

1 = 1 i
La ecuacin dada no tiene races reales; sus races son complejas conjugadas, es decir, x
1
= 1 +i
y x
2
= 1 i.
Denicin 15.3 Mdulo de un nmero complejo
Lleva el nombre de mdulo del nmero complejo z = a +bi, la longitud del vector que corresponde
a dicho nmero. Es decir el nmero real r =

a
2
+b
2
.
El nmero r es positivo y se anula slo cuando a = 0, b = 0. El mdulo de un nmero complejo
se designa con dos lneas verticales a cada lado del nmero. Es decir
[a +bi[ =
_
a
2
+b
2
En el caso particular cuando b = 0, tendremos
[a + 0i[ =
_
a
2
+ 0
2
= [a[
CAPTULO 15. NMEROS COMPLEJOS 628
es decir, el modulo de un nmero real es el valor absoluto de ese nmero. Por eso, el mdulo de un
nmero complejo se llama tambin valor absoluto de ese nmero.
De la denicin resulta inmediato que
Re(z) [z[ es decir x
_
x
2
+y
2
Im(z) [z[ es decir y
_
x
2
+y
2
Todos los nmeros complejos de mdulo igual a la unidad se representan por los puntos de una
circunferencia unitaria con centro en el origen de coordenadas.
Durante la solucin de muchos problemas se utiliza a menudo la interpretacin geomtrica de
los nmeros complejos como puntos de un plano.
Ejemplo 15.5 Sea z ,= 1 un nmero complejo. Demuestre que:
a) Si [z[ = 1, el nmero
z 1
z + 1
es puramente imaginario;
b) Si el nmero
z 1
z + 1
es puramente imaginario, [z[ = 1.
Solucin
Sean z = a + bi y z ,= 1. Entonces es claro que z + 1 ,= 0 y la expresin
z 1
z + 1
tiene sentido.
El nmero
z 1
z + 1
es el cociente de la divisin de dos nmeros complejos, por cuya razn su forma
algebraica ser:
z 1
z + 1
=
(a 1) +bi
(a + 1) +bi
=
[(a 1) +bi][(a 1) bi]
(a +b)
2
+b
2
=
a
2
+b
2
1
(a + 1)
2
+b
2
+i
2b
(a + 1)
2
+b
2
.
De aqu se deduce que si [z[ =

a
2
+b
2
= 1, entonces a
2
+ b
2
1 = 0, es decir, el nmero
z 1
z + 1
es puramente imaginario y con ello la armacin a) queda demostrada.
Demostremos la armacin b). Sea
z 1
z + 1
un nmero puramente imaginario. Entonces
a
2
+b
2
1
(a + 1)
2
+b
2
=
0, de donde a
2
+ b
2
1 = 0, es decir, [z[ =

a
2
+b
2
= 1 y con ello la armacin b) queda de-
mostrada.
Ejemplo 15.6 Hallar los mdulos de los nmeros complejos
z = 2 i, w = 2

6 + 5i, u = i.
Solucin
Aplicando la denicin, obtenemos:
[z[ =
_
2
2
+ 1
2
=

5, [w[ =
_
(2

6)
2
+ 5
2
= 7, [u[ =
_
0
2
+ 1
2
= 1.
CAPTULO 15. NMEROS COMPLEJOS 629
Ejemplo 15.7 Dnde se encuentran los nmeros complejos z = a +bi para los cuales
log
1/2
[z 2[ > log
1/2
[z[?
Solucin
Hay que sealar, ante todo, que el primer miembro de nuestra desigualdad tiene sentido para todos
los nmeros complejos z, excepto z = 2, y el segundo miembro, para todos los nmeros z 0.
Por eso, las expresiones que forman nuestra desigualdad tienen sentido simultneo para todos los
nmeros complejos z, excepto z = 0 y z = 2. Precisamente entre estos nmeros hay que hallar la
solucin de la desigualdad.
Segn las propiedades de los logaritmos, para todos estos nmeros nuestra desigualdad es equiva-
lente a la siguiente:
[z 2[ < [z[.
Sabemos que a la igualdad [z 2[ < [z[ le satisfacen todos los nmeros complejos que se hallan en
la recta l, paralela al eje Oy, que pasa por el punto A(1, 0), ya que todos los puntos de esta recta
son equidistantes de dos puntos O(0, 0) y B(2, 0). Pero, nos hace falta hallar en el plano todos
aquellos puntos que estn ms prximos al punto B(2, 0) que al punto O(0, 0).
Claro es que stos sern los puntos del plano que se encuen-
tran por el mismo lado de la recta l donde est el punto B. De
tal modo, todos los puntos del semiplano situados a la derecha
de la recta l satisfacen la condicin [z 2[ < [z[; los puntos de
la misma recta l se anulan.
Ahora recordemos que para obtener la solucin del problema
planteado es necesario eliminar el punto B(2, 0) de este semi-
plano situado a la derecha de la recta l. Pues, la condicin del
problema la satisfacen todos los puntos del plano que se en-
cuentran a la derecha de la recta paralela al eje Oy y la que
pasa por el punto (1, 0), excepto el punto (2, 0).
Ejemplo 15.8 Cul es el sentido geomtrico de la desigualdad [z[ < 1?
Solucin
El modulo del nmero complejo z, es decir, [z[, signica la distancia del origen de coordenadas
al punto z; ya que esta distancia es menor que 1, describimos del origen de coordenadas una
circunferencia de radio r = 1. Todos los puntos interiores del crculo representan los nmeros
complejos z, para los cuales [z[ < 1. Para los puntos de la circunferencia, tendremos que [z[ = 1.
Los puntos externos son nmeros complejos que satisfacen la desigualdad [z[ < 1.
Ejemplo 15.9 En un plano est dado un punto que corresponda el nmero complejo z = a+bi.
Donde se encuentran los puntos:
a) z + 1; b) z 2 +i?
Solucin
a) Ya que el nmero z + 1 = (a + 1) + bi, entonces el punto que presenta el nmero complejo
z +1, tendr las coordenadas (a +1, b), es decir, la ordenada ha quedado la misma y la abscisa ha
aumentado en 1. Por lo tanto, el punto z + 1 resulta del punto z desplazndose a la derecha en 1.
b) Ya que el nmero z 2 + i = (a 2) + (b + 1)i, entonces el punto que presenta el nmero
complejo z 2 +i, tendr las coordenadas (a 2, b + 1), es decir, la abscisa ha disminuido en 2 y
la ordenada ha aumentado en 1. Por lo tanto, el punto z 2 +i resulta del punto z desplazndose
a la izquierda en 2 y hacia arriba en 1.
CAPTULO 15. NMEROS COMPLEJOS 630
Ejemplo 15.10 Cmo estn dispuestos los puntos complejos que satisfacen la desigualdad
[z 2[ < 3?
Solucin
El modulo de la diferencia de los nmeros z y 2, es decir, [z 2[3, denota la distancia del punto
z
1
= 2 al punto z, que debe ser menor que 3. Por eso, desde el punto 2 trazamos una circunferencia
de radio igual a 3 unidades; los puntos complejos interiores de esta circunferencia son soluciones
de la desigualdad [z 2[ < 3.
Ejemplo 15.11 Sea [z[ = 1. Dnde se encuentran los puntos 1 + 2z?
Solucin Los puntos z que satisfacen la condicin [z[ = 1, se encuentran en una circunferencia
de radio 1 con el centro en el origen de las coordenadas. Todos los puntos 2z, donde [z[ = 1, se
encuentran en una circunferencia de radio 2 con el centro en el origen de las coordenadas. El punto
2z +1 resulta del punto 2z desplazndose a la derecha en 1. Por lo tanto, los puntos 1 +2z, donde
[z[ = 1, se hallan en una circunferencia de radio 2 con el centro en el punto (1, 0).
Ejemplo 15.12 Dnde se encuentran los puntos para los cuales 2 < [z[ < 3?
Solucin
Sabemos que los puntos que satisfacen la condicin [z[ = 2,
se encuentran en una circunferencia de radio 2 con el centro en
el origen de las coordenadas. Y los puntos para los cuales [z[ > 2
se encuentran ms distantes del origen de las coordenadas que
los puntos de esta circunferencia, es decir, fuera de la misma.
Anlogamente, los puntos que satisfacen la condicin [z[ < 3 se
encuentran en el interior de una circunferencia de radio 3 con
el centro en el origen de las coordenadas. Quiere decir que los
puntos que satisfacen la condicin 2 < [z[ < 3 se encuentran
dentro de un anillo acotado por las circunferencias concntricas
con el centro en el origen de las coordenadas y los radios r
1
= 2
y r
2
= 3.
CAPTULO 15. NMEROS COMPLEJOS 631
Ejemplo 15.13 Dnde se encuentran en el plano los puntos para los cuales [z[ = 1?
Solucin
Segn la interpretacin geomtrica del mdulo de un nmero complejo, todos los puntos que
representan los nmeros complejos con [z[ = 1, se encuentran a una misma distancia, igual a 1,
desde el origen de las coordenadas, es decir, se hallan (segn la denicin) en una circunferencia
de radio 1 con el centro en el origen de las coordenadas.
Ejemplo 15.14 Sea [z[ = 2. Dnde se encuentran los puntos 3z?
Solucin
Los puntos z que satisfacen la condicin [z[ = 2, se hallan en una circunferencia de radio 2 con el
centro en el origen de las coordenadas. El punto 3z se encuentra en el mismo radio que el punto
z, pero dista del origen de las coordenadas a una distancia tres veces mayor que el punto z. Por
eso, los puntos 3z, donde [z[ = 2, se encuentran en una circunferencia de radio 6 con el centro en
el origen de las coordenadas.
Ejemplo 15.15 Hallar las soluciones completas de la ecuacin
(1 i)
x
= 2
x
Solucin
supongamos que un nmero entero k es la solucin de esta ecuacin. Entonces, de la igualdad de
los nmeros complejos (1 i)
k
= 2
k
se deduce la igualdad de sus mdulos, es decir, [(1 i)
k
[ = 2
k
.
Tomando en consideracin [1 i[ =

2, segn la propiedad del mdulo, tenemos


[(1 i)
k
[ = [1 i[
k
= (

2)
k
= 2
k
2
.
En efecto, si k es la solucin de la ecuacin inicial, entonces 2
k
2
= 2
k
, lo que es posible slo cuando
k = 0.
Ahora vamos a comprobar mediante una sustitucin, si el nmero 0 es la solucin de la ecuacin
inicial. Recordando que el nmero complejo, distinto de cero, en la potencia cero es igual a 1, segn
la denicin, deducimos que x = 0 es la raz de la ecuacin inicial.
Ejemplo 15.16 Hallar todos los nmeros complejos z, para cada nmero real a 0, que
satisfaga la igualdad
[z[
2
2iz + 2a(1 +i) = 0.
Solucin
Representemos el nmero z en la forma algebraica: z = x+yi. entonces [z[
2
= x
2
+y
2
, y la ecuacin
tomar la forma
x
2
+y
2
2ix + 2y + 2a + 2ai
Si los trminos real e imaginario los igualamos a cero, obtenemos un sistema de ecuaciones
_
x
2
+y
2
+ 2y + 2a = 0
2x + 2a = 0
De ah se deduce que x = a, y para la determinacin de y tenemos una ecuacin de segundo grado
y
2
+ 2y +a
2
+ 2a = 0
con el parmetro a. Hallemos las races reales de esta ecuacin. segn se sabe, las races de una
ecuacin de segundo grado son reales si su discriminante no es negativo; por eso nuestra ecuacin
CAPTULO 15. NMEROS COMPLEJOS 632
lleva races reales slo para tales valores de a para los cuales = 1 a
2
2a 0. Para estos
valores de a obtenemos
y
1,2
= 1
_
1 a
2
2a.
De tal modo, si el nmero a satisface la desigualdad 1 a
2
2a 0, la ecuacin inicial tiene dos
soluciones
z
1,2
= a + (1
_
1 a
2
2a)i.
(Para 1 a
2
2a = 0 estas dos soluciones coinciden, es decir, hablando en rigor, para los valores
correspondientes de a, hay una sola solucin.) Para todos los dems valores de a la ecuacin inicial
no tiene soluciones.
Nos queda por sealar los lmites de variacin de a para los cuales existen soluciones. segn la
condicin a 0; adems de esto, hemos obtenido que a ha de satisfacer la condicin de la desigual-
dad 1 a
2
2a 0, o bien a
2
+2a 1 0, que es lo mismo. La solucin de la ltima desigualdad
es el intervalo 1

2 a 1 +

2; escogiendo de este intervalo los nmeros a 0, obtenemos


0 a 1 +

2.
La solucin denitiva puede ser escrita en la forma siguiente:
para 0 a < 1 +

2, z
1,2
= a + (1

1 a
2
2a)i,
para a = 1 +

2, z = 1 +

2 i,
para a > 1 +

2, no hay soluciones.
Ejemplo 15.17 resolver en nmeros complejos el sistema de ecuaciones
_
z
3
+w
7
= 0,
z
5
w
11
= 1.
Solucin
As, pa resolver este sistema vamos a deducir lo siguiente. De la primera ecuacin tenemos z
3
=
w
2
, de la segunda, z
5
=
1
w
11
. De ambas obtenemos, respectivamente, z
15
= w
35
y z
15
=
1
w
33
,
por consiguiente, w
35
=
1
w
33
, es decir, w
33
w
35
= 1.
De esta igualdad se desprende que [w
33
w
35
[ = 1. Basndonos en las propiedades de los mdulos y
los nmeros conjugados, obtenemos [w
33
w
35
[ = [w[
33
[w[
35
= [w
68
[ = 1, as que w = 1. Volviendo a
la ecuacin w
33
w
35
= 1 escribamos su primer miembro: (w
33
w
33
)w
2
= (ww)
33
w
2
= ([w[
2
)
33
w
2
=
w
2
(aqu se ha utilizado una propiedad ms de los nmeros recprocamente conjugados). De esta
manera, se llaga a la ecuacin w
2
= 1, es decir, w
1
= i, w
2
= i, de donde w
1
= i, w
2
= i.
Ahora calculamos los valores correspondientes de z. Si w = i, entonces, al tomar la primera
ecuacin del sistema inicial, hallamos que z
3
= i
7
= i, y tomando la segunda ecuacin, obten-
emos que z
5
=
1
(i)
11
=
1
i
= i. Dividiendo la segunda igualdad obtenida por la primera tenemos
z
2
= 1 y, como z3 = i, entonces z = i. en forma anloga hallamos que z = i en el caso de que
w = i.
Teniendo presente que durante la solucin hemos considerado no el sistema inicial sin o sus equiva-
lencias, es necesario vericar si los valores hallados de las incgnitas satisfacen realmente el sistema
mencionado. Esto se verica por una sustitucin directa que nos convence de que el sistema prop-
uesto tiene dos soluciones:
z
1
= i, w
1
= i y z
2
= i, w
2
= i.
CAPTULO 15. NMEROS COMPLEJOS 633
El nmero complejo se puede considerar tambin como un vector cuyo origen se encuentra en
un plano, en el origen del sistema de coordenadas, y su extremo se halla en el punto que representa
este nmero.
Mediante tal interpretacin es fcil explicar geomtricamente
las operaciopnes de adicin y sustraccin:
Si el vector OM
1
representa el nmero z
1
= a + bi, y OM
2
es el vector que representa el nmero z
2
= c + di, la suma
de estos vectores, OM
1
+ OM
2
, es la diagonal OM
3
del par-
alelogramo OM
1
M
3
M
2
. El extremo de esta diagonal, el punto
M
3
, tiene, evidentemente, las coordenadas (a +c, b +d). De tal
modo, el vector OM
3
es el que representa el nmero complejo
z
3
= z
1
+z
2
= (a +c) +(b +d)i. Si el vector OM representa el
nmero z, entonces el nmero z se representar por el vector
ON cuyo extremo es un punto simtrico al punto M respecto
al origen de las coordenadas.
Debido a esto, la operacin de sustraccin de los
nmeros complejos admite tambin una simple inter-
pretacin geomtrica. Justamente: ya que z
1
z
2
= z
1
+
(z
2
), entonces, en lugar del vector OM
2
, que representa
el nmero z
2
, vamos a analizar el vector OM
4
simtrico
al primero respecto al origen de las coordenadas. Suman-
do, como hicimos arriba, el vector OM
1
que representa
el nmero z
1
, y el vector OM
4
, obtendremos el vector
M
5
que representa la diferencia z
1
z
2
. Est claro que la
longitud del vector OM
5
es igual a la del vector M
2
M
1
,
es decir, a la longitud de la diagonal M
1
M
2
del paralel-
ogramo OM
1
M
3
M
2
. Porque la longitud del vector OM
5
es igual al mdulo de la diferencia z
1
z
2
, la longitud de
la diagonal M
1
M
2
es tambin igual a [z
1
z
2
[. Result
una simple interpretacin geomtrica del mdulo de la
diferencia de dos nmeros complejos: [z
1
z
2
[ es una
distancia entre los puntos M
1
y M
2
que representan los
nmeros complejos z
1
y z
2
. Esta interpretacin se aplica
a menudo para la solucin de los problemas.
Ejemplo 15.18 Dnde se encuentran los puntos que representan los nmeros complejos z
para las cuales [z 1[ = 2?
Solucin
Si el punto z es un punto incgnito, la distancia entre z y 1 es igual a 2. Pero, los puntos que
se hallan desde 1 a una distancia de 2, estn en la circunferencia. En ese caso, los puntos que
representan los nmeros para los cuales [z 1[ = 2, se encuentran en una circunferencia de radio
2 con el centro en el punto (1, 0).
Se puede razonar de otra manera. Designamos z1 = w. Entonces obtenemos una igualdad [w[ = 2.
Por consiguiente, los puntos w se encuentran en una circunferencia de radio 2 con el centro en el
origen de las coordenadas. Pues z = w + 1, los puntos z resultan de los puntos w desplazndose a
la derecha en 1. De tal modo, los puntos incgnitos se hallan en una circunferencia de radio 2 con
el centro en el punto (1, 0).
Ejemplo 15.19 Dnde se encuentran los puntos que representan los nmeros complejos z
CAPTULO 15. NMEROS COMPLEJOS 634
para los cuales [z + 2i[ 1?.
Solucin
Copiemos esta condicin as: [z (2i)[ 1. Esto quiere decir que la distancia desde los puntos z
hasta el punto 2i no es mayor que 1, es decir, todos los puntos que la satisfacen, se encontrarn
en el interior o en el lmite del crculo de radio 1 con el centro en el punto (0, -2), que representa
el nmero complejo 2i.
Ejemplo 15.20 Los nmeros complejos z satisfacen la condicin 1 < [z +23i[ < 2. Dnde
se encuentran los puntos que representan estos nmeros?
Solucin
Copiemos nuestra condicin as: 1 < [z (2 + 3i)[ < 2. Todos los puntos que satisfacen esta
condicin se encuentran dentro de un anillo acotado por las circunferencias concntricas con el
centro en el punto (-2, 3) y los radios r
1
= 1 y r
2
= 2.
Ejemplo 15.21 Los nmeros complejos z satisfacen la condicin [z i[ = [z + 2[. Dnde se
encuentran los puntos que representan estos nmeros?
Solucin
El mdulo [z i[ es la distancia entre los puntos z y un punto jo que representa el nmero i.
El mdulo [z + 2[ = [z (2)[ es la distancia entre los puntos z y un punto jo que representa el
nmero -2. La condicin del problema exige hallar los puntos para los cuales estas distancias sean
iguales. Es decir, la solucin del problema ser el lugar geomtrico de los puntos equidistantes de
dos puntos jos del plano: de un punto que representa el nmero complejo i, es decir, del punto
(0, 1), y de un punto que representa el punto -2, es decir, del punto (-2, 0).
Se sabe de la geometra que el lugar geomtrico es una recta perpendicular al segmento que une los
dos puntos sealados, y que pasa por su centro. Esto quiere decir que los puntos que representan
los nmeros complejos z, que satisfacen la condicin [z i[ = [z + 2[, se encuentran en la recta
perpendicular al segmento que une los puntos con las coordenadas (-2, 0) y (0, 1), y que pasa por
el centro de este segmento.
Ejemplo 15.22 Dnde se encuentran los puntos que representan los nmeros complejos z
para los cuales [z 1[ = [z 2[ = [z i[?
Solucin
El conjunto de puntos que satisfacen la condicin [z1[ = [z2[
es una recta que pasa por el centro del segmento AB, donde A(1, 0)
y B(2, 0), perpendicularmente a ste.
El conjunto de puntos que satisfacen la condicin [z1[ = [zi[ es
una recta que pasa por el centro del segmento AC, donde A(1, 0)
y C(0, 1), perpendicularmente a ste. Ahora est claro que a la
condicin [z 1[ = [z 2[ = [z i[ le satisface solamente un punto
D que es el de interseccin de estas dos rectas. Es fcil calcular
que las coordenadas de este punto sern x = y =
3
2
satisface la
condicin del problema.
Denicin 15.4 Nmero complejo conjugado
Los nmeros complejos z = a + bi y z = a bi se llaman conjugados; se diferencian slo por el
signo ante la parte imaginaria.
Un par de nmeros complejos conjugados se representan por los puntos P y Q, simtricos respec-
to del eje de abscisas. En la gura los puntos P y Q representan los nmeros complejos conjugados.
CAPTULO 15. NMEROS COMPLEJOS 635
Es del todo evidente que (z) = z, es decir, no slo el nmero z est conjugado con el nmero z,
sino tambin z est conjugado con z. Por esta razn los nmeros z y z se denominan conjugados
recprocamente.
Teorema 15.1 Sea z un nmero complejo, entonces se cumplen:
a) [ z[ = [z[;
b) [z[ = [z[.
Demostracin
a) Si z = a +bi, entonces
[ z[ = [ a bi[
=
_
(a)
2
+ (b)
2
=
_
a
2
+b
2
= [z[.
b) Si z = a +bi, entonces
[z[ = [a bi[
=
_
a
2
+ (b)
2
=
_
a
2
+b
2
= [z[.
Teorema 15.2 Sea z
1
y z
2
nmeros complejos, entonces se cumplen:
a) z
1
z
1
= [z
1
[
2
;
b) [z
1
z
2
[ = [z
1
[[z
2
[;
c) [z
1
+z
2
[ [z
1
[ +[z
2
[.
Demostracin
a) Si z
1
= a
1
+b
1
i, entonces
z
1
z
1
= (a
1
+b
1
i)(a
1
b
1
i)
= a
2
1
b
2
1
i
2
= a
2
1
+b
2
1
= [z
1
[
2
.
CAPTULO 15. NMEROS COMPLEJOS 636
b) Si z
1
= a
1
+b
1
i y z
2
= a
2
+b
2
i, entonces
[z
1
[[z
2
[ = [a
1
+b
1
i[[a
2
+b
2
i[
=
_
a
2
1
+b
2
1
_
a
2
2
+b
2
2
=
_
(a
2
1
+b
2
1
)(a
2
2
+b
2
2
)
=
_
a
2
1
a
2
2
+a
2
1
b
2
2
+b
2
1
a
2
2
+b
2
1
b
2
2
=
_
(a
2
1
a
2
2
2a
1
a
2
b
1
b
2
+b
2
1
b
2
2
) + (a
2
1
b
2
2
+ 2a
1
b
2
b
1
a
2
+b
2
1
a
2
2
)
=
_
(a
1
a
2
b
1
b
2
)
2
+ (a
1
b
2
+b
1
a
2
)
2
= [(a
1
a
2
b
1
b
2
) + (a
1
b
2
+b
1
a
2
)i[
= [(a
1
+b
1
i)(a
2
+b
2
i)[
= [z
1
z
2
[.
Otra forma de demostrar la propiedad, es la siguiente:
[z
1
z
2
[
2
= (z
1
z
2
)(z
1
z
2
)
= z
1
z
2
z
1
z
2
= (z
1
z
1
)(z
2
z
2
)
= [z
1
[
2
[z
2
[
2
.
Tomando la raz cuadrada positiva, se obtiene que [z
1
z
2
[ = [z
1
[[z
2
[.
c) Para demostrar esta propiedad, lo haremos anlogamente a la anterior
[z
1
+z
2
[
2
= (z
1
+z
2
)(z
1
+z
2
)
= (z
1
+z
2
)(z
1
+z
2
)
= z
1
z
1
+z
1
z
2
+z
2
z
1
+z
2
z
2
= [z
1
[
2
+z
1
z
2
+z
1
z
2
+[z
2
[
2
Como la suma de un nmero complejo y su conjugado es el doble de la parte real del nmero
complejo, entonces
z
1
z
2
+z
1
z
2
= 2Re(z
1
z
2
)
2[z
1
z
2
[
es decir
z
1
z
2
+z
1
z
2
2[z
1
[[z
2
[
= 2[z
1
[[z
2
[
por lo tanto, resulta que
[z
1
+z
2
[
2
[z
1
[
2
+ 2[z
1
[[z
2
[ +[z
2
[
2
= ([z
1
[ +[z
2
[)
2
tomando la raz positiva, obtenemos la desigualdad
[z
1
+z
2
[ [z
1
[ +[z
2
[
CAPTULO 15. NMEROS COMPLEJOS 637
15.2. Operaciones con los nmeros complejos
Las operaciones que denamos con los nmeros complejos han de satisfacer las dos condiciones
exigibles a toda generalizacin:
1. Han de cumplir las mismas leyes formales que las operaciones con nmeros reales.
2. Cuando los datos sean reales, los resultados deben coincidir con los hasta ahora obtenidos.
En este principio de permanencia de las leyes formales se han inspirado las deniciones que
se darn a continuacin, para las diversas operaciones con nmeros complejos. El problema sigu-
iente consiste en aprender a operar con estas expresiones: adicionarlas, multiplicarlas, etc. Somos
nosotros mismos los que tenemos que determinar cmo hacerlo.
Vamos a basarnos en la misma idea de la operacin aritmtica: el adicionar o el multiplicar
dos nmeros signica que, segn una regla, hay que construir un tercer nmero, llamado suma
o producto, respectivamente. De esa manera, para aprender a adicionar o multiplicar nuestras
expresiones, hace falta imponer unas reglas segn las cuales es necesario realizarlo.
Denicin 15.5 Suma de nmeros complejos
Se denomina suma de los nmeros complejos z
1
= a + bi y z
2
= c + di el nmero complejo
z = (a +c) +(b +d)i, obtenido sumando, respectivamente, las primeras y segundas entradas de los
dos nmeros dados, cuyas partes real e imaginaria son iguales respectivamente a la suma de las
partes reales e imaginarias de los sumandos z
1
y z
2
.
Notemos que en la ltima expresin el signo + entre los parntesis tiene un nuevo sentido: es el
signo de adicin de las expresiones formales. Sean z
1
= a+bi y z
2
= c +di dos nmeros complejos,
entonces son vlidas las transformaciones siguientes:
z
1
+z
2
= (a +bi) + (c +di)
= (a +c) + (b +d)i.
La representacin de los nmeros complejos con vectores permite dar una sencilla interpretacin
geomtrica de las operaciones con ellos. La suma de nmeros complejos puede ser entendida ge-
omtricamente como un vector igual a la suma de los vectores correspondientes a los nmeros
complejos sumandos.
Teorema 15.3 Sean z
1
, z
2
y z
3
nmeros complejos, entonces se cumplen las siguientes propiedades:
a) z
1
+z
2
= z
2
+z
1
;
b) (z
1
+z
2
) +z
3
= z
1
+ (z
2
+z
3
);
c) z
1
+ = z
1
;
d) z
1
+ (z
1
) = .
Demostracin
a) Si z
1
= a
1
+b
1
i y z
2
= a
2
+b
2
i, entonces
z
1
+z
2
= (a
1
+b
1
i) + (a
2
+b
2
i)
= (a
1
+a
2
) + (b
1
+b
2
)i
= (a
2
+a
1
) + (b
2
+b
1
)i
= (a
2
+b
2
i) + (a
1
+b
1
i)
= z
2
+z
1
.
CAPTULO 15. NMEROS COMPLEJOS 638
b) Si z
1
= a
1
+b
1
i, z
2
= a
2
+b
2
i y z
3
= a
3
+b
3
i, entonces
(z
1
+z
2
) +z
3
= [(a
1
+b
1
i) + (a
2
+b
2
i)] + (a
3
+b
3
i)
= [(a
1
+a
2
) + (b
1
+b
2
)i] + (a
3
+b
3
i)
= (a
1
+a
2
+a
3
) + (b
1
+b
2
+b
3
)i
= (a
1
+b
1
i) + [(a
2
+a
3
) + (b
2
+b
3
)i]
= (a
1
+b
1
i) + [(a
2
+b
2
i) + (a
3
+b
3
i)]
= z
1
+ (z
2
+z
3
).
c) Si z
1
= a
1
+b
1
i, = 0 + 0i, entonces
z
1
+ = (a
1
+b
1
i) + (0 + 0i)
= (a
1
+ 0) + (b
1
+ 0)i
= a
1
+b
1
i
= z
1
.
d) Si z
1
= a
1
+b
1
i, entonces
z
1
+ (z
1
) = (a
1
+b
1
i) + (a
1
b
1
i)
= (a
1
a
1
) + (b
1
b
1
)i
= 0 + 0i
= .
Teorema 15.4 Sean z
1
y z
2
nmeros complejos, entonces se cumple:
z
1
+z
2
= z
1
+z
2
Demostracin
Si z
1
= a
1
+b
1
i y z
2
= a
2
+b
2
i, entonces
z
1
+z
2
= (a
1
+b
1
i) + (a
2
+b
2
i)
= (a
1
+a
2
) + (b
1
+b
2
)i
= (a
1
+a
2
) (b
1
+b
2
)i
= (a
1
b
1
i) + (a
2
b
2
i)
= z
1
+z
2
.
Ejemplo 15.23 Hallar la suma de los nmeros complejos:
z
1
= 2 +i y z
2
= 3 2i.
Solucin
tenemos
z
1
+z
2
= (2 +i) + (3 2i)
= (2 + 3) + (1 2)i
= 5 i.
Denicin 15.6 Diferencia de nmeros complejos
Por sustraccin de un numero complejo z
1
= a + bi de otro numero complejo z
2
= c + di se
sobrentiende la determinacin de un numero z = (a c) + (b d)i que sumando al sustraendo z
2
nos da el minuendo z
1
.
CAPTULO 15. NMEROS COMPLEJOS 639
Para indicar la sustraccin se usa el signo ordinario de resta, de modo que el contenido de esta
denicin puede expresarse as:
z
1
z
2
= (a +bi) (c +di)
= (a c) + (b d)i.
En la sustraccin de dos nmeros complejos se restan separadamente sus partes reales e imag-
inarios. Geomtricamente la sustraccin de nmeros complejos signica la resta de sus correspon-
dientes vectores.
Teorema 15.5 Sean z
1
y z
2
nmeros complejos, entonces se cumple:
[z
1
[ [z
2
[ [z
1
z
2
[ [z
1
[ +[z
2
[
Demostracin
Por un lado, tenemos que
[z
1
z
2
[ = [z
1
+ (z
2
)[
[z
1
[ +[ z
2
[
= [z
1
[ +[z
2
[
Por otro lado, tenemos
[z
1
[ = [(z
1
z
2
) +z
2
[
[z
1
z
2
[ +[z
2
[
Con los resultados obtenidos, se establece que
[z
1
[ [z
2
[ [z
1
z
2
[ [z
1
[ +[z
2
[
Ejemplo 15.24 Hallar la diferencia de los nmeros complejos:
z
1
= 2 +i y z
2
= 3 2i.
Solucin
tenemos
z
1
z
2
= (2 +i) (3 2i)
= (2 3) + (1 + 2)i
= 1 + 3i.
CAPTULO 15. NMEROS COMPLEJOS 640
Denicin 15.7 Producto de nmeros complejos
Se denomina producto de los nmeros z
1
= a+bi y z
2
= c +di el nmero complejo z = (ac bd) +
(ad + bc)i. La multiplicacin se indica cuando no exista peligro de confusiones, puede omitirse el
signo de multiplicacin.
Dos nmeros complejos z
1
y z
2
se multiplican segn la regla ordinaria del producto de poli-
nomios; en el resultado i
2
se sustituye por -1 y se separa la parte real de la imaginaria:
z
1
z
2
= (a +bi)(c +di)
= ac +adi +bci +bdi
2
= (ac bd) + (ad +bc)i.
Podemos observar que el producto de dos nmeros complejos tambin es un nmero complejo.
Esta regla de la multiplicacin se extiende tambin a un nmero mayor de factores complejos.
De producto de nmeros complejos puede resultar un nmero real. En particular, esto ocurrir al
multiplicar dos nmeros complejos conjugados:
zz = (a +bi)(a bi)
= a
2
+abi abi b
2
i
2
= a
2
+b
2
= r
2
donde r es el mdulo de cada uno de los factores. As pues, el producto de dos nmeros complejos
conjugados es un nmero real, igual al cuadrado de su mdulo comn. Mientras que la denicin
adoptada para la adicin de los nmeros complejos es aceptada inmediatamente como natural por
los estudiantes, stos se quedan perplejos por el carcter aparentemente articioso de la denicin
de multiplicacin y siempre preguntan las razones por las que se adoptan.
Puesto que los nmeros complejos son nuevos objetos para los cuales las nociones de igualdad,
adicin y multiplicacin no estn denidas inicialmente, es privilegio nuestro denir estas nociones
como nos plazca, esforzndonos solamente por hacerlo de modo tal que todas las propiedades
fundamentales de las operaciones algebraicas con nmeros reales conserven su validez para los
nmeros complejos, y que, adems, los nmeros complejos sujetos a tales propiedades pueden
reemplazar a los nmeros imaginarios hasta ahora sin sentido.
Teorema 15.6 Sean z
1
, z
2
y z
3
nmeros complejos, entonces se cumplen las siguientes propiedades:
a) z
1
z
2
= z
2
z
1
;
b) (z
1
z
2
) z
3
= z
1
(z
2
z
3
);
c) 1 z
1
= z
1
;
d) z
1
z
1
1
= 1.
Demostracin
a) Si z
1
= a
1
+b
1
i y z
2
= a
2
+b
2
i, entonces
z
1
z
2
= (a
1
+b
1
i)(a
2
+b
2
i)
= (a
1
a
2
b
1
b
2
) + (a
1
b
2
+b
1
a
2
)i
= (a
2
a
1
b
2
b
1
) + (b
2
a
1
+a
2
b
1
)i
= (a
2
+b
2
i)(a
1
+b
1
i)
= z
2
z
1
.
CAPTULO 15. NMEROS COMPLEJOS 641
b) Si z
1
= a
1
+b
1
i, z
2
= a
2
+b
2
i y z
3
= a
3
+b
3
i, entonces
(z
1
z
2
) z
3
= [(a
1
+b
1
i) (a
2
+b
2
i)] (a
3
+b
3
i)
= [(a
1
a
2
b
1
b
2
) + (a
1
b
2
+b
1
a
2
)i] (a
3
+b
3
i)
= [(a
1
a
2
b
1
b
2
)a
3
(a
1
b
2
+b
1
a
2
)b
3
] + [(a
1
a
2
b
1
b
2
)b
3
+ (a
1
b
2
+b
1
a
2
)a
3
]i
= (a
1
a
2
a
3
b
1
b
2
a
3
a
1
b
2
b
3
b
1
a
2
b
3
) + (a
1
a
2
b
3
b
1
b
2
b
3
+a
1
b
2
a
3
+b
1
a
2
a
3
)i
= [a
1
(a
2
a
3
b
2
b
3
) b
1
(a
2
b
3
+b
2
a
3
)] + [a
1
(a
2
b
3
+b
2
a
3
) +b
1
(a
2
a
3
b
2
b
3
)]i
= (a
1
+b
1
i)[(a
2
a
3
b
2
b
3
) + (a
2
b
3
+b
2
a
3
)i]
= z
1
(z
2
z
3
).
c) Si z
1
= a
1
+b
1
i y 1 = 1 + 0i, entonces
1 z
1
= (1 + 0i)(a
1
+b
1
i)
= (1 a
1
0 b
1
) + (1 b
1
+ 0 a
1
)i
= a
1
+b
1
i
= z
1
.
d) Si z
1
= a
1
+b
1
i y z
1
1
=
a
1
a
2
1
+b
2
1

b
1
a
2
1
+b
2
1
i entonces
z
1
z
1
1
= (a
1
+b
1
i)
_
a
1
a
2
1
+b
2
1

b
1
a
2
1
+b
2
1
i
_
=
_
a
1

a
1
a
2
1
+b
2
1
+b
1

b
1
a
2
1
+b
2
1
_
+
_
a
1

b
1
a
2
1
+b
2
1
+b
1

a
1
a
2
1
+b
2
1
_
i
=
_
a
2
1
a
2
1
+b
2
1
+
b
2
1
a
2
1
+b
2
1
_
+
_

a
1
b
1
a
2
1
+b
2
1
+
b
1
a
1
a
2
1
+b
2
1
_
i
=
a
2
1
+b
2
1
a
2
1
+b
2
1
+
a
1
b
1
+b
1
a
1
a
2
1
+b
2
1
i
= 1 + 0i
= 1.
Teorema 15.7 Sean z
1
, z
2
y z
3
nmeros complejos, entonces se cumplen que:
z
1
(z
2
+z
3
) = z
1
z
2
+z
1
z
3
Demostracin
Si z
1
= a
1
+b
1
i, z
2
= a
2
+b
2
i y z
3
= a
3
+b
3
i, entonces
z
1
(z
2
+z
3
) = (a
1
+b
1
i) [(a
2
+b
2
i) + (a
3
+b
3
i)]
= (a
1
+b
1
i) [(a
2
+a
3
) + (b
2
+b
3
)i]
= [a
1
(a
2
+a
3
) b
1
(b
2
+b
3
)] + [a
1
(b
2
+b
3
) +b
1
(a
2
+a
3
)]i
= (a
1
a
2
+a
1
a
3
b
1
b
2
b
1
b
3
) + (a
1
b
2
+a
1
b
3
+b
1
a
2
+b
1
a
3
)i
= [(a
1
a
2
b
1
b
2
) + (a
1
a
3
b
1
b
3
)] + [(a
1
b
2
+b
1
a
2
) + (a
1
b
3
+b
1
a
3
)]i
= [(a
1
a
2
b
1
b
2
) + (a
1
b
2
+b
1
a
2
)i] + [(a
1
a
3
b
1
b
3
) + (a
1
b
3
+b
1
a
3
)i]
= z
1
z
2
+z
1
z
3
.
CAPTULO 15. NMEROS COMPLEJOS 642
Teorema 15.8 Sean z
1
y z
2
nmeros complejos, entonces se cumple:
z
1
z
2
= z
1
z
2
Demostracin
Si z
1
= a
1
+b
1
i y z
2
= a
2
+b
2
i, entonces
z
1
z
2
= (a
1
+b
1
i)(a
2
+b
2
i)
= (a
1
a
2
b
1
b
2
) + (a
1
b
2
+b
1
a
2
)i
= (a
1
a
2
b
1
b
2
) (a
1
b
2
+b
1
a
2
)i
= (a
1
a
2
b
1
a
2
i) (b
1
b
2
+a
1
b
2
i)
= (a
1
b
1
i)a
2
(b
1
+a
1
i)b
2
= (a
1
b
1
i)a
2
+i(b
1
+a
1
i)b
2
i
= (a
1
b
1
i)a
2
(a
1
b
1
i)b
2
i
= (a
1
b
1
i)(a
2
b
2
i)
= z
1
z
2
.
Ejemplo 15.25 Multiplicar los nmeros complejos
z
1
= 4(Cos60

+iSen60

) y z
2
= 3(Cos210

+iSen210

).
Solucin
Tenemos que
z
1
z
2
= [4(Cos60

+iSen60

)][3(Cos210

+iSen210

)]
= 12(Cos60

+iSen60

)(Cos210

+iSen210

)
= 12
_
1
2
+

3
2
i
__

3
2

1
2
i
_
= 3(1 +

3 i)(

3 i)
= 3(

3 i 3i

3 i
2
)
= 3(

3 i 3i +

3)
= 12i.
Denicin 15.8 Divisin de nmeros complejos
Se llama cociente de la divisin de dos nmeros complejos z
1
= a + bi y z
2
= c + di el nmero
complejo z = x +yi que multiplicado por el divisor nos da el dividendo.
De este modo, si los coecientes c y d son simultneamente distintos de cero, suponiendo que
a +bi
c +di
= x +yi
tendremos:
a +bi = (c +di)(x +yi)
o bien
a +bi = cx dy + (dx +cy)i.
CAPTULO 15. NMEROS COMPLEJOS 643
De la condicin de igualdad de dos nmeros complejos se deduce que
_
cx dy = a
dx +cy = b
Resolviendo este sistema, hallamos que:
x =
ac +bd
c
2
+d
2
e y =
cb +ad
c
2
+d
2
Por lo tanto,
a +bi
c +di
=
ac +bd
c
2
+d
2
+
cb +ad
c
2
+d
2
i
Este resultado se puede obtener ms simplemente multiplicando el dividendo y el divisor por un
nmero conjugado al divisor:
a +bi
c +di
=
(a +bi)(c di)
(c +di)(c di)
=
ac +bd + (cb ad)i
c
2
+d
2
=
ac +bd
c
2
+d
2
+
cb ad
c
2
+d
2
i
De las cuatro operaciones fundamentales, la adicin y la multiplicacin se llaman operaciones
directas y por medio de ellas se denen las operaciones inversas: sustraccin y divisin.
Teorema 15.9 Sean z
1
y z
2
nmeros complejos no nulos, entonces se cumplen:
a) z
1
1
=
z
1
[z
1
[
2
;
b) [z
1
1
[ =
1
[z
1
[
;
c) (z
1
z
2
)
1
= z
1
1
z
1
2
;
d) (z
1
1
) = (z
1
)
1
.
Demostracin
a) Si z
1
= a
1
+b
1
i, entonces
z
1
1
=
1
z
1
=
a
1
a
2
1
+b
2
1

b
1
a
2
1
+b
2
1
i
=
1
a
2
1
+b
2
1
(a
1
b
1
i)
=
1
[z
1
[
2
z
1
=
z
1
[z
1
[
2
.
CAPTULO 15. NMEROS COMPLEJOS 644
b) Para demostrar esta propiedad, utilizaremos la anterior

z
1
1

z
1
[z
1
[
2

=
[z
1
[
[z
1
[
2
=
[z
1
[
[z
1
[
2
=
1
[z
1
[
.
c) Como z
1
y z
2
son nmeros complejos no nulos, entonces
(z
1
z
2
)
1
=
z
1
z
2
[z
1
z
2
[
2
=
z
1
z
2
[z
1
[
2
[z
2
[
2
=
z
1
[z
1
[
2

z
2
[z
2
[
2
= z
1
1
z
1
2
.
d) Como z
1
es un nmero complejo no nulo, entonces
(z
1
1
) =
_
z
1
[z
1
[
2
_
=
z
1
[z
1
[
2
=
z
1
[z
1
[
2
= (z
1
)
1
.
Teorema 15.10 Sean z
1
y z
2
nmeros complejos con z
2
,= 0, entonces se cumplen:
a)

z
1
z
2

=
[z
1
[
[z
2
[
;
b)
_
z
1
z
2
_
=
z
1
z
2
.
Demostracin
a) Para demostrar esta propiedad, hacemos lo siguiente

z
1
z
2

= [z
1
z
1
2
[
= [z
1
[[z
1
2
[
= [z
1
[
1
[z
2
[
=
[z
1
[
[z
2
[
.
CAPTULO 15. NMEROS COMPLEJOS 645
b) Si z
2
,= 0, entonces
_
z
1
z
2
_
= (z
1
z
1
2
)
= z
1
z
1
2
= z
1
(z
2
)
1
=
z
1
z
2
.
Ejemplo 15.26 Dividir el nmero z
1
= 2 + 3i por el nmero z
2
= 1 + 4i.
Solucin
Tenemos
z
1
z
2
=
2 + 31
1 + 4i
=
(2 + 3i)(1 4i)
(1 + 4i)(1 4i)
=
14 5i
17
=
14
17

5
17
i
Ejemplo 15.27 Simplique la expresin:
3 +i
(1 +i)(1 2i)
Solucin
Despus de multiplicar los nmeros en el denominador, obtenemos
3 +i
(1 +i)(1 2i)
=
3 +i
1 2i +i + 2
=
3 +i
3 i
A continuacin, tenemos la posibilidad de multiplicar y dividir por el conjugado del numero com-
plejo 3 i, entonces
3 +i
3 i
=
(3 +i)(3 +i)
(3 i)(3 +i)
=
9 + 6i 1
9 + 1
=
4
5
+
3
5
i
Ejemplo 15.28 Qu lugar geomtrico ha de describir el ajo del nmero complejo z para
que los ajos de z, iz e i, estn alineados?
Solucin
Si z = a + bi, entonces iz = b + ai. Para que z, iz e i estn alineados, el rea del tringulo que
determinan debe ser cero, es decir

1 1 1
a b 0
b a 1

= 0 a
2
+b
2
a b = 0
CAPTULO 15. NMEROS COMPLEJOS 646
por lo que (a, b) estar en la circunferencia de radio
_
1
2
,
1
2
_
y radio
1
2
.
Ejemplo 15.29 Demuestre que los tres ajos de z
1
, z
2
y z
3
forman un tringulo equiltero si
y slo si
z
2
1
+z
2
2
+z
2
3
= z
1
z
2
+z
2
z
3
+z
3
z
1
Solucin
Supongamos primero que z
1
, z
2
y z
3
son los vrtices de un tringulo equiltero. Entonces z
3
es
uno de los complejos obtenidos complejo m =
z
1
+z
2
2
, punto medio de z
1
y z
2
, un complejo n
perpendicular a z
1
z
2
y con mdulo

3
2
[z
1
z
2
[, es decir
z
3
= m+n =
1
2
(z
1
+z
2
)

3
2
(z
1
z
2
)i
(2z
3
z
1
z
2
)
2
= 3(z
1
z
2
)
2
4z
2
3
+z
2
1
+z
2
2
4z
1
z
3
4z
2
z
4
+ 2z
1
z
2
= 3z
2
1
3z
2
2
+ 6z
1
z
2
4z
2
1
+ 4z
2
2
+ 4z
2
3
= 4z
1
z
2
+ 4z
2
z
3
+ 4z
3
z
1
z
2
1
+z
2
2
+z
2
3
= z
1
z
2
+z
2
z
3
+z
3
z
1
Recprocamente, si z
1
, z
2
y z
3
son nmeros complejos que satisfacen z
2
1
+z
2
2
+z
2
3
= z
1
z
2
+z
2
z
3
+z
3
z
1
,
resolviendo como ecuacin en z
3
obtenemos
z
2
3
(z
1
+z
2
)z
3
+z
2
1
+z
2
2
z
1
z
2
= 0
z
3
=
z
1
+z
2

_
(z
2
1
+z
2
2
+ 2z
1
z
2
) 4(z
2
1
+z
2
2
z
1
z
2
)
2
=
z
1
+z
2

_
3(z
2
1
+z
2
2
2z
1
z
2
)
2
=
z
1
+z
2
2

3
2
(z
1
z
2
)i
es decir, z
1
, z
2
y z
3
son los vrtices de un tringulo equiltero.
15.3. Tarea
1. Efectuar las siguientes operaciones con nmeros complejos:
a)
5 i
5 +i
; b)
1 +i
1 i
; c)
2 + 4i
3 + 5i
; d) (1+i)(1+

3 i); e)
(3i)
2
(1 2i)
2 + 2i
;
f )
(1 +i)
2
(1 i)
2
; g) (2 3i)(1 + 2i); h)
_

2
2
+

2
2
i
_
4
; i) (3 7i)(3 + 7i).
2. Efectuar las siguientes operaciones:
a)
1 +i
2 i

3 + 2i
1 + 3i
; b)
(2 +i)
2
+ (1 i)
2
1
3
2
i
; c)
(1 + 2i)(2 +i)
2 i
+
(1 2i)(2 i)
2 +i
.
3. Represente grcamente los nmeros 2 i, 3 + 2i, 5i, 1 2i, 3, 3 i, 3 + 3i.
CAPTULO 15. NMEROS COMPLEJOS 647
4. Hallar los vrtices del polgono correspondiente a los ajos del complejo
5

i.
5. Hallar los vrtices del polgono correspondiente a los ajos del complejo
6

1.
6. Dado el nmero complejo

3 +i, hallar su opuesto, su conjugado y representelos grca-
mente.
7. Hallar los vrtices del polgono correspondiente a los ajos del complejo
4
_
2

3 + 2i.
8. Determine los nmeros reales x e y tal que 3(x + 2) + 2iy ix + 5y = 7 + 5i.
Resp: x =
23
11
, y =
16
11
.
9. Simplique las expresiones:
a) (3 + 5i) + (5 + 2i) (4 + 7i)
2
; b)
5 2i
4 +i
+
2 + 5i
3i
; c)
3
4(5 i)(4 + 6i)
;
d) (2 + 3i)(5 3i)(4 + 5i
5
); e)
(5 +i)(1 +i)
3 i
+i; f )
_
2i
37
(2 +i)(3 + 4i)
_
2
.
10. Si z = a +bi, determine:
a)
Re(z)
iIm(iz)
; b) [1 Re(z) +iIm(z)][1 Re(z) iIm(z)].
Resp: a) i; b) 1 +a
2
+b
2
2a.
11. Si z
1
= 1 + 2i, z
2
= 2 +i, z
3
= 1 i, calcular:
a) 2z
1
+ 3z
2
+ 3; b)
z
1
iz
2
; c) z
2
1
+ 2z
2
3
; d)
z
1
+z
3
1 +z
2
.
12. Represente en el plano:
a) z C/Re(z) = 2; b) z C/1 Re(z) 2, 0 Im(z) 3;
c) z C/[z[ 1; d) z C/[z i[ 4;
e) z C/Re((z 1)
2
) = Re(2z(z 1)); f ) z C/(z + 1)(z +1) +2Re(z +1) 0.
Resp: a) Recta de ecuacin x = 2; c) Crculo con centro en (0, 0) y radio menor o
igual a 1; e) Hiprbola 1 = x
2
y
2
.
13. Determine z C tal que:
a) [z[ z = 1 + 2i; b) [z[ +z = 2 +i; c) z z + 3(z z) = 4 3i;
d) z z + 2z = 3 +i.
Resp: a)
3
2
2i; b)
3
4
+i; c)

15
2

1
2
i,

15
2

1
2
i.
14. Descomponer en pares de factores complejos:
a) a
2
+ 9b
2
; b) 4a
2
+ 16b
2
; c) a
2
+
b
2
4
.
15. Simplicar la expresin

1 +a +i

1 a

1 +a i

1 a

1 a +i

1 +a

1 a i

1 +a
.
16. Simplicar la expresin:
a)
(3 + 3i)(4 2i)
2 2i
; b)
2 + 3i
(4 + 2i)(1 +i)
.
CAPTULO 15. NMEROS COMPLEJOS 648
17. Resuelva las ecuaciones:
a) z
2
+ 3[z[ = 0; b) z
2
+ 2[z[ = 1; c) z
2
+[z[
2
= 0; d) z
2
+z[z[ +[z
2
[ = 0.
18. Demuestre la desigualdad [[z[ [u[[ [z +u[ [z[ +[u[.
19. Demuestre que [z u[, es decir, el mdulo de la diferencia de los nmeros complejos z y
u, es igual a la distancia entre los puntos z y u en el plano complejo.
20. Hallar el conjunto de puntos del plano complejo prejado con la condicin:
a) [z + 1[ = 1; b) [z i[ < [z +i[; c) [z + 2i 1[ 2; d) log [z 10i[ < 1;
e) [z 2[ +[z + 2[ = 26; f ) [z 2[
2
+[z + 2[
2
= 26; g) [z[
2
+ 3z + 3z = 0.
21. Resuelva los sistemas de ecuaciones:
a)
_
[z + 1[ = [z + 2[
[3z + 9[ = [5z + 10i[
; b)
_
(1 i)z = (1 +i)z
[z
2
+ 51i[ = 1
.
c) [z + 1 i[ = [3 + 2i z[ = [z +i[;
22. Simplique las expresiones:
a)

(2 +i)(3 + 4i)(5 3i)


(3 4i)(5 + 3i)

; b)

(3 + 5i)(5 2i)
5 + 2i

2
3i

.
Resp: a)

5; b)
2
3

34.
23. Encuentre los nmeros complejos z que satisfacen las relaciones siguientes:

z 12
z 8i

=
5
3
y

z 4
z 8

= 1
Resp: z = 6 + 17i, z = 6 + 8i.
24. La suma de dos nmeros complejos es 3+2i. La parte real de uno de ellos es 2, El cociente
entre ellos es imaginario puro. Hallar ambos nmeros.
Resp: z
1
= 2 + (1 +

3)i, z
2
= 1 + (1

3)i; z
1
= 2 + (1

3)i, z
2
= 1 + (1 +

3)i.
25. Determine z C tal que [z[ =
1
[z[
= [1 z[.
Resp: z =
1
2
+

3
2
i, z =
1
2

3
2
i.
26. Si z, w C y [z[ = 1, demuestre que

z +w
z w + 1

= 1.
27. Dnde se halla el punto z del plano complejo si el punto z
2
pertenece al eje imaginario?
28. Dnde se halla el punto z
2
, si el punto z pertenece a la recta Im(z) = 1?
CAPTULO 15. NMEROS COMPLEJOS 649
29. Sea z ,= 1. Demuestre que el punto
z 1
z + 1
pertenece al eje imaginario si y slo si el punto
z pertenece a la circunferencia de radio R = 1 con centro en el punto z = 0.
30. Determine el valor de k en la expresin
k + 3i
3 + 2i
para que sea un nmero complejo imaginario
puro.
31. Determine el valor de k en la expresin
3 2ki
4 + 3i
para que sea un nmero complejo imagi-
nario puro, sea un nmero complejo real puro.
32. Dado el nmero complejo z =
k +i
2 +i
, hallar el valor de k para que el mdulo de z sea

2.
33. Hallar el valor del nmero real k para que (25 ki)
2
sea un nmero complejo imaginario
puro.
34. Dado el nmero complejo z =
1
2
+

3
2
i, compruebe que 1 +z +z
2
= 0.
35. Dado el nmero complejo z =
1
2
+

3
2
i, compruebe que
1
z
= z
2
.
36. Hallar m y n para que se verique la igualdad (2 +mi) + (n + 5i) = 7 2i.
37. Hallar el valor de k para que se cumpla
k +i
1 +i
= 2 i.
38. Hallar a y b para que se cumpla (a +bi)
2
= 3 + 4i.
39. Hallar a y b para que se cumpla (2 ai)(3 bi) = 8 + 4i.
40. Hallar a y b para que se cumpla (a 3i) =
2 +bi
5 3i
.
41. Dados los nmeros complejos 3ki y 2

5+

5i, hallar el nmero k para que sus mdulos


sean iguales.
42. Hallar el valor de k para que el nmero
k + 2 +ki
k +i
sea imaginario puro.
43. Determine el valor de k para que (k 2i)
2
sea imaginario puro.
44. Hallar el valor de k para que la expresin (k + 2 +ki)(k i) sea un real puro.
CAPTULO 15. NMEROS COMPLEJOS 650
45. Hallar dos nmeros complejos, cuyo producto sea 2i y el cubo de uno de ellos dividido por
el otro sea
1
2
.
46. Hallar dos nmeros complejos, cuyo producto sea -8 y uno de ellos sea el cuadrado del otro.
47. Hallar una ecuacin de segundo grado, sabiendo que sus races son 1 +i y 2 3i.
48. Simplicar las expresiones:
a)
2i
(4 3i)(1 2i)(2 3i)
;
b)
2 3i
(1 + 2i)(1 2i)(3 2i)
;
c)
1 2i
(1 + 2i)(2 i)(2 2i)
;
d)
(1 2i)(2 i)
(4 2i)(1 3i)(2 +i)
;
e)
(1 + 3i)
2
(1 + 2i)
2
(2 3i)
2
(3 2i)
2
;
f )
3i + (1 i)
3
(1 +i)
2
(1 i)
2
+ (1 +i)
2
;
g)
(2 + 5i)
4
+i
(2 5i)
4
i
;
h)
(3 + 4i)
2
(4 3i)
2
(3 4i)
2
+ (4 + 3i)
2
;
i)
(3 5i)(1 + 3i)
((1 3i)
2
+ (1 + 2i)
2
.
49. Determine los nmeros reales x e y que satisfacen las ecuaciones:
a)
x(2 i)
2
+y(3 2i)
2
i
2 + 3i
= 3 2i; b)
(3 +i)x
iy
=
2x 4i
x + 2y
;
c)
_
2 i
2 +i
_
2

1
2x iy
= 2 i.
50. Calcular dos nmeros complejos cuya suma sea 1 + 4i, cuyo cociente sea imaginario y de
manera que la parte real de uno de ellos sea -1.
51. Si z
1
= 3 4i, z
2
= 1 + 5i y 2z
1
z
3
= 3z
2
, encontrar z
3
y z
1
3
.
52. Si z
1
= 2 +i, z
2
= 2 2i, z
3
= 1 + 3i. Hallar el valor numrico de las expresiones:
a)

2z
1
z
2
3
2z
1
+z
2
+ 3

; b)
z
2
z
2

z
3
z
3
; c) Im
_
z
1
z
3
z
2
_
; d) Re
_
3z
2
+ 2z
2
1
3z
2
3
_
.
53. Simplicar la expresin
(2

3 i)
3
(1 2i)(

3 i)
(3 3

3 i)(1 i)
2
.
54. Sea z = 5 2i, calcular
2
2z 1
+
1
3z
.
55. Hallar x, y R tales que z = x+iy sea una raz cuadrada de 25i, es decir, (x+iy)
2
= 25i.
En general, dados a, b R, si z = x + iy es una raz cuadrada de a + ib, es decir,
(x +iy)
2
= a +ib, expresar x e y en funcin de a y b.
56. Hallar z tal que
2 3i
2z(1 + 2i)
= 6 3i.
CAPTULO 15. NMEROS COMPLEJOS 651
57. Si z =
2 3i
4 +i
y w =
6 + 5i
3 + 2i
, determine [z +w[.
58. Si z =
(2 i)(

2 +i)(2 3i)
(1 + 2i)(

2 i)
, determine [z[.
59. Si z C, resolver [z 2i[
2
+z 1 = 2 3i, Re(z) 0.
60. Hallar z = x +iy que satisfacen la condicin dada:
a) [z + 4i[ [z i[ < 5; b) [z + 5[ [z 2[ 6; c)

z 3i
z + 3i

< 3;
d)

z + 4i
z 4i

> 1.
61. Resolver el sistema de ecuaciones:
a)
_
(1 +i)x + (2 i)y = 1 3i
(1 i)x (3 +i)y = 2 i
; b)
_
ix (3 + 2i)y = 1 +i
(2 +i)x + (2 i)y = 2 +i
.
62. Resuelva el sistema
_
(1 +i)z iu = 2 +i
(2 +i)z + (2 i)u = 2i
, z, u C
Resp: z =
1
13
(6 9i), z =
1
13
(16 + 11i).
63. Identicar el lugar geomtrico de los puntos que representan los nmeros complejos z, tales
que:
a) [z 1[ [2z + 1 5i[ = 2; b) [2z[ [z 3[ = [z i[; c) [2z + 3 i[ = 5;
d) [z + 2 3i[ +[z 2 3i[ = 4.
64. Encuentre el conjunto de puntos del plano que satisface la relacin [z + 2 3i[ = 3.
65. Si a, b y c son reales, usando nmeros complejos, demostrar que
(a
2
+b
2
)(c
2
+d
2
) = (ac bd)
2
+ (ad +bc)
2
66. Demuestre que los nicos elementos de C cuyo cuadrado es -1, son i y i.
67. Determine una ecuacin de segundo grado a coecientes reales, que admita la solucin
z =
i
1 +i +
i
1i+
i
1+i
68. Determine la parte real y la parte imaginaria del nmero complejo
z =

3 + 4i +

3 4i
CAPTULO 15. NMEROS COMPLEJOS 652
69. Demuestre que si la ecuacin z
2
+ (a + bi)z + (c + di) = 0 tiene una raz real, se verica
que d
2
abd +cb
2
= 0.
70. Los nmeros complejos z y w tienen suma y producto reales. Demuestre que z y w son
nmeros complejos conjugados.
71. Dado el nmero complejo z = a +bi ,= 0 + 0i, determine un nmero complejo w = x +yi,
tal que zw = 1.
72. Si w es una raz cbica compleja de la unidad, demuestre que
(1 +w)(1 + 2w)(1 + 3w)(1 + 5w) = 21
73. Determine las races de la ecuacin 4x
3
3x + 1 = 0 sabiendo que
(x +a +b)(c +aw +bw
2
)(x +aw
2
+bw) = x
3
3abx +a
3
+b
3
74. Resuelva la ecuacin z
6
+ 7z
3
8 = 0.
75. Determine las races de la ecuacin x
3
= 1, sabiendo que las races de x
3
= 1 son x
1
= 1,
x
2
=
1
2
(1 i

3) y x
3
=
1
2
(1 +i

3).
76. Determine las races de la ecuacin x
3
= i, sabiendo que las races de x
3
= 1 son x
1
= i,
x
2
=
1
2
(1 i

3) y x
3
=
1
2
(1 +i

3).
77. Si a es una raz compleja de la ecuacin z
n
1 = 0, demuestre que 1+a+a
2
+...+a
n1
= 0.
78. Calcular S = 1 +i +i
2
+i
3
+... +i
n1
.
79. Calcular S = 1 + 2i + 3i
2
+ 4i
3
+ 5i
4
+... + (4n)i
4n1
.
80. Determine un nmero complejo z = x +iy tal que z
2
= p +iq.
81. Dados los nmeros complejos z y w con [z[ = 1, demuestre que

z +w
1 +zw

= 1.
82. Sea z C con z ,= 1. Demuestre que
1 +z
1 z
es imaginario si y slo si [z[ = 1.
83. Determine los nmeros complejos z tales que z,
1
z
, z 1 tengan el mismo mdulo.
84. Sean z
1
= 1 +i y z
2
=

3 +i. Calcular el mdulo y el argumento de z


1
y z
2
.
CAPTULO 15. NMEROS COMPLEJOS 653
85. Resolver en C la ecuacin z
3
+z
2
+z + 1 = 0.
86. Resolver en C la ecuacin (1 i)z
2
(6 4i)z + 9 7i = 0.
87. Sea z =
1
2
+

3
2
i. Calcule:
a) z
2
; b) 1 +z +z
2
; c)
1 +z
(1 z)
2
+
1 z
(1 +z)
2
.
88. Sean z
1
=
1 +

3 i
1

3 i
y z
2
=
1

3 +i)
. Calcule:
a) z
1
+z
2
; b) z
1
z
2
; c) z
2
1
+z
2
2
; d) z
3
1
+z
3
2
.
89. Determine el conjunto de nmeros z tales que:
a) Re
_
z 1
z i
_
= 0; b) Im
_
z 1
z i
_
= 0; c) Re(z
3
)=Im(z
3
).
90. Sean z
1
= 4 +4i y a
2
= 1

3 i nmeros complejos. Determine el mdulo y el argumento


de:
a) z
2
1
; b) z
1
z
2
; c) z
3
1
; d)
z
1
z
2
; e)
z
2
z
1
.
91. Sean z
1
y z
2
nmeros complejos de mdulo 1, tales que z
1
z
2
,= 1. Muestre que
z
1
+z
2
1 +z
1
z
2
es un nmero real.
92. Demuestre que Re
_
z
z +w
_
+Im
_
w
w +z
_
+ 1.
93. Demuestre que Re(zw) = Re(z)Re(w) Im(z(Im(w).
94. Demuestre que Im(zw) = Re(z)Im(w) +Im(z)Re(w).
95. Demuestre que si k es un nmero real, el nmero complejo
1 +ki
1 ki
tiene mdulo 1.
96. Calcule el nmero complejo z tal que

z 12
z 81

=
5
3
y

z 4
z 8

= 1.
97. Determine los nmeros complejos tales que su mdulo sea 5 y la parte real de su cuadrado
sea 7.
Resp: 4 + 3i, 4 3i, 4 3i, 4 + 3i.
98. Encuentre el nmero complejo z tal que [z[
2
= 37 y Im(z
2
) = 12.
Resp: 6 +i, 6 i, 1 6i, 1 + 6i.
CAPTULO 15. NMEROS COMPLEJOS 654
99. Demuestre que para todo z, w C, se cumple que
[z +w[
2
+[z w[
2
2
= [z[
2
+[w[
2
.
100. Determine el conjunto de nmeros complejos z tal que
z + 2i
z i
tenga mdulo 2.
101. Demuestre que
1
z
=
z
[z[
2
. Compruebe el resultado para z = 1 2i y z =

3 +i.
102. Encuentre los nmeros complejos z tales que Re(z
2
)=Im(z
2
) y Re(z)+Im(z)=1.
103. Sean w, z C tales que w =
z a
az 1
con 0 < a < 1. demuestre que [w[ < 1 si y slo si
[z[ < 1.
104. Demuestre que si [a[ = 1 o [b[ = 1, entonces

a b
1 ab

= 1. Qu excepcin debe hacerse si


[a[ = [b[ = 1?
105. Sea w =
1 +z
1 z
con w = u +iv y z = x +iy, x, y, u, v R. Demuestre que
x =
u
2
+v
2
1
(u + 1)
2
+v
2
, y =
2v
(u + 1)
2
+v
2
106. Sea z = a +ib. Demuestre que existen p, m y n independientes de z (z ,= 0, 1) tales que
(a
2
+b
2
)(a
2
+b
2
+ 2a + 1)
a
2
b
2
+a (1 + 2a)bi
= p +mz +nz
2
Hallar p, m y n.
107. Sabiendo que [z
1
[ = [z
2
[ = ... = [z
n
[ = 1, demuestre que
[z
1
+z
2
+... +z
n
[ =

1
z
1
+
1
z
2
+... +
1
z
n

108. Demuestre que para todo par de nmeros complejos z y w se tiene


[z +w[
2
+[z w[
2
= 2[z[
2
+ 2[w[
2
109. Sabiendo que todo par de nmeros complejos z
1
y z
2
verica la igualdad
2[z
1
[
2
+ 2[z
2
[
2
= [z
1
+z
2
[
2
+[z
1
z
2
[
2
demuestre que todo par de complejos a y b verica la igualdad
[a +
_
a
2
b
2
[ +[a
_
a
2
b
2
[ = [a +b[ +[a b[
CAPTULO 15. NMEROS COMPLEJOS 655
110. Sabiendo que todo par de complejos p y q verica la igualdad
[p +
_
p
2
q
2
[ +[p
_
p
2
q
2
[ = [p +q[ +[p q[
demuestre que todo par de complejos a y b verica la igualdad

a +b
2
+

ab

a +b
2

ab

= [a[ +[b[
111. Si a es un complejo de mdulo menor que uno. [a[ < 1, demuestre que

z a
1 az

es menor, igual o mayor que 1, segn sea [z[ menor, igual o mayor que uno.
112. La ecuacin x
3
9x
2
+33x65 = 0 tiene una raz compleja cuyo mdulo es 13. Determine
las tres races de la ecuacin.
113. Si w
1
y w
2
son races de la ecuacin x
2
+ 2 = 0, hallar el valor de:
a) w
2
1
w
2
2
; b) w
1
w
2
w
2
1
; c) w
2
1
+w
2
2
.
114. Demuestre que todo nmero complejo z de mdulo r con z ,= r se puede poner en la
forma z = r
1+it
1it
siendo t un nmero real.
115. Sea p(x) =
n

k=0
a
k
x
k
un polinomio a coecientes reales, tal que p(z) = a + ib. Demuestre
que p(z) = a ib.
116. Determine la parte real y la imaginaria de cada uno de los nmeros complejos z =

i y
w =
1

i
.
117. Determine que curva debe recorrer el nmero complejo z para que w =
z + 1
z 1
sea imagi-
nario puro.
118. Sea = a + ib un nmero complejo jo y z = x + iy un nmero complejo que recorra la
recta y = mx +n. Determine que curva recorre el nmero complejo w = +z.
119. La suma de dos nmeros complejos variables z
1
y z
2
dividida por la diferencia de ellos da
un imaginario puro. Demuestre que los nmeros complejos z
1
y z
2
se desplazan sobre una
circunferencia con centro en el origen.
120. Un nmero complejo z = x + iy se mueve sobre la recta 3x + 4y + 5 = 0. Demuestre que
el valor mnimo de [z[ es uno.
CAPTULO 15. NMEROS COMPLEJOS 656
121. Un nmero complejo z = x +iy se desplaza en el plano xy de modo que [2z 1[ = [z 2[,
determine que curva recorre.
122. Determine el lugar geomtrico de un nmero complejo z = x +iy que verica la condicin
[(1 +i)z (1 + 3i)[ 1.
123. Los vrtices A, B y C de un tringulo se mueven de modo que
C A
C B
= k
1
+ ik
2
= k
donde k es constante. Demuestre que el tringulo ABC permanece semejante a si mismo.
124. Los nmeros complejos z = x +iy y w = u +iv verican siempre la igualdad w = z +
a
2
z
,
a R.
125. Los nmeros complejos z = x +iy y w = u +iv, verican siempre la igualdad w = z
2
+a,
donde a es un nmero real. Determine el lugar geomtrico de w, cuando z recorre:
a) La circunferencia x
2
+y
2
= 1 b) La recta x = y
126. Los nmeros complejos z y w verican siempre la relacin w = (4 +i) +
3 3i
z i
. Determine
el lugar geomtrico de w cuando z recorre:
a) La circunferencia [z[ = 1 b) El eje de ordenadas.
127. Los nmeros complejos A y B son las races de la ecuacin z
2
(8 + 5i)z + (8 + 26i) = 0.
Determine un nmero complejo C = x+iy, de tal modo que el tringulo ABC sea equiltero.
128. Los cuatro nmeros complejos A, B, C y D son concclicos. Demuestre que el nmero
(C A)(D B)
(C B)(D A)
es real.
129. Los vrtices opuestos A y C de un rombo ABCD estn dados por las races de la ecuacin
z
2
6(1+i)z+16i = 0. Determine una ecuacin de segundo grado que d los otros dos vrtices.
130. En un cuadrado ABCD, los vrtices opuestos A y C son las races de la ecuacin z
2
(6+
8i)z + (1 + 30i) = 0. Determine los otros dos vrtices.
131. Sobre los lados de un cuadriltero ABCD se construyen hacia el exterior tringulos rec-
tngulos issceles: ABP, BCQ, CDR y DAS. Demuestre que los trazos PR y QS son iguales
y perpendiculares.
132. A, B, C, P Q y R son nmeros complejos tales que det[[1, 1, 1], [A, B, C], [P, Q, R]] = 0.
Demuestre que los tringulos ABC y PQR son semejantes.
CAPTULO 15. NMEROS COMPLEJOS 657
133. Los nmeros complejos A, BB y C son los vrtices de un tringulo equiltero. Demuestre
que A
2
+B
2
+C
2
= A B +B C +C A.
134. Demuestre que una circunferencia de centro C y radio r, se puede expresar por P =
C +
1 +it
1 it
r, donde t es una variable real.
135. Dados los nmeros complejos A y B, determine un nmero complejo C, de tal modo que
el tringulo ABC sea equiltero.
136. Calcule la parte real e imaginaria de
z
1 +z
2
donde z Ci; i.
Resp:
x +x
3
3xy
2
(1 +x
2
y
2
)
2
+ 4x
2
y
2
y
x 3x
2
y +y
3
(1 +x
2
y
2
)
2
+ 4x
2
y
2
137. Calcular

(2 +

5i)(1 +

3i)
3

5 +

3i

.
Resp: 6

2
138. Calcule los nmeros complejos z tales que w =
2z i
2 +iz
es:
a) Un nmero real; b) Un nmero imaginario puro.
Resp: a) z est en la circ unferencia de centro
_
0,
5
4
_
y radio
3
4
; b) w es imaginario
puro si, y slo si, x = 0, es decir, z est en el eje imaginario.
139. Calcule los nmeros complejos z tales que w =
z 1 i
z + 1 +i
:
a) Es un nmero real; b) Tiene mdulo 1.
Resp: a) w es real si, y slo si, y = x ,= 1, es decir, z est en la bisectriz de los
cuadrantes primero y tercero y z ,= (1 +i); b) z est en la bisectriz de los cuadrantes
segundo y cuarto.
140. La suma de dos nmeros complejos variables z
1
y z
2
dividida por la diferencia de ellos da
un imaginario puro. Demuestre que los nmeros complejos z
1
y z
2
se desplazan sobre una
circunferencia con centro en el origen.
141. Resuelva la ecuacin cuadrtica az
2
+ bz + c = 0 donde a, b, c, son nmeros complejos
conocidos y a ,= 0.
142. Un nmero complejo z = x + iy se mueve sobre la recta 3x + 4y + 5 = 0. Demuestre que
el valor mnimo de [z[ es uno.
143. Dados dos nmeros complejos distintos a, b C, justique que para z ,= b el nmero
z a
z b
es real si, y slo si, z est en la recta que pasa por a y por b; y es real negativo si, y slo si,
z est en el segmento que une a con b.
CAPTULO 15. NMEROS COMPLEJOS 658
144. Dados dos nmeros complejos a, b y un nmero positivo q ,= 1, justique que el conjunto
_
z C/

z a
z b

= q
_
representa una circunferencia en el plano cuyo centro y radio debes
calcular.
145. a) Sea [z
1
[ = [z
2
[ = [z
3
[ = 1. Pruebe que z
1
, z
2
, z
3
son vrtices de un tringulo equiltero
si, y slo si, z
1
+z
2
+z
3
= 0.
b) Deduzca de lo anterior que si el baricentro y el circuncentro de un tringulo coinciden,
dicho tringulo debe ser equiltero.
146. Indique condiciones que deben cumplir los nmeros complejos a, b y c para que las races
de la ecuacin az
2
+bz +c = 0 formen con el origen un tringulo equiltero.
147. Realice las operaciones indicadas y expresa el resultado en forma binmica:
a) (7 2i)(5 + 3i); b) (i 1)
3
; c) i
2
(1 +i)
3
; d)
3 +i
2 +i
; e)
1 + 2i
2 i
;
f ) (1 +i)
2
; g)
(4 i)(1 3i)
1 + 2i
; h) (1 +i)(2 +i)(3 +i).
148. Calcule las siguientes cantidades;
a) [(1 +i)(2 i)[; b)

4 3i
2

5i

; c)

(1 +i)
20

; d) [

2 + (

2 + 1)i[.
149. Calcule los nmeros complejos z ,= 1 tales que
1 +z
1 z
es:
a) Un nmero real; b) Un nmero imaginario puro.
150. Demuestre la llamada igualdad del paralelogramo:
[z +w[
2
+[z w[
2
= 2([z[
2
+[w[
2
), z, w C
y explique su signicado geomtrico.
151. Dados dos nmeros complejos a y b, calcule el mnimo valor para z C de la cantidad
[z a[
2
+[z b[
2
.
152. Demuestre que

x a
1 az

< 1 si, y slo si, [z[ < 1 y [a[ < 1 o bien [z[ > 1 y [a[ > 1.
153. Si z, w C y [z[ = 1, demuestre que

z +w
1 +z w

= 1.
154. Calcular:
a)

z
w

, si
z +w
z w
= 1 + 4i tal que z, w C;
b)

1
z
+
1
u

, si z = 3 4i, u = 4 + 3i;
CAPTULO 15. NMEROS COMPLEJOS 659
c)

1
z z
2

, si z = 2i.
Resp: a)

5
2
; c)

5
10
.
155. Justique que si z
1
, z
2
, ..., z
n
son nmeros complejos de mdulo 1 y tales que [z
1
+ z
2
+
... +z
n
[ = n entonces se verica que todos son iguales z
1
= z
2
= ... = z
n
.
156. Sean z
1
, z
2
, z
3
C distintos dos a dos. Explicar el signicado geomtrico de las relaciones:
Im
z
3
z
1
z
2
z
1
= 0 y Re
z
3
z
1
z
2
z
1
= 0
157. Sean z
1
, z
2
C con z
1
,= 0. Demuestre que
z
2
z
1
es imaginario si y slo si [z
1
+z
2
[ = [z
1
z
2
[.
Deducir que un paralelogramo tiene sus diagonales iguales si y slo si es un rectngulo.
158. Demuestre que el tringulo cuyos vrtices son los puntos z
1
, z
2
, z
3
es equiltero si y slo si
z
2
1
+z
2
2
+z
2
3
= z
1
z
2
+z
2
z
3
+z
3
z
1
.
159. Cul es el lugar geomtrico de los puntos del plano complejo tales que [z +16[ = 4[z +1[?
160. Demuestre que si las races de una ecuacin son los vrtices de un paralelogramo cuyo
centro es el origen de coordenadas, la ecuacin es bicuadrada.
161. La suma de dos nmeros complejos z
1
y z
2
dividida por la diferencia de ellos da un nmero
imaginario puro. Demuestre que los nmeros complejos z
1
y z
2
se desplazan sobre una cir-
cunferencia con centro en el origen.
162. Un nmero complejo z = a + ib se mueve sobre recta 3a + 4b + 5 = 0. Demuestre que el
valor mnimo de [z[ es uno.
15.4. Potencia de un nmero complejo
15.4.1. Potencia de la unidad imaginaria
Es preciso aclarar la propiedad fundamental del nmero complejo i. Es fcil comprobar que
i
2
= i i = (0 + 1i)(0 + 1i) = 1 + 0i = 1.
De tal modo, se puede actuar con los nmeros complejos segn las mismas reglas que se aplican
para las operaciones con los nmeros reales, en este caso hay que sustituir siempre i
2
por -1. La
igualdad i
2
= 1 se puede interpretar como sigue: el nmero i es la raz de la ecuacin x
2
+1 = 0.
Precisamente ester problema consiste en la solucin de la ecuacin x
2
+ 1 = 0 que no tiene races
reales, sirvi de motivo para crear la teora de los nmeros complejos.
CAPTULO 15. NMEROS COMPLEJOS 660
Utilizando la igualdad i
2
= 1, se puede determinar fcilmente una potencia entera positiva
cualquiera de la unidad imaginaria. As, tendremos:
i
3
= i
2
i = 1i = i; i
4
= i
2
i
2
= 1; i
5
= i
4
i = i; i
6
= i
4
i
2
= 1; i
7
= i; i
8
= 1, etc.
Esto demuestra que los valores de la potencia i
n
, donde n es un nmero entero positivo, se
repiten peridicamente al aumentar el exponente en 4. Por eso, para elevar el nmero i a una
potencia entera positiva, hay que dividir el exponente por 4 y elevar i a la potencia cuyo ndice es
igual al resto de la divisin.
Ejemplo 15.30 Calcular i
17
+i
18
+i
19
+i
20
Solucin
i
17
+i
18
+i
19
+i
20
= (i
2
)
8
i + (i
2
)
9
+ (i
2
)
9
i + (i
2
)
10
= (1)
8
i + (1)
9
+ (1)
9
i + (1)
10
= i 1 i + 1
= 0.
15.4.2. Potenciacin de un nmero complejo
La elevacin de un numero complejo a una potencia entera positiva se realiza por la regla de
potenciacin de un binomio, puesto que es un caso particular del producto de factores complejos
iguales.
Ejemplo 15.31 Calcular z
2
+z = 0
Solucin
z
2
+z = (a +bi)
2
+ (a bi)
2
= a
2
+ 2abi b
2
+a bi
= (a
2
b
2
+a) + (2ab b)i
= 0 + 0i
De donde establecemos el sistema siguiente:
_
a
2
b
2
+a = 0
2ab b = 0

_
a
2
b
2
+a = 0
b(2a 1) = 0

_
a
2
b
2
+a = 0
b = 0, 2a 1 = 0
Es muy fcil resolver estos sistemas:
_
a
2
b
2
+a = 0
b = 0, a =
1
2

_
b = 0
_
a = 0
a = 1
a =
1
2
_
b =

3
2
b =

3
2
CAPTULO 15. NMEROS COMPLEJOS 661
15.4.3. Extraccin de la raz cuadrada de un nmero complejo
Supongamos que se quiere extraer la raz cuadrada del nmero a+bi. Quiere decir que debemos
hallar un nmero complejo x = yi tal que su cuadrado sea igual a a +bi. Tendremos que:

a +bi = x +yi
donde x e y son nmeros reales. En tal caso
a +bi = (x +yi)
2
= x
2
y
2
+ 2xyi.
Utilizando la condicin de igualdad de dos nmeros complejos, obtendremos:
x
2
y
2
= a, 2xy = b.
Resolvemos este sistema con respecto a las incgnitas x e y. De la segunda ecuacin hallamos que
y =
b
2x
. En tal caso
x
2

b
2
4x
2
= a
de donde
4x
4
b
2
4ax
2
= 0 4x
4
4ax
2
b
2
= 0
por lo tanto
x
2
=
2a

4a
2
+ 4b
2
4
x
2
=
a

a
2
+b
2
2
Puesto que

a
2
+b
2
, ante el radical hay que tomar el signo ms para que x
2
sea un nmero positivo
o cero; por lo tanto,
x
2
=
a

a
2
+b
2
2
Sustituimos este valor de x
2
en la ecuacin x
2
y
2
= a y obtendremos:
y
2
=
a +

a
2
+b
2
2
Los valores de x e y los hallamos de las igualdades
x
2
=
a +

a
2
+b
2
2
e y
2
=
a +

a
2
+b
2
2
x =

a +

a
2
+b
2
2
e y =

a +

a
2
+b
2
2
La ecuacin 2xy = b demuestra que el producto xy tiene el mismo signo que el nmero b. Por lo
tanto, si b > 0, x e y tienen signos iguales; si b < 0, x e y tienen diferentes signos. Por eso, para
b > 0 tendremos:

a +bi =
_
_

a +

a
2
+b
2
2
+i

a +

a
2
+b
2
2
_
_
Para b < 0, tendremos:

a +bi =
_
_

a +

a
2
+b
2
2
i

a +

a
2
+b
2
2
_
_
CAPTULO 15. NMEROS COMPLEJOS 662
En la prctica estas frmulas no se utilizan, sino se realiza el paso dado de los clculos de x e y en
cada caso por separado.
Ejemplo 15.32 Extraer la raz cuadrada del nmero complejo

1 + 2i.
Solucin
Aplicamos la denicin

1 + 2i = x +yi
Elevamos al cuadrado ambos miembros de la igualdad y obtenemos
1 + 2i = x
2
y
2
+ 2xyi
Igualamos la parte real y la parte imaginaria y establecemos el sistema de ecuaciones
_
x
2
y
2
= 1
xy = 1
resolviendo este sistema, obteniendo
_
x
2

1
x
2
= 1
y =
1
x

_
x
4
x
2
1 = 0
y =
1
x

_
_
_
x =
_
1+

5
2
y =
_
1+

5
2
Finalmente

1 + 2i =
_
_

1 +

5
2
+i

1 +

5
2
_
_
Ejemplo 15.33 Calcular el valor de (1 +w)
3
+(1 +w
2
)
9
si w es la raz cbica de 1 y distinta
de la unidad.
Solucin
Como w es raz cbica de 1 entonces w
3
= 1, de aqu obtenemos w
3
1 = 0, factorizando tenemos
(w 1)(w
2
+w + 1) = 0. Como w ,= 1 entonces w
2
+w + 1 = 0 as, 1 +w = w
2
y 1 +w
2
= w
entonces
(1 +w)
3
+ (1 +w
2
)
9
= (w
2
)
3
+ (w)
9
= w
6
w
9
= (w
3
)
2
(w
3
)
3
= 2.
Ejemplo 15.34 Escriba el nmero complejo
z =

5 + 12i

5 12i

5 + 12i +

5 12i
En forma algebraica a condicin de que las partes reales de las races

5 + 12i y

5 12i son
negativas.
Solucin
Para extraer la raz cuadrada del nmero 5 + 12i hagamos

5 + 12i = x +iy
CAPTULO 15. NMEROS COMPLEJOS 663
entonces
5 + 12i = x
2
+ 2xyi y
2
y, por lo tanto, x e y satisfacen el sistema de ecuaciones
_
x
2
y
2
= 5
xy = 6
Despus de resolverlo, obtenemos dos soluciones (3, 2) y (-3, -2). De acuerdo con el planteamiento,
la parte real de

5 + 12i es negativa y, por ello

5 + 12i = 3 2i
De forma anloga, hallamos

5 12i = 3 + 2i
As, pues
z =
3 2i (3 + 2i)
3 2i + (3 + 2i)
=
2i
3
.
15.5. Tarea
1. Simplicar las expresiones:
a) 5i
476
3i
258
+ 4i
327
8i
932
+ 4i
441
; b)
i
52
+i
421
+i
65
+i
74
+i
33
i
2541
+i
3244
+i
2460
+i
3581
+i
2723
;
c)
i
5
+i
15
+i
49
i
18
+i
400
+ 2i
14
i
6
+i
50
i
23
+i
35
i
441
; d)
(1 +i)
9
1 +i
9
; e)
(1 +i)
9
(1 i)
7
.
Respuesta: a) 0; b) 1; c) 3; d) 16; e) 2.
2. Simplicar la expresin
_
_

8
4

16
_
_
3n
(

1)
13n+5
(

1)
2n+8
.
Respuesta: 1.
3. Calcular el valor de i
2
+ 2i
4
+ 3i
6
+ 4i
8
+ 5i
10
+... + (4n)i
8n
.
Respuesta: 2n.
4. Calcular el valor de (

12 + 5i +

12 5i)(

4 + 3i +

4 3i).
Respuesta: 30.
5. Calcular el valor de
(1 +i)
3
(1 +i)
2
(1 i)
6
.
Respuesta:
1
4
.
6. Calcular el valor de (1 i
1
+i
2
i
3
+i
4
i
5
+... i
223
)
2
.
Respuesta: 0.
CAPTULO 15. NMEROS COMPLEJOS 664
7. Calcular el valor de 3i + 5i
2
+ 7i
3
+ 9i
4
+ 11i
5
+... + (8n + 1)i
4n
4n.
Respuesta: 4ni.
8. Calcular el valor de
x +y
x y
, si se cumple que (1 +i)
2
+(1 +i)
4
+(1 +i)
6
+(1 +i)
8
= x+yi.
Respuesta:
1
3
.
9. Calcular el menor valor de n que verica (1 +i)
n
= 32i.
Respuesta: n = 10.
10. Calcular el valor de n en la expresin
(1 +i)
7n
+C
n
1
(1 +i)
7n7
(1 i)
7
+C
n
2
(1 +i)
7n14
(1 i)
14
+... + (1 i)
7n
= 2
61
Respuesta: n = 180.
11. Calcular el valor de i + 2i
2
+ 3i
3
+ 4i
4
+... +ni
n
.
Respuesta: 2n.
12. Calcular el valor de i
2
+ 3i
4
+ 5i
6
+ 7i
8
+... + (2n 1)i
2n
.
Respuesta: n.
13. Calcular el valor de (1 +i)
200
(1 i)
200
.
Respuesta: 0.
14. Calcular el valor de
(1 +i)
n
(1 i)
n2
.
Respuesta: 2i
n1
.
15. Calcular x e y sabiendo que el polinomio a
2
+6a +2ai +xyi tiene raz cuadrada exacta.
Respuesta: x = 8, y = 6.
16. Calcular (1 +i) + (2 +i
2
) + (3 +i
3
) +... + (4n +i
4n
).
Respuesta: 2n(4n + 1).
17. Si
x
2
y
2
+ (x
2
+y
2
)i
x y + (x +y)i
= 3, hallar el valor de x.
Respuesta: 3.
18. Si z
1
=
_
a
2b
b
2a
+ 2a
b
b
a
i y z
2
=
_
b
2a
a
2b
+ 2a
b
b
a
i, siendo z
1
+z
2
= ab(1 +b), calcu-
lar
b
a1
a
+
a
b1
b
.
Respuesta: 1.
CAPTULO 15. NMEROS COMPLEJOS 665
19. Simplique la expresin

a
ai
+ 1

a
ai
1

a
ai
+ 1 +

a
ai
1
+
(a
ai
b
bi
)(a
ai
+b
bi
)

a
2ai
1

b
2bi
1
+

b
bi
+ 1

b
bi
1

b
bi
+ 1 +

b
bi
1
hallar la relacin entre a y b.
Respuesta: a b = 0.
20. Simplicar la expresin
1
512
_
2 +

20
_
40 8

5
_
5
.
Respuesta: 64.
21. Hallar el mdulo del nmero complejo
(4+3i)
2
(1+i)
4
(

3+i)
5
.
Respuesta: r =
25
8
.
22. El cociente de dos nmeros complejos es imaginario puro, su suma es real y vale 5. El
mdulo del dividendo es el doble que el del divisor. Hallar el divisor.
Respuesta: 1 2i.
23. Calcular la raz cuadrada de 5 + 12i.
Respuesta: 3 2i.
24. Hallar el valor de (1 w)
2
(1 w
2
)
2
(1 w
4
)
2
(1 w
5
)
2
.
Respuesta: 81.
25. Hallar el valor de (1 +w w
2
)
3
(1 w +w
2
)
3
.
Respuesta: 0.
26. Hallar el valor de (5 + 7w + 7w
2
)
12
.
Respuesta: 4096.
27. Simplicar la expresin w
273
+w
542
+w
115
+w
439
+w
855
+w
668
.
Respuesta: 0.
28. Simplicar la expresin (1 +w
2
)
10
+ (1 w +w
2
)(1 +w w
2
)w 5w, siendo w y w
2
las
races cbicas de la unidad.
Respuesta: 0.
29. Simplicar la expresin (1 + w + w
2
+ w
3
+ ... + w
25
)(1 w
5
+ w
10
w
15
+ ... + w
220
),
siendo w y w
2
las races cbicas complejas de la unidad.
Respuesta: 2w.
CAPTULO 15. NMEROS COMPLEJOS 666
30. Calcular el valor de n si (1 w)
2n
= 2187w.
Respuesta: n = 7.
31. Si 1, w, w
2
son las races cbicas de 1, hallar el valor de n que cumple con la igualdad
(1 +w) + (1 +w
2
)
2
+ (1 +w
3
)
3
+ (1 +w
4
)
4
+... + (1 +w
3n
)
3n
= 584.
Respuesta: n = 3.
32. Sabiendo que 1, w, w
2
son las races cbicas de 1, calcular el valor de
1 +w
1
+w
2
+w
3
+... +w
54
1 +
_
53
1
_
w +
_
53
2
_
w
2
+... +
_
53
52
_
w
52
+w
53
Respuesta: w
2
.
33. Sabiendo que 1, w, w
2
son las races cbicas de 1, calcular el valor de
(mn)(wmw
2
n)(w
2
mwn).
Respuesta: m
3
n
3
.
34. Sabiendo que 1, w, w
2
son las races cbicas de 1, calcular el valor de
(5 + 7w + 5w
2
)
9
+ (3 + 3w w
2
)
3
.
Respuesta: 448.
35. Calcular el valor de b para que la expresin
2a +bi
3 2i
+
2a + 3bi
3 + 2i
sea real.
Respuesta: 0.
36. Cul debe ser el valor de b para que se cumpla
(i 1)
1
(i + 1)
1
+ (i 1)
1
(1 i)
1
+ (i + 1)
1
(1 i)
1
= a +bi.
Respuesta:
1
2
.
37. Hallar el mdulo de
(2 + 3i)
3 5

1 i
(

5 + 2

2 i)
2
5

1 +i
.
Respuesta:

13
13
.
38. Hallar el mdulo de un nmero complejo, sabiendo que ste, su conjugado y el origen del
plano cartesiano forman un tringulo equiltero; adems la suma del nmero complejo con
su conjugada es 4.
Respuesta:
4

3
3
.
CAPTULO 15. NMEROS COMPLEJOS 667
39. Si z
1
= (a 3)i
3
+ (b 2)i
2
ai + 2b y z
2
= (b + 1)i
3
+ (1 a)i
2
+ 3i 1 son opuestos,
hallar b.
Respuesta: 1.
40. Simplicar la expresin (2 + 5w + 2w
2
)
3
(2 + 2w + 5w
2
)
3
.
Respuesta: 54.
41. Simplicar la expresin (1 +w w
2
)(1 +w
2
w
4
)(1 +w
4
w
8
)(1 +w
8
w
16
).
Respuesta: 4
3n
.
42. Simplicar las expresiones:
a) 2i
_
1
2
+

3
2
i
__

1
2
+

3
2
i
_
; b)
1 +i
1 i
+
1 i
1 +i
; c)
13 + 12i
6i 8
+
(1 + 2i)
2
2 +i
;
d)
(1 + 2i)
2
(1 i)
3
(3 + 2i)
3
(2 +i)
2
.
43. Hallar el nmero complejo z en forma algebraica:
a) z =
_
i
2

3
2
_
12
; b) z =
_
i
8
+

3 i
5
4
_
5
; c) z =
(1 +i)
9
(1 i)
7
;
d) z =
_
1

3 i
2
_
12
; e) z =
_
1 +

3 i
1 i
_
10
.
44. Calcule m y n para que se cumpla la identidad (2 +mi) + (n + 5i) = 7 2i.
Respuesta: m = 7 y n = 5.
45. Con qu valores enteros de n es vlida la igualdad (1 +i)
n
= (1 i)
n
?
46. Hallar todos los valores de
n

z si:
a) z = 8i, n = 3; b) z = 1 +i, n = 8.
47. Determine k para que el cociente
k +i
1 +i
sea igual a 2 i.
Respuesta: k = 3.
48. Calcule a y b de modo que se verique (a +bi)
2
= 3 + 4i.
Respuesta: a = 2, b = 1 y a = 2, b = 1.
49. Calcular a y b para que se verique a 3i =
2 +bi
5 3i
.
Respuesta: a =
11
5
y b =
108
5
.
CAPTULO 15. NMEROS COMPLEJOS 668
50. Resuelva las ecuaciones:
a) z
3
= 1 +i; b) z
4
+ 1 = 0; c) z
6
+ 64 = 0; d) z
2
= z
3
.
51. Determine con qu valores reales de x e y los nmeros complejos z = y
2
7y + 9xi y
u = 12 + 20i +x
2
i son iguales.
52. Hallar la parte imaginaria de z si:
a) z = (2 i)
3
(2 + 11i); b) z =
2 3i
1 + 4i
+i
6
; c) z =
5 + 2i
2 5i

3 4i
4 3i

1
i
.
53. Determine con qu valores reales de x e y los nmeros complejos z = 8x
2
20i
9
y
u = 9x
2
4 + 10yi
3
son conjugados.
54. Resuelva la ecuacin (1 + 2i)(z i) + (4i 3)(1 iz) + 1 + 7i = 0.
55. En el plano complejo se dan los puntos z, u, v que son los vrtices de un tringulo. Hallar
el punto de interseccin de sus medianas.
56. En el plano complejo se dan los puntos z, u, v que son los tres vrtices consecutivos de
cierto paralelogramo. Hallar el cuarto vrtice de la gura.
57. En el plano complejo estn dados los puntos z = 6+8i, u = 43i. Hallar los nmeros com-
plejos que corresponden a los puntos en la bisectriz del ngulo formado con los vectores z y u.
58. Simplicar la expresin
_

2
2
+

2
2
i
_
4
.
59. Simplicar la expresin
(2 +i)
2
+ (1 i)
2
1
3
2
i
.
60. Calcular:
a) i
45
; b) i
52
; c) i
143
; d) i
86
.
61. Hallar los vrtices del polgono correspondiente a los ajos del complejo
5

i.
62. Hallar los vrtices del polgono correspondiente a los ajos del complejo
6

1.
63. Hallar los vrtices del polgono correspondiente a los ajos del complejo
4
_
2

3 + 2i.
64. Dado el nmero complejo z =
1
2
+

3
2
i, compruebe que
1
z
= z
2
.
CAPTULO 15. NMEROS COMPLEJOS 669
65. Determine el valor de k para que (k 2i)
2
sea imaginario puro.
66. Simplicar las expresiones:
a)
2i
(4 3i)(1 2i)(2 3i)
; b)
2 3i
(1 + 2i)(1 2i)(3 2i)
; c)
(2 + 5i)
4
+i
(2 5i)
4
i
;
d)
(1 2i)(2 i)
(4 2i)(1 3i)(2 +i)
; e)
(1 + 3i)
2
(1 + 2i)
2
(2 3i)
2
(3 2i)
2
; f )
3i + (1 i)
3
(1 +i)
2
(1 i)
2
+ (1 +i)
2
;
g)
1 2i
(1 + 2i)(2 i)(2 2i)
; h)
(3 + 4i)
2
(4 3i)
2
(3 4i)
2
+ (4 + 3i)
2
; i)
(3 5i)(1 + 3i)
(1 3i)
2
+ (1 + 2i)
2
.
67. Calcular z =
(1 +i)
5
1 +i
5
.
68. Separar la parte real y la parte imaginaria del nmero complejo z =
8
(1 i)
5
.
69. Determine los nmeros reales x e y que satisfacen las ecuaciones:
a)
x(2 i)
2
+y(3 2i)
2
i
2 + 3i
= 3 2i; b)
_
2 i
2 +i
_
2

1
2x iy
= 2 i.
70. Obtener z, de los siguientes literales:
a) z =
3(3 2i)
(1 + 2i)
5

6i
1280
(

2 + 3i)
2
+
3i
2
; b) z =
i
5
i
11
+i
17
3 i
5
+i
12
i
16
.
71. Obtener z, de los siguientes literales:
a) z =
2ai 2a
a 2bi
+
2b 5bi + 3ai
2ai + 3b
; b) z = 35(i
25
i
85
+i
105
)
3
2(2 i)
3
+6(3 +2i)
3
.
72. Calcular:
a)
(2 + 3i)
11
(2 3i)
9
; b)
(2 i)
5
+ 1
(2 +i)
5
1
; c)
2i (3i 1)
3
2i + (3i + 1)
3
; d)
(i 1)
5
(i + 1)
5
(i + 1)
5
+ (i 1)
5
.
73. Si z
1
= 2 +i, z
2
= 2 2i, z
3
= 1 + 3i. Hallar el valor numrico de las expresiones:
a)

2z
1
z
2
3
2z
1
+z
2
+ 3

; b)
z
2
z
2

z
3
z
3
; c) Im
_
z
1
z
3
z
2
_
; d) Re
_
3z
2
+ 2z
2
1
3z
2
3
_
.
74. Simplicar la expresin
(2

3 i)
3
(1 2i)(

3 i)
(3 3

3 i)(1 i)
2
.
75. Hallar x, y R tales que z = x+iy sea una raz cuadrada de 25i, es decir, (x+iy)
2
= 25i.
En general, dados a, b R, si z = x + iy es una raz cuadrada de a + ib, es decir,
(x +iy)
2
= a +ib, expresar x e y en funcin de a y b.
76. Hallar z tal que
2 3i
2z(1 + 2i)
= 6 3i.
77. Si z =
(2 i)(

2 +i)(2 3i)
(1 + 2i)(

2 i)
, determine [z[.
CAPTULO 15. NMEROS COMPLEJOS 670
78. Calcular z
3
, siendo z = [4 3i[

3 i.
79. Si z C, resolver [z 2i[
2
+z 1 = 2 3i, Re(z) 0.
80. Hallar z = x +iy que satisfacen la condicin dada:
a) [z+4i[[zi[ < 5; b) [z+5[[z2[ 6; c)

z 3i
z + 3i

< 3; d)

z + 4i
z 4i

> 1.
81. Expresar en la forma a +ib el nmero complejo z = 10
2 log
2i
1i
.
82. Determine una ecuacin de segundo grado a coecientes reales, que admita la solucin
z =
i
1 +i +
i
1i+
i
1+i
83. Determine la parte real y la parte imaginaria del nmero complejo z =

3 + 4i +

3 4i.
84. Demuestre que si la ecuacin z
2
+ (a + bi)z + (c + di) = 0 tiene una raz real, se verica
que d
2
abd +cb
2
= 0.
85. Determine todos los nmeros complejos z, tales que z
3
= 1.
86. Si w es una raz cbica compleja de la unidad, demuestre que
(1 +w)(1 + 2w)(1 + 3w)(1 + 5w) = 21
87. Determine las races de la ecuacin 4x
3
3x + 1 = 0 sabiendo que
(x +a +b)(c +aw +bw
2
)(x +aw
2
+bw) = x
3
3abx +a
3
+b
3
88. Resuelva la ecuacin z
6
+ 7z
3
8 = 0.
89. Determine las races de la ecuacin x
3
= 1, sabiendo que las races de x
3
= 1 son x
1
= 1,
x
2
=
1
2
(1 i

3) y x
3
=
1
2
(1 +i

3).
90. Determine las races de la ecuacin x
3
= i, sabiendo que las races de x
3
= 1 son x
1
= i,
x
2
=
1
2
(1 i

3) y x
3
=
1
2
(1 +i

3).
91. Si a es una raz compleja de la ecuacin z
n
1 = 0, demuestre que 1+a+a
2
+...+a
n1
= 0.
92. Calcular S = 1 +i +i
2
+i
3
+... +i
n1
.
CAPTULO 15. NMEROS COMPLEJOS 671
93. Calcular S = 1 + 2i + 3i
2
+ 4i
3
+ 5i
4
+... + (4n)i
4n1
.
94. Determine un nmero complejo z = x +iy tal que z
2
= p +iq.
95. Calcular
3

1728.
96. Calcular las races de las expresiones:
a) (1 3i)
1
4
; b) (2 i)
1
5
; c) (3 +i)
1
6
; d) (2 3i)
1
2
; e) (3 + 2i)
1
5
;
f ) (

3 2i)
2
3
; g) (1 +

5 i)
3
5
; h) (

2 +

3 i)
2
5
.
97. Calcular:
a) (2 i)
n
; b) (

2 +i)
n
; c) (

3 +

2 i)
n
;
98. Calcular:
a) (

2 i)
n
; b) (

3 i)
n
; c) (1 +

3 i)
n
.
99. Sea k el mximo comn divisor de los enteros positivos m y n. Demuestre que las races
de x
k
= 1 son races de x
m
= 1 y x
n
= 1.
100. Calcular el mdulo del nmero complejo z =
(2 3i)
4
(1 i)
3
(5 +i)
3
.
101. Dados los nmeros complejos z y w con [z[ = 1, demuestre que

z +w
1 +zw

= 1.
102. Resolver en C la ecuacin z
3
+z
2
+z + 1 = 0.
103. Resolver en C la ecuacin (1 i)z
2
(6 4i)z + 9 7i = 0.
104. Determine las races cbicas de 1 +i.
105. Sea z =
1
2
+

3
2
i. Calcule:
a) z
2
; b) 1 +z +z
2
; c)
1 +z
(1 z)
2
+
1 z
(1 +z)
2
.
106. Sean z
1
=
1 +

3 i
1

3 i
y z
2
=
1

3 +i
. Calcule:
a) z
2
1
+z
2
2
; b) z
3
1
+z
3
2
.
107. Determine el conjunto de nmeros z tales que Re(z
3
)=Im(z
3
).
CAPTULO 15. NMEROS COMPLEJOS 672
108. Sean z
1
= 4 +4i y a
2
= 1

3 i nmeros complejos. Determine el mdulo y el argumento


de:
a) z
2
1
; b) z
3
1
; c)
z
1
z
2
; d)
z
2
z
1
.
109. Sean z
1
y z
2
nmeros complejos de mdulo 1, tales que z
1
z
2
,= 1. Muestre que
z
1
+z
2
1 +z
1
z
2
es un nmero real.
110. Calcule el nmero complejo z tal que

z 12
z 81

=
5
3
y

z 4
z 8

= 1.
111. Demuestre que
1
z
=
z
[z[
2
. Compruebe el resultado para z = 1 2i y z =

3 +i.
112. Sea z =
1
2
+

3
2
i. Demuestre que z
3u
= 1, z
3u+1
= z, z
3k+2
= z
2
, 1 +z +z
2
= 0.
113. Determine el nmero natural n de modo que (

3 +i)
n
sea:
a) Un nmero real positivo; b) Un nmero imaginario puro.
114. Determine las soluciones de la ecuacin z
3
= 4

2(1 +i). Utilizando las races cbicas de


la unidad, exprese las soluciones de la ecuacin anterior en forma algebraica.
115. Demuestre que (1 +i)
11
= 32(1 +i).
116. Resulva la ecuacin z
3
+
1
z
3
= 0.
117. Encuentre los nmeros complejos z tales que Re(z
2
)=Im(z
2
) y Re(z)+Im(z)=1.
118. Sean w, z C tales que w =
z a
az 1
con 0 < a < 1. demuestre que [w[ < 1 si y slo si
[z[ < 1.
119. Sea w =
1 +z
1 z
con w = u +iv y z = x +iy, x, y, u, v R. Demuestre que
x =
u
2
+v
2
1
(u + 1)
2
+v
2
, y =
2v
(u + 1)
2
+v
2
120. Sea z = a +ib. Demuestre que existen p, m y n independientes de z (z ,= 0, 1) tales que
(a
2
+b
2
)(a
2
+b
2
+ 2a + 1)
a
2
b
2
+a (1 + 2a)bi
= p +mz +nz
2
Hallar p, m y n.
CAPTULO 15. NMEROS COMPLEJOS 673
121. Sabiendo que [z
1
[ = [z
2
[ = ... = [z
n
[ = 1, demuestre que
[z
1
+z
2
+... +z
n
[ =

1
z
1
+
1
z
2
+... +
1
z
n

122. Demuestre que para todo par de nmeros complejos z y w se tiene


[z +w[
2
+[z w[
2
= 2[z[
2
+ 2[w[
2
123. Sabiendo que todo par de nmeros complejos z
1
y z
2
verica la igualdad
2[z
1
[
2
+ 2[z
2
[
2
= [z
1
+z
2
[
2
+[z
1
z
2
[
2
demuestre que todo par de complejos a y b verica la igualdad
[a +
_
a
2
b
2
[ +[a
_
a
2
b
2
[ = [a +b[ +[a b[
124. Sabiendo que todo par de complejos p y q verica la igualdad
[p +
_
p
2
q
2
[ +[p
_
p
2
q
2
[ = [p +q[ +[p q[
demuestre que todo par de complejos a y b verica la igualdad

a +b
2
+

ab

a +b
2

ab

= [a[ +[b[
125. Si a es un complejo de mdulo menor que uno. [a[ < 1, demuestre que

z a
1 az

es menor, igual o mayor que 1, segn sea [z[ menor, igual o mayor que uno.
126. La ecuacin x
3
9x
2
+33x65 = 0 tiene una raz compleja cuyo mdulo es 13. Determine
las tres races de la ecuacin.
127. Calcular todas las races que se indican:
a) Las races cbicas de z = 2 2

3 i; b) Las races cuartas de z = 1

2 i;
c) Las races quintas de z = 3

2 i.
128. Simplicar:
a)
4

1 2i

2 +i
; b)
6

1 + 2i

2 i
; c)
8
_
1 +i
1 i
.
129. Si w
1
y w
2
son races de la ecuacin x
2
+ 2 = 0, hallar el valor de:
a) w
2
1
w
2
2
; b) w
1
w
2
w
2
1
; c) w
2
1
+w
2
2
.
130. Si z = 2 2i y w = 2 + 2i, determine z
n
+w
n
, donde n es un nmero entero.
CAPTULO 15. NMEROS COMPLEJOS 674
131. Si z = (1

2)i (1 +

2), hallar Re(z


8
).
132. Sea p(x) =
n

k=0
a
k
x
k
un polinomio a coecientes reales, tal que p(z) = a + ib. Demuestre
que p(z) = a ib.
133. Determine la parte real y la imaginaria de cada uno de los nmeros complejos:
z =

i, w =
1

i
134. Demuestre que el producto de las n races de la ecuacin x
n
= a es p = (1)
n+1
a, a > 0.
135. Resuelva la ecuacin
_
1 +i

3
2
_
x
+
_

1 +i

3
2
_
x
= 2.
136. Demuestre que la suma de las races cbicas del nmero complejo z = 23(1 +i

3) es nula.
137. Si x es nmero real y n entero positivo, resuelva la ecuacin
_
1 +ix
1 ix
_
n
= 1.
138. Determine las races de la ecuacin (x + 1)
n
(x 1)
n
= 0.
139. Si w
1
,w
2
,w
3
, ..., w
n
son las races n-simas de la unidad, demuestre que
1
1 w
1
x
+
1
1 w
2
x
+... +
1
1 w
n
x
=
n
1 x
n
140. Si w es una raz n-sima primitiva de la unidad, demuestre que
1
1 x
+
w
w x
+
w
2
w
2
x
+... +
w
n1
w
n1
x
=
n
1 x
n
141. Si w
1
, w
2
, w
3
, ..., w
n
son las races n-simas de la unidad y k un entero, calcular la suma
S = w
k
1
+w
k
2
+w
k
3
+... +w
k
n
142. Si w es una raz cbica compleja de la unidad, demuestre que el tringulo cuyos vrtices
son z, zw, zw
2
es equiltero.
143. Los nmeros complejos z = x +iy y w = u +iv verican siempre la igualdad w = z +
a
2
z
,
a R.
144. Los nmeros complejos z = x +iy y w = u +iv, verican siempre la igualdad w = z
2
+a,
donde a es un nmero real. Determine el lugar geomtrico de w, cuando z recorre:
a) La circunferencia x
2
+y
2
= 1; b) La recta x = y
CAPTULO 15. NMEROS COMPLEJOS 675
145. Los nmeros complejos z y w verican siempre la relacin w = (4 +i) +
3 3i
z i
. Determine
el lugar geomtrico de w cuando z recorre:
a) La circunferencia [z[ = 1; b) El eje de ordenadas.
146. Los nmeros complejos A y B son las races de la ecuacin z
2
(8 + 5i)z + (8 + 26i) = 0.
Determine un nmero complejo C = x+iy, de tal modo que el tringulo ABC sea equiltero.
147. Los vrtices opuestos A y C de un rombo ABCD estn dados por las races de la ecuacin
z
2
6(1+i)z+16i = 0. Determine una ecuacin de segundo grado que d los otros dos vrtices.
148. En un cuadrado ABCD, los vrtices opuestos A y C son las races de la ecuacin z
2
(6+
8i)z + (1 + 30i) = 0. Determine los otros dos vrtices.
149. Calcule la parte real e imaginaria de
z
1 +z
2
donde z Ci; i.
Resp:
x +x
3
3xy
2
(1 +x
2
y
2
)
2
+ 4x
2
y
2
y
x 3x
2
y +y
3
(1 +x
2
y
2
)
2
+ 4x
2
y
2
150. Calcular

(2 +

5i)(1 +

3i)
3

5 +

3i

.
Resp: 6

2
151. Resuelva la ecuacin cuadrtica az
2
+ bz + c = 0 donde a, b, c, son nmeros complejos
conocidos y a ,= 0.
152. Calcule las soluciones de la ecuacin z
4
+ (1 +i)z
2
+ 5i = 0.
153. Calcule las soluciones de las ecuaciones:
a) z
4
+ 2z
3
+ 7z
2
18z + 26 = 0; b) z
4
+ (5 + 4i)z
2
+ 10i = 0.
154. Indique condiciones que deben cumplir los nmeros complejos a, b y c para que las races
de la ecuacin az
2
+bz +c = 0 formen con el origen un tringulo equiltero.
155. Realice las operaciones indicadas y expresa el resultado en forma binmica:
a) i
2
(1 +i)
3
; b)
(4 i)(1 3i)
1 + 2i
; c) (1 +i)
2
; d) (1 +i)(2 +i)(3 +i).
156. Calcule las siguientes cantidades:
a) [(1 +i)(2 i)[; b)

4 3i
2

5i

; c)

(1 +i)
20

; d) [

2 + (

2 + 1)i[.
CAPTULO 15. NMEROS COMPLEJOS 676
157. Exprese los siguientes nmeros complejos en forma binmica:
a) (1 +

3i)
11
; b)
_
1 +i
1 i
_
5
; c)
_
1 +

3i
1 i
_
6
; d) (

3 +i)
13
.
158. Calcule todas las soluciones de las siguientes ecuaciones:
a) z
3
= 1 +i; b) z
3
= 1 +

3i; c) z
8
= 1; d) z
2
+

32iz 6i = 0.
159. Demuestre la llamada igualdad del paralelogramo:
[z +w[
2
+[z w[
2
= 2([z[
2
+[w[
2
), z, w C
y explique su signicado geomtrico.
160. Dados dos nmeros complejos a y b, calcule el mnimo valor para z C de la cantidad
[z a[
2
+[z b[
2
.
161. Determine para cada una de las igualdades siguientes si hay algn nmero complejo z C
con [z[ = 1 que la verica:
a) [z
3
+z
2
+ 1[ = 3; b) [z
4
2z i[ = 4. c) [z
6
+z
3
+ 2[ = 4 +[4 + 4z
2
[.
162. Demuestre que el tringulo cuyos vrtices son los puntos z
1
, z
2
, z
3
es equiltero si y slo
si z
2
1
+z
2
2
+z
2
3
= z
1
z
2
+z
2
z
3
+z
3
z
1
.
163. Represente la curva y = x
4
+16 y calcule todas las races de la ecuacin x
4
+16 = 0. Saque
una conclusin en relacin con la grca dibujada.
15.6. Forma trigonomtrica
15.6.1. Forma trigonomtrica de un nmero complejo
Como ya se expreso anteriormente, el nmero complejo z = a+bi, distinto de cero, se representa
como un vector 0P; adems la longitud de este vector es el modulo del nmero complejo:
r =
_
a
2
+b
2
.
El ngulo entre el sentido positivo del eje 0X y el vector 0P se llama argumento del nmero
complejo z = a + bi. Este ngulo cuenta desde el eje 0X al vector 0P, lo que est indicado en el
dibujo por una echa. Si el nmero complejo es igual a cero, el vector 0P se convierte en un punto
y no hay necesidad de hablar de su sentido. Por eso, se considera que el nmero complejo nulo no
tiene argumento.
Denicin 15.9 Argumento de un nmero complejo
Recibe el nombre de argumento del nmero complejo z ,= 0 el ngulo entre el sentido positivo del
eje real y el vector z, con la particularidad de que dicho ngulo se considera positivo si contamos en
sentido antihorario y negativo, si contamos en sentido horario. Para el nmero z = 0 argumento
no se determina.
CAPTULO 15. NMEROS COMPLEJOS 677
A diferencia del mdulo, el argumento del nmero complejo se determina no unvocamente.
Cualesquiera dos argumentos de un nmero complejo se distinguen entre s por un nmero mlti-
plo de 2. Para designar el conjunto de todos los argumentos del nmero z = a + bi se hace uso
del smbolo Arg(z) o bien Arg(a +bi). Si se trata de cualquiera de los argumentos, ste, por regla,
se designa con la letra .
Es evidente que cada nmero complejo, distinto de cero, tiene un conjunto innito de valores
del argumento; estos valores se diferencian entre s en un nmero entero de vueltas completas,
es decir, en la magnitud 2k, donde k es un nmero entero cualquiera. El valor del argumento,
tomado en los lmites de la primera circunferencia, es decir, de 0 a 2 se llama principal.
Por la gura, tendremos:
_
a = rCos
b = rSen
de donde
a +bi = rCos +irSen
= r(Cos +iSen).
La expresin r(Cos + iSen) se llama forma trigonomtrica
del nmero complejo, a diferencia de la forma a + bi que se
llama algebraica.
Para determinar el argumento utilizamos las frmulas
_
Cos =
a

a
2
+b
2
Sen =
b

a
2
+b
2
En funcin del signo de las partes real e imaginaria se toma el correspondiente cuadrante, en el
que debe terminar el ngulo .
La forma trigonomtrica de un nmero complejo z, que tambin se le conoce como forma polar,
se abrevia poniendo
z = r(Cos +iSen)
= rCis.
Se sabe de Trigonometra que este sistema de ecuaciones tiene un conjunto innito de soluciones;
adems, si es una de las soluciones, todas las dems soluciones se deducen de la primera segn
la frmula:
=
0
+ 2k,
k es un nmero entero cualquiera. De esta manera, cualquier nmero complejo z ,= 0 tiene una
cantidad innita de argumentos y todos ellos pueden ser obtenidos de uno solo, segn la frmula
sealada.
Notemos que entre los argumentos del nmero complejo z siempre hay uno que satisface las
desigualdades 0 < 2; a veces, a este valos de se le denomina argumento del nmero z. No
CAPTULO 15. NMEROS COMPLEJOS 678
obstante, esta limitacin resulta a menudo incmoda. Vamos a seguir la denicin arriba expuesta
aplicando el trmino el valor principal del argumento para el valor de en el intervalo de 0 a
2. En correspondencia con esto, a continuacin, siempre que sea necesario hallar un argumento
de cualquier nmero complejo z, nos limitamos a buscar uno de sus argumentos (no es obligatorio
que sea el principal). Este argumento incgnito se designa frecuentemente por el smbolo Argz.
El argumento de un nmero complejo z adquiere el sigu-
iente sentido geomtrico. Si consideramos al nmero complejo
z = a +bi ,= 0 como el vector OM, el valor principal del argu-
mento del nmero z ser un ngulo , al cual hace falta girar
en sentido antihorario el semieje positivo Ox hasta que coinci-
da con el vector OM. El argumento del nmero considerado z
es una magnitud de cualquier ngulo que diere de ste en un
nmero entero de ngulos completos.
La forma trigonomtrica de los nmeros complejos est ligada estrechamente con su inter-
pretacin geomtrica; naturalmente, la frmula r(Cos+iSen) se deduce de las consideraciones
geomtricas. Cuando nosotros denimos el mdulo y el argumento del nmero complejo, su forma
trigonomtrica se dedujo automticamente. Prestemos atencin a que esta frmula se deduce, en
este libro de texto, valindose del dibujo en el cual el punto M(a, b) se encuentra en el primer
cuadrante. Sin embargo, esta frmula es vlida para cualquier situacin del punto M; el estudiante
tiene que saber demostrar su validez en cada caso.
Por ejemplo, el punto M(a, b) se encuentra en el segundo cuadrante, segn se seala en la gura.
En este caso OM
1
= rCos, OM
2
= rSen y = . Ya que para los puntos del segundo
cuadrante a < 0 y b > 0, entonces OM
1
= a, OM
2
= b y, por consiguiente
a = rCos = rCos( ) = rCos,
b = rSen = rSen( ) = rSen,
es decir
a +bi = rCos +irSen = r(Cos +iSen);
la frmula es vlida si el punto M se encuentra en el segundo cuadrante.
Los argumentos del nmero complejo z = a +bi, a ,= 0, se pueden determinar de la ecuacin
Tan =
b
a
Esta ecuacin no es equivalente al sistema dado, ella tiene mayor cantidad de races, pero la
seleccin de las races necesarias (argumentos del nmero complejo) no presenta dicultades, ya
que de la forma algebraica de anotacin del nmero complejo siempre se advierte en qu cuadrante
del plano complejo se encuentra l.
Teorema 15.11 Para todo nmero complejo no nulo, dado en forma polar, se tiene
a) Arg(z) = Arg(z);
b) Arg
_
1
z
_
= Arg(z);
c) Arg(z
1
+z
2
) = Arg(z
1
) +Arg(z
2
).
CAPTULO 15. NMEROS COMPLEJOS 679
Teorema 15.12 Para todo nmero complejo no nulo, dado en forma polar, se tiene
z = r[Cos() +iSen()]
Demostracin
En efecto, de
z = a +bi
= r(Cos +iSen)
resulta
_
a = rCos
b = rSen
entonces
z = a bi
= r(Cos iSen)
= r[Cos() +iSen()].
Denicin 15.10 Nmeros complejos iguales
Dos nmeros complejos escritos en forma trigonomtrica
z
1
= r
1
(Cos
1
+iSen
1
) y z
2
= r
2
(Cos
2
+iSen
2
)
son iguales si y slo si
r
1
= r
2
,
1
=
2
+ 2k, k Z
es decir, cuando los mdulos de los nmeros son iguales y los argumentos se diferencian en 2k,
donde k es cierto nmero entero.
Ejemplo 15.35 Hallar el argumento de un nmero complejo z
1
= z
2
z, si z = Cos+iSen.
Solucin
Unos clculos sencillos muestran que
z
1
= (Cos +iSen)
2
(Cos +iSen)
= Cos
2
Sen
2
+ 2iCosSen Cos iSen
= (Cos2 Cos) +i(Sen2 Sen)
= 2Sen

2
_
Sen
3
2
+iCos
3
2
_
.
De tal manera
[z
1
[ =

4Sen
2

2
_
Sen
2
3
2
+Cos
2
3
2
_
= 2

Sen

.
En correspondencia con la denicin del valor absoluto tenemos que considerar tres casos:
i) Si Sen

2
= 0, es decir, = 2k, donde k es un nmero entero cualquiera, entonces [z
1
[ = 0
y por eso tambin z
1
= 0. De esta manera, para = 2k (k es un nmero entero cualquiera) el
argumento del nmero z
1
queda determinado.
CAPTULO 15. NMEROS COMPLEJOS 680
ii) Si Sen

2
> 0, lo que tiene lugar para 2k <

2
< (2k + 1), es decir, cuando 4k < <
(4k + 2). k es un nmero entero cualquiera, entonces [z
1
[ = 2Sen

2
, de donde se sigue que la
forma trigonomtrica del nmero complejo z
1
ser la siguiente:
z
1
= 2Sen

2
_
Cos
+ 3
2
+iSen
+ 3
2
_
.
Por consiguiente, si satisface la condicin, entonces
Arg(z
1
) =
+ 3
2
.
iii) Si Sen

2
< 0, es decir (4k +2) < < (4k +4), k es un nmero entero cualquiera, entonces
[z
1
[ = 2Sen

2
_
Cos
3 + 3
2
+iSen
3 + 3
2
_
.
Por consiguiente, si satisface la condicin, entonces
Arg(z
1
) =
3 + 3
3
.
Es curioso dar una interpretacin geomtrica de la solucin que presentaremos slo en el caso de
0 < < . El nmero z
1
= z
2
z = z
2
+ (z) es una suma de dos nmeros complejos
z
2
= Cos2 +iSen2
y
z = Cos +iSen
= Cos( +) +iSen( +).
cuyos mdulos son iguales a 1. Para determinar su suma
hay que hallar la diagonal del paralelogramo construido a
base de los vectores OM
1
y OM
2
que representan respec-
tivamente los nmeros z
2
y z. Pues, este paralelogramo
es un rombo. Por consiguiente, la diagonal incgnita OM
3
es la bisectriz del ngulo, que forma el vector OM
1
y OM
2
por razn de que el ngulo, que forma el vector OM
3
con la
direccin positiva del eje Ox, es una semisuma de ngulos
formados por los vectores OM
1
y OM
2
con esta direccin,
es decir
Arg(z
1
) = Arg(z
2
z)
=
2 + +
2
=
+ 3
2
.
Ejemplo 15.36 Hallar los argumentos de los nmeros complejos z = i, w = 1, u = 1 +i.
Solucin
Habiendo construido los vectores z, w, u hallamos uno de los argumentos de cada nmero:

1
=

2
,
2
= 0,
3
=
3
4
CAPTULO 15. NMEROS COMPLEJOS 681
Por consiguiente
Arg(z) =

2
, Arg(w) = 2k, Arg(u) =
3
4
+ 2k
Donde k es un nmero entero tomado al azar.
Ejemplo 15.37 Hallar los argumentos del nmero complejo z = 1

3i.
Solucin
En este caso a = 1, b =

3. El sistema tiene la forma


_
Cos =
1
2
Sen =

3
2
Resolviendo este sistema, hallamos

k
=
4
3
+ 2k, k Z.
Por lo tanto,
Arg(z) =
4
3
+ 2k, k Z.
Ejemplo 15.38 Hallar los argumentos del nmero complejo z =

3 +i.
Solucin
Cada uno de los argumentos del nmero z =

3 + i satisface la ecuacin Tan =


1

3
. De
esta ecuacin se desprende que

k
=

6
+k, k Z.
Como el nmero z =

3 + i est situado en el segundo cuadrante del plano complejo, sus


argumentos sern los nmeros con los valores impares de k, de modo que
Arg(z) =

6
+(2n + 1)
=
5
6
+ 2n, n Z.
Ejemplo 15.39 Dnde se encuentran los puntos que satisfacen la condicin Arg(z) =

3
?
Solucin
Todos los puntos que se encuentran en el radio saliente del origen de las coordenadas bajo el ngulo

3
respecto al eje Ox, satisfacen esta condicin. Conviene subrayar, que no toda la recta sino que
un solo rayo, sin su origen, satisface esta condicin.
Ejemplo 15.40 Hallar la forma trigonomtrica del nmero complejo
z = 1 +iTan
donde < , , ,=

2
.
Solucin
Es natural escribir el nmero dado en la forma
z = 1 +iTan
=
1
Cos
(Cos +iSen).
CAPTULO 15. NMEROS COMPLEJOS 682
Este es el momento cuando muchos estudiantes cometen un error grave al armar que esta es
precisamente la forma trigonomtrica del nmero dado. Sin embargo, esto es correcto slo cuando
1
Cos
> 0, es decir, cuando

2
< <

2
(segn la condicin se analizan los valores de slo en el
intervalo de a ). Si
1
Cos
< 0, que tiene lugar para < <

2
y para

2
< < , entonces,
presentemos la igualdad escrita anteriormente en la forma:
z =
1
Cos
(Cos iSen)
=
1
Cos
[Cos( +) +iSen( +)].
La ltima expresin es precisamente la forma trigonomtrica del nmero z para < <

2
y
para

2
< < .
Este problema se puede resolver tambin valindose de la regla general para hallar la forma
trigonomtrica; con este n hay que hallar el mdulo y el argumento del nmero z. El mdulo
de este nmero se halla de la siguiente forma:
r = [z[
=
_
1 +Tan
2

=
1
[Cos[
.
y el argumento es cualquier solucin del sistema:
_
Cos = [Cos[,
Sen = Tan [Cos[.
(15.1)
Para la solucin de este sistema necesitamos examinar dos casos:
a) Cos > 0, es decir, se encuentra en el intervalo

2
< <

2
. En este caso [Cos[ = Cos,
y el sistema (17) toma el aspecto
_
Cos = Cos,
Sen = Sen.
Es evidente que una de las soluciones de este sistema es = y, por consiguiente, para

2
< <

2
la forma trigonomtrica ser:
z =
1
Cos
(Cos +iSen).
b) Cos < 0, es decir, se encuentra en el intervalo < <

2
, o bien, en el intervalo

2
< < . En este caso [Cos[ = Cos, y el sistema (17) toma la forma
_
Cos = Cos,
Sen = Sen.

_
Cos = Cos( +),
Sen = Sen( +).
Su solucin ser, en particular, = +. Por consiguiente, para < <

2
y para

2
< <
la forma trigonomtrica ser:
z =
1
[Cos[
[Cos( +) +iSen( +)].
CAPTULO 15. NMEROS COMPLEJOS 683
Ejemplo 15.41 Hallar la forma triginomtrica del nmero complejo z = 6 8i.
Solucin
Est claro que [z[ = 10 y Tan =
b
a
=
4
3
. Como se ve en la gura, Arg(z) = +, donde es un
ngulo agudo tal que Tan =
4
3
. Por eso = ArcTan
4
3
, es decir, = +ArcTan
4
3
, por razn de
que la forma trigonomtrica tiene un aspecto:
z = 10(Cos +iSen), = +ArcTan
4
3
.
Notemos que el concepto de la forma trigonomtrica de un nmero complejo, distinto de cero,
est denido con absoluta exactitud: precisamente, esta es la notacin del nmero complejo z 0
en la forma de
z = r(Cos +iSen)
donde r, que es el mdulo del nmero z, es nmero positivo, y el coseno y el seno se toman del
mismo ngulo , que es el argumento del nmero z; entre ellos est obligatoriamente el signo +.
Por ejemplo, veamos los siguientes nmeros complejos:
z
1
= Cos

4
+iSen
_

4
_
, z
2
= 2
_
Cos

3
+iSen

3
_
,
z
3
= Cos

2
iSen

2
, z
4
= Sen30

+iCos30

.
que no son escritos en forma trigonomtrica. La forma trigonomtri-
ca de estos nmeros complejos sern formas de su anotacin,
que a continuacin se dan:
z
1
= Cos
7
4
+iSen
7
4
, z
2
= 2
_
Cos
4
3
+iSen
4
3
_
,
z
3
= Cos
_
2

2
_
+iSen
_
2

2
_
, z
4
= Cos60

+iSen60

.
Las cuestiones expuestas se plantean a menudo en forma de problemas; adems de esto, una
serie de propiedades esenciales de los nmeros complejos, que son muy tiles, se deducen al analizar
las operaciones con nmeros complejos en forma trigonomtrica.
15.6.2. Producto de nmeros complejos dados en forma trigonomtrica
Multipliquemos los dos nmeros complejos:
z
1
= r
1
(Cos
1
+iSen
1
) y z
2
= r
2
(Cos
2
+iSen
2
).
Obtendremos:
z
1
z
2
= r
1
(Cos
1
+iSen
1
)r
2
(Cos
2
+iSen
2
)
= r
1
r
2
Cos
1
Cos
2
+ir
1
r
2
Sen
1
Cos
2
+ir
1
r
2
Cos
1
Sen
2
r
1
r
2
Sen
1
Sen
2
= r
1
r
2
[Cos(
1
+
2
) +iSen(
1
+
2
)]
= r
1
r
2
Cis(
1
+
2
).
El resultado nos muestra que el modulo del producto es igual al producto de los mdulos de los
factores, y el argumento del producto es igual a la suma de los argumentos de los factores.
CAPTULO 15. NMEROS COMPLEJOS 684
Por consiguiente,
[zw[ = r
1
r
2
, Arg(zw) =
1
+
2
+ 2k, k Z
As, pues, el mdulo del producto de dos nmeros complejos es igual al producto de los mdulos
de dichos nmeros, la suma de los argumentos de los factores es el argumento del producto.
En la gura, el vector 0Q corresponde al nmero complejo z
1
= r
1
(Cos
1
+ iSen
1
) y el
vector 0R, al nmero z
2
= r
2
(Cos
2
+ iSen
2
). El vector 0P corresponde al producto z
1
z
2
=
r
1
r
2
[Cos(
1
+
2
) + iSen(
1
+
2
)]. El vector 0P se obtiene del vector 0Q girando el ngulo
2
y variando su longitud r
1
, r
2
veces. Si r
2
> 1, se dice que el vector 0Q se dilata, y si r
2
< 1, se
contrae. En el caso particular, cuando el nmero complejo z
1
se multiplica por i, el vector 0Q gira
un ngulo recto

2
, conservando, en este caso, la longitud r
1
sin variacin.
15.6.3. Divisin de nmeros complejos dados en forma trigonomtrica
Hallamos el mdulo y el argumento del cociente
z
1
z
2
=
r
1
(Cos
1
+iSen
1
)
r
2
(Cos
2
+iSen
2
)
Multiplicando el numerador y el denominador del segundo miembro por Cos
2
iSen
2
; obten-
dremos
z
1
z
2
=
r
1
(Cos
1
+iSen
1
)(Cos
2
iSen
2
)
r
2
(Cos
2
+iSen
2
)(Cos
2
iSen
2
)
=
r
1
(Cos
1
+iSen
1
)(Cos
2
iSen
2
)
r
2
(Cos
2

2
+Sen
2

2
)
=
r
1
r
2
[Cos(
1

2
) +iSen(
1

2
)]
=
r
1
r
2
Cis(
1

2
).
Por lo tanto, el mdulo del cociente es igual al cociente de los mdulos del dividendo y divisor, y
el argumento del cociente, es igual a la diferencia de los argumentos del dividendo y del divisor.
CAPTULO 15. NMEROS COMPLEJOS 685
Utilizando esta regla se puede demostrar que
(Cos +iSen)
1
=
1
Cos +iSen
=
Cos0 +iSen0
Cos +iSen
= Cos() +iSen()
= Cos iSen.
Por consiguiente

z
1
z
2

=
r
1
r
2
, Arg
_
z
1
z
2
_
=
1

2
+ 2k, k Z
As, pues, el mdulo del cociente de dos nmeros complejos es igual al cociente de los mdulos
de dichos nmeros, la diferencia de los argumentos del dividendo y el divisor es el argumento del
cociente.
Teorema 15.13 Para todo nmero complejo no nulo, dado en forma polar, se tiene
1
z
=
1
r
[Cos() +iSen()]
Demostracin
En efecto
1
z
= z
1
=
z
[z[
2
=
1
r
[Cos() +iSen()]
Teorema 15.14 Para todo nmero complejo no nulo, dado en forma polar, se tiene
Arg
_
z
1
z
2
_
= Arg(z
1
) Arg(z
2
)
15.6.4. Potenciacin de un nmero complejo dado en forma trigonomtri-
ca
Puesto que la n-sima potencia, donde n es un nmero entero positivo, es el producto de n
factores iguales, por la regla de la multiplicacin de nmeros complejos obtenemos
[r(Cos +iSen)]
n
= r
n
(Cos +iSen)
n
= r
n
(Cos +iSen)(Cos +iSen)...(Cos +iSen)
. .
n veces
= r
n
[Cos( + +... +
. .
n veces
) +iSen( + +... +
. .
n veces
)
= r
n
(Cosn +iSenn).
Despus de la reduccin, tendremos que:
(Cos +iSen)
n
= Cosn +iSenn
CAPTULO 15. NMEROS COMPLEJOS 686
Esta forma se llama frmula de Moivre. En particular, nos permite obtener el coseno y el seno de
los arcos, mltiplos del dado.
Supongamos que n = 2, en tal caso
(Cos +iSen)
2
= Cos2 +iSen2
o bien,
Cos
2
Sen
2
+ 2iSenCos = Cos2 +iSen2
de donde
Cos2 = Cos
2
Sen
2
y Sen2 = 2SenCos
Cuando n = 3, tendremos que:
(Cos +iSen)
3
= Cos3 +iSen3
o bien
Cos
3
3CosSen
2
+i(3Cos
2
Sen Sen
3
) = Cos3 +iSen3
de donde
Cos3 = Cos
3
3CosSen
2

o bien
Cos3 = 4Cos
3
3Cos y Sen3 = 3Cos
2
Sen Sen
3

o tambin
Sen3 = 3Sen 4Sen
3

Si ambos miembros de la ltima igualdad del prrafo anterior los elevamos a la potencia n, obten-
dremos:
[(Cos +iSen)
1
]
n
= [Cos() +iSen()]
n
o bien
(Cos +iSen)
n
= Cos(n) +iSen(n)
La ltima igualdad muestra que
(Cos +iSen)
n
= Cosn +iSenn
se cumple tambin para los exponentes enteros negativos.
Ejemplo 15.42 Eleve a la novena potencia el nmero complejo z =

3 i.
Solucin
El mdulo de z es igual a 2 y uno de los argumentos es el ngulo =

6
, por lo que el mdulo
del nmero z
9
es igual a 2
9
y el argumento del nmero z, a 9 =
3
2
. Por lo tanto
(

3 i)
9
= 2
9
_
Cos
_

3
2
_
+isen
_

3
2
__
= 512i.
CAPTULO 15. NMEROS COMPLEJOS 687
Ejemplo 15.43 Calcular (1 +i)
42
.
Solucin
Como z =

2 (Cos135

+iSen135

), entonces
(1 +i)
42
=
_

2 (Cos135

+iSen135

)
_
42
= (

2)
42
[Cos(42 135)

+iSen(42 135)

]
= 2
21
(Cos5,670

+iSen5,670

)
= 2
21
[Cos(15 360 + 270)

+iSen(15 360 + 270)

]
= 2
21
(Cos270

+iSen270

)
= 2
21
i.
Ejemplo 15.44 Simplique la expresin
_
10

3+10i
5+5i
_
6
.
Solucin
Transformamos la expresin en su forma trigonomtrica
_
10

3 + 10i
5 + 5i
_
6
=
_
5 + 5i
10

3 + 10i
_
6
=
_
5

2 (Cos45

+iSen45

)
20(Cos30

+iSen30

)
_
6
=
_
5

2
20
(Cos(45

30

) +iSen(45

30

))
_
6
=
_

2
4
(Cos15

+iSen15

)
_
6
=
_

2
4
_
6
(Cos90 +iSen90)
=
1
8
i.
15.6.5. Radicacin de nmeros complejos dados en forma trigonomtrica
Supongamos que se quiere extraer la raz n-sima del nmero complejo Z = r(Cos+iSen).
Esto signica que se debe hallar un nmero complejo z = (Cos+iSen), que elevado a la n-sima
potencia nos d el nmero Z, es decir,
[(Cos +iSen)]
n
= r(Cos +iSen)
o bien

n
(Cosn +iSenn) = r(Cos +iSen).
Basndonos en la condicin de igualdad de dos nmeros complejos deducimos que sus mdulos
deben ser iguales, y los argumentos se pueden diferenciar en un nmero mltiplo de 2, es decir,
r =
n
; n = + 2k, donde k es un nmero entero, de donde obtenemos:
=
n

r, =
+ 2k
n
CAPTULO 15. NMEROS COMPLEJOS 688
De este modo, el resultado de la radicacin se presenta de la siguiente forma
z =
n
_
r(Cos +iSen)
=
n

r
_
Cos
+ 2k
n
+iSen
+ 2k
n
_
donde
n

r es el valor aritmtico de la raz.


Si en esta frmula damos al nmero k los valores 0, 1, 2, 3, ..., n1, obtendremos los siguientes
n valores de la raz:
si k = 0, z
0
=
n

r
_
Cos

n
+iSen

n
_
;
si k = 1, z
1
=
n

r
_
Cos
+ 2
n
+iSen
+ 2
n
_
;
si k = 2, z
2
=
n

r
_
Cos
+ 4
n
+iSen
+ 4
n
_
;

si k = n 1, z
n1
=
n

r
_
Cos
+ 2(n 1)
n
+iSen
+ 2(n 1)
n
_
.
Los argumentos de estos valores de la raz, es decir, los ngulos

n
;
+ 2
n
;
+ 4
n
; ... ;
+ 2(n 1)
n
van en orden creciente; se comprueba fcilmente que cada uno de ellos es menor que un ngulo
completo, o bien 2. Para ello es suciente demostrar que el mayor de ellos
+ 2(n 1)
n
< 2
En realidad, el valor principal del argumento de un nmero complejo es menor que un ngulo
completo: 0 < 2, y por eso
+ 2(n)
n
<
2 + 2(n 1)
n
= 2;
+ 2(n 1)
n
< 2
De la trigonometra se sabe que en los limites de una circunferencia dos ngulos distintos no pueden
tener simultneamente valores iguales del seno y valores idnticos del coseno; por lo tanto, todos los
n valores de la raz sern distintos. Con el aumento ulterior del nmero k (k = n, n + 1, n + 2, ...),
ya no se obtienen nuevos valores de la raz; por ejemplo, para k = n tendremos:
z
n
=
n

r
_
Cos
+ 2k
n
+iSen
+ 2k
n
_
=
n

r
_
Cos
_

n
+ 2
_
+iSen
_

n
+ 2
__
=
n

r
_
Cos

n
+iSen

n
_
= z
0
.
Se ha obtenido el mismo valor que para k = 0. Si k = n + 1, obtenemos z
1
, para k = n + 2,
obtenemos z
2
, etc.
CAPTULO 15. NMEROS COMPLEJOS 689
Ejemplo 15.45 Representar

i en forma trigonomtrica.
Solucin
Representemos i en forma trigonomtrica:
i = Cos

2
+iSen

2
entonces:

i =
_
Cos

2
+iSen

2
= Cos

2
+ 2k
2
+iSen

2
+ 2k
2
.
Para k = 0, obtendremos:

i = Cos

4
+iSen

4
=

2
2
+

2
2
i
=

2
2
(1 +i).
Para k = 1, obtendremos:

i = Cos
5
4
+iSen
5
4
=

2
2

2
2
i
=

2
2
(1 +i).
Ejemplo 15.46 Representar z =
4

1 en forma trigonomtrica.
Solucin
Tenemos que:
1 = Cos +iSen.
Luego
4

1 =
4

Cos +iSen
= Cos
+ 2k
4
+iSen
+ 2k
4
para k = 0, obtendremos:
z
1
= Cos

4
+iSen

4
=

2
2
(1 +i)
para k = 1, obtendremos:
z
2
= Cos
3
4
+iSen
3
4
=

2
2
(i 1)
CAPTULO 15. NMEROS COMPLEJOS 690
para k = 2, obtendremos:
z
3
= Cos
5
4
+iSen
5
4
=

2
2
(1 +i)
para k = 3, obtendremos:
z
4
= Cos
7
4
+iSen
7
4
=

2
2
(1 i).
Ejemplo 15.47 Hallar cuatro valores de x =
4

1.
Solucin
4

1 =
4
_
1(Cos0 +iSen0)
=
4

1
_
Cos
0 + 2k
4
+iSen
0 + 2k
4
_
= Cos
k
2
+iSen
k
2
= x
k
Si k = 0, 1, 2, 3, obtendremos:
x
1
= Cos0 +iSen0 = 1; x
2
= Cos

2
+iSen

2
= i
x
3
= Cos +iSen = 1; x
4
= Cos
3
2
+iSen
3
2
= i
Vamos a dar una interpretacin geomtrica a los resultados obtenidos. Construimos los puntos
correspondientes a los cuatro valores hallados. Estos sern los puntos P, Q, R, S, que representan
los vrtices del cuadrado inscrito en la circunferencia dada.
De un modo semejante, extrayendo la raz cbica de 1, hallamos tres nmeros complejos:
Cos0 +iSen0, Cos
2
3
+iSen
2
3
y Cos
4
3
+iSen
4
3
.
Si construimos sus puntos correspondientes, stos estarn sobre una circunferencia de radio unitario
y sern los vrtices de un triangulo equiltero inscrito.
CAPTULO 15. NMEROS COMPLEJOS 691
Geomtricamente la extraccin de la raz n-sima de 1 se reduce a la construccin de un
polgono regular de n vrtices inscrito en el circulo de radio unitario; adems, si n es impar, uno
de los vrtices se encontrara sobre el eje de abscisas a la derecha de 0; si n es par, se tendrn dos
vrtices sobre el eje de abscisas.
Ejemplo 15.48 Interpretar geomtricamente el producto (2 + 2i)i.
Solucin
Al nmero complejo z
1
= 2 + 2i corresponde el vector r
1
= (2, 2); adems, el vector r
1
forma con
el eje 0X el ngulo
Tan = 1 =

4
.
El nmero complejo i se representa por el vector unitario r
2
= (0, 1), dirigido bajo el ngulo

2
al
eje 0X. De acuerdo a la explicacin dada anteriormente, el producto de (2 + 2i)i signica el giro
del vector r
1
el ngulo

2
en sentido contrario al movimiento de las agujas del reloj, conservndose
la longitud del vector.
Por lo tanto, al producto dado corresponde el vector r
3
, que forma con el eje un ngulo igual a

4
+

2
=
3
4
.
La longitud del vector r
3
es igual a la longitud del vector r
1
, es decir,
[r[ =
_
2
2
+ 2
2
= 2

2.
Ejemplo 15.49 Escriba los nmeros complejos z = 1 i, u = 2, w = i en forma
trigonomtrica.
Solucin
Como [z[ =

2 y
1
=
3
4
, entonces
z =

2
_
Cos
_

3
4
_
+iSen
_

3
4
__
El mdulo de u es igual a 2, y uno de los argumentos de u es el ngulo
2
= , por ello
u = 2(Cos +iSen)
Tomando en consideracin que [w[ = 1,
3
=

2
es uno de los argumentos de w, obtenemos
w = Cos

2
+iSen

2
.
Ejemplo 15.50 Escriba los nmeros
z = 2Cos
7
4
2iSen

4
y u = Cos

17
+iSen

17
en forma trigonomtrica.
Solucin
Para escribir z y u en forma trigonomtrica no hay necesidad de hallar previamente sus mdulos
y argumentos, aunque hacerlo no representa ninguna dicultad. Hagamos uso de que
Cos
7
4
= Cos
_

4
_
CAPTULO 15. NMEROS COMPLEJOS 692
en tanto que
Sen

4
= Sen
_

4
_
y obtenemos de inmediato la forma trigonomtrica del primer nmero
z = 2
_
Cos
_

4
_
+iSen
_

4
__
Por analoga, teniendo en cuenta que
Cos

17
= Cos
_


17
_
= Cos
16
17
en tanto que
Sen

17
= Sen
_


17
_
= Sen
16
17
obtenemos
u = Cos
16
17
+iSen
16
17
.
Ejemplo 15.51 Hallar el producto de los nmeros
z =

2
_
Cos
11
4
+iSen
11
4
_
y u =

8
_
Cos
3
8
+iSen
3
8
_
.
Solucin
Como [z[ =

2, [u[ =

8, entonces [zu[ = 4. El argumento del producto zu ser la suma

1
+
2
=
11
4
+
3
8
=
25
8
.
Por consiguiente
zu = 4
_
Cos
25
8
+iSen
25
8
_
,
o bien
zu = 4
_
Cos
9
8
+iSen
9
8
_
.
Ejemplo 15.52 Escriba en forma trigonomtrica el nmero complejo
v =
_
Cos

3
iSen

3
_
(

3 +i)
i 1
.
Solucin
El nmero z = Cos

3
iSen

3
tiene mdulo igual a 1 y argumento
1
=

3
; el nmero u =

3+i)
tiene mdulo 2 y argumento
2
=

6
; el nmero w = i 1 tiene mdulo

2 y argumento
3
=
3
4
.
Por ello
[v[ =
[z[[u[
[w[
=
2

2
=

2.
y el argumento
=
1
+
2

3
=

3
+

6

3
4
=
11
12
.
Por lo tanto
v =

2
_
Cos
_

11
12
_
+iSen
_

11
12
__
.
CAPTULO 15. NMEROS COMPLEJOS 693
Ejemplo 15.53 Hallar todos los valores de
4

16.
Solucin
Escribamos el nmero z = 16 en forma trigonomtrica
z = 16(Cos +iSen).
De acuerdo con la frmula general, obtenemos
w
k
= 2
_
Cos
_

4
+
2k
4
_
+iSen
_

4
+
2k
4
__
, k = 0, 1, 2, 3.
Por consiguiente
w
0
= 2
_
Cos

4
+iSen

4
_
=

2 +i

2;
w
1
= 2
_
Cos
3
4
+iSen
3
4
_
=

2 +i

2;
w
2
= 2
_
Cos
5
4
+iSen
5
4
_
=

2 i

2;
w
3
= 2
_
Cos
7
4
+iSen
7
4
_
=

2 i

2.
En la gura estn representados los cuatro valores de
4

16. Los puntos correspondientes a los


nmeros w
0
, w
1
, w
2
, w
3
se hallan en los vrtices de un cuadrado inscrito en la circunferencia de
radio R = 2 con centro en el punto z = 0.
Ejemplo 15.54 Hallar todos los valores de
3

8.
Solucin
Como z = 8(Cos180

+iSen180

), entonces
3

8 =
3
_
8(Cos180

+iSen180

)
=
3

8
_
Cos
180

+ 360

k
n
n +iSen
180

+ 360

k
n
n
_
para todo k=1, 2, 3.
Si k = 0 entonces
w
0
= 2(Cos60

+iSen60

)
= 2
_
1
2
+

3
2
i
_
= 1 +

3 i.
Si k = 1 entonces
w
1
= 2(Cos180

+iSen180

)
= 2(1 + 0i)
= 2.
CAPTULO 15. NMEROS COMPLEJOS 694
Si k = 2 entonces
w
2
= 2(Cos300

+iSen300

)
= 2
_
1
2

3
2
i
_
= 1

3 i.
Ejemplo 15.55 Determine m y n reales tal que z = 1+i sea raz de la ecuacin z
5
+mz
3
+n =
0.
Solucin
Como z =

2 (Cos45

+iSen45

), entonces
z
5
=
_

2 (Cos45

+iSen45

)
_
5
=
_

2
_
5
(Cos225

+iSen225

)
=
_

2
_
5
_

2
2

2
2
i
_
= 4 4i.
z
3
=
_

2 (Cos45

+iSen45

)
_
3
=
_

2
_
3
(Cos125

+iSen125

)
=
_

2
_
3
_

2
2
+

2
2
i
_
= 2 + 2i.
De z
5
+mz
3
+n = 0 obtenemos
(4 4i) +m(2 + 2i) +n = 0 + 0i
entonces
_
4 2m+n = 0
4 + 2m = 0
Resolviendo el sistema de ecuaciones, obtenemos m = 2 y n = 8.
15.6.6. Tarea
1. Dados los nmeros complejos u = 5
_
Cos

4
+iSen

4
_
, v = 2
_
Cos

12
+iSen

12
_
y w = 4i.
Obtenga en forma polar:
a) uw; b)
u
v
2
; c)
u
3
vw
2
; d)
uv
3
w
.
Respuesta: a) 10
_
Cos

3
+iSen

3
_
, b)
5
4
_
Cos

12
+iSen

12
_
,
c)
125
32
_
Cos
5
3
+iSen
5
3
_
, d) 10.
CAPTULO 15. NMEROS COMPLEJOS 695
2. Hallar las seis races de 1. Represntelas y exprselas en forma polar.
Respuesta: 1,
1
2
+

3
2
i,
1
2
+

3
2
i, 1,
1
2

3
2
i,
1
2

3
2
i.
3. Resuelva la ecuacin z
3
+ 27 = 0. represente sus soluciones.
Respuesta:
3
2
+
3

3
2
i, 3,
3
2

3

3
2
i.
4. Calcular:
a)
3

i; b)
4
_
8 + 8

3 i; c)
3
_
2+2i
1+

3 i
.
Respuesta: a) i,

3
2

1
2
i,

3
2
+
1
2
i; b)

3+i, 1+

3 i, 1

3 i,

3i;
c)
6

2
_
Cos
5
36
+iSen
5
36
_
,
6

2
_
Cos
29
36
+iSen
29
36
_
,
6

2
_
Cos
53
36
+iSen
53
36
_
.
5. Dos nmeros complejos tienen el mismo mdulo. Uno de ellos es conjugado del otro. Sus
argumentos suman 510

. Calcular los argumentos de ambos nmeros complejos.


Respuesta:
1
= 300

,
2
= 210

.
6. Simplicar la expresin
_
1
2
+

3
2
i
_
45
.
Respuesta: 1.
7. Calcule m para que el nmero complejo 3 mi tenga el mismo mdulo que 2

5 +

5 i.
Respuesta: m = 4 y m = 4.
8. Resuelva las ecuaciones:
a) x
4
+ 1 = 0; b) x
6
+ 64 = 0.
Respuesta: a)

2
2
+

2
2
i,

2
2
+

2
2
i,

2
2

2
2
i,

2
2

2
2
i;
b)

3 +i, 2i,

3 +i,

3 i, 2i,

3 i.
9. El nmero 4 + 3i es la raz cuarta de un cierto nmero complejo z. Hallar las otras tres
races.
Respuesta: 3 + 4i, 4 3i, 3 4i.
10. Hallar dos nmeros complejos, cuyo cociente sea 3, la suma de sus argumentos

3
y la suma
de sus mdulos sea 8.
11. Simplicar la expresin
(2 i)
_
Cos

3
+iSen

3
_
(1 + 3i)
_
Cos

3
iSen

3
_.
12. Sean z
1
= 1 +i y z
2
=

3 +i:
a) Calcular el mdulo y el argumento de z
1
y z
2
.
b) Encontrar la representacin trigonomtrica y cartesiana del nmero complejo z =
z
1
z
2
.
Deducir el valor exacto de Cos

12
y Sen

12
.
CAPTULO 15. NMEROS COMPLEJOS 696
13. Sean z
1
= 4 +4i y a
2
= 1

3 i nmeros complejos. Determine el mdulo y el argumento


de:
a) z
2
1
; b) z
1
z
2
; c) z
3
1
; d)
z
1
z
2
; e)
z
2
z
1
.
14. Determine el conjunto de nmeros complejos z tal que
z + 2i
z i
tenga argumento

2
.
15. Demuestre que:
a) (1 +i)
n
= 2
n
2
_
Cos
n
4
+iSen
n
4
_
, n N;
b) (

3 i)
n
= 2
n
_
Cos
n
6
+iSen
n
6
_
, n N.
16. Verique que:
(1 +i)(1 +

3 i)(Cos +iSen) = 2

2
_
Cos
_
7
12
+
_
+iSen
_
7
12
+
__
17. Sea z = Cos
2
5
+iSen
2
5
:
a) Verique que z
5
1 = 0. Deduzca que 1 +z +z
2
+z
3
+z
4
= 0;
b) Exprese z, z
2
, z
3
, z
4
en forma trigonomtrica.
18. Determine las soluciones de la ecuacin z
3
= 4

2(1 +i) y deduzca los valores de Cos


11
12
y Sen
11
12
.
19. Encuentre las races cbicas de z = 8
_
Cos

3
iSen

3
_
.
20. Exprese los nmeros complejos en su forma polar:
a) z = 2

5 +3i; b) z =
1
3
(

7 2i); c) z = 3 +2

3 i; d) z = 3

2 +2

2 i.
21. Si z
1
= 3Cis20
o
, z
2
= 4Cis30
o
y z
3
= 2Cis10
o
, hallar z
1
z
1
2
z
3
.
22. Simplicar la operacin indicada y expresar el resultado en la forma a +ib:
a)
(2Cis12
o
)(3Cis54
o
)(2Cis25
o
)
(4Cis60
o
)(Cis45
o
)
; b)
(Cos150
o
iSen750
o
)(Cos250
o
+iSen1250
o
)
Cos215
o
+iSen435
o
.
23. Escribir en la forma polar (5 3

2 i)(6 + 6i)(2

2 + 15i).
24. Si z = 2 3i, w = 3 + 2i, gracar el lugar geomtrico de los nmeros complejos que satis-
facen la relacin Arg
_
z +w
z w
_
= 0.
25. Simplicar
_
1 Senx iCosx
1 +Senx +iCosx
_
4
.
CAPTULO 15. NMEROS COMPLEJOS 697
26. Para z = Cis
_
2
3
_
, hallar:
a) El mdulo y el argumento de (1 iz)
4
; b) Im(1 iz)
4
en sumas.
27. Hallar el modulo del nmero complejo z = 1 +Cos
8
7
+iSen
8
7
.
28. Hallar los argumentos del nmero complejo:
a) z = 2 2i; b) z = Sen

9
iSen

9
; c) z = 1 +Cos

7
+iSen

7
.
29. Entre los nmeros complejos z que satisfacen la condicin [z +1 i[ = 1, hallar el nmero
que tenga el argumento positivo menor.
30. Represente el nmero complejo z en forma trigonomtrica:
a) z =

3 +i; b) z = Cos

12
iSen

12
; c) z = 1 +Cos
10
9
+iSen
10
9
;
d) z = Tan1 i.
31. Escriba el nmero complejo z en las formas algebraica y trigonomtrica:
a) z =
i
_
Cos
5
3
+iSen
5
3
_
Cos

6
+iSen

6
; b) z =
1
Cos
4
3
+iSen
4
3
; c) z =
Cos
5
12
+iSen
5
12
Cos
13
12
iSen
13
12
;
d) z =
_
Cos

3
iSen

3
_
_
1
2
+i

3
2
_
.
32. Representar en forma trigonomtrica el nmero complejo z:
a) z =
5i(Cos100

+iSen100

)
3(Cos40

+iSen40

)
; b) z =
Sen
2
5
+i
_
1 Cos
2
5
_
i 1
.
33. El vector z = 2 + 5i, despus de girarlo a un ngulo

2
en sentido horario y alargar doble-
mente, se convierte en el vector u. Hallar el nmero complejo que corresponde al vector u.
34. El vector z = 2 + 3i est girado a 180

y alargado 1,5 veces. Hallar el nmero complejo


que corresponde al vector obtenido.
35. Escriba el nmero complejo z en forma trigonomtrica:
a) z =
_

3i + 1
i 1
_
6
; b) z =
(1 +i)
2n+1
(1 i)
2n1
, n N; c) z = (Tan2 i)
4
;
d) z =
_
Sen
6
5
+i
_
1 +Cos
6
5
__
5
.
36. Si z = 1 4i y w = 2 3i, hallar el coseno del ngulo comprendido entre z +w y z w.
37. Si z +
1
z
= 2Cosa, demuestre que z
n
+
1
z
n
= 2Cosna.
CAPTULO 15. NMEROS COMPLEJOS 698
38. Expresar en forma polar el nmero complejo z = (Sen Sen) +i(Cos Cos).
39. Determine mdulo y argumento del nmero complejo z = Cos +iSen 1.
40. Demuestre que (1 +i

3)
n
+ (1 i

3)
n
= 2
n+1
Cos
n
3
.
41. Si Cos+Cos+Cos = Sen+Sen+Sen = 0, usando nmeros complejos demuestre
que
Cos3 +Cos3 +Cos3 = 3Cos( + +)
Sen3 +Sen3 +Sen3 = 3Sen( + +)
42. Si z
1
= Cis, z
2
= Cis, z
3
= Cis, demuestre que
(z
1
+z
2
)(z
2
+z
3
)(z
3
+z
1
) = 8z
1
z
2
z
3
Cos

2
Cos

2
Cos

2
43. Usando nmeros complejos, demuestre que
Sen
4
Cos
3
=
1
64
(Cos7 Cos5 3Cos3 + 3Cos)
44. Demuestre que
C = Cos +Cos2 +... +Cosn =
Sen
n
2
Sen

2
Cos
n + 1
2

S = Sen +Sen2 +... +Senn =
Sen
n
2
Sen

2
Sen
n + 1
2

45. Si es variable, A y B nmeros complejos jos, demuestre que la circunferencia de dimetro
AB se expresa por: 2P = A(1 +Cis) +B(1 Cis).
46. Demuestre que en todo tringulo ABC se tiene:
a
3
Cos3 + 3a
2
bCos( 2) + 3ab
3
Cos(2 ) +b
3
Cos3 = c
3
47. Determine Arg(zw) y Arg
_
z
w
_
supuestos conocidos Arg(z) y Arg(w).
48. Exprese en forma polar los siguientes nmeros complejos:
a) 1 +i; b)

3 +i
1 +i
; c)
1
1 +

3i
.
Resp: a)

2
_
Cos
3
4
+iSen
3
4
_
; b)

2
_
Cos
7
12
+iSen
7
12
_
;
c)
1
2
_
Cos
2
3
iSen
2
3
_
.
CAPTULO 15. NMEROS COMPLEJOS 699
49. Calcule los nmeros complejos z tales que w =
2z 1
z 2
:
a) Tiene argumento principal igual a

2
; b) Tiene argumento principal igual a

2
.
Resp: a) Arg(w) =

2
cuando z est en la semicircunferencia de centro
_
5
4
; 0
_
y radio
3
4
que est contenida en el semiplano inferior; b) Arg(w) =

2
cuando z est en la
semicircunferencia de centro
_
5
4
; 0
_
y radio
3
4
que est contenida en el semiplano superior.
50. Expresa en forma polar los siguientes nmeros complejos:
a)

3 i; b)

3 +i; c)
3

3 +i
; d)
1 +

3i
(1 +i)
2
.
51. Supuesto que [z[ = 1 y z ,== 1, pruebe que
Arg
_
z 1
z + 1
_
=
_

2
, si Imz > 0

2
, si Imz < 0
52. Sea z = x +iy. Supuesto que [z[ = 1, z ,= 1, z ,= i, pruebe que
Arg
_
z 1
z + 1
_
=
_

4
, si 1 x +y > 0

3
4
, si 1 x +y < 0
53. Represente grcamente el conjunto de nmeros complejos que verican la igualdad [z[ =
+Argz.
54. Empleando la frmula de Moivre, calcular:
a)
_

2
2
+

2
2
i
_
100
; b)
_
3
2
+
1
2
i
_
217
; c) Partiendo de la frmula de Moivre, de-
ducir las frmulas para el Cosk y Senk para n = 2, 3, 4.
Resp: a) 1; b)

3
2
+
1
2
i; c) Cos2 = Cos
3
3CosSen
2
,
Sen2 = 2SenCos, Cos3 = Cos
3
3CosSen
2
, Sen3 = 3SenCos
2
Sen
3
,
Cos4 = Cos
4
6Cos
2
Sen
2
+Sen
4
, Sen4 = 4SenCos
3
4Sen
3
Cos.
55. Haciendo uso de la frmula de De Moivre demuestre que:
a) Sen3 = 3Sen 4Sen
3
. b) Cos4 = 8Cos
4
8Cos
2
+ 1.
c) Sen5 = 5Sen 20Sen
3
+ 16Sen
5
.
56. Hallar el mdulo y el argumento principal de 1 +Cos +iSen, donde .
57. Hallar la parte real y la imaginaria, el mdulo y un argumento de
1 +Cosx +iSenx
1 +Cosy +iSeny
.
58. Sean R y a C tales que 2Cos = a +
1
a
. Obtener 2Cos en funcin de a.
CAPTULO 15. NMEROS COMPLEJOS 700
59. Si x +iy = (2 +Cos +iSen)
1
con , x, y R, hallar x e y en funcin de y probar
que el punto (x, y) est siempre en la circunferencia de dimetro el segmento que une los
puntos
_
1
3
, 0
_
y (1, 0).
60. Demuestre que 1 + Cos
2
5
+ Cos
4
5
+ Cos
6
5
+ Cos
8
5
= 0 y dar una interpretacin
geomtrica. Demuestre que Cos

5
=

5 + 1
4
y Cos
2
5
=

5 1
4
.
61. Demuestre que
1 Cos

5
+Cos
2
5
Cos
3
5
+Cos
4
5
= 0
Sen

5
+Sen
2
5
Sen
3
5
+Sen
4
5
= 0
62. Demuestre que si n N se tiene
_
1 +Tan

12
_
n
+
_
1 iTan

12
_
n
= 2
_

2
_
n
Cos
n
12
15.7. Forma exponencial de un nmero complejo
En las diferentes partes de la moderna matemtica, as como en sus aplicaciones, se utiliza
la forma exponencial del nmero complejo, basada en la frmula de Euler, que relaciona las fun-
ciones trigonomtricas del argumento real con la funcin exponencial del argumento imaginario.
Exponemos la primera frmula de Euler sin deduccin:
e
i
= Cos +iSen
Donde el nmero e, tomado como base de los logaritmos naturales, es irracional e 2, 718; este
nmero es tan importante como el numero .
Si en la frmula z = r(Cos + iSen) sustituimos la expresin Cos + iSen por e
i
, ob-
tendremos z = re
i
. Precisamente sta es la forma exponencial del nmero complejo z. En esta
notacin r es el modulo del nmero complejo, es el argumento del nmero complejo z. Susti-
tuyendo en la frmula de Euler por , obtendremos la segunda frmula de Euler
e
i
= Cos() +iSen()
o bien
e
i
= Cos iSen.
De las frmulas de Euler
e
i
= Cos +iSen y e
i
= Cos iSen
se pueden obtener importantes resultados. Sumando miembro a miembro estas igualdades, obten-
emos:
e
i
+e
i
= 2Cos
de donde
Cos =
e
i
+e
i
2
CAPTULO 15. NMEROS COMPLEJOS 701
Restando miembro a miembro las mismas igualdades, tendremos:
Sen =
e
i
e
i
2i
Estas igualdades se llaman tambin frmulas de Euler; ellas expresan las funciones trigonomtricas
del argumento real por las funciones exponenciales del argumento imaginario. Las frmulas se
cumplen tambin cuando se sustituye por un nmero complejo z cualquiera; esta sustitucin nos
da:
_
Cosz =
e
zi
+e
zi
2
Senz =
e
zi
e
zi
2i
sta igualdades se toman como denicin del seno y del coseno del argumento complejo.
Demostremos que las funciones trigonomtricas de argumento complejo tambin son peridicas,
de periodo p = 2. En efecto
Cos(z + 2) =
e
(z+2)i
+e
(z+2)i
2
=
e
zi
e
2i
+e
zi
e
2i
2
=
e
zi
+e
zi
2
= Cosz
puesto que por las frmulas de Euler
e
2i
= Cos2 +iSen2 = 1 y e
2i
= Cos2 iSen2 = 1
La periodicidad de la funcin exponencial de argumento complejo se revela fcilmente; su periodo
es p = 2i.
En efecto
e
z+2i
= e
z
e
2i
= e
z
1 = e
z
.
Se observa que todas las frmulas de la trigonometra ordinaria son validas en el campo complejo.
Teorema 15.15 El nmero complejo z = r(Cos + iSen) se puede expresar como z = re
i
donde e
i
= Cos +iSen, entonces:
a) e
i
1
e
i
2
= e
i(
1
+
2
)
;
b)
_
e
i
_
1
= e
i
;
c)
e
i
1
e
i
2
= e
i(
1

2
)
;
d)
_
e
i
_
k
= e
ik
.
Demostracin
a) Aplicando la denicin, obtenemos
e
i
1
e
i
2
= (Cos
1
+iSen
1
)(Cos
2
+iSen
2
)
= [Cos
1
Cos
2
Sen
1
Sen
2
+ (Sen
1
Cos
2
+Cos
1
Sen
2
)i]
= [Cos(
1
+
2
) +iSen(
1
+
2
)]
= e
i(
1
+
2
)
.
CAPTULO 15. NMEROS COMPLEJOS 702
b) Aplicando la denicin, obtenemos
_
e
i
_
1
= (Cos +iSen)
1
=
1
Cos +iSen
=
Cos0 +iSen0
Cos +iSen
= Cos() +iSen()
= Cos iSen
= e
i
.
c) Aplicando la denicin, obtenemos
e
i
1
e
i
2
=
(Cos
1
+iSen
1
)
(Cos
2
+iSen
2
)
=
(Cos
1
+iSen
1
)(Cos
2
iSen
2
)
(Cos
2
+iSen
2
)(Cos
2
iSen
2
)
=
(Cos
1
+iSen
1
)(Cos
2
iSen
2
)
(Cos
2

2
+Sen
2

2
)
= Cos(
1

2
) +iSen(
1

2
)
= e
i(
1

2
)
.
d) Aplicando la denicin, obtenemos
_
e
i
_
k
= (Cos +iSen)
k
= Cosk +iSenk
= e
ik
Ejemplo 15.56 Representar en forma exponencial el nmero complejo z = 3 + 4i.
Solucin
El mdulo
r =
_
3
2
+ 4
2
= 5
Hallamos el argumento . Puesto que Tan =
4
3
, tendremos que
= ArcTan
4
3
3 + 4i = 5e
0,93i
Ejemplo 15.57 Representar en forma exponencial el nmero complejo z =

3 i.
Solucin
Hallamos el mdulo
[z[ =

3 + 1 = 2
El argumento lo hallamos de la correlacin Tan =
1

3
. Por lo tanto
=

3 i = 2e

6
i
CAPTULO 15. NMEROS COMPLEJOS 703
Ejemplo 15.58 Calcular e
2+i
.
Solucin
Tenemos que
e
2+i
= e
2
e
i
= e
2
(Cos1 +iSen1)
e
2
(0,540 + 0,842i)
= 7,39(0,540 + 0,842i)
= 3,99 + 6,22i.
Ejemplo 15.59 Calcular Cosi y Seni.
Solucin
Poniendo en la igualdad deducida anteriormente z = i, obtendremos:
Cosi =
e
ii
+e
ii
2
=
e
1
+e
1
2
.
Result que Cosi es un nmero real, mayor que 1, lo que no nos debe extraar.
Calculemos Seni:
Seni =
e
ii
e
ii
2i
=
i(e
1
e
1
)
2
.
En consecuencia, Seni es un nmero imaginario.
Ejemplo 15.60 Represente en forma exponencial el nmero complejo z =
1
8
(

3 i).
Solucin
Hallamos el mdulo del nmero
[z[ =
_
1
64
(3 + 1) =
1
4
y uno de sus argumentos
Tan =
1

3
=

6
Ya que z se encuentra en el cuarto cuadrante. Por consiguiente
z =
1
4
e

i
6
.
Ejemplo 15.61 Escriba en forma exponencial el nmero complejo
z =
(i

3)
_
Cos

12
iSen

12
_
1 i
Solucin
Representemos cada uno de los nmeros i

3, Cos

12
iSen

12
, 1 i en forma exponencial
i

3 = 2e
5i
6
; Cos

12
iSen

12
= Cos
_


12
_
iSen
_


12
_
= e

i
12
; 1 i =

2e

i
4
.
CAPTULO 15. NMEROS COMPLEJOS 704
Empleando las frmulas deducidas anteriormente, obtenemos
z =
2e
5i
6
e

i
12

2e

i
4
=

2e
i(
5
6


12
+

4
)
=

2e
i
.
Ejemplo 15.62 Represente en forma exponencial el nmero complejo z = (1 +i)
5
.
Solucin
Escribamos en forma exponencial la base de la potencia:
(1 +i)
5
=
_

2 e
3i
4
_
5
= 4

2 e
15i
4
= 4

2 e

i
4
.
Ejemplo 15.63 Escriba todos los valores de la raz
4
_

3 +i en forma exponencial.
Solucin
Representemos el nmero

3 +i en forma exponencial:
4
_

3 +i =
4
_
2e
i
6
=
4

2 e
(12k+1)i
24
, k = 0, 1, 2, 3.
Ejemplo 15.64 Demuestre que Sen
3
=
3
4
Sen
1
4
Sen3.
Solucin
Transformamos a la forma exponencial
Sen
3
=
_
e
i
e
i
2i
_
3
=
(e
i
)
3
3(e
i
)
2
e
i
+ 3e
i
(e
i
)
2
(e
i
)
3
8i
3
=
1
8i
(e
3i
3e
i
+ 3e
i
e
3i
)
=
3
8i
(e
i
e
i
)
1
8i
(e
3i
e
3i
)
=
3
4
e
i
e
i
2i

1
4
e
3i
e
3i
2i
=
3
4
Sen
1
4
Sen3.
15.8. Tarea
1. Sea w = e
2i
13
. Demuestre que w +w
3
+w
4
+w
9
+w
10
+w
12
=

13 1
2
.
2. Sea w = e
2i
7
. Calcular
6

n=1
z
n
2
.
CAPTULO 15. NMEROS COMPLEJOS 705
3. Sea z = x +iy; hallar el mdulo y los argumentos del nmero e
z
.
4. Represente z en la forma algebraica:
a) z = e
2i
; b) z = e

3i
2
+12i
; c) z = e
3i+7+3i
i
2
.
5. Represente en forma exponencial el nmero complejo z:
a) z =

12 2i; b) z = Cos

7
+iSen

7
.
6. Escriba en las formas exponencial y algebraica el nmero complejo z:
a) z = 5e
i
4
0,2e
i
6
_
Cos
5
12
isen
5
12
_
; b) z =
_
1
2
e
i
12
_
3
;
c) z =
1
(Cos12

+iSen12

)
5
; d) z =
e

i
3
(1 +

3i)
7
i
.
7. Demuestre la frmula
(1 +Cosx +iSenx)
2n
=
_
2Cos
x
2
_
2n
e
inx
, n N, x R.
8. Empleando la frmula
n

z =
n

r e
(
+2k
n
)i
escriba en forma exponencial todos los valores
de
n

z:
a) z = 1, n = 3; b) z = 4 +

48 i, n = 3; c) z = 1

3 i, n = 4.

Вам также может понравиться